You are on page 1of 777

29 mm 8.

5 X 11 inch
IIT-JEE ADVANCED
VOLUME I
About The Author
Shashi Bhushan Tiwari is a distinguished academician and Physics guru. He graduated
from IIT Kharagpur in year 1995 and has been mentoring students for IIT JEE for more
than two decades.
IIT-JEE ADVANCED
VOLUME I

Shashi Bhushan Tiwari


McGraw Hill Education (India) Private Limited
PREFACE

In the past decade and a half, the entrance exam for IITs has seen many changes – in structure as well as in design of the
question paper. No doubt, it has become more challenging. It requires high level of conceptual clarity and analytical skill,
besides promptness and comprehension ability to excel in this exam. There are frequent surprises in terms of problems
which require mathematical rigor or in depth understanding of physical conditions.
This book is being presented with a very simple objective – it will test you and nurture you on all parameters which
are required to excel in JEE exam.
Every chapter in the book has been divided into three sections –
y LEVEL 1 – This section will test you on all basic fundamentals of the chapter. Problems are not very rigorous though
they may be very conceptual.
y LEVEL 2 – This section will develop all necessary skills required to score a high rank in JEE exam. Few problems
in this section may appear lengthy but they are the ones which test your confidence and patience. Don’t be scared of
them.
y LEVEL 3 – This section contains problems that may require exceptional reasoning skill or mathematical ability.
Since difficulty level is quite subjective and may vary from person to person — few problems may appear to you as
misplaced in three sections described above. I have judged them to the best of my ability besides taking help from some
very bright minds.
I have not tried to include every other problem that is available in this universe. Most of the books available in market
have this issue – in the name of being exhaustive, they have become repetitive. Believe me, while solving problems from
this book you will not feel like wasting your time in doing similar problems again and again..
Most of the solutions are quite descriptive so that a serious student can understand on his/her own. Diagrams have
been included wherever possible to make things lucid.
JEE exam being objective, one may challenge the sanctity of a subjective book. Have no doubts in your mind —
pattern of a question paper or type of question will never deter you if you have sound grasp of the subject and have
developed right kind of temperament. Physics as a subject is notorious and can be learned only by subjecting yourself to
the true rigor and complexity. While doing a subjective problem you cannot make a guess and bluff yourself!
This collection of problems will appear to you as fresh and challenging. Start and enjoy learning physics!
Suggestions are welcome.
CONTENTS

Prefacev

Chapter 1 Basic Maths 1.1 – 1.12

Chapter 2 Kinematics 2.1 – 2.101

Chapter 3 Newton's Laws 3.1 – 3.134

Chapter 4 Work - Power - Energy 4.1 – 4.69

Chapter 5 Momentum and Center Of Mass 5.1 – 5.87

Chapter 6 Rotational Motion 6.1 – 6.133

Chapter 7 Gravitation 7.1 – 7.40

Chapter 8 Fluids 8.1 – 8.52

Chapter 9 Surface Tension 9.1 – 9.19

Chapter 10 Viscosity 10.1 – 10.8

Chapter 11 Elasticity 11.1 – 11.9

Chapter 12 Simple Harmonic Motion 12.1 – 12.48

Chapter 13 Wave Motion 13.1 – 13.47


01 BASIC MATHS

Initially, the height of liquid in the bowl is H0.


Level 1 H0
It becomes in time t0. How much more
Q.1 In an experiment mileage of a car was measured 2
to be 24 kmpl (Kilometer per liter of fuel time will be needed for the height of liquid to
consumed). After the experiment it was found that H
become 0 .
4 % of the fuel used during the experiment was 4
leaked through a small hole in the tank. Calculate Q.7 Show that the volume of a segment of height h of
the actual mileage of the car after the tank was a sphere of radius R is
repaired. 1
V = π h 2 ( 3R – h )
Q.2 A man is standing at a distance of 500m from a 3
building. He notes that angle of elevation of the
top of the building is 3.6°. Find the height of
the building. Neglect the height of the man and
take = 3.14.
Q.3 A Smuggler in a hindi film is running with a bag R
0.3 m × 0.2 m × 0.2 m in dimension. The bag is
supposed to be completely filled with gold. Do
you think than the director of the film made a
technical mistake there? Density of gold is 19.6
g/cc.
Q.4 A particle moves along the curve 6y = x3+2. h
Find the points on the curve at which the
y-coordinate is changing 8 times as fast as the Q.8 The amount of energy a car expends against air
x-coordinate. resistance is approximately given by
Q.5 The area of a regular octagon of side length a is E = K ADv2
A.
where E is measured in Joules. K is a constant, A
(a) Find the time rate of change of area of the
is the cross-sectional area of the car viewed from
octagon if its side length is being increased the front (in m2), D is the distance traveled (in m),
at a constant rate of b m/s. Is the time rate and v is the speed of the car (in m/s). Julie wants
of change of area of the octagon constant to drive from Mumbai to Delhi and get good fuel
with time? mileage. For the following questions, assume that
(b) Find the approximate change in area of the
the energy loss is due solely to air resistance.
octagon as the side length is increased from (a) Julie usually drives at a speed of 54 Km/hr.
2.0 m to 2.001 m. How much more energy will she use if she
drives 20% faster?
Level 2 (b) Harshit drives a very large SUV car, and Julie
drives a small car. Every linear dimension of
Q.6 Spirit in a bowl evaporates at a rate that is
Harshit’s car is double that of Julie’s car. Find
proportional to the surface area of the liquid.
the ratio of energy spent by Harshit’s car to
1.2 Problems in Physics for JEE Advanced

Julie’s car when they cover same distance. your formula tell you how ‘t’ depends on
Speed of Harshit was 10% faster compared initial speed V0? What can you predict about
to Julie’s car. the constant obtained in the formula?
(c) Write the dimensional formula for K. Will
Q.14 Assume that maximum mass m1 of a boulder
you believe that K depends on density of air? swept along by a river, depends on the speed V of
Q.9 The volume flow rate Q (in m3 s–1) of a liquid the river, the acceleration due to gravity g, and the
through pipe having diameter d is related to density d of the boulder. Calculate the percentage
viscosity of water ‘h’ (unit Pascal. s) and the change in maximum mass of the boulder that can
be swept by the river, when speed of the river
pressure gradient along the pipe dP [pressure increases by 1%.
dx
dP
gradient is rate of change of pressure per unit Q.15 A massive object in space causes gravitational
dx
length along the pipe], by a formula of the form lensing. Light from a distant source gets
 dP 
c deflected by a massive lensing object. This was
Q = kη a d b   first observed in 1919 and supported Einstein’s
 dx  general theory of relativity.
Where K is a dimensionless constant. Find a,b
and c. The angle by which light gets deflected due
to a massive body depends on the mass (M)
Q.10. The potential energy (U) of a particle can be
of the body, universal gravitational constant
A2 BMm
expressed in certain case as U = 2
– (G), speed of light (c) and the least distance (r)
2mr r between the lensing object and the apparent path
Where m and M are mass and r is distance. Find of light. Derive a formula for using method of
the dimensional formulae for constants. dimensions. Make suitable assumptions.
Q.11. In the following expression V and g are speed and Q.16 The Casimir effect describes the attraction
acceleration respectively. Find the dimensional between two unchanged conducting plates placed
formulae of a and b parallel to each other in vacuum. The astonishing
VdV force ( predicted in 1948 by Hendrik Casimir) per
∫ g – bV 2 = a unit area of each plate depends on the planck’s
Q.12 The maximum height of a mountain on earth is constant (h), speed of light (c) and separation
limited by the rock flowing under the enormous between the plates (r).
weight above it. Studies show that maximum (a) Using dimensional analysis prove that the
height depends on young’s modulus (Y) of the formula for the Casimir force per unit area on
rod, acceleration due to gravity (g) and the density the plates is given by
of the rock (d). hc
F = k 4 where k is a dimensionless constant
(a) Write an equation showing the dependence r
of maximum height (h) of mountain on Y, g (b) If the force acting on 1x1 cm plates separated
and d. It is given that unit of Y is Nm–2. by 1 m is 0.013 dyne, calculate the value of
(b) Take d = 3 × 103 kg m–3, Y = 1 × 1010 Nm–2 constant k.
and g = 10 ms–2 and assume that maximum Q.17. Scattering of light is a process of absorption
height of a mountain on the surface of earth and prompt re-emission of light by atoms
is limited to 10 km [height of mount Everest and molecules. Scattering involving particles
is nearly 8 km]. Write the formula for h. smaller than wavelength (l) of light is known
Q.13 A particle of mass m is given an initial speed V0. It as Rayleigh scattering. Let ai be amplitude of
experiences a retarding force that is proportional incident light on a scatterer of volume V. The
to the speed of the particle (F = aV). a is a scattered amplitude at a distance r from the
constant. 1
scatterer is as. Assume and as a ai , as and
a aV. r
(a) Write the dimensional formula of constant a. s

(b) Using dimensional analysis, derive a formula (i) Find the dimensions of the proportionality
for stopping time (t) of the particle. Does constant occurring in the expression of as
Basic Mathematics And Dimensional Analysis 1.3

(ii) Assuming that this constant depends on l, Sirius has a maximum parallax angle of d = 0.74
as ± 0.02 arc second when observed at six month
find the dependence of ratio on l. interval. The distance between two positions
ai
of earth (at six – month interval) is r = 3.000 ×
(iii) Knowing that intensify of light I a a2 find
I 1011m
the dependence of s on l. Earth
Ii
dx
Q.18 It is given that ∫ 2
= tan −1 x + c . Using
1+ x Sun
dx
methods of dimensions find ∫ 2 r
a + x2
Star
Q.19

Earth
Calculate the distance of Sirius from the Sun with
B uncertainty, in unit of light year. Given 1 ly =
A 9.460x1015 m. ; p = 3.14
A B

C Level 3
Q.22 You inhale about 0.5 liter of air in each breath
and breath once in every five seconds. Air has
Two point sources of light are fixed at the centre about 1% argon. Mass of each air particle can be
(A) and circumference (point B) of a rotating assumed to be nearly 5 × 10–26 kg. Atmosphere
turn table. A photograph of the rotating table is can be assumed to be around 20 km thick having a
taken. On the photograph a point A and an arc uniform density of 1.2 kg m–3. Radius of the earth
BC appear. The angle q was measured to be q = is R = 6.4 × 106 m. Assume that when a person
10.8° ± 0.1° and the angular speed of the turntable breathes, half of the argon atoms in each breath
was measured to be w = (33.3 ± 0.1) revolution have never been in that person’s lungs before.
per minute. Calculate the exposure time of the Argon atoms remain in atmosphere for long-long
camera. time without reacting with any other substance.
Q.20 The speed (V) of wave on surface of water is Given : one year = 3.2 × 107s
given by (a) Estimate the number of argon atoms that
al 2p b passed through Newton’s lungs in his 84
V= + years of life.
2p rl
(b) Estimate the total number of argon atoms in
where l is the wavelength of the wave and r is the Earth’s atmosphere.
density of water. a is a constant and b is a quantity
that changes with liquid temperature. (c) Assume that the argon atoms breathed by
Newton is now mixed uniformly through the
(a) Find the dimensional formulae for a and b. atmosphere, estimate the number of argon
(b) Surface wave of wavelength 30 mm have a atoms in each of your breath that were once
speed of 0.240 ms–1. If the temperature of in Newton’s lungs.
water changes by 50°C, the speed of waves Q.23 A rope is tightly wound along the equator of a
for same wavelength changes to 0.230 ms–1. large sphere of radius R. The length of the rope
Assuming that the density of water remains is increased by a small amount  (<< R) and it is
constant at 1 × 103 kg m–3, estimate the pulled away from the surface at a point to make it
change in value of ‘b’ for temperature change taut. To what height (h) from the surface will the
of 50ºC. point rise ?
Q.21 The line of sight of the brightest star in the sky, If the radius of the earth is R=6400 km and  =
1.4 Problems in Physics for JEE Advanced

10 mm, find the value of h. Does the value surprise h


you.
q3
[For small q take tanq = q + and
3
q2 2
secq = 1 + Also take (2.3) 3 = 1.74 ]
2
R

ANSWERS

1. 25 kmpl 14. 6%
2. 31.40 m GM
15. q = k
3. Yes. cr 2
16. (b) k = 6.5 × 10–3
31
4. (– 4, - ), (4, 11) 17. (i) [k] = [L–2]
3
5. (a) 4 ( 2 + 1) a b. No, it is not a constant as
(ii) µ l -2
(b) 0.0019 m2 ai
t0 Is a2
6. (iii) µ s2 µ l -4
2 Ii ai
8. (a) 44% higher (b) 4.84 (c) [ML–3]; Yes -1 Ê x ˆ
18. a tan Á ˜ + c
9. a = – 1; b = 4; c = 1 Ë a¯
10. [A] = [M1L2T–1] [B] = [M–1L3T–2] 19. (0.054 0.003)s
11. [a] = L; [b] = L–1 20. (a) [a] = [MºL1T–2]; [b] = [M1LºT–2]
Y  (b) b = – 0.022 kg s–2
12. (a) h = k   ; k = a const
 gd  21. 8.84 ± 0.24 ly
22. (a) 3.2 × 1028 (b) 2.5 × 1042
Y 
(b)
h = 0.03   (c) 1.5 × 106
 gd 
23. 5.6 m
m
13. (a) [a] = [M1T–1] (b) t = k t
a

SOLUTIONS

2. p 3.14 ¥ 3.6
q = 3.6∞ = ¥ 3.6 rad = = 0.0628 radian
180 180

q = 3.6°

500
Basic Mathematics And Dimensional Analysis 1.5

h = 500 tan q 500.q = 500 × 0.0628 m = 31.40 m


3. Calculate the mass of gold using – mass = volume × density. The gold in the bag will weigh 235.2 kg. How
can a smuggler run with such a heavy bag!
4. As per the question
dy
=8
dx
3x2
fi =8
6
&x =±4

Corresponding Y co ordinates are
(±4)3 + 2 = 11, -31
6 3
5. (a) (
Area of an octagon is = A = 2 1 + 2 a 2 [Prove this yourself] )
dt
dA
= 4 2 +1 a
da
dt
( )
= 4 2 + 1 ab ( )
This expression is not a constant because it depends on side length which itself is time dependent.
(
(b) A = 2 1 + 2 a 2 )
dA
= 4 2 + 1 a
da
( )
DA 
dA
da
Da = 4 2 + 1 a Da ( )
DA = 4 ( 2 + 1) × 2.0 × 0.001 = 0.0019 m2
6.

A
dh
h

Let the area of cross section of the bowl at height h be A.


Let the height decrease by dh in further interval dt
Volume that evaporates = Adh
Adh
As per the question aA
dt
A dh

= – kA
dt
Where k is a positive constant. We have placed a negative sign because h is decreasing with time and
dh
is a negative quantity.
dt
dh = – k dt
h t

Ú dh = – k Ú dt
H0 0

h – H0 = – kt
h = H0 – kt
1.6 Problems in Physics for JEE Advanced

H0
Now h = at t = t0
2
H
\ 0 = H 0 – kt0
2
H0
fi = kt0
2 ...........(1)
H0
Let height be at time ‘t’
4
H0
= H 0 – kt
4
3H0 H0 H0
4
=
2t0
t [using (1) k = ]
2t0
3
fit= t0
2
t0
Required answer is
2
Note: It is easy to see from equation (1) that depth of the liquid decreases at a constant rate. Hence it is obvious
H0 t
that if it takes time t0 for the level to fall from H0 to , then it will take 0 time for level to further fall through
2 2
H0
.
4
7. Consider a disc shaped element of thickness dx at a distance x from the centre

R
x
dx
r

r2 = R2 – x2
Volume of disc element dV = pr2 dx p (R2 – x2)dx
\ Required Volume is
R
V= Ú dV
x= R–h

R
=p Ú ( R 2 - x 2 )dx
( R - h)
Basic Mathematics And Dimensional Analysis 1.7

È R È x3 ˘ ˘
R

= Íp ÈÎ R 2 x ˘˚ –Í ˙ ˙
Í R–h ÍÎ 3 ˚˙ R – h ˙
Î ˚

È 1 1 3˘
= p Í R 2 ( R – R + h ) – R3 + ( R – h ) ˙
3 3
Î ˚
È R3 1 3 ˘
= p Í R2 h –
ÍÎ
(
+ R – h3 – 3R 2 h + 3Rh 2
3 3
)˙˙˚

p
= h 2 [3 R – h ]
3

8. (a) If Julie increases her speed by 20%, then she multiplies her speed by 1.2. Thus the required energy is multiplied
by (1.2)2 = 1.44, which is an increase of 44%.
(b)
The area of cross section of Harshit’s car is 4 times that of Julie’s car and his speed is 1.1 times that of Julie’s
car.

E Harshit 4 ¥ (1.1)
2
= = 4.84
EJulie 1

[E ] È ML2T –2 ˘
Î ˚ = È ML–3 ˘
(c) [K ] = =
È ADV 2 ˘ È L2 LL2T –2 ˘ Î ˚
Î ˚ Î ˚
12. (a) let h = kY agbd c [k is a contant]
Dimensions on LHS = Dimensions on RHS
M0 L1 T 0 = [M L–1T–2]a [L1 T–2]b [M1 L–3]c
Equating dimensions of M, L and T
a+c=0 (1)
– a + b – 3c = 1 (2)
– 2a – 2b = 0 (3)
Solving a = 1, b = – 1, c = – 1
ÊY ˆ
\h = k Á ˜
Ë gd ¯
Y
(b) h = k
gd

3 1010
10 ¥ 10 = k ◊
10 ¥ 3 ¥ 103
3
fik= = 0.03
100
[F ]
13. (a) [ a] = = [ MT -1 ]
[V ]
(b) let t = k ax myV0z
It can be shown that
x = – 1; y = 1; z = 0
1.8 Problems in Physics for JEE Advanced

m
\t = k (a)
a
As per this expression, the time of motion is independent of initial speed ! This cannot be true. It must grow
with V0. A large initial speed (V1) will need some non zero time to slow down to V2 fter this the particle will
take as much time to halt as it would take when projected with initial speed V2.
The only way equation (a) can be sensible is by having k = .
14. Let mass of the boulder that can be swept by the river be given by.
m = k Vx gydz
m] = [V]x [g]y [d]z
[M1 L0 T0] = [L1T–1]x [L1 T–2]y [M1 L–3]z
[Mz Lx+y–3z T–x–2y]
∴ z = 1 ; x + y – 3z = 0 ; – x – 2y = 0
Solving; x = 6; y = – 3, z = 1
\ m = k V g–3 d
6

Dm DV
\
¥ 100 = 6 ¥ 100
m V
The percentage change in mass is 6 % when speed changes by 1 %.
15. [G] = [M–1L3T–3]
[c] = [L1T–1]
[M] = [M1]
[r] = [L1]
The angle q is dimensionless.
Simple observation indicates that mass is present in expression of G and M only. Hence for dimension of mass to
be zero we must have expression like q k (GM)x (c)y (r)z
[q] = [GM]x [c]y [r]z
[M0L0T0] = [L3T–2]x [L1T–1]y [L1]z = [M0 L3x+y+z T–2x–2y]
\∴ 3x + y + z = 0
2 + 2y = 0
fiy=–x
and z = – 2x
Assuming x = 1
GM
q=k
cr 2

G2 M 2
If x = 2 q=k
c2 r 4
q is dimensionless and all expressions with x = 1,2,3 are correct. The fact of the matter is that we can also have

1 1
x = , etc. All expressions are dimensionally correct. However, the correct expression is
2 3
GM
q=k [k = 4]
c2r
Basic Mathematics And Dimensional Analysis 1.9

16. (a) Let F = k hx cy rz


F is force per unit area
[F] = [h]x [c]y [r]z
[M1 L–1 T–2] = [M L2 T–1]x [L1 T–1]y [L]z
Ê Energy ˆ
ÁË Note : h = frequency ˜¯

\∴x = 1
2x + y + z = – 1
–x–y=–2
Solving x = 1; y = 1 ; z = – 4
hc
\F = k
r4
(b) h = 6.63 × 10–27 g cm2 s–1
c = 3 × 1010 cm s–1
r = 10–4 cm
6.63 ¥ 10 -27 ¥ 3 ¥ 1010
\k
(10 )
-4 4

13
\k = ¥ 10 -2 = 6.5 ¥ 10 -3
6 . 63 ¥ 3
Vai
17. (i) as µ
r
Va
as = k i
r
V
\k is dimensionless
r
[k] = [L–2]
(ii) k µ l–2
as
\ µ l –2
ai
I s as2
(iii) a µ l –4
I i ai2

18. Denominator is a2 + x2, which means that [a] = [x].


Pretend that x is a length. Then the integral has dimensions [L–1]
Therefore, we may expect the answer to have a term like a tan–1x. Also the argument of tan–1 should be dimensionless.
Hence, we can replace x by x/a.
Ê xˆ
Therefore, the integral can be written equal to a tan–1 Á ˜ + c
Ë ¯a
19. Let the exposure time be t and the uncertainty in its value be t
33.3 ¥ 360∞
w=
= 200∞s-1
60
1.10 Problems in Physics for JEE Advanced

q 10.8
t= =
= 0.054s
w 200
Dt Dq Dw
And = +
t q w
Dq Dw
fi Dt = t
+t
q w

Dq Dw t
= +
w w
0.1 0.1 ¥ 0.054
=+
33.3 33.3

= 0.003
∴ t = (0.054 ± 0.003)s

al 2p b
20. (a) V 2 = +
2p rl
È ˘
ÈV 2 ˘ = [ al ] = Í b ˙
Î ˚
Î rl ˚

2
È M 0 L1T –1 ˘ = [ a ] È L1 ˘ = [b ]
Î ˚ Î ˚ È ML–3 ˘ È L1 ˘
Î ˚Î ˚
\∴ [a] = [M0 L1 T –2]
[b] = [M1 L0 T–2]

(c) Given data says (in SI units)

al 2p b1
(0.240)2 = + 3
2p 10 ¥ 30 ¥ 10 -3

al 2p b1
0.242 = +
2p 30 ……………(1)
Similarly,

al 2p b2
(0.23)2 = +
2p 30 ……………(2)
(2) - (1)
2p
(b – b ) = (0.23)2 – (0.24)2
30 2 1

30
Db = – – (0.23 + 0.24 )(0.24 – 0.23)
2p

= – 0.022 kg s–2
Basic Mathematics And Dimensional Analysis 1.11

21. Required distance = x


r r r
2
x=  a2 =
tan ( a
2 ) 2 a

= 0.740 arc second



Ê 0.74 ˆ 0.74 p
=Á = ¥
Ë 3600 ˜¯ 3600 180 radian

r 3.000 ¥ 1011
=\x = m
a 0.74 p
¥
3600 180

3.000 ¥ 1011 ¥ 3600 ¥ 180


= ly
0.74 ¥ p ¥ 9.460 ¥ 1015
= 8.84 ly
r
x=
a
r
Dx = Da
a2
Ê 0.02 p ˆ
3.000 ¥ 10 11 ÁË 3600 ¥ 180 ˜¯
= 2
¥ ly
Ê 0.74 p ˆ 9.460 ¥ 1015
ÁË 3600 ¥ 180 ˜¯

= 0.24 ly
22. (a) Number or argon atoms in each breath
Mass of air inhaled 1
N= ¥
Mass of one particle 100

=
(1.2kgm ) (0.5 ¥ 10 -3 -3
m3 )¥ 1
= 1.2 ¥ 1020
-26
5 ¥ 10 100
Number of times Newton breath in his life time is
n = (frequency of breaths) × 84 years
n = 0.2 × 84 × 3.2 × 107 = 5.4 × 108
1
Number of argon atom that Newton breathed in his lifetime = N .n
2
1
[factor is there to account for re-breathed atoms]
2
1
= ¥ 1.2 ¥ 1020 ¥ 5.4 ¥ 108 = 3.2 ¥ 1028
2
(b) Volume of atmosphere v = 4pR2. h.
We have estimated that half a litre (= 0.5 × 10–3 m3) air has 1.2 × 1020 argon atoms, hence number of argon
atoms in atmosphere
1.12 Problems in Physics for JEE Advanced

4p R 2 h
N0 = 3
¥ 1.2 ¥ 1020
0.5 ¥ 10

( )
2
4 ¥ 3.14 ¥ 6.4 ¥ 106 ¥ 20 ¥ 103
= ¥ 1.2 ¥ 1020 = 2.5 ¥ 10 42
5 ¥ 10 -4
(c) Fraction of argon in atmosphere which must have visited Newton’s lungs
3.2 ¥ 1028
= = 1.3 ¥ 10 -14
2.5 ¥ 10 42
∴ Required answer is 1.3 × 10–14 N = 1.5 × 106
23. P

h
B
A R C

R R
O

 
Length AP = arc AB + = Rq +
2 2

\ R tan q = Rq +
2
È q3 ˘ 
fi R Íq + ˙  Rq +
ÍÎ 3 ˙˚ 2

1
Rq 3  Ê 3 ˆ 3
fi  fiq  Á ˜
3 2 Ë 2R ¯
Now , OP = R sec q
È q2 ˘
R + h = R Í1 + ˙
ÍÎ 2 ˙˚

2
R R Ê 3 ˆ 3
fi h = q2 = Á ˜
2 2 Ë 2R ¯
For earth R = 6400 km = 6.4 × 106 m
And given l = 10 mm = 10–2 m
2

6.4 ¥ 106 Ê 3 ¥ 10 -2 ˆ
3

h= Á ˜

2 Ë 2 ¥ 6.4 ¥ 106 ¯

h = 3.2 × (2.3)2/3 = 3.2 × 1.74 = 5.6 m


02 KINEMATICS

(i) negative velocity but acceleration in positive


Level 1 X direction.
Q. 1. A particle is travelling on a curved path. In (ii) positive velocity but acceleration in negative
an interval t its speed changed from v to 2v. X direction.
However, the change in magnitude of its velocity (iii) received a sharp blow (a large force for
 negligible interval of time)?
was found to be ∆V = v . What can you say X
about the direction of velocity at the beginning
and at the end of the interval ( t)?
G
Q. 2. Two tourist A and B who are at a distance of H
A D E
40 km from their camp must reach it together in F
t
O
the shortest possible time. They have one bicycle
and they decide to use it in turn. ‘A’ started B C
walking at a speed of 5 km hr–1 and B moved on
the bicycle at a speed of 15 km hr–1. After moving
Q. 5. A particle is moving along positive X direction
certain distance B left the bicycle and walked
and is retarding uniformly. The particle crosses
the remaining distance. A, on reaching near the
the origin at time t = 0 and crosses the point
bicycle, picks it up and covers the remaining
x = 4.0 m at t = 2 s.
distance riding it. Both reached the camp together.
(a) Find the average speed of each tourist. (a) Find the maximum speed that the particle can
possess at x = 0.
(b) How long was the bicycle left unused?
(b) Find the maximum value of retardation that
Q. 3. The position time graph for a particle travelling
the particle can have.
along x axis has been shown in the figure. State
whether following statements are true of false. Q. 6. The velocity time graph for two particles (1 and
2) moving along X axis is shown in fig. At time
(a) Particle starts from rest at t = 0.
t = 0, both were at origin.
X
(a) During first 4 second of motion what is
maximum separation between the particles?
At what time the separation is maximum?
t1
O t
t2 t3 (b) Draw position (x) vs time (t) graph for the
particles for the given interval.
v (m/s)

(b) Particle is retarding in the interval 0 to t1 and
accelerating in the interval t1 to t2. 4
(c) The direction of acceleration has changed 1
once during the interval 0 to t3
Q. 4. The position time graph for a particle moving 2
along X axis has been shown in the fig. At which t (s)
O 4
of the indicated points the particle has
2.2 Problems in Physics for JEE Advanced

Q. 7. A ball travelling in positive X direction with speed v (m/s)


V0 hits a wall perpendicularly and rebounds with
speed V0. During the short interaction time ( t)
10
the force applied by the wall on the ball varies as
shown in figure.
A
Fx
B

O t (s)
2 4
t
O t Q. 10. A particle starts from rest (at x = 0) when an
acceleration is applied to it. The acceleration
of the particle changes with its co-ordinate as
shown in the fig. Find the speed of the particle at
x = 10m.
ax (m/s2)
Draw the velocity-time graph for the ball during
the interval 0 to t 8
Q. 8. For a particle moving along a straight line consider
following graphs A, B, C and D. Here x, v and t
are position, velocity and time respectively. X (in m)
O 8 10
(i) In which of the graphs the magnitude of
acceleration is decreasing with time? Q. 11. Acceleration vs time graph for a particle moving
along a straight line is as shown. If the initial
(ii) In which of the graphs the magnitude of
velocity of the particle is u = 10 m/s, draw a plot
acceleration is increasing with time?
of its velocity vs time for 0 < t < 8.
(iii) If the body is definitely going away from the a (m/s )
2

starting point with time, which of the given


graphs represent this condition. 10

X v
8
t (in s)
parabola 4

O t O t
(a) (b)
Q. 12. The velocity (V) – time (t) graphs for two particles
v v
A and B moving rectilinearly have been shown in
the figure for an interval of 2 second.
(a) At t = 1 s, which of the two particles (A or B)
has received a severe blow?
(b) Draw displacement (X) – time (t) graph for
O t O t both of them.
(c) (d)
V (m/s) V (m/s)
Q. 9. Two particles A and B start from same point and 4 4
move along a straight line. Velocity-time graph o 1 2 t(s)
o t(s)
for both of them has been shown in the fig. Find 1 2
the maximum separation between the particles in –4 –4
the interval 0 < t < 5 sec. (a) (b)
Kinematics 2.3

Q. 13. A particle starts moving rectilinearly at time t = 0 interval of time it takes the chain to pass a point
such that its velocity(v) changes with time (t) as 2L below the point of support, if all of the chain is
per equation – a freely falling body.
v = (t2 – 2t) m/s for 0 < t < 2 s
Q. 19. Two nearly identical balls are released
2
= (–t + 6t – 8) m/s for 2 < t < 4 s simultaneously from the top of a tower. One
of the balls fall with a constant acceleration of
(a) Find the interval of time between t = 0 and g1 = 9.80 ms –2 while the other falls with a
t = 4 s when particle is retarding. constant acceleration that is 0.1% greater than g1.
(b) Find the maximum speed of the particle in [This difference may be attributed to variety of
the interval 0 < t < 4 s. reasons. You may point out few of them]. What
Q. 14. Our universe is always expanding. The rate at is the displacement of the first ball by the time
which galaxies are receding from each other is the second one has fallen 1.0 mm farther than the
given by Hubble’s law (discovered in 1929 by E. first ball?
Hubble). The law states that the rate of separation Q. 20. Two projectiles are projected from same point
of two galaxies is directly proportional to their on the ground in x-y plane with y direction as
separation. It means relative speed of separation vertical. The initial velocity of projectiles are
of two galaxies, presently at separation r is given 
by v = Hr V1 = Vx1 iˆ + Vy1 ˆj

H is a constant known as Hubble’s parameter. V2 = Vx 2 iˆ + Vy 2 ˆj
Currently accepted value of H is 2.32 × 10 –18 s–1 It is given that Vx1 > Vx2 and Vy1 < Vy2. Check
(a) Express the value of H in unit of whether all of the following statement/s are True.
Km. s −1 (a) Time of flight of the second projectile is

Mega light year greater than that of the other.
(b) Range of first projectile may be equal to the
(b) Find time required for separation between
range of the second.
two galaxies to change from r to 2r.
(c) Range of the two projectiles are equal if
Q. 15. A stone is projected vertically up from a point Vx1 Vy1 = Vx2 Vy2
on the ground, with a speed of 20 m/s. Plot the
(d) The projectile having greater time of flight
variation of followings with time during the entire
can have smaller range.
course of flight –
(a) Velocity Q. 21. (a) A particle starts moving at t = 0 in x-y plane
such that its coordinates (in cm) with time (in
(b) Speed
sec) change as x = 3t and y = 4 sin (3t). Draw
(c) Height above the ground the path of the particle.
(d) distance travelled

(b) If position vector of a particle is given by
Q. 16. A ball is dropped from a height H above the ground.
It hits the ground and bounces up vertically to a

( ) ( )
r = 4t 2 - 16t iˆ + 3t 2 - 12t ˆj , then find
distance travelled in first 4 sec.
height where it is caught. Taking origin at the
Q. 22. Two particles projected at angles q1 and q2 (<q1)
point from where the ball was dropped, plot the to the horizontal attain same maximum height.
variation of its displacement vs velocity. Take Which of the two particles has larger range? Find
vertically downward direction as positive. the ratio of their range.
Q. 17. A helicopter is rising vertically up with a velocity Q. 23. A ball is projected from the floor of a long hall
of 5 ms –1. A ball is projected vertically up from having a roof height of H = 10 m. The ball is
the helicopter with a velocity V (relative to the projected with a velocity of u = 25 ms–1 making an
ground). The ball crosses the helicopter 3 second angle of q = 37° to the horizontal. On hitting the
after its projection. Find V. roof the ball loses its entire vertical component of
Q. 18. A chain of length L supported at the upper end is velocity but there is no change in the horizontal
hanging vertically. It is released. Determine the component of its velocity. The ball was projected
2.4 Problems in Physics for JEE Advanced

from point A and it hits the floor at B. Find 2 : 1. Find the ratio of this range R to the maximum
distance AB. possible range for the projectile assuming the
projection speed to be same in all cases.
Q. 28. A boy ‘A’ is running on a circular track of
u H
radius R. His friend, standing at a point O on the
 circumference of the track is throwing balls at
A B speed u = gR. Balls are being thrown randomly
Q. 24. In a tennis match Maria Sharapova returns an in all possible directions. Find the length of the
incoming ball at an angle that is 4° below the circumference of the circle on which the boy is
horizontal at a speed of 15 m/s. The ball was completely safe from being hit by a ball.
hit at a height of 1.6 m above the ground. The
opponent, Sania Mirza, reacts 0.2 s after the ball oA
is hit and runs to the ball and manages to return it R
just before it hits the ground. Sania runs at a speed C
of 7.5 m/s and she had to reach 0.8 m forward,
from where she stands, to hit the ball.
(a) At what distance Sania was standing from
O
Maria at the time the ball was returned by
Maria? Assume that Maria returned the ball Q. 29. A rectangular cardboard ABCD has dimensions
directly towards Sania. of 40 cm × 30 cm. It is moving in a direction
(b) With what speed did the ball hit the racket of perpendicular to its shorter side at a constant
Sania? speed of 2 cm/s. A small insect starts at corner A
and moves to diagonally opposite corner C. On
[g = 9.8 m/s2]
reaching C it immediately turns back and moves
Q.25. A player initially at rest throws a ball with an to A. Throughout the motion the insect maintains
a constant speed relative to the board. It takes

initial speed u = 19.5 m/s at an angle
10 s for the insect to reach C starting from A.
 12  Find displacement and distance travelled by the
θ = sin −1   to the horizontal. Immediately
 13  insect in reference frame attached to the ground
after throwing the ball he starts running to catch in the interval the insect starts from A and comes
it. He runs with constant acceleration (a) for first back to A.
C
2 s and thereafter runs with constant velocity. He D
just manages to catch the ball at exactly the same
height at which he threw the ball. Find ‘a’. Take 30 cm
g = 10 m/s2. Do you think anybody can run at a 2 cm/s
speed at which the player ran?
A
Q. 26. In a cricket match, a batsman hits the ball in air. 40 cm B
A fielder, originally standing at a distance of
Q. 30. Two particles A and B separated by 10 m at time
12 m due east of the batsman, starts running 0.6
t = 0 are moving uniformly. A is moving along
s after the ball is hits. He runs towards north at a
line AB at a constant velocity of 4 m/s and B is
constant speed of 5 m/s and just manages to catch
moving perpendicular to the velocity of A at a
the ball 2.4 s after he starts running.
constant velocity of 5 m/s. After what time the
Assume that the ball was hit and caught at the two particles will be nearest to each other?
same height and take g = 10 m/s2 g = 10 m/s2 5 m/s
Find the speed at which the ball left the bat and
the angle that its velocity made with the vertical.
Q. 27. The time of flight, for a projectile, along two A 4 m/s B
different paths to get a given range R, are in ratio 10 m
Kinematics 2.5

Q. 31. Four cars are moving along a straight road in A train(T) is running uniformly on a straight
the same direction. Velocity of car 1 is 10 m/s. track. A car is travelling with constant speed
It was found that distance between car 1 and 2 is along section AB of the road which is parallel
decreasing at a rate of 2 m/s, whereas driver in car to the rails. The driver of the car notices that the
4 observed that he was nearing car 2 at a speed of train is having a speed of 7 m/s with respect to
8 m/s. The gap between car 2 and 3 is decreasing him. The car maintains the speed but takes a right
at a rate of 3 m/s. turn at B and travels along BC. Now the driver of
20 m 20 m 20 m the car finds that the speed of train relative of him
4 3 2 1 is 13 m/s. Find the possible speeds of the car.
10 m/s
B V2 A
V1
(a) If cars were at equal separations of 20 m at time
t = 0, after how much time t0 will the driver
of car 2 see for the first time, that another car A police car B is chasing a culprit’s car A.
overtakes him? Car A and B are moving at constant speed V1 =
(b) Which car will be first to overtake car 1? 108 km/hr and V2 = 90 km/hr respectively along
Q. 32. Acceleration of a particle as seen from two a straight line. The police decides to open fire and
reference frames 1 and 2 has magnitude 3 m/s2 a policeman starts firing with his machine gun
and 4 m/s2 respectively. What can be magnitude directly aiming at car A. The bullets have a velocity
of acceleration of frame 2 with respect to u = 305 m/s relative to the gun. The policeman
frame 1? keeps firing for an interval of T0 = 20 s. The Culprit
experiences that the time gap between the first and
Q. 33. A physics professor was driving a Maruti car the last bullet hitting his car is Dt. Find Dt.
which has its rear wind screen inclined at
Q. 36. A chain of length L is supported at one end and
q = 37° to the horizontal. Suddenly it started
is hanging vertically when it is released. All of
raining with rain drops falling vertically. After
the chain falls freely with acceleration g. The
some time the rain stopped and the professor found
moment, the chain is released a ball is projected
that the rear wind shield was absolutely dry. He
up with speed u from a point 2 L below the point
knew that, during the period it was raining, his car
of support. Find the interval of time in which the
was moving at a constant speed of Vc = 20 km/hr.
ball will cross through the entire chain.
[tan 37° = 0.75] Q. 37. Jet plane A is moving towards east at a speed of
(a) The professor calculated the maximum speed 900 km/hr. Another plane B has its nose pointed
of vertically falling raindrops as Vmax. What towards 45° N of E but appears to be moving in
is value of Vmax that he obtained. direction 60° N of W to the pilot in A. Find the
(b) Plot the minimum driving speed of the car vs. true velocity of B. [sin 60° = 0.866 ; sin 75° =
angle of rear wind screen with horizontal (q) 0.966]
so as to keep rain off the rear glass. Assume
that rain drops fall at constant speed Vr
N
Q. 34. T

45°
60°
E
C B

A B E
A

Q. 38. A small cart A starts moving on a horizontal


surface, assumed to be x-y plane along a straight
line parallel to x-axis (see figure) with a constant
acceleration of 4 m/s2. Initially it is located on the
C
positive y-axis at a distance 9 m from origin. At
2.6 Problems in Physics for JEE Advanced

the instant the cart starts moving, a ball is rolled B


along the surface from the origin in a direction
making an angle 45° with the x-axis. The ball 10 cm 2 cm/s
moves without friction at a constant velocity and
hits the cart. Y A
(a) Describe the path of the A
Q. 41. Two particles A and B are moving uniformly in a
ball in a reference frame plane in two concentric circles. The time period
attached to the cart. of rotation is TA = 8 minute and TB = 11 minute
(b) Find the speed of the 45° respectively for the two particles. At time t = 0,
ball. O X the two particles are on a straight line passing
Q. 39. (a) A boy on a skateboard is sliding down on a through the centre of the circles. The particles are
smooth incline having inclination angle q. rotating in same sense. Find the minimum time
He throws a ball such that he catches it back when the two particles will again fall on a straight
after time T. With what velocity was the ball line passing through the centre.
thrown by the boy relative to himself ? Q. 42. A particle moves in xy plane with its position
vector changing with time (t) as

r = ( sin t ) iˆ + ( cos t ) ˆj (in meter)

Find the tangential acceleration of the particle as a
function of time. Describe the path of the particle.

q
Q. 43. Two paper discs are mounted on a rotating vertical
shaft. The shaft rotates with a constant angular
(b) Barrel of an anti aircraft gun is rotating speed w and the separation between the discs is
in vertical plane (it is rotating up from H. A bullet is fired vertically up so that it pierces
the horizontal position towards vertical through the two discs. It creates holes H1 and
orientation in the plane of the fig). The length H2 in the lower and the upper discs. The angular
of the barrel is L = 2 m and barrel is rotating separation between the two holes (measured with
with angular velocity w = 2 rad/s. At the respect to the shaft axis) is q. Find the speed (v)
instant angle q is 45° a shell is fired with a of the bullet. Assume that the speed of the bullet
velocity 2 2 m/s with respect to the exit point does not change while travelling through distance
of the barrel. The tank recoils with speed 4 H and that the discs do not complete even one
m/s. What is the launch speed of the shell as revolution in the interval the bullet pierces
seen from the ground? through them.
V

q
H2


Q. 40. long piece of paper is10 cm wide and is moving


uniformly along its length with a velocity of
H1
2 cm/s. An ant starts moving on the paper from

point A and moves uniformly with respect to the 
paper. A spider was located exactly opposite to the
ant just outside the paper at point B at the instant
the ant started to move on the paper. The spider, Q. 44. (a) A car moves around a circular arc subtending
without moving itself, was able to grab the ant 5 an angle of 60° at the centre. The car moves
second after it (the ant) started to move. Find the at a constant speed u0 and magnitude of its
speed of ant relative to the paper.
Kinematics 2.7

instantaneous acceleration is a0. Find the Q. 47. (a) A point moving in a circle of radius R has
average acceleration of the car over the a tangential component of acceleration that
60° arc. is always n times the normal component of
(b) The speed of an object undergoing uniform acceleration (radial acceleration). At a certain
circular motion is 4 m/s. The magnitude of the instant speed of particle is v0. What is its
change in the velocity during 0.5 sec is also speed after completing one revolution?
4 m/s. Find the minimum possible centripetal (b) The tangential acceleration of a particle
acceleration (in m/s2) of the object. moving in xy plane is given by at = a0 cos q.
Q. 45. A particle is fixed to the edge of a disk that is Where a0 is a positive constant and q is the
rotating uniformly in anticlockwise direction angle that the velocity vector makes with
about its central axis. At time t = 0 the particle is the positive direction of X axis. Assuming
on the X axis at the position shown in figure and it the speed of the particle to be zero at
has velocity v x = 0, find the dependence of its speed on its
y x co-ordinate.
Q. 48. A particle is rotating in a circle. When it is at point
A its speed is V. The speed increases to 2 V by the
v at t = 0
time the particle moves to B. Find the magnitude
of change in velocity of the particle as it travels
  
x
from A to B. Also, find VA DV ; where VA is its

velocity at point A and DV is change in velocity
as it moves from A to B.
Q. 49. A particle starts from rest moves on a circle with
(a) Draw a graph representing the variation of the its speed increasing at a constant rate of . Find
x component of the velocity of the particle as the angle through which it 0.8 ms–2 would have
a function of time. turned by the time its acceleration becomes 1 ms2.
(b) Draw the y-component of the acceleration of Q. 50. In the arrangement shown in the fig, end A of the
the particle as a function of time. string is being pulled with a constant horizontal
velocity of 6 m/s. The block is free to slide on
Q. 46. A disc is rotating with constant angular velocity w
the horizontal surface and all string segments are
in anticlockwise direction. An insect sitting at the
horizontal. Find the velocity of point P on the
centre (which is origin of our co-ordinate system)
thread.
begins to crawl along a radius at time t = 0 with a
constant speed V relative to the disc. At time t = 0 A
6 m/s
the velocity of the insect is along the X direction.

(a) Write the position vector ()
of the insect at P
time ‘t’. 
(b) Write the velocity vector ()
of the insect at Q. 51. In the arrangement shown in the fig, block A is
time ‘t’. pulled so that it moves horizontally along the line
(c) Show that the X component of the velocity AX with constant velocity u. Block B moves along
of the insect become zero when the disc has the incline. Find the time taken by B to reach the
rotated through an angle q given by pulley P if u = 1m/s. The string is inextensible.
1
tanq = . P
q y w
m
12
O A
X X
v u
2m
B q=30
0


2.8 Problems in Physics for JEE Advanced

(c) Find Akanksha’s average speed for covering


Level 2 distance L.
Q. 52. Two friends A and B are running on a circular track (d) How long does it take Harshit to cover the
of perimeter equal to 40 m. At time t = 0 they are distance?
at same location running in the same direction. Q. 56. There are two cars on a straight road, marked as
A is running slowly at a uniform speed of 4.5 x axis. Car A is travelling at a constant speed of
km/hr whereas B is running swiftly at a speed of VA = 9 m/s. Let the position of the Car A, at time
18 km/hr. .
t = 0, be the origin. Another car B is L = 40 m ahead
(a) At what time t0 the two friends will meet of car A at t = 0 and starts moving at a constant
again? acceleration of a = 1 m/s2 (at t = 0). Consider the
(b) What is average velocity of A and B for the length of the two cars to be negligible and treat
interval t = 0 to t = t0? them as point objects.
A $ VA = 9 m/s B $ a = 1m/s2
Q. 53. A particle is moving along x axis. Its position as a
function of time is given by x = x(t). Say whether x
following statements are true or false. x=0

(a) The particle is definitely slowing down if L = 40 m

(a) Plot the position–time (x–t) graph for the


d2x dx
2 > 0 and <0 two cars on the same graph. The two graphs
dt dt intersect at two points. Draw conclusion from
(b) The particle is definitely moving towards the this.
d ( x2 ) (b) Determine the maximum lead that car A can
origin if <0 have.
dt
  Q. 57. Particle A is moving with a constant velocity of
Q. 54. Graph of position (x) vs inverse of velocity   VA = 50 ms–1 in positive x direction. It crossed the
 
for a particle moving on a straight line is as shown. origin at time t = 10 s. Another particle B started
Find the time taken by the particle to move from at t = 0 from the origin and moved with a uniform
x = 3 m to x = 15 m. acceleration of aB = 2 ms –2 in positive x direction.
(a) For how long was A ahead of B during the
x(m) subsequent journey?
(b) Draw the position (x) time (t) graph for the
15 two particles and mark the interval for which
A was ahead of B.
Q. 58. (a) A particle is moving along the x axis and its
velocity vs position graph is as shown. Is
the acceleration of the particle increasing,
3 decreasing or remains constant?
1 v
O (sm–1)
2 8 v

Q. 55. Harshit and Akanksha both can run at speed v


and walk at speed u(u < v). They together start on
a journey to a place that is at a distance equal to L.
Akanksha walks half of the distance and runs the x
second half. Harshit walks for half of his travel (b) A particle is moving along x axis and its
time and runs in the other half. velocity (v) vs position (x) graph is a curve
(a) Who wins? as shown in the figure. Line APB is normal to
(b) Draw a graph showing the positions of both the curve at point P. Find the instantaneous
Harshit and Akanksha versus time. acceleration of the particle at x = 3.0 m.
Kinematics 2.9

v (m/s) been shown in the figure. Find the time when the
two particles collide. Also find the position (x)
A where they collide. It is given that x0 = ut0, and
that the particle 2 was at origin at t = 0.
P
Q. 62. Two stations A and B are 100km apart. A passenger
train crosses station A travelling at a speed of
x (in m)
50 km/hr. The train maintains constant speed for
O C B 1 hour 48 minute and then the driven applies
(3,0) (4,0)
brakes to stop the train at station B in next 6 minute.
Q. 59. A particle has co-ordinates (x, y). Its position Another express train starts from station B at the
vector makes on angle q with positive x direction. time the passenger train was crossing station A.
In an infinitesimally small interval of time the The driver of the express train runs the train with
particle moves such that length of its position uniform acceleration to attain a peak speed v0.
vector does not change but angle q increases by Immediately after the train attains the peak speed
dq. Express the change in position vector of the v0, he applies breaks which cause the train to stop
particle in terms of x, y, dq and unit vectors î at station A at the same time the passenger train
and ĵ . stops at B. Brakes in both the trains cause uniform
y retardation of same magnitude. Find the travel
time of two trains and v0.
P(x,y)
Q. 63. Particle A starts from rest and moves along a
straight line. Acceleration of the particle varies
r with time as shown in the graph. In 10 s the
velocity of the particle becomes 60 m/s and the
q acceleration drops to zero. Another particle B
O
x starts from the same location at time t = 1.1 s and
has acceleration – time relationship identical to A
Q. 60. A rope is lying on a table with one of its end at
with a delay of 1.1 s. Find distance between the
point O on the table. This end of the rope is pulled
particles at time t = 15 s.
to the right with a constant acceleration starting
a
from rest. It was observed that last 2 m length of
the rope took 5 s in crossing the point O and the
last 1m took 2 s in crossing the point O. B
A

O
(a) Find the time required by the complete rope
O t(s)
to travel past point O. 1.1 10

(b) Find length of the rope. Q. 64.


Q. 61. ax
x v ay

x0 u a0 a0

O t O t
t0 2t0 3t0 4t0 t0 2t0 3t0 4t0

t t
O t0 O t0 A particle is moving in x–y plane. The x and y
components of its acceleration change with time
Two particles 1 and 2 move along the x axis. The according to the graphs given in figure. At time
position (x) - time (t) graph for particle 1 and t = 0, its velocity is v0 directed along positive
velocity (v) - time (t) graph for particle 2 has
2.10 Problems in Physics for JEE Advanced
v0
y direction. If a0 , find the angle that the bodies 1 and 2 value of b is 2.5 kg/s and 3.0 kg/s
t0 respectively. Neglect all other forces apart from
velocity of the particle makes with x axis at time gravity and the resistive force, while answering
t = 4t0. following questions : [Hint : acceleration =
Q. 65. A particle is moving along positive x direction force/mass]
and experiences a constant acceleration of (i) With what speed v10 and v20 will the two
4 m/s2 in negative x direction. At time t = 3 second bodies hit the bed of the pond.
its velocity was observed to be 10 m/s in positive [Take g = 10 m/s2]
x direction.
(ii) Which body will acquire speed equal to half
(a) Find the distance travelled by the particle in the terminal speed in less time.
the interval t = 0 to t = 3 s. Also find distance
travelled in the interval t = 0 to t = 7.5 s.. Q. 69. A prototype of a rocket is fired from the ground.
The rocket rises vertically up with a uniform
(b) Plot the displacement – time graph for the 5
interval t = 0 to 7.5 s. acceleration of m/s2. 8 second after the start
4
Q. 66. A bead moves along a straight horizontal wire of a small nut gets detached from the rocket. Assume
length L, starting from the left end with velocity that the rocket keeps rising with the constant
v0. Its retardation is proportional to the distance acceleration.
that remains to the right end of the wire. Find (a) What is the height of the rocket at the instant
the initial retardation (at left end of the wire) if the nut lands on the ground
the bead reaches the right end of the wire with a
(b) Plot the velocity – time graph for the motion
v0
velocity . of the nut after it separates from the rocket
2 till it hits the ground. Plot the same velocity–
Q. 67. A ball is projected vertically up from the ground time graph in the reference frame of the
surface with an initial velocity of u = 20 m/s. O rocket. Take vertically upward direction as
is a fixed point on the line of motion of the ball positive and g = 10 m/s2
at a height of H = 15 m from the ground. Plot a Q. 70. An elevator starts moving upward with constant
graph showing variation of distance (s) of the ball acceleration. The position time graph for the
from the fixed point O, with time (t). [Take g = 10 floor of the elevator is as shown in the figure. The
m/s2]. Plot the graph for the entire time of flight ceiling to floor distance of the elevator is 1.5 m.
of the ball. At t = 2.0 s, a bolt breaks loose and drops from the
ceiling.
(a) At what time t0 does the bolt hit the floor?
O
(b) Draw the position time graph for the bolt
starting from time t = 0.
[take g = 10 m/s2]
H y (In meter)

4.0

Q. 68. Two bodies 1 and 2 of different shapes are


released on the surface of a deep pond. The mass O t (In second)
2.0
of the two bodies are m1 = 1 kg and m2 = 1.2 kg
respectively. While moving through water, the Q. 71. At t = 0 a projectile is projected vertically up with
bodies experience resistive force given as R = bv, a speed u from the surface of a peculiar planet.
where v is speed of the body and b is a positive The acceleration due to gravity on the planet
constant dependent on shape of the body. For changes linearly with time as per equation g = at
where a is a constant.
Kinematics 2.11

(a) Find the time required by the projectile to y(m) vy


attain maximum height. v0
(b) Find maximum height attained.
38.4
x
(c) Find the total time of flight. 19.2

Q. 72. A wet ball is projected horizontally at a speed of –v0


u = 10 m/s from the top of a tower h = 31.25 m
t(s)
high. Water drops detach from the ball at regular O 2.4
intervals of Dt = 1.0 s after the throw. (a) Find the angle of projection a
(a) How many drops will detach from the ball (b) Find the inclination (q) of the incline.
before it hits the ground.
(c) If the ball is projected with same speed but
(b) How far away the drops strike the ground
at an angle q (= inclination of incline) to the
from the point where the ball hits the ground?
horizontal, will it hit the incline above or
Q. 73. Two stones of mass m and M (M > m) are dropped below the point where it struck the incline
Dt time apart from the top of a tower. Take time earlier?
t = 0 at the instant the second stone is released.
Q. 76. (i) A canon can fire shells at speed u. Inclination
Let Dv and Ds be the difference in their speed
of its barrel to the horizontal can be changed
and their mutual separation respectively. Plot the
in steps of q = 1° ranging from q1 = 15° to
variation of Dv and Ds with time for the interval
q2 = 85°. Let Rn be the horizontal range for
both the stones are in flight. [g = 10 m/s2]
projection angle q = n°.
Q. 74. A particle is moving in the xy plane on a sinusoidal
∆Rn = Rn − Rn +1
course determined by y = A sin kx, where k and A
are constants. The X component of the velocity For what value of n the value of Rn is
of the particle is constant and is equal to v0 and maximum? Neglect air resistance.
the particle was at origin at time t = 0. Find the (ii) A small water sprinkler is in the shape of a
magnitude of the acceleration of the particle when hemisphere with large number of uniformly
p spread holes on its surface. It is placed on
it is at point having x co ordinate x = . ground and water comes out of each hole
2k
with speed u. Assume that we mentally divide
Q. 75. A ball is projected from a cliff of height h = 19.2 the ground into many small identical patches
m at an angle a to the horizontal. It hits an incline – each having area S. What is the distance
passing through the foot of the cliff, inclined at of a patch from the sprinkler which receives
an angle q to the horizontal. Time of flight of the maximum amount of water ?
ball is T = 2.4 s. Foot of the cliff is the origin of
the co-ordinate system, horizontal is x direction A gun fires a large number of bullets upward.
and vertical is y direction (see figure). Plot of y Due to shaking of hands some bullets deviate as
co-ordinate vs time and y component of velocity much as 1° from the vertical. The muzzle speed
of the ball (vy) vs its x co-ordinate (x) is as shown. of the gun is 150 m/s and the height of gun above
x and y are in m and time is in s in the graph. the ground is negligible. The radius of the head of
[g = 10 m/s2] the person firing the gun is 10cm. You can assume
y that acceleration due to gravity is nearly constant
for heights involved and its value is g = 10 m/s2.
The gun fires 1000 bullets and they fall uniformly
over a circle of radius r. Neglect air resistance.
A a
You can use the fact sin q ~ q when q is small.
(a) Find the approximate value of r.
(b) What is the probability that a bullet will fall
h
on the person’s head who is firing?

q
Three stones are projected simultaneously with
x
O same speed u from the top of a tower. Stone 1 is
2.12 Problems in Physics for JEE Advanced

projected horizontally and stone 2 and stone 3 are P


projected making an angle q with the horizontal
as shown in fig. Before stone 3 hits the ground, the
distance between 1 and 2 was found to increase at
a constant rate u. u u
2
A q a q B
O
q
1 x0
q
(b) Calculate the inclination of the wall to the
3 horizontal (a) [g = 10 m/s2]
Q. 81. AB is a pipe fixed to the ground at an inclination of
37°. A ball is projected from point O at a speed of
u = 20m/s at an angle of 53° to the horizontal and
it smoothly enters into the pipe with its velocity
parallel to the axis of the pipe. [Take g = 10 ms–2]
(a) Find q
(b) Find the rate at which the distance between 2
and 3 increases.
A
u
A horizontal electric wire is stretched at a height

L
h = 10 m above the ground. A boy standing on the
ground can throw a stone at a speed u = 20 ms –1. 53° 37° B
Find the maximum horizontal distance x at which O
a bird sitting on the wire can be hit by the stone.
(a) Find the length L of the pipe
x
(b) Find the distance of end B of the pipe from
point O.
Q. 82. (a) A boy throws several balls out of the
window of his house at different angles to
h u
the horizontal. All balls are thrown at speed
u = 10 m/s and it was found that all of them
hit the ground making an angle of 45° or
q
larger than that with the horizontal. Find the
height of the window above the ground [take
Q. 80. A wall OP is inclined to the horizontal ground g = 10 m/s2]
at an angle a. Two particles are projected from

(b) A gun is mounted on an elevated platform
points A and B on the ground with same speed (u)
AB. The distance of the gun at A from the
in directions making an angle q to the horizontal
edge B is AB = 960 m. Height of platform
(see figure). Distance between points A and B is
is OB = 960 m. The gun can fire shells with
x0 = 24 m. Both particles hit the wall elastically
a velocity of u = 100 m/s at any angle. What
and fall back on the ground. Time of flight (time
is the minimum distance (OP) from the foot
required to hit the wall and then fall back on to the of the platform where the shell of gun can
ground) for particles projected from A and B are reach?
4 s and 2 s respectively. Both the particles strike u
the wall perpendicularly and at the same location.
[In elastic collision, the velocity component of B
the particle that is perpendicular to the wall gets A
reversed without change in magnitude]
(a) Calculate maximum height attained by the O P
particle projected from A. Q. 83 An object A is kept fixed at the point x = 3 m
Kinematics 2.13

and y = 1.25 m on a plank P raised above the Q. 85. A city bus has a horizontal rectangular roof and
ground. At time t = 0 the plank starts moving a rectangular vertical windscreen. One day it was
along the + x direction with an acceleration raining steadily and there was no wind.
1.5 m/s2. At the same instant a stone is projected (a) Will the quantity of water falling on the roof
from the origin with a velocity u as shown. A in unit time be different for the two cases (i)
stationary person on the ground observes the the bus is still (ii) the bus is moving with
stone hitting the object during its downwards speed v on a horizontal road ?
motion at an angle of 45º to the horizontal. All the
motions are in x-y plane. Find u and the time after (b) Draw a graph showing the variation of
which the stone hits the object. Take g = 10 m/s2 quantity of water striking the windscreen in
A
unit time with speed of the bus (v).
y
P Q. 86. A truck is travelling due north descending a hill of
1.25 m slope angle q = tan–1 (0.1) at a constant speed of
90 km/hr. At the base of the hill there is a gentle
curve and beyond that the road is level and heads
u
30° east of north. A south bound police car is
x travelling at 80 km/hr along the level road at the
O 3m
base of the hill approaching the truck. Find the
velocity of the truck relative to police car in terms
Q. 84. (a) A particle is thrown from a height h
horizontally towards a vertical wall with a of unit vectors iˆ, ˆj and k̂ . Take x axis towards
speed v as shown in the figure. If the particle east, y axis towards north and z axis vertically
returns to the point of projection after upwards.
suffering two elastic collisions, one with the Q. 87. Two persons A and B travelling at 60 km/hr–1
wall and another with the ground, find the in their cars moving in opposite directions on a
total time of flight. [Elastic collision means straight road observe an airplane. To the person A,
the velocity component perpendicular to the the airplane appears to be moving perpendicular
surface gets reversed during collision.] to the road while to the observe B the plane
appears to cross the road making an angle of 45°.
v
(a) At what angle does the plane actually cross
h the road (relative to the ground).
(b) Find the speed of the plane relative to the
(b) Touching a hemispherical dome of radius R ground.
there is a vertical tower of height H = 4 R.
A boy projects a ball horizontally at speed Q. 88.
u from the top of the tower. The ball strikes C

R
the dome at a height from ground and
2
rebounds. After rebounding the ball retraces l u
back its path into the hands of the boy. Find u.
u

A B
L
Two friends A and B are standing on a river bank
H L distance apart. They have decided to meet
at a point C on the other bank exactly opposite
to B. Both of them start rowing simultaneously
on boats which can travel with velocity
R
V = 5 km/hr in still water. It was found that both
O reached at C at the same time. Assume that path of
2.14 Problems in Physics for JEE Advanced

both the boats are straight lines. Width of the river ground at the instant when the particle was
is l = 3.0 km and water is flowing at a uniform projected.
speed of u = 3.0 km/hr. Q. 91. A ball is projected in vertical x–y plane from a
(a) In how much time the two friends crossed the car moving along horizontal x direction. The car
river. is speeding up with constant acceleration. Which
(b) Find L. one of the following trajectory of the ball is not
possible in the reference frame attached to the
Q. 89. On a frictionless horizontal surface, assumed to be car? Give reason for your answer. Explain the
the x-y plane, a small trolley A is moving along a condition in which other trajectories are possible.
straight line parallel to the y-axis (see figure) with Consider origin at the point of projection.
a constant velocity of ( 3 – 1) m/s. At a particular y y
instant, when the line OA makes an angle of 45°
with the x-axis, a ball is thrown along the surface
from the origin O. Its velocity makes an angle f
with the x-axis and it hits the trolley.
y x x
A
(a) (b)

y
y

45°
O x

(a) The motion of the ball is observed from the


x x
frame of the trolley. Calculate the angle q (c) (d)
made by the velocity vector of the ball with
the x-axis in this frame. Q. 92. A boy standing on a cliff 50 m high throws a ball
with speed 40 m/s directly aiming towards a man
(b) Find the speed of the ball with respect to the
standing on ground at B. At the same time the

surface, if φ = . man at B throws a stone with a speed of 10 m/s
3
directly aiming towards the boy.
Q. 90. A large heavy box is sliding without friction
down a smooth plane having inclination angle q. A
From a point P at the bottom of a box, a particle is
projected inside the box. The initial speed of the
particle with respect to box is u and the direction
of projection makes an angle a with the bottom as
50 m
shown in figure


P Q
C B
50 m

 (a) Will the ball and the stone collide? If yes, at


(a) Find the distance along the bottom of the box what time after projection?
between the point of projection P and the (b) At what height above the ground the two
point Q where the particle lands. (Assume objects collide?
that the particle does not hit any other surface (c) Draw the path of ball in the reference frame
of the box. Neglect air resistance) of the stone.
(b) If the horizontal displacement of the particle Q. 93. A man walking downhill with velocity V0 finds
as seen by an observer on the ground is zero, that his umbrella gives him maximum protection
find the speed of the box with respect to the from rain when he holds it such that the stick is
Kinematics 2.15

perpendicular to the hill surface. When the man applied and the train begins to retard at a uniform
turns back and climbs the hill with velocity V0, rate that is equal to its previous acceleration (a)
he finds that it is most appropriate the hold the
(a) Will the ball hit the wall BC or wall CD or the
umbrella stick vertical. Find the actual speed of corner C?
raindrops in terms of V0. The inclination of the
hill is q = 37°.
(b) What is speed of the ball, relative to the
wagon at the instant it hits a wall ?
Q. 96. Five particles are projected simultaneously from
the top of a tower that is h = 32 m high. The initial
velocities of projection are as shown in figure.
V0 Velocity of 2 and 5 are horizontal.
15 m/s
V0
4 3 10 m/s

53° 37°
5
10 m/s
15 m/s 37° 2
q 
1
Q. 94. There are two hills A and B and a car is travelling 10 m/s

towards hill A along the line joining the two


hills. Car is travelling at a constant speed u. There
h = 32m
is a wind blowing at speed u in the direction of
motion of the car (i.e., from hill B to A). When
the car is at a distance x1 from A and x2 from B
it sounds horn (for very short interval). Driver
hears the echo of horn from both the hills at the
same time.
(a) Which particle will hit the ground first?
Wind (u)
(b) Separation between which two particles is
x2 x1
maximum at the instant the first particle hits
the ground?
(c) Which two particles are last and last but one
B u A to hit the ground? Calculate the distance
between these two particles (still in air), at
1
a time 0.3s after the third particle lands on
Find the ratio taking speed of sound in still air ground.
to be V. 2
3
[g = 10 m/s2, tan 37° = ]
Q. 95. The figure shows a square train wagon ABCD 4
which has a Q. 97. From the top of a long smooth incline a small
C D body A is projected along the surface with speed
smooth floor and
side length of 2 L. u. Simultaneously, another small object B is
The train is mov- thrown horizontally with velocity v = 10 m/s,
ing with uniform 2L
from the same point. The two bodies travel in the
acceleration (a) in same vertical plane and body B hits body A on the
a direction parallel incline. If the inclination angle of the incline is
to DA. A 'ball is u 4
B A θ = cos −1   find
rolled along the 5
2L
floor with a veloci-
(a) the speed u with which A was projected.
ty u, parallel to AB, with respect to the wagon.
The ball passes through the centre of the wagon (b) the distance from the point of projection,
floor. At the instant it is at the centre, brakes are where the two bodies collide.
2.16 Problems in Physics for JEE Advanced

B V where q is the angle made by position vector


A of the rocket with respect to the vertical.
y
u
Rocket

r
q

Q. 98. A man is on straight road AC, standing at A. He


wants to get to a point P which is in field at a
distance ‘d’ off the road (see figure). Distance AB x

is l = 50. The man can run on the road at a speed (a) Neglect atmospheric resistance and take
v1 = 5 m/s and his speed in the field is v2 = 3 m/s. g = 9.8 m/s2 at the concerned height. Neglect
l height of radar. Calculate the height of the
A B C
rocket above the ground.
(b) Two points A and B are moving in X - Y plane
(
with constant velocity of VA = 6iˆ - 9 ˆj m/s )
d
( )
and V = iˆ + ˆj m/s respectively. At time
B

t = 0 they are 15 m apart and both of them


lie on y axis with A lying away on positive
P Y axis with respect to B. What is the angular
velocity of A with respect to B at t = 1 s?
(a) Find the minimum value of ‘d’ for which man
can reach point P in least possible time by Q. 101. A stone is projected horizontally with speed u
travelling only in the field along the straight from the top of a tower of height h.
line AP. (a) Calculate the radius of curvature of the path
(b) If value of ‘d’ is half the value found in (a), of the stone at the point where its tangential
what length the man must run on the road and radial accelerations are equal.
before entering the field, in order to reach ‘P’ (b) What shall be the height (h) of the tower so
in least possible time. that radius of curvature of the path is always
Q. 99. Two particles, A and B are moving in concentric less than the value obtained in (a) above.
circles in anticlockwise sense in the same plane Q. 102. A stick of length L = 2.0 m is leaned against a
with radii of the circles being gA = 1.0 m and gB = wall as shown. It is released from a position when
2.0 m respectively. The particles move with same q = 60°. The end A of the stick remains in contact
angular speed of w = 4 rad/s. with the wall and its other end B remains in
Find the angular velocity of B as observed by A if contact with the floor as the stick slides down.
Find the distance travelled by the centre of the
(a) Particles lie on a line passing through the
stick by the time it hits the floor.
centre of the circle.
A
(b) Particles lie on two perpendicular lines
passing through the centre.
Q. 100. (a) An unpowered rocket is in flight in air. At a
moment the tracking radar gives following
data regarding the rocket.
r = distance of the rocket from the radar =
q
dr dθ
4000 m, = 0, = 1.8 deg/sec; B
dt dt
Kinematics 2.17

Q. 103. (a) A line PQ is moving on a fixed circle of ball is 10 m/s parallel to the incline XO.
radius R. The line has a constant velocity v x
perpendicular to itself. Find the speed of B
point of intersection (A) of the line with the
y
circle at the moment the line is at a distance 10
d = R/2 from the centre of the circle. m/
s
V

A 60° 30°
P Q
d O

Q. 106. A meter stick AB is lying on a horizontal table. Its


end A is pulled up so as to move it with a constant
velocity VA = 4ms–1 along a vertical line. End B
slides along the floor.
(b) In the figure shown a pin P is confined to VA

move in a fixed circular slot of radius R. The


pin is also constrained to remains inside the A

slot in a straight arm O'A. The arm moves


with a constant angular speed w about the
hinge O'. What is the acceleration of point
P?
B
A B
P
(a) After how much time (t0) speed (VB) of end B
becomes equal to the speed (VA) of end A ?
O' O (b) Find distance travelled by the end B in
R R time t0.
Q. 107. One end of a rope is fixed at a point on the ceiling
the other end is held close to the first end so that
the rope is folded. The second end is released
C from this position. Find the speed at which the
Q. 104. A flexible inextensible cord supports a mass M fold at F is descending at the instant the free end
as shown in figure. A1, A2 and B are small pulleys of the rope is going down at speed V.
in contact with the cord. At time t = 0 cord PQ
is horizontal and A1, A2 start moving vertically
down at a constant speed of v1, whereas B moves
up at a constant speed of v2. Find the velocity of
mass M as a function of time. V
L L L L
P Q
A1 A2
B

Q. 108. Block A rests on inclined surface of wedge B


M which rests on a horizontal surface. The block A is
connected to a string, which passes over a pulley
Q. 105. In the arrangement shown in the figure A is an P (fixed rigidly to the wedge B) and its other end
equilateral wedge and the ball B is rolling down is securely fixed to a wall at Q. Segment PQ of
the incline XO. Find the velocity of the wedge (of the string is horizontal and Q is at a large distance
course, along OY) at the moment velocity of the
2.18 Problems in Physics for JEE Advanced

from P. The system is let go from rest and the Q. 111. While starting from a station, a train driver was
wedge slides to right as A moves on its inclined instructed to stop his train after time T and to
face. Find the distance travelled by A by the time cover maximum possible distance in that time.
it reaches the bottom of the inclined surface. (a) If the maximum acceleration and retardation
P Q for the train are both equal to ‘a’, find the
A
maximum distance it can cover.
5m
(b) Will the train travel more distance if
maximum acceleration is ‘a’ but the
C q = 30° B
maximum retardation caused by the brakes is
‘2a’? Find this distance.
Q. 109. Two frictionless ropes connect points A & B in Q. 112. Two particles 1 and 2 start simultaneously from
vertical plane. Bead 1 is allowed to slide along origin and move along the positive X direction.
the straight rope AB and bead 2 slides along the Initial velocity of both particles is zero. The
curved rope ACB. Which bead will reach B in less acceleration of the two particles depends on their
time? displacement (x) as shown in fig.
a1 a2
A
1
2a0 2a0

a0 a0
2
O X0 X O X0 X
C B

(a) Particles 1 and 2 take t1 and t2 time respectively


Level 3 for their displacement to become x0. Find 2
.
1

Q. 110. A car manufacturer usually tells a optimum speed (b) Which particle will cover 2x0 distance in
(V0) at which the car should be driven to get least time? Which particle will cross the point
maximum mileage. In order to find the optimum x = 2x0 with greater speed?
speed for a new model, an engineer of the car (c) The two particles have same speed at a certain
company experimented a lot and finally plotted time after the start. Calculate this common
a graph between the extreme time t (defined as speed in terms of a0 and x0.
number of hours a tank full of petrol lasts) vs the
Q. 113. A cat is following a rat. The rat is running with a
constant speed V at which car was run.
constant velocity u. The cat moves with constant
t (hour) speed v with her velocity always directed towards
the rat. Consider time to be t = 0 at an instant when
both are moving perpendicular to each other and
20
separation between them is L.
(a) Find acceleration of the cat at t = 0.
(b) Find the time t0 when the rat is caught.
(c) Find the acceleration of the cat immediately
4 before it catches the rat.
3
O 10 150
-1
V(km hr ) (d) Draw the path of the rat as seen by the
cat.
(a) Calculate the optimum speed V0 for this new Q. 114.(a) Prove that bodies starting at the same time t = 0
model. from the same point, and following frictionless
(b) If the fuel tank capacity of this car is 50 litre, slopes in different directions in the same
what maximum mileage can be obtained vertical plane, all lie in a circle at any
from this car? subsequent time.
Kinematics 2.19

(b) Using the above result do the following Q. 118. A valley has two walls inclined at 37° and 53° to
problem. A point P lies above an inclined the horizontal. A particle is projected from point
plane of inclination angle a. P is joined to the P with a velocity of u = 20 m/s along a direction
plane at number of points by smooth wires, perpendicular to the incline wall OA. The Particle
running in all possible directions. Small hits the incline surface RB perpendicularly at Q.
bodies (in shape of beads) are released from Take g = 10 m/s2 and find:
P along all the wires simultaneously. Which (a) The time of flight of the particle.
body will take least time to reach the plane.
P
(b) Vertical height h of the point P from
horizontal surface OR.
 3
 tan 37° = 4 
 
B

A u
a Q

Q. 115. The acceleration due to gravity near the surface of


 
the earth is g . A ball is projected with velocity u h
from the ground. 37° 53°
O R
(a) Express the time of flight of the ball.
(b) Write the expression of average velocity of Q. 119.
the ball for its entire duration of flight.
 
Express both answers in terms of u and g .
h
Q. 116. A ball is projected from point O on the ground.
It hits a smooth vertical wall AB at a height h
and rebounds elastically. The ball finally lands
at a point C on the ground. During the course of
motion, the maximum height attained by the ball
is H.
B 

A ball is released in air above an incline plane


inclined at an angle a to the horizontal. After
falling vertically through a distance h it hits the
incline and rebounds. The ball flies in air and
C
then again makes an impact with the incline. This
O A
way the ball rebounds multiple times. Assume
h OA 1 that collisions are elastic, i.e., the ball rebound
(a) Find the ratio if
H OC 3 without any loss in speed and in accordance to
(b) Find the magnitude of average acceleration the law of reflection.
of the projectile for its entire course of flight (a) Distance between the points on the incline
if it was projected at an angle of 45° to the where the ball makes first and second impact
horizontal. is l1 and distance between points where the
Q. 117. A boy can throw a ball up to a speed of
ball makes second and third impact is l2.
u = 30 m/s . He throws the ball many a times, Which is large l1 or l2?
ensuring that maximum height attained by the ball (b) Calculate the distance between the points on
in each throw is h = 20 m. Calculate the maximum the incline where the ball makes second and
horizontal distance at which a ball might have fifth impact.
landed from the point of projection. Neglect the Q. 120. A terrorist ‘A’ is walking at a constant speed of
height of the boy. [g = 10 m/s2] 7.5 km/hr due West. At time t = 0, he was exactly
2.20 Problems in Physics for JEE Advanced

South of an army camp at a distance of 1 km. At blowing at a speed of . Mark the area on the
this instant a large number of army men scattered
in every possible direction from their camp in ground that the sprinkler will now be able to wet.
search of the terrorist. Each army person walked
in a straight line at a constant speed of 6 km/hr.
(a) What will be the closest distance of an
army person from the terrorist in this search O
operation?
(b) At what time will the terrorist get nearest to
an army person?
Q. 124. A cylinder of radius R has been placed in a corner
Q. 121. A large wedge BCD, having its inclined surface
as shown in the fig. A wedge is pressed against
at an angle q = 45° to the horizontal, is travelling
the cylinder such that its inclined surfaces touches
horizontally leftwards with uniform velocity
2
u = 10 m/s the cylinder at a height of from the ground.
u 5
C Now the wedge is pushed to the left at a constant
speed V = 15 m/s. With what speed will the
cylinder move?
V V

B 45° D
A

At some instant a particle is projected vertically


up with speed V = 20 m/s from point A on ground
lying at some distance right to the lower edge B R
of the wedge. The particle strikes the incline BC
normally, while it was falling. [g = 10 m/s2]
(a) Find the distance AB at the instant the particle 2R/5
was projected from A.
(b) Find the distance of lower edge B of the Q. 125. The entrance to a harbour consists of 50 m gap
wedge from point A at the instant the particle between two points A and B such that B is due
strikes the incline. east of A. Outside the harbour there is a 8 km/hr
(c) Trace the path of the particle in the reference current flowing due east. A motor boat is located
frame attached to the wedge. 300 m due south of A. Neglect size of the boat for
answering following questions-
Q. 122. The speed of river current close to banks is nearly
zero. The current speed increases linearly from (a) Calculate the least speed (Vmin) that the motor
the banks to become maximum (= V0) in the boat must maintain to enter the harbour.
middle of the river. A boat has speed ‘u’ in still (b) Show that the course it must steer when
water. It starts from one bank and crosses the moving at Vmin does not depend on the speed
river. Its velocity relative to water is always kept of the current.
perpendicular to the current. Find the distance
Q. 126. Two small pegs (A and B) are at horizontal and
through which the boat will get carried away by
vertical separation b and h respectively. A small
the current (along the direction of flow) while it
block of mass M is suspended with the help of
crosses the river. Width of the river is l.
two light strings passing over A and B as shown in
Q. 123. A water sprinkler is positioned at O on horizontal fig. The two string are always kept at right angles
ground. It issues water drops in every possible (i.e., <APB = 90°). Find the minimum possible
direction with fixed speed u. This way the gravitation potential energy of the mass assuming
sprinkler is able to completely wet a circular area the reference level at location of peg A. [Hint: the
of the ground (see fig). A horizontal wind starts potential energy is minimum when the block is at
Kinematics 2.21

its lowest position] Q. 130. In the arrangement shown in the figure, the block
b C begins to move down at a constant speed of
7.5 cm/s at time t = 0. At the same instant block
B
A is made to start moving down at constant
h acceleration. It starts at M and its speed is 30 cm/s
A when it reaches N (MN = 20 cm). Assuming that
B started from rest, find its position, velocity and
P acceleration when block A reaches N.
M

Q. 127. (a) A canon fires a shell up on an inclined plane.


Prove that in order to maximize the range
along the incline the shell should be fired in
a direction bisecting the angle between the
incline and the vertical. Assume that the shell
fires at same speed all the time.
(b) A canon is used to hit a target a distance R A
up an inclined plane. Assume that the energy M
used to fire the projectile is proportional to
square of its projection speed. Prove that the C B
angle at which the shell shall be fired to hit 20 cm
the target but use the least amount of energy
is same as the angle found in part (a)
u N

Q. 131. A rocket prototype is fired from ground at time


t = 0 and it goes straight up. Take the launch
point as origin and vertically upward direction as
Q. 128. A ball of mass m is projected from ground making positive x direction. The acceleration of the rocket
an angle q to the horizontal. There is a horizontal is given by
wind blowing in the direction of motion of the
g
ball. Due to wind the ball experiences a constant a = - kt 2 ; 0 < t £ t0

mg 2
horizontal force of in direction of its motion. = - g; t > t0
Find q for which the horizontal range of the ball
will be maximum. 3g
Where t0
Q. 129. A projectile is projected from a level ground 2k
making an angle q with the horizontal (x (a) Find maximum velocity of the rocket during
direction). The vertical (y) component of its the up journey.
velocity changes with its x co-ordinate according (b) Find maximum height attained by the rocket.
to the graph shown in figure. Calculate q. Take (c) Find total time of flight.
g = 10 ms–2.
vy (m/s) Q. 132. A man standing inside a room of length L rolls a
ball along the floor at time t = 0. The ball travels
at constant speed v relative to the floor, hits the
front wall (B) and rebounds back with same
speed v. The man catches the ball back at the wall
45° 10 A at time t0. The ball travelled along a straight
x (In m)
O line relative to the man inside the room. Another
observer standing outside the room found that
the entire room was travelling horizontally at
constant velocity v in a direction parallel to the
2.22 Problems in Physics for JEE Advanced

two walls A and B. rim of the umbrella has a radius of r = 0.5 m and
B
v
it is at a height of H = 1.8 m from the floor. The
man holding the umbrella gradually increases the
angular speed to make it 2 w. Calculate the area of
the floor that will get wet due to water drops spun
off the rim and hitting the floor. [g = 10 m/s2]
L

(a) Find the average speed of the ball in the time


interval t = 0 to t = t0 as observed by the r = 0.5 m
observer outside the room.
(b) If the room has acceleration in the direction H = 1.8 m
of its velocity draw a sketch of the path of
the ball as observed by the observer standing
outside. Assume that velocity of room was v
at the instant the ball was released.
Q. 133. There is a tall cylindrical building standing in a Q. 135. A ball is projected vertically up from ground. Boy
field. Radius of the cylinder is R = 8 m. A boy A standing at the window of first floor of a nearby
standing at A (at a distance of 10 m from the building observes that the time interval between
centre of the cylindrical base of the building) the ball crossing him while going up and the ball
knows that his friend is standing at B behind the crossing him while going down is t1. Another
building. The line joining A and B passes through boy B standing on the second floor notices that
the centre of the base of the building. Distance time interval between the ball passing him twice
between A and B is 50 m. A wants to throw a ball (during up motion and down motion) is t2.
to B but he realizes that the building is too tall and (a) Calculate the height difference (h) between
he cannot throw the ball over it. He throws the the boy B and A.
ball at a speed of 20 m/s such that his friend at B
(b) Assume that the height of boy A from the
has to move minimum distance to catch it.
point of projection of the ball is also equal to
h and calculate the speed with which the ball
R = 8m was projected.
Q. 136. A stick of length L is dropped from a high tower.
An ant sitting at the lower end of the stick begins
10 m 40 m to crawl up at the instant the stick is released.
Velocity of the ant relative to the stick remains
A O B constant and is equal to u. Assume that the stick
(a) What is the minimum distance that boy at B remains vertical during its fall, and length of the
stick is sufficiently long.
will have to move to catch the ball?
(b) At what angle to the horizontal does the boy
at A throws the ball?
Assume that the ball is released and caught at
same height above the ground. L
2 –1
[Take g = 10 m/s and sin (0.75) ~ 48.6°
Q. 134. A wet umbrella is held upright (see figure). The
man holding it is rotating it about its vertical
shaft at an angular speed of w = 5 rad s–1. The
Kinematics 2.23

(a) Calculate the maximum height attained by the ant an insect jumps from point A on the windshield,
measured from its initial position. with a velocity u = 2.64 m/s (relative to ground)
(b) What time after the start the ant will be at the in vertically upward direction. It falls back at
same height from where it started? point B on the windshield. Calculate distance AB.
Assume that the insect moves freely under gravity
Q. 137. Two balls are projected simultaneously from the and g = 10 m/s2.
top of a tall building. The first ball is projected
horizontally at speed u1 = 10 m/s and the other
4
one is projected at an angle θ = tan −1   to the
B

3
horizontal with a velocity u2. [g = 10 m/s2] A
u2
37°

q
u1
a = 5 m/s2

Q. 140. Two persons are pulling a heavy block with the


help of horizontal inextensible strings. At the
instant shown, the velocities of the two persons
are v1 and v2 directed along the respective strings
(a) Find minimum value of u2 (= u0) so that with the strings making an angle of 60° between
the velocity vector of the two balls can get them.
perpendicular to each other at some point of
(a) Find the speed of the block at the instant
time during their course of flight.
shown.
(b) Find the time after which velocities of the (b) For what ratio of v1 and v2 the instantaneous
two balls become perpendicular if the second velocity of the block will be along the
one was projected with speed u0. direction of v1.
Q. 138. There is a large wedge placed on a horizontal v1
surface with its incline face making an angle of
37° to the horizontal. A particle is projected in
vertically upward direction with a velocity of u
= 6.5 m/s from a point O on the inclined surface. 60°
At the instant the particle is projected, the wedge
begins to move horizontally with a constant
acceleration of a = 4 m/s2. At what distance from
point O will the particle hit the incline surface if v2

(i) direction of a is along BC? Q. 141. A heavy block 'B' is sliding with constant velocity
u on a horizontal table. The width of the block is
(ii) direction of a is along AB?
L. There is an insect A at a distance d from the
block as shown in the figure. The insect wants to
cross to the opposite side of the table. It begins to
A crawl at a constant velocity v at the instant shown
in the figure. Find the least value of v for which
O 37° the insect can cross to the other side without
getting hit by the block.
C B
u
B
Q. 139. The windshield of a truck is inclined at 37° to
the horizontal. The truck is moving horizontally L
with a constant acceleration of a = 5 m/s2. At the
d A
instant the velocity of the truck is v0 = 0.77 m/s,
2.24 Problems in Physics for JEE Advanced

Q. 142. A projectile is thrown from ground at a speed v0 A projectile is thrown from a point on ground,
at an angle a to the horizontal. Consider point with initial velocity u at some angle to the
of projection as origin, horizontal direction as X horizontal. Show that it can clear a pole of height
axis and vertically upward as Y axis. Let t be the h at a distance d from the point of projection if
time when the velocity vector of the projectile
becomes perpendicular to its position vector. u2 > g [h + h 2 + d 2 ]
A particle rotates in a circle with angular speed
(a) Write a quadratic equation in t.
w0. A retarding force decelerates it such that
(b) What is the maximum angle a for which angular deceleration is always proportional to
the distance of projectile from the point of square root of angular velocity. Find the mean
projection always keeps on increasing? angular velocity of the particle averaged over the
[Hint: Start from the equation you obtained whole time of rotation.
in part (a)]

ANSWERS

The two velocities are perpendicular. (i) B and C


(a) 7.5 km/hr–1 (ii)
D
(b) 2 hr 40 min (iii) A, B, C, D
3. (a)
F 10 m
(b)
T v = 12 m/s
(c)
T
4. (a)
E, v (m/s)

(b)
D,G 30
(c)
B,C
5. (a) 4 m/s 10
(b) 2 m/s2 t
O 4 8
(a)
Xmax = 4 m ; t = 2 s
12. (a) particle A
(b) x (m)
(b) see solution for graph
(a) l < t < 2 s and 3 < t < 4 s
8 (b) 1 m/s
14. (a) 22 (Km) (s–1) (MLy–1)
1n(2)
(b)
t (s) (a)
4
v V (m / s)
20
v0

2 4
t t (s)
t O

–20
–v0


Kinematics 2.25

(b) V (m / s) 20 (1 + 2) m
20 24. (a) 12.13 m
(b) 16 m/s
O 2 4
t (s) a = 5.19 m/s2
4 2
u = 16 m/s; θ = tan −1  
 15 
(c) h (m) 4

5
20 m 4
pR
3
Displacement = 40 cm
O 2 4
t (s) Distance = (30 5 + 10 13) cm
(d) Distance (m) 40
s
41
40
31. (a) t0 = 5 s
(b) car 4
1 m/s2 to 7 m/s2
(a) Vmax = 12 km/hr
O 4
t (s) (b)
Vcmin
y

Up
m
ot ion
io ot 0° 
n m 90°
H/2 wn
Do
5 m/s, 12 m/s
V
V0 O V0
Dt = 23.33 s
2
L
V = 20 ms–1
u
2L  2 − 1 807 kph
∆t =  
g 38. (a) Parabolic path
1 m (b) 6 m/s
20. All statements are true 1
(a) Tg cosq Perpendicular to the incline
y 2
(b) 4 2 ms–1
x 2 2 cms–1
88
min
3
40 m at = 0; path is circular
The one that is projected at q2
wH
tan θ 2 n=

1
= q
2 tan θ1
2.26 Problems in Physics for JEE Advanced

3 2uv
(a) a = a0 (c)
p u v
(b) 8.37 m/s2 2L
45. (a) vx (d)
u v
56. (b) 0.5 m
v
57. (a) 10 5 s
t
x
(b)
B
–v A

a
(b) y

O t (s)
10 t1 t2
 t
(a) r = vt [cos (wt) î + sin (wt)] ĵ
(b) 

Vp = V [cos(w t ) - w t sin(w t )] 58. (a) Acceleration is increasing

î + V [sin (wt) + wt cos (wt)] ĵ (b) 1 m/s2

(a) v0e2pn
(
59. ∆r = − yiˆ + xjˆ dθ )
(b)
V = 2a0x 60. (a) 8.5 s
(b) 2.41 m
3 u , zero
61. t = (2 – 2)t0 ; x = ( 2 – 1)x0
3
rad 62. 2.2 hr ; 90.9 km/hr
8
2 m/s 63. 66 m
1.59 s −1  3 
64. θ = tan  
52. (a) t0 = 32 s ; 2
3
15 3 65. (a) 48 m, 68.5 m
< VA > = < VB > =
(b) m/s

53. Both are true x(m)
(b)
60 s
55. (a) Harshit
x
60.5
(b)
B D 52.5
HA
KS
HIT

AN
RS

AK
HA

C
t(m)
O 5.5 7.5

A
3v02
t 66.
O tH tH tA 4L
2
Kinematics 2.27

67. v
73.
s (m)

10 m/s

t(s)

s (m)
5

t(s)
O
1 2 3 4

68. (i) v10 = v20 = 4m/s 5

(ii) Both will take same time t(s)


O
69. (a) 90 m
(b) V Ak0v02
−1  3 
4
t (s) 75. (a) α = tan  
4
1
θ = tan −1
(b)
–30 2
(c) The ball will hit at a point lower than the earlier
(c) V (m/s) spot.
O 4 76. (i) n = 84°
t
u
(ii)
g
77. (a) 80 m
– 45
(b) 1.6 × 10–3
70. (a) 2.5 s
78. (a) q = 60°
(b) y (In meter)
(b) 3 u
79. 20 2 m
6.25
80. (a) 11.25 m
5.5
8
1.5 (b) tan −1  
t
5
O 2.0 2.5
81. (a) L = 14.58 m
2u (b)
OB = 41.66 m
71. (a) t0 =
α 82. (a) 5 m

(2 )
3/ 2 (b) 480 m
(b) 1/ 2 u = 7.29 m/s, t = 1 s.

h
84. (a)
(c) g

72. (a) 2 (b)


u = 21gR
(b) zero
2.28 Problems in Physics for JEE Advanced

85. (a) No 3
(b) rad / sec
(b)
Q 2
2
101. (a) R = 2 2u
g
u2
h
(b)
O V 2g
p
86. (40iˆ + 158.9 ˆj - 8.9kˆ ) 102.
3
m

87. (a) q = tan–1 (2) 2v


103. (a)
(b) 60 5 kmhr –1 3
3 (b) 4w2R
88. (a) hr
4 dy 2v12 t 2 ( v1 + v2 ) t
2

(b) 4.5 km 104. v = = +


dt L2 + v12 t 2 L2 + ( v1 + v2 ) t 2
2

89. (a) 45°


(b) 2 m/s 10
105. m/s
3
u sin 2a2

90. (a) 1
g cos q
106. (a) t0 = s
4 2
u cos(a + q )
(b)
cos q Ê 1 ˆ
91. (b) ÁË 1 -
(b) ˜m

92. (a) yes, 2 s 107. V/2
(b) zero 108. 10 sin 15°
(c) straight line 109. Bead 2
73 110. (a) 80 kmhr –1
93. V0
3 (b) 17 kml –1
x1 v + u 1 2
94. = 111. (a) aT
x2 v − u 4
95. (a) Corner C
(b) yes,
1 2
(b)
u aT
3
96. (a) particle 1
112. (a) 2
(b) Particle 2 and 5
(b) particle 1 will cover 2x0 in lesser time. Both will
(c) particle 3 and 4 ; 50.94 m cross 2x0 with same speed.
97. (a) u = 8 m/s, (c) v = (2 + 2) a0x0
(b) 18.75 m uv
113. (a)
200 L
98. (a) dmin
3 vL
(b) t0 = 2
(b) 25 m v − u2
99. (a) w = 4 rad/s (c) Zero
(b) w = 4 rad/s (d) The path will be like a spiral
100. (a) 1600 m 114. (b) Body travelling along a line making an angle
Kinematics 2.29

with vertical 3g 2

  X0
(b)
2u.g 16 k
115. (a) t = −  2
g 3 3g
T
(c)
2 2k
  
  g u . g
Vav = u −
(b)
( ) 132. (a) 2 v
 2 (b) path is as shown
g

16
116. (a)
25
(b)
2g
117. 40 5 m 133. (a) 40 m
118. (a) 2.5 s (b) 24.3° or 65.7°
(b) 4.05 m
134. 21.2 m2
119. (a) l2 > l1
g (t12 − t22 )
(b) 72 h sin a 135. (a) h =
8
3
120. (a) km u=
g
2t12 − t22
5 (b)
2
(b) 8 min
121. (a) 15 m u2
136. (a) H max
(b) 15 m 2g
(c) parabolic u
(b)
V0 l g
122. 2u
137. (a) u0 = 37.5 m/s
123. A circle of same size shifted from the original circle
(b)
t = 1.5 m/s
u2
by ∆ X = in the direction of wind. 138. (i) 3.38 m
2g
(ii) 2.5 m
124. 20 m/s
139. AB = 0.57 m
48
125. (a) km / hr 2 2 2
37 140. (a) 1 + 2 − 1 2
3
1
126. U min = − Mg  h + b − h 
2 2

(b) 2
1
2
2
128. q = 60°
uL
129. q = 45° 141. vmin =
d 2 + L2
130. Position: 40 cm up from starting position
()
VB = 45 cm/s ≠ 142. (a) t −
2 3v0 sin α 2v2
t + 20 = 0
g g
aB = 22.5 cm/s2 (≠) sin 1
8
(b)
9
g3 w0
131. (a) Vmax 18k
144.
3
2.30 Problems in Physics for JEE Advanced

SOLUTIONS

1. The initial and final velocities are perpendicular to each other. (see figure).
vi v –vi v

vf
2v
2v

vf v

‘A’ must travel 20 km on bicycle & remaining 20 km on foot.


20 km 20 km 4 16
Time of travel t = −1
+ −1
= +4 = hr .
15 km hr 5 km hr 3 3
40 km –1
(a) Average speed Vav 7.5 km/hr 
16
hr
3
(b) Time for with bicycle was unused
20 km 20 km
=
-
5 km hr -1 15 km hr -1
4 8
= 4- = = 2hr 40 min
3 3
3. (a) slope of X-t graph is not zero at x = 0. This means velocity is not zero.
(b) Magnitude of slope (= speed) is decreasing in the interval 0 to t1. This means particle is retarding. In the interval
t1 to t2 the magnitude of slope is increasing. This means it is accelerating.
(c) During 0 to t1 velocity is negative but acceleration is positive. From t1 to t2 velocity is positive and acceleration
is also positive. After this the velocity is positive but acceleration is negative.
4. (i) At indicated point E the slope is negative which means velocity is negative but the speed is decreasing. This
means acceleration is opposite to velocity, i.e., in positive direction.
(ii) At D and G velocity is positive (slope is positive) but the slope is decreasing. This means acceleration is
negative.
(iii) There is abrupt change in velocity at B and C which means the acceleration is large. This indicates that a large
force has acted on the particle.
(a) displacement
Vav =
time
u+v
Also Vav = for uniformly accelerated motion
u+v 4
∴ =
2 2
u+v
= 2 m/s
2
Hence u cannot be greater than 4 m/s
u < 4 m/s
Kinematics 2.31

(b) The speed of the particle decreases by a maximum of 4 m/s in 2 second.


∴ a max = 2 m/s2
6. (a)
v (m/s)

A
4
1
B
2

C t (s)
O 2 4

Separation will be maximum at t = 2 s when V1 = V2


Maximum separation = area (OABC) – area (OBC)
1 1
= ¥ 2 ¥ (2 + 4 ) - ¥ 2 ¥ 2 = 4 m
2 2

(b) Motion of both the particles is uniformly accelerated /retarded. Hence, X – t graph is parabolic
x

t
O 4

7. Hint: When the force (i.e., acceleration) is small the velocity time graph has a smaller slope. When force is maximum
the slope of the graph becomes maximum.
9. After t = 4 s, velocity of B becomes lesser than velocity of A and distance between the particles start decreasing.
Separation is maximum at t = 4 s
dmax = (XA at t = 4) – (XB at t = 4)
1 1
= × 4 × 10 − × 2 × 10 = 10
2 2
dv
V =a
dx
v x =10

∫ vdv =
0

x =0
adx

v2
= area under the a – x graph
2
1
= × 8 × ( 8 + 10 ) = 72 ∴ = 12 m/s
2
Change in velocity = area under a-t graph
2.32 Problems in Physics for JEE Advanced

For 0 < t < 4


1
v−u = × 4 × 10
2
v − 10 = 20
v = 30 m /s

In interval 4 to 8 sec, the velocity decreases by 20 m/s and becomes equal to initial velocity (10 m/s).
Also, slope of v-t graph gives acceleration. Hence slope of v-t graph is decreasing from t = 0 to t = 4 s. Afterwards,
the slope becomes negative with increasing magnitude.
v (m/s)

30

10

t
O 4 8

(a) Particle A has suffered a sudden change in its velocity at t = 1 s.


It shows that it experienced tremendous acceleration, i.e., a big force.
(b) In o < t < 1s both moved with a constant acceleration of a = –4 m/s2. For A the acceleration remains –4 m/s2 in
the interval 1 < t < 2 s
For B the acceleration in the later half was + 4 m/s2
x – t graph for constant acceleration motion is a parabola.
XB (m)
XA (m)

o 1 2
t (s)
o 1 2
t
-2

-2
-4

(a) The graph of v = t2 – 2t is a parabola with v = 0 at t = 0 and t = 2 s. The graph of v = – t2 + 6t – 8 is also a
parabola with v = 0 at t = 2 s and t = 4 s.

V2

1
o t (s)
2 3 4

-V1


Particle is said to be retarding when its speed is decreasing. It is retarding during intervals 1 < t < 2 s and
3<t<4s
Kinematics 2.33

V1 = 12 − 2 (1) = 1 m /s
(b)
V2 = – 32 + 6(3) – 8 = 1 m/s
Maximum speed = 1 m /s
(a) 1 Mega light year = 9.46 × 1021 m
H. (km) (s–1) (Mly–1) = 2.32 × 10–18 s–1
2.32 × 10−18
∴ = −1
= 21.95
103 × 9.46 × 1021 

(b) If r is instantaneous separation
V = Hr
dr
= Hr
dt
2r t
dr 0


r
r
= H ∫ dt
0

In ( 2 )
In ( 2 ) = Ht0 ⇒ t0 =
H
O
Downward journey
v2 y
v 2 = 2 gy or, y =
2g
H
graph of y vs v will be parabolic with v positive. v

Ball hits the ground with velocity

v0 = 2gH
v0
It rebounds with speed u - = gH
y
2
Upward journey
O

H–y
u

v2 = u2 – 2g (H – y)
v −u
y= +H
g

(with v negative)
graph is again a parabola
2.34 Problems in Physics for JEE Advanced
y

H/2

V
–V0 O V0
2

In 3 s the helicopter rises, by h = 5 × 3 = 15 m. It means the ball is at a height of 15 m from its initial point of
projection after 3 s.
1
y = ut + a t 2
2
1
15 = V × 3 − × 10 × 32 ⇒ V = 20 m /s
2

5 m/s

End ‘A’ pass through point P at time t1 given by

1 2L
L= g t12 fi t1 = B
2 g

End ‘B’ pass through point P at time t2 given by


L
1
2L g t2 2
2
A
L
t2 2
g

L
Required interval t = t2 – t1

2L  2 − 1
P
=  
g

Kinematics 2.35

1 2
h= gt
2
1
∴ h1 = g1t 2 . ......(1)
2
1 2
∆ h1 = t ∆ g1 ......(2)
2
(2) ∏ (1)
∆h1 ∆g1
=
h1 g1
 g 
∴ h1 = ∆ h1  1 
 ∆ g1 
 
 1  1   ∆ g1 
h1 = ∆ h1   = (10−3 )    × 100 = 0.1
∆g
 1  0. 001   g1 
 g 
 1 

= 1.0 m
(a) Hint: Eliminate t between the equations to get the trajectory equation
(b) Distance can be obtained if we know the speed.
dr
Velocity V = = ( 8t - 16 ) iˆ + ( 6t - 12 ) ˆj
dt

(8t − 16 ) + ( 6t − 12 ) = 5(4 − t )
2 2
Speed v =

[since speed is positive don’t write it as 5 (t – 4)]


4

Distance = ∫ v dt = 40 m
0

22. The two projectiles attain the same height. It means


uy1  = uy2-
uy ux1 uy1 tan q1 tan q 2
Also ux = tan q \ =
ux 2 uy 2 tan q
=
tan q1
2
y

uy u

q
X
ux
uxu y
Range =
g
R1 u x1 tan θ 2
∴ = =
R2 u x 2 tan θ1

2.36 Problems in Physics for JEE Advanced

y u H


x
A C B

4
u x = u cos 37° = 25 ×
= 20 m /s
5
3
u y = u sin 37° = 10 × = 15 m /s
5
Let the ball hit the roof at time ‘t’
1 2
H = uyt −
gt
2
1
10 = 15t − × 10t 2 ⇒ t 2 − 3t + 2 = 0
2
3 ± 9 − 8 3 ±1
⇒ t = = ⇒ t = 1s, 2s [2 s is unacceptable. Why?]
2 2
AC = (ux) (1s) = 20 m
After collision at P, vertical component of velocity is zero. Time of travel from P to B is given by
2H 2 × 10
t' = = = 2s
g 10

CB = uxt' = 20 2
AB = 20 + 20 2 = 20 (1+ 2) m

x

15 m/s
y

1.6 m

M S
R 0.8m 2.78m

Time of flight of the ball
1 2 1
1.6 = (15 sin 4∞) t +
2
gt or 1.6  ¥ 9.8 ¥ t 2
2
[Q 15 sin 4∞  0 ]

fi T = 0.57 s
Range R = (15 cos 4°) (T) ~ 15 × 0.57 [q cos 4° ~ 1]
= 8.55 m
Kinematics 2.37

Time for which Sania runs t0 = 0.57 – 0.2 = 0.37 s


Distance run by Sania = 7.5 × 0.37 = 2.78 m.
(a) Distance between the two players
= 8.55 + 0.8 + 2.78 = 12.13 m
(b)
Vx ~ 15 ; Vy ~ 9.8 × 0.57 = 5.6

\ V = Vx2 + Vy2 = 16 m /s

For the ball :


2u y 2u sin θ 2 × 19.5 12
Time of flight, T = = = ×
g g 10 13
v (m/s)
5
Range, R = T .u x = 3.6 × 19.5 × = 27 m
15 v0
V – t Graph is as shown
V0 = at = 2.a
Displacement = area under the graph = 27 m
1
∴ V0 [3.6 + 1.6] = 27
2 0 3.6 t(s)
1
× 2a [5.2] = 27
2
a = 5.19 m /s 2

y N

E
O F X
12 m

The ball was hit at 0 and caught by the fielder at point M.


Fielder runs for 2.4 s
FM = 2.4 × 5 = 12 m
2.38 Problems in Physics for JEE Advanced

z vertical

u
uv


O uH M

Range of the projectile (the ball) = OM = 12 2 m. Time of flight of the projectile T = 2.4 + 0.6 = 3.0 s
uv
T
g

[uv = vertical component of initial velocity]


2uv
∴ 3.0 =
10
uv = 15 m/s
R = uH. T [uH= Horizontal component of initial velocity]
R 12 2
∴ uH = = = 4 2 m /s
T 3
Speed of projection

(4 2 )
2
u = u H 2 + uv 2 = + 152 16 m /s

Angle made by initial velocity with vertical
uH 4 2
tan θ = =
uv 15

Let u = velocity of projection
Let a1 & a2 be two possible angle of projection to get a given range.
Time of flight for two cases are
2u sin α1
T1 =
g
2u sin α 2
T2 =
g

1 2
Given
2 1
2u sin α1 g
∴ × =2
g 2u sin α 2

Þ sin a1 = 2 sin a2 .........(1)


p
But a1 = - a 2 [for same range]
2
Kinematics 2.39

cos a2 = 2 sin a2
cot a2 = 2
u 2 sin 2α 2 2u 2 sin α 2 .cos α 2
Range R = =
g g
2u 2 1 2 4 u2
= . =
g 5 5 5 g
Rmax corresponds to a = 45°
u2
∴ Rmax =
g

4u 2 g 4
Ratio 5 g × u 2 = 5
=

u2
Maximum range of balls Rmax R
g
F

D R E

Circular region where


balls can land


From geometry < DCE = 120°
Boy is safe on the arc DFE, i.e, on two third of the circle.
2 4
Required length is .2π = π
3 3
Displacement in ground frame is simply the displacement of point A of the cardboard.
Distance travelled is speed multiplied by time.
50cm
Speed in reference frame of cardboard v = = 5cms −1
10s
Speed in ground frame while travelling from A to C
4
Vg = 22 + 52 + 2.2.5 cos θ where cos θ =
5
Vg = 45cms −1

Distance travelled when moving from A to C = 10 45 = 30 5 cm
While travelling back Vg = 22 + 52 + 2.2.5 cos(180 − θ ) = 13 cms −1

Distance travelled while returning = 10 13 cm


Total distance = 30 5 cm + 10 13 cm
2.40 Problems in Physics for JEE Advanced

5 m/s

X
A 4 m/s B

10 m

Let us fix the origin of our coordinate system at the original position of A.
X axis is along AB and Y axis as shown.
At time t position of B relative to A is given by
  
rBA = rB − rA

= 10i + 5tj − 4ti = (10 − 4t )i + 5tj

Velocity of B relative to A is
   
VBA = VB − VA = 5 j − 4i

The two particles are closest to each other when


  40
rBA .VBA = 0 fi -4(10 - 4t ) + 5t ¥ 5 = 0 fi t = s.
41
Acceleration of particle w rt frame 1
   
a p = a p − a ……………(1)
Acceleration of particle w rt frame 2
    
a p = a p − a …………..(2)
(1)-(2)
           
a p1 - a p 2 = a2 - a1 fi a p1 - a p 2 = a2 - a1
 
1 £ a p1 - a p 2 £ 7

  
1 £ a2 - a1 £ 7.

33. (a)

Vr

Vc 

Front

The velocity of rain with respect to the car must make an angle a with the horizontal such that a < q
Kinematics 2.41

Vr
tan a = £ tan q ................(1)
Vc

V rc
Vr

- Vc

fi Vr £ Vc tan q
Vmax = Vc tan q
= 20 × 0.75 = 12 km/hr
Vr
(b) From equation (1) Vc ≥
tanq
Vr
Vc min =
tan q
Vcmin


0° 90°

Let the speed of the car be u towards right. Velocity of train can be (u + 7) (") or (u – 7) (")
T

u+7
T

u–7

[Velocity of train can also be considered as (7 – u) (!). But (7 – u) (!) = (u – 7) (")]

+ ( − 7)
2
+ ( + 7 ) or
2 2 2
After the car turns, relative speed becomes

u2 + (u + 7) = 13
2
Solving

we get u = 5 m/s

u2 + (u - 7) = 13
2
solving

we get u = 12 m/s
2.42 Problems in Physics for JEE Advanced

35.
B V2 = 25 m/s A
V1= 30 m/s

Let X = distance between the car at the instant first bullet is fired (say at time t = 0)
Speed of bullet, relative to ground is
Vb = u + V2 = 305 + 25 = 330 m/s
Velocity of bullet, relative to car A is
VbA = 330 – 30 = 300 m/s
\ Time when the first bullet hits the car A is
X X
t1 = = .......................(1)
VbA 300
Distance between the car when the last bullet is fired (at time t = T0 = 20 s) is
X1 = X + VAB T0 = X + 5 × 20 = X + 100
Time when the last bullet hits the car A is
X X
t2 = T0 + =
VbA 300

X + 100 .......................(2)
= 20 +
300
The interval t = t2 – t1
100
= 20 + = 23.33 sec.
30
36. In the reference frame attached to the chain, the ball appears to be moving up with a constant velocity ‘u’.
\ Required interval of time is
L
Dt =
u
     
VBA = VB - VA fi VBA + VA = VB 75°
VB A
vB vA VB
From the fig. =
sin 60∞ sin 75∞
45° 60°
sin 60∞ 0.866
v A = 900 = 900 ¥ = 807 kph. VA
sin 75∞ 0.966
With respect to the cart, ball follows a parabolic path. y
2
In this frame it has a constant acceleration of 4 m/s
A
in negative X direction.
Due to similarity with projectile motion we can write
2
4 m/ s
u2 sin 2q
Range = OA =
a
u sin 90∞
2 x
fi9= fi u = 6 m /s O
4
Kinematics 2.43

(a) The boy will be able to catch the ball only if he sees the ball moving perpendicular to the incline (i.e., in y
direction)
The acceleration of the ball in the reference frame of the boy is g cos q in negative y direction.

q
os
gc

y
q

x
[The ball and the boy both have same acceleration (= g sin q) in the ground frame. Hence the boy does not see
any acceleration in the ball in this direction]
In boy’s frame if initial velocity of the ball is V then
2V 1
T= \ V = T .g.cos q
g cos q 2

(b) The velocity of tip of the barrel


wL = 2 2 m/s
Velocity of the shell wrt the body of the gun is vector sum of velocity relative to the exit point and the velocity
of the tip of the barrel. This is equal to

( ) + (2 2 )
2 2
2 2 = 4 m /s
In vertically upward direction.
Velocity relative to ground will be vector sum of this velocity and the recoil velocity of the gun. Resultant
velocity is -

42 + 42 = 4 2 m /s
In ground frame the ant must move perpendicular to the edge of the paper. Y component of its velocity
10 cm
Vy = = 2 cms -1
5s
y

Vy

A
x
Vx

X component of ant’s velocity in ground frame must be zero.


\ Vx = 2 cms–1
\ Speed of ant relative to the paper is
V = Vx 2 + Vy 2 = 22 + 22 = 2 2 cms-1

2.44 Problems in Physics for JEE Advanced

wB

wA

A B


Let the two particles be on a straight line passing through the centre after time t. In this time, A will be one complete
rotation ahead of B.
Ê 2p 2p ˆ Ê1 1 ˆ
w A t = 2p + w B t fi Á - ˜ t = 2p fi Á - ˜ t = 1
Ë TA TB ¯ Ë 8 11¯

wB

wA

2400

O A B

A1

B1
88
t= min.
3
t 88
In this time, A has completed = 3
TA 8
11 2
= rotations = 3 + rotations.
3 3

B has completed + rotations.

The two particles meet on the line O A´B´


= +

= = -

= = + = ( )
Tangential acceleration = =
Path
x = sin t
y = cos t
x2 + y2 = 1
Path of the particle is a circle.
Time of travel for the bullet from one disc to the other
Kinematics 2.45

H
t= \q = w t
V

w
q=w fi =
q
(a)

u0

u0

/3

=

Average acceleration


D = + - ( )
The two velocity to be added have same magnitude and angle between them is 120°

Vi
u0

600
u0
u0

-Vi V Vf

V = u0
p p
Time taken D = =

\ = = =
p p p

p
(b) The particle must rotate through an angle of for change in its velocity to be 4 m/s.
Distance travelled in 0.5 s is 2 m.
2.46 Problems in Physics for JEE Advanced

p
R = 2.
3
6
R=
p
v2
a= = 8.37 m /s2
R y
45. At any time t, particle is at P and wr = v v
vx = – v sin wt 
90°
vy = v cos wt aC P
 t
(b) ay = – aC sin wt x

v2

=– sin wt
r
y

P
r
q
A X
O


At time ‘t’ particle is at point p such that
r = vt and q = wt
\ = È w + w ˘
Î ˚

(b) velocity = = È w + w ˘+ È- w w +w w ˘
Î ˚ Î ˚

= [ w -w w ] + [ w +w w ]
(c) Vx = 0
cos (wt) = wt sin (wt)

fi q=
q

(a) Given at = n ar

fi =

Or, = [s = arc length traversed by the particle = Rq]

È ˘
Or, = Í = ˙
Î ˚

Or, =

Integrating
Kinematics 2.47

Ú = Ú [One complete rotation means s = 2pR]

\ = p = p

\ v = v0e2pn
at = a0 cos q
(b)

È ˘
= Í q= ˙
Î ˚

Ú=
= Ú
=

=

Tangential acceleration at = 0.8 ms–2
Let Radial acceleration = ar
-
\ + = fi + = fi =

& w 2r  = 0.6 ..................(1)
at 0.8
Angular acceleration a = =
r r
Let angular displacement be q by the time angular speed increases from zero to w
w2 = 02 + 2 a q

w = q fiw = q fiq = = rad.

If the block moves by X when point A moves by XA then,

3X = XA [Because the length of the string is constant]

\ =

3Vblock = 6 Vblock = 2 m/s
Since length of the string from B to P is fixed, point P will also move with a velocity of 2 m/s (!)
2.48 Problems in Physics for JEE Advanced

Length of the string = 12 + (12 sin 30° – 2). Final position has been shown in fig. below
B

16 m
4m

A
30° ut

(ut)2 = 162 – 42
252
\ t= = 1.59 s
10
(a) Speed of A and B
UA = 4.5 km/hr = 1.25 m/s
UB = 18 km/hr = 5 m/s
B will complete the circle in 8 sec. At that time A will travel through quarter of the circle.
In next t second let A travel through distance ‘s’. B will meet him if he travels a distance (10 + s) in interval
t

10 8
\ 5 t = 10 + 1.25 t fi Dt = = second 10m
3.75 3 B
8 10 B
S = 1.25 ¥ = m S
3 3
8 32
\ Both meet after, t0 = 8 + = sec
3 3 A
Meeting point is at a distance
10 40
10 + = m from the starting point
3 3
Displacement of both is same = OM
O " starting point, M " Meeting point

R 60°

120°
O


40
OM = R sin 60° × 2 [R = radius of circle = 2p ]
40 3
= . ¥2
2p 2
20 3
= m
p
Kinematics 2.49

2 3
Average velocity for A; < VA > =
Ê 32 ˆ
p ¥Á ˜
Ë 3¯
15 3
= m /s (along OM)
8p
Average velocity for B; <VB > = <VA>
the equation of the straight line shown in the graph is
2
x = -1
v
dx
But v =
dt
2dt
\ x= -1 fi x dx = 2 dt - dx
dx
15 t 15
1
fi Ú x dx = 2 Ú dt - Ú dx fi
2
[225 - 9] = 2t - [15 - 3]
3 0 3

fi 108 + 12 = 2t fi t = 60 s

1 2
56. (a) x B = 40 +
at = 40 + 0.5t 2 (parabola)
2
xA = 9t (straight line)
m B
x

90

72

40
A

t (s)
O 8 10

The two cars are at same position if xB = xA


0.5 t2 + 40 = 9 t
Solving, t = 8 s, 10 s
Conclusion:
First the car A is moving at greater speed. At t = 8 sec (when VA = 9 m/s > VB = 8 m/s)
Car A overtakes car B. But soon B will overtake A as its speed keeps on increasing. This happens at t = 10 sec
(VB = 10 m/s, VA = 9 m/s). After this the two cars never meet as speed of B keeps on increasing.
(b) At t = 8 ; xA = xB = 72 m.
Car A keeps on taking lead till VA > VB
2.50 Problems in Physics for JEE Advanced

After t = 9 sec, speed of B exceeds A and the two starts getting closer.
Lead is maximum at t = 9 s
DL = xA – xB (at t = 9)
= 9 × 9 – (0.5 × 92 + 40)
= 0.5 m
1 2
xB = aB t = t 2
2
xA = VA (t – 10) = 50 (t – 10)
A and B are at same location if
xB = xA
t2 = 50t – 500 \ t2 – 50t + 500 = 0
50 ± 2500 - 2000
t=
2
= 25 + 5 5
t1 = 25 – 5 5 [At this time A crossed ‘B’]
t2 = 25 + 5 5 [B is moving with increasing speed. This is the time when B overtakes A]
\ Interval for which A is ahead
t = t2 – t1 = 10 5 s
(b)
x – t graph for B is a parabola.
x – t graph for A is a straight line.
x

t (s)
t1 t2

t

58.
dv
(a) a = v
dx
dv
Æ constant
dx
v Æ increasing
\ a Æ increasing
v0
(b) Slope of line APB = -
1
[v0 = velocity at P]
Let slope of tangent at P = m
Kinematics 2.51

Ê v ˆ
m ¥ Á - 0 ˜ = -1
Ë 1¯
1
m=+
v0

Ê dv ˆ 1
ÁË ˜¯ =
dx P v0
Ê dv ˆ 1
\ a = vP Á ˜ = v0 . = 1 m /s
P Ë dx ¯ P v0

( )

. The change in position vector Dr has length rdq where r = x 2 + y2
y

rd q

q
d r

q
O x
 
Direction of Dr is perpendicular to
y

rd q
Dr

x

\ Dr = – rdq sin q î + rdq cos q ĵ
But r sin q = y and r cos q = x

\ Dr = - ydq iˆ + xdq ˆj

(a) t = time needed for rope to cross point O.


a = acceleration
L = length
1 2
L = 2 at ..........…(i)

1
( L - 1) = a (t - 2 )
2

2 ..........…(ii)

( L - 2 ) = 1 a (t - 5)2 ..........…(iii)
2
1 È2
a t - (t - 2 ) ˘ ..........…(iv)
2
(i)-(ii) 1 =
2 Î ˚
2.52 Problems in Physics for JEE Advanced

1 È
a (t - 2 ) - (t - 5) ˘ ..........…(v)
2 2
(ii)-(iii) 1 =
2 Î ˚

\ t2 – (t – 2)2 = (t – 2)2 – (t – 5)2


& 4t – 4 = 6t – 21 & 2t = 17
t = 8.5 s
1
From (iv) a ÈÎ8.52 - 6.52 ˘˚ = 1
2
1
a (15 ¥ 2 ) = 1
2
1
a= m /s 2
15
1 1 1
\ L = ¥ at 2 = ¥ ¥ 8.52
2 2 15
= 2.41 m
61. x co-ordinate of particle 1 at time ‘t’ is
Êx ˆ
x1 = x0 - Á 0 ˜ t = x0 - ut …..........(i)
Ë t0 ¯
[This is the equation of the straight line given in the question]
For particle 2-
u
Acceleration = - [Slope of v – t graph]
t0
x co-ordinate at time ‘t’ is-
1Ê uˆ 2
x2 = ut - t
2 ÁË t0 ˜¯ ….......... (ii)

Collision occurs when x1 = x2


1Ê uˆ Ê u ˆ
\ x0 - ut = ut - Á ˜ t 2 fi Á ˜ t 2 - 2ut + x0 = 0
2 Ë t0 ¯ Ë 2t0 ¯
2u
2u ± 4u 2 - x
t0 0 2u ± 4u2 - 2u2
\t = =
u u
t0 t0
= t0 ÈÎ2 ± 2 ˘˚

Since t cannot be larger than t0
t = (2 – 2)t0
Put this in (i) x1 = x0 – (2 – 2) ut0
= ( 2 – 1)x0
62. v – t graph for two trains has been shown.
Distance travelled by passenger train in
1 hr 48 min = 50 × 1.8 = 90 km
\ Area of PQR = 10 km
Kinematics 2.53

v (Km/hr)
1
¥ 50 ¥ t0 = 10
2
2
t0 = = 0.4 hr M
5 Express train
from B to A
\ Total travel time = 2.2 hr = travel time for express
train from B to A
Area of OMR = 100 50
P

1 Passenger
¥ 2.2 ¥ v0 = 100 train from
2 A to B
100
v0 = = 90.9 km /hr Q R
t(hr)
1.1 O 1.8 2.2
t0
Area under ax vs t graph gives change in vx
È v0 ˘
vx = a0t0 + a0t0 = 2a0t0 = 2v0 ÍQ a0 = t ˙
Î ˚

Area under ay vs t graph = change in vy


1
\ vy - v0 = ¥ t0 ¥ a0 ¥ 4 = 2 a0 t0 = 2 v0
2
3v
\ vy = 3v0 \ tan q = 0
2 v0
Ê 3ˆ
q = tan -1 Á ˜
Ë 2¯

Let velocity after displacement ‘x’ be v.
(L–x)

V0
V

dv
v
dx
= -k (L - x ) [k = a constant ]
v0 / 2 L

Ú v dv = - Ú k ( L - x ) dx
v0 0

1 Èv ˘
2
È 2 kL2 ˘ 3 2
Í - v 2

0
= - ÍkL - ˙ fi v0 = kL
2

2Î 4 ˚ Î 2 ˚ 4
3v02
Initial retardation = kL =
6L

= -

Ê - ˆ
= ÁË - ˜¯

2.54 Problems in Physics for JEE Advanced

È ˘
= ( - - t
) Ít =
Î
˙
˚

(i) =

¥
= =

¥
= =

(ii) The body with smaller value of t will take lesser time.

For body 1 t = = =

For body 2 t = = =

Both will take same time.


69. (a) At the instant the nut gets detached from the rocket, its speed and height are
5
v = 0 + ¥ 8 = 10 m /s (≠)
4 Path of nut
1 5 10 m/s
h0 = 0 + ¥ ¥ 82 = 40 m
2 4
10 m/s
For motion of the nut
u = 10 m/s (-­is +ve)
h = – 40 m (. is –ve)
a = 10 m/s2

\ = +
h0 = 40m

- = ¥ - ¥ ¥

Solving, t = 4 s
Nut will hit the ground in next 4 s.
Further height gained by the rocket in 4 s is
1 5
h1 = 10 ¥ 4 + 2 ¥ 4 ¥ 4
2

= 40 + 10 = 50 m
Required height of the rocket = h0 + h1 = 90 m V (m/s)

(b) Velocity of nut in the reference frame of ground is v = 10 – 10 t 10


4
In frame of rocket O t(s)
1
Initial velocity u = 0
5 45
Acceleration a = 10 + = m /s2 (Ø)
4 4
v = 0 + at –30
Kinematics 2.55

45
v = - t [– ve because acceleration is downward]
4

O t

– 45 (m/s)

70. The elevator moves up with constant acceleration, hence y – t graph must be a parabola.
Let y = kt2
at t = 2, y = 4
k = 1 y = t2
dy
\ = 2 t = 4 m /s (at t = 2)
dt
d2 y
= 2.0 m /s2
dt 2
In the reference frame of the elevator the acceleration of bolt is 12 m/s2 and its initial velocity is zero. Time required
for a displacement of 1.5 m in this frame is
1
y= ¥ 12 ¥ t 2
2
1
1.5 = ¥ 12 ¥ t 2 fi t = 0.5 s
2
Bolt hits the floor at t = 2.5 s

In 0.5 second, the displacement of bolt (in reference frame of ground) is


1
Dyb = 4 ¥ 0.5 - ¥ 10 ¥ 0.52 = 0.75 m
2
The event as seen in reference frame of elevator and ground has been shown in figure.
R.F. of elevator

u=0

a = 12

1.5 m


2.56 Problems in Physics for JEE Advanced

R.F. of ground

F1 F1
u=4
0.75 m

F F

Graph in R.F. of ground


y Co-ordinate
y (In meter)
y (In meter) of ceiling

y Co-ordinate
6.25 of bolt

5.5

1.5
t
O 2.0 2.5
y Co-ordinate
of floor

71. For upward motion


v = 0 t0

v x0
t

g
u

t=0

dv
= -a t
dt
0 t0

Ú dv = -a
u
Ú t dt
t =0

t02
u=a
2

2u
\ t0 = = time for upward motion
a
Kinematics 2.57

Also, velocity at time ‘t’


v t

Ú dv = -a Ú t dv
u 0

t2
v = u -a
2
dx t2
\ = u -a
dt 2
x0 t0 t
a 2 ...............…(i)

Ú dx = u Ú dt -
0 0
2 Ú0
t dt

a 3 2u a 2u 2u 2u 2u
x0 = ut0 - t0 = u - =
6 a 6 a a 3 a
Total time of flight
0 t t
a 2
From (i) Ú dx = uÚ dt -
0 0
2 Ú0
t dt

a 3
fi ut - t =0
6
6u
fit =
a t=0
2
73. At time t = 0, speed of first stone is
u1 = 0 + g × 1 = 10 m/s s0
and separation between the stones is
1 1
S0 = 0 + ¥ g ¥ 12 = 5 m
2
At time t u1

Speed of first stone is


v1 = 10 + gt = 10 + 10 t
Speed of second stone is
v2 = gt = 10 t
v = v1 – v2 = 10 = constant.
Displacement of first stone from top of the tower is
1
S1 = 5 + 10t +
¥ 10 ¥ t 2
2
= 5 + 10t + 5t
2

v s (m)

10 m/s

t(s) t(s)
O
2.58 Problems in Physics for JEE Advanced

Displacement of second stone from top of the tower is


1 2
S2 =
gt = 5t 2
2
DS = S1 - S2 = 10t + 5
74. x = v0t

r = x iˆ + y ˆj


 dr dx dy ˆ
v= = iˆ + j
dt dt dt
dx ˆ
= i + Akv0 cos ( v0 tk ) ˆj = v0 iˆ + Akv0 cos ( v0 tk ) ˆj
dt

dv
= − Ak 2 v02 sin ( v0 tk ) j = − Ak 2 v02 sin ( kx ) j
dt
π
at x =
2k ’

a = − Ak 2 v02 ˆj

The ball hits the incline at the same horizontal level from which it is projected. (first graph).
Time of flight
2uy
= 2.4 fi uy = 12 m /s [= v0 shown in 2nd graph]
g
vy = 12 – 10 t and x = uxt
10 x
Eliminating 't', vy = 12 -
ux
Slope of vy vs x graph
-10 2v
=- 0
ux 38.4
ux = 16 m/s
uy 12 3
(a) tana = = =
ux 16 4
h 19.2 1
tan q =
(b) = =
x 38.4 2
q<a
(c)
And q and a both are less than 45°. So by decreasing the angle of projection the horizontal range of an usual
projectile will decrease. Hence the ball will hit the incline at a lower point.
R
u2 sin 2q
76. (i) Range R =
g
B
The graph of R vs. q is as shown.
In the context of the question, the graph ranges from A to A
B to C.
C
Slope of the graph is maximum at C.
[graph is symmetrical about q = 45°, slope at q = 15° is
O
same as slope at 75°]. 0° 15° 45° 85° 90°
Kinematics 2.59

DR
It means is maximum at C. Hence answer is q = 84°
Dq
Range will change by maximum amount when q is increased from 84° to 85°.
(ii) If we consider small cones of semi vertical angle Dq with apex at the sprinkler, each of the cone will receive
DR
same amount of water. This water spreads on smallest possible area on ground at position where is
minimum. This happens at q = 45°. Dq
u2
The required distance is R45∞ =
g
77. Minimum Angle of projection with horizontal is q = 90° – 1° = 89°
For this angle range will be maximum. (look at the graph in solution to previous problem)

u2 sin 2q (150)2 ¥ sin 178∞


Rmax = radius of circle (r) = = [Q sin (180 – q) = sin q]
g g
p
But ∞ = ¥ p radian. Hence, ∞@

¥ ¥p
\ = @ 80 m
¥

Probability that a bullet fired will fall on person’s head = = = 1.6 × 10–6
Probability that any of the fired bullets falls on head is
n = 1.6 × 10–6 × 1000 = 1.6 × 10–3
78. (a) For an observer falling down with an acceleration g, all the stones appear to move with constant velocities. The
velocity of 2 relative to 1 has magnitude equal to u. It means q = 60° (see fig)
u21 u2
u2
u
q = 60° u
1200
u1 u
-u1

(b) Velocity of 3 relative to 2 will have magnitude

= + + ∞=

u2

120°

60°

-u2 u3

u32
2.60 Problems in Physics for JEE Advanced

79. If the stone is thrown at an angle of q = 45°, its range is = = = . In this case the maximum height

( q)
=
attained by the stone will be . But a stone thrown at q > 45° can have a path as shown in path 2 in

the figure and thereby it can hit the bird sitting at a larger distance. Therefore, we need to calculate the maximum

possible x co-ordinate of the projectile for a given height h = 10 m. Trajectory equation is

y Path 2

Path 1

Ê ˆ
= q- ÁË ˜

Put y = 10 m, g = 10 m/s2 and u = 20 m/s

= q- q

= q- ( + q )

fi =( q) - - q


fi ( ) q -( ) q+ ( + )=
This is a quadratic equation in tanq. For tanq to be real we must have
(80x)2 – 4(x2) (800 + x2) > 0
fi - - ≥ fi ≥ fi ≥

\ =

80. The path of both the projectiles to the wall, if considered together, makes the complete path of either of the particles
in absence of the wall.
P

uy uy


A ux O ux B
Kinematics 2.61

In absence of wall, the time of flight will be


4 2
T= + =3s
2 2
2u y
fi =3 fi uy = 15 m /s
g

152uy2
H max =
(a) =
= 11.25 m
2 g 2 ¥ 10
(b)
x0 = Range of projectile in absence of wall
24
24 = ux T fi ux = =8m/s
3
P

vx

vy v

a
O

Particle projected from A, hits the wall normally, 2s after its projection. It means its velocity vector makes an
angle a to the vertical after 2 s.
vx ux 8
\ tan a = = =
vy uy - gt 15 - 10 ¥ 2
8
tan a =
5
81. (a) ux = 20 cos 53° = 12 m/s
uy = 20 sin 53° = 16 m/s

When the ball enters the pipe its velocity vector makes an angle of 37º with the horizontal.

A
vx
37°

h vy v

x
vy
tan 37∞ =
vx

3 vy
=
4 12
vy = 9 m/s
2.62 Problems in Physics for JEE Advanced

vy2 = uy2 - 2 gh

92 = 162 – 2 × 10 × h & h = 8.75 m

h 8.75 ¥ 5
Now = sin 37∞ fi L = = 14.58 m
L 3
(b) Let the ball be at A at time 't'
vy = uy = gt
& –9 = 16 – 10.t & t = 2.5 s
x = ux.t = 12 × 2.5 = 30 m
4
OB = 30 + L cos 37° = 30 + 14.58 ¥ = 41.66 m
5
ux
82. (a) vy = uy + 2 gy ...........…(i)
2 2

and vx = ux ...........…(ii) uy u

vy
tanq =
vx
As per question minimum value of q is 45°. vx
x
& ratio of minimum vy and maximum vx is tan 45° = 1 q
y
(vy)min = 2gy when uy = 0 v
vy
and(vx)max = u when stone is projected horizontally.
2 gy
\ =1
u
y=5m

(b)

A B

O P

The shell cannot land in the region OP.

Equation of trajectory for projectile = q- + q


For point B, x = 960 m and y = 0
¥ ¥
q¥ - q + =
¥ ¥

tan q = 3/4, 4/3


1
For tanq = 4/3, OP is smallest. Hence the trajectory equation is-
N
Ê ˆ
= - ÁË ˜¯
Kinematics 2.63

For point P y = –960 m

- = -

Solving x = 1440 m
OP = x – 960 = 480 m
84. (a) As the vertical component of velocity does not change during collision, the time of flight is not affected by a
collision.

Time to touch the ground =

The total time of flight will be twice this time since the particle will bounce back to same height from which it
was thrown.
(b) The ball must strike the sphere normally (i.e., along radius) so that it can rebound back along the same path.
R/2 1 u
sin q = R = 2
q = 30°
The velocity of the ball at the instant of hitting the sphere is 7R
inclined at q to the horizontal. 2
vy
\ tanq =
vx
q
Ê 7R ˆ
2g Á ˜
1 Ë 2 ¯ v
= R
R
3 u 2

\ u = 21gR

(a) Hint: The velocity of rain relative to the bus has same vertical component in both cases.
Velocity of truck

ˆ ˆ ˆ
V T = 0 i + 90 cos q j - 90 sin q k
Where q = tan–1 (0.1)

V T = (89.6 ĵ – 8.94 k̂ ) km/hr
Velocity of police car

Q V p = – 80 sin 30° î – 80 cos 30° ĵ + 0 k̂
= – 40 î – 69.3 ĵ
Relative velocity
  
V TP = V T - V P
(
= 40 iˆ + (158.9) ˆj - 89 kˆ km /hr )
VA = 60 kph VB = 60 kph

VPA

VPB
VP
q 45°

–VA 60 kph 60 kph


–VB
2.64 Problems in Physics for JEE Advanced

From the above relative velocity diagram


VPA = 120 kph
120
and tanq = =2
60
VP = 602 + 1202 = 60 5 kph
V = velocity of boat relative to water.
VA and VB = actual velocity of two boats.
From the condition given in the problem it follows that
VAy = VBy
& V cos q = V cos q ' C
& q = q'
Also, V sin q ' = u [Q VBx = 0] y
5 sin q ' = 3
sin q ' = 3/5 x
V VA V
Ê 3ˆ VB
q = q ’ = sin -1 Á ˜
Ë 5¯ q q'
4
\ VAy = VBy = V cos q = 5 ¥ = 4 km /hr A u u
5 L
3.0 km
time to cross the river t =
4.0 km /hr
3
= hr.
4
3
For A, VAx = V sin q + u = 5 ¥ + 3 = 6 km /hr
5
3
\ L = VAx t = 6 ¥ = 4.5 km
4
90. (a) Let X direction be along the incline and Y direction be perpendicular to it.
u is the relative velocity of particle with respect to the box in x– direction.
x
uy is the relative velocity with respect to the box in y – direction

in 
gs

 sin Q
sin g
in u
gs u y= s
u co
u x=
P



Considering motion in Y direction relative to box
uy = + u sin a ; ay = – g cos q

= + fi = a - q¥

a
fi =
q
Considering motion in X direction relative to box
Kinematics 2.65

ux = + u cos a; ax = 0
a a
= + fi = a¥ =
q q

(b) For the observer on ground, to see the horizontal displacement equal to zero, the horizontal velocity of the
particle (as observed from the ground) shall be zero.
a +q
=
q
92. (a) In reference frame of the stone, the path of the ball will be a straight line. This is because, relative acceleration
of the two objects will be g (.) – g (.) = 0
A

40 m/s

50 2

50 m

10 m / s

45°

50 m B

In reference frame attached to the stone, the ball travels a straight line path of length 50 2 m with a velocity of
50 m/s.
50 2
\ Time for collision t = = 2 sec
50
(b) Time of flight for stone
2 ¥ 10 ¥ sin 45∞
T= = 2 sec
10
\ The two objects collide just before hitting the ground.
(c) Straight line.
93. Take x axis along the incline and y direction perpendicular to it.

Vrm

V0

Velocity of rain relative to the man is perpendicular to the incline in this case (i.e., along the umbrella stick. This
keeps canopy perpendicular to the rainfall and provides maximum safety).
2.66 Problems in Physics for JEE Advanced

  
V rm = V r - Vm
( )
= Vx iˆ + Vy ˆj - V0 iˆ = (Vx - V0 ) iˆ + Vy ˆj

Since V rm has no x component
\ Vx = V0

When the man is walking up, V rm is directed vertically downward.

( ) (
V rm = Vx iˆ + Vy ˆj - -V0 iˆ )
= V0 iˆ + Vy ˆj + V0 iˆ
= 2V0 iˆ + Vy ˆj
From diagram

Vrm

V0

x y
Vrm

2V0
tanq =
Vy
3 2V0 8V
= fi Vy = 0
4 Vy 3

73
\ Vr = Vx2 + Vy2 = V0
3
94. In the reference frame of the car, there is no wind (the air is still). In this frame the sound wave travels at speed
v and hill A travels to left at a speed u.

x2 x1

B A

u u

Sound has constant speed in this reference frame while approaching the hill A and after getting reflected from it.
x1
Time for sound to travel from car to hill A; t1 =
v+u
Kinematics 2.67

x1
Time for sound to travel back from hill to car t1 =
v+u

\ Time after which echo is heard T1 = 2t1 = 2 x1 .


v+u
If sound reaches hill B in time t1' then x2 + ut1' = vt1'
x2
\ t1´ =
v-u
Same amount of time is needed for the sound to return back.
Time after which echo is heard
2 x2
T2 =
v-u
T1 = T2
x1 x
= 2
v+u v-u
x1 v + u
=
x2 v - u

95. In the reference frame of the wagon, the ball has an acceleration a in + y direction. After brakes are applied the ball
has same acceleration in –y direction.
y

C D

L
a

O a
L

x A
B
L

The Vy – t graph for the ball will be as shown. If the ball covers a displacement y in y direction in time 't' (when it is
at centre O) then it will again travel same distance y in next interval 't'. This is V y

evident from area under v – t graph. In x direction the ball keeps moving
uniformly. It means that it will take equal amount of time for the ball to cover
first and second half of displacements each equal to L.
Hence, without any calculation we can say that the ball will hit the corner C.
At that moment Vy = 0 and Vx = u
96. (a) Particle 1 will hit the ground first. Let it hit the ground after time t. O t 2t t

1
32 = (10 sin 37∞) t + ¥ g ¥ t2
2
32 = 6 t + 5 t 2 fi t = 2 s
(b) Relative acceleration of any two particles is zero.
Hence, particles with maximum initial relative speed will be at maximum separation. Particle 2 and 5 have
maximum relative speed
2.68 Problems in Physics for JEE Advanced

U25 = 25 m/s
Separation between them at t = 2 s will be
25 × 2 = 50 m
(c) Particle 1, followed by 2 and 5 will land on ground. Time at which 2 and 5 land on ground is
2 ¥ 32
t2 = = 6.4 = 2.53 s
10
Time at which separation between 3 and 4 is to be calculated is t' = 2.33 + 0.3 = 2.83
[you can check that both 3 & 4 have time of flight greater than 2.83 s]
Relative velocity of 3 and 4 is

V34 = 62 + 172 = 18.0 m /s


\ Required separation = 2.83 × 18 = 50.94 m

P V
q

q
g sin q
g cos q g sin q
g

q
x
(a) If we fix our co-ordinate axes along the incline (x) and perpendicular to it (y), the acceleration of both objects
in x direction is same equal to ax = g sin q.
\ The two bodies will collide if the x component of their initial velocities is equal.
& u = v cos q
4
\ u = 10 ¥ = 8 m /s
5
(b) The time of flight of the projectile can be calculated by consideration of its motion in y direction.
1
O = (v sin q ) t - ( g cosq ) t 2
2
2v sin q 2 ¥ 10 ¥ 3 3
fit = = = sec
g cos q 10 ¥ 4 2
1
\ Distaance PQ = ut + ( g sin q ) t 2
2
2
3 1 3 Ê 3ˆ
=8¥
+ ¥ 10 ¥ ¥ Á ˜
2 2 5 Ë 2¯
= 18.75 m
98. Let the man travel a distance 'x' on road in order to reach 'P' in minimum time
Kinematics 2.69

A V1 Q B
x (l - x)

V2

d 2 + (l - x )
2
x
t = v + v2 .....................(A)
1

Time to travel from A to P depends on x.


This time (t) is minimum when
dt
=0
dx
1 1 -x
fi + =0
v1 v2 d + ( - x )
2 2

v2 d
Solving, x=l±
v12 - v22

The positive sign in unacceptable as x cannot be larger than l
v2 d
\ x=l-
v12 - v22

v2 d
≥l
(a) If v12 - v22

v12 - v22 4 200


fid≥ = 50 ¥ = m
v2 3 3

In this case x is negative or zero.


But it makes no sense to have x < 0.
Hence, man will need to walk along straight line AP in order to cover the distance in minimum time.
v2 d
\ If l >
(b) then x is positive.
v12 - v22

v2 d
x=l-
v12 - v22
100

= 50 - 3
5 - 32
2

= 50 - 25 = 25 m
Putting this in equation (A)
2
Ê 100 ˆ
˜ + (50 - 25)
2
ÁË
tmin = 25 + 3 ¯ 170
= s
5 3 9
2.70 Problems in Physics for JEE Advanced

VB - VA w (g B - g A )
99. (a) w BA = = =w
gB -g A (g B - g A )

gA

A
VA

B
VB

gA VA
tanq =
(b) .
gB
q
wBA = (velocity of B wrt A perpendicular to AB) / AB gA A
VB cos q + VA sin q
w BA =
AB
gB gA gB
wg B . + wg A .
g A +gB
2 2
g A + g B2
2 q
= =w
g A2 + g B2 VB
B

The questions can be answered by simple observation as well. Imagine yourself as particle A. You can see that
particle B rotates around you and completes one revolution in the same time as you complete your rotation.
dr
= 0 , the velocity of the rocket is perpendicular to r.
100. (a) Since
dt
Ê dq ˆ p
v = w r = Á ˜ .r = 1.8 ¥ ¥ 4000
Ë dt ¯ 180
= 125.6 m /s
ar V

The radial acceleration of the rocket is
r g
v 2 (125.6 )
2

ar = = = 3.94 m /s2 q
r 4000
But the only acceleration that the rocket is having is g(­.)

r
h
q


\ g cosq = ar
9.8 cosq = 3.94
cosq = 0.4
Height h = r cosq = 4000 × 0.4 = 1600 m
Kinematics 2.71

A u

Vx = u
B


Vy V


(a) Tangential and radial accelerations will be equal at a point where q = 45° [i.e. vx = vy]
g
at = ar =
2
Speed of stone at this point
v= 2u
v2 g 2 2 u2
\ = fiR= ...........…(i)
R 2 g
(b) If height of tower is less than AB, then R will always be less then value given by (i)
v = 0 + 2 gh
2 2
y

u2
= h = height AB for which vy becomes equal to u
2g
u2
\ h<
2g y

102. In the given co-ordinate system A


L L

x = cos q ; y = sin q
2 2
2
Ê Lˆ x C
\ x +y =Á ˜
2 2
Ë 2¯
L 2
That is, the centre C will move on a circle of radius = = 1.0 m y
y 2 2

x
O B
C

q 
x
O


The centre C will rotate through an angle q = 60° on the circle of radius 1.0 m.
2p R
\ Distance travelled S =
6
[ R = 1.0 m ]

p
= m
3
2.72 Problems in Physics for JEE Advanced

103. (a) Let speed of point A be VA. Then VA


y
VA sin q = V [Q point is always on the rod]
V x
\ VA =
sinq q

At the instant given


O
d 1
cos q = =
R 2
3 2V
\ sin q = \ VA =
2 3
(b) See the figure.
It is clear from the geometry of the figure that
P

q f
O'
O R


f = 2q
df dq
\ =2
dt dt
dq
But = w = angular velocity of P figure about O'
dt
df
And = w 0 = angular velocity of P figure about O
dt
\ w0 = 2w
\ Angular velocity of P about O is constant.
\ Tangential acceleration at = 0
And, Radial (Centripetal) acceleration ar = w02R = 4w2R
\ Acceleration of P is 4w2R directed towards O.
The position at time t has been shown in figure. The shape of string has been shown by dotted lines.
Length of string between P and Q is
B'

P L A1
V2t A2
Q
B
V1t

A2'
A1'
M

Kinematics 2.73

PA1' + A1'B' + B'A2' + A2'Q

= ÈÍ L2 + (v1t ) + L2 + ( v1 + v2 ) t 2 ˘˙ ¥ 2
2 2

Î ˚
Increase in length of string between PQ must be
equal to distance(y) through which M moves up
È 2 2 2 ˘
\ y = 2 ÍÎ L + v1 t + L + (v1 + v2 ) t ˙˚ - 4 L
2 2 2

Hence velocity of M is
2 (v1 + v2 ) t
2
dy 2v12 t
v= = +
dt L2 + v12 t 2 L2 + (v1 + v2 ) t 2
2

This is the required answer.
105. Wall AB of the wedge is vertical. For ball to remain in contact with the wedge the velocity component of the
ball perpendicular to the wall AB must be equal to velocity component of the wedge in horizontal direction (i.e.,
perpendicular to wall AB)
B

60°
10 A
m/
s
60°
30°

\ 10 cos 60° = V cos 30°


3
5=V
2
10
\V = m /s
3
106. (a) Length of the stick is fixed. Hence velocity component of end A along the length of the stick = velocity
component of end B along the length of the stick.
VA sinq VA
\ VB cosq = VA sinq
Initially, q is small (which means cosq is large and sinq is small).
Therefore, VB is smaller than VA. A
L
As q increases, VB will increase.
h VB cosq
when VA = VB
cosq = sinq & tanq = 1 q
C B
& q = 45° VB

1
\ h = L sin 45∞ = [Q L = 1 m ]
2
h 1
\ t0 = = s
VA 4 2

2.74 Problems in Physics for JEE Advanced

1 Ê 1 ˆ
BC = L cos 45∞ =
(b) \ Distance travelled by end B = Á 1 -
2 Ë ˜

107. 2 (x0 – x) + x = L = length of the rope

x0

dx0 dx dx0 1 Ê dx ˆ 1
\ 2 - =0 \ = = V
dt dt dt 2 ÁË dt ˜¯ 2

108.
A

 A'
C C'

Because length of string does not change


\ AR = RA' = 5 m
<ARC' = q = 30°
5m R
A
15°
15°

5m

C'

Distance travelled by A
= AC´ = [5 sin 15°] × 2 = 10 sin 15°
110. Equation of the line given in graph is

=- + ...................(1)

When car moves at constant speed V for


time t, it will cover a distance

= =- + ...................(2)
Kinematics 2.75

Graph of X vs V is parabolic as shown X


–1
X is maximum for V = 80 km/hr  [Q parabola is
symmetric curve and vertex will lie midway between
V = 10 and V = 150]
For V = 80, value of X is

=- ¥ + ¥
200

= = V
10 80 150
This is the maximum distance that the car can travel on a
tank full of petrol.

Maximum mileage =

= 17 km/l –1
. (a) To cover maximum possible distance, the train shall accelerate for half the time and retard for the remaining
half with both acceleration and retardation equal to a. V – t graph for the case is as shown.
V

0

q
O t
T T
2
q=

fi =

Distance travelled = area of the triangular graph

= ¥ ¥ =

You can plot the graph for any other case and show that it is not possible to cover a distance greater then
and also come to rest at t = T. For example –

(i) If train accelerates for > , it cannot be stopped at t = T


V

t
O T T
2

(ii) If train is accelerated for < , then moved with constant velocity and then braked, the distance will be less
2.76 Problems in Physics for JEE Advanced

then
V

t
O T T
2

(b) Again Maximum distance is covered when train is allowed to travel with maximum acceleration for largest
amount of time
V

V0

a b
t
O 2T T
3

tan a = a ; tan b = 2a

\ = =

Distance = ¥ ¥ =

V
112. (a) ..................(1) G
= ( ) 1B
E
V1 A 2

..................(2) V2 D
=
O t1 c t2 F t
(b)
V – t graph for motion of both particles is shown.
Obviously, particle 1 will cover 2X0 distance in lesser time.
Area OABCO = 2X0
Area ODEFO = 2X0
-
Area under a – x graph gives = where V = find speed; u = 0 = initial speed. For both 1 and 2 this

area is same for their displacement 2X0


Hence, both will cross 2X0 with same speed.
Kinematics 2.77

(c) In above graph OAGD is a parallelogram. V is the common speed acquired at time t.
V
G
V
V2
A
V1 2 t1

V2
D

O t1 2 t1 t t

V = V1 + V2
È ˘
= + = ( + ) = ( + ) Í
ÍÎ
= ˙
˙˚


\ = ( + )
113. (a)
u t
R
u

Dq
L v

v v

V
V
C

In a small interval, the rat moves a distance u Dt. The direction of velocity of cat changes towards the rat, by an
D
angle Dq = [ DÆ ]
\ Change in velocity of cat has magnitude
uD
D = u Dq =
D u
=
D D
Acceleration is =
DÆ D
(b) Let the cat be at C' and the rat be at R' at any time t
u t
o X
R R'
X
u

y v

c'

c
2.78 Problems in Physics for JEE Advanced

Relative velocity of approach is v – u cosq


\ Cat catches the rat at the instant when–

Ú - q = fi - Ú q = ...................(1)

Also = Ú q

=Ú q ...................(2)

From (1) and (2)


(c) Zero
(d) The path will be like a spiral
114. (a) The acceleration is largest along vertical direction (= g) and the body falling along this path will cover maximum
distance in any time t
P

= = ( )

Naturally P is the top point and PQ the diameter of any such circle.
Now consider a body moving at an angle q to the vertical. Distance travelled in time t will be

= q [Q acceleration along PR = g cos q]

= d cos q
It can be easily proved that d cos q is the length of cord of the circle with diameter d and P as top point; the cord
making an angle q with PQ.
Hence the point following any path lies on such a circle.
(b) Now we consider two possible paths
PQ0 (vertically down)
and – (PQ1 perpendicular to incline)
Acceleration along PQ0 is maximum (= g) but path involved is also large. Acceleration along PQ1, is less but
path involved is shortest. We assume that path of least time lies somewhere in between PQ0 and PQ1.
Kinematics 2.79

Q1

Q0 P


The problem is solved easily by using the result proved in part (a). Bodies 2
starting from P at the same time and travelling in different directions,
always form a circle that grows with time and P as its top most point. O
After some time the circle with touch the inclined plane, with the plane 
tangential to the circle at the contact point Q’ (see fig). Thus, body
travelling along PQ’ reaches the incline plane before any other such body. Q'
a
It is easy to see that < =
a
\ Body travelling along line PQ' making an angle with vertical 
reaches the plane in least time.
115. (a) Displacement at time t is

= +

At the end of flight is perpendicular to

\ = fi ( ) + ( ) =

È ˘
fi = - ÍÎ
=
˙˚

This is time of flight.

(b) = = +

= +
Ê
Á-
ˆ
˜= -
( )
Á ˜
Ë ¯

116. In elastic collision, the horizontal velocity component will get inverted. There is no change in vertical motion of
the projectile (Q wall is smooth)
G E

h y

X
A
C O 4X0 F
3X0 X0
(a) In absence of wall, path of projectile would have been DEF. After collision, the path become DGC (mirror
2.80 Problems in Physics for JEE Advanced

image of DEF, with wall as mirror)


2u y
Time of flight T =
g

Time of flight from O to D is = =

[Q Horizontal displacement is of the range of a usual projectile]

Ê ˆ
\ = - = - ÁË ˜¯

È ˘ È ˘
= Í - ˙= Í = ˙
Î ˚ ÎÍ ˚˙

\ =

D
(b) =
D
The y component of average acceleration is certainly –g
Change in X component of velocity is
DVx = – ux – ux = – 2ux
\ X Component of average acceleration is
È ˘
( ) =- =- Í = ˙
Î ˚
È ˘
=- Í = ∞= ˙
ÍÎ ˙˚
\ = + =

118. At P : ux = 20 sin 37° = 12 m/s


uy = 20 cos 37° = 16 m/s

Q
Vx
uy u=20 0
53
V
37° Vy
P uX
y
h
53° x
37°
O R

At Q :
Vx = ux = 12 m/s

¥
= ∞ fi = =

Kinematics 2.81

Q
y

P’ H
y

h 53°
37°
o R
X1 15 X2

Time of flight = time interval in which the vertical velocity component charges from 16 m/s (-­) to 9 m/s (.)
Vy = uy – gt
& –9 = 16 – 10 t
& t = 2.5 s
Height of Q above P is

= -

= ¥ - ¥ ¥ =

Horizontal distance covered by the projectile from P to Q is


X = 12 × 2.5 = 30 m
X1 + X2 + 15 = 30
X1 + X2 = 15

+ =

+ ( + )=

Substituting the value of y and solving we get h = 4.05 m
119.
y

u
h

a a u

o
A
B
C  D
x

Speed of ball just before striking the incline, u = 2gh.


After first hit the velocity is u making an angle a with y axis.
ux = u sin a ; uy = u cos a
ax = g sin a ; ay = –g cos a
Time of flight from O to A is obtained by the fact that from O to A displacement in y direction is 0
2.82 Problems in Physics for JEE Advanced

= a - a

\ =

Just after each impact, the y component of velocity remains uy = u cos a. Hence, time of flight between any two
impacts is same = T.
Required distance
= OD – OA
È ˘ È ˘
=Í ( )+ ( ) ˙-Í + ˙
Î ˚ Î ˚

= +

Substituting for ux, T and ax
AD = 72 h sin a
120. Let be velocity of an army person and be velocity of the terrorist. With respect to the terrorist, velocity of
an army person will be


= ( )
+ -

Represents (- )
… represent in various possible directions.
.. are relative velocities for various possible directions of . When AB gets tangential to the circle
shown, we get maximum value of angle a. This is the case for which an army person will get closest to the terrorist.
For this case

a= =
- Ê ˆ
Terrorist sees one army person walking along a direction a = ÁË ˜¯ south of East (with respect to himself) who
gets nearest to him.

= a

= ¥ =

A 7.5 kph -VT O W E


S
B1
Va
VaT 6 kph
B2
B


Kinematics 2.83

A


3
km
5
1 km
C


dmin
T 4
 km
5
VaT

(b)
Time required

= =
-
=
121. In the reference frame of the wedge, initial velocity of the particle is

= +

Hence, the motion in this frame is that of a usual projectile

y
M
45°
20m/s Vx x

Vy

B 45°
A
10m/s E

This projectile hits the wedge normally, means its velocity at the instant of hit is inclined at 450 to the horizontal.

\ ∞=

10 = 10 t – 20
È = - ˘
\ 10 = 3 sec Í ˙
ÎÍ ˚˙
Height of projectile at this instant

= ¥ - ¥ ¥ =

= ∞ fi = =

And AE = 10 × 3 = 30 m
2.84 Problems in Physics for JEE Advanced

(a)
AB = 30 – 15 = 15 m
(b)

B will be 15 m to the left of A.
(c) Path is parabolic as shown above.
122.

For £
B A
15 m
= =

\ =

fi Ú = Ú
=

Ê ˆ
ÁË ˜¯ = fi =

Ê ˆ
For ≥ fi = ÁË - ˜¯
dy ul
\ =
dx 2V0 (l - y ) y
l
X
2V0 Ú (l - y ) dy = u l
x

l
Ú x0
dx
l
2
2
2
l V0 È V l˘ xo
= u l Íx - 0 ˙
4 Î 4u ˚ l
Vl 2
fi x= 0 X
2u           O

123.


The sprinkler wets a circle of radius R where R = maximum range of a projectile fired at speed u.

= = [ = ] ..........................(i)

When there is wind, the velocity of wind gets added to the velocity of the drops. This will not change the time of
flights because the vertical velocity component remains unaffected.
Kinematics 2.85

Let the wind be along X direction. Due to wind there will be additional shift (DX) of each drop along X direction.

For the drop whose vertical velocity is (i.e., those drops which fall on the circumference of the circle shown
above)
Ê ˆ
D =Á
Ë ˜¯ ( )= [Using (1)]

The region which will get wet is again a circle of radius R, shifted from the original circle by DX = R/2 (see fig)
Wind

R2

O O'


125. (a) The situation has been shown in fig.
50 m
A B

300 m
u0 = 8 km/hr

y
v
vy x

vx M

Let n = velocity of boat relative to water


= -n +n
Resultant velocity component in x direction is
Vx = u0 – vx
Resultant velocity component in y direction is
Vy = vy È = ˘
Î ˚
Time taken to travel 300 m in Y direction is

=
n

To enter the harbour, drift along X direction must be between O and 50 m


\ 0 < (u0 – vx)t < 50
n
\ -n £ =

( n ) .................(a)
2.86 Problems in Physics for JEE Advanced

And 0 < u0 – vx
& u0 > vx .................(b)
Now speed of motor boat is

n = n +n

But n + n ≥ n + ( -n ) .................(c)

[Q vy > 6(u0 – vx) from (a)]

Ê ˆ
fi n + n ≥ Án - ˜¯ +
Ë

\ n + n is minimum if n - =

fi n =

[This condition is consistent with (b)]

\ n +n( ) =

\n = (n +n ) =

= ¥ =

(b) For vmin we have

\ n = n -n

Ê ˆ
= -Á ˜¯
Ë

Ê ˆ
= ÁË - ˜¯ =

\ If q is angle made by n with y direction.


n
q=
n
= ( )

126. Given <APB = 90°
It means that A, P and B lie on a semicircle with AB as diameter.
Kinematics 2.87

b B

C
h
h/2
M
A
P4
P1
P3
P2
P0

= = +
The two strings will form a right angle if the block lies on any of the points P1, P2, P3, P4 etc. But the PE will be
minimum when the block is at the lowest point P0 (CP0 is vertical line through the centre)
È ˘
= - Í = ˙
Î ˚

=
+
- = ( + - )
\ =- = È + - ˘
Î ˚

127. Let the X and Y axes be as shown.


ax = – g sin a ; ay = – g cos a
ux = u cos (q – a) ; uy = u sin (q – a)
u
y
x

A
g sin  g cos 

O

If time of flight is T.

= +

= (q - a ) - a

q -a
\ =
a

Range along the incline = OA (= R say)

= +

q -a q -a q -a
= - a
a a
2.88 Problems in Physics for JEE Advanced

q -a È a q -a ˘
= Í (q - a ) - ˙
a Î a ˚

q -a
= È (q - a ) a- q -a a ˘˚
a Î

q -a q …………..(1)
=
a
R will be maximum when =
q


q
[ q -a q] =

fi q -a q- q q -a =
fi q -a =
p u
fi q -a =

a p
fiq = +

           
p a
In this case one can show that b = g = -
(b) From (1)
q -a q
= a
We want u2 to be minimum for a given R.
This is possible if sin (q – a). cos q is maximum, which is same as the condition in part (a).
Hence, least energy will be needed when the shell is fired exactly bisecting the angle between the vertical and the
incline.

128.

y Wind g
ax 
3

u g


x
R

The time of flight depends only on vertical component of velocity and acceleration.
q
\ = =

Horizontal range = +
Kinematics 2.89

q Ê qˆ È ˘
fi = q◊ + ◊Á ˜¯ = Í q q+ q˙
Ë Î ˚

For R to be maximum =
q

fi- q+ q+ q q=

p p
fi q=- qfi q=- fi q= fiq =

129. From graph

uy = 10 m/s

vy becomes zero when =

\ Range = 20 m \ =

\ ux = 10 m/s \ q= = \ q = 45°
130. For A
V2 = u2 + 2as
(30)2 = 0 + 2 × a × 20 & a = 22.5 cm/s2
And V = u + at

30 = 0 + 22.5 × t & =

In time , block C descends by ¥ =

Now length of the string

XA + 2XC + XB = constant ….......(i)

\ DXA + 2DXC + DXB = 0 \ DXB = –[DXA + 2DXC]


= – [20 + 2 × 10] = –40 cm
& XB decreases by 40 cm. Hence, B goes up by 40 cm.
From (i)
È ˘
= -Í + ˙ …........ (ii)
Î ˚
= – [30 + 2 × 7.5] = – 45.0 cm/s
–ve sign indicates XB decreases with time, i.e., B moves up.
Differentiating (ii) once more

=- +

2.90 Problems in Physics for JEE Advanced

= =- + =-

Negative sign once again indicates upward acceleration.

131. Initially, acceleration is positive. It becomes zero at time =


At this time velocity of the rocket is maximum.
For t > t1 the acceleration becomes negative (i.e., downwards) and the rocket retards. Finally, it stops at B (at t = t2).
Thereafter it falls back.

= - [for interval 0 <t < t0]             


x
    
B
t  t2

v0

A x0
t = t1

t=0
O

= -

Ú = Ú - Ú

= - ............…(i)

If v becomes zero at time t

= -

È ˘
fi Í - ˙= fi =
Î ˚

And = = =

Actually, much before t0 the acceleration of the rocket has turned negative [at time t1 it becomes zero and thereafter
it becomes negative]. Velocity decreases after t1 and becomes zero at t0.
(a) Maximum velocity during up journey is
È ˘
= = - Í = = ˙
ÎÍ ˚˙
Kinematics 2.91

= ◊ - ◊

= =

(b) From (i)

= -

= -

Ú = Ú - Ú
= -

È ˘
= Í - ˙
Î ˚
È ˘
= Í - ˙
Î ˚

=

(c) Time to fall down from height x0 (acceleration being g downward) will be given by

= fi =

Total time of flight = + = +

132. (a) The path of the ball as observed by the observer outside the room has been shown in the figure.
The time t0 can be calculated easily in the reference frame of the room.
v

=

Displacement of room in this interval

= ◊ =

2.92 Problems in Physics for JEE Advanced

Distance travelled by the ball in reference frame outside the room is

= ¥ + =

\ Average speed = = =

(b) The speed of ball remains constant in y direction but it increases continuously in x direction.
The path will be as shown.
Wall B

x
Wall A Wall A

Sol.132 (b)
133. (a) The figure shows the top view of the situation. A will throw the ball so that it passes tangentially (just missing)
to the building.
The ball can fall on line AX at different locations for different angles of projection.
Boy at B will have to move a minimum distance if the ball lands at C such that BC ^ AX.

10 m 40 m

A
B

D
d min

In similar, triangles
DAOD and DABC

= [R = 8 m]

\ dmin = 40 m

(b) Now AC2 = AB2 – BC2


= 502 – 402
\ AC = 30 m
Therefore, the horizontal range of the projected ball must be 30 m.
Note that the ball moves in a vertical plane passing through the line AX.
Kinematics 2.93

q ¥ q
= \ =

- Ê ˆ
fi q= \ q= ÁË ˜¯

2q = 48.6° or 131.4°
\q = 24.3° or 65.7°
134. A water drop leaves the rim at a horizontal velocity v = wr.



¥
Time of flight = = =

Horizontal range of drops


R = vT = wr × 0.6
For w = 5 rad s –1
R1 = 5 × 0.5 × 0.6 = 1.5 m
For w = 10 rad s –1
R2 = 10 × 0.5 × 0.6 = 3.0 m
Locus of drops when w = 5 rad s –1 will be circle of radius

= + = +
=
Similarly, locus of drops when w = 10 rad s–1 will be a circle of radius.

= + = +
=


rim

r x

v Top view


2.94 Problems in Physics for JEE Advanced

Hence, area of the floor that is wet is


=p ( - )
= 3.14 [9.25 – 2.5]
= 21.2 m2
135.

VB

h
VA


(a) = fi =

= fi =

\ = - fi - =

-
\ =
(b) Now = -
\ = +

= + -

= ( - )
= -
136. In reference frame of the ground
Initial velocity of the ant = u (-­)
Acceleration of the ant = g (.)

\
(a) =

(b) =
Kinematics 2.95

137. (a)
y
v2
u2

 u
x

v1

After time 't' the velocity of two balls will become
= - = -
= q + q-
Ê ˆ Ê ˆ
=Á ˜¯ + ÁË - ˜¯
Ë
For the two velocities to be perpendicular, ◊ =
Ê ˆ
\ ◊ - ÁË - ˜¯ =

& 3u2 – t (4u2 – 50t) = 0

& 50t2 – (4u2)t + 3u2 = 0 ........(i)


For this quadratic equation to have real solutions
D>0
(4u2)2 – 4 × 50 × 3u2 > 0
75
fi u2 ≥ m /s
2
75
\ u0 = = 37.5 m /s
2
Solution of equation (i) [when D = 0] is
4u2 4 ¥ 75
t = 2 ¥ 50 = 100 ¥ 2 = 1.5 s

138. (i) When a is directed horizontally along BC, the particle will hit the wedge at same height. In ground frame path
of the particle will be a straight line. In reference frame attached the wedge, the particle will have a horizontal
acceleration along CB equal to a = 4 m/s2 : apart from its vertical acceleration g (.).

O P 37

a
2.96 Problems in Physics for JEE Advanced

2u
Time of flight; T= = 1.3 s
g
1 1
OP = aT 2 = ¥ 4 ¥ 1.32 = 3.38 m
2 2
(ii) In this case the particle will hit the incline at a height above its point of projection.
Let the particle strike the surface at a height H above its point of projection.

37
O A

Two points (O & Q for example) separated by height H (and lying on the line of greatest slope) are separated
horizontally by
4
x = H cot 37∞ = 3 H

Q'
x
Q
H
O'
O

O and Q are two points on the wedge along the line of greatest slope. Particle is projected from O. The particle
lands back at point Q which has moved a distance X (to position Q') by that time.
1 2 3 3 2
\ X= at = 2t 2 \ H = 4 x = 2 t
2
For vertical motion of the particle from O to the top and then back to Q' we can write
1 2
H = ut - gt
2
3 2
fi t = 6.5t - 5t 2 fi 6.5t = 6.5 fi t = 1.0 s
2
\ x = 2t2 = 2.0 m
H = 1.5 t2 = 1.5 m
\ OQ = x 2 + H 2 = 4 + 1.52 = 2.5 m ( )
139. We will study the motion of the insect in the reference frame of the truck.
x
x
y u y

a
37 v0
A

g
37 37
Kinematics 2.97

Initial velocity of the insect in this reference frame will be vector sum of two velocities
u = 2.64 m/s (-) and v0 = 0.77 m/s (")
We will take x and y direction as shown.
Components of initial velocity
ux = 2.64 sin 37° + 0.77 cos 37°
3 4 7.92 + 3.08
= 2.64 ¥ + 0.77 ¥ =
5 5 5
11
= m / s = 2.2 m / s
5
uy = 2.64 cos 37° – 0.77 sin 37°
4 3 10.56 - 2.31
= 2.64 ¥ - 0.77 ¥ =
5 5 5
8.25
= m / s = 1.65 m / s
5
Components of acceleration
Ê 3 4ˆ
a x = - ( g sin 37∞ - a cos 37∞) = - Á 10 ¥ - 5 ¥ ˜ = -2 m / s2
Ë 5 5¯
Ê 4 3ˆ
a y = - ( g cos 37∞ - a sin 37∞) = - Á 10 ¥ + 5 ¥ ˜ = -11 m / s2
Ë 5 5¯

Consider motion in direction


1
y = uy t + a y t 2
2
To get time of flight put y = 0
1 1.65 ¥ 2
1.65t -¥ 11 ¥ t 2 = 0 fi t = = 0.3 s
2 11
Now consider motion in x direction
1
AB = ux t + a x t 2
2
1
= 2.2 ¥ 0.3 - ¥ 2 ¥ 0.32
2
= 0.66 - 0.09 = 0.57 m

140. Let the velocity of the block be v in a direction making an angle q with the direction of v1.
v1


v
60 

v2

2.98 Problems in Physics for JEE Advanced

For a string to be taut, the objects at the two ends must have same velocity component along the length of the string.
\ v cos q = v1 .............(1)
v cos (60 – q) = v2
È1 3 ˘
fi v Í cos q + sin q ˙ = v2
ÍÎ 2 2 ˙˚
v1 v 2 - v12
using (1) cos q = and sinq =
v v
È1 v 3 v - v1 ˘
2 2

\ vÍ 1 + . ˙ = v2
ÍÎ 2 v 2 v ˙˚

v1 + 3 v 2 - v12 = 2v2
( )
3 v 2 - v12 = 4v22 + v12 - 4v1v2
fi 3v = 4v + 4v12 - 4v1v2
2 2
2

2 .............(2)
\v = v12 + v22 - v1v2
3
(b) For q = 0°
v = v1 [from (1)]
Putting into (2) we get
2
v1 = v12 + v22 - v1v2
3
3v12
= v12 + v22 - v1v2
4
fi ( v1 - 2 v2 ) = 0
2
fi v12 - 4v1v2 + 4v22 = 0
v
fi v1 = 2 v2 fi 1 =2
v2

141. Let the insect move with speed v along the direction shown in the figure.
D C

u
B L
V



A
L
Time to cross is t =
v cosq
sin q
DC = v sin q .t = L = L tan q.
cos q
The block will just miss the insect if distance travelled by the block in time t is just less than d + DC
L
fi u.t = d + L tan q fi u.
v cos q
[= d + L tan q ]
uL
fi v=
d cos q + L sin q
The maximum value of d cos q + L sin q is d 2 - L2

Kinematics 2.99

uL
\ vmin =
d 2 + L2
g
142. x = v0t cos a, y = v0t sin a  –  2 t2,
vx = v0 cosa, vy = v0 sin a  –  gt
The stone is at the greatest distance from the origin when
its velocity is perpendicular to its position vector.
The condition for this is that the dot product of velocity vector and position
vector is zero.
y vx
=
x vy ,
This yields a quadratic equation for time t at which this happens;
3v0 sina 2v2
t2 - t + 20 = 0
g g
If this is not to happen, the discriminant of this equation must be negative i.e.,
2
Ê 3v0 sin a ˆ Ê 2v02 ˆ
ÁË ˜¯ < 4 ÁË g 2 ˜¯ .
g

Thus, for the stone to the permanently moving away from the thrower, we must have sin a < 8 / 9 = 0.94. i.e.,
a < 70.5°.

143. Method 1 : Trajectory equation


1 gx 2
y = x tan q -
2 u2 cos2 q
gx 2
y = x tan q - 2 1 + tan 2 q
2u
( )
Ê gx ˆ 2
2
Ê gx 2 ˆ
ÁË 2u2 ˜¯ tan q - x tan q + ÁË y + =0
2u2 ˜¯

Putting x = d & y = h
(gd ) tan
2

q -2
d tan q +
Ê
h +
gd 2 ˆ
=0
2u 2 ÁË 2u2 ˜¯
If projectile clears the pole then roots of above equation must be real i.e. D > 0
Ê gd 2 ˆ Ê gd 2 ˆ
d2 - 4 Á 2 ˜ Á h + 2 ˜ ≥ 0
Ë 2u ¯ Ë 2u ¯
Ê g ˆ Ê 2u h + gd ˆ
2 2
1- Á 2 ˜ Á
Ëu ¯Ë u 2 ˜¯ ≥ 0

u4 – (2gh) u2 – g2d2 > 0 ………….(i)
4 2 2 2
Let, u – (2gh) u – g d = 0

2 gh + 4 g 2 h 2 + 4 g 2 d 2
u0 2 =
2
u0 = g h + h + d 2 ˘
2 È 2
Î ˚
2.100 Problems in Physics for JEE Advanced

If u2 > u02 then expression (i) is greater than or equal to zero.

i.e. if u2 > g ÈÎh + h + d ˘˚


2 2

then projectile will clear the pole.



Method 2 :
h
sin a =
h + d2
2

Maximum range of projectile on inclined plane is


u2
Rmax = g 1 + sina
( )
If Rmax > h 2 + d 2 ; then projectile will clear the pole
u2
≥ h2 + d 2
g (1 + sina )

u 2 = g Èh + h 2 + d 2 ˘
Î ˚
Total angular displacement (q)
144. Mean (average) angular velocity =
Total time taken ( T )
dw
Given = - k w [where k is a constant and – ve sign corresponds to retardation]
dt
w t
dw
\ Ú = - Ú kdt
w0 w 0

kt
fi w0 - w = +
2
2
Ê kt ˆ
fi w = Á w0 - ˜
Ë 2¯
kt
\ w = 0 ; when w 0 -
2
=0 [Particle stops]
2 w0
\ T=
k
2
Ê kt ˆ
Now w = Á w 0 - ˜
Ë 2¯
2
dq Ê kt ˆ
\ = Á w0 - ˜
dt Ë 2¯
q T 2
Ê kt ˆ
Ú0 dq = Ú0 ÁË w 0 - 2 ˜¯ dt
[q = angular displacement when particle stops, i.e., t = T]
Kinematics 2.101

T
ÈÈ kt ˘ ˘
3
Í Í w0 - ˙ ˙
Î 2˚ ˙
\q = Í ˙
Í 3k
Í - ˙
ÍÎ 2 ˙˚ 0
È 2 w0 ˘
2 È
( ) -( ) ˘˙˚
3 3
=- w0 - w0 w0 ÍQT = ˙
3k ÍÎ ÍÎ k ˙˚
2 3/ 2
= w0
3k
q w0
\ <w > = =
T 3
03 NEWTON’S LAWS

Level 1 of F = 40 N is applied to the block A (see figure).


Find the contact force between block D and the
  front vertical wall of the cart.
Q. 1. Let u be the initial velocity of a particle and F be
A
the resultant force acting on it. Describe the path B
C
that the particle can take if F D
  
(a) u ¥ F = 0 and F = constant
  
(b) u.F = 0 and F = constant
In which case can the particle retrace its path.
Q. 2. A ball is projected vertically up from the floor of Q. 6. (i) Three blocks A, B and C are placed in an
a room. The ball experiences air resistance that ideal Atwood machine as shown in the
is proportional to speed of the ball. Just before figure. When the system is allowed to move
hitting the ceiling the speed of the ball is 10 m/s freely it was found that tension in the string
and its retardation is 2g. The ball rebounds from connecting A to C was more than thrice the
the ceiling without any loss of speed and falls on tension in the string connecting A and B. The
the floor 2s after making impact with the ceiling. masses of the three blocks A, B and C are m1,
How high is the ceiling? Take g = 10 m/s2. m2 and m3, respectively. State whether the
Q. 3. A small body of super dense material, whose following statements are true or false [All
mass is half the mass of the earth (but whose size masses have finite non zero values and the
is very small compared to the size of the earth), system has a non zero acceleration].
starts from rest at a height H above the earth’s
surface, and reaches the earth’s surface in time
t. Calculate time t assuming that H is very small
compared to the radius of the earth. Acceleration
due to gravity near the surface of the earth is g.
Q. 4. N identical carts are connected to each other using
strings of negligible mass. A pulling force F is m1 A C m3
applied on the first cart and the system moves
without friction along the horizontal ground. The
tension in the string connecting 4th and 5th cart is m2 B
twice the tension in the string connecting 8th and
(a) m3 can have any finite value
9th cart. Find the total number of carts (N) and
tension in the last string. (b) m1 > 2m2
N N–1 2 1 F (ii) In an Atwood machine the sum of two masses

is a constant. If the string can sustain a
Q. 5. A toy cart has mass of 4 kg and is kept on a smooth Ê 24 ˆ
tension equal to Á ˜ of the weight of the
horizontal surface. Four blocks A, B, C and D Ë 30 ¯
of masses 2 kg, 2 kg, 1 kg and 1 kg respectively sum of two masses, find the least acceleration
have been placed on the cart. A horizontal force of the masses. The string and pulley are light.
3.2 Problems in Physics for JEE Advanced

(iii) A load of w newton is to be raised vertically over the smooth pulley and two masses m and M
through a height h using a light rope. The (> m) are connected to its ends (see figure). Find
greatest tension that the rope can bear is the reading of the scale when the two masses are
hw  (h > 1). Calculate the least time of ascent left free to move.
if it is required that the load starts from rest
M0
and must come to rest when it reaches a
height h.
Q. 7. In the arrangement shown in the figure the system
is in equilibrium. Mass of the block A is M and that m M
of the insect clinging to block B is m. Pulley and Stand
string are light. The insect loses contact with the
block B and begins to fall. After how much time
the insect and the block B will have a separation
L between them.
Q. 10. In the given arrangement, all strings and pulleys
are light. When the system was released it was
observed that M and m0 do not move. Find the
masses M and m0 in terms of m1 and m2. Find the
acceleration of all the masses if string is cut just
above m2.

A B

Q. 8. Two blocks of equal mass, M each, are connected


M
to two ends of a massless string passing over a
massless pulley. On one side of the string there is
M m0
a bead of mass .
2

m1
m2

Q.11 The system shown in the fig. is in equilibrium.


Pulleys A and B have mass M each and the block
M C has mass 2M. The strings are light. There is an
2 insect (D) of mass M/2 sitting at the middle or the
right string. Insect does not move.
M
S1
M

(a) When the system is released from rest the A


bead continues to remain at rest while the two
E
blocks accelerate. Find the acceleration of the
blocks.
B
(b) Find the acceleration of the two blocks if it D
was observed that the bead was sliding down S2
with a constant velocity relative to the string.
Q. 9. A pulley is mounted on a stand which is placed
over a weighing scale. The combined mass of the
C
stand and the pulley is M0. A light string passes
Newton’s Laws 3.3

(a) Just by inspection, say if the tension in the Q. 17. A ball of mass M is in equilibrium between a
string S1 is equal to, more than or less than vertical wall and the inclined surface of a wedge.
9/2 Mg. The inclination of the wedge is q = 45° and its
(b) Find tension in the string S2, and S1. mass is very small compared to that of the ball. The
coefficient of friction between the wedge and the
(c) Find tension in S2 if the insect flies and sits at
floor is m and there is no friction elsewhere. Find
point E on the string.
minimum value of m for which this equilibrium is
Q. 12. A block slides down a frictionless plane inclined possible.
at an angle q. For what value of angle q the
horizontal component of acceleration of the block
is maximum? Find this maximum horizontal
acceleration. M
Q. 13. A tall elevator is going up with an acceleration
of a = 4 m/s2. A 4 kg snake is climbing up the
vertical wall of the elevator with an acceleration
of a. A 50 g insect is riding on the back of the
q = 45°
snake and it is moving up relative to the snake
at an acceleration of a. Find the friction force
Q.18 A helicopter of mass M = 15000 kg is lifting a
between the elevator wall and the snake. Assume
cubical box of mass m = 2000 kg. The helicopter
that the snake remains straight.
is going up with an acceleration of a = 1.2 m/s2.
Q. 14. Due to air drag the falling bodies usually acquire The four strings are tied at mid points of the sides
a constant speed when the drag force becomes of the square face PQRS of the box. The strings
equal to weight. Two bodies, of identical shape, are identical and form a knot at K. Another string
experience air drag force proportional to square KH connects the knot to the helicopter. Neglect
of their speed (Fdrag = kv2, k is a constant). mass of all strings and take g = 10 m/s2. Length of
The mass ratio of two bodies is 1 : 4. Both are each string AK, BK, CK and DK is equal to side
simultaneously released from a large height and length of the cube.
very quickly acquire their terminal speeds. If
(a) Find tension T in string AK.
the lighter body reaches the ground in 25 s, find
the approximate time taken by the other body to (b) Find tension T0 in string KH.
reach the ground. (c) Find the force (F) applied by the atmosphere
Q. 15. A cylinder of mass M and radius r is suspended at on the helicopter. Assume that the atmosphere
the corner of a room. Length of the thread is twice exerts a negligible force on the box.
the radius of the cylinder. Find the tension in the (d) If the four strings are tied at P,Q,R and S
thread and normal force applied by each wall on instead of A, B, C & D, how will the quantities
the cylinder assuming the walls to be smooth. T, T0 and F change? Will they increase or
decrease? Assume that length of the four
Thread identical strings remains same.

H
K

Q. 16. A rod of mass M and length L lies on an incline


P
having inclination of q = 37°. The coefficient of A D S
friction between the rod and the incline surface is C
Q R
B
m = 0.90. Find the tension at the mid point of the
rod.
3.4 Problems in Physics for JEE Advanced

Q. 19. A pendulum has a bob connected to a light wire.


Bob ‘A’ is in equilibrium in the position shown.
The string is horizontal and is connected to a M
block B resting on a rough surface. The block B is m
on verge of sliding when q = 60°.

q = 60° wire
Q. 23. A small body A starts sliding down from the top
of a wedge (see fig) whose base is equal to l .
B The coefficient of friction between the body and
A String
wedge surface is m = 1.0. At what value of angle q
will the time of sliding be least?
(a) Is equilibrium possible if q were 70°? A

(b) With q = 60°, calculate the ratio of tension


in the pendulum wire immediately after the
string is cut to the tension in the wire before
the string is cut.

Q. 20. Two blocks of equal mass have been placed on
two faces of a fixed wedge as shown in figure. The Q. 24. Three blocks A, B and C each of mass m are
blocks are released from position where centre of placed on a smooth horizontal table. There is no
one block is at a height h above the centre of the friction between the contact surfaces of the blocks
other block. Find the time after which the centre as well. Horizontal force F is applied on each of
of the two blocks will be at same horizontal level. A and B as shown. Find the ratio of normal force
There is no friction anywhere. applied by the table on the three blocks (i.e., RA :
mg
RB : RC). Take F =
2 3
h

F A C B F
RA RC RB
30° 30°
60° 30°
Q. 25. A U shaped container has uniform cross sectional
Q. 21. In the system shown in the figure, all surfaces are area S. It is suspended vertically with the help
smooth. Block A and B have mass m each and of a spring and two strings A and B as shown in
mass of block C is 2 m. All pulleys are massless the figure. The spring and strings are light. When
and fixed to block C. Strings are light and the force water (density = d ) is poured slowly into the
F applied at the free end of the string is horizontal. container it was observed that the level of water
Find the acceleration of all three blocks. remained unchanged with respect to the ground.
B Find the force constant of the spring.

A C Spring
F

A q q B
Q. 22. A particle of mass M rests on a rough inclined
Ê 4ˆ
plane at an angle q to the horizontal Á sinq = ˜ . It
Ë 5¯
is connected to another mass m as shown in fig.
The pulley and string are light. The largest value
of m for which equilibrium is possible is M. Find
the smallest value of m for which equilibrium is
possible.
Newton’s Laws 3.5

Q. 26. A uniform light spring has unstretched length of you convince your friend?
3.0 m. One of its end is fixed to a wall. A particle In a typical situation, car without ABS needs
of mass m = 20 g is glued to the spring at a point 20 m as minimum stopping distance. Under
1.0 m away from its fixed end. The free end of the identical conditions, what minimum distance a
spring is pulled away from the wall at a constant car with ABS would need to stop? Coefficient of
speed of 5 cm/s. kinetic friction between tyre and road is 25% less
Assume that the spring remains horizontal (i.e., than the coefficient of static friction.
neglect gravity). Force constant of spring = 0.6 Q. 29. Starting from rest a car takes at least ‘t’ second to
N / cm. travel through a distance s on a flat concrete road.
(a) With what speed does the particle of mass m Find the minimum time that will be needed for
move? it to climb through a distance ‘s’ on an inclined
(b) Find the force applied by the external agent concrete road. Assume that the car starts from rest
pulling the spring at time 2.0 s after he started and inclination of road is q = 5° with horizontal.
pulling. Coefficient of friction between tyres and the
3.0 m concrete road is m = 1.
Q. 30. A table cloth of length L is lying on a table with
1.0 m
one of its end at the edge of the table. A block is
kept at the centre of the table cloth. A man pulls
m the end of the table cloth horizontally so as to take
5 cm / s
it off the table. The cloth is pulled at a constant
Q. 27. It was observed that a small block of mass m
speed V0. What can you say about the coefficient
remains in equilibrium at the centre of a vertical
of friction between the block and the cloth if the
square frame, which was accelerated. The block
block remains on the table (i.e., it does not fall off
is held by two identical light strings as shown.
the edge) as the cloth is pulled out.
[Both strings are along the diagonal]
L L
(a) Which of 1, 2, 3 & 4 is/are possible direction/s — —
2 2
of acceleration of the frame for block to
remain in equilibrium inside it?
(b) Find the acceleration of the frame for your
answers to question (a).
Q. 31. A block rests on a horizontal surface. A horizontal
force F is applied to the block. The acceleration
3
(a) produced in the block as a function of applied
2 1 force (F) has been plotted in a graph (see figure).
Find the mass of the block.
4
a (ms–2)

Q.28 In an emergency situation while driving one 3


has tendency to jam the brakes, trying to stop
in shortest distance. With wheels locked, the car
slides and steering get useless. In ABS system the F (N)
O 6 18
electronic sensors keep varying the brake pressure
so as to keep the wheels rolling (without slipping) Q. 32. Repeat the last problem if the graph is as shown
while ensuring that the friction remains limiting. below.
Your friend has an old car with good brakes. He a (ms–2)
boasts saying that all the four wheels of his car 3
get firmly locked and stop rotation immediately
after the brakes are applied. You know that your
new car which has a computerized anti lock
F (N)
braking system (ABS) is much safer. How will O 6 18
3.6 Problems in Physics for JEE Advanced

Q. 33. A solid block of mass m = 1 kg is resting on a C


horizontal platform as shown in figure. The z C
direction is vertically up. Coefficient of friction f
between the block and the platform is m = 0.2. The f
platform is moved
 with a time dependent velocity B
( )
given by V = 2tiˆ + tjˆ + 3tkˆ m/s . Calculate the B
magnitude of the force exerted by the block on the f
platform. Take g = 10 m/s2 D D
z
y
f
A

x (b) A particle is moving along an expanding


spiral (shown in fig) such that the normal
Q. 34. In the system shown in the figure, the string is force on the particle [i.e., component of force
light and coefficient of friction between the 10 kg perpendicular to the path of the particle]
block and the incline surface is m = 0.5. Mass of remains constant in magnitude.  The possible
the hanger, H is 0.5 kg. A boy places a block of
mass m on the hanger and finds that the system
direction of acceleration a of the particle ()
has been shown at three points A, B and C on
does not move. What could be values of mass m? its path. At which of these points the direction
3 of acceleration has been represented correctly.
tan 37∞ = and g = 10 m/s2
4 A
VA

a
10 kg a

B a
H VC

37° m VB
C

Q. 35. A disc of mass m lies flat on a smooth horizontal


(c) A particle is moving in XY plane with a
table. A light string runs halfway around it as 
velocity. v = 4iˆ + 2tjˆ ms –1 . Calculate its rate
shown in figure. One end of the string is attached
of change of speed and normal acceleration at
to a particle of mass m and the other end is being
t = 2 s.
pulled with a force F. There is no friction between
the disc and the string. Find acceleration of the end Q. 37. (i) A spinning disk has a hole at its centre. The
of the string to which force is being applied. surface of the disk is horizontal and a small
block A of mass m = 1 kg is placed on it.
F
m w

Q. 36. (a) A car starts moving (at point A) on a horizontal r A


circular track and moves in anticlockwise
sense. The speed of the car is made to increase
uniformly. The car slips just after point D. The
figure shows the friction force (f) acting on
the car at points A, B, C and D. The length
of the arrow indicates the magnitude of the
friction and it is given that D > B > C. At
which point (A, B, C or D) the friction forces
represented is certainly wrong ? B
Newton’s Laws 3.7

Block A is tied to a light inextensible string, with respect to time. Take t = 0 when block 1
other end of which passes through the hole gets detached.
and supports another block B of mass M
= 2 kg. The coefficient of friction between
A and the disk surface is 0.5. It was observed
that the disk is spinning with block A
remaining at rest relative to the disk. Block B
was found to be stationary. It was estimated
that length of horizontal segment of the string
(r) was anywhere between 1.0 m to 1.5 m. With
6
this data what estimate can be made about the 4
2
angular speed (w) of the disk. [g  =  10  m/s2] 5
3
(ii) A spring has force constant equal to 1
k  =  100  Nm–1. Ends of the spring are joined to Q. 40. Two monkeys A and B are holding on the two
give it a circular shape of radius R = 20 cm. sides of a light string passing over a smooth pulley.
Now the spring is rotated about its symmetry Mass of the two monkeys are mA= 8 kg and mB =
axis (perpendicular to its plane) such that the 10 kg respectively [g = 10 m/s2]
circumference of the circle increases by 1%.
Find the angular speed (w). Mass of one meter (a) Monkey A holds the string tightly and B goes
length of the spring is l = 0.126 gm–1. down with an acceleration ar = 2 m/s2 relative
to the string. Find the weight that A feels of
Q. 38. Two particles of mass m1 and m2 are in space at his own body.

separation r [vector from m1 to m2]. The only (b) What is the weight experienced by two
force that the two particles experience is the monkeys if A holds the string tightly and B
mutual gravitational pull. The force applied by goes down with an acceleration ar = 4 m/s2

 d 2 r  relative to the string.
m1 on m2 is F . Prove that m 2 = F Where
dt
m1m2
m is known as reduced mass for the two
m1 + m2
particle system.
m1

r A B
F m2

Q. 39. Six identical blocks – numbered 1 to 6 – have


been glued in two groups of three each and have Level 2
been suspended over a pulley as shown in fig.
The pulley and string are massless and the system Q. 41. Two strange particles A and B in space, exert no
is in equilibrium. The block 1, 2, 3, and 4 get force on each other when they are at a separation
detached from the system in sequence starting greater than x0 = 1.0 m. When they are at a distance
with block 1. The time gap between separation less than x0, they repel one another along the line
of two consecutive block (i.e., time gap between joining them. The repulsion force is constant
separation of 1 and 2 or gap between separation and does not depend on the distance between
of 2 and 3) is t0. Finally, blocks 5 and 6 remain the particles. This repulsive force produces an
connected to the string. acceleration of 6 ms–2 in A and 2 ms–2 in B when
(a) Find the final speed of blocks 5 and 6. the particles are at separation less than x0. In one
experiment particle B is projected towards A with
(b) Plot the graph of variation of speed of block 5
a velocity of 2 ms–1 from a large distance so as to
3.8 Problems in Physics for JEE Advanced

hit A head on. The particle A is originally at rest M M M


and the system of two particles do not experience masses M , , , ..................respectively. All
2 4 8
any external force. surfaces are smooth. Find the contact force
(a) Find the ratio of mass of A to that of B. between the block 1 and 2 after the system is
(b) Find the minimum distance between the released from rest. Also find the acceleration of
particles during subsequent motion. the wedge.

(c) Find the final velocity of the two particles.


4
Q. 42. A light string passing over a smooth pulley holds 3
two identical buckets at its ends. Mass of each
2
empty bucket is M and each of them holds M mass
A 1 B
of sand. The system was in equilibrium when a
small leak developed in bucket B (take this time W
as t = 0). The sand leaves the bucket at a constant
rate of m kg/s. Assume that the leaving sand
particles have no relative speed with respect to the
bucket (it means that there is no impulsive force Q. 45. In the system shown in fig, mass of the block
on the bucket like leaving exhaust gases exert on is m1 = 4 kg and that of the hanging particle is
a rocket). Find the speed (V0) of the two bucket m2 = 1 kg. The incline is fixed and surface is
when B is just empty. smooth. Block is initially held at the top of the
incline and the particle hangs a distance d = 2.0
m below it. [Assume that the block and the particle
are on same vertical line in this position]. System
is released from this position. After what time will
the distance between the block and the particle be
minimum ? Find this minimum distance. [g = 10
m/s2.]

A B

Q. 43. A chain is lying on a smooth table with half its


length hanging over the edge of the table [fig(i)]. If 30°
the chain is released it slips off the table in time t1.
Now, two identical small balls are attached to the Q. 46. A uniform chain of mass M = 4.8 kg hangs in
two ends of the chain and the system is released vertical plane as shown in the fig.
[fig(ii)]. This time the chain took t2 time to slip off (a) Show that horizontal component of tension is
the table. Which time is larger, t1 or t2? same throughout the chain.
(b) Find tension in the chain at point P where the
chain makes an angle q = 15° with horizontal.
(c) Find mass of segment AP of the chain.
[Take g = 10 m/s2; cos 15° = 0.96, sin 15° = 0.25]
A
60°
(i) (ii)

Q. 44. A triangular wedge W having mass M is placed


on an incline plane with its face AB horizontal.
Inclination of the incline is q. On the flat horizontal B
surface of the wedge there lies an infinite tower of q P 30°
rectangular blocks. Blocks 1, 2, 3, 4 ………. have
Newton’s Laws 3.9

Q. 47. Block A of mass M is placed on an incline plane, Q. 50. Blocks A and B have dimensions as shown in the
connected to a string, passing over a pulley as fig. and their masses are 8 kg and 1 kg respectively.
shown in the fig. The other end of the string also A small block C of mass 0.5 kg is placed on the top
carries a block B of mass M. The system is held in left corner of block A. All surfaces are smooth. A
the position shown such that triangle APQ lies in a horizontal force F = 18 N is applied to the block B
vertical plane with horizontal line AQ in the plane at time t = 0. At what time will the block C hit the
of the incline surface. ground surface? Take g = 10 m/s2.
C
P

4.0 m
2.0 m F = 18N
A B
a M
MA Q
B 4.0 m 2.75 m

Q. 51. Three identical smooth balls are placed between


q two vertical walls as shown in fig. Mass of each
5R
ball is m and radius is r = where 2R is
Find the minimum coefficient of friction between 9
separation between the walls.
the incline surface and block A such that the
system remains at rest after it is released. Take q = (a) Force between which two contact surface is
a = 45°. maximum? Find its value.
Q. 48. Figure shown a fixed surface inclined at an angle (b) Force between which two contact surface is
q to the horizontal. A smooth groove is cut on the minimum and what is its value?
incline along QR forming an angle f with PR. A
small block is released at point Q and it slides
down to R in time t. Find t.
Q

q

P R

Q. 49. In the system shown in the figure AB and CD 2R


are identical elastic cords having force constant Q. 52. A horizontal wooden block has a fixed rod OA
K. The string connected to the block of mass M standing on it. From top point A of the rod, two
is inextensible and massless. The pulley is also wires have been fixed to points B and C on the
massless. Initially, the cords are just taut. The end block. The plane of triangle OAB is perpendicular
D of the cord CD is gradually moved up. Find the to the plane of the triangle OAC. There are two
vertical displacement of the end D by the time the identical beads on the two wires. One of the wires
block leaves the ground.
A

C
q C
A O
q
a
M
B
B
3.10 Problems in Physics for JEE Advanced

is perfectly smooth while the other is rough. floor. The monkey remains at height h till the
The wooden block is moved with a horizontal block crosses it. At the instant the block is
acceleration (a) that is perpendicular to the line crossing the monkey it begins climbing up the
OB and it is observed that both the beads do not rope. Find the minimum acceleration of the
slide on the wire. Find the minimum coefficient of monkey relative to the rope, so that the block
friction between the rough wire and the bead. is not able to hit the floor. Do you think that a
Q. 53. In the arrangement shown in the fig. the pulley, monkey can climb with such an acceleration?
the spring and the thread are ideal. The spring is (g = 10 ms –2)
stretched and the two blocks are in contact with
a horizontal platform P. When the platform is
gradually moved up by 2 cm the tension in the
string becomes zero. If the platform is gradually
moved down by 2 cm from its original position
one of the blocks lose contact with the platform.
Given M = 4 kg; m = 2 kg.
(a) Find the force constant (k) of the spring A

(b) If the platform continues to move down after h


one of the blocks loses contact, will the other M
block also lose contact? Assume that that the h
platform moves very slowly.

Q. 55. An ideal spring is in its natural length (L) with two


objects A and B connected to its ends. A point
2L

P on the unstretched spring is at a distance
3
from B. Now the objects A and B are moved by
4 cm to the left and 8 cm to the right respectively.
Find the displacement of point P.
4 cm 2L
3 8 cm
M m P
P

Q. 54. In the arrangement shown in the fig. a monkey of L


mass M keeps itself as well as block A at rest by Q. 56. The fig. shows an infinite tower of identical springs
firmly holding the rope. Rope is massless and the each having force constant k. The connecting
pulley is ideal. Height of the monkey and block A
from the floor is h and 2h respectively [h = 2.5 m] F
A
(a) The monkey loosens its grip on the rope and
slides down to the floor. At what height from
B1
the ground is block A at the instant the money
hits the ground?
(b) Another block of mass equal to that of A is
stuck to the block A and the system is released.
The monkey decides to keep itself at height
h above the ground and it allows the rope to
slide through its hand. With what speed will
the block strike the ground?
(c) In the situation described in (b), the monkey
decides to prevent the block from striking the
Newton’s Laws 3.11

bars and all springs are massless. All springs are Q. 59. In the arrangement shown in the fig. all pulleys are
relaxed and the bottom row of springs is fixed to mass less and the strings are inextensible and light.
horizontal ground. The free end of the top spring is Block A has mass M.
pulled up with a constant force F. In equilibrium, (a) If the system stays at rest after it is released,
find find the mass of the block B.
(a) The displacement of free end A of the top (b) If mass of the block B is twice the value
spring from relaxed position. found in part (a) of the problem, calculate the
(b) The displacement of the top bar B1 from the acceleration of block A.
initial relaxed position.
Q. 57. In the system shown in the fig. there is no friction
and string is light. Mass of movable pulley P2
is M2. If pulley P1 is massless, what should be
value of applied force F to keep the system in
equilibrium?

P3
M A

B
P1

Q. 60. In the fig. shown, the pulley and string are mass
less and the incline is frictionless. The segment
P2
AP of the string is parallel to the incline and the
segment PB is perpendicular to the incline. End of
the string is pulled with a constant force F.
m
(a) If the block is moving up the incline with
Q. 58. In the system shown in the fig., the bead of mass acceleration while being in contact with the
m can slide on the string. There is friction between incline, then angle q must be less then q0.
the bead and the string. Block has mass equal to Find q0
twice that of the bead. The system is released from q0
(b) If q= find the maximum acceleration
rest with length l of the string hanging below the 2
bead. Calculate the distance moved by the block with which the block can move up the plane
before the bead slips out of the thread. Assume the without losing contact with the incline.
string and pulley to be massless. F
A
B

P
m

bead

Q.61. A triangular wedge A is held fixed and a block B


is released on its inclined surface, from the top.
Block B reaches the horizontal ground in time t. In
another experiment, the wedge A was free to slide
P2
on the horizontal surface and it took t’ time for the
block B to reach the ground surface after it was
2m released from the top. Neglect friction and assume
3.12 Problems in Physics for JEE Advanced

that B remains in contact with A. horizontal, and coefficient of friction between


(a) Which time is larger t or t´? Tell by simple shoes and the platform is 0.6.
observation. Q. 64. A wedge is placed on the smooth surface of a fixed
(b) When wedge A was free to move, it was incline having inclination q with the horizontal.
observed that it moved leftward with an The vertical wall of the wedge has height h and
g there is a small block A on the edge of the horizontal
acceleration and one of the two measured
4 surface of the wedge. Mass of the wedge and the
times (t & t´) was twice the other. Find the small block are M and m respectively.
inclination q of the inclined surface of the
(a) Find the acceleration of the wedge if friction
wedge.
between block A and the wedge is large
B enough to prevent slipping between the two.
(b) Find friction force between the block and the
wedge in the above case. Also find the normal
force between the two.
(c) Assuming there is no friction between the
A
block and the wedge, calculate the time in
which the block will hit the incline.
Q. 62. A block A is made to move up an inclined plane
of inclination q with constant acceleration a0 as
shown in figure. Bob B, hanging from block A A
by a light inextensible string, is held vertical and
q
is moving along with the block. Calculate the
magnitude of acceleration of block A relative to h
the bob immediately after bob is released.
a0 q

A Q. 65. In the system shown in figure, all surfaces are


smooth, pulley and strings are massless. Mass of
both A and B are equal. The system is released
from rest.
B

Q. 63. A 50 kg man is standing at the centre of a 30 kg


platform A. Length of the platform is 10 m and
coefficient of friction between the platform and B
the horizontal ground is 0.2. Man is holding one A
end of a light rope which is connected to a 50 kg
box B. The coefficient of friction between the box  
and the ground is 0.5. The man pulls the rope so (a) Find the a A . a B immediately after the system
as to slowly move the box and ensuring that he  
is released. a A and a B are accelerations of
himself does not move relative to the ground. If
the shoes of the man does not slip on the platform, block A and B respectively.

calculate how much time it will take for the man (b) Find a A immediately after the system is
to fall off the platform. Assume that rope remains released.
B Q.66. A block is placed on an incline having inclination
50 kg
q. There is a rigid L shaped frame fixed to
30 kg
the block. A plumb line (a ball connected to a
A
thread) is attached to the end A of the frame. The
10 m system is released on the inline. Find the angle
Newton’s Laws 3.13

that the plumb line will make with vertical in its placed in contact with the wedge B as shown. The
equilibrium position relative to the block when coefficient of friction between the block C and the
3 m
(a) the incline is smooth vertical wedge wall is m = 4 . Find the ratio for
M
(b) there is friction and the acceleration of the which the block C will not slide with respect to the
block is half its value when the incline is wedge after the system is released?
smooth A
A m C
M
4
B M
30° B

Q. 70. A smooth rod is fixed at an angle a to the


horizontal. A small ring of mass m can slide along
q the rod. A thread carrying a small sphere of mass
Q. 67. A wedge of mass m is placed on a horizontal M is attached to the ring. To keep the system in
smooth table. A block of mass m is placed at the equilibrium, another thread is attached to the ring
mid point of the smooth inclined surface having which carries a load of mass m0 at its end (see
length L along its line of greatest slope. Inclination figure). The thread runs parallel to the rod between
of the inclined surface is q = 45°. The block is the ring and the pulley.
released and simultaneously a constant horizontal All threads and pulley are massless.
force F is applied on the wedge as shown. (a) Find m0 so that system is in equilibrium.
(a) What is value of F if the block does not slide (b) Find acceleration of the sphere M immediately
on the wedge? after the thread supporting m0 is cut.
(b) In how much time the block will come out of
the incline surface if applied force is 1.5 times m
that found in part (a)
m0
L a
m
F
M

q m Q. 71. In the system shown in figure all surfaces are


smooth and string and pulleys are light. Angle of
Q. 68. A rod is kept inclined at an angle q with the Ê 3ˆ
horizontal A sleeve of mass m can slide on the wedge q = sin -1 Á ˜ . When released from rest
Ë 5¯
rod. If the coefficient of friction between the rod it was found that the wedge of mass m0 does not
and the sleeve is m, for what values of horizontal M
acceleration a of the rod, towards left, the sleeve move. Find .
m
will not slide over the rod?

a m M

m0 q
q

Q. 72. In the last problem take M = m and m0 = 2 m and


Q. 69. In the arrangement shown in figure, a block A of
calculate the acceleration of the wedge.
mass m has been placed on a smooth wedge B of
mass M. The wedge lies on a horizontal smooth Q. 73. In the system shown in the figure all surfaces are
M smooth, pulley and string are massless. The string
surface. Another block C of mass has been
4 between the two pulleys and between pulley and
3.14 Problems in Physics for JEE Advanced

block of mass 5 m is parallel to the incline surface


d 2 x1 d 2 x2
of the block of mass 4 m. The system is released (b) Find the value of 2
and
from rest. Find the acceleration of the block of dt dt 2
immediately after string AB is cut.
mass 4 m. ÈÍ tan 37∞ = ˘˙
3
Î 4˚ d 2 x1 d 2 x2
(c) Find the value of and
dt 2 dt 2
immediately after spring 2 is cut.
Q. 76. In the figure shown, the pulley, strings and springs
5m
are mass less. The block is moved to right by a
8m
4m distance x0 from the position where the two
37° springs are relaxed. The block is released from this
position.
Q. 74. In the system shown in figure, the two springs S1
and S2 have force constant k each. Pulley, springs K1
and strings are all massless. Initially, the system T0
M
is in equilibrium with spring S1 stretched and S2 K2
relaxed. The end A of the string is pulled down
slowly through a distance L. By what distance Smooth
does the block of mass M move?
(a) Find the acceleration of the block immediately
after it is released.
(b) Find tension (T0) in the support holding the
S1
pulley to the wall, immediately after the block
is released.
M
Assume no friction.
Q.77. The system shown in figure is in equilibrium.
Surface PQ of wedge A, having mass M, is
horizontal. Block B, having mass 2M, rests on
A wedge A and is supported by a vertical spring. The
S2
spring balance S is showing a reading of 2 Mg.
There is no friction anywhere and the thread QS
Q. 75. The system shown in figure is in equilibrium. is parallel to the incline surface. The thread QS
Pulley, springs and the strings are massless. The is cut. Find the acceleration of A and the normal
three blocks A, B and C have equal masses. x1 contact force between A and B immediately after
and x2 are extensions in the spring 1 and spring 2 the thread is cut.
respectively.

S
2M Q
P B
A M A

Spring 1 B

Spring 2

q = 45°

Q.78. A triangular wedge of mass M lies on a smooth


d 2 x2 horizontal table with half of its base projecting
(a) Find the value of immediately
dt 2 out of the edge of the table. A block of mass m is
after spring 1 is cut. kept at the top of the smooth incline surface of the
Newton’s Laws 3.15

wedge and the system is let go. Find the maximum È 3 2 ˘


M Ítan q = 4 ; g = 10 m / s ˙
value of for which the block will land on the Î
˚
m
x(m)
table. Take q = 60°. A
x 2.0
m F
0.5 A
t (sec)
x=0 O 1.0 2.0
–0.5
B
q
–2.0 A q

M
q Q.82. Block B of mass m has been placed on block A
of mass 3 m as shown. Block A rests on a smooth
horizontal table. F1 is the maximum horizontal
L L
2 2 force that can be applied on the block A such that
there is no slipping between the blocks. Similarly,
Q.79. In the system shown in the figure all surfaces are F2 is the maximum horizontal force that can be
smooth and both the pulleys are mass less. Block applied on the block B so that the two blocks move
on the incline surface of wedge A has mass m. Mass together without slipping on each other. When
of A and B are M = 4 m and M0 = 2 m respectively. F1 and F2 both are applied together as shown in
Find the acceleration of wedge A when the system figure.
is released from rest.
m F2
B

3m F1
A
m
Smooth
B
(a) Find the friction force acting between the
q A blocks.
(b) Acceleration of the two blocks.
Q.80. A block of mass m requires a horizontal force F0 to (c) If F2 is decreased a little, what will be direction
move it on a horizontal metal plate with constant of friction acting on B.
velocity. The metal plate is folded to make it a right
Q. 83. (i) In the arrangement shown in the figure the
angled horizontal trough. Find the horizontal force
coefficient of friction between the 2 kg block and
F that is needed to move the block with constant
the vertical wall is m = 0.5. A constant horizontal
velocity along this trough.
force of 40 N keeps the block pressed against the
wall. The spring has a natural length of 1.0 m and
F0
F
45° its force constant is k = 400 Nm–1. What should
be the height h of the block above the horizontal
Q.81. Block A of mass mA = 200 g is placed on an incline floor for it to be in equilibrium. The spring is not
plane and a constant force F = 2.2 N is applied on tied to the block.
it parallel to the incline. Taking the initial position
of the block as origin and up along the incline
as x direction, the position (x) time (t) graph of F = 40 N
2 kg
the block is recorded (see figure (b)). The same
experiment is repeated with another block B of
mass mB = 500 g. Same force F is applied to it
h
up along the incline and its position – time graph
is recorded (see figure (b)). Now the two blocks
are connected by a light string and released on
the same incline as shown in figure (c). Find the (ii) A block of mass M is pressed against a rough
tension in the string. vertical wall by applying a force F making an
3.16 Problems in Physics for JEE Advanced

angle of q with horizontal (as shown in figure). Q. 86. A block of mass m = 1 kg is kept pressed against a
Coefficient of friction between the wall and the spring on a rough horizontal surface. The spring is
block is m = 0.75. compressed by 10 cm from its natural length and to
m keep the block at rest in this position a horizontal
F
force (F) towards left is applied. It was found that
the block can be kept at rest if 8 N F 18 N.
q Find the spring constant (k) and the coefficient of
M friction (m) between the block and the horizontal
surface.

F
(a) If F = 2 Mg, find the range of values of q so m
that the block does not slide
[Take tan 37° = 0.75; sin 24° = 0.4] Q. 87. An experimenter is inside a uniformly accelerated
(b) Find the maximum value of q above which train. Train is moving horizontally with constant
equilibrium is not possible for any magnitude acceleration a0. He places a wooden plank AB in
of force F. horizontal position with end A pointing towards
Q. 84. A block is projected up along a rough incline with the engine of the train. A block is released at end
a velocity of u = 10 m/s. After 4 s the block was A of the plank and it reaches end B in time t1. The
at point B at a distance of 5 m from the starting same plank is placed at an inclination of 45° to the
point A and was travelling down at a velocity of horizontal. When the block is released at A it now
v = 4 m/s. t2 5
climbs to B in time t2. It was found that =2 4.
/s t1
4m What is the coefficient of friction between the
v=
/s block and the plank?
1 0m B
u= Direction of acceleration of the train

B
A 5m

q
B A
45
(a) Find time after projection at which the block A
came to rest.
Q. 88. Two hemispheres of radii R and r (< R) are fixed on
(b) Find the coefficient of friction between the
a horizontal table touching each other (see figure).
block and the incline.
A uniform rod rests on two spheres as shown. The
Take g = 10 m/s2 coefficient of friction between the rod and two
Q. 85. A long piece of paper is being pulled on a spheres is µ. Find the minimum value of the ratio
horizontal surface with a constant velocity u along
r
for which the rod will not slide.
its length. Width of the paper is L. A small block R
moving horizontally, perpendicular to the direction
of motion of the paper, with velocity v slides onto
the paper. The coefficient of friction between the
block and the paper is m. Find maximum value of R r
v such that the block does not cross the opposite O1 O2
edge of the paper.
Q. 89. In order to lift a heavy block A, an engineer has
u designed a wedge system as shown. Wedge C is
L
v fixed. A horizontal force F is applied to B to lift
block A. Wedge B itself has negligible mass and
mass of A is M. The coefficient of friction at all
Newton’s Laws 3.17

surfaces is m. Find the value of applied force F at and strings are mass less. Mass of block C is M.
which the block A just begins to rise. [g = 10 m/s2]

A A

B
q
B

q q
C (fixed)
C
Q.90. A 60 kg platform has been placed on a rough (a) Find value of M for which block B does not
incline having inclination q = 37°. The coefficient
accelerate
of friction between the platform and the incline
(b) Find maximum value of M for which A does
is m = 0.5. A 40 kg man is running down on the
not accelerate.
platform so as to keep the platform stationary.
What is the acceleration of the man? It is known Q.93. In the arrangement shown in figure, pulley and
that the man cannot manage to go beyond an string are light. Friction coefficient between the
È 3˘ two blocks is m whereas the incline is smooth.
acceleration of 7 m/s2. Ísin 37∞ = 5 ˙ Block A has mass m and difference in mass of
Î ˚
the two blocks is m. Find minimum value of m
40 kg 0.5
m for which the system will not accelerate when
released from rest.

60 kg

A
q 
B

Q. 91. In the system show in figure, mass of the


block placed on horizontal surface is M =

4 kg. A constant horizontal force of F = 40 N
is applied on it as shown. The coefficient of Q. 94. In the arrangement shown in figure pulley P can
friction between the blocks and surfaces is move whereas other two pulleys are fixed. All of
m = 0.5. Calculate the values of mass m of the them are light. String is light and inextensible. The
block on the incline for which the system does not coefficient of friction between 2 kg and 3 kg block
È 3 ˘ is m = 0.75 and that between 3 kg block and the
move. Ísin 37∞ = ; g = 10m / s2 ˙ table is m = 0.5. The system is released from rest
Î 5 ˚
 2 kg
M
F
 3 kg
m

P
q = 37°

M
Q. 92. In the arrangement shown in the figure, block A
of mass 8 kg rests on a horizontal table having (i) Find maximum value of mass M, so that the
coefficient of friction m = 0.5. Block B has a system does not move. Find friction force
mass of 6 kg and rests on a smooth incline having between 2 kg and 3 kg blocks in this case.
Ê 2ˆ (ii) If M = 4 kg, find the tension in the string
inclination angle q = sin -1 Á ˜ . All pulleys
Ë 5¯ attached to 2 kg block.
3.18 Problems in Physics for JEE Advanced

(iii) If M = 4 kg and m1 = 0.9, find friction force D E


between the two blocks, and acceleration of
B C
M. 30°

(iv) Find acceleration of M if m1  = 0.75, m2  =  -0.9


A Vertically down
and M = 4 kg.
60°
Êp ˆ
Q. 95. A rope of length Á + 1˜ R has been placed on a (a) Find m
Ë2 ¯
smooth sphere of radius R as shown in figure. End (b) x is distance measured along the length of the
A of the rope is at the top of the sphere and end B rope starting from point A. Plot the variation
is overhanging. Mass per unit length of the rope is of tension in the rope (T) with distance x.
. The horizontal string holding this rope in place (c) Find the maximum tension in the rope.
can tolerate tension equal to weight of the rope.
Find the maximum mass (M0) of a block that can Q. 98. (i) Four small blocks are interconnected with
be tied to the end B of the rope so that the string light strings and placed over a fixed sphere as
does not break. shown. Blocks A, B and C are identical each
having mass m = 1 kg. Block D has a mass of m´
String
= 2 kg. The coefficient of friction between the
blocks and the sphere is m = 0.5. The system is
A
released from the position shown in figure.
A
D B

Rope 37° 37°


53°
C
B

Q.96. A uniform rope has been placed on a sloping


surface as shown in the figure. The vertical
separation and horizontal separation between the
end points of the rope are H and X respectively. (a) Find the tension in each string. Which string
The friction coefficient (m) is just good enough has largest tension?
to prevent the rope from sliding down. Find the
value of m. (b) Find the friction force acting on each block.
A
È 3 2˘
ÍTake tan 37∞ = 4 ; g = 10 m / s ˙
Î ˚
(ii) A fixed square prism ABCD has its axis
H
horizontal and perpendicular to the plane
of the figure. The face AB makes 45° with
the vertical. On the upper faces AB and
B
X BC of the prism there are light bodies P
and Q respectively. The two bodies (P and
Q.97. A uniform rope ABCDE has mass M and it is Q) are connected using a string S1 and
laid along two incline surfaces (AB and CD) and strings S0 and S2 are hanging from P and Q
two horizontal surfaces (BC and DE) as shown respectively. All strings are mass less, and
in figure. The four parts of the rope AB, BC, CD inextensible. String S1 is horizontal and the
and DE are of equal lengths. The coefficient of other two strings are vertical. The coefficient
friction (m) is uniform along the entire surface of friction between the bodies and the prism
and is just good enough to prevent the rope from is m . Assume that P and Q always remain in
sliding. contact with the prism.
Newton’s Laws 3.19

S1 that is vertically above the centre of the sphere.


The other end of the string is connected to a small
P B Q particle of mass m that rests on the sphere. The
string makes an angle a = 30° with the vertical.
45°
A C Find the acceleration of the sphere immediately
after it is released. There is no friction anywhere.
S0 S2
D
Q. 102. A light rod AB is fitted with a small sleeve of
mass m which can slide smoothly over it. The
(a) If tension in S0 is T0, find the minimum sleeve is connected to the two ends of the rod
tension (T1) in S1 to keep the body P at rest. using two springs of force constant 2k and k (see
fig). The ends of the springs at A and B are fixed
(b) A mass M0 is tied to the lower end of
and the other ends (connected to sleeve) can move
string S0 and another mass m2 is tied to S2.
along with the sleeve. The natural length of spring
Find the minimum value of m2 so as to keep
connected to A is l0. Now the rod is rotated with
P and Q at rest.
angular velocity w about an axis passing through
Q. 99. A metal disc of radius R can rotate about the k
vertical axis passing through its centre. The top end A that is perpendicular to the rod. Take mw 2 = h
surface of the disc is uniformly covered with and express the change in length of each spring
dust particles. The disc is rotated with gradually (in equilibrium position of the sleeve relative to
increasing speed. At what value of the angular the rod) in terms of l0 and h.
speed (w) of the disc the 75% of the top surface
w
will become dust free. Assume that the coefficient
of friction between the dust particles and the metal B
A
disc is m = 0.5. Assume no interaction amongst 2K m K
the dust particles.

Q. 103. A metallic hemisphere is having dust on its



surface. The sphere is rotated about a vertical axis
passing through its centre at angular speed w =
R
10 rad s –1. Now the dust is visible only on top
20% area of the curved hemispherical surface.
Radius of the hemisphere is R = 0.1 m. Find the
coefficient of friction between the dust particle
and the hemisphere [g = 10 ms –2].
w
Q.100. In the last question, the axis of the disc is tilted Dust
slightly to make an angle q with the vertical. Redo
the problem for this condition and check the result
by putting q = 0 in your answer.
Q. 101.
Q. 104. Civil engineers bank a road to help a car negotiate a
 curve. While designing a road they usually ignore
friction. However, a young engineer decided to
include friction in his calculation while designing
a road. The radius of curvature of the road is R
and the coefficient of friction between the tire and
the road is m.
(a) What should be the banking angle (q ) so that
A sphere of mass M is held at rest on a horizontal car travelling up to a maximum speed V0 can
floor. One end of a light string is fixed at a point negotiate the curve.
3.20 Problems in Physics for JEE Advanced

(b) At what speed (V1) shall a car travel on a road Q.108. Three small discs are connected with two
banked at q0 so that there is no tendency to identical massless rods as shown in fig. The rods
skid. (No tendency to skid means there is no are pinned to the discs such that angle between
static friction force action on the car). them can change freely. The system is placed on
(c) The driver of a car travelling at speed (V1) a smooth horizontal surface with discs A and B
starts retarding (by applying brakes). What touching a smooth wall and the angle ACB being
angle (acute, obtuse or right angle) does the 90°. A force F is applied to the disc C in a direction
resultant friction force on the car make with perpendicular to the wall. Find acceleration of
the direction of motion? disc B immediately after the force starts to act.
Masses of discs are mA = m; mB = 2 m; mC = m
Q. 105. A turn of radius 100 m is banked for a speed of [wall is perpendicular to the plane of the fig.]
20 m/s A
m
(a) Find the banking angle
(b) If a vehicle of mass 500 kg negotiates the
curve find the force of friction on it if its
m
speed is – (i) 30 m/s (ii) 10 m/s C
90°
Assume that friction is sufficient to prevent F

skidding and slipping. wall


[Take tan 22° = 0.4, sin 22° = 0.375, cos 22° = 2m
0.93, g = 10 ms –2] B

Q. 106. A horizontal circular turning has a curved length


L and radius R. A car enters the turn with a speed Q. 109. Figure shows two blocks in contact placed on an
V0 and its speed increases at a constant rate f. If incline of angle q = 30°. The coefficient of friction
the coefficient of friction is m, between the block of mass 4 kg and the incline is
m1, and that between 2 kg block and incline is m2.
(a) At what time t0, after entering the curve, will Find the acceleration of the blocks and the contact
the car skid? (Take it for granted that it skids force between them if –
somewhere on the turning) (a) m1 = 0.5, m2 = 0.8
(b) At a time t (< t0) what is the force of friction (b) m1 = 0.8, m2 = 0.5
acting on the car? (c) m1 = 0.6, m2 = 0.1 [Take g = 10 m/s2]

Q. 107. A 70 kg man enters a lift and stands an a weighing
scale inside it. At time t = 0, the lift starts 2k
g
g
moving up and stops at a higher floor at t = 9.0 4k
s. During the course of this journey, the weighing
scale records his weight and given a plot of his
weight vs time. The plot is shown in the fig. = 30°
[Take g = 10 m/s2]
(a) Find F0 Q. 110. A small collar of mass m = 100 g slides over the
surface of a horizontal circular rod of radius R =
(b) Find the magnitude of maximum acceleration
0.3 m. The coefficient of friction between the rod
of the lift.
and the collar is m = 0.8. Find the angle made with
(c) Find maximum speed acquired by the lift.
vertical by the force applied by the rod on the collar
F
when speed of the collar is V = 2 m/s.
800N

700N 6.0 9.0


o t (s)
0.2 2.8 3.0

Fo
Newton’s Laws 3.21

Q. 111. A flat race track consists of two straight section is suspended at the centre of the room tied to
AC and DB each of length 180 m and one semi three inextensible strings as shown. String BA is
circular section DC of radius R = 150 m. A car horizontal with A being the centre point of the
starting from rest at A has to reach B in least wall. Find the ratio of tension in the string BA and
possible time (the car may cross through point BC.
B and need not stop there). The coefficient of Q.114. Two identical smooth disc of radius R have been
friction between the tyres and the road is m = placed on a frictionless table touching each other.
0.6 and the top speed that the car can acquire Another circular plate is placed between them as
is 180 kph. Find the minimum time needed shown in figure. The mass per unit area of each
to move from A to B under ideal conditions. object is s, and the line joining the centers of the
Braking is not allowed in the entire journey plate and the disc is q
[g = 10 m/s 2]
(a) Find the minimum horizontal force F0 that
must be applied to the two discs to keep them
together.
C R D
O (b) Angle q can be changed by changing the size
of the circular plate. Find F0 when q 0.
È q2 ˘
180 m 180 m Íuse cos q = 1 - and sin q = q for small q ˙
Î 2 ˚

(c) Find F0 when q Æ p . Explain the result.


A B 2

Q. 112. A small insect is climbing slowly along the



inner wall of a hemispherical bowl of radius R.
The insect is unable to climb beyond q  =  45°.
Whenever it tries to climb beyond q  =  45°, it F R F
slips.
(a) Find the minimum angular speed w with
which the bowl shall be rotated about its
vertical radius so that the insect can climb Q. 115. Three identical smooth cylinders, each of mass
upto q = 60°. m and radius r are resting in equilibrium within
(b) Find minimum w for which the insect can a fixed smooth cylinder of radius R (only a part
move out of the bowl. of this cylinder has been shown in the fig). Find
w the largest value of R in terms of r for the small
cylinders to remain in equilibrium.

Q. 113. C
D
Q. 116. A massless spring of force constant K and natural
length l0 is hanging from a ceiling. An insect of
B A mass m is sitting at the lower end of the spring
and the system is in equilibrium . The insect starts
slowing climbing up the spring so as to eat a bug
A room is in shape of a cube. A heavy ball (B) sitting on the ceiling. Assume that insect climbs
3.22 Problems in Physics for JEE Advanced

mg (a) Find the maximum value of F (call it F0)


without slipping on the spring and K . Find so that all the four blocks move with same
l0
1 acceleration.
the length of the spring when the insect is at th
4 (b) Will the value of F0 increase or decrease if
of its original distance from the bug.
another block (E) of mass m2 is placed above
block D and coefficient of friction between E
Bug
and D is m?
Q. 119. A chain with uniform mass per unit length lies
in a vertical plane along the slope of a smooth
hill. The two end of the chain are at same height.
K
If the chain is released from this position find its
acceleration.

insect

Q. 117. In the system shown in fig., all pulleys are mass


less and the string is inextensible and light.
(a) After the system is released, find the
acceleration of mass m1
pR
(b) If m1 = 1 kg, m2 = 2 kg and m3 = 3 kg then Q. 120. A uniform rope of length has been placed on
2
what must be value of mass m4 so that it fixed cylinder of radius R as shown in the fig. One
accelerates downwards? end of the rope is at the top of the cylinder. The
coefficient of friction between the rope and the
cylinder is just enough to prevent the rope from
sliding. Mass of the rope is M.
(a) At what position, the tension in the rope is
m1 m2 m3 maximum?
(b) Calculate the value of maximum tension in
the rope.

m4 R
Q. 118. In the system shown in fig., block A and C are
placed on smooth floors and both have mass equal
to m1. Blocks B and D are identical having mass
m2 each. Coefficient of friction Q. 121. In the last problem, the rope is placed on the
cylinder as shown. Find maximum tension in the
D
rope.
C F

B 45° 45°

Between A and B and that between C and D are


both equal to m. String and pulleys are light. A
horizontal force F is applied on block C and is Q. 122. A 4 kg block is placed on a rough horizontal
gradually increased. surface. The coefficient of friction between the
Newton’s Laws 3.23

block and the surface is m = 0.5. A force F = 18 N Q. 125. A smooth cylinder is fixed with its axis horizontal.
is applied on the block making an angle q with the Radius of the cylinder is R. A uniform rope (ACB)
horizontal. Find the range of values of q for which of linear mass density (kg/m) is exactly of length
the block can start moving. R and is held in semicircular shape in vertical
ÈTake g = 10 m/s2 , tan -1 (2) = 63∞˘ plane around the cylinder as shown in figure. Two
Í ˙ massless strings are connected at the two ends of

Í -1 Ê 10 ˆ ˙ the rope and are pulled up vertically with force T0
Í sin ÁË ˜¯ = 84 ∞ ˙
Î 9 1.25 ˚ to keep the rope in contact with the cylinder.

F = 18 N
(a) Find minimum value of T0 so that the rope
does not lose contact with the cylinder at any

point.
4kg
(b) If T0 is decreased slightly below the minimum
m = 0.5 value calculated in (a), where will the rope
lose contact with the cylinder.
Q. 123. Two rectangular blocks A and B are placed on
T0 T0
a horizontal surface at a very small separation.
The masses of the blocks are mA = 4 kg and mB =
5 kg. Coefficient of friction between the horizontal
surface and both the blocks is m = 0.4. Horizontal A
O
B
forces F1 and F2 are applied on the blocks as
shown. Both the forces vary with time as R

F1 = 15 + 0.5 t
F2 = 2t C

Where ‘t’ is time in second. Q. 126. A block of mass m placed on an incline just begins
Plot the variation of friction force acting on the to slide when inclination of the incline is made
two blocks ( A and B) vs time till the motion q = 45°. With inclination equal to q = 30°, the
starts. Take rightward direction to be positive for block is placed on the incline. A horizontal force
(F) parallel to the surface of the incline is applied
B and leftward direction to be positive for A.
5 kg to the block. The force F is gradually increased
4 kg from zero. At what angle a to the force F will the
F1 F2 block first begin to slide?
A B

Level 3 a
F

Q. 124. A rope of mass m is hung from a ceiling. The


centre point is pulled down with a vertical force q
F. The tangent to the rope at its ends makes an
angle a with horizontal ceiling. The two tangents
at the lower point make an angle of q with each Q. 127. In the last problem if it is allowed to apply the
other. Find q. force F in any direction, find the minimum force
Fmin needed to move the block on the incline.
a a
Q. 128. A block A has been placed symmetrically over
two identical blocks B and C. All the three blocks
q have equal mass, M each, and the horizontal
surface on which B and C are placed is smooth.
The coefficient of friction between A and either
of B and C is m. The block A exerts equal pressure
F
on B and C. A horizontal force F is applied to the
3.24 Problems in Physics for JEE Advanced

block B. (a) Find tension in the string.


A (b) Find the acceleration of the hemisphere
immediately after the string is cut.
F
B C Q. 131. A semicircular ring of radius R is fixed on a smooth
horizontal table. A small block is projected with
speed u so as to enter the ring at end A. Initial
(a) Find maximum value of F so that A does velocity of the block is along tangent to the ring at
not slip on B or C and the three blocks move A and it moves on the table remaining in contact
together. with the inner wall of the ring. The coefficient of
(b) If F is increased beyond the maximum found friction between the block and the ring is m.
in (a) where will we see slipping first- at (a) Find the time after which the block will exit
contact of A and B or at the contact of A and the ring at B.
C.
(b) With what speed will the block leave the ring
(c) If F is kept half the maximum found in (a), at B.
calculate the ratio of friction force between A A
and B to that between A and C. Does this ratio
change if F is decreased further? u

Q. 129. In the arrangement shown in the figure the


coefficient of friction between the blocks C and
D is m = 0.7 and that between block D and the
horizontal table is m = 0.2. The system is released
from rest. [ Take g = 10 ms –2] Pulleys and threads
are massless.
C B
3 kg
D Q. 132. A long helix made of thin wire is held vertical.
1.5 kg The radius and pitch of the helix are R and r
A respectively. A bead begins to slide down the
3 kg
0.5 kg B helix.

(a) Find the acceleration of the block C. (a) Find the normal force applied by the wire on
the bead when the speed of the bead is v.
(b) Block B is replaced with a new block. What
shall be the minimum mass of this new block (b) Eventually, the bead acquires a constant
so that block C and D accelerate in opposite speed of v0. Find the coefficient of friction
direction? between the wire and the bead.

Q. 130. A hemisphere of mass M and radius R rests on a


smooth horizontal table. A vertical rod of mass m
is held between two smooth guide walls supported
on the sphere as shown. There is no friction
between the rod and the sphere. A horizontal r
string tied to the sphere keeps the system at rest.

R
m
Q. 133. A wedge of mass m is kept on a smooth table and
its inclined surface is also smooth. A small block
of mass m is projected from the bottom along the
M R String incline surface with velocity u. Assume that the
block remains on the incline and take q = 45°,
q g = 10 m/s2.
Newton’s Laws 3.25

(a) Find the acceleration of the wedge and the string AO is twice the tension (T2) in string
x and y components of acceleration of the BO
block. (d) Assume that both strings are taut when the
(b) Draw the approximate path of the block as string AO breaks. What will be nature of path
observed by an observer on the ground. At of the particle moment after AO breaks ?
what angle does the block hit the table? Q. 136. A sphere of mass m and radius r = 3m is placed
(c) Calculate the radius of curvature of the path inside a container with flat bottom and slant
of the block when it is at the highest point. sidewall as shown in the figure. The sphere
touches the slant wall at point A and the floor at
x
point B. It does not touch any other surface. The
container, along with the sphere, is rotated about
y
the central vertical axis with angular speed w.
u The sphere moves along with the container, i.e.,
m m it is at rest relative to the container. The normal
q = 45 force applied by the bottom surface and the slant
surface on the sphere are N1 and N2 respectively.
Q. 134. A cylinder with radius R spins about its horizontal There is no friction.
axis with angular speed w. There is a small block
(a) Find the value of w above which N2 becomes
lying on the inner surface of the cylinder. The
larger than N1
coefficient of friction between the block and the
cylinder is m. Find the value of w for which the (b) Find the value of w above which the sphere
block does not slip, i.e., stays at rest with respect leaves contact with the floor.
w
to the cylinder.
w

r
R A
B 60°
2.0 m

Q. 135. A particle of mass m is attached to a vertical rod Q. 137. A car is being driven on a tilted ground. The
with two inextensible strings AO and BO of equal ground makes an angle q with the horizontal.
lengths l. Distance between A and B is also l. The The driven drives on a circle of radius R. The
setup is rotated with angular speed w with rod as coefficient of friction between the tires and the
the axis. ground is m.
(a) What is the largest speed for which the car
will not slip at point A? Assume that rate of
A
l
change of speed is zero.
O (b) What is the largest constant speed with which
l
m the car can be driven on the circle without
slipping?
l
B

O
(a) Find the values of w for which the particle B A
remains at point B.
(b) Find the range of values of w for which q
tension (T1) in the string AO is greater than
mg but the other string remains slack Q. 138. A particle P is attached to two fixed points O1
(c) Find the value of w for which tension (T1) in and O2 in a horizontal line, by means of two
3.26 Problems in Physics for JEE Advanced

light inextensible strings of equal length l. It is (a) Find the normal force applied by each of the
projected with a velocity just sufficient to make it blocks on the sphere at the instant separation
describe a circle, in a vertical plane, without the between the blocks is a = 2R; R = 1.0 m
strings getting slack and with the angle < O2O1P being the radius of the ball.
= <O1O2P = q. When the particle is at its lowest
point, the string O2P breaks and the subsequent
path of the particle was found to be a circle of C
radius l cos q. Find q.

A B
O1 O2 V a V
q q

(b) How much force must be applied on each of


P the two blocks (when a = 2R) so that they do
Q. 139. The arrangement shown in figure is in equilibrium not have any acceleration. Assume that the he
with all strings vertical. The end A of the string horizontal surface is smooth.
is tied to a ring which can be slid slowly on the Q. 141. In the figure all pulleys (P1, P2, P3 …….) are
horizontal rod. Pulley P1 is rigidly fixed but P2 massless and all the blocks (1,2,3 …..) are
can move freely. A mass m is attached to the identical, each having mass m. The system
centre of pulley P2 through a thread. Pulleys and consist of infinite number of pulleys and blocks.
strings are mass less. Strings are light and inextensible and horizontal
A surfaces are smooth. Pulley P1 is moved to
left with a constant acceleration of a0. Find the
acceleration of block1. Assume the strings to
P1
remain horizontal.
P3
P2
P1
3
M 2
P2 a0 1

m Q. 142. A small disc P is placed on an inclined plane


forming an angle q with the horizontal and
(a) Which block will move up as A is moved imparted an initial velocity v0. Find how the
slowly to the right? velocity of disc depends on the angle f which its
(b) Will the block of mass m have horizontal velocity vector makes with the x axis (see figure).
displacement? The coefficient of friction is m = tan q and initially
p
(c) Is it possible, for a particular position of A, f0 = .
that M has no acceleration but m does have 2
an acceleration? If this happens when string
from P2 to A makes an angle q with vertical, P
find the acceleration of m at the instant.
v f
Q. 140. A smooth spherical ball of mass M = 2 kg is
resting on two identical blocks A and B as shown
in the figure. The blocks are moved apart with
q
same horizontal velocity V = 1 m/s in opposite
directions (see figure).
x
Newton’s Laws 3.27

ANSWERS

(a) straight line (b) 22400 N


(b) Parabolic (c) 190400 N
20 m (d)
T0 and F do not change. T will increase.
19. (a) No
4H
3. (b) 1 : 4
3g
F F
N = 12; Tension = = 2h
N 12 2
5. 16 N g
6. (i) (a) True
(b) True aA = aB = F ; ac = 0
2m
g
(ii) 22. 3M/5
5
q = 62.5°
(iii)
2 ηh 4. RA : RB : RC = 3 : 1 : 2
(η − 1)g K = 2 Sdg
5
2( M − m)L 26. (a) cms –1
7. t= 3
Mg
(b) 6 N
g 27. (a) 2, 4
(a) a =
4 (b) In both cases acceleration of the frame must be ‘g’.
g 15 m
(b) a =
5
6t
4 Mmg
+ M0 g 36 – p
M+m
v02
8m1m2 30. m £
m0 = 4m1m2 ; M = gL
m1 + m2 m1 + m2
31. 4 kg
All masses will fall down with acceleration g
4.8 kg
11. (a) More than 9/2 Mg
(b) Tension in S2 = Mg/2, Tension in S1 = 5 Mg 174 N
(c) Tension in S2 = Mg/6 1.5 kg m 9.5 kg
q 45°, g/2
5F
73.1 N
m
12.5 s (a) At C
Mg (b) At C
T = 2 Mg; N =
2
(c) 2 m/s2 and 2 m/s2
Zero.
mmin = 1 (i) 15 <w 16.67 rad/s
(a) 6467 N (ii) 500 rad s–1
3.28 Problems in Physics for JEE Advanced

8gt 4
(a) 0 It is mg.
15 3
(b) sin q
mmin =
8
— gt0
15
cos q + tan 2 q
2

(a) K = 2.5 N/cm


gt0
——
5
(b) No
(a) The block is at height h = 2.5 m
o 2 t0 3t0 t
t0
(b)
V = 5 2 m/s
(a) 80 N
(c) 25 m/s2 ( )
640 Zero
(b) N for both
9
2F
(a)
(a) m A = 1 K
mB 3
F
(b)
(b)
Xmin = 0.75 m K
With pulley P1 having zero mass, equilibrium is not
(c) VA¢ = Ê 3 + 3 ˆ ms -1; VB¢ = Ê 3 - 1 ˆ ms -1 possible
ÁË 2 ˜ ÁË 2 ˜
2¯ 2¯
l

Mg È Ê 4ˆ ˘ 3
V0 = 4ln Á ˜ - 1˙
m ÍÎ Ë 3¯ ˚
(a) M
43. t2 g
(b)
3
Mg cos2 q
N12 =
1 + 2 sin 2 q p
. (a) q 0 =
3g sin q 4
a=
1 + 2 sin 2 q È Êpˆ Êp ˆ˘
amax = g Ícos Á ˜ - sin Á ˜ ˙
(b)
1.0 s, 1.0 m Î Ë 8 ¯ Ë 8¯˚
(b) TP = 21.65 N (a) t > t1
(c) 3.05 kg
-1 Ê 1 ˆ
(b) q = tan Á ˜
Ë 12 ¯
5–2 2
a0 cos q
2 –1
10
1 2d t s
t= 3
sin θ sin φ g
(a) g sin q
5Mg
49. 1
2K (b) = mg sin 2q
2
50. 2.9 s N = mg cos2 q
51. (a) Force between the wall and the middle ball is
maximum. It is 4 mg
t=
(c)
(
2h M + m sin 2 q )
(b) Force between upper ball and wall is least. ( M + m ) g sin 2
q
Newton’s Laws 3.29
 
65. (a) a A ⋅ aB = 0 immediately after release 4k1k2 x0
(b)
k1 + k2
 g
a A = ( ←)
(b) a=
g
; NAB = 0
2 2
q
(a) 78. 3

Ê sin q ◊ cos q ˆ 6g
tan -1 Á
(b)
Ë 2 - sin 2 q ˜¯ 47

(a) 2 mg 2 F0
T = 0.49 N
3L
t=
(b) (a) Zero
2g
F1 F2
(b) =
g (sin q - m cos q ) g (sin q + m cos q ) 3m m
68. £a£
(cosq + m sin q ) (cosq - m sin q ) (c) To right
(a) q 13°
m 20
= = 16.7 (b) 37°
M 3 3–4
13
(a) (M + m) g sin a (a) s
7
( M + m ) g sin 2 a
(b) (b) 0.18
m + M sin 2 a
-u 2 u4
+ (2m gL )
2
Vmax = +
M 1 2 4
=
m 5
k = 130 Nm–1; m = 0.5
48g
a0 = Ê
m = 3a0 – 4 g
ˆ
199 ÁË 4a + 3g ˜¯
0

44 g

205 Êrˆ 1 + m2 - m
Á ˜ =
Ë R ¯ min 1 + m2 + m
74.
2L
5
Mg È m cos q + sin q ˘
F= Í m + cos q - m sin q ˙
1 - m2 Î ˚
d2 x 3g
(a) 2 =
dt 2 2 5 m/s2 a 7 m/s2
2 kg m 30 kg
d2 x d 2 x2
(b) 1 = 2 g; = 2g
dt 2 dt 2 960
(a) kg
95
d2 x g d 2 x2 3g
(c) 21 ; 480
dt 2 dt 2 2 (b) kg
61
4k1k2 x0 Dm
(a) mmin = tan q
( k1 + k2 ) M 2m
3.30 Problems in Physics for JEE Advanced

(i) 2.5 kg; 12.5 N 103. 2.45


Ê V0 2 ˆ
50
(ii) N Á Rg - m ˜
3 (a) q 0 = tan -1 Á ˜
Á mV02 ˜
40 5 ÁË 1 + Rg ˜¯
(iii) N,a m/s2
3 3
V1 = rg tanq 0
(b)
5
(iv) m /s2 (c) Obtuse
6
(a) 22°
π
M 0 = λR  – 1 (b) (i) 2315 N, 1389 N
2 

( )
1

H  R 2 µ2 g 2 − f 2  4 − V
(a) t =   0

x 0
f

(a) m =
3 +1
= 0.4
(V0 + ft )4
(b) m + f2
3+5 R2

(b) T
(a) 93.3 N
10
0.17 Mg (b) m/s2
7
0.1 Mg
(c) 4 m/s
0.07 Mg
F

X 5m
A B C D E

(c) Tmax = 0.17 Mg 109. (a) contact force = 0, acceleration of 4 kg block is


0.7 m/s2 and that of other block is zero
(i) (a) TBC = 10 N; TAB = 12 N; TAD = 7 N
(b) contact force = 1.4 N, acceleration of both = 0
(b) C = 0; B = 4N; A = 5N; D = 5N (c ) Contact force = 5.74 N, acceleration of both =
Ê 1 – m0 ˆ 1.27 m/s2
(ii) (a) T1 = Á T0
Ë 1 + m0 ˜¯ -1
q = cos Á
Ê 3 ˆ
Ë 41 ˜¯
2
Ê 1 – m0 ˆ 111. 30.1 s
(b) m2 =
ÁË 1 + m ˜¯ M 0
0

(a)
(
g 2 3 –1 )
w= g R 3 3 +1( )
R

(b) g
g
w= (cosq - 2 sin q ) R
R
2
3mg
3
3m + 4 M

(a) F = sp R g (1 - cos q )
2 2
l
102. x = 0 0
3h – 1 2 sin q .cos q
Newton’s Laws 3.31

(b)
F0 = 0
(c)
127. Fmin =
mg
2 2
( 3 −1 )
R = r (1 + 2 7) 3
128. (a) Fmax = m Mg
5l0 4
116.
4 (b) Between A and B
(c) 2, No
 
(a) a1 = g 1 – 4 M 129. (a) 2 ms–1
 m1 
(b) 2.1 kg
18 (a) T = N cos q
m4 >
(b) kg
11
mg
(b)
 m + m1  M tan q + m cot q
F0 = 2mm2 g  2  ; increase
 2m2 + m1  R pm
(a) t = Èe - 1˘˚
119. Zero mu Î

(a) q = 45° from vertical diameter. u


V=
(b)
eπµ
Ê 2 - 1ˆ
(b)
Tmax = 2 Á ˜ Mg Ê v 2 cos q ˆ
2
Ë p ¯
(a) mg cos q 1 + Á
Ë Rg ˜¯
121. Zero
21° < q < 33° tan q
fA (N) 2 r
(b) Ê v 2 cos q ˆ where tan q =
1+ Á 2p R
Ë Rg ˜¯
16
 g
34 t (s) 133. (a) a wedge = iˆ
O 3
2 12.67 23.33
-16 –g
a x block =
3
fB (N)
2g
a y block =
3
20
(b) The block hits the table normally.

4
O t (s)
2 12.67 34

ÈÊ mg ˆ ˘
124. q = 2 tan -1 ÍÁ 1 + ˜¯ cot a ˙
ÎË F ˚
3u2
(a) T0 = 2 Rg (c)
16 g
(b) At the lowest point
−1  1  w≥ g 1 + m2
sin  
 3 Rm
3.32 Problems in Physics for JEE Advanced

g Ê 1 ˆ
135. (a) w > q = tan -1 Á
l Ë 5 ˜¯

139. (a) Block with mass M will move up.


g 2g
(b) < w £ (b) yes
l l
g (1 – cos q)
(c)
(c) 6g (a) (10 2 – 8) N
l
(b) (5 – 4 2)N
(d) parabolic
3a0
g
(a) 2
3
v0
(b) 3g
1 + cosf
. (a) [g2 R2 (m2 cos2 q – sin2 q)]1/4

gR ( m cos q - sin q )
(b)

SOLUTIONS

 
1. (a) Initial velocity is parallel to F or anti parallel to F . Hence particle moves in a straight line and speed may
increase or decrease.
(b) Path is parabolic with speed increasing.
In case (a) the particle may retrace its path.
2. Just before striking the ceiling, retardation is 2 g. If air resistance force is R at this instant, then
v R

a R v
mg
mg

    ma = mg + R
m (2 g) = mg + R
    R = mg
After impact, the air resistance force will be upward but its magnitude will remain mg. This is because speed has
not changed.
After impact net force on the ball = 0
Ball will fall down with constant speed
H = (10 m/s) (2 s) = 20 m.
Newton’s Laws 3.33

Attraction of the earth will produce an acceleration of g in the body. According to Newton’s third law the falling
g
body applies equal and opposite force on the earth. This will produce an acceleration of in the earth since mass
2
of the earth is twice that of the body.
3g
Relative acceleration of approach =
2
2H 4H
∴t = =
3g / 2 3g

F
Acceleration a =
Nm
Where m = mass of each cart.
Let, T1 = tension between 4th and 5th cart
Considering motion of last (N – 4) carts we get (N – 4) ma = T1 ….. (1)
Similarly, tension between 8th and 9th cart (T2) can be written considering the motion of last (N – 8) carts
(N – 8) ma = T2 .....….. (2)
As per the question T1 = 2T2
(N – 4) ma = 2 (N – 8) ma
N – 4 = 2N – 16
   N = 12
F
Tension in the last string T = ma =
N
5. Acceleration of the entire system is
F 40
a= =
m A + mB + mC + mD + mcart 2 + 2 + 1 + 1 + 4
Why does the cart accelerate? It is because of the force that D applies on it.
ND = mcart . a = 4 × 4 = 16 N
If m1 + m2 is accelerating down (m1 + m2)g – T1 = (m1 + m2)a
T1 = (m1 + m2) (g–a) And m2g –T2 = m2a
T2 = m2 (g – a) Q T1 > 3T2
(m1 + m2) (g – a) > 3 m2 (g – a)
m1 + m2 > 3 m2
m1 > 2 m2
The same result is obtained even if the system is moving in opposite direction.

The ratio of tension T1 & T2 is independent of mass m3.

T1
T1
m1

m1 g m3

T2
m3 g
m2

m2 g
3.34 Problems in Physics for JEE Advanced

Let the heavier mass be x.


The other mass = M – x
xg – T = xa …………(i)
T – (M – x) g = (M – x) a …………(ii)
Eliminating x between the two equations we get
T Ta  g a 
T – mg + = Ma – ⇒ T 1 + +  = Ma + Mg
(g – a) g–a  g – a g – a

∴T =
(
M g2 – a2 )
2g

T T
M-x x

a
(M-x)g xg

Mg
Smaller ‘a’ means higher T. For a = 0, T =
2
Which is higher than permissible limit.
If a increases above zero, T decreases.

 24 
T must decrease to  mg or below it .
 50 
24
T ≤ Mg
50

(
M g2 – a2 ) ≤ 24 Mg ⇒ 1 − a 2

24
2g 50 2 25
g
2
a 1 g
⇒ 2
≥ ⇒a≥
g 25 5

(iii) The load will travel through a distance h (rest to rest) in minimum time if it moves with maximum possible
acceleration (a1) and then retards with maximum possible retardation (a2). Obviously,
v

v0

t
t1 t2
Newton’s Laws 3.35

(h - 1)w
a1 = = (h - 1)g
M
And a2 = g when tension in the rope = 0.
The v – t graph for the situation has been shown in the figure.
v0 = a1t1 = a2t2
(h – 1) gt1 = gt2 (h – 1)t1 = t2
Area under v – t graph = h
1
∴ (t1 + t2) v0 = h
2
1
[t1 + (h – 1)t1] (h – 1) gt1 = h
2
2h 2h
fi t12 = fi t1 =
gh(h - 1) gh(h - 1)

tmin = t1 + t2 = t1 + (h– 1)t1 = ht1
2hh
=
g(h - 1)

Mass of A; mA = M
Mass of B; mB = M – m
Acceleration of B after the insect falls is
 m – mB 
aB =  A g(↑)
 m A + mB 

mg
=
2M – m
Acceleration of the insect = g (Ø) mg 2 Mg
The two objects separate with a relative acceleration of a = g + =
2M – m 2M – m
1
∴ at 2 = L
2
 Mg  2
  t = L
2 M – m
(2 M − m ) L
t=
Mg

8. (a) The friction force between the bead and the string causes the tension to change in the string on the two sides of
the bead.

T2

Mg T1
2
a T2 T1
a

Mg Mg
3.36 Problems in Physics for JEE Advanced

Mg
For the bead T2 = T1 + …..(1)
2
For the blocks T
2 – Mg = Ma …..(2)
Mg – T1 = Ma …..(3)
(2) + (3) T
2 – T1 = 2Ma
Mg g
Using (1) = 2 Ma ⇒ a =
2 4
(b) In this case acceleration of the bead is same as acceleration of the two blocks.
In above solution equation (1) changes to
Mg M
2 + T1 – T2 = 2 a

The other two equations remain same.


Solving the three equations we get
g
a =
5
9.

a
T T

m
M
mg
mg

For motion of the two masses we can write :



Mg – T = Ma…..(1)

T – mg = ma….. (2)
2 Mmg
Solving T=
M+m
Now we consider the equilibrium of the stand and pulley system

T T

M0 g

N = normal force between the stand and the scale = Reading of the scale N = 2T + Mog

Newton’s Laws 3.37

4 Mmg
N= + M0 g
M+m
10 Tension in string connecting m1 and m2 is
2m1m2
T= g
m1 + m2
m0g = 2T [for equilibrium of m0]

4 m1m2
m0 =
m1 + m2

For equilibrium of M
Mg = 4T

8m1m2
M=
m1 + m2

11. (a) Weight of the system = 9/2 Mg.


The support of the string S2 applies a downward force (T2)
9
Tension in S1 is Mg + T2
2
(b)

S1

T'1 T'1
B
D
T2 Mg T2
T1

C
2Mg

For pulley B, 2T2 + Mg = T1’ ...............(1)

For insect T1 ’=
M
g + T1 ...............(2)
2
For C T1 + T2 = 2 Mg ...............(3)

Mg
Solving T2 =
2

9
Tension in S1 = Mg + T2 = 5Mg
2
3.38 Problems in Physics for JEE Advanced

(c)
S1

T1
T'1
T'1
B

T2 Mg T2

C
2 Mg

The equations are


2T2 + Mg = T´1 …………(1)
M
T1´ + g = T1 …………(2)
2
T1 + T2 = 2Mg ................(3)
(1) + (2) + (3)
Mg
3T2 =
2
Mg
T2 =
6
a = g sin q
1
ax = g sin q cos q =
g sin 2q
2
g
(a x )max
2
When 2q = 90°
q = 45°
In reference frame of ground acceleration of snake is 8 m/s2 ( ) and that of the insect is 12 m/s2 ( ).
f2

f1

insect

mg

f1

Snake
Elevator
wall Mg

For insect
1– mg = m(12)
Newton’s Laws 3.39

[ 1= friction between insect and snake]


 50 
f1 =  kg × 22 = 1.1N
 1000 
For snake
ƒ2 – Mg – ƒ1 = M (8)
[ƒ2 = friction between snake and elevator]
ƒ2 = 4 × 18 + 1.1 = 73.1N

kVT2 = mg
V m1 1 1
T 1 = = =
VT 2 m2 4 2
h / 25 1
= [h = height from which bodies have been dropped]
h/t 2
t = 12.5 s

−1  2 r 
The thread makes an angle of sin   = 45° with the vertical.
 2r 

2r

2N
N

If N is normal force by each wall on the cylinder


T
2N =
2
T
And Mg =
2
T = 2Mg

Mg
And N =
2

17.
N1

N2 N1
Mg

N3

45° f
3.40 Problems in Physics for JEE Advanced

For Ball N1 = N 2 And N1 = Mg N1 = 2 Mg


2 2
For Wedge N 3 = N1 = Mg
2
Wedge will not move if

N1
fmax ≥
2

2 Mg m 1
mMg ≥
2
18. Considering the helicopter + Box as our system:
F – (M + m)g = (M + m)a

F = (M + m) (g + a)
= 17000 × 11.2 = 190400 N
For helicopter alone
F – Mg – T0 = Ma
T0 = F – Mg – Ma
= 190400 – 15000 × 11.2
= 22400 N
And T0 = 4T cos q  …(1)
T0

T q
T
T

2
 a
a2 –  
 2

T0 = 4T
a
T0 = 2 3T
22400
T 6467 N
2 3
If the strings are fixed at PQRS, value of cos q in equation (1) will decrease.
T will increase

T0 and F will not change
Newton’s Laws 3.41

q
T

T'
f

mg

T cos q = mg …...... (1)


and T sin q = T´ …...... (2)
[T´ = max when the block is about to slide]
When q increases, T will increase (equation.1)
From (2) T´ will increase and the block will slide.
(b) Before string is cut tension in wire is [equation … (1)]
mg
T1 = = 2 mg
cos 60∞

T2

mg sin  mg cos 
mg

After the string is cut, tension in wire is

mg T2 1

T2 = mg cos 60° = =
2 T1 4
20. Acceleration of the two blocks

mg sin 60° − mg sin 30°  3 − 1 


a=
=
 4 
g
3m  
If the two blocks move a distance x along respective inclines in time t
1 2
x= at
2
The centre of two blocks will be at equal height if

x sin 60° + x sin 30° = h
 3 1
x +  =h
 2 2

1  3 – 1 2  3 + 1

2  4 
gt   =h
 4 
3.42 Problems in Physics for JEE Advanced

16h 8h
t2 = =

( 3 –1 )( )
3 +1 g g

h
t=2 2
g
21. Block ‘C’ will not move as net horizontal force on it is zero.
F
T

T F

Horizontal forces on C have been shown in the fig.



Accelerations of A and B are same equal to

F
a=
2m
22. Hint: When m is maximum possible take limiting friction to be down the incline and when m is minimum take the
limiting friction to be up the incline.

2s 2l sec q
t= a
=
g(sin q - m cos q )

2l
=
g(sin q cos q - m cos2 q )

This is minimum when the z = sin q cos q – m cos2 q is maximum.

dz
= 0 gives q = 62.5°
dq

24.
N1 N2
A N1 N2 B
F C
RC RB
RA F
30°

mg mg mg

N1
For A: F = N1 sin 30° = … (1)
2
 mg  3
And RA = mg + N1 cos 30° = mg + 2  .
 2 3  2
3
= mg … (2)
2
N
For B: F = N2 sin 30° = 2
2
mg
N2 = 2F = …(3)
3
Newton’s Laws 3.43

mg 3 mg
And RB = mg – N2 cos 30° = mg – =
. ...........(4)
3 2 2
For C: N1 sin 30° = N2 sin 30°

mg [from (3)] ...........(5)


N1 = N2 =
3
And RC + N1 cos 30° = N2 cos 30° + mg
RC = mg ...........(6)
3 1
RA : RB : RC = : : 1 = 3 : 1 : 2
2 2
25. Mass of water added when the level of water rises by x (wrt container)
m = 2Sxd
The spring stretches by x so that the level of water does not change wrt ground.
Kx = 2Sxdg
K = 2Sdg
26. (a) Displacement (and hence speed) of each point on the spring will be proportional to its distance from the wall.
1 5
Speed of the particle. V = 5 × = cm / s
3 3
(b) The free end moves by 10 cm in 2.0 s
Tension in the spring = Kx = 0.6 × 10 = 6 N
Hint : The resultant of mg and the pseudo force ma must be along the diagonal or they must add to zero.
u2
x=
2m g
u2
For Car without ABS 20 =
2 m k .g
u2
For Car with ABS x=
2m s g
x mk
=
20 m s

x = 20 × 0.75 = 15 m
29. On flat road
1 2
S= at
2
1
S= m gt 2 [Q amax = mg]
2
1 2 Q
S= gt [m  =  1]
2
On inclined road
a´ = g (m cos q – sin q)
= g (cos q – sin q)
= g (1 – q) [for small angle cos q 1, sin q q]
3.44 Problems in Physics for JEE Advanced

1
S a´t´2
2

t
1 2 1  π 
gt = g(1 - q )t ¢ 2 t´ = p Q5º = 36 rad 
2 2 1-  
36
6t
=
36 – π
30. Time required to pull out the cloth is
L
t0 =
v0
If we observe the motion of the block in the reference frame of the cloth, it has initial velocity v0 ( ) and acceleration
mg ( ).
Note that friction between the block and the cloth is kinetic friction since the cloth moves at speed v0 but initial
velocity of the block is zero.

L
In the reference frame of the cloth the displacement of the block (taking left as positive) should exceed at or
before t0. 2

1 L
v0t0 – mgt02 ≥
2 2
2
1 Ê Lˆ L
L – mg Á ˜ ≥
2 Ë v0 ¯ 2

L 1 L2
≥ mg 2
2 2 v0

v02
≥m
gL
Block starts to move when F 6N
N

f
mg

If means max = 6N
When F = 18N, acceleration is 3ms–2.
F – max = ma
[Once the motion starts the friction remains constant at 6N]
18 – 6 = m × 3 m = 4 kg
In this case ms > mk
Block does not accelerate till F = 6 N.
ms N = 6 ms mg = 6N
Once F becomes slightly larger than 6N, the block has an acceleration a = 0.5 ms–2
Newton’s Laws 3.45

6N

fk
mg

6 – mk. mg = m. (0.5) …....(1)


When F = 18N
18 – mk mg = m (3) …....(2)
Subtracting equation – (1) from (2) gives 12 = (2.5) (m)
12
m 4.8 kg
2.5

Acceleration of the platform


 
dv
ap = = 2iˆ + ˆj + 3kˆ
dt
Horizontal and vertical acceleration

aH = 4 + 1 = 5 m/s2

av = 3 m/s2
Normal force on the block

N = m (g + av) = 1 × 13 = 13N
Maximum possible friction = mN
= mm (g + av)
Maximum acceleration that friction can provide
amax = m(g + av) = 0.2 × 13 = 2.6 m/s2

Q amax > aH

Block will not slide on the platform.

Value of friction force on the block
= maH = 5 N

Force by the platform on the block is

F = N 2 + f 2 = 169 + 5 = 174 N
Component of weight of 10 kg block down the incline is = 10 g sin 37° = 60 N
Maximum friction on the block is m10g cos 37° = 40 N
The block on the incline will not slide down until weight of the hanger + weight in it does not remain below
60 – 40 = 20 N. It means the additional mass in the hanger shall not be less than 1.5 kg.
The block on the incline will not slide up until weight of the hanger + weight in it does not exceed 60 + 40 = 100
N. It means the additional mass in the hanger should not exceed 9.5 kg.
35. For pulley 2F = ma
For particle F = ma’
Hence, a = 2a´ = 2 F/m
3.46 Problems in Physics for JEE Advanced

Acceleration of the point of application of the force is = 2a + a’ = 5a/2 = 5F/m


36. (a) The inclination of friction force with the tangent will go on increasing because the tangential acceleration is
constant but the radial acceleration goes on increasing
Normal force (i.e., normal acceleration) of the particle is constant.
v2
∴ = Const.
R
[v = speed
R = radius of curvature of the path]

R is increasing, hence v is increasing.

Component of acceleration along the tangent must be in the direction of velocity. Correct representation is at C.
Representation B means speed is decreasing and A means speed is not changing.
 dv
(c ) a = = 2 ˆj
dt
Rate of change of speed = tangential acceleration =

a.v 4t
=
v 16 + 4t 2
At t = 2s , tangential acceleration at = 2 m/s2

( 2)
2

an = a 2 – at2 = 22 – = 2 ms –2

(i) If value of mw2r (centrifugal force) + max is less than T (= Mg), block A will slide radially inward.


r T mr
fmax

Mg

m (w2r)min + max = Mg
2
(w r)min = 20 – 0.5 × 10 = 15N ..........…(1)
Also, if value of mw2r is higher then max + Mg, the block A will slide radially outward.
w

T
mw2r
fmax

Mg
Newton’s Laws 3.47

m (w r)max = Mg + max
(w2r)max = 20 + 5 = 25 N .........…(2)
If r = 1.0 m
w2min = 15   from (1)
And w2max = 25   from (2)
If r = 1.5m
15
w 2 min = = 10
1.5
25
w 2 max = = 16.67
1.5
Because r can be anywhere between 1.0 and 1.5 m, hence to be absolutely sure that A does not slip it is required
that
15 < w2 < 16.67
15 < ω < 16.67
(ii) Increase in length of the spring
1 2 × 3.14 × 20
∆l = (2πR) × = cm
100 100
= 1.26 cm
Tension in the spring T = k l
1.26
= 100 ×
= 1.26 N
100
If we consider a small element on the spring having angular width q, the centripetal force to its is provided by
the tension.
Dq
2T .sin = l ( R Dq )w 2 . R
2
For q 0
Dq
2T = l R 2 Dq .w 2
2
T 1.26
w2 = =
lR 2
0.126 ¥ 10 -3 ¥ (0.2)2

102
Þ fiw = = 500 rad s –1
0.2

T T
Dq

38. m1
r
F12 m2
r1 F21

r2


O
3.48 Problems in Physics for JEE Advanced


Let F12 = force on m1 due to m­2

F21 = force on m2 due to m1

d 2 r1 
m1 2 F12
dt

d 2 r2 
m2 2 F21
dt
2
 2
  
d r2 d r F F
2
– 21 = 21 – 12
dt dt m2 m1
 
(
d 2 r2 – r1 ) = F + F
21 21
 
Q F 12 = – F 21 
dt 2 m m  
2 1


d 2r  1 1  
∴ 2 = +  F 21
dt  m1 m2 

1 1 1
= +
m m1 m2

d 2
r 
\ m 2 = F21
dt
(a) Let mass of each block be m.
Acceleration after block 1 gets detached is
 3m – 2m 
a1 =  g
 3m + 2 m 
g
=
5
gt
Speed acquired in time t0 V1 = 0 + a1t0 = 0
5
At this moment block 2 gets detached and the masses on two sides become equal. The speed remains constant
for next interval t0
 2m – m  g
Then block 3 falls. Acceleration becomes a2 =   g=
 2m + m  3
Speed at time t0 after block 3 falls is
gt gt 8 gt0
V2 = V1 + a2 t0 = 0 + 0 =
5 3 15
Now block 4 falls and once again the acceleration becomes zero.
8gt0
final speed for 5 and 6 is V2 =
(b) 15
v

8
— gt0
15

g0
— t0
5

o t0 2 t0 3t0 t

Newton’s Laws 3.49

40. (a) Net force on B in downward direction = 20 N


Tension in string = 80 N
A will remain at rest and will feel his true weight.
(b) Hint: Let acceleration of A be ‘a’ in downward direction. Then acceleration of the rope where B is holding it
will be ‘a’. This means the acceleration of B is (4-a) in downward direction( rope is slipping from his hand).
Write equation of motion for both the monkeys and solve.
From Newton’s third law the particles exert equal and opposite force on one another.
mAaA = mBaB
m a 2 1
A = B = =
mB a A 6 3
(b) Till x = x0, A does not move and B moves uniformly.
The two particles will be nearest when both have same velocity. Let this happen at a time t1 after B reaches at
a separation x0 = 1.0 m to A.
A 2 m/s B
1.0 m
A V B
V
6 ms–2 –2
2 ms
Xmin

VA = VB
0 + 6t1 = 2 – 2t1
1
t1 s
4
1 3 –1
VA = VB = V (say) = 6 × = ms
4 2
Let displacement of A and B be XA and XB by time t1
2
1 1  1 3
X A = × 6 × t12 = × 6 ×   = m
2 2  4 16

2
1 2  1 7
X B = 2t1 – × 2 × t12 = –   =
2 4  4 16
Xmin = 1.0 + XA – XB = 0.75 m
(c) The particles continue to repel till distance between them becomes 1.0 m. Let this happen a time t after they
attain minimum separation.
3 1 2 3 1 2
 2 t + 2 × 6 × t  –  2 t + 2 × 2 × t  = 0.25
1
t=
8
Final velocities are
3 1  3 3  −1
VA ´ = + 6. = +  ms
2 8 2 2

3 1  3 1  −1
VB ´ = – 2. = –  ms
2 8 2 2
3.50 Problems in Physics for JEE Advanced

42. Time required to empty the bucket B


M
t0 =
m
Mass of B at time ‘t’
= 2M – mt
 m – mB 
Acceleration at time t is a =  A g
 m A + mB 
È2 M – (2 M – m t )˘˚ g

2 M + 2 M – mt
mt
= g
4 M – mt

If v is the speed of the buckets at time ‘t’



dv Ê m t ˆ
= g
dt ÁË 4 M – m t ˜¯

vo to
(4 M – mt ) dt + 4 Mg t o
dt
Ú dv = – Ú g

o o
4 M – mt Ú 4M – m
o

4 Mg
ÈÎln ( 4 M – Mt )˘˚ o
t
Vo = – gto –

m o

4 Mg
= – gto – Èln ( 4 M – m to ) - ln ( 4 M )˘˚
m Î
Q mto = M
4 Mg
\ V0 = – gto – Èln (3 M ) – l n ( 4 M )˘˚
m Î
4 Mg Ê 4 ˆ
= – gto + lnÁ ˜
m Ë 3¯

Mg È Ê 4ˆ ˘  M
=
Í 4ln Á ˜ – 1˙ Qto = 
m Î Ë 3¯ ˚  µ

43. Let the mass of the chain be m and its length be L


The mass of unit length of the chain= m/L
When a part of the chain of length x is hanging, the acceleration of the chain will equal to

a=
( m / L ) xg = gx
m L
Initially, when x = L/2 , the acceleration equals g/2, but then it increases.
After identical masses M are attached to the ends of the chain, then acceleration at the moment when a length x of
the chain is hanging will be
 ML 
gx  ( )
( m / L ) xg + Mg m +
a´ = = x 
 
m + 2M L  m + 2M 
 
Newton’s Laws 3.51

To decide whether a is larger or a’ let us assume


a > a’
 ML 
 (m) + x  ML
1>   ⇒ m + 2M > m +
 m + 2M  x
 
2x > L, which is true.
Hence, the chain without the balls will slip off faster.
44. Let the acceleration of wedge be ‘a’ down the incline.
Sum of masses of all rectangular blocks
= Mo = M + M + M + M + ...............
2 4 8
 1 1 1 
  = M 1 + + + + .............
 2 4 8 
1
=M = 2M
 1
 1 – 
  2

For finding a we can treat the system as shown below.


Acceleration of Mo will be a sin q ( )
Mo
q
N2
M N1

a
q
a Mg

For wedge N2 = (N1 + Mg) cos q ...............(1)


And (N1 + Mg) sin q = Ma ...............(2)
For Mo
N1

a sinq
M0g

Mo g – N1 = Mo a sin q

2Mg – N1 = 2 M a sin q--------------(3)
Multiply (3) with sin q and add to (2)
3Mg sin q = Ma (1 + 2 sin2 q)
3g sin q
\a =
1 + 2 sin 2 q
Acceleration of rectangular blocks
3g sin 2 q
= a sin q Ø =
() ()
1 + 2 sin 2 q
Ø
3.52 Problems in Physics for JEE Advanced

Mass of all rectangular blocks except 1 is M. Replace blocks 2, 3, 4, 5, ………… with a single block of mass M
Considering motion of this mass M,
Mg – N12 = Ma sin q

3 Mg sin 2 q
\ N12 = Mg –
1 + 2 sin 2 q

Mg ÈÎ1 – sin 2 q ˘˚ Mg cos2 q


= =
ÈÎ1 + 2 sin 2 q ˘˚ 1 + 2 sin 2 q

45. Acceleration of the system


1
m1g sin 30° − m2 g 4 × 10 × 2 − 1 × 10 2
a= = = 2 m/s
m1 + m2 4 +1
N12
a1 =
a
a2=a 60° M

1 a sinq

Mg
Acceleration of block 1 wrt particle 2 is
  
a12 = a1 – a 2

a12 = a 2 + a 2 + 2.a.a cos 60°


= 3 a = 2 3 m/s2
a1
30°

-a2

a12

a12 makes an angle of 30° with the vertical.
This means that to particle 2, block 1 appears to move in a direction making an angle of 30° to the vertical
(Along line AB)
A

300
c
d

dmin B
motion of 1
2 as observed by 2

Drop perpendicular from 2 on to the line AB. Length of this perpendicular is the desired minimum distance.
1
∴ dmin = d sin 30° = 2 × = 1.0 m
2
Time required to reach this position is calculated as
1
a t 2 = ( AC )
2 12
Newton’s Laws 3.53

1
× 2 3 × t 2 = 2 cos 30°
2
t2 = 1
t = 1.0 s.
46. (a) If we consider the horizontal equilibrium of any segment of the chain, one can easily show that horizontal
component of tension is same everywhere.
(b) Considering equilibrium of entire chain
TA
TA sin 60°

TA cos 60°

TB sin 30°
TB

TB Cos 30°
Mg y
x
TA cos 60° = TB cos 30° = (X component of tension everywhere)
TA = 3 TB ---------------- (1)
And TA sin 60° + TB sin 30° = Mg

3 T
TA + A = Mg [using (1)]
2 2 3

3Mg
∴ TA =
2

Mg
And TB
2
Considering equilibrium of segment AP
TA
TA sin 60°

60°
TA cos 60° A

mg

q P TP cos 15°
q
TP
TP sin 15°

Horizontal component of tension must be same everywhere


TP cos 15° = TA cos 60°

TP × 0.96 = 3 2
Mg [M = 4.8 kg, g = 10 m/s ]
4
3.54 Problems in Physics for JEE Advanced

TP = 21.65 N
(c) Considering vertical equilibrium of AP
mg + TP sin 15° = TA sin 60°
3
mg = Mg – 21.65 × sin 15°
4
3 21.65
m = × 4.8 − × 0.25
4 10
14.4 − 2.165
= = 3.05 kg
4

47. Fig. (a) shows component of tension (acting on A) along horizontal and vertical. Fig. (b) shows forces along the line
of greatest slope (XX’) and perpendicular to the incline. In equilibrium
= mg [considering block B] -------(1)
T
X

T sina
T T

2

A Q
T
T cosa 
2

X'
Fig (a)

Mg T  2 – 1
N= – = Mg  
2 2  2  ------------(2)
Forget about the friction for a moment.
T
T sin a =
2
T T
cos q =
2 2 X
T
N sin q= T
2 2

q=
in
M gs
Mg
q Mg cos q =
2
X'
Mg
Fig (b)
Unbalanced force along XX´ =
Mg T  2 – 1
– = Mg   ...............(3)
2 2  2 
T Mg
And there is a force along AQ = = ...............(4)
2 2
The friction will balance the resultant of the force given by (3) and (4)
Newton’s Laws 3.55

The resultant has been shown in fig. (c)


X

 2  1
Mg 
 2 
 

Mg
2
R
Fig (c)

2 2
 2 – 1  1 
∴ f = Mg   +  
 2  2

5–2 2
= Mg
2
mN

5–2 2 Ê 2 – 1ˆ
Mg £ m Á ˜ Mg
2 Ë 2 ¯

5–2 2
⇒ ≤µ
2 −1
49. Block will move up when tension in the string becomes larger than Mg.
Tension in string = Tension in cord AB
Mg
Mg = Kx, [x1= extension in AB] x1 =
K
x
If AB stretches by x1, the pulley moves up by 1 .
2
Tension in cord CD = twice the tension in string
Kx2 = 2Mg [x2 = extension in CD]

2 Mg
x2 =
K

End D moves up by a distance
x1
= x2 +
2
2 Mg Mg
= +
K 2K
5Mg
=
2K
3.56 Problems in Physics for JEE Advanced

50. Mass of A = 8 kg
Mass of B = 1 kg
F
Acceleration of (A + B); a 2 m/s2.
9 kg
We will solve the problem in the reference frame attached to the blocks (A + B). In this frame acceleration of C is
2.0 m/s2 ( )
Time required for C to travel a distance of 4.0 m is given as
1
4.0 =
× a × t12 ⇒ t1 = 2.0 sec
2
[Q a = 2 m/s2]
Velocity of C at this time is
u = at1 = 2 × 2 = 4 m/s ( )
Now, the block C falls off the edge of A. Its motion in our reference frame has horizontal acceleration of 2 m/s2 (
and a vertical acceleration of (g = 10 m/s2). Time needed to fall through a height of 2.0 m is given by
1
2.0 = gt22
2
t2 = 0.4 = 0.63 sec
Horizontal distance travelled in this time interval is
1 2
x = ut + at
2
1
= 4 × 0.63 = × 2 × (0.63)2 = 2.92 m
2
This is larger than width of block B. It means C will fall off the edge of A and land directly on the ground.
Time of fall can be directly calculated as
1
4.0 =
× g × t32 t3 = 0.8 = 0.9 sec
2

Total time needed to hit the ground is

t1 + t3 = 2 + 0.9 = 2.9 sec
51.

r q

2R

r + r + 2 r cos q = 2 R

R
\1 + cosq =
r
Newton’s Laws 3.57

9 4 3
cos q = - 1 = fi sin q =
5 5 5

N4
N6
N6
N3
N5

N5
N2 N1

Various contact forces are as shown.
Considering all 3 balls together
N1 = 3 mg ..............(1)
And N2 + N4 = N3 ..............(2)
Vertical equilibrium of lowest ball
N5 sin q + mg = N1
3
∴ N 5 . = 2 mg
5
10
N5 mg
3
Vertical equilibrium of top ball
N6. sin q = mg

5
⇒ N6 = mg
3

Horizontal equilibrium of top ball
5 4 4
N 4 = N 6 cosq = mg ¥ = mg

3 5 3
Horizontal equilibrium of middle ball

Ê 5 10 ˆ 4
N 3 = ( N 6 + N 5 ) cosq = Á + ˜ mg = 4mg

Ë3 3 ¯ 5

4 8
From (1) N 2 = 4mg – mg = mg
3 3
Hence N3 is largest and its value is 4 mg
4

N4 is smallest and its value is mg
3
52. Bead on wire AC can remain at rest relative to the wire even if the wire is smooth. This is not possible with the other
bead (It will become clear when we draw the force diagram). Let’s first study the equilibrium of bead on AC in the
reference frame of the wooden block.
3.58 Problems in Physics for JEE Advanced

N cos q
A
N

ma N sin q ma = pseudo force

mg q C

N sin q = ma
N cos q = mg
a
fi tan q =
fi a = g tan q ..............(1)
g

This must be the acceleration of the block. Now we will study the equilibrium of the other bead in reference frame
attached to the block.
N1 f

q
sin
g q
m s
co
g
m
q
mg

There are two other force perpendicular to the plane of the fig.
(1) Pseudo force ma that is perpendicularly into the plane of the fig.
(2) Another component of normal force N2 that is perpendicular to the plane of the fig.
(coming out towards you)
N1 = mg cos q
N2 = ma = mg tan q
And = mg sin q

But f £ m N12 + N 22

mg sin q
m mg cos2 q + tan 2 q

sin q £m
cos2 q + tan 2 q

53. (a) As per the problem, the string loses tension if platform goes up by2 cm.
It means the initial stretch in the spring was 4 cm (Why?)
At the instant block of mass m loses contact, the stretch in the spring is 8 cm.
k (8 cm) = mg
2 × 10 N
∴k = = 2.5 N/cm
8 cm
(b)
No
The block of mass m will remain in equilibrium. It will gradually move up so that the spring does not stretch
further.
Newton’s Laws 3.59

54. (a) Mass of A = mass of monkey.


Tension is actually the friction force between the hands of the monkey and the rope. Since forces on the monkey
and the block are identical hence they will have same acceleration
When monkey falls through ‘h’ the block also falls through ‘h’
Hence, the block is at height h from the floor at the moment the monkey hits the ground.
(b) To keep the monkey at rest, tension T = mg
Downward acceleration of the block is given by
2 Ma = 2 Mg – T
g
∴a = = 5 m/s2
2
Speed of the block at the instant it hits the ground is
V2 = 0 + 2a (2h)
  V = 2 × 5 × 2 × 2.5 = 5 2 m/s
(c) At the instant the block crosses the monkey its downward velocity is
Vo = 2 ah = gh = 5 m/s
After this the block must retard with a = 5 m/s2 so as to stop after moving through a further distance h.
Tension in the rope increases to T’ such that
g
T´ – 2 Mg = 2M 2
T´ = 3 Mg
Upward acceleration of the monkey will be given by
T´ – Mg = Ma
a = 2 g
g
Rope on the side of the monkey will have acceleration in downward direction.
2
Acceleration of the monkey relative to the rope is
g
= 2 g + = 5 ↑
2
g
2
()
= 25 m/s2 ( )
Let the top spring stretch by x.
Since net force on bar B1 must be zero (Q it has zero mass), hence the two spring connected below B1 will get stretched
x
by each.
2
x
Similarly, one can argue that the next layer of four springs will have extension of each and so on
4
(a) Displacement of top point A will be
x x x
x A = x + + + + ................
2 4 8
 
 1 
= x  
1 – 1 
 2
F
= 2x = 2
K
3.60 Problems in Physics for JEE Advanced

(b) Displacement of B1
F
XB1 = 2x – x = x =
K

a1

mg

2T
a2

2mg

Friction between the bead and the thread = tension in the string carrying the bead = T (say)
For bead
mg – T = ma1 .................(1)
For block
2 mg – 2T = 2 m a2
mg – T = ma2 .................(2)
From (1) and (2)
a1 = a2 = a (say)
The end of the string below the bead will move up with acceleration of 2a as the block goes down with acceleration
of a.
Hence relative acceleration of the bead and the end of the string = 3a
1
\l =
(3a ) t 2 .................(3)
2
The block falls through
1 2 l
s = at = [from (3)]
2 3
59.

T1
T1
T1 P2

A 2T1

P1
mg
T2 T2
B

mg
Newton’s Laws 3.61

(a) For equilibrium


T1 = Mg
And T2 = mg
But 2T2 = 2T1 (for equilibrium of pulley P1)
m = M
(b) When m = 2M, let acceleration of A be a (-).

a
Acceleration of pulley P2 will be
2
(↓)
a
Acceleration of P1 = acceleration of P2 = ↓ .
2
( )

Acceleration of B = 2 × acceleration of P2 = a ( ).
For pulley P1; 2T1 = 2T2 T1 = T2 ...............(1)
For A; T1 – Mg = Ma ...............(2)
For B; 2Mg – T2 = 2Ma ...............(3)
Solving (1), (2) and (3)
g
a =
3

60. (a)
F
N
F

q
sin
mg

q mg cos q

For block to accelerate up the incline
F > mg sin q
For block to remain in contact
F < mg cos q
This is possible only if q < 45°
p
\ q o = 45∞ =
4

qo p
(b) If \ q o = = , maximum allowed value of F
2 8
so that block does not lose contact is
 π
F = mg cos  
 8

π π
∴ mamax = mg cos   − mg sin  
8 8
3.62 Problems in Physics for JEE Advanced

 π  π 
∴ amax = g cos   − sin   
 8  8 

61. When A is fixed


1
S= ( g sinq ) t 2   [S = length of incline] ----- (1)
2


When A is free to move
In reference frame attached to A. The acceleration of B will be given by

mg
ma = mg sin q +
cos q
4

g
m—
4

a
mg q


1 Ê 1 ˆ 2
\S = g Á sin q + cos q ˜ t´
2 Ë 4 ¯
2
1 Ê 1 ˆÊ tˆ t
= g Á sin q + cos q ˜ Á ˜ [Q t´ = ] -----(2)
2 Ë 4 ¯ Ë 2¯ 2

From (1) and (2)

Ê 1 ˆ1
sin q = Á sin q + cos q ˜
Ë 4 ¯4

1
3 sin q = cos q
4
1
tanq =
12

62. Let the horizontal and vertical directions be X and Y directions respectively.

a A = ao cos q î + ao sin q ĵ

a B = ao sin q ĵ (since length of string is constant)

fi a AB = ao cos q î
Newton’s Laws 3.63

63.
T
T B
f2
f2
A

f3
a f1 = 0.55010 = 250N

For block B to move slowly T = 1 = 250 N.


Maximum possible friction on shoes of the man
Sol.63
2 max = 0.6 × 500 = 300N
For man to remain at rest relative to the ground
2 = T = 250 N
Acceleration of platform A is
30. a = 2 – 3
30. a = 250 – 0.2 × 800
a = 3 m/s2
Man will fall off the platform once he moves 5m.
Hence, time required is given by

1 2
at = 5
2
1
× 3 × t2 = 5
2

10
t s
3
64. (a) In this case the mass (M + m) will slide down the incline with acceleration
a = g sin q
(b) The horizontal and vertical components of the above acceleration are
ax = g sin q. cos q
ay = g sin q sin q
ax

q
in ay
gs q

y
For motion of block A
N
ax A

ay f
mg
3.64 Problems in Physics for JEE Advanced

= max
= mg sin q . cos q
= 1 mg sin 2θ
2
And mg – N = may
N = mg – mg sin2 q = mg. cos2 q
(c) In case of no friction, the wedge slides down the incline with acceleration a and the small block will move
vertically down with acceleration a sin q.
N N0 N N0
A
s i nq
N)
mg a Mg g+
q (M q (Mg + N) cos q
a sin q

For Block: mg – N = ma sin q ….. (1)


For Wedge: No = (Mg + N) cos q
and (Mg + N) sin q = Ma ….. (2)
Solving (1) and (2)
( M + m)g sin q
a=
M + m sin 2 q
acceleration of the block = a sin q

=
( M + m ) g sin 2 q
M + m sin q
2

Now, for displacement of the block


1 Ê ( M + m ) g sin q ˆ 2
2

h= t
2 ÁË M + m sin 2 q ˜¯

(
2h M + m sin 2 q )
t =
( M + m ) g sin 2
q

65. (a) Immediately after release none of the blocks have any speed.

a A is towards ( )

a B is
 
A .a B = a A .aB .cos 90° = 0
∴ a
(b) Immediately after release, the constraint that length of string is constant means that
 
a A = a B = a(say)
For B:
Mg – T = Ma ….. (1) [Normal force between the blocks is zero]
For A:
T = Ma …..(2)
(1) + (2) gives
a = g/2
Newton’s Laws 3.65

(a) The block will slide down with an acceleration a = g sin q


For the ball to have same acceleration, the thread shall be perpendicular to the incline. Tension will balance
mg cos q and the ball will have a resultant force of mg sin q down the incline.
It means that the thread makes an angle q to the vertical.
1
(b) The acceleration of the block is a = g sinq . In reference frame of the block there is a pseudo force (= ma) on
the ball. 2

a
ma
a T
q

mg q

Taking equilibrium in horizontal and vertical direction –


T sin a = ma cos q …..(i)
T cos a + ma sin q = mg
T cos a = mg – ma sin q ….. (ii)
Taking ratio of (i) and (ii)
1
g sin q .cos q
a cos q
tan a = =2
g - a sin q 1
g - g sin 2 q
2
sin q .cos q
tan a =
2 - sin 2 q

-1 Ê sin q .cos q ˆ
a = tan ÁË ˜
2 - sin 2 q ¯

67. (a) Let acceleration of wedge be ao.


In the reference frame attached to the wedge, the block is at rest.
N cos q

N sin q ma0

q mg

N sin q = mao ….. (1)


N cos q = mg ….. (2)
ao
(1) ∏ (2) gives tanq =
g
\ ao = g tan q = g tan 45° = g
\ Force on wedge Fo = 2m.g
3.66 Problems in Physics for JEE Advanced

(b)
F = 1.5 × 2 mg = 3 mg
Let ao = acceleration of wedge
a = acceleration of block relative to wedge (up along the incline)
sq
a
co
N ma 0

ma0

q
sin q
mg sin
ma 0
q mg

mg
co
sq
N = mg cos q + mao sin q ….....(1)
ma = mao cos q – mg sin q
a = ao cos q – g sin q ….....(2)
For wedge
N sin q

N cos q N
a0
R 3 mg

mg

3mg – N sin q = mao


\ N sin q = 3mg – mao….. (3)
Eliminating N between (1) and (3) and writing
1
sin 45° = cos 45° = , we get
2
5
ao = g
3
Putting in (2), we get

2g
a
3
L
The block will fall off the wedge after moving through a distance up the incline.
2
L 1 2
\ = at
2 2
2 3L
⇒ t = 2g

3L
t = 2g
Newton’s Laws 3.67

68. Figure shows the free body diagram of the sleeve in a reference frame attached to the rod when a is small, the rod
has a tendency to slide down, hence friction is up the rod.
N
µN

ma cos q

ma

q
sin
mg

ma
mg
mg

sin
co

q
sq
For the minimum value of a for which the sleeve does not slide, friction will take its maximum possible value,
i.e., mN
N = m (a sin q + g cos q)

and mN + ma cos q = mg sin q
m
m (a sin q + g cos q) + ma cos q = mg sin q

a=g
(sin q - m cosq )

(cosq + m sin q )
This is the minimum value of a for which the sleeve does not slide.

ma cos q

ma
µN

q
sin
mg
ma
mg

sin

mg
co

q
sq

When a increases the sleeve has a tendency to move up.


Thus friction is directed down the rod.
a is maximum when friction is mN
N = m (a sin q + g cos q)
and mg sin q + mN = ma cos q
or
mg sin q + mm (a sin q + g cos q) = ma cos q
g(sin q + m cos q )
or a = cos q - m sin q

(sin q - m cosq ) £ a £ g (sin q + m cosq )


g
(cosq + m sin q ) cos q - m sin q
3.68 Problems in Physics for JEE Advanced

69. Let ao = acceleration of B (towards right)


a = acceleration of A wrt B
FBD of C in the reference frame attached to B is shown in figure
µN1

C
N1
moao

mog

M
mo =
= mass of C.
4

N1 = moao ….. (1)
and mN1 = mog …..(2)
g
or ao = ….. (3)
m
FBD of A is shown
N2

ma0 sin q A
a ma0

sq q
co os
ma 0 c
mg
q mg
sin
mg

N2 = mg cos q – ma sin q
……(4)
and ma = mg sin q + mao cos q
or a = g sin q + ao cos q ……(5)

FBD of B is also shown
N2 sin q
R
a0
N2
N2 cos q N1
q

mg

N2 sin q – N1 = Mao……(6)

Using (i)
N2 sin q = (M + mo)ao

Eliminating N2 using (4) we get
mg sin 2q
ao =

(
2 M + mo + m sin 2 q )
g mg sin 2q
=
From (3)
(
m 2 M + mo + m sin 2 q )
Newton’s Laws 3.69

or 2M + 2mo × 2m sin2 q = mm sin 2q

or 2 M + M + 2m sin 2 q = 3 m sin 2q
2 4
5 3 
or M = m  sin 60° − 2 sin 2 30° 
2 4 

 
or 5 M = m 3 3 – 2
2  
 4 2 4

 
or 5 M = m 3 3 – 1
2  2 
 8
m 20 20
or = = = 16.7
M 3 3 – 4 1.196

70. (a) Equilibrium


N
T2

T2
mg T1
a T1
mog

Mg

T1 = Mg and T
2 = mog
For ring:
T2 = (T1 + mg) sin a
mog = (M + m)g sin a
mo = (M + m) sin a
(b) Relation between a1 and a2 is
N

a1

mg T
a
T
a2

Mg

a1 sin a = a2 …....... (1)


For ring
(T + mg) sin a = ma1 …....... (2)
For sphere
Mg – T = ma2 …....... (3)
3.70 Problems in Physics for JEE Advanced

Solving (1), (2) and (3)

=
( M + m ) g sin 2 a
a2
m + M sin 2 a

71.
T a
T q T N
T
M N
a
Mg N0
q q
mg
m0g

For M: Mg – T = Ma …....... (1)


For m: T – mg sin q = ma …....... (2)
and N = mg cos q …....... (3)
For mo: T + T cos q = N sin q …....... (4)
[In horizontal direction]
Eliminating a between (1) and (2)

T T

g– = – g sin q
M m

Ê1 1ˆ
T Á + ˜ = g (1 + sinq )

Ë m M¯

mMg
T=
(1 + sinq )
m+M
Put this value of T and value of N from equation (3) into equation (4)

mMg (1 + sin q )
(1 + cosq ) = mg cosq .sin q
m+M
4 3
.
M cos q .sin q 5 5 1
\ = = =
m + M (1 + sin q ) (1 + cos q ) Ê 3 ˆ Ê 4 ˆ 6
ÁË 1 + ˜¯ ÁË 1 + ˜¯
5 5

M 1
=
m 5
72.
T
a
T N
N T
ma

T
0
co
s
q

am ma0 a0
Mg = mg
m

m
a0

g
si

mg cos q
sin
n
q

q
Newton’s Laws 3.71

Let acceleration of m wrt wedge be a () and that of the wedge be as ao ( ).


Acceleration of M is aM ( ) = a + ao …(1)
For M: mg – T = maM …(2)
For m in reference frame of wedge
3 4
T – mg + mao . = ma
… (3)
5 5
4 3
and N = mg. + mao … (4)
5 5
For wedge in horizontal direction
4 3
T + T . − N . = 2m.ao … (5)
5 5

(2) + (2) 2 mg + 4 mao = m ( a + ao ) + ma


5 5
2g + 4ao = 10a + 5ao
10a + ao = 2g … (A)

Eliminating N between (4) and (5) gives.


45T – 12 mg = 59 mao
Substituting for T from (2)
45 (mg – maM) – 12mg = 59mao
45g – 45(a + ao) – 12g = 59ao
45a + 104 ao = 33 g … (B)
Solving (A) and (B)
48g
ao =
199
Let ao = acceleration of 4 m towards right

a = acceleration of 5 m and 8 m wrt the block of mass 4 m.
We will consider motion of block of mass 8m and 5m in the reference frame attached to the block of mass 4 m.

T 5 mao
a
8 mao

N a T
8 mg 5 mg
37°

For 4 m: 8 mg – T = 8 ma …....(1)
and N = 8 mao …....(2)
For 5 m:
Perpendicular to incline
3 4
N´ + 5mao 5mg.
5 5
N’ = 4 mg – 3 mao ….... (3)
Down the incline
3.72 Problems in Physics for JEE Advanced

4 3
T + 5mao . + 5mg. = 5ma
5 5

T = 5 ma – 4 mao – 3 mg ….... (4)
For block of mass 4 m:
T
T
T N

ao

We will write equation of motion in horizontal direction. Relevant forces are shown in figure.
4 3
T. N´ − N = 4 mao
5 5

4T + 3N´ – 5N = 20 mao ….... (5)


Substituting for T, N and N´ using equations (1), (2) and (3)
4[8 mg – 8 ma] + 3 [4 mg – 3 mao] – 5 [8 mao] = 20 mao
69 ao + 32a = 44 g ….... (6)
Eliminating T between (1) and (4)
13 a – 4ao = 11 g ….... (7)
Solving (6) and (7)

44 g
ao =
205
74. Initial extension in S1 be xo
kxo = Mg
Let extension in S2 be x2 and further extension in S1 be x1 as point A moves down by L.
2x1 + x2 = L      …… (1)
k(xo + x1)

Mg 2kx2

For equilibrium of M

kxo + kx1 = Mg + 2kx2
But kxo = Mg
x1 = 2x2        ……(2)
Solving (1) and (2)
L
x2 =
5
2L
x1 =
5
Newton’s Laws 3.73

In equilibrium, tension in spring 1 is T1 = 3mg and tension in spring 2 is T2 = mg.


(a) When spring 1 is cut, there will be no immediate change in tension in spring 2.
ac = 0
A

mg
a
T

mg

T2 = mg

Let a be acceleration of A and B


2ma = T2 + mg + mg
3
a= g ↓ ( )
2
d 2 x2 3
2
= aC - aB = 0 - g Ø
2
()
dt
2
d x2 = 3g
dt 2 2
(b) When AB is cut, tension in both springs remain unchanged.
For A
T1 = 3 mg
aA

=
mg
maA = 3mg – mg
aA = 2g ( )
For B
B aB

mg

T2 = mg

maB = mg + mg
aB = 2g ( )
For C
aC = 0
d 2 x1
\ = aA = 2g
dt 2
d 2 x2
    = aC - aB = 2 g
dt 2
3.74 Problems in Physics for JEE Advanced

(c) When spring 2 is cut, (A + B) together move up.


2ma = 3mg – mg – mg

a =
g
2
()

aC = g ( )

d 2 x1 g
∴ 2
= aA =
dx 2

2
d x2 = a – a = 3g
2 A C
dt 2
76. Let stretch in 1st spring be x1
stretch in 2nd spring be x2

x1 + x2
xo =
…... (1)
2
Also k1x1 = k2x2 …... (2)
Solving (1) and (2)
 k2 
x1 = 
( 2 x0 )
 k1 + k2 
Tension in springs
2k1k2 x0
T = k1 x1 =

k1 + k2
(a)
Ma = 2T
4k1k2 x0
a=
( k1 + k2 ) M
4k1k2 x0
T0 = 2T =
(b)
k1 + k2

77. Let’s consider


Equilibrium of block A and B
N1 Fs

N2
A

B
N2 cos q
q
sin
Mg N2 Mg cos q
q Mg
N2 sin q 2 Mg

For A:
Mg sin q + N2 sin q = 2 Mg

Mg N 2
+ = 2 Mg
2 2
Newton’s Laws 3.75

N2 = Mg
For B:
FS + N2 = 2Mg
FS = Mg
Immediately, after the string is cut, tension in the spring (FS) will not change.
Let acceleration of A be ‘a’ down the incline.

a
Acceleration of B will be a sinq = in vertically downward direction.
2
We will use the same force diagram as drawn above (Now T = 0). For motion of A.
(N2 + Mg) sin q = Ma
N2 + Mg = 2 Ma …...... (1)
For motion of B
2 Mg – N2 = 2M . a sin q
2Mg – N2 = 2 2 Ma …...... (2)
From (1) and (2)
N2 = 0
g
and a =
2
Let a0 = acceleration of the wedge
a = acceleration of the block relative to the wedge. We consider motion of the block in reference frame attached to
the wedge
N

ma0 sin q

mao
ma0 cos q
mg sin q
a
mg cos q q
mg

N + ma0 sin q = mg cos q ..........(1)


ma0 cos q + mg sin q = ma
a0 cos q + g sin q = a ..........(2)
For motion of the wedge

N sin q

a0 N
N cos q
R
q

Mg
N sin q = Ma0 .......... (3)
Eliminating N between (1) and (3)
3.76 Problems in Physics for JEE Advanced

mg sin q cos q
a0 =

( M + m sin q )
2

Putting this in (2) gives

a=
( M + m ) g sin q

( M + m sin q )
2

a M+m
=
a0 m

For the block to remain on the table, it is required that by the time displacement of the block relative to the wedge
L
becomes equal to ‘S’, the horizontal displacement of the wedge must have become larger than .
2

L
––
2

1 2
at = S …........ (1)
2
1 2 L
and a0 t ≥ …........ (2)
2 2
For limiting case let’s take the ratio [ (1) (2)]
a 2S
=
a0 L

M + m 2S
=
m L
M 2  1
+1 = =4 cos 60° = 2 
m cos θ  

M
∴ =3
m

79. Let a = acceleration of m relative to the wedge A


a1= acceleration of wedge A towards right
a2 = acceleration of Block B towards left
A careful observation will tell you that
a = a1 + a2 …........(1)

Consider motion of m in reference frame of wedge A
Newton’s Laws 3.77

q
sin N
ma 1

ma1
a
sq
co q mg cos q
ma
1 sin
mg
q mg

ma1 sin q + N = mg cos q


…........ (2)
ma1 cos q + mg sin q – T = ma …........ (3)

Consider motion of wedge A
We will write equation for horizontal motion only
T cos q
T

N sin q T

N
a1

q R

Mg

T + N sin q – T cos q = Ma1 ...........(4)


For motion of Block B

a2 R´

Mog

The equation for horizontal motion is


T = M0a2 …........(5)
First let’s eliminate N between 2 and 4
ma1 sin2 q – T (1 – cos q) = mg cos q.sin q – Ma1
a1 [m sin2 q + M] – T (1 – cos q) = mg sin q cos q

3 4
Put sin q = ;cos q = and M = 4 m
5 5

9  T 12
ma1  + 4  – = mg
 25  5 25
109 ma1 – 5 T = 12 mg …........ (6)
Put value of T from (5) into (3) & (6)
ma1 cos q + mg sin q – M0a2 = ma
3.78 Problems in Physics for JEE Advanced

4 3
a1 + g – 2a2 = a
5 5
4 a1 –10 a2 = 5 a – 3 g …........ (7)
And 109 a1 – 10 a2 = 12 g …........ (8)
Solving (1), (7) and (8)
4a1 – 10 a2 = 5 (a1 + a2) – 3 g
a1 + 15 a2 = 3 g
Multiply this equation with (2) and add it to equation (8) multiplied by 3
6g
a1 =
47
80. If friction coefficient is m.
F0 = mmg
N N

45°

Mg

When the block is in the trough



N cos 45° + N cos 45° = Mg
2 N = Mg

Mg
N=
2

Friction force on the block is


= mN + mN [for two contact surfaces]
= 2 m Mg
Required force F = 2 m Mg = 2 F0
81. Block A moved up along the incline with acceleration aA such that

1
X A = 0 + aAt 2
2
1
2= a A × 22
2

aA = 1.0 m/s2
Force equation for block A

N F

q
in
gs mAg cos q
m A

fA
Newton’s Laws 3.79

F – mAg sin q – mAmAg cos q = mA.aA


3 4
2.2 - 0.2 ¥ 10 ¥
- m A ¥ 0.2 ¥ 10 ¥ = 0.2 ¥ 1
5 5
mA = 0.5
For B

1
X B = 0 + aB t 2
2

1
–2 = ¥ aB ¥ 22
2
aB = – 1.0 ms–2

Force equation for B


F fB
2
/s
m
.0
=1
aB

q
in
gs
m B

mBg sin q – F – mB. mBg cos q = mBaB



0.5×10 × 0.6 – 2.2 – mB × 0.5 × 10 × 0.8 = 0.5 × 1

3
m B = = 0.075
40
With both blocks together

fA
q
a sin B
g
mA

q fB
sin A
g
m B

(mB + mA)g sin q – mAmAg cos q – mBmB cos q = (mA + mB)a

3
0.7 × 10 × 0.6 – 0.5 × 0.2 × 10 × 0.8 – × 0.5 × 10 × 0.8 = 0.7a
40
4.2 – 0.8 – 0.3 = 0.7a
a = 4.43 m/s2
3.80 Problems in Physics for JEE Advanced

Considering only A

fA

q
sin
g T a
m A

mAg sin q + T – A = mAa


1.2 + T – 0.8 = 0.2 × 4.43
T = 0.49 N
82. Let coefficient of friction between the blocks be m.
F1 = 4 m (mg)
mg is the maximum acceleration that friction can impart to block B.
mg
Similarly, when force is applied to B, friction can impart a maximum acceleration of to the block A.
3
Ê mg ˆ
F2 = 4 m Á ˜
Ë 3 ¯

(a) With both F1 and F2 acting, friction will adjust itself to zero, as the two blocks will have equal accelerations

4 F F
(b) a = mg = 1 = 2
3 3m m
83. (i) Consider the case when the spring is compressed and the block is in equilibrium.
kx = mg +
kx

40 N N

mg f

(kx)max = mg + max
400 xmax = 20 + 0.5 × 40
xmax = 0.1 m
In this position h = 0.9 m
Above this height block can be kept anywhere.
If compression in the spring is more than 0.1 m, the block will slide up.
(ii)
F cos
f

N  F sin

Mg
Newton’s Laws 3.81

N = F cos q
F sin q + Mg = ƒ
But ƒ mN
F sin q + Mg m F cos q
Mg < F (m cos q – sin q) …........(1)
0.5 < m cos q – sin q

0.5 < m 2 + 1 sin (a - q ) [where a = tan–1 m = tan–1 0.75 = 37°]


0.5 < 1.25 sin (a – q)
0.4 < sin (a – q)
sin 24° sin (a – q)
24° < 37° – q q 13°
(b) From equation (1), F will have a finite positive value only if
m cos q – sin q 0
m > tan q
0.75 > tan q
37° > q
84. Retardation while going up will be a1 = g sin q + mg cos q = x + y [ say]
Acceleration while coming down a2 = g sin q – mg cos q = x – y
C

a2
a1
s B
m/ /s
10 4m

5m
A

Let time for up journey = t1


Time for down journey (C to B) = t2

AC = S1 and CB = S2
VC = VA – a1t1 0 = 10 – (x + y)t1

10
=t
x+y 1
And VB = VC + a2t2 x – y t2

4
=t
x–y 2
Given t1 + t2 = 4
10 4
\ + = 4 …........(1)
x+y x–y
10
VC2 = VA2 – 2a1S1 = S1
2 ( x + y)
3.82 Problems in Physics for JEE Advanced

Similarly, 42
= S2
2 ( x – y)
Given S1 – S2 = 5m

50 8
– = 5 …(2)
x+y x–y

1 1
Let = a and =b
x+y x–y

Equation (1) and (2) can be written as


10a + 4b = 4 5a + 2b = 2
And 50a – 8b = 5
Solving these two equations we get

13 15
a=
and b =
70 28
13x + 13y = 70
And 15x – 15y = 28
Solving these two equations we get
x = 3.62 and y = 1.76
10 13 13
t1 =
(a) = 10 ⋅ a = 10 ⋅ = s
x+ y 70 7
(b) x = 3.62 g sin q = 3.62 sin q = 0.362

cos q = 1 - (0.362 ) = 0.94


2

y = 1.76 mg cos q = 1.76


1.76
\ m= = 0.18
10 ¥ 0.94

85. We will observe the motion of the block in the reference frame attached to the moving paper.

In this frame, the initial velocity of the block is in a direction making an angle
q with Y direction, as shown. The block will travel in this direction with friction force exactly opposite to velocity.
y

v L
q
x
u
f

We will consider the motion of the block in y direction only.


Initial velocity uy = v
y component of friction = mg. cos q [along – ve y]
\ y component of acceleration = mg cos q
Newton’s Laws 3.83

[along – ve y]

The maximum v can be obtained by assuming that the y component of velocity of the block becomes zero just when
the block travels through a distance L in y direction.
v2y = u2y – 2ay .L


2 m gL
v=
u2 + v2
v2 (u2 + v2) = 4m2g2L2
v4 + u2. v2– 4m2g2L2 = 0

Minimum force (Fmin) is needed when the force and limiting friction (acting to left) together balance the spring
force (kx).

Fmin Fmin

kx kx

fmax fmax
Fmin + max = kx
8+ max = kx .............(1)
Maximum force that can be applied without moving the block can be calculated as
Fmax = max + kx
18 = max + kx .............(2)
The block will begin to move towards left if F is increased beyond this value.
Solving (1) and (2)
kx = 13N
k (0.1) = 13 k = 130 N/m
max = 5N
mmg = 5
m × 1 × 10 = 5
m = 0.5
87. When the plank is horizontal
N
a1
mai

m mg

B A

mg
3.84 Problems in Physics for JEE Advanced

Let acceleration of block in reference frame of train be a1.


ma1 = ma0 – mmg
a1 = a0 – mg .............(1)
When the plank is inclined, let acceleration in reference frame of train be a2.
a2
N
B

ma0

mN

45°
mg A

and

..........(2)

Let time to travel from A to B in horizontal position = t1
Time in inclined position = t2
25/4 t1= t2

4 2 a2 = a1 ..........(3)
Substituting for a1 and a2 from (1) and (2) in equation (3)
4 [(a0 – g) – m (a0 + g)] = a0 – mg

88.
N1
A
f1
N2
f2
B
R Mg
r

O1 O2

Figure (a)
Newton’s Laws 3.85

A
(R–r )
q B
C

O1 O2
R+r

Figure (b)
In figure (b)

.............. (1)

The equilibrium of rod gives


N1 + N2 = Mg cos q .............. (2)
and m(N1 + N2) = mg sin q .............. (3)
Assuming the friction to be at its limiting value.
(3) ∏ (2) gives
tan q = m

Put in (1)

If the ratio h is decreased, q will increase and the rod will begin to slide.
89.
A N2
mg f2
N1
f1
f1 N1
F N3 f3
B


For A:
N2 = 1max
N2 = mN1 ….......(1)
and N1 = 2max + Mg
N1 = mN2 + Mg
3.86 Problems in Physics for JEE Advanced

N1 = m2N1 + Mg

….......(2)

For B:
N3 cos q = sin q + N1

….......(3)

and F = 1 + 3 cos q + N3 sin q


= mN1 + mN3 cos q + N3 sin q
Substituting for N3 from equation (3) we get

F=

90.
f
fm

37°
in
gs
60 0 N
6
=3

Platform

fm

40g sin 37°


= 240 N

Man

Normal reaction by the incline on the platform is


N = 100 g cos 37° = 800 N
Maximum possible friction force on the platform (by incline) is
max = mN = 400 N
The maximum friction by the man on the platform in downward direction can be m = 40 N. This means that the man
is experiencing friction force of 40 N up the plane.

Acceleration of man in this case m/s2

If m is less than this value man will have higher acceleration.

If m = 0; acceleration of man a = 6 m/s2

If acceleration of man is larger than 6 m/s2, the friction m reverses direction.


2
When a = 7 m/s
m = 40 N
Newton’s Laws 3.87

Friction on platform by incline will adjust itself to 320 N.


91. Consider the case when m moves up the incline with acceleration ‘a’.
a
T
N

f
mg
q


a
T
F


Mg

For m

T – mg sin 37° – mN = ma

…..(1)

For M

F – T – ƒ´ = Ma [put m = ]

- - = …..(2)

(1) + (2)

For a > 0

m<2
Similarly consider the case when m accelerates down the incline.
T
N
f

mg
3.88 Problems in Physics for JEE Advanced
a

T
F

Mg

mg sin 37° – T – m.mg cos 37° = ma

and T – 40 – mMg = Ma

….. (4)

(3) + (4)

for a > 0

m > 30 kg

No motion happens when
2 kg m 30 kg
92.

T2
A T1
fmax = µ mA g
P1
= 40 N B
T2 mB g sin 
= 24 N
P2
q
T3
C
mg

(a) For aB = 0, T1 = 24 N
T2 = 48 N and T3 = 96 N

m/s2

If B does not move, P1 does not move.

Acceleration of C,

M (9.5) = 96
Newton’s Laws 3.89


(b) If A does not move
T2max = 40 N
T1 = 20 N and T3 = 80 N
With T1 = 20N, B will accelerate down the incline.

m/s2 

acceleration of m/s2

acceleration of m/s2

For C
T3 – Mg = M.

93. If mA > mB, the direction of forces on the two blocks is as shown
T
f
T

mA g sin 

f
mB g sin 

There will be no acceleration if


mAg sin q < mBg sin q + +
With friction at its peak value we get
(mA – mB)g sin q < 2 mmAg cos q

If mA < mB
T

f T

mA g sin q
f

mB g sin 

q
3.90 Problems in Physics for JEE Advanced

There will be no acceleration if


mBg sin q < mAg sin q + 2

94. (i)
2 kg T 2T
f1

f1 M
3 kg T
Mg
f2

1max = 0.75 × 20 = 15 N
2max = 0.5 × 50 = 25 N
When 2 T > 25 N the system will move
For no motion 2 T 25 N
Mg 25 N
M 2.5 kg
When M = 2.5 kg
1 = T = 12.5N
(ii) If we assume that 2 kg and 3 kg block are moving together, then
2 T – 2max = 5 a .......…(i)
and 4 g – 2 T = 4 a .......… (ii)
Solving (i) and (ii) m/s2

And =

Considering only 2 kg block


T– 1=2×a

= - = =

This value of 1 is possible (as 1max = 15N).


Hence, our assumption that both the 2 kg and 3 kg blocks are moving together is correct.

(iii) 1 will still be

acceleration of M will also be same i.e.,


m/s2

(iv) 2max = 0.9 × 50 = 45 N


The 3 kg block can experience a maximum possible rightward force
= Tmax + max = 20 + 15 = 35 N
But friction 2 can go upto 45 N.
Newton’s Laws 3.91

Hence, 3 kg block will not move.


Let acceleration of M be a, then acceleration of 2 kg block will be 2a.
40 – 2T = 4.a …(1)
T – 15 = 2.(2a) …(2)
Solving, a = 5/6 m/s2
95. Let’s calculate the tension in the string when no block is attached to the rope.
A T + dT

dq
T
q
C


dT = (Rdq)g cos q

TA – TC = Rg
TA – Rg = Rg
TA = 2 Rg

96. Consider a small segment of length dl on the rope.

dl
dy
q

dx

The tangential component of weight


= dl g. sin q = g (dl sin q) = g dy
For complete rope, sum of tangential component of weight is


3.92 Problems in Physics for JEE Advanced

Maximum friction force on the segment


= m ( dl g cos q) = m g (dl cos q) = m g dx
Sum of friction on the complete rope

Rope will be in equilibrium if


max Wt
m gx gH

97. (a) Let mass of each segment of the rope be

For rope to just remain in equilibrium


mg sin 60° + mg sin 30° = m mg cos 60° + m mg + m mg cos 30° + m mg

(b) °
30
s in
TD
mg E
TD
TC f3 D f4
TB TC
30°
TB
B f2 C
°
60
sin

f1
mg

A
60°

TB = mg sin 60° – ƒ1 = 0.67 mg = 0.17 Mg [ƒ1 = m mg cos 60°]


TC = TB – ƒ2 = 0.27 mg = 0.07 Mg [ƒ2 = m mg]
TD = TC + mg sin 30° – ƒ3 = 0.4 mg = 0.1Mg [ƒ3 = m mg cos 30°]
TE = TD – ƒ4 = 0 [ƒ4 = m mg]
\ TD = m mg
T

0.17 Mg

0.1 Mg
0.07 Mg

X
A B C D E

Maximum tension is at B
Tmax = TB = 0.17Mg
Newton’s Laws 3.93

98.
N = mg cos 37°
N´ = m´g cos 37° A = 8N
= 16N TAD TAB
TAD TAB
B
D fA
fD fb
mg sin 37°
m´g sin 37° TBC = 6N
= 12N
TBC
37° 37°

mg = 10N

C =0
Bmax= m mg cos 37° = 4 N
Amax = m mg = 5 N
Dmax = m m´g cos 37° = 8 N

If we look at the system as a whole, with no friction, it will move to right. The net driving force will be 10 + 6 – 12
= 4N. Friction is strong enough to prevent the blocks from moving.
Start with block C. There is no friction
TBC = 10N
For B: mg sin 37° + TBC = 16N
Friction will assume its peak value of 4N but a tension TAB = 12 N is required to maintain the block in equilibrium.
For A: TAB = 12 N is balanced by friction Amax = 5 N and TAD = 7 N
For D: It has tendency to slide down.
Friction is up along the tangent
D = 12 – 7 = 5 N
Note that friction on D is not at its peak.

Ê ˆ
99. 75% of the surface becoming dust free means that only Á ˜ th of the disc surface will remain covered with dust.
Ë ¯
This means all particles beyond will fly away.


x
f

For a dust particle at a distance x from the centre


N = mg
and = mw2x
But mN
3.94 Problems in Physics for JEE Advanced

mw2x m mg

We want for the dust particles should fly away and for < the friction should be able to provide the
necessary centripetal force

m È ˘
w = = ÍQ m = ˙
Î ˚

100. Consider a dust particle P rotating in a circle of radius x. We are considering the particle at the lowest position of
the circular motion. This is the position where a particle has maximum chance of leaving the disc, if friction fails to
provide the necessary centripetal force.

 

N
x
f
P

mg cos  mg sin 
mg

N = mg cos q
and – mg sin q = mw2x
= mg sin q + mw2x
[If you consider the particle at the top most point friction force required will be least equal to
= mw2x – mg sin q
But mN
mg sin q + m w2x
m mg cos q

putting [for 75% dust particles to fly off]

w=
(m q- q)


Newton’s Laws 3.95

For q = 0°

This matches with the result obtained in the previous problem.


Since the particle is small, the string will be tangential to the sphere. q = 60° (see figure).
Let acceleration of the sphere be a0 immediately after release
The particle will have its initial acceleration (a) along normal to the string towards PC. And, component of a0 in the

direction must be equal to a

30°
T
N

P a0
q
C

mg


Force on the particle has been shown in the figure. The equation of motion along PC will be

…………(i)

Sphere experiences a force N along PC. It has a horizontal component = N sin q

…………(ii)

From (i) and (ii)

Let the spring connected to A get stretched by x. The other spring gets compressed by x.
In the reference frame attached to the rod, forces acting on the sleeve are –
2 kx towards A
kx towards A
l towards B.

2kx + kx = mw2 ( l 0 + x)

3.96 Problems in Physics for JEE Advanced

l l


The solid angle subtended by the spherical cap at the centre of the hemisphere is = 2 (1 – cos q
Surface area of the cap
s = R2 = 2 R2 (1 – cos q)
If s = (0.2) (2 R2) then
0.2 = 1 – cos q
cos q = 0.8 q = 37°
A dust particle at an angle q > 37° will slide. A particle at q = 37° is in limiting equilibrium in the reference frame
of the hemisphere.
w

N
f

mw2r r
R

mg

N = mg cos q – mw2r sin q


And mN = mg sin q + mw2r cos q
m (mg cos q – mw2r sin q) = mg sin q + m w2r cos q
Where r = R sin q = 0.1 × 0.6 = 0.06

ar

mg

As the speed of car increases, the friction force (in direction shown) increases in strength. For a car travelling at
speed V
Newton’s Laws 3.97

……..(1)

N cos q – sin q = mg
……..(2)
(1) ∏ (2)

As V increases, f also increases. But maximum allowed value of f is mN

(b) For friction to be zero

and N cos q = mg

\ = q

Ê V02 ˆ
Á Rg - m ˜
= Rg Á ˜
Á mV02 ˜
ÁË 1 + Rg ˜¯

(c) Since car is retarding, there is a component of friction ( 1) opposite to the direction of motion. There is another
component of friction ( 2) opposite to that shown in previous fig . At very low speed N sin q exceeds the
necessary centripetal force – and friction is directed so as to reduce the net force towards centre.
3.98 Problems in Physics for JEE Advanced

Resultant friction makes obtuse angle with the direction of motion.

f2

f1 f

105. (a) Banking angle is given by

N N cos

N sin

f cos

f
mg

f sin

q = where V = correct speed = 20 m/s

\ q= =
¥
q = tan–1 (0.4) = 22°

(a)
when V = 30 m/s
Friction is directed inwards.
mV 2
In horizontal direction N sin q + f cos q =
R
In vertical direction N cos q = ƒ sin q + mg
Eliminating N between two equations

( q q+ q) = - q

¥
fi [ ¥ + ]= - ¥ ¥

= 2315 N
Newton’s Laws 3.99

(ii) When V = 10 m/s


N cos
N
f sin

N sin f cos

mg

The vehicle has a tendency to slip in and friction is directed upwards. Thus,

q- q=

And N cos q + sin q = mg
Again eliminating N we get

[ q q+ q] = q-

Putting V = 10 m/s and q = 22° we get


= 1398 N
(a) The speed of car increases uniformly at a rate f.
V

at

ar

It means, tangential acceleration of the car is at = and motion is non – uniform circular. At time t, its speed is
V = V0 + t
Thus, radial (centripetal) acceleration at time t is

= =
( + )

The resultant acceleration of the car is
= +

=
( + ) +
a at

ar


3.100 Problems in Physics for JEE Advanced

This acceleration is provided by friction force, f which has a maximum value


= mmg
If mass times acceleration exceeds this maximum value of friction the car skids
Car skids if

( + ) + ≥m


( + ) + ≥m

(V0 + t) [R2 ( 2g2 – 2 1/4


)]

È
fi ≥ Î
(m - )˘˚ -

\
È

(m - )˘˚ -


(c) For t < t0, friction adjusts itself so as to provide the necessary acceleration a
\ f = ma

(V0 + ft )
4

= m + f2
R2
The direction of friction is in the direction of a as shown in fig.
107. (a) Total impulse of all the force acting on the man is zero in 0 to 9.0 s interval.

( )
Ú
\ - =

Scale

Mg

Integration is easily obtained by area under the graph.

¥ ¥[ + ]- ( )¥ =

F0 = 93.3 N
(b) Maximum change in weight = 100 N
70 × amax = 100

= m/s2
Newton’s Laws 3.101

(c) ¥ = ¥ ¥[ + ]

Vmax = 4 m/s
108.
aA
A
T

A
a1
TB
C
450
F
450
TA
a2

TB
aB
Component of acceleration of C can be assumed along the two rods (the rods are r) as a1 and a2.
Since rods are rigid

For A = ………..(1)

For B = ………..(2)

For C - = .………..(3)

And - = .………..(4)

Using (2) and (4)
- =

Ê ˆ
- = ÁËQ = ˜¯
3.102 Problems in Physics for JEE Advanced

Ê ˆ
\ = ÁË + ˜¯

=


N f

sin
mg
mg cos

mg

(a) If the 4 kg block is placed on the incline separately, then driving force on it is

q= ¥ ¥ =

The opposing force is friction, which can attain a maximum value of


= mN = 0.5 × mg cos 30°
= 0.5 × 4 × 10 × 0.86
= 17.2N
Therefore, 4 kg block will accelerate down if allowed to move separately.
For 2 kg block (if allowed to move separately).

Driving force = mg sin 30° = ¥ ¥ =

Opposing force = mg cos 30°


= 0.8 × 2 × 10 × 0.86 = 13.76 N
2 kg block has no tendency to slide down. When placed together on the incline, 4 kg block will accelerate
down but 2 kg block will remain stationary behind it.
Contact force = zero.
Acceleration of 4 kg block is given by
ma = mg sin 30° – mmg cos 30°
a = g (sin 30° – m cos 30°)

È ˘
= Í - ¥ ˙
Î ˚
= 0.7 m/s2
m1 = 0.8, m2 = 0.5
(b)
In this case (if allowed to move separately) driving force on 4 kg block = mg sin 30° = 20 N.
Opposing force on 4 kg block = m1mg cos 30° = 0.8 × 4 × 10 × 0.86
= 27.52 N
Driving force on 2 kg block = 10 N
Opposing force on 2 kg block = 2mg cos 30° = 8.6 N
The 4 kg block has no tendency to move but 2 kg block has. Since 2 kg block is placed behind 4 kg block, it tries
Newton’s Laws 3.103

to push the 4 kg block. Whether motion will take place depends on the magnitude of driving force and opposing
force on the combined system.
Net driving force = 20 + 10 = 30 N
Net opposing force = 27.52 + 8.6 = 36.12 N
Motion will not take place and acceleration of both = 0.
Since, the 2 kg block is pushed against the 4 kg block, there will be normal contact force (Say R) between them.
To know the contact force (R) consider the equilibrium of 2 kg block (see fig.)
R + mN = 2g sin 30°
R = 2 g sin 30° – m2g cos 30° = 1.4 N
2 kg block applies equal and opposite force on 4 kg block
N
f

sin 2g cos 30°


2g
30° 2g

m1 = 0.6; m2 = 0.1
(c)
In this case (if blocks are allowed to move separately)
Driving force on 4 kg = 20 N
Opposing force on 4 kg = 0.6 × 4 × 10 × 0.86 = 20.64 N
Driving force on 2 kg = 20 N
Opposing force on 2 kg = 0.1 × 2 × 10 × 0.86 =1.72 N
Net driving force = 30 N
Net opposing force = 22.36 N
4 kg block when left alone has no tendency to move. It is pushed by 2 kg block and motion takes place.

-
= = m/s2

To know the contact force we can consider the free body diagram of either blocks. Here we consider the 2 kg
block.
2g sin 30° – R – m (2g) cos 30° = 2a

fi Ê ˆ
= ÁË - ¥ ˜¯ - ¥

= 8.28 – 2.54 = 5.74 N


N f

a R

2g sin 30° 2g cos 30°

2g
3.104 Problems in Physics for JEE Advanced


NV

N
V f

NH


Vertical normal reaction Nv = mg = 0.1 × 10 = 1 N

¥
Horizontal normal = = =

Resultant normal force

= + =

Friction =m =

Resultant force by rod

=
+ =

Angle made by R with vertical

= = =

Ê ˆ
q = ÁË ˜¯

C R D
t = 24.2 s
O
50 m

M t = 27.2 s

L
t=5

75m

A B
t=0

Maximum speed allowed on the circular portion is


Newton’s Laws 3.105

= m = ¥ ¥ = m/s
The car will accelerate with maximum acceleration a = g = 6 m/s2

It acceleration for 5 seconds to acquire a speed of 30 m/s and covers a distance ¥ ¥ = in the period.
Car moves from L to C to D with this constant speed.
Time needed to move from L to D is

+p ¥
= =

Hence, car reaches D at t = 24.2 s. After this it accelerate to acquire the top speed of 50 m/s (= 180 kph)

-
Time required for acceleration = =

Distance covered in this period = -


=
¥
Remaining distance of 130 m is covered at a speed of 50 m/s.
Time required for this is = =

Total time = 24.2 + 3.3 + 2.6 = 30.1 Sec.


For stationary bowl –

N
f

mg cos q
m

sin
g

mg
q

Insect can move upto q = 45°


max = mg sin q [for q = 45°]
mmg cos q = mg sin q
m = 1.0
For rotating bowl
w

w
m

O
2

sin f
R

q
co N
s
q
mw R sin q
2

R sin
mg cos q
+ mg
mw2R sin2 q
m

sin
g

q
3.106 Problems in Physics for JEE Advanced

In reference frame of bowl


N = mg cos q + mw2R sin2 q

[Insect moves slowly]


If insect can just reach q = 60°
mN + mw2R sin q cos q = mg sin q
mg cos q + mw2R sin2 q + m 2
sin q cos = mg sin q [Q m = 1]
2
w R sin q (sin q + cos q) = g (sin q – cos q)

Ê ˆ Ê ˆ
w + ˜= Á - ˜
Á
Ë ¯ Ë ¯

fiw =
( - )
( + )

w=
( - )
( + )
(b) for q = 90°
w2R = g

w=

Let tension in BC & BD be T1 and that in string BA be T2.
C
M
T1

D
T0
T1

B T2

Mg

The string CB and DB make an angle of a = - Ê ˆ with vertical because the diagonal of a cube makes
ÁË ˜¯

- Ê ˆ angle with a side.


ÁË ˜¯

Line BM makes = 45° with vertical.

=
Newton’s Laws 3.107

\ q=

Resultant of tension in CB and DB is along BM equal to

= q=

Vertical component of T0 balance Mg and its horizontal component is equal to T2.


T0 cos b = Mg
T0 sin b = T2

fi b=

fi =

fi =

114.
N N

mg

F0 R F0


Radius of the circular plate is r = R (sec q – 1)
Mass of the plate is m = spr2
= spR2 (sec q – 1)2
For equilibrium of the plate -
2 N sin q = spR2 (sec q –1)2 g

fi =
sp ( q- )
q N’’

F0
R N’

Mg

3.108 Problems in Physics for JEE Advanced

For two discs to remain in contact, normal force between them; N´ 0.


If the discs are just in contact N´ = 0
For horizontal equilibrium
sp q( q- )
= q=
q

Þ =
sp ( - q)
q q

q
(b) When q Æ q - q q

q
sp
fi = Æ
q ÈÍ - q ˘
Î ˚˙
(c) When

When , the circular plate will become massive. Even through N points almost vertically, its horizontal
component still approaches infinity due to large magnitude of N.
O = centre of larger cylinder.
DO will pass through centre of lower cylinder (B)
N1 = Contact force between A and B (and A and C)
N2 = Contact force between B and the large cylinder.
N3 = Contact force between B and C.
For the vertical equilibrium of A
O

A
N1 N1
mg

N1
B C
N3
N2 mg

D E

2N1 cos 30° = mg 3 N1 = mg ..............(1)


For horizontal equilibrium of B
N1 sin 30° = N2 sin q

..............(2)
Newton’s Laws 3.109

For vertical equilibrium of whole system


2N2 cos q = 3 mg

..............(3)

(2) ∏ (3)

A
2r 2r
30° 30°

B C
r r

D E

fi 27r2 = (R – r)2 – r2

fi (R – r)2 = 28r2
R – r = 2 7 r
R = r (1 + 2 7)

A A A

l0
2
C C
2l0
P P

B’
B’
B
(A) (B) (C)

l Q
Initial extension in the spring is l

Initial distance between the insect and the bug is l


As the insect climbs up (slowly), the tension in the segment of the spring above the insect does not change (tension
in segment AP = mg) and the segment of the spring below the insect becomes relaxed. It means the length of
segment AP of the spring remains unchanged when the insect moves from B to P.
3.110 Problems in Physics for JEE Advanced

l l
When the insect is at the original distance, it is at point C such that
l
Looking at fig (A) and (B) one can realize that segment AC is of the spring (i.e., its elongated length = ). The
l
natural length of segment .

Hence, length of the spring when the insect is at C is

l l l

: Let initial stretch in the spring be x0


A

l0/2
2l0 B

kx0 = mg

l
l

Initial distance between the insect and the bug = l


(c) When insect is at B such that

l l

Only the portion AB of the spring takes the load and the length BC remains unstretched.
In other words part AB behave as a separate spring supporting the insect.

Let natural length of segment AB be l

l
Then force constant of AB is
l
l
Extension in part l

l
l

l l
or, l
l

l 
or, mg  0 − l1  = mg l1 [Ql 0 k = mg ]
 2 
l0
or, 2l 1
2
Newton’s Laws 3.111

or, l1 l0
4
Total length of spring = AB + BC

= l 0 + l 0 − l 0 
2  4 
5l 0

4

(a) Let T be the tension in the string and a1, a2, a3, a4 be acceleration of the four masses in downward direction.
The string has fixed length, which implies that sum of the displacements (and hence acceleration) of all four
masses must be zero. In other words,
a1 + a2 + a3 + a4 = 0 ………(1)
For m1
m1a1 = m1g – 2 T

⇒ a1 = g − 2T ………(2)
m1
Similar equation can be written for a2, a3 and a4 put all these in (1)

 1 1 1 1 
4 g − 2T  + + + =0

 m1 m2 m3 m4 
4g
⇒ 2T = = 4mg
1 1 1 1
+ + +
m1 m2 m3 m4

 Where 1 = 1 + 1 + 1 + 1 
 
 1 2 3 4 

From (2)
4mg
a1 = g −
m1

 4m 
= g 1 − 
 m1 

 
(b) Similarly, a4 = g 1 − 4m 
 m4 

for a4 to be positive
4
1
4
4 1
4
3.112 Problems in Physics for JEE Advanced

4 1 1 1 1
< + + +
4 1 2 3 4

3 1 1 1
< + +
4 1 2 3
3 11
4 6
18
4 kg
11
If F is increased beyond a limit, block D will slip over C. This is because friction force acting on D is responsible
to accelerate D + B + A. Hence, for all the four block to move together –
Acceleration of the system produced by F acceleration produced by friction on D in the system D + B + A

F µ m2 g
∴ ≤
2m1 + 2m2 m1 + 2m2

 m + m2 

∴ F0 = 2µ m2 g  1 
 2m2 + m1 

Let’s calculate the net force of gravity along the slope on the part AB of the chain
B
dl

dx H

A C
x=0 x x+dx

consider the small segment of the chain between
X and X + dX.
Gravitational force along the slope is

λ d lg sin θ [ = mass per unit length]



Net gravity force along the slope on part AB is

λ g ∫ dl sin θ = λ g ∫ dy = λ gH

Thus, the force does not depend on the profile of the hill. It only depends on height of B from A.
For portion BC also the force will be same. Hence, net force on the chain is zero.
Acceleration of the chain in zero.

From last question

H R
µ = = = 1.0
X R
Newton’s Laws 3.113

(a) At some lower point in the rope, if we take a small segment, the tangential component of weight is higher then
friction on the segment. As we move up, friction increases and tangential component of the weight decrease.
Tension will be increasing as we go up from bottom till the point where tangential component of weight on
an element equal the friction force on it. After this point the friction on a segment gets larger than tangential
component of weight. Tension starts decreasing.
dlgcos


dlg.sin

Tension is maximum where


m dl g cos q = ldl g sin [Q m = 1]
tan q = 1
q = 45°
(b)
TA
A

H

B
X


For segment AB of the rope (refer to last question)

wt = gH = gR sin 45° = g

 1 
and = mlgX = 1lgR (1 – cos 45°) = lgR 1 − 
 2

Tension at A is
TA = Wt – = lgR ( 2 – 1)

M
=
πR
gR ( )
2 −1

 2 −1
= 2   Mg
 π 
3.114 Problems in Physics for JEE Advanced

F sin q
F
N

F cos q

f
mg = 4 x10 = 40 N

N = 40 – F sin q …………….(1)
The block can move if
F cos q > fmax
F cos q > mN
F cos q > 0.5 (40 – F sin q)
18 [cos q + 0.5 sin q] > 20
10
cos q + 0.5 sin q
9

 1 0.5  10
⇒ 12 + 0.52  cos θ + sin θ  >
 9
2 2
 1 + 0.5 12 + 0.52

10
⇒ 1.25 sin (θ + δ ) >
9

 −1  1  
 Where δ = tan  0.5  = tan ( 2 ) = 63º 
−1

   
sin


O° 84° 96°

10
sin (q + ) >
9. 1.25
84° < q + < 96°
fi 84° – 63° < q < 96 – 63°
21° < q < 33°

124. For equilibrium of rope


2T1 sin = F + mg ......…(1)
Newton’s Laws 3.115

T1 T1

 


F
mg

Consider the equilibrium of half the rope


T1

q T2
2

θ 
1 cos α = 2 sin   ......…(2)
 2     

θ 
And also, 2T2 cos   = F ......…(3)
2

1 Ê q ˆ T cos a
(2) ÷ (3) tan Á ˜ = 1
2 Ë 2¯ F


θ
Using (1) tan   =
( F + mg ) cos α
2 sin α .F

 F + mg  
θ = 2 tan −1   cot α 
 F  

125. Consider an element of the rope having angular width .


(T+
T
) sin T+T
( 
 /2) (T+T) cos (/2)

 

T
sin



 m
in gc
T gs os N
m
T cos  mg

Length of the element =R q


Mass of the element = lR q
Tension at lower end of the element =T
Tension at upper end of the element = T + T
For tangential equilibrium of the element
3.116 Problems in Physics for JEE Advanced

∆θ   ∆θ 
(T + ∆T ) cos   − T cos   = λ R∆θ .g sin θ
 2   2 

∆θ 
For small angle q, cos   1
 2 
T + T – T = Rg q sin q
T = lRg q. sin q
For q 0
dT = lRg sin qdq .................(1)

For equilibrium in radial direction

∆θ ∆θ 
N + Rg cos = T sin  
+( +∆ ) sin  
 2   2 

∆θ ∆θ
∆N = T + (T + ∆T ) − λ Rg ∆θ .cos θ
2 2

∆θ ∆θ
[Q for small q; sin  
→ ]
 2  2

T q = 0 [product of very small quantities]


N = T q – lRg q . cos q
For rope to remain in contact
N 0 at all points
T lRg cos q at all points
At bottom (i.e., at q = 0°)
T1 lRg .................(2)
From equation (1)
dT = lRg sin q dq
T0
π /2
∫ dT = λ Rg ∫0 sin θ dθ
T1

T0 – T1 = lRg
T0 = T1 + lRg

T0 2 Rg [using (2)]

126. m = tan q0 = 1
3
Maximum friction on the block max = mg cos = mg
2
mg 
Block will begin to slide when resultant of mg sin θ  =  and F will become just larger
 
than max.
Newton’s Laws 3.117

fmax

F


3
fmax  mg
mg 2
—–
2

Fig. In the plane of


the incline

mg / 2
sin α =
From figure 3
mg
2
1
sin α =
3
127. Force shall be applied in the vertical plane through the line of greatest slope. Let the direction of the force make an
angle a with the incline plane.
N

F


mg

N = mg cos q – F sin
Block just moves if
F cos a + mg sin q = mN
F cos a + mg sin q = m (mg cos q – F sin a)

mg ( µ cos θ − sin θ )
F=
cos α + µ sin α
F is minimum when cos a + m sin is maximum

Maximum value of cos a + m sin is 1 + µ 2

mg ( µ cos θ − sin θ )
Fmin =
1+ µ2

q = 30° and m = 1
 3 1
mg  − 
2 2  mg
Fmin = 
2
=
2 2
( 3 −1)
3.118 Problems in Physics for JEE Advanced

128.
A f2
f1
f1 f2
F C
B
B
a

F
(a) Let the common acceleration be a
M
For A – = M.a
For C = Ma
Adding = 2Ma
is larger than .

2F Mg
f1 = 2 Ma = ≤µ
3 2
3
F ≤ µ Mg
4

3
Fmax = µ Mg
4
(b) Since >
The slippage will take place at surface between A and B.

3
(c) For F = µ Mg
8

µg
a =

µ Mg
f 2 = Ma =
8
– = Ma

µ Mg
f1 = 2 Ma =
4

1
2
2

129. (a) Let’s assume that there is no slipping between C and D. All four blocks move with acceleration a.
a
T1
C
T2
D
a
T1 f T2

a
A B

30N 5N
Newton’s Laws 3.119

For A: 30 – T1 = 3a …....... (1)


For B: T2 – 5 = 0.5a ….......(2)
For (C + D) T1 – T2 – = 4.5a
T1 – T2 – 0.2 × 45 = 4.5a ….......(3)
(1) + (2) + (3) 16 = 8a
a = 2 ms–2
We must check whether C can have this acceleration without the friction force on it exceeding its peak limit.
a = 2ms-2

C
T1
fCD

From (1) T1 = 24 N
For C T1 – CD = 3.a
CD = 24 – 6 = 18 N
Peak value of CD = mN = 0.7 × 30 = 21 N
Hence, friction is within limit. Our assumption that all four are moving together is correct.
(b) If C and D move in opposite direction friction between them will be kinetic.
CD = mN = 21 N
For (B + D)
fCD
T
D

mg

The block D will move to right (and B will move downward) if


mg CD
mg 21
m 2.1 kg
130.

R N
N
y 
 T
x
mg
Mg Forces on the rod

(a) Forces on the hemisphere


Let R = Normal force by ground on the hemisphere
N = Normal force by the rod on the sphere
N´ = Normal force by the wall on the rod
3.120 Problems in Physics for JEE Advanced

T = Tension in string.
For hemisphere
R = Mg + N sin q ............ (1)
and T = N cos q .............. (2)
For rod
N’ = N cos q .............. (3)
And N sin q = mg …....... (4)
Put value of N from (4) into (2)
mg
T= .cos θ = mg cot θ
sin θ
(b)

R
y

O
x

From the figure


y2 + x2 = R2

dy dx
2 y + 2x = 0
dt dt

dy dx
y = −x
dt dt
[– ve sign indicates that as x increases, y will decrease i.e., the rod will move down]
We will discard the negative sign. Differentiating once again

dy dy d2y dx dx d2x
    + y. 2 =     + x. 2
 dt   dt  dt  dt   dt  dt
Just after the string is cut, velocity of the rod as well as hemisphere is zero.

dy dx
⇒ = =
dt dt

d y xd x

dt y dt
ay = (cot q) ax …..........(1)
ay = downward acceleration of the rod
ax = leftward acceleration of the hemisphere
Using the force diagram in the part (a) we can write
For hemisphere: N cos q = Max …..........(2)
Newton’s Laws 3.121

For rod mg – N sin q = may …..........(3)


Solving (1), (2) and (3)
mg
ax =
M tan θ + m cot θ

131. Let the speed be v after time t when the block has moved through a distance S.

mv
Normal reaction of the ring is N
R

dv
m = −µ N
dt

dv µv
=−
dt R
v t
dv µ

∫u v2 = − R ∫0 dt
µ
− = t
v u R

1 1 m
= + t …..........(1)
v u R

dt µ
= + t
ds u R

t πR
dt
∫ 1 µ = ∫ ds
0 + t 0
u R
 1 µt  1 µ
ln  +  − ln   = π R
 u R  u  R

 µu 
ln 1 + t = πµ
 R 

µu
+ t = eπµ
R

R πµ
t = e −  …..........(2)
µu 
Put value of t from equation (2) into (1)
Ru Ru
v=
=
R + µ ut R + R eπµ − 

u
v = πµ
e
3.122 Problems in Physics for JEE Advanced

132. The helix wire is inclined to the horizontal at an angle given by


mg


ρ
tan θ = …........(1)

The normal force applied by the wire on the bead can be thought to have two components.

N = directed horizontally towards axis of helix. (In the second fig. given above, this force is normal to the plane of
the drawing pointing towards you)
N´ = directed perpendicular to the velocity in the plane of above figure.
Force equations are (when the bead acquires constant speed)
= mg sin q …........(2)
N´ = mg cos q …........(3)

mv H
N
R

mv 2 cos 2 θ …........(4)
N=
R
Where vH = Horizontal component of velocity = v cos q
The resultant of two components of normal forces is
2
0 = + '2

2
 mv 2 cos 2 θ 
= ( mg cos θ ) + 
2

 R 

v4
= m cos θ g 2 +
cos 2 θ
R2
Now friction = mN
from (2)

v4
µ mg cos θ 1 + 2 2 cos 2 θ = mg sin θ
R g
Newton’s Laws 3.123

tan θ
µ=
2
 v 2 cos θ 
1+  
 Rg 
133. Force diagram of the wedge and that of block in the reference frame of the wedge are as shown.
N sin q

N cos q
N

N0
a0

Wedge mg
N q
sin
ma
0
ma0 sin q

ma0

os
q
o sq
c
ma 0 mgc
a q
sin
mg mg

For wedge: N sin q = ma0 …......... (1)


For block: (a = acceleration of block relative to wedge)
N + ma0 sin = mg cos q …......... (2)
and N sin q = ma0 ….........(3)
Solving (1), (2) and (3)
1 1
g .
g sin θ cos θ 2 2 =g
a0 =
= 2
1 + sin 2 θ  1  3
1+  
 2
1
2g
2 g sin θ 2 =2 2g
a= = 2
1 + sin 2 θ  1  3
1+  
 2
For block
ax = a cos q ( ) + a0 ( ) = a cos q ( ) – a0 ( )
2
= − = (←)
3 3 3
2g
a y = a sin θ =
3
(↓)
(b) Due to ax the horizontal velocity of the block decreases. It is easy to show that vx = 0 when the block lands on
the table.
3.124 Problems in Physics for JEE Advanced

The block strikes the table normally. Path is as shown.

(c) At highest point velocity is horizontal (equal to that of wedge).

Yourself show that vx = u cos θ = u


2 2 2

vx
ay
R

vx2 u2 3u 2
R= = =
a y 8 × 2 g 16 g
3

134. Consider the block at any position q shown in the figure. [In reference frame of cylinder]
f
m R
2

N
mg cos 
mg sin 
mg


N = mw2 R – mg sin q …........ (1)


= mg cos q …........ (2)
Q mN
mg cos q m mw R – mmg sin q
g [cos q + m sin q] mw2R [for all value of ‘q’]

Q maximum value of cos q + m sin q will be 1+ µ2

g 1 + µ 2 ≤ µω 2 R

ω ≥ g 1+ µ2

135.
(a) Consider a conical pendulum with one string (AO) only.
Newton’s Laws 3.125


l

mg

r = l sin q ….......(1)
2
T sin q = mw r
T sin q = mw2l sin q
T = mw2l …....... (2)
and T cos q = mg

mg g
cos θ = 2
= 2 ….......(3)
mω l ω l
For a conical pendulum to be possible
q > 0°

g
cos q < 1 <1
ω 2l

g

l
g
Hence, particle will stay at B if ω ≤
l
g
(b) When ω > , the particle begins to rotate in a circle and the tension in string becomes greater than mg.
l
The second string will get taut only when q = 60°

g 2g
From (3) ω 2 = =
l cos 60º l
g
ω=
l

g
If ω > , there is tension in BO
l
g g
Answer is <ω ≤
l l
3.126 Problems in Physics for JEE Advanced

(c)

60° T1

T2
mg
B

T1 cos 60° = T2 cos 60° + mg


T1 – T2 = 2mg …........ (4)
2
and (T1 + T2) sin 60° = m r


T1 + T2 = mw2l …........ (5)
For, T1 = 2T2, equation (4) and (5) give
T2 = 2 mg
and 3T2 = mw2l
6mg = mw2l

g
ω =
l
Draw the free body diagram in reference frame of the container.

N1
N2
m2r0

60°

60°

mg

r0 = radius of circular path of the centre of mass of the body


= 1.0 m

60° A
M

B D C
Newton’s Laws 3.127

3
BC = AM = r sin 60°
2

AC = MB = r (1 – cos 60°) =

1
CD = AC . cot 60° = . :
2 3

3 2
BD = BC – DC = − = = = 1.0
2 2 3 2 3 3
r0 = 2.0 – BD
= 2 – 1 = 1.0 m
N2 cos 60° + N1 = mg

N2
+ N1 = mg …...... (1)
2
and N2 sin 60° = mw2r0

3
N2 = mω 2 [r0 = 1]
2

2
N2 = mω 2 …...... (2)
3
mω 2
Put in (1) N1 = mg −
3

(a) For N2 > N1

2 mω 2
mω 2 > mg −
3 3

3
mω 2 > mg
3

ω >

(b) For N1 = 0
mω 2
= mg
3

ω =

137. (a) Normal force on the car N = mg cos q.


3.128 Problems in Physics for JEE Advanced

v
f
ft

fr


mg sin 

The friction has two roles to play. It has a component along the tangent ( t) which balances mg sin q, so that
there is no tangential acceleration and it provides the necessary centripetal force also.
\ t = mg sin q

mv
fr
R
friction

2
 v2 
( g sin θ )
2
f = ft 2 + f r2 = m + 
R
But mN

2
 v2 
( g sin θ )
2
m +   ≤ µ mg cos θ
R

2 2 v4
g sin θ + 2
≤ µ 2 g 2 cos 2 θ
R

v4
≤ g 2  µ 2 cos 2 θ − sin 2 θ 
R2
v [g2R2 [m2 cos2 q – sin2 q]]1/4
(b) For a given speed, the friction force will be largest at the bottom most point. To prove this consider the car at
an angle (as shown).

ft

v

fr

W cos 

W sin 
W
W = mg sin q
Newton’s Laws 3.129

mv 2
f r = W cos α +
R
t = W sin a

m 2 v 4 2mv 2
f 2 = W 2 + + W cos α
R4 R
Obviously, is maximum when cos a = 1; i.e., when a = 0°
and, maximum friction for a given speed v is ,

 mv 
f = W + 
 R 

mv 2
f = mg sin θ +
R
But m mg cos q

mv 2
mg sin θ + ≤ µ mg cos θ
R

v ≤ Rg [ µ cos θ − sin θ ]
138. Particle goes in a vertical circle of radius
R = l sin q
It can be proved that, in this case also, minimum speed at the lowest point to complete the circle is [Do it yourself].

u = 5 gR = 5 gl sin θ …......... (1)



After one string breaks, the particle will go in a horizontal circle (forming a conical pendulum) of radius
r = l cos q
O1


l cos 

mg

mu 2
T cos θ =
r
T sin q = mg

gr
tan θ =
u2
3.130 Problems in Physics for JEE Advanced

gl cos 2 θ
u 2 = …....... (2)
sin θ
gl cos 2 θ
From (1) and (2) = 5 gl sin θ
sin θ

1
tan θ =
5

m
139. For equilibrium M


A

P1

P2

M T  T

m
P2´

For horizontal equilibrium of pulley P2 it is necessary that the two segments (P1P2´ and P2´A´) of the string have
equal inclination to the vertical.
Hence, P2 will move horizontally as well as vertically.
Yes, it is possible that M does not have acceleration (i.e., it moves with constant velocity) and m has an acceleration.

mg
T T = ——
2
q q

P2

mg

mg – 2T cos q = ma

mg
mg − 2. cos θ = ma
2
mg
[Q If M does not have acceleration T Mg ]

a = g (1 – cos q)
Newton’s Laws 3.131

140.

N N
C
R
y
M N
P

V x V

Let CP = y and MN = x

x
y +   = R
 

dy 1 dx
2 y + .2 x = 0

dt 4 dt

dy x dx
=−
dt y dt

dy x  dx 
= − ( 2v ) Q dt = 2v 
dt 4y  
dy x
=− v ….......(1)
dt y

When x = a = 2 R

MP a R

R
∴y =

At this moment velocity of centre of the sphere is

dy 2R
=− .v = −v

dt R
2
2
[– sign indicates that y is decreasing]
Differentiating (1) once again

 dx dy 
d y y −x 
= −v  dt dt 
dt  y 
 

\ Acceleration of centre C when
3.132 Problems in Physics for JEE Advanced

R dx
y ;x 2 R; 2v
2 dt

dy
= −v
dt

 R 
2 .2v − 2 R ( −v ) 
d y
a = 2 = −v  2

dt   R 
2 
   
  2 

v2
=−
R
(
4 2 )
12 2
=− × 4 2 = −4 2 m/s
1
– sign indicates that the acceleration is downward.
Force equation for the ball
Mg – 2N cos 45° = M.a
2 × 10 – 2 × 4 2 = N 2
N = 10 2 – 8

(b) =5−4 2
2
141. Let’s consider the problem in a reference frame moving to left with an acceleration of a0.
In this frame P1 is at rest and all the blocks experience a force ma0 towards right.

P3

P2 T2

T1
P1
ma0
T2
ma0
T1
ma0

a1

Clearly T1 a0 …....... (1)


In this frame let the acceleration of block 1 be a1 ( ). Pulley P2 will move towards right with an acceleration of a1.
If an observer is located in reference frame attached to P2 [which is moving to left with an acceleration of (a0 – a1)],
he will similarly conclude that
T2 (a0 – a1) …....... (2)
Newton’s Laws 3.133

T1 a0
=
T2 a0 − a1

But pulley is massless


T1 = 2T2

0
=2
0 − 1

0
1
2

3
In ground frame, acceleration of block 1 will be (←)
2
142. Normal reaction on the particle is always N = mg cos q

µmg cos q
v


mg sin q

Friction force on the particle is = mg cos q and is always opposite the direction of instantaneous velocity.
Another force on the particle in the plane of the incline is mg sin which is always directed along x direction. Force
on the particle in tangential direction (i.e., in direction of its velocity) is

Ft = mg sin q . cos f – mg cos q

Acceleration in tangential direction is


at = g sin q (cos f – 1) [Q = tan q] ….......... (a)

Similarly, acceleration of disc along x – direction is

ax = g sin q – (mg cos q) cos

= g sin q (1 – cos f) ….......... (b)


at + ax = 0    [Adding (a) and (b)]
Integrating, we get
v + vx = c [c is a constant)
    
But vx = v cos f
3.134 Problems in Physics for JEE Advanced

v + v cos f = c

π
Initially φ = ; = 0
2
c = v0

= 0

1 + cos φ
04 WORK - POWER - ENERGY

m
Level 1
Q. 1. (i) The cause of increases in kinetic energy
when a man starts running without his feet u
slipping on ground is asked to two students.
Their answers are–
(a) the ground (b) the wedge.
Harshit: Cause of increase in kinetic energy is [Take g = 10 ms–2]
work done by friction force. Without friction
the man cannot run. Q. 4. In an industrial gun, when the trigger is pulled
a gas under pressure is released into the barrel
Akanksha: Cause of increase in kinetic behind a ball of mass m. The ball slides smoothly
energy is work done by internal (muscle) inside the barrel and the force exerted by the gas
forces of the body. on the ball varies as
Who is right?
(ii) An inextensible rope is hanging from a tree.
A monkey, having mass m, climbs to a height Where L is length of the end of the barrel from the
h grabbing the rope tightly. The monkey starts initial position of the ball and x is instantaneous
from rest and ends up hanging motionlessly displacement of the ball from its initial position.
on the rope at height h. Neglect any other force on the ball apart from that
(a) How much work is done by gravity on the applied by the gas. Calculate the speed (V) of the
monkey? ball with which it comes out of the gun.
(b) How much work is done by the rope on the L

monkey?
x
(c) Using work – energy theorem, explain
the increase in mechanical energy of the
monkey. Gas Cylinder
Q. 2. A man of mass M jumps from rest, straight up,
Q. 5. A particle of mass 3 kg takes 2 second to move
from a flat concrete surface. Centre of mass of the
from point A to B under the action of gravity and
man rises a distance h at the highest point of the
another constant force
motion. Find the work done by the normal contact 
force (between the man’s feet and the concrete = -  +
( 
) , where the unit vector
floor) on the man. k̂ is in the direction of upward vertical.

Q. 3. A block of mass m = 10 kg is released from the top The position vector of point B is
of the smooth inclined surface of a wedge which
is moving horizontally toward right at a constant

= - - ( )
and velocity of the

velocity of u = 10 m/s. Inclination of the wedge particle when it reaches B is



is q = 37°. Calculate the work done by the force
applied by the wedge on the block in two seconds
= ( +- 
) .

in a reference frame attached to - (a) Find the velocity, of the particle when it
4.2 Problems in Physics for JEE Advanced

was at A. at a constant speed of 40 m/s when the engine


 developed a power of 53 KW. When the engine
(b) Find position vector, of point A.
 developed an output of 2 KW the car was able to
(c) Find work done by the force as the particle travel on a horizontal road at a constant speed of
moves from A to B. 10 m/s.
(d) Find change in gravitational potential energy (a) Find the power that the engine must deliver
of the particle as it moves from A to B. for the car to travel at a constant speed of
Q. 6. A particle can move along a straight line. It is at 40 m/s on a horizontal road.
rest when a force (F) starts acting on it directed (b) The car is able to climb a hill at a constant
along the line. Work done by the force on the speed of 40 m/s with its engine working
particle changes with time(t) according to the at a constant rate of 69 KW. Calculate the
graph shown in the fig. Can you say that the force
inclination of the hill (in degree)
acting on the particle remains constant with time?
W Q. 11. A particle moves along the loop A–B–C–D–A
while a conservative force acts on it. Work done
by the force along the various sections of the path
are – WA " B = – 50 J ; WB " C = 25 J; WC"D =
60  J. Assume that potential energy of the particle
O t
is zero at A. Write the potential energy of particle
Q. 7. A particle is moving on a straight line and all the when it is at B and D.
forces acting on it produce a constant power P B
calculate the distance travelled by the particle in
the interval its speed increase from V to 2V.
A
C
Q. 8. Work done and power spent by the motor of an
escalator are W and P respectively when it carries D
a standing passenger from ground floor to the Q. 12. A moving particle of mass
     m is acted  upon 
by five
first floor. Will the work and power expended forces and . Forces and F 3 are
by the motor change if the passenger on moving conservative and their potential energy functions
escalator walks up the staircase at a constant are U and W respectively. Speed of the particle
speed? changes from Va to Vb when it moves from
Q. 9. (i) A block is connected to an ideal spring on a position a to b. Which of the following statement
horizontal frictionless surface. The block is is/are true –
  
pulled a short distance and released. Plot the (a) Sum of work done by and =
variation of kinetic energy of the block vs the Ub – Ua + Wb – Wa
spring potential energy.   
(b) Sum of work done by and =
(ii) A ball of mass 200 g is projected from the
top of a building 20 m high. The projection
- Ê ˆ
speed is 10 m/s at an angle q = ÁË ˜¯ (c) Sum of work done by all five forces =
from the horizontal. Sketch a graph of kinetic
energy of the ball against height measured  
from the ground. Indicate the values of kinetic (d) Sum of work done by and =
energy at the top and bottom of the building (Ub + Wb) – (Ua + Wa).
and at the highest point of the trajectory, Q. 13.
specifying the heights on the graph. Neglect F (in N)
air resistance and take g = 10 m/s2
1
Q. 10. A car of mass m = 1600 kg, while moving on any
X (in m)
road, experiences resistance to its motion given O 1 2.5 3.5 5.5
by (m + nV2) newton; where m and n are positive -1
constants. On a horizontal road the car moved
Work - Power - Energy 4.3


The given graph represents the total force in x (a) Find the work done by the snake against
direction being applied on a particle of mass gravity (Wg)
m = 2 kg that is constrained to move along x (b) Will the answer to part (a) be different if the
axis. What is the minimum possible speed of the
snake were of half the length but of same
particle when it was at x = 0?
mass.
Q. 14. A vertical spring supports a beaker containing
Q. 17. A uniform rope of linear mass density l (kg/m)
some water in it. Water slowly evaporates and the
passes over a smooth pulley of negligible
compression in the spring decreases. Where does
dimension. At one end B of the rope there is a
the elastic potential energy stored in the spring
small particle having mass one fifty of the rope.
go?
Initially the system is held at rest with length
L of the rope on one side and length on the

other side of the pulley (see fig). The external


agent begins to pull the end A downward. Find
the minimum work that the agent must perform
so that the small particle will definitely reach the
pulley.

Q. 15. A pan of negligible mass is supported by an ideal


spring which is vertical. Length of the spring is Lo.
A mass M of sand is lying nearby on the floor. A L
4
boy lifts a small quantity of sand and gently puts A
it into the pan. This way he slowly transfers the Force
entire sand into the pan. The spring compresses L by external agent

by . Assume that height of the sand heap on the


floor as well as in the pan is negligible. Calculate
B
the work done by the boy against gravity in
transferring the entire sand into the pan.
Q. 18. A particle of mass m = 100 g is projected
vertically up with a kinetic energy of 20 J form
a position where its gravitational potential energy
is – 50 J. Find the maximum height to which the
L0
particle will rise above its point of projection.
[g = 10 m/s2]
M
Q. 19. A physics student writes the elastic potential
Q. 16. A snake of mass M and length L is lying on an energy stored in a spring as = + ,
incline of inclination 30°. It craws up slowly and where L is the natural length of the spring, x is
overhangs half its length vertically. Assume that extension or compression in it and K is its force
the mass is distributed uniformly along the length constant. A block of mass M travelling with speed
of the snake and its hanging part as well as the V hits the spring and compresses it.
part on the incline both remain straight. L
V
M
L L
2 Find the maximum compression caused.
Q. 20. A block of mass m = 4 kg is kept on an incline
connected to a spring (see fig). The angle of
30°
the incline is q = 30° and the spring constant is
4.4 Problems in Physics for JEE Advanced

K = 80 N/m. There is a very small friction between (b) At what distance from A the mass will come
the block and the incline. The block is released to rest for the first time after being released.
with spring in natural length. Find the work done A
by the friction on the block till the block finally
comes to rest.[g = 10 m/s2]

k
m
B

Q. 25. Two blocks 1 and 2 start from same point A on


a smooth slide at the same time. The track from
A to B to C is common for the two blocks. At C
= 30° the track divides into two parts. Block 1 takes the
route C–D–E and gets airborne after E. Block 2
Q. 21. A body is projected directly up a plane which is moves along CFGH. Point E is vertically above G
inclined at an angle q to the horizontal. It was and the stretch GH is horizontal. Block 1 lands at
found that when it returns to the starting point its point H.
speed is half its initial speed.

(a) Where is the other block at the time block 1
(a) Was dissipation of mechanical energy of the lands at H? Has it already crossed H or yet to
body, due to friction, higher during ascent or reach there?
descent?
(b) Which block will reach at H with higher
(b) Calculate the coefficient of friction (m) speed ?
between the body and the incline.
A 2
1
Q. 22. A tanker filled with water starts at rest and then 1
rolls, without any energy loss to friction, down E
a valley. Initial height of the tanker is h1. The B
tanker, after coming down, climbs on the other D 2
C
side of the valley up to a height h2. Throughout F G H
the journey, water leaks from the bottom of the
tanker. How does h2 compare with h1? Q. 26 In the arrangement shown in the figure, block B
of mass M rests on a weighing scale. Ball A is
released from a position where spring is in its
natural length and the scale shows the correct
weight of block B. Find the mass of ball A so
h1
that the minimum reading shown by the scale
subsequently is half the true weight of B.

Q. 23. A stone with weight W is thrown vertically upward


into air with initial speed u. Due to air friction a
constant force R acts on the stone, throughout its
flight. Find –
(a) the maximum height reached and k

(b) speed of stone on reaching the ground. A


Q. 24. A mass m = 0.1 kg is attached to the end B of an
B M
elastic string AB with stiffness k = 16 N/m and
natural length l0 = 0.25 m. The end A of the string
is fixed. The mass is pulled down so that AB is
2l0 = 0.5 m and then released. Q. 27 In an aircraft carrier warship the runway is a
20 m long strip inclined at q = 20° to the
(a) Find the velocity of the mass when the string horizontal. The launcher is effectively a large
gets slack for the first time. spring that pushes an aircraft of mass m = 2000
Work - Power - Energy 4.5

kg for first 5 m of the 20 m long runway. The jet position x as shown in the figure.
engine of the plane produces a constant thrust of When the particle is at x = x0, it is given a kinetic
6 × 104 N for the entire length of the runway. The energy (k) such that 0 < k < 4U0
plane needs to have a speed of 180 kph at the end
of the runway. Neglect air resistance and calculate (a) Does the particle ever reach the origin?
the spring constant of the launcher. [sin 20° = 0.3 (b) Qualitatively describe the motion of the
and g = 10 m/s2] particle.
U(x)

6U0
m
20
5U0

q
U0
Q. 28 A block of mass M is placed on a horizontal
x
surface having coefficient of friction m. A constant O x0

pulling force = is applied on the block to


Q. 32 A pillar having square cross section of side length
displace it horizontally through a distance d. Find L is fixed on a smooth floor. A particle of mass m
the maximum possible kinetic energy acquired by is connected to a corner A of the pillar using an
the block. inextensible string of length 3.5 L. With the string
Q. 29 A small block is made to slide, starting from rest, just taut along the line BA, the particle is given a
along two equally rough circular surfaces from A velocity v perpendicular to the string. The particle
to B through path 1 and 2. The two paths have slides on the smooth floor and the string wraps
equal radii. The speed of the block at the end of around the pillar.
the slide was found to be V1 and V2 for path 1 and
2 respectively. Which one is larger V1 or V2? m
3.5 L B
A 1 A

L
L
2 (a) Find the time in which the particle will hit the
pillar.
B
(b) Find the tension in the string just before the
particle hits the pillar.
Q. 30 A particle can move along x axis under influence
Neglect any energy loss of the particle.
of a conservative force. The potential energy of
the particle is given by U = 5x2 – 20x + 2 joule Q. 33 (i) A simple pendulum consist of a small
where x is co-ordinate of the particle expressed in bob of mass m tied to a string of length L.
meter. Show that the total energy of oscillation of
The particle is released at x = –3 m the pendulum is  when it is
(a) Find the maximum kinetic energy of the oscillating with a small angular amplitude q0.
particle during subsequent motion. Assume the gravitational potential energy to
(b) Find the maximum x co-ordinate of the be zero of the lowest position of the bob.
particle. (ii) Three identical pendulums A, B and C are
Q. 31 A particle is constrained to move along x axis suspended from the ceiling of a room. They
under the action of a conservative force. The are swinging in semicircular arcs in vertical
potential energy of the particle varies with planes. The string of pendulum A snaps when
4.6 Problems in Physics for JEE Advanced

it is vertical and it was found that the bob fell Q. 36 A child of mass m is sitting on a swing suspended
on the floor with speed V1. The string of B by a rope of length L. The swing and the rope have
breaks when it makes an angle of 30° to the negligible mass and the dimension of child can be
vertical and the bob hits the floor with speed neglected. Mother of the child pulls the swing till
V2. The string of pendulum C was cut when the rope makes an angle of q 0 = 1 radian with the
it was horizontal and the bob falls to the floor vertical. Now the mother pushes the swing along
hitting it with a speed V3. Which is greatest the arc of the circle with a force and
and which is smallest among V1,V2 and V3?
releases it when the string gets vertical. How high
Q. 34 AB is a mass less rigid rod of length 2l. It is free will the swing go?
to rotate in vertical plane about a horizontal axis
[Take cos(1 radian) ~ 0.5]
passing through its end A. Equal point masses
(m each) are stuck at the centre C and end Bof the Q. 37. A particle of mass m is suspended by a string
rod. The rod is released from horizontal position. of length l from a fixed rigid support. Particle is
Write the tension in the rod when it becomes imparted a horizontal velocity . Find
vertical. the angle made by the string with the vertical
A l C l B when the acceleration of the particle is inclined to
the string by 45°?
Q. 38 A particle of mass m is moving in a circular
path of constant radius r such that its centripetal
acceleration ac is varying with time t as
ac = k2rt2, where k is a constant. Calculate the
power delivered to the particle by the force acting
on it.
Q. 35 A rigid mass less rod of length L is rotating in Q. 39 A ball is hanging vertically by a light inextensible
a vertical plane about a horizontal axis passing string of length L from fixed point O. The ball of
through one of its ends. At the other end of the mass m is given a speed u at the lowest position
rod there is a mass less metal plate welded to the such that it completes a vertical circle with centre
rod. This plate supports a heavy small bead that at O as shown. Let AB be a diameter of circular
can slide on the rod without friction. Just above path of ball making an angle q with vertical as
the bead there is another identical metal plate shown. (g is acceleration due to gravity)
welded to the rod. The bead remains confined
between the plates. The gap between the plates B
q
is negligible compared to L. The angular speed of
O
the rod when the bead is at lowest position of the
circle is w = . How many times a clink of A
u
the bead hitting a metal plate is heard during one m

full rotation of the rod ? (a) Let TA and TB be tension in string when ball is
at A and B respectively, then find TA – TB.
 
(b) Let and be acceleration of ball when
it is at A and B respectively, then find the
 
value of .
Q. 40 A ball suspended by a thread swings in a vertical
 plane so that the magnitude of its total acceleration
in the extreme position and lowest position are
equal. Find the angle q that the thread makes with
the vertical in the extreme position.
Q. 41 A particle of mass m oscillates inside the smooth
surface of a fixed pipe of radius R. The axis of the
Bead
pipe is horizontal and the particle moves from B
Work - Power - Energy 4.7

to A and back. At an instant the kinetic energy of Q. 43. A completely filled cylindrical tank of height H
the particle is K (say at position of the particle contains water of mass M. At a height h above the
shown in the figure). What is the force applied by top of the tank there is another wide container.
particle on the pipe at this instant? The entire water from the tank is to be transferred
into the container in time t0 such that level of
water in tank decreases at a uniform rate. How
will the power of the external agent vary with
A B time?

Level 2
H
Q. 42.

Q. 44. A uniform chain of mass m0 and length l rests on


a rough incline with its part hanging vertically as
shown in the fig. The chain starts sliding up the
O
R incline (and hanging part moving down) provided
the hanging part equals h times the chain length
(h < 1). What is the work performed by the
h friction force by the time chain slides completely
off the incline. Neglect the dimension of pulley
and assume it to be smooth.
(i) There is a vertical loop of radius R. A small block
of mass m is slowly pushed along the loop from
bottom to a point at height h. Find the work done
by the external agent if the coefficient of friction
is m. Assume that the external agent pushes
tangentially along the path.
q
(ii) A block of mass m slides down a smooth slope of
height h, starting from rest. The lower part of the Q. 45. A large flat board is lying on a smooth ground. A
track is horizontal. In the beginning the block has disc of mass m = 2 kg is kept on the board. The
potential energy U = mgh which gets converted coefficient of friction between the disc and the
into kinetic energy at the bottom. The velocity board is m = 0.2 . The disc and the board
 are moved
at bottom is . Now assume that an with velocity and
observer moving horizontally with velocity respectively [in reference frame of the ground].
towards right observes the sliding Calculate the power of the external force applied
block. She finds that initial energy of the block on the disc and the force applied on the board. At
what rate heat is being dissipated due to friction
is = + and the final energy of the between the board and the disc? [g = 10 ms –2]
block when it reaches the bottom of the track is
X
zero. Where did the energy disappear?
u

h Y
V

Q. 46. A car can pull a trailer of twice its mass up a


certain slope at a maximum speed V. Without
4.8 Problems in Physics for JEE Advanced

the trailer the maximum speed of the car, up the the length of its base l and coefficient of friction
same slope is 2 V. The resistance to the motion between the body and hill surface is m. What
is proportional to mass and square of speed. If is the work done if body is moved along some
the car (without trailer) starts to move down the alternative path shown by the dotted line, friction
same slope, with its engine shut off, prove that coefficient being same.
eventually it will acquire a constant speed. Find
this speed.
Q. 47 Force acting on a particle in a two dimensional
 F
XY space is given as . Show m h

that the force is conservative.



Q. 48. In a two dimensional space the potential
energy function for a conservative force acting l
on a particle of mass m = 0.1 kg is given by 
U = 2 (x + y) joule (x and y are in m). The particle Q. 52. In previous problem what is the work done by
is being moved on a circular path at a constant if the body started at rest at the base and has a
speed of V = 1 ms –1. The equation of the circular velocity v on reaching the top?
path is x2 + y2 = 42. Q. 53. A block of mass M is placed on a horizontal
(a) Find the net external force (other than the smooth table. It is attached to an ideal spring of
conservative force) that must be acting on the force constant k as shown. The free end of the
particle when the particle is at (0, 4). spring is pulled at a constant speed u. Find the
maximum extension (xo) in the spring during the
(b) Calculate the work done by the external force subsequent motion.
in moving the particle from (4, 0) to (0, 4). M
k u
Q. 49. A particle of mass m moves in xy plane such that
its position
 vector, as a function of time, is given
by ; where
b and k are positive constants. Q. 54. A spring block system is placed on a rough
horizontal floor. Force constant of the spring is k.
(a) Find the time t0 in the interval when
The block is pulled to right to give the spring an

the resultant force acting on the particle has zero elongation equal to x0 and then it is released. The
power. block moves to left and stops at the position where
(b) Find the work done by the resultant force acting the spring is relaxed. Calculate the maximum
kinetic energy of the block during its motion.
on the particle in the interval
Q.50. A block of mass 2 kg is connect to an ideal spring
and the system is placed on a smooth horizontal
surface. The spring is pulled to move the block
and at an instant the speed of end A of the spring Q. 55. In the fig shown, a block of mass M is attached
and speed of the block were measured to be to the spring and another block of mass 2M has
6 m/s and 3 m/s respectively. At this moment the been placed over it. The system is in equilibrium.
potential energy stored in the spring is increasing The block are pushed down so that the spring
at a rate of 15 J/s. Find the acceleration of the compresses further by . System is released.
block at this instant.
2M
A M
2kg

Q. 51. A body of mass m is slowly hauled up a rough hill k


as shown in fig by a force F which acts tangential
to the trajectory at each point. Find the work
performed by the force, if the height of hill is h,
Work - Power - Energy 4.9

(a) At what height above the position of release, Q. 59. A plank is moving along a smooth surface
the block of mass 2M will lose contact with with a constant speed V. A block of mass M is
the other block? gently placed on it. Initially the block slips and
(b) What is maximum height attained by 2M then acquires the constant speed (V) same as the
above the point of release? plank. Throughout the period, a horizontal force
is applied on the plank to keep its speed constant.
Q. 56. Block A and B are identical having 1 kg mass
each. A is tied to a spring of force constant k and (a) Find the work performed by the external
B is placed in front of A (touching it). Block ‘B’ force.
is pushed to left so as to compress the spring by (b) Find the heat developed due to friction
0.1 m from its natural length. The system is released between the block and the plank.
from this position. Coefficient of friction for both
the blocks with horizontal surface is µ = 0.2. M
V
K
A B
Smooth
= 0.2
Q. 60. A block of mass m1 is lying on the edge of a rough
table. The coefficient of friction between the
(a) Take . Kinetic energy of the
block and the table is µ. Another block of mass m2
system comprising of the two blocks will be is lying on another horizontal smooth table. The
maximum after travelling through a distance two block are connected by a horizontal spring of
x0 from the initial position. Find x0. Find the force constant K. Block of mass m2 is pulled to
contact force between the two blocks when the right with a constant horizontal force F.
they come to rest.
(a) Find the maximum value of F for which the
(b) Take k = 100 N/m. What distance (x1) will the block of mass m1 does not fall off the edge.
block travel together, after being released, (b) Calculate the maximum speed that m2 can
before B separates from A. acquire under condition that m1 does not fall.
Q. 57. In the arrangement shown in the fig. string, m2
m1 K
springs and the pulley are mass less. F
Both the springs have a force
constant of k and the mass of block k
B resting on the table is M. Ball A is Q. 61. A vertical spring supports a block in equilibrium.
released from rest when both the The spring is designed to break
springs are in natural length and just when extension in it crosses a
k A limit. There is a light thread
taut. Find the minimum value of
mass of A so that block B leaves attached to the block as shown.
M B The thread is pulled down with a
contact with the table at some stage.
force F which gradually increases
Q. 58. Two block A and B are connected to a spring (force
from zero. The spring breaks
constant k = 480 N/m) and placed on a horizontal
when the force becomes F0.
surface. Another block C is placed on B. The
Instead of gradually increasing
coefficient of friction between the floor and block thread
the force, if the thread were pulled
A is µ1 = 0.5, whereas there is no friction between
by applying a constant force, for F
B and the floor. Coefficient of friction between
what minimum value of the
C and B is µ2 = 0.85. Masses of the blocks are
constant force the spring will break?
MA = 50 kg; MB = 28 kg and MC = 2 kg. The system
is held at rest with spring compressed by x0 = 0.5 Q. 62 Two liquid A & B having densities 2r and r
m. After the system is released, find the maximum respectively, are kept in a cylindrical container
speed of block B during subsequent motion. separated by a partition as shown in figure. The
C
height of each liquid in the container is h and
area of cross section of the container is A. Now
A B
the partition is removed. Calculate change in
4.10 Problems in Physics for JEE Advanced

gravitational potential energy (DU) of the system (ii) If magnitude of a is twice the minimum
value found in (i), find the maximum height
A (measured from its original level at the
bottom of the trough) to which the block will
rise.
B O
R
a A
(a) assuming that the two liquids mix uniformly. T
(b) Assuming that the two liquids are immiscible.
What do you conclude from the sign of DU in the Q. 66. A semicircular wire frame of radius R is
above two cases? standing vertical on a horizontal table. It is
Q. 63. A particle is projected at an angle q  =  30º with pulled horizontally towards right with a constant
the horizontal. Two students A and B have drawn acceleration. A bead of mass m remain in
the variation of kinetic energy and gravitational equilibrium (relative to the semicircular wire) at
potential energy of the particle as a function of a position where radius makes an angle q with
time taking the point of projection as the reference horizontal. There is no friction between the wire
level for the gravitational potential energy. Who and the bead. The bead is displaced a little bit
is wrong and why? in upward direction and released. Calculate the
(a) KE/PE speed of the bead relative to the wire at the instant
it strikes the table. Assume that all throughout the
KE
semicircular wire keeps moving with constant
acceleration.
PE m
t R
(b) KE/PE
q

KE
Q. 67. A ideal spring of force constant k is connected
to a small block of mass m using an inextensible
PE light string (see fig). The pulley is mass less and
t
friction coefficient between the block and the
Q. 64. Two small rings each of mass ‘m’ are connected horizontal surface is . The string between
to a block of same mass ‘m’ through inextensible
the pulley and the block is vertical and has length
light strings. Rings are constrained to move along
l. Find the minimum velocity u that must be given
a smooth horizontal rod. Initially system is held
to the block in horizontal direction shown, so that
at rest (as shown in figure) with the strings just
subsequently it leaves contact with the horizontal
taut. Length of each string is ‘l’. The system is
released from the position shown. Find the speed surface. [Take ]
of the block (v) and speed of the rings (u) when l
K
the strings make an angle of q  =  60º with vertical.
(Take g = 10 m/s2)
l
m u
 

Q. 65. A toy truck T at rest, has a hemispherical trough of Q. 68. A light spring is vertical and a mass less pan
radius R in it [O is the centre of the hemisphere]. is attached to it. Force constant of the spring is
A small block A is kept at the bottom of the k. A block of mass m is gently dropped on the
trough. The truck is accelerated horizontally with pan. Plot the variation of spring potential energy,
an acceleration a. gravitation potential energy and the total potential
(i) Find the minimum value of a for which the energy of the system as a function of displacement
block is able to move out of the trolley. (x) of the block. For gravitational potential energy
Work - Power - Energy 4.11

take reference level to be the initial position of the (b) What is the period of this circular motion?
pan. Q. 72. A small block is placed on the top of a smooth
m
inverted hemispherical bowl of radius R.
Ref. level

R
K q

(a) The bowl is given a sudden impulse so that


Q. 69. A particle of mass m = 1.0 kg is free to move it begins moving horizontally with speed V.
along the x axis. It is acted upon by a force which Find minimum value of V so that the block
is described by the potential energy function immediately loses contact with the bowl as it
represented in the graph below. The particle is begins to move.
projected towards left with a speed v, from the
origin. Find minimum value of v for which the (b) The bowl is given a constant acceleration ‘a’
particle will escape far away from the origin. in horizontal direction. Find maximum value
U (in joule) of ‘a’ so that the block does not lose contact
with the bowl by the time it rotates through
an angle q relative to the bowl. You can
make suitable mathematical approximations
justified for small value of angle q.
10
Q. 73 A pendulum bob is projected form its lowest
position with velocity (u), in horizontal direction,
6
that is just enough to make the string horizontal
3 (position OC). At angular position q, at point B,
the speed (V) of the bob was observed to be half
its initial projection speed (u).
-8 -5 O 2 4 7 11 X O
(in m)
C
Q. 70. A particle of mass m = 1 kg is free to move along
x axis under influence of a conservative force. The 
potential energy function for the particle is
L
ÈÊ ˆ Ê ˆ ˘ V
= ÍÁ ˜ - Á ˜ ˙
ÍË ¯ Ë ¯ ˙˚
Î
B

Where b = 1.0 m and a = 1.0 J. If the total


A
mechanical energy of the particle is zero, find u
the co-ordinates where we can expect to find the (a) Find q
particle and also calculate the maximum speed of
the particle. (b) Plot variation of magnitude of tangential
acceleration with q.
Q. 71. A particle of mass m moves under the action of
a central force. The potential energy function is (c) Let the travel time from A to B be t1 and
given by U(r) = mkr3 that from B to C be t2. Looking at the graph
obtained in part (b), tell which is larger – t1 or
Where k is a positive constant and r is distance of
t2?
the particle from the centre of attraction.
Q. 74. A small ball is attached to an end of a light
(a) What should be the kinetic energy of the
string of length R. It is suspended in vertical
particle so that it moves in a circle of radius
plane supported at point A. B and C are two nails
a0 about the centre of attraction?
4.12 Problems in Physics for JEE Advanced

(of negligible thickness) at a horizontal distance shown in figure. The particle is given a horizontal
0.3 R from A and a vertical distance 0.4 R above A velocity u. At some point, its motion ceases to be
respectively. The ball is given a horizontal velocity circular and eventually the object passes through
u = 5gR at its lowest point. Subsequently, after the line AB. At the instant of crossing AB, its
the string hitting the nails, the nails become the velocity is horizontal. Find u.
centre of rotation. Assume no loss in kinetic O
A
energy when the string hits the nails. It is known
that the string will break if tension in it is suddenly
L
increased by 200% or more. 8
C L

0.4R

B u
A
0.3R A

R Q. 77 A simple pendulum has a bob of mass m and


string of length R. The bob is projected from
lowest position giving it a horizontal velocity just
u enough for it to complete the vertical circle. Let
Will the string break during the motion? If yes, the angular displacement of the pendulum from
where? What is tension in the string at the instant its initial vertical position be represented by q.
the string breaks? Plot the variation of kinetic energy (kE) of the bob
Q. 75. A spherical ball of mass m is kept at the highest and the tension (T) in the string with q. Plot the
point in space between two fixed concentric graph for one complete rotation of the pendulum.
spheres A and B (see figure). The smaller sphere Q. 78 A light thread is tightly wrapped around a fixed
has a radius R and the space between the two disc of radius R. A particle of mass m is tied to
spheres has a width d. The ball has diameter just the end P of the thread and the vertically hanging
less than d. All surfaces are frictionless. The ball part of the string has length R. The particle is
is given a gentle push (towards the right). The
angle made by the radius vector of the ball with imparted a horizontal velocity . The
upward vertical is denoted by q.
string wraps around the disc as the particle moves
up. At the instant the velocity of the particle makes
an angle of q with horizontal, calculate.
d q Sphere B

R O O R

Sphere A

(a) Express the total normal reaction force


exerted by the spheres as a function of q.
V0 p
(b) Let NA and NB denote the magnitudes of
normal reaction forces on the ball exerted by (a) speed of the particle
the spheres A and B respectively. Sketch the (b) tension in the string
variations of NA and NB as function of cos q Q. 79 An experimenter is inside a train. He observes
in the range of 0 < q < p by drawing two that minimum speed at lowest position needed by
separate graphs. a pendulum bob to complete a vertical circle is
Q. 76. A particle is suspended vertically from a point O 10 m/s. Calculate the minimum speed (u) needed
by an inextensible mass less string of length L. A at the lowest position so as to complete the vertical
circle when the train is moving horizontally
vertical line AB is at a distance of from O as
at an acceleration of a = 7.5 m/s2. Find the
Work - Power - Energy 4.13

maximum tension in the string during the motion. (b) At what value of q the compression (or
[g = 10 m/s2]. tension) in the rod changes to tension (or
compression)?

q
Q. 80 A track (ACB) is in the shape of an arc of a circle. O
It is held fixed in vertical plane with its radius OA
horizontal. A small block is released on the inner Q. 83 A pendulum has length L = 1.8 m. The bob is
surface of the track from point A. It slides without released from position shown in the figure. Find
friction and leaves the track at B. What should the tension in the string when the bob reaches the
be value of q so that the block travels the largest lowest position. Mass of the bob is 1 kg.
horizontal distance by the time it returns to the
Bob
horizontal plane passing through B?
A O

B 30°
Horz

C Q. 84 A small body of mass m lies on a horizontal plane.


Q. 81 Bob of a simple pendulum of length L is projected The body is given a velocity v0, along the plane.
horizontally with a speed of , from the (a) Find the mean power developed by the
lowest position. Find the distance of the bob from friction during the whole time of motion, if
vertical line AB, at the moment its tangential friction coefficient is µ = 0.3; m = 2.0 kg and
acceleration becomes zero. v0 = 3 m/s.
A (b) Find the maximum instantaneous power
developed by the friction force, if the friction
coefficient varies as µ=ax, where a is a constant
and x is distance from the starting point.
O
Q. 85 Two particles of masses M and m (M >m) are
connected by a light string of length pR.
L
The string is hung over a fixed circular frame of
radius R.
u

Q. 82 A light rigid rod has a bob of mass m attached to


one of its end. The other end of the rod is pivoted m M
O
so that the entire assembly can rotate freely in a
vertical plane. Initially, the rod is held vertical
as shown in the figure. From this position it is
allowed to fall.
(a) When the rod has rotated through q , Initially the particles lie at the ends of the
what kind of force does it experience– horizontal diameter of the circle (see figure).
compression or tension? Neglect friction.
4.14 Problems in Physics for JEE Advanced

(a) If the system is released, and if m remains mass at the level of the pulleys to be zero. Neglect
in contact with the circle, find the speed of dimension of the pulley and treat the strings to be
the masses when M has descended through a massless. Distance between the pulleys is 2d.
distance Rq q p . d
d
(b) Find the reaction force between the frame U=0
and m at this instant.
(c) Prove that m1 will certainly remain in contact
with the frame, just after the release, if
3m > M.
Q. 86 A small object is sliding on a smooth horizontal q q
floor along a vertical wall. The wall makes a
smooth turn by an angle q . Coefficient of friction A
C
between the wall and the block is µ. Speed of B
the object before the turn is u. Find its speed (V)
(a) Write the potential energy of the system as a
just after completing the turn. Does your answer
function of angle q.
depend on shape of the curve? [The turn is smooth
(b) Knowing that potential energy of the system
and there are no sharp corners.]
will be maximum or minimum in equilibrium
u position, find value of q for equilibrium.
 (c) Tell if the equilibrium is stable or unstable.
Q. 89 Three identical masses are attached to the ends
of light strings, the other ends of which are
connected together as shown in the figure. Each
V
of the three strings has a length of 3 m. The three
masses are dropped through three holes in a table
Q. 87 AB is a vertically suspended elastic cord of and the system is allowed to reach equilibrium.
negligible mass and length L. Its force constant A
is k = mg . There is a massless platform
L
attached to the lower end of the cord. A monkey
of mass m starts from top end A and slides down 1m 1m
g
the cord with a uniform acceleration of . Just Three holes
before landing on the platform, the monkey loses in the table
B 1 C
grip on the cord. After landing on the platform m
3
the monkey stays on it. Calculate the maximum
extension in the elastic cord. (a) What is total length of the strings lying on the
A table in equilibrium?
(b) Select a point K inside the DABC such that
AK + BK + CK is minimum, use the result
obtained in (a) and the fact that potential
L energy of the system will be minimum when
it is in equilibrium.
Q. 90 A particle of mass m is attached to an end of a
B
light rigid rod of length a. The other end of the
rod is fixed, so that the rod can rotate freely in
Level 3 vertical plane about its fixed end. The mass m is
given a horizontal velocity u at the lowest point.
Q. 88 In the arrangement shown in the fig. all the three
blocks have equal mass m. The length of the (a) Prove that when the radius to the mass makes
strings connecting A to C and B to C is L each. an angle q with the upward vertical the
Assume the gravitational potential energy of any horizontal component of the acceleration of
Work - Power - Energy 4.15

the mass (measured in direction of u) is O. The particle is projected horizontally with a


[g (2 + 3 cos q) – u / a] sin q2 velocity v0 from its lowest position A. When the
angular displacement of the string is more than
(b) If 4ag < u2 < 5ag, show that there are four 90°, the particle leaves the circular path at B. The
points at which horizontal component of string again becomes taut at C such that B,O,C are
acceleration is zero. Locate the points. collinear. Find v0 in terms of l and g.
Q. 91 A weightless rod of length l with a small load of
mass m at one of its end is held vertical with its
lower end hinged on a horizontal surface. The B
load touches a wedge of mass M in this position. O
A slight jerk towards right sets the system in
motion (see figure), with rod rotating freely in
vertical plane about its lower end. There is no C
friction.
m v0
A

Q. 94. The teeter toy consists of two identical weights


l hanging from a peg on dropping arms as shown.
M The arrangement is surprisingly stable. Let us
consider only oscillatory motion in the vertical
plane. Consider the peg and rods (connecting the
weights to the peg) to be very light. The length
(a) For what mass ratio will the rod form an of each rod is l and length of the peg is L. In the
angle q p with the vertical at the moment position shown the peg is vertical and the two
the load separates from the wedge? weights are in a position lower than the support
point of the peg. Angle a that the rods make with
(b) What is speed of the wedge at that moment? the peg remains fixed.
Neglect friction.
(a) Assuming the zero of gravitational potential
Q. 92 A tube of mass M hangs from a thread and two energy at the support point of the peg
balls of mass m slide inside it without friction evaluate the potential energy (U) when the
(see figure). The balls are released simultaneously peg is tilted to an angle q to the vertical. The
from the top of the ring and slide down on opposite tip of the peg does not move.
sides. q defines the positions of balls at any time
as shown in figure. (b) Knowing that U shall be minimum in stable
equilibrium position prove that q = 0 is the
stable equilibrium position for the toy if the
two weights are in a position lower than the
support point of the peg.
q q

a a
l l
L

(a) Show that ring will start to rise if .


(b) If M = 0, find the angle q at which the tube m m
begins to rise.
Q. 93 A heavy particle is attached to one end of a light
string of length l whose other end is fixed at
4.16 Problems in Physics for JEE Advanced

ANSWERS
1. (i) Akanksha is right. spring energy gets converted into the gravitational
(ii) (a) – mgh potential energy of beaker + water.
(b) 0
.
(c) internal (muscle) forces of the body
perform work
2. Zero (a)   (b) Yes.

(a) 960 J
(b) zero

20 m

(a) = ( + + )
. – 5 J

(b) = - + ( - ) (a) Same in both

(c)
W = 138J (b)

DUg = – 60 J
(d) h h1
6. No, the force is decreasing with time.
(a)

7.
(b)
The power will not change but work done will
decrease. (a)
(i) KE
(b) zero
25. (a) Block 2 has already crossed H.
(b) Both reach H with same speed.


PE

(ii) K in joule 27. k = 2.096 × 105 N/m


50
28.
10.0
6.4 V1
height in m
O 20 21.8
(a) Kmax = 125 J
(a) 53 KW
(b) Xmax = 7m
q = 1.43°
(b)
(a) No
UB = 50 J ; UD = –35 J
(b) Oscillations about x0
12. (b), (c)
0.5 m/s 32. (a)
When a small amount of water evaporates, the spring
relaxes a little bit. Water remaining in the beaker (b)
gains gravitational potential energy. Therefore, the
Work - Power - Energy 4.17

(i) V1 = V2 = V3 58. 4 m/s


Tension in segment (a) MV2
(b)
Tension in Segment
35. 2 60. (a)
36. The swing gets horizontal
37. tan–12 (b)
2 2
. mk r t
(a) 6 mg cos q 61.
(b)
. (a)
40.
(b) –r Ah2 g
41.
The positive sign of DU means external work will be
required to mix the two liquids uniformly.
42. (i) DU is negative in second case which means the
heavier liquid will automatically move to lower
43. side.
A is wrong. Under given conditions the two curves

cannot touch.
45. u= =
46.
(i) amin = g
48. (a) (ii) 2R
(b) Zero
66.
49. (a)

(b) 67.
68.
50. 2.5 m/s2 U

l . Work done is path independent and Uo


will be same for the alternative path Us

52. l

O X
53. x0
U
2mg
54. x0 =
K
Ug 2m2g2
55. (a) -Uo U0 =
K

(b) 69. 2 3 m/s

57. 70. - £ £ = -
4.18 Problems in Physics for JEE Advanced

82. (a) Compression


(a)
(b)
(b)
83. 35 N
(a) –9 Watt
72. (a)
(b)
(b)

Ê ˆ 85. (a)
(a) q = ÁË ˜¯

(b)
a t (b)

V = ue–mq0; No the answer does not depend on the
shape of the curve.

87.
q
O 
(c)
t1> t2
88. (a)
The string will break on hitting the second nail at C. q
(b)
T = 8.6 mg
(c) Stable
.
89. (a)
77.
KE T
The four points are represented by –
2.5 mgR 6 mg
q , , and

0.5 mgR
q q
O p 2p O p 2p
,
78. (a)

 1 92.
mg  3 + 
(b)
 2

79. u = 115 m /s

Ê ˆ (a) 2 mg cos q [L – l cos ]


80. q = ÁË ˜¯

81.
Work - Power - Energy 4.19

SOLUTIONS

1. (i) While walking the contact point of the shoes do not move relative to the ground, hence no work is done by the
friction. Internal forces of a system can always cause a change in KE. The muscle forces perform work which
results in change in KE.
(ii) Similar reasoning can be given for increase in PE of the monkey.
3. (a) The reference frame attached to the wedge is inertial.
In frame attached to the wedge acceleration of the block is
= q= ∞= = -

Direction of the acceleration is down the incline. Displacement in 2 s in this frame is

Displacement of the wedge


Displacement of the block in the ground frame is vector sum of Xw and Xo The component of resultant
displacement perpendicular to the incline (i.e., in the direction of normal force) is X0 sin q
Work done by N (= mg cos q) is
N
WN = mg.cos q X0 sin q


X0
(b) In the frame attached to the wedge, there in no
displacement in the direction of N.
\ WN = o
4. Work done by the force on the ball is Xw

Ê ˆ È ˘
= Ú ÁË - ˜¯ = Í - ˙
Î ˚
Work energy theorem

5. (a) let position vector of be

Position vector of B,
 
Net force on particle

\ Acceleration

       [Taking g = 10 m/s2]


  

4.20 Problems in Physics for JEE Advanced

È = ˘
Í ˙
ÎÍ = ˚˙




  
(b) Displacement


  

 
(c) = 138 J
(d) Change in potential energy = mg × (Vertical displacement)
= mg (–2) = – 60J
Q

6. Slope of the graph = a constant.

With increase in speed of the particle, the force must decrease so that P = Fv = a constant.
7. Fv = P

m v2dv = P dx

Integrating and simplifying gives

8. If the passenger moves up an escalator at a constant speed, the average force that he exerts on the staircase remains
unaltered equal to his weight. Therefore the force with which the motor must drive the staircase will also remain the
same. The power which has to be exerted by the motor remains the same. However a walking man will reach the
top of the escalator sooner and therefore the work done by the motor of the escalator in raising a moving man will
be less than that done in raising a stationary one (the part of the work is done by the man).
9. (ii) Mechanical energy at the time of projection

   = 10 + 40 = 50 J
Work - Power - Energy 4.21

y
H

Kinetic energy at height y is


K = E  –  mgy
K = 50 – 2y ………………….(i)
q ¥
= + = + = + =
¥ ¥

Graph is plot of equation (1)


K in J

A
50

C
10.0
6.4 B
O y
20 21.8
height in m

point A $ bottom of the building


point B $ highest point in trajectory
point C $ top of the building
If we follow the motion of the ball, the graph starts at C, moves to B and then moves from B to A.
10. Resistance force FR = (m + nV2)
Power = FR .V
\ (m + nV2) V = P
For V = 40 m/s
[m + n (40)2] × 40 = 53 × 103
& m + 1600 n = 1325 …………(1)
For V = 10 m/s
[m + n (10)2] × 10 = 2 × 103
m + 100n = 200 …………(2)
(1) - (2)
1500 n = 1125
n = 0.75
put in (2) m = 125
\ FR = 125 + 0.75 V2
(a)
P = FR.V
= [125 + 0.75 (40)2] 40 = 53000 W = 53 KW
4.22 Problems in Physics for JEE Advanced

(b) V
FR

mgsinq

q
(mg sin q FR)V = 69 × 103
mg sinq V + FRV = 69000
From last part of the question
FAV = 53000 W for V = 40 m/s

q in radian will be approximately

In degree = 1. 43°

11. Work done by a conservative force in a closed path is zero


\ WAB + WBC + WCD + WDA = 0
–50 + 25 + 60 + WDA = 0
& WDA = – 35 J
UB – UA = –WA"B
UB – O = 50 J & UB = 50 J
And UD – UA = – WA"D
  UD – O = – (+ 35) & UD = – 35 J
13. Work done by the force in position of the particle changing from x = 0 to x = 5.5 m is
W = area under the given graph

= 1.75 – 2.0 = – 0.25 J


From work – energy theorem
Kf – Ki = –  0.25
\ Ki = Kf + 0.25
Since Kf > 0
\ Ki > 0.25

& u > 0.5 m/s


15. If the height of the pan at an instant is l and the boy transfers a small mass m of the sand into it, he performs a work
= mg l (against gravity).
This work done increases the gravitational potential energy of the sand mass which loses a part of it in compressing
the spring.
Therefore, total work done by the boy will be finally found as gravitational potential energy of the sand plus the
Work - Power - Energy 4.23

elastic potential energy of the spring.


As per the question.

………………(i)

Work done

[using (1)]

16. (a) Initially, the COM of the snake is ∞= vertically below the top of the incline.

In final position, the COM is h below the top point, where

+ ∞
=

Centre of mass rises by

Work done against gravity

(b) If length were

17. Total mass of the system


The external agent must ensure that more than half the mass of the system gets to the right of the pulley. After that
he can leave the system. Ball will accelerate to the pulley.
Take PE to be zero at the horizontal level of the centre of the pulley.
Minimum work needed = Increase in gravitational PE of the system between two positions shown in fig. below.

L
4 L
2
L 3L
4

mass on both sides equal


4.24 Problems in Physics for JEE Advanced


È l l l ˘
= Í- - - ˙
Î ˚
18. Increase in gravitational PE = Loss in KE
mgh = 20 J

= =
¥

19. Hint: Change in spring PE = , when the spring compresses or stretches by x.


20. The block will finally come to rest when the extension in the spring is X0 such that
KX0 = mg sinq
¥ ¥
= =
The friction will dissipate all the energy possessed by the system initially, which is
E = mgh = 4 × 10 × (X.sinq)
\ Wf  =  – 5 J
21. (a) The friction force has same magnitude during up and down journey.

v=0

\ Work done by friction while going up is same as work done by it while coming down.

(b) Total loss of mechanical energy

\ m q = ...........….. (1)

Retardation during up journey = g (sin q + m cos q)


\ u2 = 2g (sin q + m cos q). L

Using (1) = q+

& ...........…..(2)

From (2) and (1)

q =
m

m= q
Work - Power - Energy 4.25

22. Hint: There is no change in speed of the tanker due to leakage of water.
23. (a) Let the stone reach a height h
Forces on stone are
Gravity W and
Air resistance R
Both act opposite to displacement during the upward motion.
\ Work done on stone
W R
Wo = Wg + WR
= –Wh – Rh [both W and R are constant force]
= –(W + R)h
Change in kinetic energy of stone in moving from ground to top of its path is
D k = kf – ki

From work energy theorem
Wo = D k

&

È W˘
& ÍQ m = g ˙
Î ˚
(b) For downward motion
R
Wg = Wh (positive)
WR = – Rh (negative)
D k = kf – ki


W
[Ki = KE at the top = 0]
\ From work energy theorem

&

Substituting for h

24. Consider zero of gravitational PE at initial position of the mass.


The string becomes slack when it regains its natural length.

4.26 Problems in Physics for JEE Advanced

Mechanical energy conservation:

l0

V
& 2l0 String slack

& =
Let the mass come to rest at height h above the initial position.
Energy conservation
Ug = 0
B

h = 0.5 m
Therefore, the mass will come to rest at point A.
26. When ball A is at its lowest position, let the stretch in the spring be x. At this instant tension acting on B is kx.
\ N = Mg – T
T
For N to be equal to ,
N
=
B
\ The spring must stretch by x such that
(when A comes to rest) Mg

Energy Conservation:
Loss in PE of A= gain in spring PE

&

kx  =  2mg

&

1 2
27. Wspring + Wthrust = mv + mgh
2

[180 kph = 50 m/s; g = 10 m/s2]


=2.096 × 105 N/m
Work - Power - Energy 4.27

28. N = mg – F sin q N F

\ f = mN = m (Mg – F sin q)

WE Theorem:

f
Mg

= Fd (m sin q + cos q) – mMgd

Maximum value of (m sin q + cos q) is m +


When tan q = m
\ = m + -m

= m + -m

È ˘
Í m +
= - m˙
Í ˙
ÍÎ ˙˚
29. Hint : Along path 1 the normal force is less and friction force is small. Along path 2 the normal force is high.
30. (a) U = 5x2 – 20x + 2
The variation of U with x is as shown.
U(J)

2 x
–3 O
– 18


The PE is minimum when
& 10x – 20 = 0
& x = 2m
At x = 2m, U = 5 (2)2 – 20 (2) + 2 = – 18 J
At x = –3m, U = 5 (– 3)2 – 20 (– 3) + 2 = 107 J
When particle is released at x = –3, it experiences a force in positive x direction. Its KE is maximum at x = 2 m
kmax + (– 18) = 107
& kmax = 125 J
(b) Total energy of the particle is 107J . When it is at rest its its KE = 0.
\ U = 107 & 5x­2 – 20x + 2 = 107
& x2 – 4 x – 21 = 0
& x = –3, 7 m \ xmax = 7 m
4.28 Problems in Physics for JEE Advanced

31. Hint: For reaching the origin the total mechanical energy of the particle must be greater than 6U0.
A minima of PE curve is the position of stable equilibrium and the particle performs oscillations about this position.
32. From P to Q the particle rotates in a circle of radius 3.5 L with speed V.
From Q to R it rotates in a circle of radius 2.5L (with centre at D ) with speed V .
From R to S it rotates in a circle of radius 1.5L with centre at C .
From S to U it goes in a circle of radius 0.5L with speed V .
S

P
U
A B

D
R
C

The speed of the particle does not change because the string tension force acting on it is always perpendicular to its
velocity. The power of the tension force is always zero. In absence of any work done on it the kinetic energy of the
particle does not change.

\
(a)

(b) Just before hitting the pillar, the particle is moving in a circle of radius 0.5 L with speed V

33. (i) E = PE at extreme position [since KE at extreme is zero]

\ E = mgh = mg L (1 – cosq
q0
 L

Q 
h


(ii) The total mechanical energy of the pendulum bob at any location is same. Therefore, energy conservation gives
V1 = V2 = V3.
Work - Power - Energy 4.29

34. Energy Conservation gives :


A

T2

wl C

T1

T1

wl B


Let T1 be tension in segment BC . For circular motion of mass at B
T1 – mg = mw2 (2l)

For circular motion of mass at C


T2 – T1 – mg = mw2l

35. Initially the bead will be in contact with the outer plate so that it receives a normal force towards centre (think of
tension in a pendulum). Since velocity at bottom is (which is less than ), there will be a point
(A) where bead will need no normal force (apart from weight) towards centre. Beyond A (up to B) it needs a radially
outward force. Hence, at A the bead falls on to the inner plate producing a clink (sound when two metal collide).
After B the bead will once again hit the outer plate. This produces the second sound.

B A
q q
O

Inner plate
Outer plate

36. Let the swing rise to an angle q


4.30 Problems in Physics for JEE Advanced

q q
L

F V
mg

0.5 + (1 – 0.5) = 1 – cos q


[Q q cosq ]
& cosq
&q
\ Swing will become horizontal.
O
37. At position q,
Radial acceleration of the particle is and q

Tangential acceleration is at = g sin q


ar
l V
As per the question 45°
a

& ........….... (1)


at
From conservation of mechanical energy, u

&

= 2g – 2g + 2g cos q
= 2g cos q … (2)
From (1) and (2). sin q = 2 cos q
& tan q & q = tan–12.
38. ac = k2rt2

Or, = k2rt2
Or, v = krt
Therefore, tangential acceleration, at = = kr
Or, Tangential force,
Ft = mat = mkr
Work - Power - Energy 4.31

Only tangential force performs work.


Power = Ft v = (mkr) (krt)
Or, Power = mk2 r2 t
39. (a) The difference in K.E. at positions A and B is

KA – KB = = mg (2L cosq) = 2mgL cosq .... (1)

TA = + mg cosq

TB = – mg cosq

\ TA – TB = + 2 mg cosq ....(2)

from equation (1) and (2)


TA – TB = 6 mg cos q
(b) The component of accelerations of ball at A and B are as shown in figure. B
ar

&   O
g sin q
A

= = + q g sin q
40. In extreme position, total acceleration is equal to tangential acceleration. This is because the radial acceleration is
zero. It means total acceleration is g sin q.
At mean position (lowest position), total acceleration is the radial acceleration = l as tangential acceleration is
zero.
Conservation of energy gives

= ( - q)

& = ( - q)

& g sin q = 2 g (1 – cos q)


q q q
& =

q -
& = fi q=

41. Let velocity of particle at point P be v.


From conservation of mechanical energy A q B
h
R
mv2 = K = mgh v P

Let N be the normal reaction between the particle and the pipe at this instant. Then

N – mg sinq =
4.32 Problems in Physics for JEE Advanced

But, = and

Hence , N = mg + = +

(Q K = mgh)
Hence, N = = force on the pipe.

42. (i)

Fext
O
N
q qo R

mg
cos
f q h

mg
sin mg
q

Forces acting at position q have been shown.


Fext = mg sin q + f
= mg sin q + m mg cos q
Work done in small angular displacement dq is dWext = Fext.Rdq
q

Ú( q)
\ = q+m q

From geometry,

And

È - ˘
\ = Í - ˙+m -
Î ˚


(ii) In the reference frame of the ground, the normal force of the track does not perform any work on the block. But
in the reference frame of the moving observer, the normal force does perform negative work on the block.
Work - Power - Energy 4.33

43. Mass of water in unit length of the cylinder is x


The water level decreases at a constant rate
dx

=

Level falls by x at time

Mass of a length dx of water . =


If this dx length of water is transferred to the wide container in time dt then

44. l
B
N
and l
l
Mass of BC, l
l
C
A
Mass of AB, l
l q
Mg
mg
Since chain is on verge of sliding, friction is at its limiting value
f = mN = m mg cosq
m = (1 – h) mo g cos q
Chain will just start sliding if
Net driving force = Net resisting force
& Mg = mg sin q + f
& hmog = (1 – h) mog sin q + m(1 – h) mog cos q

Now as the chain moves, friction force changes due to change in N.


We consider a situation when BC = x
And AB = l – x
Mass of


-
=m q
4.34 Problems in Physics for JEE Advanced

If the chain moves by a distance dx


[displacement (dx) and friction (f) are oppositely directed. ]

m m0 g cos q l

\ Wf = -
l Ú (l - x)dx
hl

[Q x = length of BC changes from hl to l]

m m0 g cos q l 2
=- (1 - h)2
l 2

  


45. Velocity of disc relative to the board is

\ Friction force acting on the board will be along while that acting on the disc will be along

.
Friction force mmg = 0.2 × 2 × 10 = 4 N


Friction on board


\ External force on the board

Power of this force = =


Friction force on the disc


External force on the disc
 
Power

Heat dissipated = -
+ =
46. The important thing to notice is that car engine can develop a certain maximum power (Pmax)
When car moves up with trailer the net force against the motion is F = 3 mg sin q R
Where R = resistance to motion = k(3m)V2 & [k = a constant]
Work - Power - Energy 4.35

q
in
gs q 3 mg
3m

\ F = 3 mg sinq kmV2
\ Pmax = F.V = ( 3mg sinq kmV2)V
When moving without trailer
Pmax = 2V (mg sin q + km (2V)2)
= 2V (mg sin q + 4 km V2)
\ (3 mg sinq + 3 km V2) V = 2V (mg sin q + 4km V2)

……………………. (a)

When car moves down the slope, with engine shut off, initially it accelerated due to mg sin q, R being small due
to small speed. But as the speed grows, R increases and eventually R becomes equal to mg sin q. After this car
experience no acceleration and moves with constant speed.

q
sin
mg mg

Let this constant speed be u.
Then kmu2 = mg sin q
q
= = ÎÈ ( )˚˘

47. Work done by the force for a displacement is given by performing following integration with limit changing from
initial to final position

W=

= (dX î + dY )

Taking : X2 + Y2 = t
2XdX + 2YdY = dt

& XdX + YdY =


4.36 Problems in Physics for JEE Advanced

& W=

This integral can be evaluated easily if the co ordinates of initial and final position are known. In order to evaluate
the integral we need not know the path taken by the particle. It means work done is path independent. Hence the
force is conservative.
48. (a) U = 2 (x + y)
= - ∂ = -

= - ∂ = -


Conservative force is
To keep the particle moving uniformly on a circle, net force is centripetal force. When the particle is at (0, 4)
net force acting on it is
y
V

F0
X
O

¥
= (- ) =-

+ =

=- (
- - - )

Wext = DK + DU
(b)
= 0 + U (at 4, 0) – U (at 0, 4)
= 8 – 8 = 0.


49.





(a) When power of the force is zero


 

 
fi ◊ =
& bk2 sin kt (bk – bk cos kt) – bk2 cos kt (bk – bk sin kt) = 0
Work - Power - Energy 4.37

& b2k3 sin kt = b2k3 cos kt)


& tan kt = 1
;

(b) Velocity at



ÈÊ ˆ Ê ˆ ˘
= ( ) ÍÁ - ˜¯ + ÁË - ˜¯ ˙
ÍÎË ˙˚

Velocity at

=
\ V2 = 2bk

(+ )
= ( ) È -
Î ( - )˘˚ =
50. Elongation in the spring is x = xA– xB
Where xA= displacement of end A measured from original natural length position
xB =displacement of block measured from original position.

= 6 – 3 = 3 m/s

Potential energy in the spring =

\ acceleration of the block

51. Work done by friction can be calculated as–

ds
} dh
f dl

Consider a small displacement ds of the body at an intermediate point where slope angle of hill is q. (see fig)
Force of friction is f = mmg cos q (down the slope)
\ Work done by friction in small displacement ds is dWf = – f ds = – mmg (ds cos q)
4.38 Problems in Physics for JEE Advanced


=-m [ = q ]
l l

Since the body is hauled slowly, at each point its velocity is nearly zero.
\ Dk = 0
But WN + Wmg + Wf + WF = Dk = 0 ……………….(a)
& WF = – (WN + Wmg + Wf )
= (0 – mgh – µmgl) = mg (h + ml)
52. Body acquires a velocity means it is accelerated during the course of its motion.
However, WN ,Wmg and Wf remain same as in previous question.
\ From equation (a) of previous solution.

Q

fi = - [- -m l]

53. Observe the system from a reference frame moving to right with velocity u. In this frame the free end of the spring
remains as rest (as if tied to a wall) and the initial velocity of the block is u to left. Hence, maximum
extension (or compression) is given by

54. Let the kinetic friction force on the block be f


From work energy theorem:
Wf = DU + DK
Ê ˆ
- =Á - ˜¯ +
Ë

As the block is released from extreme position it accelerates because the spring force exceeds the friction.
At position where extension in the spring is , the friction force equals the spring force. After this the friction

force will exceed the spring force and the block will retard.

\ Speed is maximum when extension = . Again using work energy theorem gives-
Wf = DU + DK

\- =- +
Work - Power - Energy 4.39

55. (a)

Natural V0
length
x0

3x0

Equilibrium (B) (C)


(A)
In equilibrium, the spring is compressed by [fig. (A)]

The system is released from position shown in fig (B). Compression in the spring at this position is 4x0. The
upper block will lose contact with the lower block when the spring acquires its natural length position. [Why?]

Hence, answer to first part of the question is

(b) Let’s apply law of conservation of mechanical energy to get the speed of the block when the spring reaches its
natural length.

Further height attained by the block of mass 2M is given by

\ Height attained above point of release is

56. (a) Initial PE is spring =

= ¥ ¥( ) = =

Work done by friction when the two blocks get displaced by x is
Wf = f.x = – 0.2 × 2 × 10 × x = – 4x
Till the spring returns to natural length the work done by friction will be
Wf = – 4 × 0.1 = – 0.4J
But energy available for dissipation is 0.33 J only.
\ Blocks will stop before the spring returns to its natural length.
4.40 Problems in Physics for JEE Advanced

KE will be increasing till the spring force exceeds friction.


It will be maximum where
kx = m (2mg) [x = compression in spring]

x = 0.06 m
\ x0 = 0.1 – 0.06 = 0.04 m = 4 cm
let x = compression when the two blocks come to rest.

ÈÎ - ˘ = m( )( - )
˚
Solving for x we get
x = 0.02 m = 2 cm
In this position, spring force on block A is

The maximum static friction on A can be


mmg = 0.2 × 1 × 10 = 2 N
Hence, friction on A will adjust to and balance the spring force. There will be no contact force between A
and B.
(b) As in (a) you can check that the blocks will cross the natural length position of the spring. Just when the spring
starts getting stretched retardation of A will exceed the retardation of B and the two blocks will get separated.
57.

kx0 S1 X0

kx0 S2 A
2 M
X
B

When spring S1 Stretches by x0, tension in it is kx0

At this instant tension in S2 shall be


It means S2 is stretched by .

When S1 and S2 stretch by x0 and , the ball A will fall through a distance

= + = ..................(1)

[we are assuming that B does not move]


If A falls through x before coming to rest,

\ Spring force on
Work - Power - Energy 4.41

B will just leave the table at this instant if

..................(2)

When ball A falls through x (before coming to rest) principle of conservation of energy says loss in PE of A = Gain
in spring PE

& kx = 10 mg
From (2) 5 Mg = 10 mg

58. Spring force is maximum when the system is released


F S max = kx0 = 240 N
The limiting friction on A can be
FlA = m1MA g = 250 N
Hence, block A remains fixed and does not move at all.
The limiting friction on C can be
Flc = m2MC g
Thus maximum acceleration that friction can provide to C is
acmax = m2g = 8.5 m/s2
Just after the release spring force is maximum and it will cause maximum acceleration in B. Let us assume that there
is no slipping between B and C. In that case the maximum acceleration is

= =
+
Friction can easily provide this acceleration to block C hence it will not slip over B.
Speed is maximum when the spring acquires its natural length.


& vmax = 4 m/s
59. (a) Friction force between the block and the plank = f
f M
f
F
V

Acceleration of the block – .

If the slipping stops in time ‘t’


V = 0 + at

...................(1)

For the plank external force F = friction (f)


Power of the external force P = FV
4.42 Problems in Physics for JEE Advanced

Work done by external force in time ‘t’


W = Pt = FVt = ftV = MV2 [Q ft = MV from (1)]
(b) The work done by the external force can be interpreted as
W = Heat produced + gain in kE of the block.
\ = - =

Note: In the reference frame of the plank the block has initial velocity V and final velocity zero. It has lost
amount of KE. This energy has been lost as heat.

60. Maximum friction on m1 is f1max = mm1g


m
\ Extension in the spring needed to move m1 is =
Therefore, F shall be such that extension in the spring does not exceed x0. Let F0 be the force for which the block
of mass M2 moves through x0 before coming to rest.
WE theorem
=

m m
= = =

(a) Speed of m2 is maximum when it reaches the equilibrium position.


kx = f0

WE Theorem
1 1
F0 . x = k x 2 + m2V 2
2 2

61. In equilibrium the spring is stretched by = , where W = weight of the block.


Let the spring break when it is stretched further by x

N.L
x0

Fspring x

Breaking
Position

W F0
Work - Power - Energy 4.43

k (x + x0) = F0 + W ..................(1)
When a constant force is applied, the block starts to accelerate and it gains speed. It will not come to rest at position
where k (x + x0) = W + F. It will be having a speed at that point and will move further down. The
spring can break only if the spring stretches further by x before the block stops

N.L

X0
Start
Position
X

Block stops
and spring just breaks

W F

Applying work energy theorem between starting position and the position where the block stops.

From (1) kx + kx0 = F0 + W and kx0 = W

62. (a) Assuming the reference level at the bottom of the container

= r +r = r

If the liquids mix uniformly the entire mixture will have a density of and the final PE will be

= r = r

Change in PE, DU = - = r

(b) In this case, liquid A will settle at the bottom and B will be above it. Final PE will be

Change in PE, DU = - = r - r = -r

The positive sign of DU in first case tells us that some external work will be required to mix the two liquids
uniformly.
4.44 Problems in Physics for JEE Advanced

In second case DU is negative. The heavier liquid will automatically move to lower side as this decreases the
overall PE.

63. = + - q

È ˘
= = Í q - ˙= q-
Î ˚

= fi - q+ =

Discriminant of this equation is – ve for q  = 30º


\ the two curves do not touch u u
64. v cos q = u sin q
..........…(i)

mg l cosq = mu2 + ..........…(ii) q

l l v
u=
65. (i) In reference frame of the truck there are two forces on the block which perform work on it –
The pseudo force and
Gravitational force
R
O
P

R
ma

mg

Applying work energy theorem for the situation when the block just manages to reach the top of the hemisphere
(change in KE = 0)
maR – mgR = 0
a=g
(ii) Once again applying work energy theorem in reference frame of the truck

Where v is vertically upward velocity of the block in reference frame of the truck at the instant it leaves the
truck.
Given a = 2g
Work - Power - Energy 4.45

Hence, =

Which means the block will rise further by R after leaving the truck.
Therefore, answer is 2R
66. In a reference frame attached to the wire, the force on the bead has been shown in the fig.
masin  o
N

ma

mg
co
mg

s
qo

qo
A C B

For equilibrium ma sinq0 = mg cosq0 ....................(1)


If q increases a little bit the tangential force ma sin q will rise and mg cos q will decrease. Therefore, the bead will
rise on the ring and then it will fall on other side to hit the table at end A.
Work done by the pseudo force = ma [R + R cosq0]
work done by mg is = mg R sin q
\ = [ + q ]+ q

v2 = 2aR (1 + cos q0) + 2gR sin q0

Ê + q ˆ
\ = ÁË q ˜¯

67.

q
l T
q

A B
mg

The block will leave the horizontal surface if it can reach a point B where
È = ˘
T cosq mg..................(i) Í ˙
Î = ˚
But T = kx = kl [secq – 1] [x = extension in spring = l(sec q –1)]
Using (i) l [at B]
4.46 Problems in Physics for JEE Advanced

& q = 60°
From work energy theorem
Work done by friction = Dk +DU

-m l ∞=- + ÈÎl ( ∞ - )˘˚


l l l l l
l

l
68. The maximum compression in the spring can be calculated as

Ê ˆ
=- ( ) =- ÁË ˜¯ = -

Ê ˆ
And = ( ) = ÁË ˜¯ =


Graph has been shown in the answer.

69. = fi = fi =
Note that energy to be given is 6 J and NOT 10 J. Why?

70. Putting a =1 and b = 1


U = x4 – 5x2

= = = =-

At x = 0; U = 0
at x = 0; U = 0
U (J)
at = = - =-

5 25
x=- ; U = - J = U min
2 4
Graph of U vs x is as shown in the fig.
– 5/2 5/2
O X
– 5 5

-25
Since K + U = 0 4
Work - Power - Energy 4.47

\ particle will remain between

5
Vmax = m/s
2
71. (a) Force on the particle,

The – ve sign indicates attractive force.


For the motion to be circular

& ..............… (i)


For r = a0

\ KE

p p p
(b) Time period, = = =

72. (a) In reference frame of bowl, the block acquires a velocity V , in horizontal direction.
N
V

mg

V

R.F. of bowl
Block will lose contact (N = 0) if the force mg is not good enough to provide sharp enough turning to the path
of the block.

V > Rg
(b) In reference frame of the bowl, we use work energy theorem for motion of the block
N

ma

v q
mg

O
4.48 Problems in Physics for JEE Advanced

For small q, sin q = q and cos q = 1

\ ..............… (1)

Also 

= mg – maq – 2maq
For N > 0
mg > 3maq

&

73. (a) Energy conservation

But as per question

and

&

&

(b) " increases with time. The graph is as shown below.


at
at

g
g

A2
A1

 O t
/2 t1 t2
Work - Power - Energy 4.49

(c) Area A1 under the graph gives change in magnitude of velocity from A to B

\
Similarly, A2 = change in magnitude of velocity from B to C .

For A1 = A2, looking at the graph one can easily say that
t1 > t2
74. Particle is projected from P and follows a circular path C1 till it reaches Q .

S C2

q V1
V2 C
C3
0.4R q
B
A Q
0.3 R

C1

u = 5gR
p
At Q (just before the thread hits the nail at B ) its speed is V1 .

&

Tension at this moment is


Just after the thread hits the nail at B, particle starts moving on circle C2 (centered at B )
Tension at Q just after the thread hits the nail at B is

% change in tension is

The particle will be at point S when the thread hits the nail at C .
Energy Conservation between S and Q

[Q ; CS = 0.7 R – 0.5 R = 0.2 R]

\
4.50 Problems in Physics for JEE Advanced

Tension just before the string hits the nail C is

Just after the string hits the nail, particle moves on a circle of radius 0.2R

= 8.6 mg
% change in tension

Hence, the string breaks.


75. (a) The centre of the ball moves in a circle of radius .

v

sq
co
mg

mg

From figure ............… (a)

From conservation of energy …................ (b)

Eliminating v2 from (a) and (b) we get


N = mg(3 cosq – 2) …......... (c)
Initially, for small q , (3 cosq – 2) > 0 and N is positive.
This means ball is in contact with the sphere A which exerts an outward normal force.

When ; N = 0 and ball loses contact with A.

Naturally, it comes in contact with B and direction of N gets reversed. In fact, this is envisaged in the equation (c)
as N is < 0 for
Work - Power - Energy 4.51

\ NA = mg(3 cosq –2) for

NB = mg(2 – 3 cosq for

(b) The two graphs look as shown in figure


NA
N
mg 5 mg

mg

cos q
O 2/3 1 2/3
cos q –1

Note that when q ° (cosq


NA = mg
And, when q (cosq –1)
NB = 5 mg
76. Let the string slack at point D (see figure).
A
v
E

F 90–q D
q

q
O

C u
B

Particle moves in a circular path from C to D. At D it leaves the circular path and follows a parabolic path (a projectile).
It crosses the line AB, when it is moving horizontally. It means while crossing AB at point E it is at the top of the
parabolic path.
At D , string tension = 0

\        ...............(a)

[v = velocity at D]
From conservation of energy

v2 = u2 – 2gL (1 + sin q) ..................(b)

Horizontal range of projectile (starting from D at an angle (90 – q with horizontal)


4.52 Problems in Physics for JEE Advanced

[From formula for range]

From geometry of figure

&

& [Q from (a), = q]

&

& &

& &q

Now from (a)

Substituting in (b) u2 = v2 + 2gL (1 + sinq

77. At position q, let the speed be V and tension T.

v
R
 T
R mg cos q

mg
u

Energy conservation + ( q )=

     ….............(1)

[Q = ]
Work - Power - Energy 4.53

\ Kinetic energy is

Equation for force in radial direction

T = mg cosq mg + 2mg cosq [using (1) to get the value of ]


T = 3mg(1 + cosq ..............… (2)
The graphical plots are
KE T

2.5 mgR 6 mg

0.5 mgR
O q q
p 2p O p 2p

78. The desired position is shown.

O C
B
q

u A

q
p' D
q = p/3
p

Arc

The vertical height difference between p' and p is


h = R – CD
È ˘
=p Í q+ p q˙
Î ˚

\
4.54 Problems in Physics for JEE Advanced

(b) Writing equation for centripetal force

79. When train is at rest


[L = length of pendulum]

&

When the train is moving horizontally at an acceleration ‘a’ , we can assume that inside the train effective acceleration
due to gravity is
a = 7.5

g = 10
g eff
VA
= 12.5 m/s2
A
TA

37°
mg eff
O

q
In accelerated train, minimum tension will be at point A.
g eff
u
p

In limiting case TA = 0

.......….....(1)

Applying energy conservation between points P and A


Work - Power - Energy 4.55

80.
A R O
V


B
x

v = speed with which the block leaves the surface at B

= q

v2 = 2gR cos q ............… (1)


Range of the projectile released from B is

q
= = q q    [from (1)]

= 4 R cos2 q. sin q
x is maximum when

& – 2 cos q sin2 q + cos3 q = 0

& cos q [cos2 q – 2 sin2 q] = 0


cosq is not acceptable [It will give zero range]
\ cos q – 2 sin2 q = 0
2

81. Let the particle leave the circular path at an angle q (as shown)

T V

q
R P
Q
q

Energy conservation

= [ + q]
4.56 Problems in Physics for JEE Advanced

& = [ + q ] .............… (1)

Centripetal force [with tension becoming zero]

= q ...............… (2)

From (1) and (2) g cos q = 4g – 2g – 2g cos q

q=

From (2)

After this the particle goes in a parabolic path. Tangential acceleration becomes zero when it is at the top most point
of its trajectory (at point T)
Required distance is RQ = L sin q – Half the range of projectile.

q q
=

82. (a) T

°
30
s
co

g
m R
30° V

mg


When q
Let the force exerted by the rod on the bob be T as shown.

Then q = .............… (i)

Where R = length of the rod


From conservation of energy

&

&
Work - Power - Energy 4.57

From (i)

&

\ Rod exerts a force T on the bob radially outwards.


\ Force exerted on the rod by the bob is radially inward, i.e., compressive.
(b)

sq
V

co


g
m
mg

At the point where compression changes to tension, we have


T=0

& V2 = gR cos q …........... (i)


From Energy conservation

& gR cos q = 2gR (1 – cos q)

& cosq &

83. The bob will experience free fall for a distance of L = 1.8 m.
Speed of the bob just before the string gets taut is

L O

30° 60°
O
30°
L L
L cos 60°=
2

ur ut
ut
u
V
4.58 Problems in Physics for JEE Advanced

As the string gets taut, the radial component of velocity vanishes.


Only the tangential component will remain.


Energy conservation

V2 = 27 + 10 × 1.8 = 45

84. (a) Mean or average power

Time of motion can be calculated as


v = u + at
& 0 = v0 – mgt    [Q f = mmg \ a = mg]

&

Work done by friction = loss in kinetic energy [whole kinetic energy is lost in doing work against friction]

\ Work done by friction, =

m
\ < >=- = m = ¥ ¥ ¥ ¥ = watt
(b) When body is at a distance x from the starting point, retarding friction force is
f = mmg

\ = -m

\ = -m

\ = -m

Ú = -a Ú

-
& = -a

\ = -a
Work - Power - Energy 4.59

& = -a

Instantaneous power of friction force is


 

= f v cos 180°

= -m -a

= -a -a

p is maximum when

È - a ˘
& -a Í -a + ˙=
Í -a ˙
Î ˚

& -a -a =

&

&

Ê ˆ
\ = -a Á ˜ -a
Ë a ¯ a

   =- a
85. (a) When M descends through Rq , radius vector of m rotates through q as shown in figure. The normal contact
force does not perform any work. Only other force is gravity (which is conservative).

Rq
R sin q

Rq

\ loss in PE = gain in KE

Loss in PE of M–gain in PE of m

(Since speed of both M and m will be same.)


\
4.60 Problems in Physics for JEE Advanced

(b) Consider the motion of m at this instant.


N V

mg sin q
q

mg

= q-

& = q-

q- q
= q-
+
È + q- q+ q ˘
= Í ˙
Î + ˚

=
+
[ + q- q]

(c) Just after release


q Æ
sin q = q

\ =
+
[ + q- q] = [ - q]
+
For contact to be maintained N > 0
& 3m > M
86. Consider the object at point P on the curve. Let its velocity at this point be V making an angle q with the original
direction. Let the centre of curvature of the path at this point be O and the radius of curvature be R.
f

N V
R
q dq

The normal force is


Work - Power - Energy 4.61

Kinetic friction force on the object is


Work done by the friction in small angular displacement dq of the block is dWf = – f (Rdq) = –mmV2dq
From work energy theorem, work done by the friction is equal to change in KE of the object

dK = mV dV

\ mVdV = –mmV2dq

& &

& &
- mq
& =

& V = ue –mq0
Answer does not depend on the shape of the curve.
87. Monkey slides down with a constant acceleration of . It means the tension in the elastic cord just before the
monkey lands on the platform is .

Extension in the cord by this time = x0

Then

Elastic potential energy stored in the cord

Kinetic Energy of monkey just at the time of hitting the platform


[Q monkey accelerates over a length of ]

\ =

Let the cord stretch further by x.


Applying energy conservation
Loss in KE + loss in gravitational PE = gain in spring PE


4.62 Problems in Physics for JEE Advanced

& 9L2 + 16 Lx = 32 x2 + 8L.x

& 32x2 – 8 Lx – 9 L2 = 0

      [–ve sign is unacceptable]

\ Maximum extension
88. d d

q q

C
A m B
m m

(a) PE of the system in position shown is


U = – mg d cot q – 2 mg [L – d cosec q]
  = – 2mgL – mgd [cot q – 2 cosec q]
È q- ˘
=- - Í
  Î q ˙˚

È - q˘
=- + Í
  Î q ˙˚

(b)
U is maximum or minimum when

q + q - - q q
q = & ◊ =
q
& q + q - - q q=

& q+ q- q=

& q= & q= ∞
Work - Power - Energy 4.63

\ Equilibrium is at q = ∞

You can verify this using simple force diagram.

(c) If > , equilibrium is stable


q

If < , equilibrium is unstable


q
Ê - qˆ
= ÁË ˜
q q ¯
È q q - - q q q ˘
\ 2
= ◊Í ˙
q ÎÍ q ˚˙
At q = 60°

È ˆ˘
Í ◊ ◊ Ê ˆÊ
-Á - ◊ ˜Á ◊ ◊ ˜˙
Í Ë ¯Ë ¯˙
\ = ◊Í ˙
q2 Í Ê ˆ ˙
Í Á ˜ ˙
ÍÎ Ë ¯ ˙˚
\ >
q2

\ q = 60° is position of stable equilibrium

89. (a) If m is mass of each particle, tension in each string is T = mg.


For equilibrium of the knot it is necessary that the three strings form 120° angle with each other. The situation
has been shown in figure.
A, B and C are holes and K is the knot.
A

T
120

1m 1m
12

T K T

D
B 1 C
m
3

\
4.64 Problems in Physics for JEE Advanced

And

\ length of string on the table is


(b) The point K shown in the figure above is the desired point. The equilibrium position is the one with the lowest
potential energy of masses, that is the one with the most string hanging below the table. In other words, in
equilibrium least length of the strings will be lying on the table.
90. Let P be the point where rod makes an angle q with the upward vertical. v is velocity of mass at this point.
v

sq
q
mg sin q

co
O mg
mg

u
A

T is force exerted y rod on the particle towards centre. By conservation of mechanical energy

& v2 = u2 – 2ag (1 + cosq) ................ (a)

At point P

Radial acceleration = = - + q

Tangential acceleration at = g sin q (in the direction shown)

ar sin q at sin q

q ar
at

Resolving ar and at in horizontal directions, we get the horizontal component of acceleration (in the direction of u)
as
aH = at cos q – ar sin q

= q◊ q- q+ + q q
Work - Power - Energy 4.65

È ˘
=Í q+ - ˙ q
ÍÎ ˙˚
(b) When aH = 0
È ˘
qÍ ( q+ )- ˙=
ÍÎ ˙˚
& sin q & q or

Also when q+ - =

-
q= ..............… (b)

Since 4 ag < u2 < 5 ag ; (u2 – 2 ag) < 3 ag and a real value of q exists satisfying (b)

-
Ê - ˆ
\q= Á ˜
Ë ¯
Ê - -
ˆ
and q = p - Á ˜
Ë ¯
Thus, horizontal acceleration is zero at four points given by

q = 0, , and

Figure shows the four points A


D C

a a

B
 
( 
)
Note: If u2 > 5ag, then

As

Thus, only two solutions result, q and p.


91. What is the physical condition for the load getting separated from the wedge?

q
M

O
4.66 Problems in Physics for JEE Advanced

As the mass m moves in a vertical circle, it pushes the wedge towards right. At any moment, the horizontal component
of displacement of m and M are equal. This means, the horizontal component of velocity of m is equal to the
velocity of M. However, at a certain value of q , the horizontal component of acceleration of m becomes zero and
thereafter becomes negative. From this point onwards the horizontal component of velocity of m starts decreasing.
But the velocity acquired by M cannot decrease as there are no retarding forces on it. Thus, the two masses get
separated at the moment the horizontal acceleration of m changes from positive to negative (i.e., becomes zero).
\ Condition when contact is lost is
ax = horizontal acceleration of m = 0
At the moment contact is lost, let the velocity of m be v.
Velocity of M will be equal to horizontal component of velocity of m .

v cos 

l
v
q v sin 

X
O

\ Velocity of M is vx = v cosq
As there are no dissipative forces, mechanical energy of the system is conserved.
\ loss in P.E of m = gain in K.E of m + gain in K.E. of M

\ - q = + q

- q
& =      ...........… (a)
+ q
Now the mass m has two accelerations

Radial
Tangential at = g sin q
[Note that contact force N, between m and M is zero at the instant contact is lost. Therefore, force in tangential
direction is only mg sinq .]
The acceleration of mass m in horizontal direction is

= q◊ q- q    [See figure.]

v2
sin q g sin q cos q
l

g sin q
v2
l
q

For loosing contact ax = 0


- q q
\ q◊ q- =    [using (a)]
+ q
Work - Power - Energy 4.67

& 2 mg – 2mg cos q – mg cos q – Mg cos3 q = 0     [sinq is not acceptable]


It is given that this happens when q = p/3
p
\ - ¥ = p

&

&

& M = 4m

(b) from (a)


- p
= =
+ p

\ Speed of M at the instant is

vx = v cos = p

92. Why at all the tube will rise? As the balls slide down, N
N cos q N cos q
N Force on
tube wall
initially they are in contact with inner wall of the tube and
at a certain point they leave the inner circle and get in touch N sin 
N sin q
with outer circle, so that normal reaction is inward on the
balls.  N
R
The balls exert an equal and opposite force on the outer wall
of tube (see figure)
When 2N cos q Mg (weight of tube), the tube will begin to
rise. mg
Considering the dynamics of ball

= q+ …........... (a)

From conservation of energy

= mgR (1 – cos q) .......…..... (b) [For each ball]

From (a) and (b), eliminating v2, we get


N = mg (2 – 3 cos q)
For tube to rise
2 N cos q > Mg
2 m cos q (2 – 3 cos q) > M .....................(c)
For the tube to just start rising, the maximum value of
Z = cos q (2 – 3 cos q) must satisfy the above inequality.
Z is maximum if


4.68 Problems in Physics for JEE Advanced

&

\ from (c)

& for the tube to rise.

(b) Equation (c) gives the value of q .


When M = 0,
2 m cos q (2 – 3 cos q) = 0     [In limiting case]
& cosq

93.

v0
A

Let v = speed at B.
Tension in the string at B = 0

& v2 = gl cos q ............… (a)


After B , the motion of particle is that of a projectile . One can consider it a projectile thrown down the incline BOC.
The particle hits the incline at C .
v

q
B

b  q
C

\ Range along the incline = BO + OC = 2l


Incline angle b –q
For such a projectile, you can prove that the range along the incline will be
q◊ ∞ q
= 2l cot2 q
q
Work - Power - Energy 4.69

\ 2l = 2l cot2 q
\ cot2 q
Or, q
Now we can use conservation of energy between points A and B , which gives

= + + q

& = + + q
Using (a)
= q+

[Q q ]

94. As shown in sketch. If we take the zero of gravitational potential energy at the pivot (point O), we have
U(q) = mg [L cos q – l cos (a + q)] + mg [L cos q – l cos (a – q)]
= 2 mg cos q [L – l cos a]

l cos (a+q) l
L l l cos (a–q)
Lcosq
m
O m
For equilibrium,

=- q (L – l cos a) = 0
q
& sin q = 0 or q = 0, as we expect from symmetry.
To investigate the stability of the equilibrium position, we must examine the second derivative of the potential
energy. We have

=- q [L – l cos a]
q
At equilibrium, =- a [L – l cos a]
q q=

For the second derivative to be positive, we have L – l cos a < 0 or L < l cos a.
In order for the teeter toy to be stable, the weights must hang below pivot.
05 MOMENTUM AND CENTER OF MASS

(b) Which force performed greater work?


LEVEL 1
Q. 4. An observer O1 standing on ground finds
Q. 1. A particle is acted upon by a force for 1 second that momentum of a projectile of mass 2 kg

whose X component remains constant at Fx =
30 N but y and z components vary with time as
(
changes with time as P01 = 4t iˆ + 20 t kˆ kg m/s )
shown in the graph. Calculate the magnitude of

Acceleration due to gravity is g = 10 kˆ m/s2 ( )
change in momentum of the particle in 1 s. What and there is a wind blowing in horizontal direction.
angle does the change in momentum P make Another observer O2 driving a car observes that
with X axis? momentum of the same projectile changes with
(N) time as -

80 ( )
P02 = 8t iˆ − 16t 2 ˆj + 20t kˆ kg m/s. Find the ac-
FZ
1
celeration of the car at t = s
40 8
Fy Q. 5. Water flows through a tube assembly as shown
t (s)
in the fig. Speed of flow (marked as V and 2V),
O 1 cross sectional area (A, A/2 and A/4) and the
Q. 2. Two block A and B of equal mass are connected angles between segments has been shown in fig.
using a light inextensible string passing over two Calculate the force applied by the water flow on
light smooth pulleys fixed to the blocks (see fig). the tube. Take density of water to be r.
The horizontal surface is smooth. Every segment A/2
of the string (that is not touching the pulley) is
V
horizontal. When a horizontal force F1 is applied 0
to A the magnitude of momentum of the system, 60
A V
cos
–1 3
comprising of A + B, changes at a rate R. When a
4
horizontal force F2 is applied to B (F1 not applied)
the magnitude of momentum of the system A + B A 2V
4
once again changes at the rate R. Which force is
larger - F1 or F2? Q. 6. A man is running along a road with speed u.
A B On his chest there is a paper of mass m and area
F1
m m F2
S. There is a wind blowing against the man at
speed V. Density of air is r. Assume that the air
molecules after striking the paper come to rest
Q. 3. A particle of mass m = 1 kg is moving in space in relative to the man. Find the minimum coefficient
X direction with a velocity of 10 ms–1. A 4 N force of friction between the paper and the chest so that
acting in Y direction is applied on it for a time the paper does not fall?
interval of 5.0 s. Later a 5 N force was applied on
it in Z direction for 4.0 s Q. 7. Two particles A and B of mass 2 m and m
(a) Calculate the total work done by both the respectively attract each other by mutual
forces. gravitational force and no other force acts on
5.2 Problems in Physics for JEE Advanced

them. At time t = 0, A was observed to be at rest A


and B was moving away from A with a speed u. At u
a later time t it was observed that B was moving
B
towards A with speed u. Assume no collision
has taken place by then. Find work done by the
gravitational force in the time interval 0 to t wall
Q. 8. (i) A block of mass m moving towards right
with a velocity V strikes (head on) another Q. 11. Two identical balls A and B are moving as shown
block of mass M which is at rest connected in the fig. Ball A hits a smooth floor head on with a
to a spring. The coefficient of restitution velocity u and at the same instant ball B strikes A
for collision between the blocks is e = 0.5. head on with a horizontal velocity u. The collision
between A and B is perfectly inelastic whereas the
M
Find the ratio
for which the subsequent coefficient of restitution for collision between A
m and the floor is e = 0.5. At what time the two balls
compression in the spring is maximum. There will collide again? Assume friction to be absent
is no friction. everywhere.
V

m M A u

(ii) Ball A collides head on with another identical B u


ball B at rest. Find the coefficient of restitution
if ball B has 80% of the total kinetic energy of
Q. 12. Two blocks of mass m and M are lying on a
the system after collision.
smooth table. A spring is attached with the block
Q. 9. A ball having mass m and velocity u makes a head of mass M (see fig). Block of mass m is given a
on collision with another ball. After collision velocity towards the other block. Find the value
velocity of the ball of mass m was found to be M
V in the direction of its original motion. The of for which the kinetic energy of the system
m
interaction force between the two balls during
will never fall below one third of the initial kinetic
their collision has been shown in the graph. The
energy imparted to the block of mass m.
area of the shaded part of the graph is same as
the area of the not shaded part. Find the velocity m M
of the balls at the instant they were having equal
velocity.
F
m u Q. 13. Two identical blocks A and B have two identical
springs fixed to them (see fig). Mass of each block
t is M and force constant of each spring is K. The
Q. 10. Ball A is about to hit a wall at an angle of two blocks have been placed on a smooth table.
incidence of q = 30°. But before hitting the wall Another block C of mass m (< < M) is placed
it made a head on collision with another identical between A and B and is held close to A so as to
ball B. The ball B then collides with the wall. The compress the spring attached to A by X0. From
coefficient of restitution for collision between this position the system is released. C moves to
two balls is e1 = 0.8 and that between a ball and push B and then is back to push A. The sequence
the wall is e1 = 0.6. Find the final velocity of ball continues until all interactions between the blocks
B. Initial velocity of A was u = 5 ms–1. Neglect cease. Find the speeds eventually acquired by A
friction. and B.
È -1 Ê 2.25 ˆ ˘ A C B
Ítan ÁË 2.34 ˜¯ = 44∞˙
Î ˚
Momentum and Center of Mass 5.3

Q. 14 A particle of mass m is flying horizontally at rest and the impact parameter d is equal to
velocity u. It strikes a smooth inclined surface and ÷3 R where R is radius of each ball. Due to impact
its velocity becomes vertical. the direction of motion of ball A changes by 30°.
(a) Find the loss in kinetic energy of the particle Find the velocity of B after the impact. It is given
due to impact if the inclination of the incline that collision is elastic.
is 60° to the horizontal. A
(b) Can the particle go vertically up after collision u
if inclination of the incline is 30°?

d = 3R
B

u m

Q. 18. (a) Two identical balls are moving along X axis


and undergo an elastic collision. Plot the
60° position time graph for the two balls.
(b) Consider five identical balls moving along
Q. 15. A block A of mass 2 kg is moving to right with X axis. What is the maximum number of
a speed of 5 m/s on a horizontal smooth surface. collisions that is possible? Assume that more
Another block B of mass 2 kg, with a mass less than two balls do not collide at the same time
spring of force constant K =200 N/m attached to and collisions are elastic.
it, is moving to left on the same surface with a
speed of 3 m/s. Block A collides with the spring Q. 19. Two particles of mass m each are attached to
attached to B. Calculate the end of a mass less spring. This dumb-bell
is moving towards right on a smooth horizontal
(a) the final velocity of the block A.
surface at speed V with the spring relaxed. Another
(b) the minimum kinetic energy of the system of identical dumb-bell is moving along the same line
two blocks during subsequent motion. is opposite direction with the same speed. The
(c) Repeat part (b) if there is no spring and the two dumb-bells collide head on and collision is
two blocks collide head on. Assume that the elastic. Assuming collisions to be instantaneous,
blocks are made of perfectly elastic material. how many collisions will take place?
3 m/s
2 kg 5 m/s 2 kg V V
A B

Q. 16. A box of mass M is at rest on a horizontal surface. Q. 20. Two blocks of mass m1 and m2 are moving along
A boy of mass m (< M) wants to push the box by a smooth horizontal floor. A non-ideal spring is
applying a horizontal force on it. The boy knows attached at the back of mass m2. Initial velocities
that he will not be able to push the box as the of the blocks are u1 and u2 as shown; with u1 >
coefficient of friction µ between his shoes and u2. After collision the two blocks were found to
ground is almost equal to that between the box and be moving with velocities V1 and V2 respectively.
the ground. He decides to run, acquire a speed u Find the ratio of impulse (on each block) during
and then bang into the box. After hitting the box, the deformation phase of the spring and that
the boy keeps pushing as hard as possible. What during its restoration phase. [By non ideal spring
is the maximum distance through which the box we mean that it does not completely regain its
can be displaced this way? original shape after deformation. You can neglect
m the mass of the spring.]
M m1 m2
u1 u2

Q. 17. A smooth ball A travels towards another


identical ball B with a velocity u. Ball B is at
5.4 Problems in Physics for JEE Advanced

Q. 21. A ball moving with velocity V0, makes a head on blocks. A bullet of mass m0, moving with speed u
collision with another identical ball at rest. The hits the block of mass M and gets embedded into
velocity of incident ball and the other ball after it. The collision is instantaneous. Assuming that
collision is V1 and V2 respectively. m always stays over M, calculate the maximum
(a) Using momentum conservation write an extension in the spring caused during the
equation having V1 and V2 as unknowns. Plot subsequent motion.
a graph of V1 vs V2 using this equation. K = 8960 N/m ; u = 400 m/s
support
(b) Assuming the collision to be elastic write an m
K
equation for kinetic energy. Plot a graph of V1
vs V2 using this equation. m0
u M
(c) The intersection point of the above two
graphs gives solution. Find V1 and V2 . Q. 24. Starting from a height H, a ball slips without
(d) In a particular collision, the plot of graphs friction, down a plane inclined at an angle of 30°
mentioned above is as shown in figure to the horizontal (fig.). After leaving the inclined
V1 (m/s) plane it fall under gravity on a parabolic path and
hits the horizontal ground surface. The impact is
perfectly elastic (It means that there is no change
2 (0.5, 1.5) in horizontal component of ball’s velocity and its
vertical velocity component gets inverted. There
(1.5, 0.5) is no change in speed due to collision). Will the
V2 .(m/s) ball rise to a height equal to H or less than H after
O 2 the impact?

30°
O

H
Find V1 and V2 for this collision. Also write the
percentage loss in kinetic energy during the
collision.
Q. 22. A particle having charge +q and mass m is
approaching (head on) a free particle having mass
M and charge 10 q. Initially the mass m is at large Q. 25. Hailstones are observed to strike the surface of
distance and has a velocity V0, whereas the other the frozen lake at an angle of 30° with the vertical
particle is at rest. and rebound at 60° with the vertical. Assuming
the contact to be smooth, find the coefficient of
(a) Find the final velocity of the two particles
restitution.
when M = 20 m.
(b) Find the final state of the two particles if M = Q. 26. A ball of mass m approaches a heavy wall of mass
m. M with speed 4 m/s along the normal to the wall.
q +10 q The speed of wall before collision is 1m/s towards
V0 the ball. The ball collides elastically with the wall.
m What can you say about the speed of the ball after
M
collision? Will it be slightly less than or slightly
Q. 23. In the system shown in fig. block of mass M is higher than 6 m/s ?
placed on a smooth horizontal surface. There is
a mass less rigid support attached to the block. Q. 27. A particle is thrown upward with speed 20÷2
Block of mass m is placed on the first block and m/s. It strikes the inclined surface as shown in the
it is connected to the support with a spring of figure. Collision of particle and inclined surface is
force constant K. There is no friction between the perfectly inelastic. What will be maximum height
Momentum and Center of Mass 5.5

(in m) attained by the particle from the ground (g ball 1. After the collision ball 2 itself goes straight
= 10 m/s2) into the hole at A. Prove that ball 1 will fall into
the hole at C. Assume that the balls are identical
and their dimensions are too small compared to
the dimensions of a hole. All collisions are elastic
20 2 m/s
A B
45°
?

O
4m
1
Q.28. 2n identical cubical blocks are kept in a straight
line on a horizontal smooth surface. The 2
D C
separation between any two consecutive blocks is
same. The odd numbered blocks 1, 3, 5,.....(2n–1) Q. 32. Blocks shown in figure have been placed on a
are given velocity v to the right whereas blocks smooth horizontal surface and mass of (n + 1)th
2, 4, 6,......2n are given velocity v to the left. All
1
collisions between blocks are perfectly elastic. block is 20 times the mass of nth block (where
Calculate the total number of collisions that will n = 1, 2, 3, 4,.....). The first block is given an
take place. initial velocity u towards the second block.
All collision are head on elastic collisions.
If u = 10 m/s then how many blocks must
be kept so that the last one acquires speed
Q.29. A small ball with mass M = 0.2 kg rests on top
equal to or greater than the escape speed
of a vertical column with height h = 5 m. A bullet
(= 11.0 km s–1)
with mass m = 0.01 kg, moving with velocity
v0 = 500 m/s, passes horizontally through the  40 
[Take log10  = 0.28 and log10 (11) = 1.04]
center of the ball. The ball reaches the ground at  21 
a horizontal distance s = 20 m from the column. u
Where does the bullet reach the ground? What part
of the kinetic energy of the bullet was converted 1
2 3 4 5 ..........
into heat when the bullet passed through the ball?
Neglect resistance of the air. Assume that g = 10
Q. 33. There is a long narrow and smooth groove in a
m/s2.
horizontal table. Two identical blocks A and B
Q. 30. Figure shows a circular frictionless track of radius each of mass m are placed inside the groove at
R, centred at point O. A particle of mass M is some separation. An ideal spring is fixed to A as
released from point A (OA = R/2). After collision shown. Block A is given a velocity u to the right
with the track, the particle moves along the track. and it interacts with B through the spring.
(a) Find the coefficient of restitution e.
A u B
(b) What will be value of e if the velocity of
the particle becomes horizontal just after
collision?
O A (a) What will be final state of motion of the two
blocks?

(b) During their course of interaction what is the
minimum kinetic energy of the system?

(c) The spring is removed and the two blocks are
tied using a mass less string. Now A is set
Q. 31. A rectangular billiard table has dimensions AB =
into motion with speed u. What will be the
4÷3 feet and BC = 2 feet. Ball 1 is at the centre of
final state of motion of the two blocks in this
the table. Ball 2 moving perpendicular to CD hits
case ? How much kinetic energy is lost by the
5.6 Problems in Physics for JEE Advanced

system? Where goes this energy? distance of the centre of mass of the particle
A u B system from the centre of the polygon.

(ii) In the last problem you have been asked to
remove any one particle from the system so
that the centre of mass of the remaining 2014
string
particles lies farthest from the geometrical
Q. 34. A carpet lying on ground has length L, width a centre of the polygon. Which particle will
and a small thickness d. It is rolled over a light you remove?
L Q. 37. Two identical block A and B each having mass
cylindrical pipe of radius r = and
100 π m, are connected with a spring of force constant
k. The floor is smooth and A is pushed so as to
kept on a level ground. Increase in gravitational
compress the spring by x0. The system is released
potential energy of the carpet is U1 (compared
from this position
to its initial position when it was lying flat). In
another experiment the carpet was folded to (a) Calculate the maximum speed of the centre
give it a shape of a cuboid (see figure) having of mass of the system during subsequent
width b. When this is placed on level ground its motion.
gravitational potential energy is U2 higher than (b) What is acceleration of the centre of mass
its initial position (flat on ground). It is given that at the instant it acquires half its maximum
d = 10–4 L. Find b for which U1 = U2. [Take speed?
π
= 1.25 ]
2
k
a
B
A
L
a r
d
Q. 38. A dancer leaps off the floor with her centre of
mass having a velocity of 5 m/s making an angle of
q 37° to the horizontal. At the top of the trajectory
a the dancer has her legs stretched so that the centre
of mass gets closer to head by a vertical distance
of 0.25 m. By how much does the head rises

vertically from its initial position? sin 37° = 


3
b
 5
.
Q. 35. Two identical thin rods are welded as shown in
the fig. B is midpoint of rod CD. Now the system
is cut into two parts through its center of mass M.
The part AM weights 4 kg. Find the mass of the
other part.
C

30°
A
B

D
Q. 39. In order to make a jump straight up, a 60 kg
Q. 36. (i) A regular polygon has 2016 sides and r is player starts the motion crouched down at rest.
the radius of the circle circumscribing the He pushes hard against the ground, raising his
polygon. Particles of equal mass are placed centre of mass by a height h0 = 0.5 m. Assume
at 2015 vertices of the polygon. Find the that his legs exert a constant force F0 during this
Momentum and Center of Mass 5.7

motion. At this point, where his centre of mass Q. 43. Consider a uniform rectangular plate. If a straight
has gone up by h0 his feet leave the ground and line is drawn, passing through its centre of mass
he has an upward velocity of v. Centre of mass of (in the plane of the plate), so as to cut the plate
his body rises further by h = 0.8 m before falling in two parts – the two parts obtained are of equal
down [Take g = 10 m/s2] mass irrespective of the orientation of the line.
(a) Find v. Can you also say that a straight line passing
through the centre of mass of a triangular plate,
(b) Find the normal force applied by the ground irrespective of its orientation, will also divide the
on his feet just before he left the ground. triangle into two pieces of equal mass?
Q. 40. A platform is kept on a rough horizontal surface. Q. 44. Two spherical bodies of masses m and 5 m and radii
At one end A of the platform there is a man R and 2R respectively are released in free space
standing on it. The man runs towards the end B with initial separation between their centres equal
and the platform is found to be moving. In which to 12R. If they attract each other due to gravitational
direction will the platform be moving after the force only then find the distance covered by smaller
man abruptly comes to rest on the platform at B? sphere just before collision.

R 2R
12R

A B
m
5m
Q. 45. A shell is fired vertically upward with a speed of
Q. 41. Two particles P and Q have mass 1 kg and 2 kg 60 m/s. When at its maximum height it explodes
respectively. They are projected along a vertical into large number of fragments. Assume that the
line with velocity up = 20 m/s and uQ = 5 m/s fragments fly in every possible direction and all
when separation between them Q of them have same initial speed of 25 m/s
was 60 m. P was projected [Take g = 10 m/s2]
uQ
vertically up while Q was (a) Prove that after the explosion all the fragments
projected vertically down. will lie on an expanding sphere. What will be
Calculate the maximum height 60 m speed of the centre of the sphere thus formed
attained by the centre of mass of – one second after explosion?
the system of two particles, (b) Find the radius of the above mentioned
measured from the initial P uP sphere at the instant the bottom of the sphere
position of P. Assume that the touches the ground.
particles do not collide and that the ground is far
below their point of projection [g = 10 m/s2] Level 2
Q. 42. Two small motors are kept on a smooth table at
a separation L. The motors have mass M and 2M Q. 46. A car of mass M is free to move on a frictionless
and are connected by a light thread. The motors horizontal surface. A gun fires bullets on the car.
begin to wrap the thread and thereby move The bullets leave the stationary gun with speed u
closer to each other. The tension in the thread and mass rate b kg s–1. The bullets hit the vertical
is maintained constant at F. Find the time after rear surface of the car while travelling horizontally
which the two motors will collide. Neglect the and collisions are elastic. If the car starts at rest
dimensions of the motors and their stands. find its speed and position as a function of time.
2M Mass of the car M >> mass of each bullet.
M
u
M

L Q. 47. (i) Liquid of density r flows at speed u along a


flexible pipe bent into a semicircle of radius
5.8 Problems in Physics for JEE Advanced

R. The cross sectional area of the pipe is A and the block with a velocity of u = 100 m/s. The
its cross sectional radius is small compared to ball keeps bouncing back and forth between the
R. Three strings S1, S2 and S3 keep the pipe block A and fixed wall B. Each of the collisions
in place. S3 ties the two ends of the pipe and is elastic. After the ball has made 1000 collisions
the other two string have their ends secured at with the block and wall each, the distance between
A and B. Strings S1and S2 are perpendicular the block and the wall was found to be L = 1.2
to the string S3. The entire system is in m. Calculate the average force being experienced
horizontal plane. Find the tension in the three by the block due to collision at this instant. All
strings. collision are instantaneous.
B

V
V
S3 A
u
R
S1 L
S2

Q. 50. A chain (A) of length L is coiled up on the edge


A B
of a table. Another identical chain (B) is placed
(ii) A car of mass M is moving with a velocity straight on the table as shown. A very small length
V0 on a smooth horizontal surface. Bullets, of both the chains is pushed off the edge and it
each of mass m, are fired horizontally starts falling under gravity. There is no friction.
perpendicular to the velocity of the car with
a speed u relative to the car. After firing n
bullets it was found that the car was travelling B
with velocity V0 in a direction opposite to its
original direction of motion. Assume that mu
<< MV0 and also that nm << M. Find n in A
terms of other given parameters.
Q. 48. A block of mass m = 4.4 kg lies on a horizontal
rough surface. The coefficient of friction between
the block and the surface is µ = 0.5. A force F
starts acting on the block making an angle q = 37° (a) Find the acceleration of the chain B at the
to the horizontal. The force changes with time as L
instant length of it is hanging. Assume
shown in the graph. 2
no kinks in the chain so that the entire chain
(a) At what time the block begins to move? moves with same speed.
(b) Calculate the maximum speed attained by the (b) For chain A assume that velocity of each
block. element remains zero until it is jerked into
È 3 -2 ˘ motion with a velocity equal to that of the
Ítan 37∞ = 4 ; g = 10 ms ˙ falling section. Find acceleration of the
Î ˚
hanging section at the instant a length l0 has
F(N)
slipped off the table and its speed is known to
F be v0 at the instant.
q
40 Q. 51 To understand the effect of air resistance on
m the motion of a bullet let's consider a bullet of
the shape shown in the fig. The bullet is flying
t(s)
O 10 20 horizontally. The cross section of cylindrical
Q. 49. A heavy block A is made to move uniformly along part is A and the conical part has a semi vertical
a smooth floor with velocity V = 0.01 m/s towards angle of 45°. Assume that the bullet is fired with
left. A ball of mass m = 50 g is projected towards initial velocity u and moves in a gaseous medium
Momentum and Center of Mass 5.9

in which molecules are at rest (do you think (a) Assume that friction between the plank and
this assumption is necessary?). Collisions of the the table is large enough to prevent it from
molecules with the bullet are elastic. Take mass slipping and calculate the smallest normal
of bullet to be M, density of gaseous medium as r force applied by the plank on the table.
and disregard gravity. (b) Assume that friction is absent everywhere
and calculate the speed of the plank when the
rod makes q = 180°.
45°
Q. 54. A block of mass M = 5 kg is moving on a
horizontal table and the coefficient of friction is
µ = 0.4. A clay ball of mass m = 1 kg is dropped
(a) Consider two bullets one small and other on the block, hitting it with a vertical velocity of
large, made of same material. Which will u = 10 m/s. At the instant of hit, the block was
experience larger retardation due to air having a horizontal velocity of v = 2 m/s. After
resistance? an interval of t, another similar clay ball hits the
(b) Write the speed of bulled after time t. block and the system comes to rest immediately
(c) Write distance travelled by the bullet in time after the hit. Assume that the clay balls stick to the
t. block and collision is momentary. Find t. Take g
Q. 52. Two particle A and B, of mass 3m and 2m = 10 m/s2.
respectively, are attached to the
ends of a light inextensible
string which passes over a u
smooth fixed pulley of
negligible mass. After the V
M
system is released and A falls m
through a distance L, it hits a A B
horizontal inelastic table so Q. 55. Vertical strings of same length L support two
that its speed is immediately L balls A and B of mass 2m each. There is a small
reduced to zero. Assume that B monkey of mass m sitting on ball A. Suddenly,
never hits the table or the the monkey jumps off the ball A at an angle q
pulley. Find = 45° to the horizontal and lands exactly on the
(a) the time for which A is resting on the table ball B. Thereafter, the monkey and the ball B just
after the first collision and before it is jerked manage to complete the vertical circle.
off, (a) Find distance d between the two string and
(b) the difference between the total kinetic energy the speed with which the monkey jumped of
of the system immediately before A first hits the ball A.
the table and total kinetic energy immediately
(b) Find the impulse of the string tension on ball
after A starts moving upwards for the first
A during the small period when the monkey
time. Explain the loss in kinetic energy.
interacted with the ball to jump off it.
Q. 53. A light rod of length L is hinged to a plank of
mass m. The plank is lying on the edge of a d
horizontal table such that the rod can swing freely
in the vertical plane without any hindrance from L L
the table. A particle of mass m is attached to the
end of the rod and system is released from q = 0°
position (see figure) q
m
A B
q Q. 56. In the shown figure, pulleys and strings are ideal
and horizontal surface is smooth. The block
m
C (mass 2m) is given a horizontal velocity of
5.10 Problems in Physics for JEE Advanced

V0 = 3 m/s towards right and the entire system is Q. 59. A moving particle of mass m collides elastically
let go. Find the velocity of three blocks, just after with a stationary particle of mass 2m. After

the strings regain tension. Mass of A and B are 2m collision the two particles move with velocity v 1
 
and m respectively and take g = 10 m/s2. and v 2 respectively. Prove that v 2 is perpendicular
 
V0 to (2 v 1 + v 2)
Q. 60. Two identical carts are moving on parallel smooth
2m C tracks with velocities u1 = 10 ms–1 and u2 = 15
ms–1. The empty carts (with drivers) have mass
3m each. Each cart has a sack of mass m kept at
end A and end D (see figure). At the instant the
carts being to cross, the sack in cart 1 is the thrown
perpendicular (relative to cart 1) with some
A unknown velocity and it lands on cart 2 at its end
2m B D after a time t0. Immediately after the sack lands
m into cart 2, the original sack in cart 2 is thrown
Q. 57. Two identical small balls are interconnected with perpendicularly (relative to cart 2) towards cart 1
a light and inextensible thread having length in identical fashion. The sack lands on cart 1 at
L. The system is on a smooth horizontal table point M, a time t0 after the throw. Assume that the
with the thread just taut. Each ball is imparted carts are constrained to move in straight lines.
a velocity v, one towards the other ball and the (i) Find length BM if length of each cart is L
other in a direction that is perpendicular to the (ii) Find the velocity of cart 1 after the sack
velocity given to the first ball. thrown from cart 2 lands on it.
v D
C 2
v u2

(a) After how much time the thread will become


M
taut again? u1
B 1 A
(b) Calculate the kinetic energy of the system
after the string gets taut. Q. 61. A man of mass m is standing on the flat top of
a cart of mass 2m. The length and height of the
Q. 58. A particle of mass 1 kg is moving with a velocity
cart is L and H respectively and it is at rest on a
of 200 m/s. An impulsive force of 4 s duration
smooth horizontal ground. The man starts running
acts on the particle in a direction opposite to its
from end A, speeds up and jumps out of the cart
motion. The force fluctuates a little bit around
at point B with a velocity u relative to the cart in
40 N magnitude and then it dies out in next 4s
horizontal direction. Calculate the total horizontal
showing small fluctuations. An oscilloscope
distance covered by the man by the time he lands
records the force as shown. The two oscillating
on the ground.
components in the graph are identical except
m
that one is mirror image of the other. Find the
magnitude of velocity of particle after the force
stops acting.
A B
Force (N)

H
2m

40
L
Q. 62. Two blocks of masses m = 2 kg and m = 8 kg
O are connected to a spring of force constant
4s 12 Time(s) K = 1 kN/m. The spring is compressed by 20 cm
4s
and the two blocks are held in this position by
Momentum and Center of Mass 5.11

a string. The system is placed on a horizontal can touch the roof of the car.
smooth surface and given a velocity u = 3 m/s
perpendicular to the spring. The string snaps
m
while moving. Find the speed of the block of
u m
mass m when the spring regains its natural length.
2m
m

Q. 65. A heap of rope is lying on a horizontal surface.


u One free end A of the rope is pulled horizontally
with a constant velocity v. Assume that the heap
does not move and the moving part of the rope
String remains straight and horizontal (i.e. there is no
M sag). Mass per unit length of the rope is l. Find
Q. 63. Two blocks of mass M and 2M are connected to the tension at point P where the straightened part
the two ends of a light, inextensible string passing of the rope meets the heap. How much force the
over an ideal pulley as shown in figure. The external agent must apply at end A?
system is released from rest. P A
v
(a) One second after the system is
released, a particle of mass M hits the
Q. 66. In the last problem, the free end A of the rope is
block of mass M and sticks to it. The
tied to a block of mass M and the block is given
particle hits the block with a speed of
a horizontal velocity v0 (see figure). Calculate the
10 m/s while travelling downward. Find the
following quantities at the instant the block is at
total distance travelled by block of mass 2M
a distance x from the right end of the heap (here
after it is released.
‘heap’ means the coiled part of the rope that is not
(b) One second after the system is
moving).
released, a particle of mass 2M hits the
block of mass M and sticks to it. The M v0
particle hits the block with a speed of
10 m/s while travelling in upward direction.
(i) Speed of the block.
Find distance travelled by the block of mass
2M after it is released to the time it comes to (ii) Tension force applied by the rope on the
rest for the first time (g = 10 m/s)2 block.
Q. 67. (i) Three balls 1, 2 and 3 lie on a smooth
horizontal table. Ball 1 is given a velocity
towards ball 2. Kinetic energy given to
ball 1 is k0. It collides with 2 and in turn
M ball 2 hits ball 3. All collisions are head on
elastic. Masses of the balls are m, M and m
2M
respectively.
2M M M 1 2 3
k0
(a) (b) 2M m M m

Q. 64. A toy car of mass 2 m is at rest on a smooth (a) Calculate the kinetic energy (k3) of ball 3
horizontal surface. A small bob of mass m is after ball 2 hits it.
suspended by a mass less string of length L from
the roof of the car. A horizontally flying bullet of (b) Draw the variation of k3 as a function of M.
mass m enters into the car through a small window (ii) Consider 10 balls laid on a smooth surface
and sticks to the bob. Speed of the bullet is u. Find m m m m
minimum value of u (call it u0) for which the bob with masses m, , , ...... and first
2 4 8 512
5.12 Problems in Physics for JEE Advanced

ball is pushed towards the second with kinetic m m


L/2 L
energy k0. [All collisions are elastic and head B
O
A
on]. The kinetic energy acquired by the last
ball is k10. In a separate experiment the 10th
ball is pushed towards 9th ball with kinetic L
energy k0. This time the kinetic energy
acquired by 1st ball is k1.
Compare k10 and k1.
1 2 3 4 10
(a) In which direction will the combined mass
move immediately after collision – right or
left?

Q. 68. A simple pendulum is suspended from a peg on a (b) Find tension in the string immediately after
wall which is inclined at an angle of 30° with the the collision.
vertical. The pendulum is pulled away from the Q. 70. A smooth track, fixed to the ground, is in the
wall to a horizontal position (with string just taut) shape of a quarter of a circle. Two small blocks of
and released. The bob repeatedly bounces off the mass 3m and 2m are released from the two edges
A and B of the circular track. The masses slide
2 down and collide at centre O of the track. Vertical
wall, the coefficient of restitution being e = . height of A and B from O is h = 2m. Collision
5
is elastic. Find the maximum height (above O)
Find the number of collisions of the bob with attained by the block of mass 2m after collision.
the wall, after which the amplitude of oscillation

(measured from the wall) becomes less than 30°.
45°
60°
30°
3m 2m
A B

h = 2.0 m

O
wall
Q. 71. A man stands on a frictionless horizontal ground.
Q. 69. Two particles A and B, having same mass m are He slides a 10 kg block on the surface with a
tied to a common point of suspension O. A is tied speed of 3 m/s relative the ground, towards a
with the help of an inextensible string of length L vertical massive wall. The wall itself it moving
and B is tied using an elastic string of unstretched towards the man at a constant speed of 2 m/s. The
L block makes a perfectly head on elastic collision
length . The two particles are released from
2 with the wall, rebounds and reaches back to the
horizontal positions as shown in figure. The man 3 second after the throw. At the moment the
particles have been released at a time gap so that block was thrown, the wall was at a distance of
both the string and the elastic cord become vertical 10 m from the man.
simultaneously. It was observed that the length (a) Find the mass of the man.
of the cord became equal to that of the string at
this moment and the two particles collided. The (b) Find the ratio of work done by the man in
particles got stuck together and their velocity just throwing the block to the work done by the
wall on the block.
after the collision was observed to be gL .
2 Q. 72. A ball is projected from point A in horizontal
direction with a velocity of u = 28 m/s. It hits the
Momentum and Center of Mass 5.13

incline plane at point B and rebounds. Show that (b) Find the minimum kinetic energy of the
whatever be the coefficient of restitution between system (as a percentage of original kinetic
the ball and the incline, the ball will always hit energy before collision) during the process of
the incline for the second time at a point above B collision.
(i.e., it will not hit the incline below B). Assume Treat the collision to be instantaneous.
the incline to be smooth and take g = 10 m/s2
 3
sin 37° = 5 
 
A u

Q. 75. On a billiard table two balls B and C are at rest


touching each other. A third ball A, travelling with
H = 20m B speed u, strikes the two balls elastically (see fig.).
Somehow, A hits B first and within a fraction of a
second hits ball C. You may assume that B and C
are placed symmetrically with respect to the line
37°
of motion of A and that all the balls are identical.
Q. 73. A staircase has each step of height h and width What angle does the final velocity of A make with
x. A ball strikes the centre point of a step with its original direction of motion.
velocity v making an angle q with the vertical. It
rebounds and strikes the centre point of the next B
step. Once again it rebounds and hits the centre A
point of the next step and so on. u
B C

v q

A Q. 76. A toy car of mass m is placed on a smooth


D
x/2
horizontal surface. A particle of mass 3m is
h suspended inside the car with the help of a string
C of length l. Initially everything is at rest. A sudden
horizontal impulse I = 2m ÷gl is applied on the car
C
and it starts moving.
x
m

I l

3m
Assume that there is no friction between the ball
and the steps and coefficient of restitution is e.
(a) Show that each time after hitting a step, the
ball climbs to the same height (i.e., heights (a) Find the maximum angle q0 that the string
like AB and CD shown in figure are equal). will make with the vertical subsequently.
(b) Find h and x. (b) Find tension in the string when it makes angle
q0 with the vertical.
Q. 74. Two identical discs are initially at rest in contact
Q. 77. A smooth ball of mass M and radius R is lying
on a horizontal table. A third disc of same mass
on a smooth horizontal table. A smaller ball of
but of double radius strikes them symmetrically
radius r and mass m travelling horizontally on the
and comes to rest after the impact.
table with velocity u hits the larger ball. Collision
(a) Find the coefficient of restitution for the is elastic. During the interaction of the balls the
impact. larger ball does not lose contact with the table at
5.14 Problems in Physics for JEE Advanced

any instant.
along –ve x‑direction with velocity 2gl
M collides with the bob and comes to rest. The

m R bob swings and when it comes to rest for
the first time, another particle Q of mass m
r u moving horizontally along y direction collides
with the bob and sticks to it. It is observed
that the bob now moves in a horizontal circle.

(i) Calculate the velocity of the balls after


collision. l

(ii) Calculate the maximum possible interaction 2l


force between the balls during collision. Y
Q. 78. A light rigid rod has a small ball of mass m attached
X
to its one end. The other end is hinged on a table
and the rod can rotate freely in vertical plane. The (a) Find tension in string just before the second
rod is released from vertical position and while collision.
falling the ball at its end strikes a hemisphere of (b) Find the height of the circular path above the
mass m lying freely on the table. The collision floor.
between the ball and the hemisphere is elastic. (c) Find the time period of the circular motion.
The radius of hemisphere and length of the rod (d) The string breaks during the circular motion
are R and 2R respectively. Find the velocity of the at time t = 0. At what time the bob will hit the
hemisphere after collision. floor?
Q. 80. A billiard ball collides elastically with an identical
stationary ball. The collision is not head on. Show
that the directions of motion of the two balls are at
2R right angles after the collision. Solve the problem
in centre of mass frame as well as in lab frame.

R
Q. 81.
m
u R/2
Q. 79. (i) O is a fixed peg at a height H above a
u0
perfectly inelastic smooth horizontal plane.
A light inextensible string of length L (> H)
has one end attached to O and the other end A heavy ball of radius R is travelling on a smooth
is attached to a heavy particle. The particle is horizontal surface with a velocity of u0 towards
held at the level of O with string horizontal left. A horizontally moving small ball of mass m
and just taut and released from rest. Find the R
strikes it at a height above the centre while
height of the particle above the plane when 2
it comes to rest for the first time after the travelling with velocity u towards right.
release. (a) After collision the small ball moves in
0 L
vertically upwards direction with velocity u.
H Prove that this can happen only if u > 3u0
(ii) The bob of a pendulum has mass m and (b) Find the velocity of small ball after collision
the length of pendulum is l. It is initially at if the collision is elastic and the balls are
rest with the string vertical and the point of smooth.
suspension at a height 2l above the floor. A Q. 82. Two elastic balls of masses M and m (M >> m)
m are placed on top of each other with a small gap
particle P of mass moving horizontally between them. The balls are dropped on to the
2
Momentum and Center of Mass 5.15

ground with the bottom of the lower ball at height u


h above the ground. The lower ball has a radius R
and the upper ball has negligible dimension.
A L B
m
Q. 86. Two blocks A and B, each of mass m, are connected
by a spring of force constant K. Initially, the spring
M is in its natural length. A horizontal constant force
F starts acting on block A at time t=0 and at time
t , the extension in the spring is seen to be l. What
h is the displacement of the block A in time t?
F
B A

(a) Up to what height the ball of mass m will


bounce above the ground ? Q. 87. Two blocks of mass m1 and m2 are connected to
the ends of a spring. The spring is held compressed
(b) Does the result obtained above violates the
and the system is placed on a smooth horizontal
low of conservation of mechanical energy?
table. The block of mass m1 = 2 kg is kept at x =
Q. 83. Three identical particles are placed on a horizontal 1 cm mark and the other block is at x = 2 cm
smooth table, connected with strings as shown. mark. The system is released from this position.
The particle B is imparted a velocity V0 = 9 m/s It was observed that at the instant m1 was at x = 5
in horizontal direction perpendicular to the line cm mark its velocity was zero and at that moment
ABC. Find speed of particle A when it is about to m2 was located at x = –4 cm. Find mass m2 and
collide with C. unstretched length (l0) of the spring.
V0 m2 k m1
x
A l B l C -5 -4 -3 -2 -1 0 1 2 3 4 5
Q. 84. A light inextensible string, passing over a pulley, Q. 88. Two particles having masses m1 and m2 are
 
supports two particles 1 and 2 moving with velocities V1 and V2 respectively.
at its ends. An insect of mass 
V0 is velocity of centre of mass of the system.
m is sitting on particle 2 and
the system is in equilibrium. (a) Prove that the kinetic energy of the system
The sum of masses of particles in a reference frame attached to the centre
and the insect is M. Now the 1
of mass of the system is KEcm = µV 2rel .
insect crawls a distance x up m1m 2 2
relative to the string. Find the Where µ = m + m and Vrel is the relative
1 2
displacement of centre of
speed of the two particles.
mass of the system of two
particles and the insect. In 1 (b) Prove that the kinetic energy of the system in
which direction does the 2 ground frame is given by
centre of mass move and why? 1
KE = KE cm + ( m1 + m2 ) V02
2
Q. 85. Two particles (A and B) of masses m and 2m are (c) If the two particles collide head on find the
joined by a light rigid rod of length L. The system minimum kinetic energy that the system has
lies on a smooth horizontal table. The particle during collision.
(A) of mass m is given a sharp impulse so that
Q. 89. Two blocks A and B of mass m and 2m
it acquires a velocity u perpendicular to the rod.
respectively are connected by a light spring of
Calculate maximum speed of particle B during
force constant k. They are placed on a smooth
subsequent motion. By what angle q will the rod
horizontal surface. Spring is stretched by a length
rotate by the time the speed of particle B become
x and then released. Find the relative velocity of
maximum for the first time?
the blocks when the spring comes to its natural
5.16 Problems in Physics for JEE Advanced

length. of the incline and is connected to the motor using


A B
a light string. The motor is switched on and it
slowly hauls block B through a distance L = 2.0
Q. 90. Two ring of mass m and 2m are connected with meter along the incline. Calculate the work done
a light spring and can slide over two frictionless by the string tension force on the wedge plus
parallel horizontal rails as shown in figure. Ring motor system. All surfaces are frictionless.
of mass m is given velocity ‘v0’ in horizontal
direction as shown. Calculate the maximum M
stretch in spring during subsequent motion.
m v0 L

2m
A B
q
Q. 91. A disc of mass M and radius R is kept flat on a
smooth horizontal table. An insect of mass m Q. 94. An ice cream cone of mass M has base radius
alights on the periphery of the disc and begins to R and height h. Assume its wall to be thin and
crawl along the edge. uniform. When ice cream is filled inside it (so as
(a) Describe the path of the centre of the disc. to occupy the complete conical space) its mass
m becomes 5 M. Find the distance of the centre of
(b) For what value of the centre of the disc mass of the ice cream filled cone from its vertex.
M
and the insect will follow the same path? Q. 95. A flexible rope is in the shape of a semicircle
ACB with its centre at O. Ends A and B are fixed.
Radius of the semicircle is R. The midpoint C is
pulled so that the rope acquires V shape as shown
in the figure.
O R A O B
A O B
R
C
Q. 92. A metal wire having mass M is bent in the shape
of a semicircle of radius R and is sliding inside C
a smooth circular grove of radius R present in a
(a) Make a guess whether the centre of mass of
horizontal table. The wire just fits into the groove
the rope moves closer to O or moves away
and is moving at a constant speed V. Find the
from it when it is pulled?
magnitude of net force acting on the wire.
V (b) Calculate the shift in position of the centre of
mass of the rope.
Q. 96. Three small balls of equal mass (m)
R are suspended from a thread and m m
two springs of same force constant
(K) such that the distances between
k
the first and the second ball and the
second third ball are the same. Thus
m
the centre of mass of the whole system
Q. 93. A triangular wedge (A) has inclined surface
coincides with the second ball. The
making an angle q = 37° to the horizontal. A
thread supporting the upper ball is k
motor (M) is fixed at the top of the wedge. Mass
cut and system starts a free fall. Find
of the wedge plus motor system is 3m. A small
the distance of the centre of mass of m
block (B) of mass m = 1kg is placed at the bottom
Momentum and Center of Mass 5.17

the system from the second ball when both the (a) The velocity of the centre of the disc becomes
springs acquire their natural length in the falling zero for the first time at time t0. Find t0.
system. t0
(b) Find speed of the small body at time .
Q. 97. (a) A uniform chain is lying in form of on arc 3
of a circle of radius R. The arc subtends an Q. 99. Laila and Majnu are on a boat for a picnic. The
angle of 2 at the centre of the circle. Find boat is initially at rest. Laila has a big watermelon
the distance of the centre of mass of the chain which she throws towards Majnu. The man
from the centre of the circle. catches the melon and eats half of it. He throws
OO
OOOO OOOOO
back the remaining half to Laila. She eats the
OO OO
OO O half of the melon that she receives & throws the
O
remaining part to Majnu. Majnu again eats half
OO

OO
of what he receives and returns the remaining
a R part back to Laila. This continues till the melon
a lasts. The two are sitting at the two ends of the
boat which has a length L. Combined mass of
O the boat and the two lovers is M0 and the mass of
πR the water melon is M. Assume that the boat can
(b) A uniform chain of length is lying move horizontally on water without any resistive
2
symmetrically on the top of a fixed smooth force. Find the displacement of the boat when the
half cylinder (see figure) of radius R. The watermelon gets finished.
chain is pulled slightly from one side and L
released. It begins to slide. Find the speed of
the chain when its one end just touches the
floor. What is speed of centre of mass of the
chain at this instant?
OOOOOOOO
OO OO
OO O
O Q. 100. A hot air balloon (mass M) has a passenger (mass
OO

OO

R m) and is stationary in the mid air. The passenger


climbs out and slides down a rope with constant
velocity u relative to the balloon.
      O
(c) In part (b) assume that the half cylinder is not (a) Show that when the passenger is sliding
fixed and can slide on the smooth floor. Find down, there is no change in mechanical
the displacement of the cylinder by the time energy (kinetic + gravitational potential
one end of the chain touches the floor. Mass energy) of the system (Balloon + passenger).
of cylinder is equal to that of the chain. Calculate the speed of balloon.
For part (b) and (c) assume that the chain (b) Calculate the power of the buoyancy force on
remains in contact with the cylinder all the the system when the man is
while. sliding. For easy calculation,
assume that volume of man is
Q. 98. A small body of mass m is at rest inside a narrow negligible compared to the M
groove carved in a disc. balloon.
Vo
Groove is a circle of
R (c) If buoyancy force is doing
radius R concentric to groove positive work, where is this
the disc. Mass of the
work done lost? You have
disc is also m . The disc
Disc proved that sum of kinetic
lies on a smooth
and potential energy of the
horizontal floor. The small body is given a sharp
system remains constant. m
impulse so that it acquires a tangential velocity Vo
at time t = 0.
5.18 Problems in Physics for JEE Advanced

Q.101. A wooden wedge of mass 10 m has a smooth standing on the edges of two perpendicular radii
groove on its inclined surface. The groove at A and B.
is in shape of quarter of a circle of radius A
R = 0.55 m. The inclined face makes an angle P
Ê 11 ˆ R
q = cos-1 Á ˜ with the horizontal. A block
Ë 5 ¯ B
C
‘A’ of mass m is placed at the top of the groove
and given a gentle push so as to slide along the
groove. There is no friction between the wedge
and the horizontal ground on which it has been Find the displacement of the centre of the disc if
placed. Neglect width of the groove.
(a) The two men walk radically relative to the
disc so as to meet at the centre C.
(b) The two men walk along the circumference
to meet at the midpoint(P) of the are AB.
A
Q. 104. There particles A, B and C have masses m,
R 2m and m respectively. They lie on a smooth
horizontal table connected by light inextensible
q strings AB and BC. The string are taut and
<ABC = 120°. An impulse is applied to particle A
R
along BA so that it acquires a velocity u. Find the
initial speeds of B and C.
(a) Find the magnitude of displacement of the
C
wedge by the instant the block A reaches the
bottom of the groove.
(b) Find the velocity of the wedge at the instant
the block A reaches the bottom of the groove.
120°
Q. 102. A uniform bar AB of length 6a has been placed on I
a horizontal smooth table of width 5 a as shown A B
in the figure. Length 2a of the bar is overhanging.
Mass of the bar is 4m. An insect of mass m is
sitting at the end A of the bar. The insect walks Level 3
along the length of the bar to reach its other end
Q. 105. A smooth hollow U shaped tube of mass 2m is
B.
lying at rest on a smooth horizontal table. Two
6a
m small balls of mass m, moving with velocity u
A B enter the tube simultaneously in symmetrical
4m
fashion. Assume all collisions to be elastic. Find
the final velocity of the balls and the tube.
2a u
m 2m
5a

(a) Will the bar topple when the insect reaches


end B of the bar?
(b) After the insect reaches at B, another insect
of mass M lands on the end A of the bar. Find
the largest value of M which will not topple m
u
the bar.
Q. 106. There are 40 identical balls travelling along a
Q. 103. A disc of mass M and radius R lies on a smooth straight line on a smooth horizontal table. All
M
horizontal table. Two men, each of mass , are balls have equal speed v and each one is travelling
2 to right or left. All collisions between the balls is
Momentum and Center of Mass 5.19

head on elastic. At some point in time all balls will no cover around the peg?
have fallen off the table. The time at which this cover
happens will definitely depend on initial positions
of the balls. Over all possible initial positions of
the balls; what is the longest amount of time that
you would need to wait to ensure that the table
has no more balls? Assume that length of the table L x
is L.

L Q. 109. Two thin rings of slightly different radii are joined


Q. 107. A small ball of mass m is suspended from the end together to make a wheel (see figure) of radius
A of a L shaped mass less rigid frame which is R. There is a very small smooth gap between the
fixed to a block of mass m. The block is placed two ring. The wheel has a mass M and its centre
on a smooth table. The ball is given a horizontal of mass is at its geometrical centre. The wheel
impulse so as to impart it a velocity of u. The ball stands on a smooth surface and a small particle
beings to rotate in a circle of radius R about the of mass m lies at the top (A) in the gap between
point A, while the block and the frame slide on the the rings. The system is released and the particle
table. Find the tension in the string, to which the begins to slide down along the gap. Assume that
ball is attached, at the instant the ball is at the top the ring does not lose contact with the surface.
most position. The rod does not interfere with the A
string during the motion.
q R
A
R
u
string m

m
B

Q. 108. A heavy rope of mass m and length 2L is hanged (a) As the particle slides down from top point A
on a smooth little peg with equal lengths on two to the bottom point B, in which direction does
sides of the peg. Right part of the rope is pulled a the centre of the wheel move?
little longer and released. The rope begins to slide (b) Find the speed of centre of the wheel when
under the action of gravity. There is a smooth the particle just reaches the bottom point B.
cover on the peg (so that the rope passes through How much force the particle is exerting on
the narrow channel formed between the peg and the wheel at this instant?
the cover) to prevent the rope from whiplashing. (c) Find the speed of the centre of the wheel at
(a) Calculate the speed of the rope as a function the moment the position vector of the particle
of its length (x) on the right side. with respect to the centre of the wheel
makes an angle q with the vertical. Do this
(b) Differentiate the expression obtained in (a) to
calculation assuming that the particle is in
find the acceleration of the rope as a function
contact with the inner ring at desired value
of x.
of q.
(c) Write the rate of change of momentum of Q. 110. A large number of small identical blocks, each
the rope as a function of x. Take downward of mass m, are placed on a smooth horizontal
direction as positive surface with distance between two successive
(d) Find the force applied by the rope on the peg blocks being d. A constant force F is applied on
as a function of x. the first block as shown in the figure.
(e) For what value of x, the force found in (d) d
F 1 2 3 4 5
becomes zero? What will happen if there is
5.20 Problems in Physics for JEE Advanced

(a) If the collisions are elastic, plot the variation Q. 113. A massive ball (A) is dropped from height h on
of speed of block 1 with time. a smooth horizontal floor. A smaller ball (B) is
(b) Assuming the collisions to be perfectly also dropped simultaneously. Initially ball B
inelastic, find the speed of the moving blocks was just touching ball A (see fig.). Radii of both
after n collisions. To what value does this balls is much smaller than h. Ball A hits the
speed tend to if n is very large. floor, rebounds and immediately hits B. Motion
Q. 111. Two small balls, each of mass m are placed on of both the balls is vertical before the collision
a smooth table, connected with a light string of of two balls. All collision are elastic and there is
length 2l, as shown in the figure. The midpoint of no friction. Ball B lands at point P on the ground
the string is pulled along y direction by applying after colliding with A. Find OP, assuming that it
a constant force F. Find the relative speed of the is large compared to radius of A.
two particles when they are about to collide. If the
q
two masses collide and stick to each other, how
much kinetic energy is lost. B
F
A

m m
l l

Q. 112. A block of mass M is tied to a spring of force


constant K and the system is suspended vertically. O P
Consider three situations shown in fig. (a), (b) and
Q. 114. Disc A of radius R is lying flat on a horizontal
(c).
surface. Disc B is also at rest. Disc C, which is
(a) In fig. (a), an insect of mass M is clinging to
identical to B is traveling along the surface with
the block and the system is in equilibrium.
its velocity parallel to the line joining the centre
The insect leaves the block and falls. Find the
C1 and C2 of the discs A and B. The distance
amplitude of resulting oscillations.
between the line C1C2 and the line of motion of
(b) In fig. (b), an insect of mass M is resting
centre of disc C is 3.r , where r is radius of both
on the top of the block and the system is in
B and C. Impact of C with B is completely elastic.
equilibrium. The insect suddenly jumps up
Subsequently it is observed that both B and C just
M miss hitting the disc A. Find the radius (R) of A in
with a sped u = g and the block starts
K terms of r.
A
oscillating. Find amplitude of oscillation B
assuming that the insect never falls back on C2 C1
the block. 3r

(c) In fig. (c), an insect of mass M falls on the


block that is in equilibrium. The insect hits
M
the block with velocity u = g while C
K
Q. 115. A mass m moving with speed u in x direction
moving downwards and sticks to the block.
collides elastically with a stationary mass 2m.
Find the amplitude of oscillation.
After the collision, it was found that both masses
have equal x components of velocity. What angle
does the velocity of mass 2m make with the
u
x axis?
Q. 116. A ball of mass M collides elastically with another
stationary ball of mass m. If M > m, find the
(a) (b) (c) maximum angle of deflection of M.
Momentum and Center of Mass 5.21

Q. 117. A tennis ball is lying on a rigid floor. A steel ball floor. Is it possible that the tennis ball will also
is dropped on it from some height. The steel ball bounce?
bounces vertically after hitting the ball on the

ANSWERS

Ê 3 ˆ u
Dp = 10 29 Ns; q = cos-1 Á 17.
1. Ë 29 ˜¯ 2
2. F2 18. (a)
X
3. (a) 400 J
B
(b) Both performed equal work.

4. acar = −2iˆ + 2 ˆj C

Ê 7 - 3ˆ A
2
5, Á 4 ˜ rAV
Ë ¯
mg
6. mmin = t
r S (V + u )
2 O D

7. mu2 O – A – D " Ball 1.


M C – A – B " Ball 2.
8. (i) = 1
m (b) 10
(ii) 1/3 19. 2
u − u2
u+V 20. 1
9. V2 − V1
2
V1
10. 3.24 ms–1making an angle of 44° with the normal to
the wall
u 21. (a) V0
11. A time g after first collision

M
12. £2
m
V2
O V0
K
13. V = x0
2M V1
2 V0
mu
14. (a) KEloss
3
(b) No
15. (a) 3 m/s towards left (b) V0 V2
O
(b)
Kmin = 2J
(c) Kmin = 2J

m 2 u2
16.
(
2µ g M 2 − m2 )
5.22 Problems in Physics for JEE Advanced

V1
V0 3kx0
(b)
4m
38. 0.2 m
(c) O V0 V2
39. (a) 4 m/s
(b) 1560N
40. (To right)
V1 = 0, V2 = V0 41. 40.56 m
(d)
V1 = 0.5 m/s ; V2 = 1.5 m/s; % loss in KE =
37.5% 4 ML
42. t=
22. (a) The heavier particle moves with a velocity of 3F
2V0  43. No
in the direction of V0 . 44. 7.5R
21
19V0 (a) 10 m/s
Lighter particle moves with velocity
 21 (b) 100 m
opposite to V0
u
The incident particle comes to rest. The other
 46. v =
M
particle moves with V0 . 1+
2bt
23. 0.1m
47. (i) Ts1 = Ts2 = Ts3 = rAv2
24. less than H
25. 1/3 p MV0
(ii)
mu
26. Slightly less than 6 m/s
48. (a) 5 s
27. 13 m
(b) 25 ms–1
n (n + 1)
28. 49. F ~ 1200 N
2
29. 100 m, 92.8% g
(a)
1 2
30. (a) e = 0 (b) v2
3 g− 0
(b)
32. 12 l0
33. (a) A will be at rest and B will have a velocity u 51. (a) Smaller bullet
Mu
(b) (b) M + r Aut
u
(c) Both will be travelling with velocity . Loss in M È r Au ˘
2 (c) ln Í1 + t
mu 2 rA Î M ˙˚
KE
4 8L
L 52. (a) 5g
34.
260 mgL
(b)
20 5
35. kg
3 2mg
53. (a)
36. (i) 3
(b) 2 gL
(ii) A particle next to the blank vertex.
54. ∆t = 1 s
37. (a) 1 k x 12
0
2 m
Momentum and Center of Mass 5.23

55. (a) d = 90 L u = 90 gL
1
(b)
4
(b)
m 45gL
73. (b) h =
(
v 2 cos2 θ 1 − e2 )
36 30 33 2g
56. VA m / s; VB m / s ; VC m /s
7 7 7 v2
x = (e + 1) sin 2q
L 2g
57. (a)
v
9
3 74. (a)
(b) mv2 16
4
58. 40 m/s (b) 36 %
L 75. 120°
60. (i) 76. (a) q0 = 60°
5
(ii) 5 m/s–1 2mg
T=
(b)
61. 2 È L + m 2 H ˘ 3
Í ˙
3 ÍÎ g ˙˚
77. (i) v =  m − M  u; v =  2mu 
62. 5 m/s m M +m M M +m
   
5
m
63. (a)  R + r
3 Mg 
 R − r 
(ii)
96
(b) = 10.67m
9 4 sin q gR (1 - sin q ) 1
2 2 78. where sin q =
65. TP = v ; Fext = v 1 + sin q2
5
Mv0 H5
66. (i) v = 79. (i)
M + λx L4
3
M 3 v20 λ mg
(ii) (a)
T=
(ii) 4
( M + λx )3 5l
(b)
m2 M 2 4
67. (i) (a) 16k0
( M + m )4 (c) 2π
3l
4g
(b) k3
5l
(d)
2g

81. (b) u2 3u02 3uu0


82. (a) 2R + 9h
m M
83. 6 m/s
(ii)
k10 = k1
mx
68. 4 84.
M
69. (a) Left
85. 2u , 180°
(b) Zero 3
70. 2.96 m
Ft 2 l
(a) 90 kg 86. +
4m 2
5.24 Problems in Physics for JEE Advanced

87. m2 = 4 kg ; l0 = 6 cm
1 (b)
2 −1 R ( )
88. (c) ( m1 + m2 ) V02 2 2
2
3k 2 31 4u
vr = x 104. vB u, vc
89. 11 11
2m
105. vball = 0; vtube = u
2m
90. v L
3k 0 106.
mR v
91. (a) A circle of radius mu2
M+m 107. T = − 13 mg
R
m
(b) = 1 g
108. (a) v = L ( x − L )
M
2 MV 2
92. = g
π R (b) a = (x − L)
L
2
93. 1.92 J dp Ê x - Lˆ
(c) = 2 mg Á
11h dt Ë L ˜¯
94.
15   x − L 
2

(d) F = mg  1 − 2   
(a) Closer to O  L  
(b) 0.03 R
L
mg (e) x=L+
96. 2
3K 109. (a) First moves to right and then to left
R sin α
97. (a) gR
α vw = 2m
(b) M (M + m)
V =2
( )
2 − 1 gR
; Vcm =
4 2 ( )
2 − 1 gR
(b)
π π π 2m 2 gR cos2 q (1 - cos q )
(c) vw =
R ( M + m )2 + Mm co2 q
(c)
π 110. (a) See the solution for the graph
2πR
98. (a) t0 = (b) n Fd ; Fd
V0 n +1 m m
V0
(b) Fl
2
111. 2 ; Fl
2 ML m
99. 3 ( M0 + M ) 112. (a)
Mg
(b) 2
Mg
K K
mu
100. (a) 6 Mg
M+m (c)
(b)
mg. u 2 K
101. (a) 6 cm È1 ˘
113. 16 h sin 2q Í + cos 2q ˙
Î2 ˚
(b) 0.18 m/s
 3 + 1
102. (a) No 114. R =  r
 3 − 1
(b) 85 m
115. 45°
R 116. sin-1 (m/M)
103. (a)
2 2
117. Yes.
Momentum and Center of Mass 5.25

SOLUTIONS
1.  
DP = Ú Fdt = iˆ Ú Fx dt + ˆj Ú Fy dt + kˆ Ú Fz dt
Ê1 ˆ Ê1 ˆ
= iˆ (30 ¥ 1) + ˆj Á ¥ 40 ¥ 1˜ + kˆ Á ¥ 80 ¥ 1˜
Ë2 ¯ Ë2 ¯
= 30iˆ + 20 ˆj + 40 kˆ
[Area under Fy – t graph gives
∫ Fy dt ]
\ Dp = 302 + 202 + 402 = 10 29 N - s


If angle made by ∆P with x axis is q

30 3
Then , cosq = =
10 29 29
2. Rate of change of momentum = Net external force.
External forces are (F1 or F2) and the force applied by the support on the string.
When F1 is applied, the support exerts a force (T1) on the string in the direction of F1
R = F1 + T1
When F2 is applied
R = F2 – T2
[T2 = force by the support on the string]
F2 > F1

3. (a) p = Impulse

mV – mu = 4 × 5 ˆj + 5 × 4 kˆ

1 × V − 1 × 10iˆ = 20 ˆj + 20 kˆ

V = 10iˆ + 20 ˆj + 20 kˆ
∴ V = 102 + 202 + 202 = 30 ms−1
Work energy theorem gives–

W =
1
2
1 1
(
mV 2 − mu2 = × 1 × 302 − 102 = 400 J
2 2
)

(b) Let the velocity be V1 after the first force stops acting.
 
mV1 − mu = 20 ˆj

V = 10iˆ + 20 ˆj
1

V1 = 10 5ms –1
1 1
∴ W1 = mV12 − mu2
2 2
1
= × 1 × ( 500 − 100 ) = 200 J
2
∴ W1 = W2 = 200 J
5.26 Problems in Physics for JEE Advanced

4. For O1force on the projectile is





F01 =
dp01
dt
(
= 4iˆ + 20 kˆ N )
This is real force on the projectile due to gravity and blowing wind.
For O2 force on the projectile is –


F02 =
dp02
dt
(
= 8iˆ − 32t ˆj + 20kˆ N )
 
Freal + Fpseudo = 8iˆ − 32t ˆj + 20kˆ

( )
4iˆ + 20kˆ + Fpseudo = 8iˆ − 32t ˆj + 20kˆ

∴ Fpseudo = 4iˆ − 32t ˆj
1
At = s
8

Fpseudo = 4iˆ − 4 ˆj

∴−m acar = 4iˆ − 4 ˆj

a = −2iˆ + 2 ˆj
car

5. y
V
A/2

A 600
V X
cos
–1 3
4

A 2V
4
Let mass of water that gets divided into two streams in time interval t be m.
m = AVr t
Change in momentum of water in X and Y direction is
A A Ê 3ˆ
DPx = V Dt.r.V .cos 60∞ + 2V Dt r.2V Á ˜ –AV t.V = O
2 4 Ë 4¯

A A Ê 7ˆ
DPy = V Dt rV sin 60∞ - 2V .r.Dt 2V ◊ Á ˜
2 4 Ë 4 ¯
Ê 7 - 3ˆ 2
= -Á ˜ r AV Dt
Ë 4 ¯

Force on water is along negative y direction


DPy Ê 7 - 3ˆ 2
Fy = = -Á ˜ r AV
Dt Ë 4 ¯

Ê 7 - 3ˆ 2
Force on tube, Fy = Á ˜ r AV
Ë 4 ¯
Momentum and Center of Mass 5.27

6. Volume of air striking the paper in unit time is = S (V + u)


Mass of air striking the paper in unit time = rS (V+ u)
In reference frame of man, the air molecules strike at a speed (V + u) and come to rest.
Rate of charge of momentum for air particles = rS (V + u) (V + u)

Force on paper due to air = rS (V + u)2
For the paper to not fall, friction on it must balance its weight
f max > mg
rS (V + u)2 > mg
mg
m>
r S (V + u )
2

7. Momentum conservation gives 2 mv – mu = mu & v = u


A B A B
u v
m 2m u m
2m

1 1  1
W = K f − Ki =  ( 2 m ) u2 + mu2  − mu2 = mu2
2 2  2
8. (i) The compression in the spring will be maximum when maximum KE is transferred to M. This happens when
M=m
(ii) KB = 4.KA & VB = 2VA (after collision)
mu = mVA + mVB

u 2u
⇒ u = 3VA ⇒ VA = and VB =
3 3
 u 2u 

  1
∴e = − 3 3  =
u−0 3
 
 
9. The not shaded area in the graph = Impulse during the period of deformation
Shaded area = impulse during the period of restitution

(Impulse)deformation = (Impulse)Restitution
m (V0
– u) = m (V – V0) [V0 = Common velocity when the balls
V0 – u = V – V0 have maximum deformation]
u+V
∴ V0 =
2
10. For collision of A and B

VB – VA = e1 (uA – uB)

VB – VA = 0.8 × 5 = 4 .............................(1)
and mVB + mVA = mu
VB + VA = 5 ................................(2)
Solving (1) and (2) we get
VB = 4.5 ms–1 ; VA = 0.5 ms–1
Now B collides with the wall.
5.28 Problems in Physics for JEE Advanced

Velocity component parallel to the wall remains unchanged.



V11 = VB sin 30° = 2.25 m/s
Velocity component perpendicular to the wall becomes
3

V = e2 VB cos 30° = 0.6 × 4.5 × 2.34 m /s
2
If final velocity makes an angle with normal to the wall
2.25
tan a =
fi a  44∞ and V = 2.252 + 2.342 = 3.24m /s
2.34
11. Applying momentum conservation in horizontal direction, it is easy to see that the two balls will have horizontal
u
component of velocity Vx =
2
[Collision between the balls is perfectly inelastic].
Due to collision with the floor, ball A acquires a vertical velocity = 0.5 u
Both balls are traveling in horizontal direction with same velocity after collision.
Hence, they collide next where A falls back on to the floor.
2 (0.5u ) u
t= =
g g
12. KE of the system is minimum when compression in the spring is maximum. This happens when both the blocks
have same velocity (Say V). If initial velocity of mass m is u then–
mu
( M + m )V = mu ⇒ V =
M+m
According to the problem
2
( M + m ) 
1 mu  1 1 2
 > . mu
2 M + m 3 2
m 1 M
⇒ > ⇒ <2
M+m 3 m
13. Initial momentum = 0

Final momentum = 0
Hence, A and B will have equal and opposite velocity (C will have negligible momentum due to its small mass. Its
KE is also negligible)
1 1 1
∴ MV 2 + MV 2 = KX 02
2 2 2
 K 
∴V = 
 2 M  0
X
 
14. (a) Since the incline surface is smooth, the velocity component of the particle parallel to the incline will not
change.
vs v
in vcos

ucos
u

usin
Momentum and Center of Mass 5.29


V sinq = u cosq ………(1)
u
For q = 60°, V =
3
2
1 2 1  u  mu2
Loss in KE =
mu − m   =
2 2  3 3
u
(b) From (1) tan θ =
V
Since, u > V tan q >1 q < 45°
Therefore, particle cannot go vertically up if q < 45°.
15. The collision process is quite similar to collision between two elastic bodies except for one difference that in
presence of the spring the time duration of the collision gets prolonged.
(a) In head on elastic collision of two equal masses exchange of velocities takes place. Hence, final velocity of the
block A will be 3 m/s towards left.
(b) KE will be least when the spring is compressed the most. In this position both the blocks will have same
velocity (say V).
mV + mV = 5 m – 3 m & V = 1 m/s
1 1
∴ K min = × 2 × 12 + × 2 × 12 = 2 J
2 2
(c) Answer will be same as in (b) due to the reason given above.
16. Speed immediately after collision
(M + m) V = mu
mu
V=
M+m
Retarding force on (M + m) is mMg – mmg after collision
mg ( M - m )
\ Retardation a =
(M + m)
Displacement (s) before stopping can be calculates as 2 as = V2
m (M - m) g m 2 u2
2.
.s =
(M + m) ( M + m )2
m 2 u2
s=
(
2m g M 2 - m2 )
17. B will move along the line of impact.

3
sinq =
2
According to the problem ball A moves perpendicular to its original direction of motion. For momentum to remain
conserved along the line of impact the other ball must move with velocity V2

u
u cosq =
2 u

2R d

V1
5.30 Problems in Physics for JEE Advanced

18. (a) Let the position of two particles at time t = 0 be as shown in figure.
1 2
X
X=0 X0
t=0

B 1

t
O t0 D
2
Collision occurs at t = t0
X – t graph for ball 1 is O – A – D and for ball 2 it is C – A – B
The graph may appear slightly different depending on initial position & direction of motion. The figure below
shows another situation.
B

D
A

C 1 2

X=0 X0
t =0
t
O

Ball 1 Æ OAD, Ball 2 Æ CAB

(b) Any intersection on X – t graph represents a collision. In elastic collision, the X – t graph gets exchanged [as in
(a)]. Therefore, the lines (X – t graph) representing the motion of 5 balls will not change even after a collision
takes place.
5 lines can intersect at maximum number of points given by 5C2 = 10

19. There will be two collisions. The sequence of events has been shown below.
V V
Just before first collision
V V
V V
Just after first collision
V V

V V
Just before second collision
V V
V V
Just after second collision
V V
Momentum and Center of Mass 5.31
m1 m2
20. u1 u2 before collision

m1 m2

Maximum Deformation

m1 m2
v1 v2 After collision

Impulse on m2 during deformation phase is given by-


JD = m2 u – m2 u2
Where u = Common velocity in maximum deformation state

Impulse on m2 during restoration phase is given by-
JR = m2V2 – m2u

J D u − u2
∴ =
J R V2 − u

m1u1 + m 2 u2 m1V1 + m2V2


But u = =
m1 + m 2 m1 + m2
m1u1 + m 2 u 2
- u2
JD m1 + m2 u -u
\ = = 1 2
JR m V + m V V2 - V1
V2 - 1 1 2 2
m1 + m2

21. (a) mV0 = mV1 + mV2


V1 + V2 = V0 ………………..(1)
V1

V0

O V0 V2

1 1 1
(b) mV12 + mV2 2 = mV0 2
2 2 2
V1
V12 + V22 = V02 V0

O V2
5.32 Problems in Physics for JEE Advanced

(c) V1
V0

V0 V2

Two solution are V1 = 0, V2 = V0


And V1 = V0 ; V2 = 0 [This is just not possible]
(d) V1 + V2 = 2 .......................................(i)
1 1 1
mV12 + mV2 2 = h mV0 2 [h < 1]
2 2 2
2 2 2
V1 + V2 = hV0
Radius of circle = hV0

0.52 + 1.52 = h .2
h = 0.625
Percentage loss in KE = (1 – 0.625) × 100 = 37.5%
The two solutions are –
V1 = 1.5 m/s; V2 = 0.5 m/s (This is not possible)
V1 = 0.5 m/s; V2 = 1.5 m/s
22 The problem can be solved as a usual elastic collision problem. The interaction time of the two particles is prolonged
but eventually the kinetic energy will be equal to initial KE.
23. After collision velocity of (M + m0) system is given by momentum conservation.
(M + m0) V = m0 u
10. V = 0.14 × 400
V = 5.6 m/s
Now the system is equivalent to that shown in figure below.
4 kg M + m0 = 10 kg

V = 5.6 m/s

V0 V0

Extension is maximum when both blocks have same velocity


(4 + 10) V0 = 10 × 5.6 & V0 = 4 m/s
Energy conservation
1 2 1 1
Kx + (14 ) V0 2 = × 10 × 5.62

2 2 2
Solving x = 0.1 m
24. Once the ball starts moving, its horizontal component of velocity never becomes zero. The impact does not affect
the horizontal velocity. Therefore, kinetic energy of the ball never becomes zero after the impact. It will never reach
height H.
Momentum and Center of Mass 5.33

30°

H
(5/6) H

25. Since there is no friction, components of velocity before and after collision parallel to the plane are equal, So
v sin 60° = u sin30°............. (1)
Components of velocity normal to the plane, after and before collision, are related to each other as
v cos 60° = e u (cos30°).........(2)
Taking ratio of (2) and (1)
e cot 30° = cot60°
1
e=
3
26.

4 m/s
1 m/s 1 m/s

Before collision After collision


If the mass of the wall is infinite its velocity will not change. If v be the velocity of ball after collision then
Relative velocity of separation = relative velocity of approach
v–1=4+1
or v = 6 m/s (away from the wall)
If mass of the wall is huge but finite, its speed will decrease a little bit and ball will have its speed reduced (compared
to 6 m/s) by same amount.
27. Speed just before collision is given by
v2 = (20÷2)2 – 2 × 10 × 4 = 720
v2 = ÷720 m/s
After inelastic collision the particle will have a velocity along the incline surface and its velocity perpendicular to
the incline will be lost.

Velocity along the incline = ¥ =

Horizontal and vertical component of this velocity is


vx = 6÷5 m/s
vy = 6÷5 m/s
Further height attained after collision will be given by –

= =
( ) =

Height attained from ground = 9 + 4 = 13 m
5.34 Problems in Physics for JEE Advanced

28. Initially there will be n collisions. Exchange of velocity takes place between two colliding blocks. The two extreme
blocks will move out and never encounter any other collision. Remaining (2n–2) blocks will further have (n–1)
n (n + 1)
collisions and so on. This way total no of collisions = n + (n –1) + (n – 2) + (n – 3) + ……. + 3 + 2 + 1 =
2
29.
m v0

s
D

Let v = horizontal component of the velocity of the bullet just after collision

V = horizontal component of the velocity of the ball just after collision

As no horizontal force acts on the system (ball + bullet), the horizontal component of momentum of this system
before collision and after collision must be the same:
mvo = mv + MV
M
v = v0 − m V

After collision both the ball and the bullet continue a free motion in the gravitational field with initial horizontal
velocities v and V, respectively. The time of flight for both is same equal to:

2h
t=
g

The distances passed by the ball and bullet during time t are: S = Vt and D = vt, respectively.

s g
Thus, V = ; V = s
t 2h

M g
Therefore, v = v0 − s
m 2h

2h M
And D = v0 − s = 100 m
g m
mv02
The kinetic energy of the system just before collision is: E0 .
2
Immediately after the collision the total kinetic energy of the system is equal to:
2
mv 2 MV

Em , EM
2 2
Momentum and Center of Mass 5.35

The difference is converted into heat,



E = E0 – (Em + EM).
Fraction converted into heat is:
∆E Em + E M
= 1 −
E0 E0
M s2 g  v0 2h M + m 
= 2 − .
m v 2 0 2h  s g m 

 = 92.8%.

1 p
30. cosq = 2 fi q = 3
(a) After collision the particle moves along the track. This means there is no normal component of velocity. Hence
e=o

(b) Just before collision, the components of velocity along the normal and along the tangent are
un = u sin q
ut = u cos q
R/2
O A

R

ut un
u
"
During collision ut does not change. The normal component of velocity becomes eun along B O.

O



B

Just after collision

Question says that velocity of the particle is horizontal after collision, which means
ut cos q = eun sin q
π 1
u cos2 q = eu sin2 q ⇒ e = cot 2 =
3 3
31. Path of the two balls after collision has been shown in figure

2 3
tan α = = 3 ⇒ α = 60°
2
5.36 Problems in Physics for JEE Advanced

2 3

A E D
B

2 

 1
2 
O

D C

Since it is elastic collision between two identical balls both of them will travel at right angle to each other
b = 30°
2 4
∴ ED = 2 tan 30° = and DB = 4 tan 30° =
3 3
6
∴ EB = =2 3
3


& The ball 1 will fall into the hole at C.
32. It can be proved using law of conservation of momentum and energy that speed of blocks after collision in the
shown figure will be -
( M − m)u 2 Mu
v1 =
and V2 =
M+m ( M + m)

2u 2u  40 
Now V2 = = = u
m 1  21 
1+ 1+
M 20
1
2
M u
m
Before collision
v1
v2
M
m
After collision
When block 2 hits block 3, the velocity of 3 will become
2
40 40u  40 
v3 = . =  u
 21 
21 21
speed of nth block after it gets a hit is

n −1
 40 
vn =   u
 21 

If vn > 11000 m/s


n −1
 40 

  × 10 11000
21
Momentum and Center of Mass 5.37

(n − 1) log 
40 
 > log 11 + log (100 )
21 
(n – 1) × 0.28 > 1.04 + 2
(n – 1) > 10.86
n > 11.86
Since n must be an integer
n = 12
33. (a) It is just an elastic collision between two blocks of equal mass.
(b) First B will acquire the speed of A and A will come to rest after collision. Now after B moves through a distance
equal to the length of the string, the string gives a jolt to both the blocks and thereafter both will move with
u
same speed. Conservation of momentum requires that both the blocks travel with a speed = .
2
34. Let outer radius of the cylinder formed be R
Equating volume (R2 – r2) a = L.a.d
Ld L2 L2
R2 =
+ r2 = 4 + 4
π 10 π 10 π
2L
∴R = = 2r
100 π
Ld
If the height of the cuboid is h then =h
b
L2
∴h =
10 4.b
h
The centre of mass of cylinder is at a height R from the ground and the height of COM of cuboid is
2
h
if U1 = U2 then R =
2
2L L2
=
100 π 2.10 4 b
πL 1.25L L
∴b = = =
2 2.100 200 160

36. (i) If particles were present at all vertices, the COM would have been at the centre O. Assume that a particle from
A has been removed. The COM of remaining system lies somewhere on the line AO produced. Let the COM be
at a distance x from O
2015 mx = mr [OA = r]
D
r
fix=
2015 C
O

A
5.38 Problems in Physics for JEE Advanced

37. (a) The normal force of wall on B is the reason for acceleration of the COM. Just when B is about to leave the wall
(i.e. when the spring is relaxed) let the speed of A be v.
1 2 1 2 k
mv = kx0 ⇒ v = x0
2 2 m

v 1 k
Speed of COM is v0 = = x
2 2 m 0
This is the final maximum speed.
v
(b) Let compression in the spring be x when speed of A is
2
2
1 kx 2 + 1 m  v  = 1 kx 2
  0
2 2 2 2
1
kx 2 + kx02 = kx02
4

3x2 3 x0
x2 = 0 ; x =
4 2
3kx0
Normal force by the wall =
2
3kx0 3kx0
acm = =
2 ( 2m ) 4m

38. The COM follows a parabolic trajectory like a projectile.
Maximum height gained by the COM is
2
 3 2
5 × 
u2 sin 2 θ  5
H= = = 0.45 m
2 g 2 × 10


Rise in position of head = 0.45 – 0.25 = 0.2 m
39. (a)
N
v
a
CM
h
CM
h0
CM

Mg

To rise to a height h any object must be thrown up with a speed


v = 2 gh = 2 × 10 × 0.8 = 4 m / s

(b) The centre of mass (COM) acquires a velocity of 4 m/s during a displacement of h0 = 0.5 m
v2 = O2 + 2ah0

v2 42
a = = = 16 m /s2
2h0 2 × 0.5
Net force on the mass N – Mg = Ma & N = Ma + Mg = 60 × 16 + 60 × 10 = 1560 N
Momentum and Center of Mass 5.39

40. As the man moves to right, the platform moves to left. Friction (By horizontal surface) on the platform is towards
right. Thus the momentum of the system (Man + platform) increases towards right. When the man comes to rest,
the system will have a net momentum towards right.
Fext m p g + mQ g
acm =
41.
mass
=
m p + mQ
=g ↓ ( )
m p u p + mQ uQ 1 × 20 − 2 × 5 10
ucm =
m p + mQ
=
1+ 2
()
= m/s ↑
3

Initial height of COM above P is


1 × 0 + 2 × 60
h0 = = 40 m
3
Further height attained
2
 10 
u2 cm  3  5

h= = = m
2g 2 × 10 9

Total height H = 40 + 0.56 = 40.56 m


42. COM is at a distance x1 from the motor of mass M
2 ML 2 L
x1 = =
3M 3
F
Acceleration of M is a =
M
The collision will take place when mass M moves through a distance x1
1 1 F  2L
∴ at 2 = x1 ⇒   t 2 =
2 2 M 3
4 ML
∴t =
3F
43. A line through COM does not necessarily divide a mass distribution into two halves of equal masses. Fact of the
matter is that
 
 ∑ mi ri  =  ∑ mi ri 
one half other half

 ∑ mi  =  ∑ mi 
one half other half

44. Hint – The sum of distance moved by the two balls = 9R
Since COM will not move m1 x1 = m2 x2
45. (a) The COM will fall down with acceleration ‘g’. In the COM frame every particle will move uniformly with
speed v = 25 m/s. Therefore, all the particles will keep moving away from the COM at a constant speed of
25 m/s. They all will lie on a sphere whose radius will be increasing at a rate of 25 m/s.
Centre of the sphere falls down with acceleration g. Speed of centre after 1 sec = 10 m/s
(b) The particle which was ejected in vertically downward direction will hit the ground first. Height of explosion
602
h= = 180 m
2 × 10
If t = time required for sphere to touch the ground after explosion then
180 = 25.t + 1 × 10 t 2
2
2
⇒ t + 5t − 36 = 0
⇒ t = 4s
5.40 Problems in Physics for JEE Advanced

Height (above point of explosion) of the particle which was emitted vertically up is
1
h1 = 25 × 4 − × 10 × 42 = 20 m
2
Diameter of the sphere = h + h1 = 200 m
Radius of sphere = 100 m
46. Let the speed of the car at time t be v. Let dm mass of bullets hit the car in a small time interval dt.
Velocity of approach = velocity of separation
u – v = vb + u & vb = 2u – v
vb is velocity of bullet after the hit in direction opposite to its original direction. We are assuming that change in
speed of car due to impact of one bullet is negligible.
Momentum transferred to the car in interval dt is
dp = 2(u – v) dm
dp dm
Force on car F = = 2 (u − v )
dt dt
The bullets do not hit the car at the rate at which they leave the gun.
dm b ( u − v )
=
dt u
dv 2 ( u − v )
2 v t
dv 2b
∴M = b⇒∫ = ∫ dt
0 (u − v )
dt u 2 Mu 0

1 1 2bt
⇒ − =
u − v u Mu
 2bt 
 u u
M 
⇒v= =
2bt M
1+ +1
M 2bt
As t " •; v " u

47. (i) Force on water inside the pipe in y direction = rate of change of y component of momentum of water
= (mass of water entering / leaving the pipe in unit time) × (2v)
= (rAv) (2v) = 2 rAv2(in – ve y direction)
Water applies same force on the pipe in y direction
y V

V V
V
Fy = 2rAv2 Ts1 + Ts2 = 2 rAv2
From symmetry Ts1 = Ts2
Ts1 = Ts2 = rAv2
Now consider a quarter of the pipe.
Change in x component of momentum of water in this segment in unit time is the x component of force
FX = (rAv) (v) 2 rAv2
Tension in s3 balances this force
TS3 = rAv2
Momentum and Center of Mass 5.41

(ii) P
P0 = mV0 P0 P0

 v0 y

x
u
m

Let’s assume that the car is travelling in y direction and a bullet is fired in x direction (wrt the car)
Change in momentum of the car is P = mu in a direction perpendicular to original momentum P0
P = P0 q
where q = angular change in direction of motion
mu
mu = MV0 q & q = MV
0

After n firings the car will deviate from original direction by


n q=
nmu

MV0
πMV0
∴n =
mu
48. (a) For 0 < t < 10 s
F = 4t F(N)
If the block begins to move at time t0, then
F cos q = m (mg – F sin q)
4 1 3 40
4t0 × 5 = 2  4.4 × 10 − 4to × 5 
20
⇒ t 0 = 5s

t(s)
O 5 10 15 20
(b) The block accelerates in the interval 5 < t < 15 s
Hence speed is maximum at t = 15 s
p Impulse
15 15

\ mvmax = Ú F cos q dt - Ú m ( mg - F sin q ) dt
5 5
15 15 15

\ mvmax = cos q Ú Fdt - m mg Ú dt + m sin q Ú Fdt


5 5 5
15
= (cos q + m sin q ) Ú Fdt - m mg ¥ 10
5

Ê 4 1 3ˆ 1
= Á + ¥ ˜ (shaded area ) - ¥ 4.4 ¥ 10 ¥ 10
Ë 5 2 5¯ 2
11 È 1 ˘
= ¥ 20 ¥ 10 + ¥ 20 ¥ 10 ˙ - 220
10 ÍÎ 2 ˚
fi 4.4vmax = 110
110
fi vmax = = 25ms -1
4.4
5.42 Problems in Physics for JEE Advanced

49. Block move uniformly at speed V = 0.01 m/s.


After each collision with A the speed of the ball will increase by 2V [Collision with fixed wall B does not cause
change in speed]
After n = 1000 collisions, speed of the balls is

un = u + 2nV = 100 + 2 × 1000 × 0.01 = 120 m/s
V V
A
u u’ = u+2V
Relative speed of approach Relative speed of separation
= V+u = u’ -V = V+u
Notice that speed of the ball changes marginally during each collision but the change becomes significant after a
large number of collisions. Change in momentum of the ball during nth collision with the body is

P = m [u + 2nV] + m [u + 2 (n – 1)V] = m [2u + 4 nV – 2V]
For n as large as 1000, we can neglect 2V
∴ ∆P = m [2u + 4 nV ]

50
= × [2 × 100 + 4 × 1000 × 0.01]
1000
= 12 kg m / s
120 m / s
At this instant frequency of collision of the ball with body A is f = = 100 s −1
1.2 m
Force = Pf
    1200 N
50. (a) Acceleration of B is
L
λ g
Weight of hanging part g
aB = = 2 =
Mass of chain λL 2
L/2

Chain B
L/2

(b) As the falling chain jerks off an element on the table, a thrust force acts on the falling section in upward
λdxv
direction Fth = = λv 2
dt
[Here we have assumed that a segment of dx length acquires speed v in time dt]
dp λdxv
∴ Fth = = = λv 2
dt dt
Equation of motion for falling section is
dv dx
ll = l l g- l v2
dt

l v
When l = l0 . v = v0
dv v2
∴ =g− 0
dt l0
Momentum and Center of Mass 5.43

51. Let’s observe the collision of one molecular (mass = m) in the reference frame attached to the bullet. In this frame,
the bullet is at rest and molecules are moving towards it at velocity V (= velocity of bullet in ground frame)
From symmetry, we understand that there will not be any transfer of momentum perpendicular to the X direction.
We need to consider momentum transfer along X direction only.
For one molecule r = mV
mv

X
mv

In unit time, the bullet sweeps through a volume AV and mass of an molecules in this volume is rAV.
Force on bullet = rate of momentum transfer = (rAV)V = rAV2

rAV 2

Retardation = .............(1)
M
M = d.V0 [d = density of bullet V0 = volume of bullet]
For smaller bullet volume is small and hence retardation will be higher. [Ans to (a)]

dV

M = - r AV 2
dt
V t
dV rA

M Ú0
=- dt
u V2
1 1 rA
- = t
V u M
1 1 r At
\ = + .....................(i)
V u M
Mu
\V = .................(ii)
M + r Aut

dx Mu
Now =
dt M + r Aut
x t dt
\ Ú dx = Mu Ú
0 0 M + r Aut
Mu È r Au ˘
x= ln 1 + t
r Au ÍÎ M ˚˙
M È r Au ˘
x= ln Í1 + t
rA Î M ˙˚

52. Acceleration before A hits the table is


 3m − 2 m  g
a= g=
 3m + 2 m  5
Speed of A just before hitting the table is
2
u0 = 2 aL =
gL
5
This is also the speed of B at this moment. Now A comes to rest and string becomes slack. Now B moves up with
retardation g. B stops and then starts falling down. When B acquires speed u0 and comes to
5.44 Problems in Physics for JEE Advanced

B
 I I
V
B B B
A A A A V

(i) (ii) (iii) (iv)


Just after A hits Just before Just After
the table string gets taut and string is taut
A starts moving

position shown in fig. (iii), the string is about to regain tension. As soon as the string becomes taut, the speed of both
A and B will becomes same, say V.
The impulse, I applied by the string causes sudden change in momentum of A and B.
For A –I = 3mV .........……(i)
For B –I = 2mV – 2mu0 ...………..(ii)
2
From (i) and (ii) V = u0
5
The required time = time of flight of a particle projected vertically with speed u0

2u 2 2 8L
= 0 = gL =
g g 5 5g

(a) Loss in kinetic energy is


1 1
∆k =
(5m ) u02 − (5m )V 2
2 2

5m 2  4   2 
= u0 1 −  ∴ V = 5 u0 
2  5  
1 2 mgL  2 
mu0 = = ∴ u0 = gL 
2 5  5 

53. (a) Let the speed of the particle be u at position q . Energy conservation gives–
1 2
mu = mgL (1 - cosq )
2
mu2
And the equation for centripetal force is T + mg cosq =
L
T = 2mg (1 – cos q) – mg cos q
T = mg (2 – 3 cos q)

T
u
 mg
L

For the plank normal force will be minimum when T cos q is maximum
Momentum and Center of Mass 5.45

N
T


mg

That is when cos q (2 – 3 cos q) is maximum


& – sin q (2 – 3 cos q) + cos q (3 sin q) = 0
& 2 sin q = 6 sin q cos q
1
fi cosq =
3
Nmin = mg – mg cos q (2 – 3 cos q)

1 1  2mg
= mg − mg.  2 − 3.  =
3 3 3
(b) Momentum conservation tells us that the velocity of the particle and the plank must be equal and opposite in
horizontal direction when the rod gets vertical.
1 2 1 2
\ mu + mu = 2mgL fi u = 4 gL
2 2
54. During the interaction period of the clay ball and the block, the vertical impulse (due to normal force) applied by
ground is
J v = ∫ N dt = change in momentum in

vertical direction = 1 × 10 = 10 kg m/s ..............(i)
Horizontal impulse of ground friction during the same period is
J H = Ú m N dt = 0.4 ¥ 10 = 4 kg m / s ...................(ii)

JV
u
V1
V

JH
Velocity (V1) of the (block + ball) system just after the impact is given by
(M + m)V1 = MV – JH
5× 2 − 4
V1 =
=1 m / s
6
Let the velocity of (M + m) be reduced to V2 [due to friction] in interval t.
At this point another clay ball hits the block.
JV and JH given by (i) and (ii) remain same for the second impact

0 = (M + m)V2 – JH
4 2
∴ V2 = = m/s
6 3
During the interval between two impact, friction causes a retardation of
a = mg = 4 m/s2
V2 = V1 – a t
2
1−
⇒ ∆t = 3 = 1 sec.
   4 12
5.46 Problems in Physics for JEE Advanced

55. (a) Let the monkey jump off A with a velocity having horizontal and vertical components ux and uy respectively.
A will recoil to left with velocity

VA mux ux
2m 2

L u L y

VA 
x
A B
When the monkey lands on B, it imparts a horizontal velocity to B
For (monkey + B) system
mux ux
VB = 3m = 3 [What happens to vertical velocity?]
This velocity is just sufficient for completing the circle
u
∴ x = 5gL ⇒ ux = 3 5gL
3
Time of flight from A to B for the monkey
d d
T = =
ux 3 5gL
2u y d
∴ =
g 3 5gL
d g
∴ uy =
6 5L
uy d g 1 d 1
∴ tan θ = = × =
ux 6 5L 3 5gL 90 L
If q = 45° ; d = 90 L
u can be calculated, u = 90 gL

Impulse of tension muy = m 45gL


56. Before the strings get taut, block C moves with constant velocity of 3 m/s and the blocks A and B (along with the
pulley) fall with an acceleration g.
The strings recover tension, when distance travelled by C = distance travelled by the falling pulley.
1
∴ × gt 2 = 3t
2
6
⇒t= = 0.6 s
10
Speed of A and B at this instant is 6 m/s. During the short interval in which tension is regained, let the impulse of
string tension on C be ∫ Tdt = I (→ )

I
Impulse on A and B both will be 2 ( ↑ )
Momentum and Center of Mass 5.47

For (C)
I = 2mVC – 2m.3
I
= 2VC − 6......................(i)
m [VC =velocity of C after string regains tension]
I
For A − = 2mVA + 2m.6
2
I
∴ = 24 − 4VA ...............(ii)
m
I
For B − = mVB − m.6
2
I
= 12 − 2VB .................(iii)
m
And after the strings are taut, velocity of A and B relative to the falling pulley must be equal and opposite.

VA – VC = – (VB – VC)

VA + VB = 2VC ........................(iv)
From (i) and (ii) 2VC – 6 = 24 – 4 VA

VC + 2VA = 15 ........................(v)
From (i) and (iii) 2VC – 6 = 12 – 2VB

VC + VB = 9 ........................(vi)
Solving (iv), (v) and (vi)
33 36 30
VC m/s, VA m/s, VB m/s,
7 7 7

57. (a) We will study the motion of second particle in the reference frame attached to the first particle. The velocity
of second particle makes an angle of 45° with the initial line joining the two particles (see fig 2). The thread is
loose before the distance between particles again becomes L. Fig.3 shows the situation just before the string
gets taut.
L 2 L
Required time is t = =
v 2 v
(b) In the reference frame of ground, velocities just before the string gets taut, has been shown in fig.4. The velocity
component for the two particles along the string will be same for both particles after the string is taut. Fig.5
shows the situation immediately after the string gets taut.
vrel = v 2
v
v
m
L Fig. (2)
Fig. (1)

V V
v 2
v rel
v L
t> v
V
L 2
45° v v
L Fig. (4) Fig. (5)
Fig. (3)
5.48 Problems in Physics for JEE Advanced

1 mv2 1 2  2 1  3 2
Now, total K.E. = + mv  1 + 2  = mv
2 4 2  2  4
58. The total impulse of the force is 40N × 4s = 160 Ns because the impulse due to two fluctuating components will add
to zero.

mv = 1 × 200 – 160 = 40 &v = 40 m/s
59. Momentum conservation
  
mv = mv1 + 2mv2
  
v = v1 + 2v2
Taking dot product of the equation with itself gives
      
v.v = v1.v1 + 4 v1.v2 + 4 v2 .v2 .......(i)
Conservation of kinetic energy gives
1 2 1 2 1
mv = mv1 + (2m ) v22
2 2 2
    
v.v = v1.v1 + 2v2 .v2 ..................(ii)
(i) – (ii) gives
   
4v1.v2 + 2v2 .v2 = 0
  
  
∴ v2 is perpendicular to ( 2v1 + v2 )

60. (i) Relative to cart 1, cart 2 has a velocity of 25 m/s (!).
According to the problem
25 t0 = L ............... (i)
As soon as the first sack lands on cart 2, its velocity will change
5 mV2 = 4m × 15 – m × 10
[Q the sack brings a momentum 10 m (") with itself]
V2 = 10 m/s
The second sack is thrown when the carts are in position shown.
Relative velocity = 20 ms–1
The time of flight of the sack = t0
20t0 = AM
L 4L
∴ AM = 20 × =
25 5
L
∴ BM = C 2 D
5
10 m/s

             
10 m/s
B 1 A

(ii) The second sack brings in a momentum m ×10 (!) with itself
4 mV1 = 3m × 10 – m × 10
V1 = 5ms–1
Momentum and Center of Mass 5.49

61. Let displacement of the cart be x (!) by the time man reaches the edge B.
Since centre of mass of the system (Man + Cart) will remain at rest hence,
m (L – x) = 2x
L
⇒x=
3

B
A

x
x

L–x

L 2L
Horizontal displacement of man = L −
=
3 3
Let the man jump out with absolute velocity v (")
v
For momentum to remain conserved the velocity of cart must be (←)
2
v

v
2
v
A per question u (→ ) = v (→ ) − (←)
2
3v 2u
u= ∴v =
2 3
2H
Time of flight for the man from B to ground is t = .
g
2u 2 H
Horizontal distance covered = vt =
3 g
2 L 2u 2 H
Total horizontal distance travelled from the start =
+
3 3 g
62. After the string snaps, the velocity component in x direction remains unchanged.
v2

m u y

M u

v1
5.50 Problems in Physics for JEE Advanced

For conserving momentum in y direction


mv2 = Mv1
2v2 = 8v1 & v2 = 4v1
Energy conservation
1
2
( 1
)
m u2 + v22 + M u2 + v12
2
( )
1 1 1
= kx 2 + mu2 + Mu2
2 2 2
2 2 2
∴ mv2 + Mv1 = kx
2
( )
2 ( 4 v1 ) + 8 v12 = 1000 × (0.2 )
2

∴ 40 v12 = 40
v1 = 1m / s
∴ vm = u2 + v22 = 32 + 42 = 5m / s

63. (a) Acceleration of the two blocks for 1 sec after start is a =
(2 M − M ) g = g
(2 M + M ) 3
10 10
speed of the system after 1 sec is u = at = ×1= m / s
3 3
At this moment, a particle of mass M travelling with speed = 10 m/s strikes.

M
10/3 m/s
10 m/s

2M M 2M 2M
10/3 m/s

By applying momentum conservation to the system comprising of the two blocks and the striking particle,
it is easy to see that the whole system will come to rest after collision. Thereafter the system will remain in
equilibrium.
[Alternately, one can consider the effect of impulsive string tension on two masses separately to arrive at the
result that there will be no motion after impact.]
Distance travelled by 2M = distance travelled in 1s before collision
1 2 1 10 2 5
= 0 + at = × × 1 = m / s
2 2 3 3
(b) As before, speed of the system and distance travelled by 2M, after 1 sec will be
10 5
u = m / s; s= m
3 3
This time, immediately after collision, the string will become loose.
Let speed of (M + 2M) just after collision be v.
10 70
3Mv = M × + 2 M × 10 ⇒ v = m / s
3 9
Momentum and Center of Mass 5.51

String will get taut after the two blocks travel through same distance S1 in time t.

10/3 m/s g
g v

2M M 2M M

10/3 m/s 10/3 m/s


2M

10 m/s
Just before Collision Just after Collision

10 1 70 1
S = t + × 10 × t 2 = t − × 10 × t 2
1 3 2 9 2
4 200
⇒ t = s and S1 = m.
9 81
Speed of block of mass 2M just before the string gets taut is
10 4 70
v1 = + 10 ¥ = m/s
3 9 9
Speed of (M + 2M) just before string is taut is
70 4 10
v2 = − 10 × = m / s
9 9 3
Speed just after string is taut (by momentum conservation) is
70 10
5 M .v′ = 2 M × + 3M ×
9 3
46
v′ = m/s
9

v'
10/3 m/s
a

v'
2M 2M
70/9 m/s

Just before string is taut Just after the string is taut


Now the system retards with acceleration
 3M − 2 M  1
a =   g = 5 g = 2m / s
2

 3 M + 2 M 
5.52 Problems in Physics for JEE Advanced

System will stop after travelling a distance S2.


2
02 =   − 2 × 2 × S2
46
 9
529
⇒ S2 = m
81
Total distance travelled by 2M before coming to rest

= 5 + 200 + 529 = 864 = 10.67


3 81 81 81
64. The bob just manages to touch the roof. It means that velocity of bob (+ bullet) relative to the car is zero when the
string gets horizontal. Let the common velocity be v.

L
m u1 v
u m

2m

Momentum conservation in horizontal direction gives


u
(m + m + 2m)v = mu0 fi v = 0 .......(i)
4
During collision of the bullet and the bob, there is loss in mechanical energy.
Let velocity of (bullet + bob) be u1 just after collision.
u
(m + m)u1 = mu0 ⇒ u1 = 0
2
KE after collision
2
1  u0  mu02
( )  
K = 2 m =
2  2  4
This will be equal to mechanical energy of the system when the string becomes horizontal.
1 mu2
(4m ) v2 + 2mgL = 0
2 4
2
 u0  mu02
Using (i) 2m  4  + 2mgL = 4

⇒ u0 = 4 gL
65. In time ‘dt’ a small length of the rope is jerked out to move with speed v.
It means a mass dx acquired a speed v in an interval dt.
dp (λdx )(v ) dx
F=
= = λv = λv 2
dt dt dt
This force is actually the tension in the rope at point P which sets the next element into motion.
Since moving part of the rope has no acceleration hence the external agent must apply force equal to v2.
66.
v
P
M

x
Momentum and Center of Mass 5.53

(a) The block plus the x length of the rope is moving at the instant.
From momentum conservation
(M + x)v = Mv0
Mv0
⇒ v =
M + λx
Note that the heap contains loose rope and it does not apply force on the moving part. Only the moving part
applies an impulse to next small segment of the rope to bring it to motion. [Actually if you take the moving
rope and next small element, which will be brought to motion next, in your system then the remaining heap is
not applying force on this system].
dx
(b) = v [Because this is the rate at which the rope is unwinding]
dt
dv dv Mv0 λ Mv0
∴a = =v =−
dt dx ( M + λx ) M + λx
2

M 2 v02 λ
=−
( M + λx )3
M 3 v02 λ
∴ T = Ma =
( M + λx )3
67. (a) Collision between 1 and 2

u BC

m M

v1 v2 AC

m M
mv1 + Mv2 = mu ….. (1)
v −v 
Elastic Collision −  1 2  = 1
 u−0 
v2 – v1 = u ….. (2)
 2mu 
Solving (1) and (2) v2 =  M + m 

Fraction of KE transferred to 2 is
1
Mv 22
2 4mM
h = 1 = .............(i)
mu2 ( M + m )
2

2
After 2 collides with 3, the KE of 3 will be
4mM 1
k3 = . Mv22
( M + m) 2 2

= mu
(
1 2 16m M
2 2

= k .
)16
m2 M 2
0
2 ( M + m )4 ( M + m )4
5.54 Problems in Physics for JEE Advanced

(b) For M " 0 ; k3 " 0


For M " • ; k3 " 0
You can prove that k3 is maximum when M = m. Hence, graph is as shown.
k3

O M

(c) For m = m0; M = m0


2
8
h=
9
8
For m = m0; M = 2m0 also h = .
9
In both cases the KE of last ball will be same.
68.
O

30° R cos 30°

30°


u = speed of bob just before first collision.
1 2
mu = mgR cos 30° ⇒ u = 3gR
2
After ‘n’ hits the speed of bob will become
n
 2 
vn = en u =   3gR
 5
1 2
mvn < mgR [ cos 30° − cos 60°]
2
n
4  3 1
∴   3gR < 2 gR  −
5  2 2 
 
n
4
  <
5
1
3
(
3 − 1 = 0.423 )
Smallest value of n satisfying this equation is 4.
Momentum and Center of Mass 5.55

69. (a) The ball tied to elastic cord will have lesser speed, when it reaches the bottom.
Combined mass will move to left.
(b) Energy conservation for particle B and elastic cord system
2 2
1  gL  1 L
m  + k   = mgL
2  2  2 2
mgL + kL2 = 8mgL
7mg
∴k =
L

Speed of A just before collision = 2gL .


Momentum Conservation gives–

m gL
2mv = m 2 gL −
2
gL gL
v= − = 0.46 gL
2 4
Centripetal force
2
= 2mv = 2 × (0.46 )2 × mg = 0.42mg
L
L
Tension in elastic cord = k. = 3.5mg
2
Tension in cord > mg + centripetal force
Particle will not go in circle.
String will have no tension.
70. Speed of both the blocks when they reach O is

u = 2 gh = 2 × 10 × 2 = 2 10 m/s

3m 2m
Just before collision
u u

         
v2 v1 Just after collision
Momentum Conservation
2 mv1 + 3 mv2 = 3mu – 2mu
2v1 + 3v2 = u ......….. (1)
Elastic Collision
v1 − v2
= −1
−u − ( u )
v1 – v2 = 2u ….. (2)
Solving (1) and (2)
7u 7
v1 = = 2 gh
5 5
5.56 Problems in Physics for JEE Advanced

When mass 2m reaches B after collision, its speed will be given by


48
v0 = v12 − 2 gh =
gh
25
After this it will move like a projectile projected at 45° to the horizontal.
v 2 sin 2 45° 12h
H max = 0 =
2g 25
Maximum height above O is
12h 37h 37
h+
= = × 2 = 2.96m
25 25 25
72.
28 m/s x

y y
vx
x

20-y vy

37°

Let the ball hit the incline at time t.


x = 28 t and
1
y = gt 2 = 5t 2

2
20 − y 20 − 5t 2 3
= tan 37°∴ =
x 28t 4
2
80 – 20t2 = 84t & t + 4.2 t – 4 = 0
Solving t = 0.8s
Just before hitting the incline, the velocity components of the ball are
vx = 28 m/s; vy = 10 × 0.8 = 8 m/s
If we resolve this velocity parallel to the incline and perpendicular to it, we get the components as
v11 = 28 cos 37° – 8cos 53° = 17.6 m/s
and v = 28 sin 37° + 8 sin 53° = 23.2 m/s

53°

v'

v'
 v

vx = 28

vy = 8
v
Momentum and Center of Mass 5.57

Just after impact


v11 = 17.6 m/s
v1 = 23.2. e
Resultant velocity after impact (v' ) makes an angle f with the incline. If f < 53°, the ball will move up the incline.
v1′ 23.2e
tan φ =
= = 1.32e
v11 17.6
4
Since tan 53° = = 1.33 , therefore, for any value of e (0 to 1.0), tan f < tan 53°
3
he velocity v' remains to the left of the vertical line and hence the ball will move up the incline.
T

73. (a) During collision the horizontal component of the velocity of the ball does not change. So it will travel equal
horizontal distances in equal intervals. It means time of flight between two collisions is same.
This is possible only if vertical velocity component after each collision remains same. Height like AB and CD
depend on this component of velocity only. Hence AB = CD
(b) Before first collision, the horizontal and vertical components of ball’s velocity are
vx = v sin ; vy = v cos
During collision vx does not change.
After first collision v1y = evy
Just before second collision, the vertical velocity component becomes (u2y)

( )
2 2 2
u2 y = v1y + 2 gh, i.e., u2 y = evy + 2 gh

Just after second collision, the vertical velocity becomes v2y = eu2y
But this must be equal to v1y
i.e.,
v22 y = v12y

e2 u22 y = v12y
e2 ÈÎe2 vy2 + 2 gh ˘˚ = e2 vy2

\h =
(
vy2 1 - e2 )
2g

\h =
v 2 cos2 q 1 - e2( ) ................(i)
2g

Time of flight between two collisions can be calculated as

1
2
( )
h = − evy t + gt 2
2
⇒ gt − 2evy t − 2h = 0

2evy ± 4e2 vy2 + 8 gh


∴t =
2g
evy + e2 vy2 + 2 gh
t= [ −ve sign is not acceptable ]
g
5.58 Problems in Physics for JEE Advanced

Substituting for h from (1)


Ê e + 1ˆ
t=Á v
Ë g ˜¯ y
vy v cos q
= (e + 1) = (e + 1)
g g
v2
\ x = v sin q t = (e + 1) sin 2q
2g

74. (a)





Just before Collision
r 1
sin q = =
3r 3




Just after impact

The two smaller discs will move along the respective lines of impact.
Momentum Conservation along x gives -
mu = mv cos q + mv cos q
u 3u
∴ v = 2 cos θ =
2 8
Coefficient of restitution

 v−0  9
e = −  0 − u cosθ  = 16

(b) The KE is least, when deformation is maximum. This occurs when relative velocity along the line of impact
becomes zero.
Situation at this moment is as shown with v = v0 cos


Momentum and Center of Mass 5.59

Momentum Conservation along x


2 mv cos q + mv0 = mu 2 v0 cos2 q + v0 = u

 8 
⇒  2 × + 1 v0 = u
 9 
9u
⇒ v0 =
25
99uu 8 8 3 38u8u
∴∴vv== × × = =
2525 3 3 2525
11 2 2 11
∴∴KE min== mv
KEmin mv × × 2+ 2 + mvm02v02
23 22
11 99××8 8 2 2 1 1 81812 2
== mm. . u u× 2× +2 + m.m. u u
22 625 625 2 2 625 625
1 1 2 2144 + 81  
144 + 81=  mu 1 1 2 2 9 9
= − mumu   −  mu  ,
2 2   625 
625    2 2  25  25
KE 99
∴∴ KEmin
min=
11 2 2 =2525
mu
22 mu
9
In percentage this is equal to × 100 = 36%
25
75. In an oblique elastic collision of two identical balls, the two will move perpendicular to each other after collision.
The figures show the centers of the balls. After first collision, B moves along the line of impact and A moves
perpendicular to that.
Then A hits C will velocity vA1 which makes 30° angle with line of impact. C goes along the impact and A is
deflected perpendicular to that.
vA2 makes 120° with original direction of motion.
vB
B

30° u 30°
A 60° 30°
60° C

vC
vA2
v A1 v A1

76. (a) Immediately after the impulse, velocity


I
2 gl .
of the car is v0
m
The string will make maximum angle with the vertical when there is no relative motion between the particle
and the car and both move horizontally with common velocity v.
v0
mv + 3mv = mv0 ⇒ v =
4
Energy Conservation

5.60 Problems in Physics for JEE Advanced

1 1
(3m ) v 2 + mv 2 = mgl (1 - cos q )
2 2

v2
fi 1 - cos q 0 = 0
8gl
4 gl
fi cos q 0 = 1 -
8gl
fi q 0 = 60∞
(b) In the reference frame attached to the car, the particle has no radial acceleration when at maximum deflection.

60°

ma0

60°

mg

If a0 is acceleration of the car towards left at this moment, then


ma0 cos 30° = mg cos 60°
g
a0 = 3
If we consider the car only then,
a0

60°
T

T sin 60° = ma0


3 mg 2mg
T = i.e., T =
2 3 3
77. (a) Momentum Conservation along horizontal direction gives
mv1 + Mv2 = mu … (1)
    
Since collision is elastic

u cos θ − 0
v cos θ − v cos θ = −1    … (2)
1 2
⇒ v2 − v1 = u
Solving (1) and (2)

v =
(m − M ) u ; v = 2mu
1
M+m
2
M+m
Momentum and Center of Mass 5.61

For ball of mass M to remain in contact with the table


Mg
F sin q < Mg i.e., F <
sin q

Ê R + rˆ
F < Mg Á
Ë R - r ˜¯

78.

R 1
tan q = =
2R 2
u = speed of ball just before collision.

1 2
mu = mg2 R (1 - sin q ) i.e., u = 2 gR (1 - sin q )
2
Let v0 and v be velocities of the hemisphere and the ball just after collision.



Relative velocity of approach = relative velocity of separation


u = v0 sin q + v       … (1)
Momentum conservation in horizontal direction gives
mv0 – mv sin q = mu sin q
v0 – v sin q = u sin q     … (2)
From (1) and (2)
v0 – (u – v0 sin q sin q = u sin q
v0 [1 + sin2 q = 2u sin q

2u sin q 4 sin q gR (1 - sin q ) 1


v0 = 2
= 2
where sin q =
1 + sin q 1 + sin q 5

79. (i) When the bob hits the floor the string makes an angle a with the horizontal.

H
sin α =
L
5.62 Problems in Physics for JEE Advanced

Velocity component parallel to the


H
horizontal surface is vx = v sin α = v ; where v = 2 gH
L
During collision the velocity component perpendicular to the horizontal surface is lost and the particle slides
along the surface with velocity vx. After it reaches a point on left side of the vertical line through O where its
distance from O is L , the string suddenly gets taut and the particle swings up like a pendulum. Just when the
string gets taut the velocity is perpendicular to the string equal to vx sin (the component along the length of
the string is lost). If the height attained is h then from conservation of energy it follows that –

mgh = 1 m ( v sin α )2 O
x
2 H a
4
H5
1
( ) 2 gH  H 
2
h= v sin 2 α =   = v
2g 2g  L  L4
80. The problem is solved easily in centre of mass frame.

mu u
vcm =
= (→ )
m+m 2
vcm remains unchanged even after collision, due to conservation of momentum.
Before collision, the velocities of the two particles in COM frame is as shown below.

COM frame
After collision, the two particles must move with equal and opposite momentum in the COM frame (because
momentum of the system always remains zero in the COM frame)
u
To keep the kinetic energy conserved, the two particles will have same speed, in the
2
COM frame. After collision the situation in COM frame is as shown in figure.


Direction of
 motion of COM


The velocity in lab frame is obtained by adding vcm to the velocity of the two particles in COM frame.
v2 cm
u/2
v2  v2cm  vcm


 u
vcm 
2
p-q
p-q


v1 cm
u/2

v1  v1cm  vcm
Momentum and Center of Mass 5.63

θ π−θ π
From the figure, angle between v1 and v2 is 2 + 2 = 2
 

Here I am presenting a neat solution in lab frame.


Momentum Conservation gives
  
mu = mv1 + mv2
  
i.e, u = v1 + v2
Taking dot product of this equation with itself
      
u.u = v1.v1 + 2 v1.v1 + v2 v2 ...............(i)
Conservation of kinetic energy
1 2 1 2 1 2
mu = mv1 + mv2
2 2 2
    
⇒ u.u = v1.v1 + v2 .v2 ...............(iii)
Subtract equation (ii) from (i)
 
v1.v2 = 0
 
⇒ v1 ⊥ v2

81. (a)
u

u R/2
R
u0

Since the bigger ball is heavy, its velocity will not change much due to collision.
| Relative velocity of separation along radius |
= e | Relative velocity of approach along radius |
u cos q – u0 sinq = e (u sin q + u0 sin q)

u cos q - u0 sin q
\e =

(u + u0 ) sin q
1 3
u − u0
e = 2 2 = u − 3u0

(u + u0 ) 23 3 (u + u0 )
e must be positive u> 3u0
(b)
V

u0


Let the velocity of small ball be V in the direction shown. Since collision is elastic
5.64 Problems in Physics for JEE Advanced

V cos f – u0 sin q = u sin q + u0 sin q


V cos f = (u + 2u0) sin q ................(a)
Velocity component of the ball along the tangent remains unchanged.
u cos q = v sin f ................(b)
From (a) and (b)
V2 sin2 f + V2 cos2 f = (u cos q)2 + [(u + 2u0) sin q]2
& V2 = u2 cos2q + u2 sin2q + 4u20  sin2q + 4 uu0 sin2 q
2 2
2  3  3
=u + 4u02   + 4uu0 
2 2
 = u + 3u0 + 3uu0
 2   2 
∴ V = u2 + 3u02 + 3uu0

82. u = 2 gh
3u

u
u
u u'
2R
u

Just before lower ball Just after the lower ball Just after the two balls
hits the ground hits the ground collide u' u

During collision between the balls, speed of M does not change much (M >> m).
And, relative speed of separation = relative speed of approach.

& The smaller bounces off with speed 3u.
Height attained by the ball above the ground is
( 3u )2
H = 2R + = 2 R + 9h Q u2 = 2 gh 
2g

83.
Vy

y
B
x
Vy
A
VX

Particle A and C will collide when the strings BA and BC become parallel to y axis. At this time B will be moving
along y with velocity Vy (say).
Component of velocity of A & C along y axis will also be Vy. Particle A has a component of velocity along X axis
as well (say Vx).
Momentum conservation along y direction gives mV0 = 3mVy
V0
⇒ Vy = = 3 m/s
3
Energy conservation
Momentum and Center of Mass 5.65

1 1  V0 
2
1   V 2 
mV0 = m   + ( 2m )   0  + Vx2 
2
2 2  3  2  3  
 
 2 
∴Speed of A and C = V 2 + V 2 =  V0  + V 2 
 y y  3  x

 
 
Solving, Vx = 3 3 m/s

(3 3 )
2
∴ Speed of A, VA = + 32 = 6 m/s

84. Let the mass 1 move up by X1. Mass 2 will go down by the same distance because the string is inextensible.
The insect goes up by (x – X1)

1'

X1 X3= x- X1
1 x = relative
displacement
2

X2= X1

2'

m1∆X1 + m2 ∆X 2 + m3 ∆X3
∆X cm =
M
m1∆X1 − m2 ∆X1 + m ( x − ∆X1 )
=
M
( m1 − m2 − m ) ∆X1 + mx
=
M
mx ∴ For equilibrium 
= m + m = m 
M  2 1 
The impulses of impulsive tension during the crawling of the insect are responsible for moving the COM up.
mu u
Vcm
85. 3m 3
2u u
In COM frame, the system rotates with speed of A = and speed of B =
3 3
2u u/3
3 u/3 u/3

A cm B B cm A

fig (a) u fig (b) 2u


3 3
5.66 Problems in Physics for JEE Advanced

The two figures show the velocities of A and B in COM frame.


In ground frame B will have maximum velocity when it is in position indicated in fig (b)
2u
∴ (VB )max =
3
86. Acceleration of the COM of the system is
F
a0 =
2m
Displacement of COM in time t is
1 Ft 2
x0 = a0 t 2 =
2 4m
Let x1 and x2 be the displacement of block A and B respectively.

mx + mx2 x1 + x2
x0 = 1 =
2m 2
Ft 2
∴ x1 + x2 = ....................(i)
2m
And x1 – x2 = l  ........... (ii)
Solving (i) and (ii)
Ft 2 l
x1 = +
4m 2
87. let x1 = displacement of m1 to right and
x2 = displacement of m2 to left
m2 x2 = m1 x1
(So that COM does not move)
When m1 is at x = 5; x1 = 4 cm
And m2 is at x = – 4 cm; x2 = 2 cm
m2 × 2 = 2 × 4 & m2 = 4 kg
At x = 5; m1 is at rest. At that time m2 must also be at rest (to keep the linear momentum conserved). This is the time
when extension in the spring is maximum.
Initial compression in the spring = Maximum extension in the spring
[Since kinetic energy is zero at both instances]
⇒ l0 − 3 = 9 − l0

l 0 = unstretched length of the spring 
⇒ l 0 = 6 cm
 
 m1V1 + m2V2
88. (a) V0 =
m1 + m2
Velocity of the two particles in COM frame
 
   m2 V1 − V2 ( )
V1cm = V1 − V0 =
m1 + m2
 
   m1 V2 − V1
V2 cm = V2 − V0 =
( )
m1 + m2
1 2 1 2
∴ KEcm = m1 V1cm + m2 V2 cm
2 2
Momentum and Center of Mass 5.67

1  m1m22 m2 m12    2
=
2  m + m
+  V1 − V2
 1 2 m1 + m 2 
1 m1m2 2 1 2
= Vrel = µVrel
2 m1 + m2 2

1
KEcm +
(b) ( m1 + m2 ) V02
2
  2
1 m1m2    m + m2   m1V1 + m2V2 
( )
2
= V1 − V2 + 1  m + m 
2 m1 + m2  2  1 2 

   
=
1
2(m1 + m2 ) 
1 2 1 ( 2 1 2 1 1 )
 m m V 2 + V 2 − 2V .V + m 2V 2 + m 2V 2 + 2 m m V .V 
2 2 1 2 1 2

1  m1V12 ( m2 + m1 ) + m2V22 ( m1 + m2 ) 
=
2 ( m1 + m2 )  
1 1
= m1V12 + m2V22 = KE in ground frame
2 2

(c) Using the above result when Vrel = 0, the KE is minimum.


1
∴ KEmin = ( m1 + m2 ) V02
3
89. The result obtained in the last problem will help us produce an easy solution to this problem.

In the COM frame (a reference frame travelling with the velocity of the centre of mass of the system) the initial
mechanical energy of the system is equal to the spring potential energy as velocity of both blocks is zero.

This energy gets converted into KE when the spring regains its natural length.

1 2 1 2
kx = µv r .....(i)
2 2
(m )(2m ) = 2 m
Here, = reduced mass of the blocks =
m + 2m 3
and vr = relative velocity of the two.
Substituting in Equation (1), we get

kx 2 = µvr2

2m 2 3k
kx 2 = vr ⇒ vr = x
3 2m
90. Maximum expansion in spring is given by

1 2 1

2
kx max = µv20
2
[µ = Reduced mass]
µ 2m
⇒ x max = . v0 = v0
k 3k
5.68 Problems in Physics for JEE Advanced

91. (a)

O cm I

r1 r2

Distance of COM of the system (disc + Insect)


from the centre (O) of the disc is
mR
r1 =
M+m
Distance of insect (I) from COM is
MR
r2 =
M+m
The COM of the system remains at rest and both the insect and the centre (O) of the disc move in circle of radii
r2 and r1 respectively. The angular speed of both is same so that O, CM and I always remain on a straight line.

(b) For m = M, both are equidistant from the COM. They will move along the same circle

I
R/2

R/2 cm
O

2R
92. The COM moves in a circle of radius r =
π
V
Angular speed of COM = angular speed of the wire ω =
R
2
Net force F = Macm = M. r
2 2
 V   2 R  2 MV
= M  ⋅   =
R  π  π R
V

cm

93. Let the wedge move by a distance x towards right by the time the block reaches the top of the incline.
Actual horizontal displacement of the block will be 2 cos 37° – x = 1.6 – x
COM of the entire system will remain undisplaced
Momentum and Center of Mass 5.69

⇒ m (1.6 − x ) = 3mx

1.6
⇒x= = 0.4m
4
Work done by tension on the wedge + motor system is
WT = ( T cos θ ) x = mg sin 37°.cos 37°. x

3 4
= 1 × 10 × × × 0.4 = 1.92 J .
5 5
3h
94. The cream is like a solid cone of mass 4M and height h. Its COM is at a distance of from the apex.
2h 4
The biscuit cone is hollow and its COM is at a distance from the apex. Here I will derive the result for an empty
3
cone and doing the derivation for a solid cone is being for students.
We divide our hollow cone into multiple rings as shown in the fig.

r
d

y
l

r = y tan q l = y sec θ

d l = (dy ) sec θ

Mass of the ring element is

dm = σ ( 2π rd l )

dm = 2πσ . y tan θ sec θ dy
h 2
∫ ydm 2π tan θ sec θ .σ ∫ y dy
o
∴ ycm = =
σ .π Rl σ .π Rl
2 hh
= tan θ .sec θ .h
3 Rl
h h 2h
Put = cot θ and = cosθ to get ycm =
R l 3

2h
The cone can be replaced with a particle of mass M at a distance from the apex and the cream can be replaced
3h 3
with another point mass 4M kept at a distance from the apex.
4

3h 2h
4 M. + M.
∴ ycm = 4 3 = 11h
5M 15
5.70 Problems in Physics for JEE Advanced

95. (a) The centre of mass gets closer to O.


(b) For semicircle, the distance of COM from O is
2R
y1 = = 0.64 R
π
πR
For V shaped rope l = BC =
2
A O B
A B
2R h
p 2
cm cm
l

h = OC = l2 − R 2

π2
= − 1 R = 1.21R
4
Distance of COM from O is
h
y2 = 2 = 0.61R
Shift y = y1 – y2 = 0.03 R
96. The two spring have same force constant but different tensions. Therefore, they must have different natural lengths.
Let the natural lengths of the upper and lower springs be L1 and L2 respectively.
Spring forces in equilibrium are F2 = mg
mg
⇒ Kx2 = mg ⇒ x2 = = x0 ( say )
K
m m
And F1 = mg + F2 & F1 = 2mg

2 mg 
∴ x1 = = 2 x0 k
K
Given, L1 + 2x0 = L2 + x0 & L2 = L1 + x0
m
When springs are in natural length, distance of centre of mass from the top ball will be 
0 + mL1 + m ( L1 + L2 )  k
xcm =

3m
2 L1 + L2 2 L1 + L1 + x0 x m
= = = L1 + 0
3 3 3
x0 mg
COM is at a distance of (i.e., ) from the second ball
3 3K
97. (b) Distance of COM of the chain from O is
R sin α R sin 45° 2 2 R
r= = =
α π π
4
Momentum and Center of Mass 5.71

The COM of the chain moves on a circle of radius r as the chain slides.
oooooo
ooo
oo
C1

oo
oo
r C2

ooo
0
45 r

oo
O

C1 and C2 are initial and final positions of COM of the chain


Energy conservation

1
mV 2 = mgr [1 − cos 45°]
2
 1  2 2R 2 − 1
∴ V 2 = 2 gr  1 −  = 2g
 2 π 2

V=
4 ( )
2 − 1 gR
π
Angular speed of COM and the chain must be same

∴ Vcm =
Vr 2 2
=
4 ( )
2 − 1 gR
R π π
(c) COM of the system (chain + Half cylinder) will suffer no displacement along the horizontal direction.
If X is displacement of the half cylinder towards left, then m (r sin 45° – X) = mX
r R
\X = =
2 2 p
V0
98. The COM of the (disc + body) system moves in a straight line with velocity
2
In COM frame, the body and the centre of the disc always have equal and opposite velocity. In this frame, the body
R
(A) and the centre of the disc (C) move in circle of radius with angular speed given by –
2
V / 2 V0
ω = 0 =
R/2 R

A Vo /2

R/2
CoM
R/2

C
(a) The velocity of the centre (C) will become zero in ground frame after it has completed one rotation in the COM
frame.
2πR / 2 2πR
∴ t0 = =
V0 / 2 V0
t0 2π
(b) In time , the body A rotates through radian in COM frame.
3 3
5.72 Problems in Physics for JEE Advanced

Its velocity in this frame is as shown.

Vo
C Vcm = 2
2/3
1200

Vo /2 Vo
2
In ground frame velocity of A is obtained by adding the velocity of COM to this velocity.

V0
Resultant of two vectors also has a magnitude of .
2
99. Mass of water melon eaten by Majnu is

M M 1 1 M 1 1 1 1
mm = + . . + . . . . + .....
2 2 2 2 2 2 2 2 2
M M M
= + + + ................ + .......∞ terms
2 8 32
M
2M
= 2 =
1 3
1−
4
Mass eaten by Laila
M
mL
3
2M
So, a mass has effectively moved from
3
left end of the boat to its right end. Let the boat move to the left by x
2
ML
2M  M 3
( L − x ) =  M0 +  x ∴ =x
3  3  M0 + M
2 ML
x=
3 ( M0 + M )

100. (a) Let man move down with velocity Vm (wrt ground) and the balloon go up with velocity Vb (wrt ground)
Vm + Vb = u & Vb = u – Vm
Momentum conservation gives –
mVm = M (u – Vm)
Mu
∴ Vm =
M+m
mu
And Vb =
M+m
As COM of the system does not move, we have –

mX m ( ↓ ) = M X b ( ↑ ) ..................(a)
Momentum and Center of Mass 5.73

Change in PE = – mgXm + MgXb = 0 [using (a)]


Both man and balloon move with constant speed
there is no change in KE.
KE + PE = const
(b)
FB = (M + m)g

mu
PB = FB .Vb = ( M + m ) g. = mgu
M+m
(c) Heat is generated in rubbing.

101. COM of the block + wedge system will not get displaced in x and y direction.
If x0 and y0 are components of displacement of the wedge in the shown directions, then m (R – x0) = 10 mx0



s
co
R

R
⇒ x0 = ...........(i)
11
And m [R cos q – y0] = 10 my0
R cos θ R 11 11
⇒ y0 = = = R .........(ii)
11 11 5 55
Displacement of wedge is
6 R 6 × 0.55
r = x02 + y02 = = = 0.06 m
55 55
= 6 cm
(b) Let the velocity of block and wedge at the instant block leaves the wedge be
 
vb = −v1 ˆj − v2 kˆ and vw = v0 . ˆj
Momentum conservation along y direction
mv1 = 10 mv0 & v1 = 10v0 ............ (i)
Velocity of block relative to wedge
  
vbw = vb − vw = − ( v0 + v1 ) ˆj − v2 kˆ
This velocity makes an angle

11
θ = cos−1 with horizontal
5
5.74 Problems in Physics for JEE Advanced

v2
∴ tan θ =
v0 + v1

14 v
= 2 ⇒ v2 = 154v0 .........(ii)
11 11v0
Mechanical en
nergy conservation
1
2
( )
1
m v12 + v 22 + × 10 mv02 = mgR sin θ
2
14
(10v0 )2 + 154v02 + 10v02 = 2 × 10 × 0.55 ×
5
264 v02 = 8.23 ∴ v0 = 0.18 m/s
102. (a) Since there is no external force, in horizontal direction on the system, its COM will not move.
Let the bar move a distance x towards left by the time 1st insect reaches B.
6
Then 4mx = m ( 6a − x ) ⇒ a=x
5

A m
B
C D

5a

6a 4a
Length hanging out of the table (on right) is 2 a − =
5 5
a
Length hanging out on left =
5
The bar will topple if the vertical line through centre of mass of the system passes outside the table. The COM
of the system lies at a distance
m × 0 + 4m × 3a
from end B
5m
12 4a
= a>
5 5
The bar will not topple.

(b) When the second insect with large mass sits at end A, the bar has a tendency to topple about C (see figure ). If
M increases COM of the system shifts to left. M is maximum (for not toppling) when COM is at C

M × 0 + 4m × 3a + m × 6a
Distance of COM from A =
M + 5m
M m
A
B
C D
5a

a 18ma
⇒ =
5 M + 5m
Momentum and Center of Mass 5.75

& M + 5m = 90 m
& m = 85 m
MR
R
103. (a) The initial co-ordinates of COM of the system is x = 2 =
2M 4
M
.R
R
y= 2 =
2M 4

M P
2

X
M M
2


When the men are at centre (C), the centre must have shifted to the original position of the COM.
Displacement of centre of the disc
R
= x 2 + y2 =
2 2
(b) In our co-ordinate system (whose origin is fixed to the table at original position of the centre of the disc), initial

R
distance of COM from origin is rCM
2 2
Y

men

CM

C
r

 X
O
Let the centre of the disc translate by r. The distance of point P from origin is R + r

M ∆r + M ( R + ∆r ) R
∴ =
2M 2 2
∴ [2∆r + R ] 2 = R
2 2∆r = − R ( 2 −1 )
⇒ ∆r = −
R ( 2 −1 )
2 2

Centre moves by a distance


R ( 2 −1 )
2 2
in direction opposite to that shown in the figure.

5.76 Problems in Physics for JEE Advanced

104. Let the impulse of tension in the string AB and BC be I1 and I2 respectively.
Particle C receives an impulse along CB. There is no other force on it. Therefore, it will start moving along CB. Let
its initial velocity be v
m
v C

I m

A B

For : C mv = I2 ……….(i)

For : A mu = I – I1   …….(ii)
Where I = external impulse applied
C

A
u u B



Component of velocity of B in direction – BA must be u
Let its velocity component perpendicular to BA be v1
Velocity component of B along CB is – v1 cos 30° + u sin 30°
This must be equal to v (= velocity of C)
3 u
∴− v1 + = v...............(iii)
2 2
Let’s write impulse momentum theorem for B in two perpendicular directions –BA and perpendicular to it.

I1 – I2 cos 60° = 2mu
I
I1 − 2 = 2mu

2
2 I1 − I 2 = 4mu ...................(iv)
And I2 cos 30° = 2mv1
3
I2 2mv1 .................(v)
2
We need to solve the set of equations (i), (iii) and (v).
(i) and (v) gives
3mv = 4 mv1 ⇒ 3v = 4v1

put in (iii)
Momentum and Center of Mass 5.77

3 3  u
− v + =v
2  4  2
11 u 4u
v= ⇒v=
8 2 11
3v 3u
and v1 = =
4 11
Speed of B is

2 2 2 31
vB = u + v1 = u
11
105. It is like 2 m mass hitting another 2 m mass (at rest) head on.
u
m
2m

m
u

106. During each elastic collision the balls will exchange velocities. It is given that all balls are identical (the two
colliding balls have no names), we could say that the balls simply pass through each other moving independent of
each other. If a ball was at the edge it will take longest time to fall off the other edge
L
∴ tmax =
v
107. Let v1 = velocity of ball at the top

v2 = velocity of the block at this instant


Momentum conservation gives:
mv2 – mv1 = mu
v2 – v1 = u ...............(i)
Energy conservation gives
1 2 1 2 1
mv1 + mv2 + mg (2 R ) = mu2
2 2 2
2 2 2
v1 + v2 = u – 4 gR...............(ii))
Squaring (i) v22 + v12 − 2v1v2 = u2
& u2 – 4gR – 2v1 v2 = u2
& 2v1 v2 = –4gR [This product can be negative !]
Now, (v2 + v1)2 = v22 v12 4v1v2
5.78 Problems in Physics for JEE Advanced

(v2 + v1)2= u2– 4gR – 8gR


(v2 + v1)2 = u2– 12gR
v2 + v1 = ± u2 − 12 gR

Negative sign is not acceptable since v1 + v2 is velocity of ball relative to point A, which cannot be towards right.

∴ v2 + v1 = u2 − 12 gR ......................(iii)

Solving (i) and (iii) we get v2 and v1


At the top, string is vertical. Therefore, there is no acceleration of the block (i.e., point A). With respect to point A,
the ball is rotating in a circle of radius R with speed

v1 ( ← ) − v2 ( → ) = v1 + v2 ( ← ) = u2 − 12 gR

∴ T + mg =
(
m u2 − 12 gR )
R
2
mu
∴T = − 13 mg
R

108. (a) We will apply conservation of mechanical energy.


[since peg is small, we will neglect the semicircular part of rope lying over it]
Loss in PE = Ui – Uf

U=0
2L-x
x

L  x  2L − x  
= −λLg × 2 −  −λxg − λ (2 L − x ) g 
2  2  2  
λx 2 g λg
= −λL2 g +
2
+
2
(
4 L2 + x 2 − 4 Lx )
= λL2 g + λx 2 g − 2λgLx
1
∴ (2λL ) v2 = λg  L2 + x 2 − 2 Lx 
2
g
v2 = ( x − L )
2
L
g
v= (x − L)
L
dv g dx
2v = 2 ( x − L )
(b) dt L dt
dv g
a= = (x − L)
dt L
P = lxv – l(2L –x) v = 2lv (x – L)
(c)
Momentum and Center of Mass 5.79

dp dx dv
= 2λv + 2λ ( x − L )
dt dt dt

= 2lv2 + 2l (x – L)a

g (x − L)
= 2λ ( x − L )2 + 2 λ ( x − L ) g
L L
4λg 2 mg
= ( x − L )2 = 2 ( x − L )2
L L

dp
mg − F =
(d)
dt
F = force applied by the peg on the rope
dp 4λg
∴ F = mg − = mg − ( x − L )2
dt L
  ( x − L) 
2
= mg 1 − 2  2  
  L  
(e)
F = 0 for
x−L 1
=
L 2

L
⇒x=L+
2
109. (a) The particle starts moving down maintaining its contact with the inner ring. The centre of the ring moves to
right (the normal force applied by the particle on the ring has a rightward component). Later the particle loses
contact with the inner ring and gets into contact with the outer ring. Now the ring experiences a horizontal
force towards left. It will come to rest and then start moving towards left. This can also be seen knowing that
the COM of the system shall not get displaced horizontally.
(b) vp = velocity of particle at B
vw = velocity of wheel when particle is at B
mvp (Æ) = Mvw (¨) [Momentum conservation in horizontal direction]
Energy conservation:

1 2 1
mv p + Mvw2 = mg.2 R
2 2
2 gR
1  Mvw  1 ∴ vw = 2m
m
2  m 
+
2
Mv w
2
= 2 mgR

M ( + m)
M
(c) Let v = speed of particle with respect to the wheel
vw = speed of the wheel
When particle is in contact with the inner wheel, the wheel is moving to right.
Note: velocity of particle in ground frame is
  
v p = v + vw

v p = v 2 + vw2 + 2vvw + sin (180 − θ )

= v 2 + vw2 + 2vvw sin θ



5.80 Problems in Physics for JEE Advanced

Velocity component of the particle in horizontal direction


vx = v cos q – vw (Æ)
Momentum conservation in Horizontal direction gives –
m (v cos q – vw) = Mvw

v =
( M + m ) vw
m cosθ
Energy conservation

1 2 1
mv + Mvw2 = mgR [1 − cosθ ]
2 2
( M + m )2
2
vw2 + Mvw2 = 2mgR [1 − cosθ ]
m cos θ
 ( M + m )2 + Mm cos2 θ 
vw2   = 2mgR [1 − cosθ ]
2
 m cos θ 
 
2m 2 gR cos2 θ (1 − cosθ )
vw2 =
( M + m )2 + Mm cos2 θ
2m 2 gR cos2 θ (1 − cosθ )
vw =
( M + m )2 + Mm cos2 θ


v

110. (a) Speed of block 1 grows linearly from zero to v1 before hitting the block 2.
F 2 Fd
v12 = 02 + 2 d ⇒ v1
m m

Block 1 hits block 2 with speed v1. Block 1 comes to rest and 2 starts moving with velocity v1. Block 1 will
again begin to accelerate as the force is still acting on it. Block 2 will move a distance d, hit block 3 and come
to rest. By the time block 1 covers distance d and is about to hit block 2, it will once again have velocity equal
to v1. It will hit 2 and come to rest. The process continues like this.
Momentum and Center of Mass 5.81

(b) Block 1 will hit 2 and both will stick. 1 + 2 will hit 3 and 1 + 2 + 3 will move together. The process continues.
Let u1,2,3......,n represent the velocity of 1 + 2 + 3 + ....+ n immediately after the nth block (is hit and) starts
moving.
Let v1,2,3,......, n represent the velocity of 1 + 2 + 3 + .......+ n just before the moving system is about to hit the
(n + 1)th block.
u1 = 0
F 2 Fd
v1 = u12 + 2 d ⇒ v1 =
m m
Momentum conservation gives 2 mu12 = mv1
Fd
⇒ u12 =
2m

2 F 3Fd
v12 = u12 +2 d ⇒ v12 =
2m 2m
Again momentum conservation gives
2 3 Fd 2 Fd
u123 = =
3 2 m 3m
2 2 F 4 Fd
v123 = u123 + 2. .d ⇒ v123 =
3m 3 m
After ‘n’ collision (n + 1) blocks will be moving together
n Fd
∴ u123...................( n +1) =
n +1 m
n
If n is large →1
n +1
Fd
∴ u123.........( n +1) →
m
111. Position of the balls at a later time has been shown.
2T cos θ = F ⇒ T = F ...............(i)
2 cos θ
x = l sin q
dx = l cos q dq  ........(ii)
For motion of a ball in x direction
F

 y
T
T

y T
T
x

T sin θ = m dvx ⇒ v dvx = T sin θ


x
dt dx m
T
⇒ vx dvx = sin θ l cosθ dθ [ using (ii)]
m
Fl
= sin θ dθ [ using (i)]
2m
vx Fl 0
∴ ∫ vx dvx = −
2m ∫π / 2
sin θ dθ
0
5.82 Problems in Physics for JEE Advanced

[we have placed negative sign as q is decreasing]


vx2 F l Fl
= fi vx =
2 2m m
Relative velocity of approach before collision
Fl
= 2vx = 2
m
[The two balls will always have zero relative velocity in y direction]
It follows from momentum conservation that after collision the two balls will lose all their velocity in x direction.
1
loss in KE = 2 × mvx2 = Fl
2

112. (a) Amplitude of oscillation

Normal
Length
x1

x2

Equilibrium with
block

Equilibrium with
block + insect

(b) As the insect jumps the block recoils with same speed (Momentum conservation)
M
Speed of recoil v = u = g
K

Normal
Length
x1

x2
u
x3
Equilibrium with
block

Equilibrium with
block + insect

Extreme position for block


Momentum and Center of Mass 5.83

Applying energy conservation


1 2 1 1
Kx3 − Mg ( x3 − x2 ) = Kx22 + Mv 2
2 2 2
2 2 2 2
1 2 2M g 2M g M 2 g2
Kx3 − Mgx3 + = +
2 K K 2K
2 2
2 Mg M g
x32 − x3 − =0
K K2
2 Mg 4 M 2 g2 4 M 2 g2
± 2
+
x3 =
K K K2
2
Mg 
= 1 + 2 
K 
[Negative sign is not acceptable]
Mg
Amplitude A = x3 − x1 = 2
K
(c) The spring is stretched by x1 when the insect strikes. Speed after the hit
u g M
v = =
2 2 K
Let stretch in spring be x4 at extreme position
Energy conservation:
1 2 1 1
Kx + 2 Mv 2 = Kx42 − 2 Mg ( x4 − x1 )
2 1 2 2
2 2 2 2

1M g M g 1 2 M 2 g2
+ = Kx42 − 2 Mgx4 +
2 K 4K 2 K
2 2
Mg 5M g
x42 − 4 x4 + =0
K 2 K2
4 Mg M 2 g2
± (16 − 10 )
∴ x4 =
K K2
2
Mg  6
= 2 + 
K  2 

6 Mg
Amplitude A = x4 – x2 =
2 K
113. Speed of both balls just before A collides with floor is v0 = 2gh
Ball A rebounds with velocity v0
2v0
  2v0

A
5.84 Problems in Physics for JEE Advanced

Before colliding with B, A sees it moving with velocity 2v0 (Ø)



relative velocity of approach = relative velocity of separation
In reference frame of A, the velocity of ball B has been shown in the figure. After collision it is 2v0 making an angle
2q with vertical. Note that the true velocity of B has a vertical component equal to 2v0 cos 2q v0
2vx vy 2 [ 2v0 sin 2θ ][ 2 v0 cos 2θ + v0 ]
x= =
g g
8v02 1 
= sin 2θ  + cos 2θ 
g  2 
 1 
= 16h sin 2θ  + cos 2θ 
2 

114. The line of impact makes an angle of 60° with the line C1C2
3r 3
sin θ = =
2r 2
vB

r
C2
 R

P C2 60 30
30 60 C1
C3
C3

vC

Figure 1 Figure 2

Disc B will move along this line of impact and C will move perpendicular to this. From figure 2
(R + r) 2(R + r)
CC = =
1 2 cos 30° 3
(R + r)
PC1 = = 2(R + r)
cos 60°
2(R + r)
∴ PC2 = 2 ( R + r ) −
3

2r
But PC2 = [ In ∆PC2C3 ]
sin 30°
= 4r
 1 
∴ 4r = 2 ( R + r ) 1 − 
 3
2 3r
= R+r
3 −1
 3 +1
R=
 3 − 1 
r
 
Momentum and Center of Mass 5.85

m 2m

115.
In COM frame, before collision, the situation appears as shown below–
m 2m
2 /3
/3
u
In lab frame, the x component of velocity of the two particles is vx = vcm =
3
[Because x component of momentum = mu]
Since COM is moving along x, the velocity components in y direction are same in lab frame as well as COM frame.
After collision, particles travel along y in COM frame. Kinetic energy is conserved in COM frame also.

COM frame

2m

2
1 2 1  vy  1  2u 
2
1  u
2
mvy + 2 m   = m   + 2 m  
2 2  2 2  3 2  3
3vy2 2 2
= u2 ⇒ v y = u
2 3 3
In lab frame the situation will appear as shown below.
2m vx  u / 3
45

v2
vy  u / 3

116.
M u m
x Before collision

 After collision
x


m
5.86 Problems in Physics for JEE Advanced

Momentum conservation along x:


Mv1 cos q + mv2 cos f = Mu


& mv cos f = Mu – Mv cos q ...............(i)
2 1
Momentum conservation along y:
mv2 sin f = Mv1 sin q ...............(ii)
Squaring and adding equation (i) & (ii) so as to eliminate f
m 2 v22 = M 2 u2 + M 2 v12 − 2 M 2 uv1 cosθ

K 2 v22 = u2 + v12 − 2uv1 cos θ .....................(iii)
m
[Where K = ]
M
Kinetic energy conservation:

1 1 1
Mv12 + mv22 = Mu2
2 2 2
2 2 2
⇒ Kv2 = u − v1 .................(iv)
Putting the value of v22 in (iii)

( )
K u2 − v12 = u2 + v12 − 2uv1 cosθ


⇒ (1 + k ) v12 − ( 2u cosθ ) v1 + (1 − K ) u2 = 0

For v1 to be real, discriminant of this quadratic equation must be zero or positive


4u2 cos2q – 4 (1 – K2) u2 > 0
cos2 q – 1 + K2 > 0
K2 > sin2q
K > sin q

m
sin −1 >θ
 M 
m
∴θmax = sin −1  
M

M u m
Alternate: x

Mu
vcm =
M+m
In COM frame,
Mu mu
velocity of M = u − =
M+m M+m
Mu
velocity of m = 0 − vcm = −
M+m
Momentum and Center of Mass 5.87


Before collision in COM frame
M
m
  Total Momentum = 0

After collision, in COM frame, the two balls will continuous to move with same speed (otherwise KE will not be
conserved) in opposite directions
After collision in COM frame
v1cm
M


x

CoM
m

v2cm

q can assume any value.
  
Velocity v1 of mass M in lab frame can be obtained by v1 = v1cm + vcm
  
Let OA represent vcm . Keeping the tail of v1cm at A we can rotate it so that its tip, B moves on a circle of radius

 mu  
v1cm = . (This is because q can assume any value.) Vector OB gives v1 . The angle of
M+m
deflection is largest when OB is tangent to the circle
B3 B

B2 v1
v1cm
v1cm  max
B1
O v cm A O v cm A

Êv ˆ Ê mˆ
\ d max = sin -1 Á 1cm ˜ = sin -1 Á ˜

Ë vcm ¯ Ë M¯

117. As the steel ball falls on the tennis ball, it compresses it. The tennis ball gets deformed and air pressure inside it
increases. As soon as the steel ball comes to rest the tennis ball begins to expand and it pushes the steel ball up. The
moment the steel ball separates from the tennis ball the upper point of the tennis ball has same speed as the steel
ball but the lower point of the ball (in contact with the floor) is at rest. Therefore, the COM of the tennis ball will
have a nonzero velocity and it will bounce.
06 ROTATIONAL MOTION

Q. 3. A ring is rolling, without slipping on a horizontal


Level 1 surface with constant velocity. Speed of point A
Q. 1. The pulley of radius R can rotate freely about its (at the top) is vA. After an interval T, the speed of
axle as shown in the figure. A thread is tightly point A again becomes vA. During what fraction
of the interval T speed of point A was greater than
wrapped around the pulley and its free end carries
a block of mass m. When the block is at a height 3 .
vA
h above the ground the system is released (i.e., 2
the pulley is made free to rotate & the block is A

allowed to fall) and at the same instant the axle is


moved up keeping it horizontal all the time. When
the block hits the floor the axle has gone up by a
distance 2h. Find the angle by which the pulley
must have rotated by this time.

Q. 4. Calculate the ratio of moment of inertia of a thin


uniform disc about axis 1 and 2 marked in the
R figure. O is the centre of the disc.
2

30°
O 45°

h
1

Q. 5. A uniform circular disc has a sector of angle 90°


Q. 2. A disc is rolling without sliding on a horizontal removed from it. Mass of the remaining disc is M.
surface. Velocity of the centre of the disc is v. Find Write the moment of inertia of the remaining disc
the maximum relative speed of any point on the about the axis xx shown in figure (Radius is R)
circumference of the disc with respect to point P. x

R
30° O
v

x
6.2 Problems in Physics for JEE Advanced

Q. 6. An Indian bread ‘‘Roti’’ is a uniform disc of mass the plate about the axes 1, 2, 3 and 4 respectively.
M and radius R. Before eating a person usually Axes 1 and 2 are diagonals and 3 and 4 are lines
folds it about its diameter (say about x axis). After passing through centre parallel to sides of the
folding it a sector of angle 60° is removed from square. The moment of inertia of the plate about
it. Find the moment of inertia of the remaining an axis passing through centre and perpendicular
‘‘Roti’’ about Z-axis. to the plane of the figure is equal to which of the
Z Z followings.
(a) I3 + I4 (b) I1 + I3
X X
1
(c) I2 + I3 (d) (I + I + I + I )
2 1 2 3 4
O O 60°
4

2 1

3
O
Q. 7. A uniform rectangular plate has side length
 and 2 . The plate is in x – y plane with its
centre at origin and sides parallel to x and y axes.
The moment of inertia of the plate about an axis
passing through a vertex (say A) perpendicular Q. 10. An asteroid in the shape of a uniform sphere
to the plane of the figure is I0. Now the axis is encounters cosmic dust. A thin uniform layer of
shifted parallel to itself so that moment of inertia dust gets deposited on it and its mass increases
about it still remains I0.Write the locus of point of by 2%. Find percentage change in its moment of
intersection of the axis with xy plane. inertia about diameter.
y
Q. 11. (i) Consider an infinitesimally thin triangular
strip having mass M and length L. Find the
A moment of inertia of the strip about on axis
passing through its tip and perpendicular to
l
x the plane. Compare the result with moment
O
of inertia of a uniform disc of mass M and
radius L about an axis passing through its
2l
centre and perpendicular to the plane the
disc. Why the two expressions are same?
Q. 8. A thin semi circular cylindrical shell has mass M
and radius R. Find its moment of inertia about a L
line passing through its centre of mass parallel to (ii) A circular fan made of paper is in shape of
the axis (shown in figure) of the cylinder. a disc of radius R. The fan can be folded
axis (various stages shown in figure (a) through
(f)) to the shape of a thin stick. The moment
of inertia of the circular fan about an axis
passing through centre O and perpendicular
1
to the plane of the figure is I 0 = MR 2 where
M = mass of the fan. 2
a b
R a b
R
a O R b
O
O
Q. 9. Consider a uniform square plate shown in the
figure. I1, I2, I3 and I4 are moment of inertia of (a) (b) (c)
Rotational Motion 6.3

O O O P

R
R R
a b A
a b ab
(d) (e) (f)

(a) How does the moment of inertia (I), about an B


axis perpendicular to the plane of the figure
passing through O, change as the fan is folded
through stage a to b to c to d to e?
C
(b) When the fan is completely folded in the
shape of a stick (fig. (f)), write its moment of
inertia about the above mentioned axis. Q. 15. Determine the moment of inertia of the shaded
Note : Moment of inertia of a uniform rod about area about y axis. The mass of the shaded area is
an axis through its end and perpendicular to it is M.
2
y
ML
.
3

Q. 12. A uniform rectangular plate has moment of inertia


about its longer side, equal to I. The moment of
inertia of the plate about an axis in its plane, h1 h2
passing through the centre and parallel to the
shorter sides is also equal to I. Find its moment x
of inertia about an axis passing through its centre L
and perpendicular to its plane. Q. 16. Two uniform semicircular discs, each of radius
Q. 13. A uniform rectangular plate has been bent as R, are stuck together to form a disc. Masses of
shown in the figure. The two angled parts of the the two semicircular parts are M and 3M. Find
plate are of identical size. The moment of inertia the moment of inertia of the circular disc about
of the bent plate about axis xx is I. Find its moment an axis perpendicular to its plane and passing
of inertia about an axis parallel to xx and passing through its centre of mass.
through the centre of mass of the plate.
D

M 3M
C
x
B
b
E
Q. 17. A stick AB of mass M is tied at one end to a light
60° A string OA. A horizontal force F = Mg is applied at
O end B of the stick and its remains in equilibrium
x in position shown. Calculate angles a and b.
O
Q. 14. Three identical rings each of mass M and radius
R are welded together with their planes mutually b
perpendicular to each other. Ring A is vertical
and B is also vertical in a plane perpendicular to
A. Ring C is in horizontal plane. Find moment A
of Inertia of this system about a horizontal axis a
B
perpendicular to the plane of the figure passing F
through point P (top point of ring A)
6.4 Problems in Physics for JEE Advanced

Q. 18. When brakes are applied on a moving car, the car (b) Does the gravitational potential energy of the
dips to the front. Why ? [That is try to show that ball increases or decreases as it climbs the
front wheels are more pressed as compared to rear step?
ones when the brakes are applied]. Assume that Q. 22. The centre of mass of an inhomogeneous sphere is
centre of mass of the car is equidistant from the at a distance of 0.3 R from its geometrical centre.
front and rear wheels. R is the radius of the sphere. Find the maximum
Q. 19. A uniform wire has been bent in shape of a inclination (q) of an incline plane on which this
semi circle. The semicircle is suspended about a sphere can be placed in equilibrium. Assume that
horizontal axis passing through one of its ends, friction is large enough to prevent slipping.
so that the semicircular wire can swing in vertical
Q. 23. Rectangular block B, having height h and width
plane. Find the angle a that the diameter of the
d has been placed on another block A as shown
semicircle makes with vertical in equilibrium.
in the figure. Both blocks have equal mass and
A
there is no friction between A and the horizontal
surface. A horizontal time dependent force
a
F = kt is applied on the block A. At what time
will block B topple? Assume that friction between
the two blocks is large enough to prevent B from
slipping.
d

B B h

Q. 20. A uniform cylindrical body of radius r has a


A F
conical nose. The length of the cylindrical and
conical parts are 4r and 3r respectively. Mass
of the conical part is M. The body rests on a Q. 24. A cylinder C rests on a horizontal surface. A
horizontal surface as shown. A ring of radius r small particle of mass m is held in equilibrium
2
connected to an overhanging string as shown. The
is to be tightly fitted on the nose of the body. What other end of the mass less string is being pulled
is maximum permissible mass of the ring so that horizontally by a force F as shown. Find F.
the body does not topple?
F
3r 4r
r

Q. 21. There is a step of height h on an incline plane. A


The step prevents a ball of radius R from rolling
down.
(a) If the inclination (q) of the incline is increased Q. 25. A hollow cylindrical pipe of mass M and radius R
gradually, at what value of q the ball will just has a thin rod of mass m welded inside it, along
manage to climb the step? its length. A light thread is tightly wound on the
surface of the pipe. A mass m0 is attached to the
R end of the thread as shown in figure. The system
stays in equilibrium when the cylinder is placed
such that a = 30°. The pulley shown in figure is a
M
disc of mass .
2
(a) Find the direction and magnitude of friction
q force acting on the cylinder.
h
Rotational Motion 6.5

(b) Express mass of the rod ‘m’ in terms of m0 and it can rotate freely in the vertical plane. There
Horizontal is a horizontal light rod fixed to the wheel below
the pivot. There is a small sleeve of mass m which
can slide along the rod without friction. The
M sleeve is connected to a light spring. The other
end of the spring is fixed to the rim as shown. The
 sleeve is at the centre of the rod and the spring
R is relaxed. Now the wheel is held at rest and the
m
sleeve is moved towards left so as to compress the
sprig by some distance. The sleeve and the wheel
are released simultaneously from this position.
Rough table A
mo (a) Is it possible that the wheel does not rotate as
the sleeve perform SHM on the rod ?
Q. 26. A sphere of radius R is supported by a rope
attached to the wall. The rope makes an angle (b) Find the value of spring constant k for
q = 45° with respect to the wall. The point where situation described in (a) to be possible. The
the rope is attached to the wall is at a distance of distance of rod from centre of the wheel is d.
3R
from the point where the sphere touches the
2 Wheel
wall. Find the minimum coefficient of friction
( between the wall and the sphere for this Pivot
Sleeve
equilibrium to be possible. O

string Spring d
45°
3R/2
Rod

R
Q. 29. A string is wrapped around a cylinder of mass
M and radius R. The string is pulled vertically
upward to prevent the centre of mass from falling
Q. 27. A uniform rod has mass M and length 4L. It rests as the string unwinds. Assume that the cylinder
in equilibrium with one end on a rough horizontal remains horizontal throughout and the thread does
surface at A. At point B, at a distance 3L from A, it not slip. Find the length of the string unwound
is supported by a fixed smooth roller. The rod just when the cylinder has reached an angular speed
remains in equilibrium when q = 30° w.
T
(a) Find the normal force applied by the
horizontal surface on the rod at point A.
(b) Find the coefficient of friction between the a
rod and the surface.

4L Mg

3L Q. 30. A mass less string is wrapped around a uniform


disc of mass m and radius r. The string passes
over a mass less pulley and is tied to a block of
q mass M at its other end (see figure). The system
A is released from rest. Assume that the string does
Q. 28. A wheel is mounted on frictionless central pivot not slip with respect to the disc.
6.6 Problems in Physics for JEE Advanced

(a) Find the acceleration of the block for the case of the string is attached to a small body of mass
M=m m and the other end is being pulled with a force
F. The circumference of the disc is sufficiently
rough so that the string does not slip over it. Find
acceleration of the small body.

F
R
Disc m
m
r m

M
M Q. 33. A uniform quarter circular thin rod of mass M
(b) Find for which the block can accelerate and radius R is pivoted at a point B on the floor.
m
It can rotate freely in the vertical plane about B.

upwards. It is supported by a smooth vertical wall at its
Q. 31. A solid uniform sphere of mass M and radius R other free end A so that it remains at rest. Find the
can rotate about a fixed vertical axis. There is no reaction force of wall on the rod.
frictional torque acting at the axis of rotation.
A light string is wrapped around the equator of A
the sphere. The string has exactly 6 turns on the
sphere. The string passes over a light pulley and
carries a small mass m at its end (see figure).
The string between the sphere and the pulley is R B
always horizontal. The system is released from
Q. 34. A ball is rolling without sliding down an incline.
rest and the small mass falls down vertically.
Is the force applied by the ball on the incline
The string does not slip on the sphere till 5 turns
larger than or less than its (ball’s) own weight ?
get unwound. As soon as 5th turn gets unwound
completely, the friction between the sphere and Q. 35. A solid sphere of mass M and radius R is covered
the string vanishes all of a sudden. with a thin shell of mass M. There is no friction
between the inner wall of the shell and the sphere.
(a) Find the angular speed of the sphere as the
The ball is released from rest, and then it rolls
string leaves it.
without slipping down an incline that is inclined at
(b) Find the change in acceleration of the small an angle q to the horizontal. Find the acceleration
mass m after 5 turns get unwound from the of the ball.
sphere.
Q. 36. A homogeneous solid sphere of radius R is resting
M on a horizontal surface. It is set in motion by a
horizontal impulse imparted to it at a height
h above the centre. If h is greater than h0, the
velocity of the sphere increases in the direction
of its motion after the start. If h < h0, the velocity
decreases after the start. Find h0

h
m

Q. 32. A disc of mass m and radius R lies flat on a


smooth horizontal table. A mass less string runs
halfway around it as shown in figure. One end
Rotational Motion 6.7

Q. 37. A boy pushes a cylinder of mass M with the help the spinning ceases.
of a plank of mass m as shown in figure. There Q. 39. A thin pencil of mass M and length L is being
is no slipping at any contact. The horizontal moved in a plane so that its centre (i.e. centre
component of the force applied by the boy on the of mass) goes in a circular path of radius R
plank is F. Find at a constant angular speed w. However, the
(a) The acceleration of the centre of the cylinder orientation of the pencil does not change in space.
(b) The friction force between the plank and the Its tip (A) always remains above the other end (B)
cylinder in the figure shown
m
(a) Write the kinetic energy of the pencil.
F
(b) Find the magnitude of net force acting on the
pencil.
M A

R
Q. 38. (i) A solid sphere of radius R is released on a
rough horizontal surface with its top point
having thrice the velocity of its bottom point
A (VA = V0) as shown in figure. Calculate the
linear velocity of the centre of the sphere
when it starts pure rolling. Q. 40. In figure (a) there is a uniform cylinder of
B
mass M and radius R. Length of the cylinder is
VB = 3V0 L = 3R. The cylinder is rolling without sliding
on a horizontal surface with its centre moving at
speed V. In figure (b) the same cylinder is moving
on a horizontal surface with its centre moving at
speed V and the cylinder rotating about a vertical
axis passing through its centre. [Place your pencil
on the table and give a sharp blow at its end.
A VA = V0
Look at the motion of the pencil. This is how the
(ii) Solid sphere of radius R is placed on a rough V
cylinder is moving]. The angular speed is ω = .
horizontal surface with its centre having R
velocity V0 towards right and its angular Write the kinetic energy of the cylinder in two
velocity being w0 (in anticlockwise sense). cases. In which case, the kinetic energy would
Find the required relationship between V0 have been higher if length of the cylinder were
and w0 so that - doubled (= 2 3R).
w

w0

C V C V
V0
R w
(a) (b)

Q. 41. There is a fixed hollow cylinder having
the slipping ceases before the sphere loses all smooth inner surface. Radius of the cylinder is
its linear momentum. R = 4m. A uniform rod of M = 4kg and length
L = 4m is released from vertical position inside
the sphere comes to a permanent rest after
the cylinder as shown in the figure. Convince
some time.
yourself that the rod will perform pure rotation
the velocity of centre becomes zero before
about the axis of the cylinder passing through O.
6.8 Problems in Physics for JEE Advanced

Fid the angular speed of the rod when its becomes the centre of its top edge to a mass less rod of
horizontal. length a. The rod can rotate about its upper end
only. In both cases the plate is pushed from its
equilibrium position so that centre of mass of the
plate acquires a speed V. In which case will the
O centre of mass of the plate rise to a greater height.
There is no friction
Q. 44. A thin carpet of mass 2m is rolled over a hollow
cylinder of mass m. The cylinder wall is thin and
radius of the cylinder is R. The carpet rolled over
Q. 42. A disc shaped body has two tight windings of light it has outer radius 2R (see figure). This roll is
threads - one on the inner rim of radius R = 1m and placed on a rough horizontal surface and given
the other on outer rim of radius 2R (see figure). It gentle push so that the carpet begins to roll and
is kept on a horizontal surface and the ends of the unwind. Friction is large enough to prevent any
two threads are pulled horizontally in opposite slipping of the carpet on the floor. Also assume
directions with force of equal magnitude F = 20N. that the carpet does not slip on the surface of the
Mass of the body and its moment of inertia about cylinder. The entire carpet is laid out on the floor
an axis through centre O and perpendicular to the and the hollow cylinder rolls out with speed V.
plane of the figure are M = 4kg and I = 8kg – m2 Find V.
respectively. Find the kinetic energy of the body 2
seconds after the forces begin to act, if
(i) the surface is smooth,
(ii) the surface is rough enough to ensure rolling
without sliding.
F

F R
2R
R

2R
O Q. 45. A uniform rod of mass M is moving in a plane
4
and has a kinetic energy of MV 2 where V is
3
speed of its centre of mass. Find the maximum
and minimum possible speed of the end point of
Q. 43. A uniform square plate has mass M and side
the rod.
length a. It is made to oscillate in vertical plane in
two different ways shown in figure (A) and (B). In Q. 46. The propeller of a small airplane is mounted in
figure (A), the plate is hinged at its upper corners the front. The propeller rotates clockwise if seen
with the help of two mass less rigid rods each of from behind by the pilot. The plane is flying
length a. The rods can rotate freely about both horizontally and the pilot suddenly turns it to the
ends. right. Will the body of the plane have a tendency
to get inclined to the horizontal? If yes, does the
nose of the plane veer upward or downward?
a a Why?
Q. 47. A massive star is spinning about its diameter with
p
an angular speed w 0 = rad/day . After its
1000
a
fuel is exhausted, the star collapses under its own
gravity to form a neutron star. Assume that the
volume of the star decreases to 10–12 times the
a original volume and its shape remains spherical.
fig. (a) fig. (b) Assuming that density of the star is uniform, find
In figure (B) the plate is rigidly connected at the angular speed of the neutron star.
Rotational Motion 6.9

Q. 48. A square plate of side length 2m has a groove of the platform after the man lies down.
made in the shape of two quarter circles joining at w0 w
the centre of the plate. The plate is free to rotate
about vertical axis passing through its centre.
The moment of inertia of the plate about this axis R/2
O O
is 4 kg – m2. A small block of mass 1 kg enters R
R/2
the groove at end A travelling with a velocity of
2m/s parallel to the side of the square plate. The
block move along the frictionless groove of the
horizontal plate and comes out at the other end B
Q. 51. A uniform block of mass M and dimensions as
with speed V. Find V assuming that width of the
shown in the figure is placed on a rough horizontal
groove is negligible.
surface and given a velocity V0 to the right. A is a
2m/s
point on the surface to the left of the block.
2m
A (a) Write the angular momentum of the block
about point A just after it begins to move
(b) Due to friction the block stops. What
happened to its angular momentum about
point A? Which torque is responsible for
change in angular momentum of the block?
B a

Q. 49. A disc of mass m and radius R lies flat on a smooth


horizontal table. A particle of mass m, moving b
horizontally along the table, strikes the disc with V0
velocity V while moving along a line at a distance A
R
from the centre. Find the angular velocity
2 Q. 52. ABCFED is a uniform plate (shown in figure).
acquired by the disc if the particle comes to rest ABC and DEF are circular arcs with common
after the impact. centre at O and having radii a and 2a respectively.
This plate is lying on a smooth horizontal table. A
particle of mass half the mass of the plate strikes
the plate at point A while travelling horizontally
V R/2
along the x direction with velocity u. The particle
hits the plate and rebounds along negative x with
u
m velocity . Find the velocity of point D of the
2
28
plate immediately after the impact. [Take  1]

Q. 50. A disc of mass M and radius R is rotating with y

angular velocity w0 about a vertical axis passing


M
through its centre (O). A man of mass
2 D
R
and height is standing on the periphery. The
2 E
A
man gradually lies down on the disc such that his
B
head is at a distance R from the centre and his
2
feet touching the edge of the disc. For simplicity
O x
assume that the man can be modelled as a thin C F
R a a
rod of length . Calculate the angular speed (w)
2
6.10 Problems in Physics for JEE Advanced

Q. 53. A uniform rod of mass m and length L is fixed (a) Find the velocity of A, B and C
to an axis, making an angle q with it as shown (b) Find the angular velocity of the body.
in the figure. The rod is rotated about this axis so y
that the free end of the rod moves with a uniform
speed ‘v’. Find the angular momentum of the rod B

about the axis. Is the angular momentum of the C


rod about point A constant?
x
A

Q. 56. A wheel is rolling without sliding on a horizontal


surface. Prove that velocities of all points on the
circumference of the wheel are directed towards
B
the top most point of the wheel.

VP
P

Q. 54. A mass m in attached to a mass less string and


swings in a horizontal circle, forming a conical Q. 57. There is a fixed half cylinder of radius R on a
pendulum, as shown in the figure. The other end horizontal table. A uniform rod of length 2R
of the string passes through a hole in the table leans against it as shown. At the instant shown,
and is dragged slowly so as to reduce the length l. q = 30° and the right end of the rod is sliding with
The string is slowly drawn up so that the depth h velocity v.
shown in the figure becomes half. By what factor
does the radius (r) of the circular path of the mass
m change?
R

O
 V
O
h l (a) Calculate the angular speed of the rod at this
m instant.
r
(b) Find the vertical component of the velocity of
the centre of the rod at this instant.
Q. 58. A disc of radius R is rolling without sliding on a
horizontal surface at a constant speed of v
Level 2 A

Q. 55. A flat rigid body is moving in x – y plane on


a table. The plane of the body lies in the x – y B
plane. At an instant it was found that some of the R/2
velocity components of its three particles A, B and V
C were VAx = 4m/s, VBx = 3m/s and Vcy = – 2m/s,
respectively. At the instant the three particles A, R
B and C were located at (0,0) (3,4) , (4,3) (all
in meter ) respectively in a co-ordinate system
attached to the table.
Rotational Motion 6.11

(a) What is speed of points A and B on the B C A


R
vertical diameter of the disc ? Given AB R
2
(b) After what time, for the first time, speed of
point A becomes equal to present speed (i.e.,
the speed at the instant shown in the figure) of
Q. 62. A wheel of radius R is rolling without sliding
point B?
uniformly on a horizontal surface. Find the
Q. 59. A uniform disc of radius R = 2 3 m is moving radius of curvature of the path of a point on its
on a horizontal surface without slipping. At some circumference when it is at highest point in its
instant its angular velocity is w = 1 rad/s and path.
angular acceleration is a = 3 rad/s2.
Q. 63. A wall is inclined to a horizontal surface at an
A
angle of 120° as shown. A rod AB of length L =
0.75 m is sliding with its two ends A and B on the
horizontal surface and on the wall respectively. At
C the moment angle q = 20° (see figure), the velocity
r
of end A is vA = 1.5 m/s towards right. Calculate

P the angular speed of the rod at this instant.
[Take cos 40° = 0.766]
B B

(a) Find acceleration of the top point A.


L
(b) Find acceleration of contact point B.
(c) Find co - oridnates (r, q) for a point P which
120°
has zero acceleration. q VA

Q. 60. A convex surface has a uniform radius of curvature A


equal to 5R. A wheel of radius R is rolling without Q. 64. In the figure the plank resting on two cylinders is
sliding on it with a constant speed v. Find the horizontal. The plank is pulled to the right such
acceleration of the point (P) of the wheel which is that the centre of smaller cylinder moves with a
in contact with the convex surface. constant velocity v. Friction is large enough to
V prevent slipping at all surfaces. Find-
R
(a) The velocity of the centre of larger cylinder.
P (b) The ratio of accelerations of the points of
contact of the two cylinders with the plank.
5R

V
R

Q. 61. AB is a non uniform plank of length L = 4R with 2R


its centre of mass at C such that AC = R. It is
placed on a step with its one end A supported by a
cylinder of radius R as shown in figure. The centre Q. 65. A wire of linear mass density (kg/m) is bent into
of mass of the plank is just outside the edge of the M

step. The cylinder is slowly rolled on the lower


step such that there is no slipping at any of its
contacts. Calculate the distance through which
the centre of the cylinder moves before the plank
q0 q 0
loses contact with the horizontal surface of the
upper step. O
6.12 Problems in Physics for JEE Advanced

an arc of a circle of radius R subtending an angle Mass of the plate with one big and eight small
2q at the centre. Calculate the moment o inertia holes is M. Find its moment of inertia about an
of this circular arc about an axis passing through axis passing through its centre and perpendicular
its midpoint (M) and perpendicular to its plane. to its plane.
Q. 66. A metallic plate has been fabricated by welding
Q. 68. ABC is an isosceles triangle right angled at A.
two semicircular discs - D1 and D2 of radii R and
Mass of the triangular plate is M and its equal sides
R are of length a. Find the moment of inertia of this
respectively (fig. a). O and O' are the centre
2 plate about an axis through A perpendicular to the
of curvature of the two discs and each disc has a plane of the plate. Use the expression of moment
mass 6m. The plate is in xy plane. Now the plate of inertia for a square plate that you might have
is folded along the y axis so as to bring the part studied.
OAB in yz plane. (fig. b). The plate is now set free B
to be able to rotate freely about the z – axis. A
particle of mass m, moving with a velocity v in a

the xy plane along the line x = d hits the plate and


sticks to it (d < R). Just before collision speed of A
the particle was v.
Z
a
B B
R C
A O A O
D1 F y d y
F Q. 69. The triangular plate described in the last question
O´ D2 m v C
C D2 has angle <A = q Now find its moment of inertia
E E
about an axis through A perpendicular to the plane
x
x of the plate.
Fig (a) Fig (b)
B
(a) Find the moment of inertia of inertia of the a

plate about z axis.


A q
(b) Find the angular speed of the plate after
collision. a
C
Q. 67. There is a square plate of side length a. It is
divided into nine identical squares each of side Q. 70. A thin uniform spherical shell of radius R is
a bored such that the axis of the boring rod passes
and the central square is removed (see fig. (i)).
3 through the centre of the sphere. The boring rod
Now each of the remaining eight squares of side R
is a cylinder of radius . Take the mass of the
a 2
length are divided into nine identical squares sphere before boring to be M.
3
and central square is removed from each of them (a) Find the mass of the leftover part
(see fig. (ii)). (b) Find the moment of inertia of the leftover
a part about the axis shown.
axis

R/2
a
R

Fig. (i) Fig. (ii)


Rotational Motion 6.13

Q. 71. Consider an equilateral prism as shown in the oscillations about the position shown in diagram?
figure. The mass of the prism is M and length of Note : Centre of mass of a solid hemisphere is at
each side of its cross section is a. Find the moment 3R
a distance of from its base.
of inertia of such a prism about the central axis 8
shown.

axis R

Q. 72. In the arrangement shown in figure the cylinder of


mass M is at rest on an incline. The string between
the cylinder and the pulley (P) is horizontal. Find
the minimum coefficient of friction between Q. 75. Two cylinders A and B have been placed in contact
the incline and the cylinder which can keep the on an incline. They remain in equilibrium. The
system in equilibrium. Also find the mass of the dimensions of the two cylinders are same. Which
block. Assume no friction between the pulley (P) cylinder has larger mass?
and the string. B
P
A
M

q
Block
q Q. 76. The ladder shown in the figure is light and stands
on a frictionless horizontal surface. Arms AB and
Q. 73. A horizontal stick of mass m has its right end
BC are of equal length and M and N are their mid
attached to a pivot on a wall, while its left end
points. Length of MN is half that of AB. A man of
rests on the top of a cylinder of mass m which in
mass M is standing at the midpoint of BM. Find
turn rests on an incline plane inclined at an angle
the tension in the mass less rod MN. Consider the
q. The stick remains horizontal. The coefficient of
man to be a point object.
friction between the cylinder and both the plane
B
and the stick is m. Find the minimum value of
m as function of q for which the system stays in
equilibrium.
M N

A C

Q. 77. A uniform metal sheet of mass M has been folded


q to give it L shape and it is placed on a rough floor
Q. 74. Consider the object shown in the figure. It consist as shown in figure. Wind is blowing horizontally
of a solid hemisphere of mass M and radius R. and hits the vertical face of the sheet as shown.
There is a solid rod welded at its centre. The The speed of air varies linearly from zero at floor
object is placed on a flat surface so that the rod is level to v0 at height L from the floor. Density of
M air is r. Find maximum value of v0 for which the
vertical. Mass of the rod per unit length is . sheet will not topple. Assume that air particles
2R
What is the maximum length of the rod that striking the sheet come to rest after collision, and
can be welded so that the object can perform that the friction is large enough to prevent the
6.14 Problems in Physics for JEE Advanced

sheet from sliding. f

Wind
L

q
L L
Q. 81. A L shaped, uniform rod has its two arms of
length l and 2l. It is placed on a horizontal table
Q. 78. Three identical cylinders have mass M each and
and a string is tied at the bend. The string is pulled
are placed as shown in the figure. The system is
horizontally so that the rod slides with constant
in equilibrium and there is no contact between B
speed. Find the angle q that the longer side
and C. Find the normal contact force between A
makes with the string. Assume that the rod exerts
and B.
uniform pressure at all points on the table.

2l

B C
q

l String
Q. 79. A spool is kept in equilibrium on an incline plane Q. 82. A uniform meter stick AB of mass M is lying in
as shown in figure. The inner and outer radii of the state of rest on a rough horizontal plane. A small
r 1 block of mass m is placed on it at its centre C. A
spool are in ratio = . The force applied on the
R 2 variable force F is applied at the end B of the stick
thread (wrapped on part of radius r) is horizontal. so as to rotate the stick slowly about A in vertical
Find the angle that the force applied by the incline plane. The force F always remains perpendicular
on the spool makes with the vertical. to the length of the stick. The stick is raised to q
and it was observed that neither the end A
 3 slipped on the ground nor the block of mass m
[Take tan −1    19° ]
 5  slipped on the stick.
F

F
B
r

F
q m

A C B
q=60°


Q. 80. A uniform hemisphere placed on an incline is
q q
on verge of sliding. The coefficient of friction
between the hemisphere and the incline is
m = 0.3.
Find the angle f that the circular base of the q q
hemisphere makes with the horizontal. Q. 83. A ladder of mass M and length L stays at rest
Given sin (tan–1 0.3) 0.29 and sin–1 (0.77) against a smooth wall. The coefficient of friction
50° between the ground and the ladder is m.
Rotational Motion 6.15

(a) Let Fwall, W and Fg be the force applied by a


wall, weight of the ladder and force applied by
ground on the ladder.Argue to show that the line
of action of these three forces must intersect. A
(b) Using the result obtained in (a) show that line q
of action of Fg makes an angle tan–1 (2 tan  q)
with the horizontal ground where q is the
angle made by the ladder with the ground.
(c) Find the smallest angle that the ladder can Q. 86. A uniform rectangular block is moving to the right
make with the ground and not slip. on a rough horizontal floor (the block is retarding
(d) You climb up the ladder, your presence due to friction). The length of the block is L and
makes the ladder more likely to slip. Where its height is h. A small particle (A) of mass equal
are you at A or B? C is the centre of mass of to that of the block is stuck at the upper left edge.
the ladder. Coefficient of friction between the block and the
floor is m = 2 . Find the value of h (in terms of
3
A
L) if the normal reaction of the floor on the block
C effectively passes through the geometrical centre
(C) of the block.
B A
q

Q. 84. A uniform rod AB has mass M and length L. It


h C
is in equilibrium supported in vertical plane by
three identical springs as shown in figure. The
springs are connected at A, C and D such that
L
AC = CD = . Assume that the springs are L
3
very stiff and the angle a made by the rod with Q. 87. A uniform cubical block of mass M and side
the horizontal in equilibrium position is very length L is lying on the edge of a rough table with
small. (All springs are nearly vertical). Calculate 1
th of its edge overhanging. When a small block
the tension in the three springs. 4
L/3 L/3 of mass m is placed on its top surface at the right
edge (see fig.), the cube is on verge of toppling.
The block of mass m is given a sharp horizontal
impulse so that it acquires a velocity towards B.
3 2 1
The small block moves on the top surface and falls
on the other side. What is maximum coefficient of
a
A
friction between the small block and the cube so
C that the cube does not rotate as the block moves
D
B over it. Assume that the friction between the cube
Q. 85. A uniform rod of length b can be balanced as and the table is large enough to prevent sliding of
shown in figure. The lower end of the rod is resting the cube on the table.
against a vertical wall. The coefficient of friction m A
B
between the rod and the wall and that between the
rod and the support at A is m. Distance of support
from the wall is a.
a L
(a) Find the ratio if the maximum value of q M
is q . b
a
(b) Find the ratio if the minimum value of q L/4
is q . b
6.16 Problems in Physics for JEE Advanced

Q. 88. A uniform rod of mass M and length L is hinged in vertical plane about point O. The string
at its end to a wall so that it can rotate freely in a an pulley are mass less. The block has mass
horizontal plane. When the rod is perpendicular equal to that of the rod. Find the acceleration
to the wall a constant force F starts acting at of the block immediately after the system is
the centre of the rod in a horizontal direction released with rod in horizontal position.
perpendicular to the rod. The force remains (b) System shown in figure 2 is similar to that in
parallel to its original direction and acts at the figure 1 apart from the fact that rod is mass
centre of the rod as the rod rotates. (Neglect less and a block of mass m is attached to the
gravity). centre of the rod with the help of a thread.
(a) With what angular speed will the rod hit the Find the acceleration of both the blocks
wall ? immediately after the system is released with
(b) At what angle q (see figure) the hinge force rod in horizontal position.
will make a 45° angle with the rod ?

L L
m
O
F m m m
Fig.(1) Fig.(2)
F
q
Q. 91.

Q
wall B

Q. 89. A rod of mass M = 5kg and length L = 1.5 m is


held vertical on a table as shown. A gentle push
is given to it and it starts falling. Friction is large P
enough to prevent end A from slipping on the
table. A
B
A light thread is wrapped tightly a few turns
around a disc P of mass M. One end of the thread
M
is fixed to the ceiling at B. The other end of the
L
thread is passed over a mass less pulley (Q) and
carries a block of mass M. All segment of the
thread (apart from that on the pulley and disc)
q are vertical when the system is released. Find the
acceleration of block A. On which object – the
block A or the ceiling at B – does the thread exert
A
more force ?
(a) Find the sum of linear momentum of all the Q. 92. An equilateral triangle is made from three mass
particles of the rod when it rotates through an less rods, each of length l. Two point masses m
angle q = 37°. are attached to two vertices. The third vertex is
(b) Find the friction force and normal reaction hinged and triangle can swing freely in a vertical
force by the table on the rod, when q = 37°. plane as shown. It is released the position shown
(c) Find value of angle q when the friction force with one of the rods vertical. Immediately after
becomes zero. the system is released, find –
3 (a) tensions in all three rods (specify tension or
[tan 37∞ = and g = 10 m / s]
4 compression),
Q. 90. (a) In the system shown in figure 1, the uniform (b) accelerations of the two masses
rod of length L and mass m is free to rotate
Rotational Motion 6.17

pivot is placed in contact with the vertical rod and a


horizontal force F is applied at the upper end of
the rod.
l
(a) Find the acceleration of the ball immediately
m
after the force starts acting.
l F

Q. 93. A B

A rod of mass M and length L is hinged about (b) Find the horizontal component of hinge force
its end A so that it can rotate in vertical plane. acting on the rod immediately after force F
When the rod is released from horizontal position starts acting.
it takes t0 time for it to become vertical. Q. 96.
(a) A particle of mass M is stuck at the end B
of the rod and the rod is once again released
from its horizontal position. Will it take
more time or less time (than t0) for the rod to
become vertical from its horizontal position. R
(b) At what distance x from end A shall the q
A
particle of mass M be stuck so that it takes
minimum time for the rod to become vertical A ring of mass M and radius R is held at rest on
from its horizontal position. a rough horizontal surface. A rod of mass M and
length L = 2 3R is pivoted at its end A on the
Q. 94. A disc is free to rotate about an axis passing horizontal surface and is supported by the ring.
through its centre and perpendicular to its plane. There is no friction between the ring and the rod.
The moment of inertia of the disc about its rotation The ring is released from this position. Find the
axis is I. A light ribbon is tightly wrapped over it acceleration of the ring immediately after the
in multiple layers. The end of the ribbon is pulled release if q = 60°. Assume that friction between
out at a constant speed of u. Let the radius of the the ring on the horizontal surface is large enough
ribboned disc be R at any time and thickness of to prevent slipping of the ring.
the ribbon be d (<< R). Find the force (F) required
to pull the ribbon as a function of radius R. Q. 97. A uniform semicircular wire is hinged at ‘A’ so
d
that it can rotate freely in vertical plane about
u
a horizontal axis through ‘A’. The semicircle
is released from rest when its diameter AB is
horizontal.
R
O B
A

Q. 95. A uniform rod of mass M and length L is hinged Find the hinge force at ‘A’ immediately after the
at its lower end on a table. The rod can rotate wire is released.
freely in vertical plane and there is no friction at
Q. 98. A uniform solid hemisphere A of mass M radius
L R is joined with a thin uniform hemispherical
the hinge. A ball of mass M and radius R =
3
6.18 Problems in Physics for JEE Advanced

shell B of mass M and radius R (see fig.). The


sphere thus formed is placed inside a fixed box as
shown. The floor , as well as walls of the box are
smooth. On slight disturbance, the sphere begins
to rotate. Find its maximum angular speed (w0) R

and maximum angular acceleration (a0) during M


the subsequent motion. Do the walls of the box
apply any force on the sphere while it rotates?
A

r
m
B
(b) Find acceleration of the mark at time t0.
A
(c) Is there any difference in magnitude of
acceleration of the mark and that of a point
Q. 99. located on the circumference at diametrically
opposite end of the pulley.
Q. 101. A point mass m = 1 kg is attached to a point P on
the circumference of a uniform ring of mass M =
3kg and radius R = 2.0 m. The ring is placed on a
O
r horizontal surface and is released from rest with
R line OP in horizontal position (O is centre of the
ring). Friction is large enough to prevent sliding.
Calculate the following quantities immediately
after the ring is released-

In the arrangement shown, the double pulley has (a) angular acceleration (a) of the ring,
a mass M and the two mass less threads have (b) normal reaction of the horizontal surface on
been tightly wound on the inner (radius = r) and the ring and
outer circumference (radius R = 2r). The block (c) the friction force applied by the surface on
shown has a mass 4 M. The moment of inertia of the ring.
the double pulley system about a horizontal axis [Take g = 10 m/s2]
passing through its centre and perpendicular to
Mr 2
the plane of the figure is I .
2 O R
(a) Find the acceleration of the center of the P
pulley after the system is released.
(b) Two seconds after the start of the motion the
string holding the block breaks. How long
after this the pulley will stop ascending? Q. 102. A light thread has been tightly wrapped around a
disc of mass M and radius R. The disc has been
Q. 100. A thread is tightly wrapped on two pulleys as
placed on a smooth table, lying flat as shown.
shown in figure. Both the pulleys are uniform disc
with upper one having mass M and radius R being M
free to rotate about its central horizontal axis. The
R
lower pulley has mass m and radius r and it is
released from rest. It spins and falls down. At the
instant of release a small mark (A) was at the top
point of the lower pulley.
(a) After what minimum time (t0) the mark will m
again be at the top of the lower pulley?
Rotational Motion 6.19

The other end of the string has been attached to inclination is q.


a mass m as shown. The system is released from (a) Find the minimum force F0 needed to roll the
rest. If m = M, which point of the disc will have log up the incline.
zero acceleration, immediately after the system is
released? (b) Find the work done by the force in moving
the log from the bottom to the top of the
Q. 103. A spool has the shape shown in figure. Radii incline if the applied force is F = 2F0
of inner and outer cylinders are R and 2 R
respectively. Mass of the spool is 3 m and its Q. 105. In the figure shown, the light thread is tightly
moment of inertia about the shown axis is 2mR2. wrapped on the cylinder and masses of plank and
Light threads are tightly wrapped on both the cylinder are same each equal to m. An external
cylindrical parts. The spool is placed on a rough agent begins to pull the plank to the right with
surface with two masses m1 = m and m2 = 2m a constant force F. The friction between the
connected to the strings as shown. The string plank and the cylinder is large enough to prevent
segment between spool and the pulleys P1 and P2 slipping. Assume that the length of the plank is
are horizontal. The centre of mass of the spool is quite large and the cylinder does not fall off it for
at its geometrical centre. System is released from the time duration concerned.
rest. (a) Find the acceleration of the cylinder. (Hint :
(a) What is minimum value of coefficient of don’t write any equations)
friction between the spool and the table so (b) Find the kinetic energy of the system after
that it does not slip? time t.
(b) Find the speed of m1 when the spool
completes one rotation about its centre.
axis R

R
F
2R

P1 Q. 106. A disc of radius r = 0.1 m is rolled from a point


P2 A on a track as shown in the figure. The part
R AB of the track is a semi-circle of radius R in a
2R vertical plane. The disc rolls without sliding and
leaves contact with the track at its highest point
m1 m2 B. Flying through the air it strikes the ground at
point C. The velocity of the center of mass of the
Q. 104. disc makes an angle of 30° below the horizontal
F
at the time of striking the ground. At the same
B
instant, velocity of the topmost point P of the disc
F
is found to be 6 m/s. (Take g = 10 m/s2).
A
B

P
q 30°
C A

A heavy uniform log of mass M is pulled up an (a) Find the value of R.


incline surface with the help of two parallel ropes (b) Find the velocity of the center of mass of the
as shown in figure. The ropes are secured at point disc when it strikes the ground.
A and B. The height of the incline is h and its (c) Find distance AC.
6.20 Problems in Physics for JEE Advanced

Q. 107. A trough has two identical inclined segments


cylinder B having mass M and radius R. The
and a horizontal segment. A ball is released on springs are connected symmetrically to the axle
the top of one inclined part and it oscillates inside of the cylinders. Moment of inertia of the two
the trough. Friction is large enough to prevent Bodies A and B about their axles are IA = MR2 and
slipping of the ball. Time period of oscillation is 1
I B = MR 2 respectively. Cylinders are pulled

T. Now the liner dimension of each part of the 2
trough is enlarged four times. Find the new time apart so as to stretch the springs by x0 and
period of oscillation of the ball. released. During subsequent motion the cylinders
do not slip.

(a) Find acceleration of the centre of mass of the


system immediately after it is released.
(b) Find the distance travelled by cylinder A by
the time it comes to rest for the first time after
Q. 108. A uniform cylinder is lying on a rough sheet being released.
of paper as shown in fig. The strip is pulled Q. 111. Two identical uniform thin rods have been

horizontally to the right with a constant connected at right angles to form a ‘T’ shape. One
acceleration of a0 = 6 m/s2. Initially the cylinder end of a rod is connected to the centre of the other
is located at a distance of L = 10 m from the left rod. Length of each rod is L. The upside down
end of the strip. Find the velocity of the centre of ‘T’ can swing like a pendulum about a horizontal
the cylinder at the instant it moves off the edge of axis passing through the top end (see fig.). Axis is
the strip. Assume that the cylinder does not slip. perpendicular to plane of the fig. The speed of the
meeting point of the two rods is u = 2 gL when
it is at its lowest position. Calculate the angular
a 0 = 6 m/s2
acceleration of the ‘T’ shaped object when it is at
extreme position of its oscillation.

L = 10 m

M
Q. 109.
m F

A hollow pipe of mass M = 6 kg rests on a plate


of mass m = 1.5 kg . The thickness of the pipe Q. 112. A uniform rod of mass M and length L is hinged
is negligible. The coefficient of friction at all at its lower end so as to rotate
contacts is m = 0.2. The system is initially at rest. freely in the vertical plane of the
A horizontal force F of magnitude 25N is applied fig. There is a small tight fitting
M M
on the plate as shown in figure. Will the cylinder bead of mass on the
6 6
slide on the plate? Find the acceleration of the 3L
centre of the cylinder. rod at a distance from
4 3L
Q. 110. the hinged end. A small mass less 4
A pin welded to the rod supports the
k
bead. The system is released from
the vertical position shown. It was
k observed that the bead just begins
B
to slide on the rod when the rod
becomes horizontal.
A hollow cylindrical pipe A has mass M and (a) Find the normal contact force between the
radius R. With the help of two identical springs rod and the bead when the rod gets horizontal.
(each of force constant k) it is connected to solid What is the direction of this force?
Rotational Motion 6.21

(b) Find the coefficient of friction between the towards each other with same speed (relative to
bead and rod. the table) along the circumference. Find the angle
Q. 113 rotated by table by the time the two men meet.
Treat the men as point masses.
cm

Two astronauts having mass of 75 kg and 50 kg 1 2


are connected by a rope of length L = 10m and
negligible mass. They are in space, orbiting their
centre of mass at an angular speed of w0 = 5 rad/s.
The centre of mass itself is moving uniformly in
space at a velocity of 10 m/s. By pulling on the
rope, the astronauts shorten the distance between Q. 116. A L shaped uniform rod has both its sides of
L length l. Mass of each side is m. The rod is placed
them to 5m . How much work is done by
2 on a smooth horizontal surface with its side
the astronauts in shortening the distance between AB horizontal and side BC vertical. It tumbles
them ? Assuming that the astronauts are athletic down from this unstable position and falls on the
and each of them can generate a power of 500 surface. Find the speed with which end C of the
watt, is it possible for the two astronauts to rod hits the surface.
reduce the distance between them to 5 m, within a C
minute?
l
Q. 114. In the figure shown a plank of mass m is lying at
rest on a smooth horizontal surface. A cylinder of B
same mass m and radius r is rotated to an angular l
A
speed w0 and then gently placed on the plank. It
is found that by the time the slipping between the
plank and the cylinder cease, 50% of total kinetic Q.117.
energy of the cylinder and plank system is lost.
Assume that plank is long enough and m is the
coefficient of friction between the cylinder and
the plank.
m, r

w0
V
m

(a) Find the final velocity of the plank. A flat horizontal belt is running at a constant
speed V. There is a uniform solid cylinder of mass
(b) Calculate the magnitude of the change in
M which can rotate freely about an axle passing
angular momentum of the cylinder about its
through its centre and parallel to its length.
centre of mass.
Holding the axle parallel to the width of the belt,
(c) Distance moved by the plank by the time the cylinder is lowered on to the belt. The cylinder
slipping ceases between cylinder and plank. begins to rotate about its axle and eventually stops
Q. 115. A horizontal turn table of mass 90kg is free to slipping. The cylinder is, however, not allowed to
rotate about a vertical axis passing through its move forward by keeping its axle fixed. Assume
centre. Two men – 1 and 2 of mass 50kg and 60kg that the moment of inertia of the cylinder about its
respectively are standing at diametrically opposite 1
axle is MR 2 where M is its mass and R its
point on the table. The two men start moving 2
6.22 Problems in Physics for JEE Advanced

radius and also assume that the belt continues does not fly off the cylinder.
to move at constant speed. No vertical force is Thread A
applied on the axle of the cylinder while holding B
it. R rope

(a) Calculate the extra power that the motor R C


O
driving the belt has to spend while the cylinder Q P
gains rotational speed. Assume coefficient of
friction = m.
L fixed
(b) Prove that 50% of the extra work done by the
motor after the cylinder is placed over it, is Q.120. A uniform cube of mass M and side length a is
dissipated as heat due to friction between the placed at rest at the edge of a table. With half of
belt and the cylinder. the cube overhanging from the table, the cube
Q.118. A uniform disc of mass M and radius R is rotating begins to roll off the edge. There is sufficient
freely about its central vertical axis with angular friction at the edge so that the cube does not slip
speed w0. Another disc of mass m and radius r is at the edge of the table. Find -
free to rotate about a horizontal rod AB. Length (a) the angle q through which the cube rotates
of the rod AB is L (< R) and its end A is rigidly before it leaves contact with the table.
attached to the vertical axis of the first disc. The (b) the speed of the centre of the cube at the
disc of mass m, initially at rest, is placed gently instant it breaks off the table.
on the disc of mass M as shown in figure. Find
(c) the rotational kinetic energy of the cube at the
the time after which the slipping between the two
instant its face AB becomes horizontal.
discs will cease. Assume that normal reaction
C
between the two discs is equal to mg. Coefficient C B
B
of friction between the two discs is m.

w0 D P
D A q0
A

A
B Q.121. A uniform frictionless ring of mass M and radius
R L R, stands vertically on the ground. A wall touches
the ring on the left and another wall of height R
touches the ring on right (see figure).There is a
small bead of mass m positioned at the top of the
ring. The bead is given a gentle push and it being
to slide down the ring as shown. All surfaces are
Q.119. P is a fixed smooth cylinder of radius R and Q is frictionless.
a disc of mass M and radius R. A light thread is m
tightly wound on Q and its end is connected to
a rope ABC. The rope has a mass m and length
πR q
and is initially placed on the cylinder with
2 A
its end A at the top. The system is released from
rest. The rope slides down the cylinder as the R
M
disc rolls without slipping. The initial separation
πR
between the disc and the cylinder was L =
2
(a) As the bead slides, up to what value of angle
(see fig). Find the speed with which the disc
q the force applied by the ground on the ring
will hit the cylinder. Assume that the rope either
is larger than Mg?
remains on the cylinder or remains vertical; it
Rotational Motion 6.23

(b) Write the torque of force applied by the bead and it rolls without sliding. The ring leaves the
on the ring about point A as function of q. track at F and attains a maximum height of h2
(c) What is the maximum possible value of h1
above
the ground. Find the ratio .
torque calculated in (b)? Using this result tell h2
.
m Q.124. A uniform sphere of radius R has a spherical
what is the largest value of for which the
M R
ring never rises off the ground ? cavity of radius (see figure). Mass of the
2
Q.122. sphere with cavity is M. The sphere is rolling
without sliding on a rough horizontal floor [the
line joining the centre of sphere to the centre of
O the cavity remains in vertical plane]. When the
centre of the cavity is at lowest position, the centre
of the sphere has horizontal velocity V. Find:
m0 (a) The kinetic energy of the sphere at this
moment.
(b) The velocity of the centre of mass at this
moment.
(c) The maximum permissible value of V ( in the
m
position shown ) which allows the sphere to
A uniform disc shaped pulley is free to rotate roll without bouncing
about a horizontal axis passing through the centre
of the pulley. A light thread is tightly wrapped
R
over it and supports a mass m at one of its end.
A small particle of mass m0 = 2m is stuck at the O
V
lowest point of the disc and the system is released
from rest. Will the particle of mass m0 climb to R
2
the top of the pulley?
Q.123.
y Q. 125. A uniform ball of mass M and radius R can

vertical D q rotate freely about any axis through its centre. Its
A 150° angular velocity vector is directed along positive
H B H F x axis. A bullet is fired along negative Z direction
H and it pierces through the ball along a line that is
2
X at a perpendicular distance r (< R) from the centre
Ground O Ground E of the ball. The bullet passes quickly and its net
effect is that it applies an impulse on the ball.
AOB is a frictionless parabolic track in vertical
Mass of the bullet is m and its velocity charges
plane. The equation of parabolic track can be
from u to v (< u) as it passes through the ball. As
3 2
expressed as y = x for co-ordinate system a result the ball stops rotating about X axis and
2H begins to rotate about y axis. The angular speed
shown in the figure. The end B of the track of the ball before and after the hit is w. Find r.
H y
lies at y . When a uniform small ring is
2
released on the track at A it was found to attain
a maximum height of h1, above the ground after
r
leaving the track at B. There is another track DEF O
X
w
which is in form of an arc of a circle of radius
H subtending an angle an angle of 150° at the
centre. The radius of the track at D is horizontal.
The same ring is released on this track at point D
6.24 Problems in Physics for JEE Advanced

Q. 126. A uniform square plate ABCD has mass M side from one end of the rod. A sharp impulse P0 =
length a. It is sliding on a horizontal smooth
2 130 kg m/s is applied along the surface at one

( )
surface with a velocity of v = v0 4iˆ + 2 ˆj . There  
end of the rod as shown in figure  tan θ = 7 
9
is no rotation. Vertex A of the plate is suddenly  
fixed by a nail. Calculate the velocity of centre of
(a) Find the angular speed of the rod immediately
the plate immediately after this.
Y
after the hit
(b) Find the impulse on the rod due to the hinge.
D C
v O C

l/4 PO q
A B
X

Q. 127. Two discs A and B are moving with their flat


circular surface on a smooth horizontal surface. Q. 130. Four thin rods of length L = 1.0 m each are joined
Mass, radius and velocity of the two discs are – to form a square ABCD. The opposite vertices of
mA = 2M, mB = M, rA = R, rB = 2R, vA = v, and vB = the square are joined by mass less rods AC and
2v. The velocities of the two discs are oppositely BD. This square frame is mounted on a horizontal
directed so that they just cannot avoid collision axis through its centre so that the frame can rotate
and stick to each other (see figure) freely in the vertical plane. Masses of rods AB
(a) Find the angular speed of the composite and BC are m = 2kg each and the rod AD and
system after collision DC have mass M = 4kg each. A monkey of mass
mo =12 kg is at rest on the horizontal rod AD and
(b) Find loss in kinetic energy due to collision
keeps the system in equilibrium. The monkey
A 2M
takes a sudden jump and rises to a height H from
v its initial position. Calculate minimum value of
R H so that the square frame is able to complete a
w
rotation about its central axis. Assume no further
2R contact between monkey and the frame.
2v
B D
A
M
Q. 128. A uniform rod of mass M and length 2L lies on
a smooth horizontal table. There is a smooth L
peg O fixed on the table. One end of the rod is O
kept touching the peg as shown in the figure.
An impulse J is imparted to the rod at its other C
B
end. The impulse is horizontal and perpendicular
to the length of the rod. Find the magnitude of Q. 131. A frustum has been mounted with its axis vertical.
impulse experienced by the peg. It has a height h and radii of its upper and lower
O 2L cross sections are R and r respectively. A particle
is projected with horizontal velocity v0 along its
J upper brim. The particle spirals down the inner
Peg surface and leaves the lower face at point B. The
inner wall of the frustum is smooth.
Q. 129. A uniform rod of mass M and length  is hinged
at point O and is free to rotate on a horizontal (a) Find the vertical component of velocity of the
 particle as it leaves the frustum at B.
smooth surface. Point O is at a distance of
4 (b) Find minimum value of h for which the
Rotational Motion 6.25

particle will never come out of the frustum. impact misses the so called sweet spot of the rod
R (and hits either above or below the sweet spot).
Take r = for solving this part of the problem.
2 After solving the above problem you know where
the sweet spot is ! You may assume that during the
v0 impact the rod is rotating about its holding hand.
A
And if you play cricket, you know that there is a
R
sweet spot in your bat too ! If the ball hits way
above or below the spot you get stung.]
Q. 134. Two identical thin rods are moving on a smooth
h table, as shown. Both of them are rotating with
angular speed w, in clockwise sense about their
r centres. Their centres have velocity V in opposite
directions. The rods collide at their edge and stick
B
v together. Length of each rod is L.
V
(a) For what value of there will be no
Q. 132. A uniform thin stick of mass M = 24kg and length ωL
L rotates on a friction less horizontal plane, with motion after collision ?
its centre of mass stationary. A particle of mass m V
L (b) If the ratio is half the value found in (a)
is placed on the plane at a distance x = from ωL
3
the centre of the stick . This stick hits the particle above, what fraction of kinetic energy is lost
elastically in the collision?
v
(a) Find the value of m so that after the collision,
w
there is no rotational motion of the stick
w
(b) For what minimum of x can we get a value of
‘m’ so that the rod has no rotational motion v
after elastic collision? Q. 135. Light thread is tightly wound on a uniform solid
cylinder of radius R. The cylinder is placed on a
smooth horizontal table and the thread is pulled
m horizontally as shown, by applying a constant
force F. How much length of the thread is
L
x unwound from the cylinder by the time its kinetic
3
energy becomes equal to K.
F

Q. 133. A uniform rod of length L is rotating in a horizontal


plane about a vertical axis passing through one of
its ends. At a distance x (< L) from the axis there
is a fixed vertical pole. The rod hits the pole and Q. 136. A horizontal disc of radius R and mas 20 M is
its direction of motion is reversed. Find x if it is pivoted to rotate freely about a vertical axis
known that during the impact the axis of rotation through its centre. A small insect A of mass
imparts no impulse to the rod. Does your answer M
depend on coefficient of restitution? M and another small insect B of mass m =
4
[NOTE : If you hit a lamp post with a rod, the are initially at diametrically opposite points on
hand holding the rod gets hurt as long as the the periphery of the disc. The whole system is
6.26 Problems in Physics for JEE Advanced

imparted an angular speed w0. Insect A walks (b) Do you think there is a chance of second
along the diameter with constant velocity v Collision? If yes, how is the system of particle
relative to the disc unit it reaches B which remains and stick moving after the second collision?
at rest on the disc. A then eats B and returns to its m
starting point along the original path with same
speed v relative to the disc. m
u
(a) Find the angular speed of the disc when A
reaches the centre after eating B. x

l
(b) Plot approximately, the variation of angular C
speed of the disc with time for the entire
journey of the insect A.
Q.137. A disc of mass m and radius R is moving on a
smooth horizontal surface with the flat circular
face on the surface. It is spinning about its centre
with angular peed w and has a velocity V (see
figure). It just manages to hit a stick AB at its end Q. 139. A ring is made to rotate about its diameter at a
A. The stick was lying free on the surface and constant angular speed of w . A small insect
stick to the disc. [The combined object becomes of mass m walks along the ring with a uniform
like a badminton racket]. Mass and length of the angular speed w relative to the ring (see figure).
stick are m and 2R respectively. Radius of the ring is R.
(a) Find the external torque needed to keep the
(a) Calculate the angular speed of the combined ring rotating at constant speed as the insect
object assuming V = Rw walks. Express your answer as a function of q.
(b) Calculate loss in kinetic energy. Why is For what value of q is this torque maximum?
energy lost? [given your answer for 0 <q <90°]
(c) If V   = h(Rw), loss in kinetic energy is (b) Find the component of force perpendicular
minimum. Find h. [Assume w is given] to the plane of the ring, that is applied by
the ring on the insect. For what value of q
is this force maximum? Argue quantitatively
w to show that indeed the force should be
V
maximum for this value of q. [Give your
answer for 0° <q <90°]
A

w0

2R

O w
B q
Q. 138. A uniform rod of mass m and length  has been
placed on a smooth table A particle of mass m,
travelling perpendicular to the rod, hits it at a

distance x = from the centre C of the rod. Q. 140. A small car took off a ramp at a speed of 30 m/s.
6 Immediately after leaving the ramp, the driver
Collision is elastic. applied brakes on all the wheels. The brakes
(a) Find the speed of the centre of the rod and the retarded the wheels uniformly to bring them to
particle after the collision. rest in 2 second. Calculate the angle by which
Rotational Motion 6.27

the car will rotate about its centre of mass in the threads. Mass of the plate is M.
2    second interval after leaving the ramp. Radius
of each wheel is r = 0.30 m . Moment of inertia
of the car along with the driver, about the relevant T1
axis through its centre of mass is IM = 80kgm2 and
the moment of inertia of each pair of wheels about
l2
their respective axles is 0.3kg m2. Assume that the l1
car remained in air for more than 2 second . Also T3 T2
assume that before take- off the wheels rolled
without sliding. l3

Q. 143. A rigid large uniform square platform is resting on


V
a flat horizontal ground supported at its vertices by
four identical spring. At vertex l a wooden block,
6 cm high, is inserted below the spring. Calculate
the change in height of the centre of the platform.
Assume change in height to be small compared to
dimension of the platform.
3 2

Q. 141. A disc of radius R stands at the edge of a table.


If the given a gentle push and it beings to fall.
Assume that the disc does not slip at A and
1
it rotates about the point as it falls. The falling 4
disc hits the edge of another table placed at same
height as the first one at a horizontal distance of
2 R. Imagine that the disc hits the edge B and
6cm
rotates (up) about the edge

(a) Find the speed of the centre of the disc at the Q. 144. A uniform rod of mass M and length L is placed
instant just before it hits the edge B. freely on a horizontal table. A horizontal force F
is applied perpendicular to the rod at one of its
(b) Find the angular speed of the disc about B ends. The force F is increased gradually from zero
just after the hit. and it is observed that when its value becomes F0,
the rod just begins to rotate about point O
(i) Find length AO
(ii) Find F0

F A
R

A B
L
R

O
Level 3
Q. 142. A uniform triangular plate is kept horizontal
B
suspended with the help of three vertical threads
as shown. The sides of the plate have length l1, Q. 145. A ring of mass M and radius R lies flat on a
l2 and l3. Find tension T1, T2 and T3 in the three horizontal table. A light thread is wound around it
6.28 Problems in Physics for JEE Advanced

and its free end is pulled with a constant velocity released from rest and the rod begins to slide.
v. Find the speed of the centre of the rod when the
string becomes vertical.
(a) Two small segment A and B (see fig.) in
the ring are rough and have a coefficient of A
friction m with the table. Rest of the ring is
smooth. Find the speed with which the ring a
moves.
C b
(b) Find the speed of the ring if coefficient of
friction is m everywhere; for all points on the a
ring.
V V
B
A B
R Q. 148. A dumb – bell has a rigid mass less stick and
two point masses at its ends. Each mass is
Top view m and length of the stick is L. The dumb- bell
leans against a frictionless wall, standing on a
frictionless ground. It is initially held motionless,
Q. 146. A uniform stick of length L is pivoted at one end with its bottom end an infinitesimal distance from
on a horizontal table. The stick is held forming the wall. It is released from this position and its
an angle q0 with the table. A small block of mass bottom end slides away from the wall where as
m is placed at the other end of the stick and it the top end slides down along the wall.
remains at rest. The system is released from rest
(a) Show that centre of mass of the dumb-bell

(a) Prove that the stick will hit the table before moves along a circle.
2 (b) When the dumb-bell loses contact with the
the block if cosq 0 ≥
3 wall what is speed of the centre of mass?
(b) Find the contact force between the block and
the stick immediately before the system is
Ê 2ˆ
released. Take q 0 = cos-1 Á ˜
Ë 3¯ L

(c) Find the contact force between the block


and the stick immediately after the system is
Ê 2ˆ
released if q 0 = cos-1 Á ˜.
Ë 3¯
Q. 149. A hexagonal pencil of mass M and sides length
a has been placed on a rough incline having
inclination q Friction is large enough to prevent
L sliding. If at all the pencil moves, during one
full rotation each of its 6 edges, in turn, serve as
q0 instantaneous axis of rotation.
(a) Show that for q > 30° the pencil cannot
Q. 147. A uniform rod BC with length a is attached to a remain at rest.
light string AC. End A of the string is fixed to the (b) For inclination of incline q < 30°) the pencil
ceiling and the end B of the rod is on a smooth will not roll on its own. A sharp impulse J is
horizontal surface. B is exactly below point A given to the pencil parallel to the incline at
and length AB is b (a < b < 2a). The system is its upper edge (see figure). Friction does not
Rotational Motion 6.29

allow the pencil to slide but it begins to rotate toy is M. Now a small block of mass m is stuck
about the edge through A with initial angular at the top of the right vertical segment of the belt
speed w0. Find w0. Moment of inertia of the and the system is released. Height of the toy is
pencil about its edge is I. h. Find the speed of the block when it is about to
(c) Find minimum value of J so that the pencil hit the ground. Assume no slipping anywhere and
will turn about A; and B will land on the neglect the dimension of the wheels.
incline.
Block
(d) If kinetic energy acquired by the pencil just
Belt
after the impulse is K0, find its kinetic energy Frame
h
just before edge B lands on the incline

Q. 152. Consider an idealized case of rolling of a solid


a ball in which the point P does not rotate in a
vertical plane. But it rotates on circular path
B PQRSTP when observed from the centre of the
ball. The radius of circular path PQRSTP is half
A
q the radius of the ball. The ball rolls without sliding
with its centre moving with speed v0 in direction
Q. 150. A rope of length L and mass per unit length l perpendicular to the plane of the figure calculate
passes over a disc shaped pulley of mass M the kinetic energy of the rolling ball. Mass of the
and radius R. The rope hangs on both sides of ball and its radius are M and R respectively.
the pulley and the length of larger hanging part
is l. The pulley can rotate about a horizontal
axis passing through its centre. The system is
R
released from rest and it begins to move. The S
pulley has no friction at its axle and the rope has O
Q
large enough friction to prevent it from slipping T
on the pulley.
(a) Find the acceleration of the rope immediately P
after it is released. Q. 153. Two identical rings, each of mass M and radius R,
(b) Find the horizontal component of the force are standing on a rough horizontal surface. The
applied by the axle on the pulley immediately rings overlap such that the horizontal line passing
after the system is released through their centre makes an angle of q = 45°with
the radius through their intersection point P. A
small object of mass m is placed symmetrically
R
on the rings at point P and released. Calculate the
acceleration of the centre of the ring immediately
after the release. There is no friction between the
small object and the rings. The friction between
l the small object and the rings, and the friction
between the rings and the ground is large enough
to prevent slipping.
P

Q. 151. A toy is made of a rectangular wooden frame with q q


four small wheels at its vertices. A tight fitting
belt of negligible mass runs around the frame
passing over the wheels. Mass of the complete
6.30 Problems in Physics for JEE Advanced

Q. 154. A uniform rope tightly wraps around a uniform at end A or B while moving in the direction of
thin ring the mass M and radius R. The mass of velocity of A or B. All collisions are elastic.
the segment of the rope around the ring (i.e. , (i) Initial angular velocity of the rod is zero and its
mass of the length 2 R of the rope) is also M. The angular velocity after nth collision is wn. Write
ends of the rope are fixed one above the other and wn + 1 in terms of wn
it is taut. The ring is let go. Find its acceleration.
Assume no slipping and thickness of the rope to (ii) Solve the above equation to get wn
be negligible. (iii) Find the limiting value of w.
m O m
r r
A B
R


Q. 155. A uniform stick AB has length L. It is tossed
up from horizontal position such that its centre Q. 157. A uniform rod of mass m and length 2L on a
receives a velocity u = gL in vertically upward smooth horizontal surface. A particle of mass m is
direction and the stick gets an angular velocity. connected to a string of length L whose other end
The stick lands back to its point of projection in is connected to the end ‘A’ of the rod. Initially the
horizontal position. During its course of flight string is held taut perpendicular to the rod and the
its angular velocity remained constant and the particle is given a velocity v0 parallel to the initial
stick made one complete rotation. Stick rotates in position of the rod.
vertical plane. (a) Calculate the acceleration of the centre of
(a) Calculate the angular velocity (w) imparted the rod immediately after the particle is
to the stick. projected.
(b) Calculate the maximum height, above the
point of projection, to which the end B of the (b) The particle strikes the centre of the rod and
stick rises. sticks to it. Calculate the angular speed of the
rod after this.
[Take solution of equation cos x = 2x to be x
A 2L B
= 0.45 and sin (0.45) = 0.43]

V0
m

u Q. 158. Two boys support by the ends a uniform rod of


A B mass M and length 2L. The rod is horizontal. The
w
two boys decided to change the ends of the rod
Q. 156. A uniform rod of mass 4m and length 2r can by throwing the rod into air and catching it. The
rotate in horizontal plane about a vertical axis boys do not move from their position and the rod
passing through its centre O. Two small balls remained horizontal throughout its flight. Find
each of mass m are attached to its ends. A fixed the minimum impulse applied by each boy on the
gun fires identical balls with speed v in horizontal rod when it was thrown.
direction. The firing is being done at suitable Q. 159 A uniform rod of mass m and length l pivoted
intervals so that the fired balls either hit the ball at one of its top end is hanging freely in vertical
Rotational Motion 6.31

plane. Another identical rod moving horizontally they remain perpendicularly connected to each
with velocity v along a line passing through its other. Find the maximum angle turned by the
lower end hits it and sticks to it. The two rods two-rod system after collision.
were perpendicular during the hit and later also

ANSWERS

3h ML2  h1 + 3h2 
15.
1.
R 6  h1 + h2 
2. 2v Ê 8 ˆ
16. 2 MR 2 Á 1 - ˜
Ë 9p 2 ¯
1
3. 17. a tan–1 (2) ; b 45°
3
3 Ê 2ˆ
4. 19. a = tan -1 Á ˜
2 Ë ¯ p
1 29
5. MR 2 20. M
4 6
6. MR 2 R−h
21. (a) cosθ =
3 R
52 (b) Decreases
7. x2 + y2 =
4
22. sin–1 (0.3)
Ê 4ˆ
8. MR 2 Á 1 - 2 ˜ 2 d.g.m
Ë p ¯ 23. t =
kh
9. a, b, c, d
24. mg
F=
10 2
10. %
3 25. (a) f = m0g towards left
(b)
m = 4m0
2
ML 1
11. (i) 26. mmin =
2 2

(ii) (a) MOI does not change Mg


27. (a)
2
1
(b) I = I 0 = MR 2 1
2 (b)
3
I
12. 5 28. (a) Yes
4
mg
7I k=
(b)
13. d
16
14. 29 MR2 R 2w 2
29.
4g
6.32 Problems in Physics for JEE Advanced

g 5gR
30. (a)
2 5 5 – 1) V
M 1
(b) <
m 3
100p mg 6
31. (a) w = m/s
R (5m + 2 M ) 5
V
(b)
 2M  49.
g R
 5m + 2 M 
24
F w= w0
32. a = 19
4m
Ê 2ˆ MV0 b
33. N = Á 1 - ˜ Mg
Ë p¯ 2
34. Less than weight

3g sinq 3u
35. a =
4 4
2R
36. h0 = 1
5 mvL sin q , No
3
4F 4
37. (a)  1
3 M + 8m  
2
3MF
(b)   9
3 M + 8m 55. (a) VA = 4i − 3 j ; VB = 3i − j ;
4
12V0  13
VC = i − 2 j all in unit of m/s
7 4
2 1
(ii) (a) V0 > w 0 R w=
(b) rad/s
5 4
2 v
(b) V0 = w0 R 57. (a) w =
5 2 3R
v
2
V0 < w 0 R
5
vy =
(b)
4
(↓)
1 3V
M (w R)2 58. (a) VA = 2V, VB =
2 2
w 2R Ê 3ˆ
t=
(b) cos-1 Á ˜
V Ë 4¯
3
MV 2
4 59. (a) 156 ms–2
(b) 2 3 ms–2

(c) (3, p /6)


Rotational Motion 6.33

5v 2 1 3 Mg
60.
6R 77. (v0 )max = L r
61. R
62. 4R Mg
78.
63. 2.26 rad/s 2
64. (a) v 79.
1
(b) 80.
2
l q q -1 Ê 1 ˆ
81. q = p - tan ÁË ˜¯
4
2 9
66. (a) mR
2 82. (a) F1

2 vd
(b) w = mg
11R 2
91
67. Ma 2 0°

486 60°

Ma 2 mmin = 3
(b)
68.
3
(c) f
1 Ê 2 qˆ
69 Ma 2 Á 1 - sin 2 ˜ (M  m )g

2 Ë 3 2¯ 4

3
70. (a) M 
2 O 45° 60°

3 3 a q
(b) MR 2
8 Ê 1 ˆ
q min = tan -1 Á
(c)
Ë 2 m ˜¯
Ma 2
71.
12 (d) Slippage is more likely when at A.
Mg Mg 7 Mg
sin q M sin q 84. T1 = ;T = ;T =
72. mmin = ; mblock = 12 2 3 3 12
1 + cos q 1 + cos q
85.
3 sin q
73.
1 + cos q (a)
a
b
1
2 Î (
= sin 2 q1 È 1 - m 2 sin q1 - 2 m cos q1 ˘ ) ˚

74. Less than


3
2
R
b 2
a
Î
1
(
(b) = sin 2 q 2 È 1 - m 2 sin q 2 + 2 m cos q 2 ˘
˚ )
75. MA < MB
L
86. h =
2
3
76. Mg 1
4 87. mmax =
2
6.34 Problems in Physics for JEE Advanced

3F 18
(b) s
88. (a) 11
ML

2p (2 m + 3M ) r
-1 Ê
1ˆ 100. (a) t0 =
(b) q = tan Á ˜ Mg
Ë 10 ¯
2g
M 2 + (4p M + M + m )
2
a=
(b)
2m + 3M
(c) Yes.
 2
(c) cos−1  
 3 101. (a) 5 rad /s2
8
3g 155
90. (a) (b) N
8 4
2g g (c) 5 N
(b) and
5 5 R
102. A point at a distance from centre.
1 2
4g 103. µmin =
91. ; Thread exerts more force on A g
11
5 mg 3 104. (a) F0 = Mg sinq
92. (a) mg, , mg 4
4 2 4
(b) 2 Mgh
3 105. (a) Zero
(b) g
4
F 2t 2
(b)
3m
21 − 3
(b) 106. (a) 1 m
6
(b)
12 m/s
2
Idu (c) 3.6 3 m
94. F =
2πR 4 107. 2T
108. t = 5 s.
3F
95. (a) 10
4M 109. No; m / s2
9
7F
(b) kx0
8 110. (a)
6M
g 6x
a= (b) 0
4 3 7
935 g
1 1 111.
Mg + 34 L
4 p2
2 Mg
15 g 15 g 112. (a) Vertically down
w0 = ;a 0 = ; Yes 123
32 R 128 R
m = 22.5
(b)
6g 113. 112.5 KJ ; No.
99. (a)
11
Rotational Motion 6.35

rw 0
114. (a) 2v
4 127. (a) w =
3R
1 2
(b) mr w 0
4 kEloss = Mv2
(b)

r 2w 02 J0
(c) 128.
32 m g 2
115. 5.8º
36 P0 cos q 36
116. 3gl 129. (a) w = =
7 m m
117. (a) P = mmg.V 2
2 Ê2 ˆ
MR 2 Lw 0 P =
(b) 0 P sin q + ÁË cos q ˜¯ = 85N - s
7
118.
2 m g ÈÎ MR 2 + mL2 ˘˚
130. 4( 2 – 1)m
Ê 8 + 3p 2 ˆ Ê mgR ˆ
119. V =
Á ˜Á ˜  
Ë p ¯ Ë 3 M + 8m ¯
2
131. (a) 2 gh − v 2 R − 1
0 2 
 6 r 
120. (a) cos−1 
 11 3
hmin =
(b) v0
3ga 2g
(b)
11 132. (a) m = 72 kg
Mga
(c) L
11 x>
(b)
2 3
-1 Ê 2 ˆ
q = cos Á ˜ 2
Ë 3¯ 133.
q q 3
1
mgR Ê m ˆ 134. (a)
(c) t max = ; =3 6
3 ÁË M ˜¯ max 49
122. No. (b)
52
h 14
123. 1 =
h2 11 2K
135.
31 3F
124. (a) MV 2
40 9
136. (a) w0
15V 8
(b)
14
(b) 9/8 
(c) V < 14gR
0

45/41 0

2 2 MR 2w
125.
5 m (u - v )
t
O t0 2t0

126. vc =
4
(
3v0 ˆ ˆ
i−j )
6.36 Problems in Physics for JEE Advanced

9 3Ja
137. (a) w 0 = w 149. (b) w 0 =
17 I
2 Mga
DE =
(b)
5
mV 2 +
7 3
mR 2 w 2 - mV Rw w=
(c)
I
(1 - cos (30∞ - q ))
68 34 17
(d) K = K0 + Mga sinq
6
h=
(c)
5 l g (2 + p R - L )
150. (a)
138. (a) Both have speed Ê Mˆ
ÁË l L + ˜¯
2
(b) The stick is at rest. Particle moves in original
direction with speed 2l 2 Rg (2 + p R - L )
(b)
Ê Mˆ
139. (a) = mR2 w0 wsin 2q is maximum for q = 45° ÁË l L + 2 ˜¯

F^ = 2mRw0 wcos q ; q = 0°
(b) mgh
151.
3 m+M
140. rad  43°
4
13
141. (a) v =
2 2 ( ) gR
2 -1 152.
10
Mv 20
3
mg
153.
M +m
w=
1 2 2 2 -1 g ( )
(b) 154.
3 3 R
g
Mg 155. (a) w =
142. T1 = T2 = T3 = L
3
(b)
hB = 5.04L
143. 1.5 cm
6v 7
144. (i) 156. (i) w n +1 = + wn
13r 13

n -1 ˘
(ii) F0 = mMg ( 2 –1) vÈ Ê 7ˆ
wn =
(ii) Í1 - ˙
r ÍÎ ÁË 13 ˜¯ ˙˚
145. (a)
v
(iii)
r
(b)
v02
157. (a)
146. (b) mg 5L
(c) zero
3v
(b) 0
2 L
 b
147. v = ga − 
  M p Lg
158.
2 3
gL
148. (b)  3  v 2   −1  3 
159. cos −1  1 −  + cos 
 
 10  5 g    10 
Rotational Motion 6.37

SOLUTIONS

1. Total length of thread unwrapped from the pulley = 3h


3h
Rq = 3h fi q =
R
2. In reference frame attached to the centre of the disc, the velocity of point P is along the tangent. A point which is
diametrically opposite to P has same velocity in opposite direction. This point has maximum speed relative to P.
This maximum speed is equal to 2v.

3. Let the Speed of centre of the ring be v. Then vA = 2v


After the ring rotates through an angle q, speed of point A is given by vector sum of v and wR = v as shown in figure

A
v
q
q

wR
=
v

Êqˆ
vq = v 2 + v 2 + 2 vv cosq = 2 v cos Á ˜
Ë 2¯

3 Êqˆ
When vq = v A = 3v fi 3v = 2v cos Á ˜
2 Ë 2¯
q 3
fi cos = fi q = 60∞
2 2
3
Speed of the point remains above v A during the interval the ring rotates from A to A1 and the from A2 to A
2 A

A2 A1
60° 60°


The required time interval is -
120∞ T
t= T=
360∞ 3
t 1

Required answer is =
T 3
6.38 Problems in Physics for JEE Advanced

2
MR 2
2  R
4. I1 = I CM + Md = +M
4  2 
3
= MR 2
4
ICM 2

I1
A
d
O R
30°
45°
d O R
A

I2

1
ICM

2
MR 2  R MR 2
I 2 = I CM + Md 2 = +M =

4  2  2
I1 3 2 3
∴ = × =
I2 4 1 2
1 2
It is important to notice that the MOI (Moment of Inertia) of the disc about the Z axis will be I zz = MR
5.
1 1 2
But Ixx = Iyy \ I xx = I zz = MR 2
2 4
y

x x
O

y
6. On folding about x axis the MOI about z axis does not change. If one third part is removed the MOI of the remaining
2
part will be 2 MR = 1 MR 2
3 2 3

7. I0 = ICM + Md2

M (x2 + y2) = I0 – ICM [= a constant]



 2 
( )
⇒ M x2 + y 2 = M  2 + 
 4
5 2 2 2
⇒x +y =
4
Rotational Motion 6.39

2R
8. The COM is at a distance of d = from the cylinder axis
p
MOI about the axis of cylinder is I = MR2
Using ICM = I – Md2
2
Ê 2R ˆ È 4 ˘
ICM = MR 2 - M Á ˜ = MR 2 Í1 - 2 ˙
Ë p ¯ Î p ˚
Iz
9. Hint: Using perpendicular axes theorem and symmetry one can show that I1 = I 2 = I 3 = I 4 =
2
2 2
10. I 0 = MR 2 and DI = mR 2 Where m = 0.02M
5 3
0.04
∴ ∆I = MR 2
3
∆I 0.04 5 10
∴ × 100 = × × 100 = %
I0 3 2 3
11. Hint: One can cut large number of triangular strips (as given in question) out of a disc of radius L and mass M. If all
such strips are pasted one over another we get the strip described in the problem.
Ma 2 Mb 2
12. According to the problem I = and I =
12 3
y

I
a

x
b

I
2 2
a b
∴ = ⇒ a = 2b
12 3
Ma 2 Mb 2 I 5I
∴ Iz = + =I+ =
12 12 4 4

14. I = IA + IB + IC

È MR 2 2 ˘ È MR
2 ˘
(
= 2 MR 2 + Í
ÍÎ 2
)
+ M (3 R ) ˙ + Í
˙˚ ÍÎ 2
(
+ M 16 R 2 + R 2 ˙
˙˚
)
58
= MR 2 = 29 MR 2
2
h2 - h1
15. y = h1 + .x
L
È h2 - h1 ˘
Area of strip of width dx is - dA = Íh1 + x ˙ dx
Î L ˚
6.40 Problems in Physics for JEE Advanced

M 2M
Mass per unit area is s = =
1 L (h1 + h2 )
L (h1 + h2 )
2
y

h1 h2
y

x
x
dx
L

MOI of strip about y axis is dI = sdA x2


È L h -h L ˘
\ I = Ú dI = s Í Ú h1 x 2 dx + 2 1 Ú x 3 dx ˙
Î 0 L 0 ˚

È L3 h - h L4 ˘ 3 È h1 h2 - h1 ˘ s L3
= s Íh1 + 2 1 ˙ = s L Í3 + =
4 ˙˚ 12 1
[h + 3h2 ]
ÍÎ 3 L 4 ˙˚ Î

ML2 Ê h1 + 3h2 ˆ
=
6 ÁË h1 + h2 ˜¯

1
16. The MOI of a uniform semicircular disc about axis AOB shown in figure is MR 2 (why?) and distance of its COM
4R 2
from O is . A
3p A

O O
R

B
B
The MOI of the composite disc about AOB is
1 1
I 0 = MR +

2
(3M ) R 2 = 2MR 2
2 2
4R
Let’s locate the COM of the composite disc. In the figure shown x0 =
3p

Xcm

M CM 3M
x

X0 X0


Rotational Motion 6.41

4R 4R
3M . -M
X cm =
3p 3p = 2 R
4M 3p

MOI of the composite disc about an axis through COM and parallel to AOB is
2
Ê 2R ˆ
ICM = I 0 - 4 M Á ˜
Ë 3p ¯

A
ICM

O
Xcm CM

2 16 MR 2
\ ICM = 2 MR -
9p 2
Ê 8 ˆ

= 2 MR 2 Á 1 - 2 ˜
Ë 9p ¯
17. Horizontal and vertical equilibrium of the stick requires -
F = T sin b Mg = T sin b ...........(i)
and mg = T cos b ...........(ii)
Solving (i) and (ii)
tan b = 1 b = 45°
T


F

Mg
T = 2 Mg
Rotational equilibrium requires torque about C to be zero (tc = 0)-
  
T cos b sin a - T sin b cos a = F cos a
2 2 2
È 1 ˘
T sin a - T cos a = 2 F cos a ÍQ cos b = sin b = ˙
Î 2˚
2 Mg (sin a - cos a ) = 2 Mg.cos a
\ tan a = 2 fi a = tan -1 (2 )
6.42 Problems in Physics for JEE Advanced

18. For rotational equilibrium, resultant torque about centre of mass of the car = 0

Nfa = Nra + (ff + fr) h

Nr Nf

CM F
a
a
h

fr ff

Mg
h
N f = Nr + f f + fr (
a
)

∴ N f > Nr
19. In equilibrium the COM of the semicircle must lie on the vertical line through A. This will result in no torque of
weight about A, hence equilibrium.
First we need to find the position of COM of a semicircular ring. For this consider an element of angular width dq
as shown.
p

ycm =
Ú ydm = Ú0 ( R sinq )(l Rdq )

Ú dm lp R

R dq

dq
q
O
R sinq

[where l is linear mass density and Ú dm = mass of the ring = lpR]


2R
\ ycm =
p
A

a R

O
CM
2R/p
B
Rotational Motion 6.43

From symmetry xcm = 0


2R / p
For equilibrium of the suspended ring tan a =
R
Ê 2ˆ
fi a = tan -1 Á ˜
Ëp¯

20. In critical case the COM of the system will lie at the centre of the circular base of the conical part (Just above A ; at
O)
1
Volume of cone = π 2
(3 ) = π 3

3
Volume of cylinder = r2 (4r) = 4 r3
Mass of cylinder is 4M
1.5r

2r
O
4M
0.75r

h 3r
COM of the cone is at a distance of = from O.
4 4
Let mass of the ring = m

m × 1.5r + M × 0.75r = 4M × 2r
29
∴m = M
6

21. (a) The ball will begin to climb the step if the line of action of its weight (Mg) passes to the left of point A. This
will cause an unbalanced torque about A.
R-h
In critical case (see figure) cosq =
R

h
R-
q
R

Mg
q

(b) In the position shown, the COM of the ball is vertically above point A at a distance R. As the ball climbs, its
centre of mass will lower from this position. (Otherwise why will the ball come down?) In the position shown
the centre of mass is at highest position.
6.44 Problems in Physics for JEE Advanced

22. For rotational equilibrium of the sphere, it is necessary that vertical line (along which force Mg acts) through COM
pass through contact point. This will ensure no torque about the contact point. Situation for maximum q is shown
in figure. Obviously, sinq 0.3.

0.3R
CM O

q
f

Mg q

If q is less than sin–1 (0.3) the sphere can be rotated so that vertical line through COM passes through contact point.
If q is larger than sin–1 (0.3), the line of Mg will pass from right of contact point and will produce an unbalanced
torque.
F kt
23. Acceleration a = =
m m
The force diagram of block B in reference frame attached to A is shown in figure. The normal force passes through
left edge at the instant the block is about to topple.
h d
The block will be on verge of toppling when ma = mg

ma d/2

h/2

f
O

mg

kt h d dgm
m
= mg ∴t =
m kh
24. For cylinder F = f (friction)
F

mg

f
Rotational Motion 6.45

And FR + f R = mg R

mg
fi 2 F = mg fi F =
2
25 For equilibrium of m0 ; T1 = m0 g
T2 T2

T1
m 30°

f T1
mg
A

m0 g

For rotation equilibrium of pulley


T2 = T1 = m0g
For translational equilibrium of cylinder + rod in horizontal direction

f = T2 = m0g (towards left)
For rotational equilibrium of cylinder + rod

mg R sin 30° = f R + T2 R
mg
= 2m0 g fi m = 4m0
2
26. Net torque about A = 0
O
q T

fs

P C A
N
Mg

3R R
= Mg.
fs
2 2
Mg È 3R 3R ˘
\ fs =
.........................(i) Í\ PA = 2 tan 45∞ = 2 ˙
3 Î ˚

T
Horizontal equilibrium N = ..............(ii)
2
T
Vertical equilibrium = Mg - fs
2

2 2 .............(iii)
T= Mg
3
6.46 Problems in Physics for JEE Advanced

2
Put in (ii) N = Mg
3
Mg 2
fs < m s N fi < m s . Mg
3 3
1


< ms
2
28. When spring is compressed by x the torque of weight of the sleeve is anticlockwise. But the torque of spring force
on the wheel is clockwise.

d
x
kx
mg

mg
These two torques can balance each other if k x d = mg. x fik=
d
29. Translational equilibrium requires
T = Mg.........(i)
T

Mg

For rotation
1
T = Ia fi T .R = MR 2 .a
2
1
fi Mg =
MRa [ using (i)]
2
2g
fia =
R

Using
2g
w 2 = w 02 + 2aq ; w 2 = 0 + 2. q
R
w2R
fiq =
4g
Rotational Motion 6.47

w2R
The cylinder rotates through angle q = by the time it acquires angular speed w.
4g

w 2 R2
Length of string unwound Rw =
4g
30. (a) For M = m acceleration of the two objects must be same. Because both experience a downward force mg and an
upward force T
Considering any one of them mg – T = ma...(i)

1 2
Rotation of the disc Tr = mr a
2
1
fiT =mra But ra = a + a
2
1
\ T = m (2 a ) fi T = ma...........(ii)
2
g
Solving (i) and (ii) a =

a1 + a2 = ra
(b) ..................(i)
Mg – T = Ma2 .................(ii)

mg – T = ma1 ................(iii)

T .r = 1 mr 2 .a fi T = 1 m a + a ................(iv)
2
( 1 2) 2

T
T a

a1
mg
M
Mg
a2
 3M − m 
Solving (ii), (iii) and (iv) a2 = g 
 3M + m 
1 M
For a2 to be negative m > 3M ⇒ >
3 m
2
31. For sphere TR = MR 2 .a
5
2
TR =
MR ( Ra ) But a = Ra = acceleration of m
5
6.48 Problems in Physics for JEE Advanced

2
∴ T = Ma.................(i)
5
For the block mg – T = ma ...........(ii)
mg 5mg
(i) + (ii) gives =afia=
2 5m + 2 M
m+ M
5
a 5mg
\a = =
R R (5m + 2 M )
Angular displacement of sphere while it rotates 5 turns = q = 2p × 5 rad

w 2 = w 02 + 2aq
10mg 100p mg
w2 = 0 + 10p =
R (5m + 2 M ) R (5m + 2 M )
100p mg
w=
R (5m + 2 M )
(b) After completion of 5 turns acceleration of m will become g

 5m 
∴ ∆a = g 1 −
 5m + 2 M 
 2M 
∆a = g 
 5m + 2 M 
32. For no slipping condition of thread acceleration of point A on the circumference of the disc = acceleration of the
small body
Ra a0 = a..............(i)
a
F

a0

A
m
T
a

Other equations are


T = ma .............(ii)
F + T = ma0 ............(iii)
1 1
And ( F - T ) R = mR 2 .a fi F -T = mRa .......(iv)
2 2

1
Using (i) in (iv) F − T = m ( a + a0 )
2
3 1
Using (ii) in this F = ma + ma ........(v)
2 2
From (ii) and (iii) F = ma0 – ma.........(vi)
Rotational Motion 6.49

F
Solving (v) and (vi) a =
4m
33. Let the normal reaction of the wall be N.
2R
Note that the force Mg will be acting at COM of the rod which is located at a horizontal distance of from the
centre p

Ê 2R ˆ
N.R = Á R - Mg
Ë p ˜¯
Ê 2ˆ
N = Á 1 - ˜ Mg
Ë p ¯
34. Friction f <mg sinq
Normal reaction N = mg cosq

Total force F = f + N < mg


35. The solid ball will not rotate. It will only translate. We can replace it with a point mass placed at the centre of the
shell. Torque about the point of contact is t = 2 MgR sinq
Moment of inertia about the rotation axis through point of contact is
Ê2 ˆ 8
I = Á MR 2 + MR 2 ˜ + MR 2 = MR 2
Ë3 ¯ 3
t 3g sin q
\a = =
I 4R
3g sin q
\ a = Ra =
4
You may also proceed by assuming a friction force on the ball and then writing equation of translation and rotation
about centre of mass. In an altogether different approach, you can use energy conservation to arrive at the same
result
Write the total mechanical energy of the rolling ball and take it to be a constant. Differentiate this equation with
respect to time to get the acceleration.
36. For h = h0 the sphere starts pure rolling. For h > h0, the angular speed acquired by the sphere is high such that
wR > V. In this case the contact point has a backward velocity. Friction acts in forward direction and the sphere
accelerates.

h0
C V
w


For h <h0, wR <V
The contact point has a forward velocity and the friction acts in backward direction. The sphere decelerates.
If linear impulse = J
J = MV...........(i)
Angular impulse about centre = Jh
6.50 Problems in Physics for JEE Advanced

2
MR 2w
Jh =
5
5 Jh 5 MV .h 5 h
\ Rw =
= = V
2 RM 2 RM 2 R
For pure rolling
2 2
V = Rw \ h =
R fi h0 = R
5 5
37. Hint : The acceleration of the plank will be twice that of the cylinder
V0
38. (i) The initial velocity of the centre of the sphere will be 2V0 and its angular speed is w 0 =
R

w0R=V0 2V0 3V0

2V0
+ 2V0 =
w0R=V0 2V0
V0

Because contact point is sliding forward, the kinetic friction will act backward.
This friction will produce a linear retardation (a) and an angular acceleration (a)

f
f fR 5f
a = and a = =
F 2 2 MR
MR 2
5
Velocity at time ‘t’ is
ft
V = 2V0 − at = 2V0 −
M
V0 5 ft
Angular velocity at time ‘t’ w = +
R 2 MR
If pure rolling starts at time ‘t’ V = wR

ft 5 ft
2V0 − = V0 +
M 2M
7 ft 2V M
∴V0 = ⇒t = 0
2M 7f
At this time speed is -
ft 2V 12V0
V = 2V0 - = 2V0 - 0 =
M 7 7
Rotational Motion 6.51


ALTERNATE METHOD :

w0=V0 /R w=V /R

2V0 V

Angular momentum is conserved about a point (like P) on the ground Lf = Li


2 2
MR 2 .w + MVR = MR 2 .w 0 + M (2V0 ) R
5 5
12
fi 7V = 12V0 fi V = V0
7
(ii) This problem can be solved by proceeding in a manner similar to the previous one.
39. Hint. The motion is translational. There is no rotation.

40. In 1
figure 2 =1 1 2 2 + 1 Iw 2
KE = (a)
MV KE
+ 2I wMV 2
2 2
2
1 1 1 ÊVˆ 3
= MV 2 + ¥ MR 2 Á ˜ = MV 2
2 2 2 Ë R¯ 4

In figure (b) MOI about rotation axis


Ê L2 R 2 ˆ Ê 3R2 R2 ˆ
I = MÁ + ˜ = M Á 12 + 4 ˜
Ë 12 4 ¯ Ë ¯
MR 2
=
2
2
1 1 Ê MR 2 ˆ Ê V ˆ
\ KE = MV 2 + Á Á ˜
2 2 Ë 2 ˜¯ Ë R ¯

3
= MV 2
4
Kinetic energy in case (a) does not depend on the length of the cylinder. It will increase in case (b) if the length is
increased.

41. Moment of Inertia of the rod about an axis through O is

ML2
I = ICM + Md 2 = + Md 2
12
4 ¥ 42
( )
2
= +4¥ 42 - 22
12
160
= kg - m 2
3
6.52 Problems in Physics for JEE Advanced

R = 4m
2m
CM d O
d O O 

2 2
= 4 −2 = 2 3

1 2
The rod is in pure rotation about O. Applying conservation of mechanical energy I w = Mgd
2
1 160
fi ¥ ¥ w 2 = 4 ¥ 10 ¥ 2 3
2 3
w 2 = 6 3 fi w = 3.2 rad /s
42. (i) Net force on the body = 0
However, it has a net torque acting on it.
t = F.2R – FR = FR = 20 × 1 = 20 N – m
t 20 5
\a = = = rad /s2
I 8 2
Angular speed at t = 2s is w = at = 5 rad/s
1 2 1 2
\ K = 2 I w = 2 ¥ 8 ¥ 5 = 100 J .

(ii) Let friction be = f


F

a
F

f

f = Ma f = 4a ...................(i)
And Ia = F.2R – FR – f.2R
8.a = 20 × 1 – 2f 4a = 10 – f
But 2 R.a = a 2a = a
10 – f = 2a ........(ii)
Rotational Motion 6.53

10 5
Solving (i) and (ii) a = m /s 2 = m /s 2
6 3
a 5
a = = rad /s2
2R 6
10 5
Hence at t = 2 s; V = at = m /s and w = a t = rad /s
3 3
1 1
\ K = MV 2 + I w 2
2 2
2 2
1 Ê 10 ˆ 1 Ê 5ˆ
= ¥ 4 ¥ Á ˜ + ¥ 8 ¥ Á ˜ = 33.33J
2 Ë 3¯ 2 Ë 3¯

1
43. In case A, motion is translational, kinetic energy given to the plate is K A = MV 2
2
In case B, the plate is in pure rotation about a horizontal axis through the upper end of the connecting rod.
1 2
\ KB = Iw
2
2
1 ÊVˆ
= ÈÎ ICM + Md 2 ˘˚ Á ˜
2 Ë d¯

3
d = distance of COM from the upper hinge = + =
2 2

Ê 3a ˆ ˘ Ê 2V ˆ
2 2
1 È Ma 2
\ KB = Í + M Á ˜ ˙Á ˜
2 ÍÎ 6 Ë 2 ¯ ˙ Ë 3a ¯
˚
1 Ê 29 ˆ
= MV 2 Á ˜
2 Ë 27 ¯

Hence, the plate has been given more KE in case (B)


Therefore, COM will rise to a greater height in case (B).
44. Energy conservation :
2
1
2
1 ÊVˆ
3m.g.2 R = mgR + mV 2 + mR 2 Á ˜
2 Ë R¯ ( )

5gR = V 2 fi V = 5gR

45. If w = angular speed


V

w
6.54 Problems in Physics for JEE Advanced

1 1Ê 1 ˆ 4
MV 2 + Á ML2 ˜ w 2 = MV 2
2 2 Ë 12 ¯ 3
1 5
= ( Lw ) = MV 2 \ (w L ) = 20V 2
2 2
24 6
wL
fi w L = 2 5V fi = 5V
2
wL
The velocity of one end of the rod is given by vector sum of its velocity of COM and = 5V (that is
perpendicular to length). 2

Speed of end point can range from ( 5 V – V) to ( 5 V + V) depending on the direction of velocity of the COM of
the rod.

( )
46. The change in angular momentum of the propeller is to the right DL if the plane turns to right. To conserve the
angular momentum the entire body of the plane will have a tendency to rotate in vertical plane. The plane will tilt
with its nose down. This will create an angular momentum in horizontal direction towards left for the plane.
4 3
pr
V 3 r
47. = 10 -12 fi = 10 -12 fi = 10 -4
V0 4 3 R
pR
3

w0

r M
M

Neutron star

Star

Angular momentum conservation gives -

2 2
MR 2w 0 = Mr 2w
5 5
2
Ê Rˆ p
( )
2
fi w = w 0 Á ˜ = 3 ¥ 10 4 = p ¥ 105 rad /s
Ër¯ 10

48. Let w = angular speed of square plate when the block moves out
Angular momentum conservation about rotation axis of the plate gives
1 × 2 × 1 = – 1 × V × 1 + 4w
V +2
fiw = ...............(i)
4
Energy conservation
Rotational Motion 6.55

1 1 1
¥ 1 ¥ 22 = ¥ 1 ¥ V 2 + ¥ 4 ¥ w 2
2 2 2
2 2
V + 4w = 4...................(ii)

2
From (i) 2  + 2
+ 4
  =4
4 
2 6
5 + 4 − 12 = 0 ⇒ = m/s
5
49. Impulse to the particle = mV (¨)
Impulse to the disc = mV (Æ)
R
Angular impulse about centre = mV

1 R
\ mR 2 .w = mV
2 2
V
\w =
R
50. No external force has torque about the rotation axis. Weight of the man is a force that is parallel to the rotation axis.
Hence, angular momentum of the system (platform + man) remains conserved.
I0 w0 = Iw
w0 w

O O

È1 M Ê 3R ˆ ˘
2 2
È1 2 M 2˘ 2 1 M Ê Rˆ
Í 2 MR + 2 .R ˙ w 0 = Í 2 MR + 12 2 ÁË 2 ˜¯ + 2 ÁË 4 ˜¯ ˙ w
Î ˚ ÍÎ ˙˚

[Note that MOI of the rod lying on disc can be written using I = ICM + md2]
24
Simplifying gives w = w0
19

b
51. (a) LA = MV0
2
N4
Neff
N3

N2 C C
N1

f A f
Mg
Mg

(b) About the centre (C) of the block there is no rotation. Mg has no torque about C. Friction has a clockwise torque
6.56 Problems in Physics for JEE Advanced

about C. The torque of normal reaction balances this. The effective normal force is to the right of C. If you look
about point A, Neff has higher torque than torque due to Mg since distance of Neff is greater
This produces an anticlockwise torque which eats away the angular momentum of the block
52. The centre of mass of the plate has co-ordinates given by (prove this yourself)
28a 28a
x=  a; y = a
9p 9p
The line of impact passes through COM hence there will be no rotation.
Momentum Conservation gives
mu mu 3u
=- + mV \ V =
2 22 4
All point on the plate will have this velocity only.
53. Angular momentum about the axis is
1 v
L = I w = mL2 sin 2 q
3 L sin q
1
fi L = mLv sin q

2
The angular momentum about point A is not a constant because it keeps changing direction.
54. The two forces on the mass are mg and T (tension). T has no torque about O. Torque of mg about z – axis is zero.
Hence the angular momentum of the mass about z axis will remain conserved as the string is dragged up
Z

O
q

l
h
T
T cosq

r m

mg

Lz = mr2w
const .......(i)
For motion of the mass

T cosq = mg............(ii)
T sinq = mw2r...........(iii)
(iii) ∏ (ii)

w 2r
tan q =
2
w2
tan q =  sin q [Q r =  sin q ]
g
g g
fiw =
=
 cos q h
Rotational Motion 6.57

g
Putting in (i) mr = a const
h
1
4
fir µhfir µ h4

1


When h becomes half, r becomes
Ê 1ˆ 4 times its original value.
ÁË ˜¯
2

Level 2
55. Two particles of a rigid body must have same velocity component along the line joining them.
y

B
3m/s

VAY 4

A x
4m/s 3


Velocity of ‘A’ along AB = velocity of ‘B’ along AB

3 4 3 4
4¥ + VAy ¥ = 3 ¥ + VBy ¥
5 5 5 5
12 + 4VAy = 9 + 4VBy
3
= VBy - VAy ....................(1)
4

Similarly, for A and C
4Vcx – 3VAy = 22...........….. (2)
And for B and C

VBy + VCx = 1 ..........,...….. (3)

9 13
Solving we get VAy = –3 m/s; VBy - m /s ; Vcx = m /s
4 4
  9  13
\ VA = 4i - 3 j; VB = 3i - j; VC = i - 2 j
4 4

Velocity of B to AB – velocity of A to AB
(b) Angular velocity = ————————————————————
AB
6.58 Problems in Physics for JEE Advanced

Ê 4 9 3ˆ Ê 4 3ˆ
ÁË 3 ¥ + ¥ ˜¯ - ÁË 4 ¥ + 3 ¥ ˜¯
5 4 5 5 5
=
5
1 1
= - rad/s ;w = rad/s

4 4

B 3

3sinq + 9/4cosq
9/4

tanq = 4/3

A q 4

4sinq + 3cosq
3

56.
R A

VP
P /2 
D
V0

V0=R
C V0


For a pure rolling motion V0 = wR
Velocity of a point like P will be along a line bisecting the direction of V0 and wR at P. One can easily show that
<APC + <PCA + <CAP = 180°
PAC is a triangle [with <APD = q/2]

VP is along PA
57. (a)

q V
O
X

R
From geometry, x =
sinq
dx dÊ 1 ˆ cos q dq
\ =R Á ˜ fi V = -R 2
dt dt Ë sin q ¯ sin q dt
Rotational Motion 6.59


− = ω = angular speed of the rod
dt
v sin 2 q
\w =
R cos q
For q = 30°
2
Ê 1ˆ
v ÁË 2 ˜¯ v
w = . =
R 3 2 3R
2
(b) Let the components of velocity of the centre of mass of the rod be vx and vy

Vx
w L2
Vy x

y
q 90-q

wL
The velocity of end A of the rod with respect to the centre will be = w R in the direction shown. For
resultant velocity of end A to be along x it is necessary that 2

v3 v
vy = w R cos q = =
2 3 2 4
58. (a) vA = v + wR = 2v and
w R 3v

vB = v + =
2 2
A
w V+wR

B V+wR/2

O V

3
(b) Let speed of A become equal to after the disc has rotated through an angle q about its centre.
2
w
A
V
q

q w
R

O V
=V


6.60 Problems in Physics for JEE Advanced

Speed of point A is v A = v 2 + v 2 + 2 vv cosq


3v 3 q
2 = v 2 (1 + cos q ) fi 2 = 2 cos 2

Ê 3ˆ
fi q = 2 ◊ cos-1 Á ˜
Ë 4¯
q q R 2R Ê 3ˆ
Time t = = = cos-1 Á ˜
w v v Ë 4¯
59. (a) With respect to COM, the acceleration of A is
aACM = aradial + a tangential

() ()
= w 2 R Ø + Ra (Æ ) = 2 3 Ø + 6 (Æ )
\ a A = a ACM + aCM
()
= 2 3 Ø + 6 (Æ ) + Ra (Æ )
=2 3 ( Ø ) + 6 (Æ ) + 6 (Æ )
=2 3 (Ø) + 12 (Æ )

(2 3 )
2
aA = + 122 = 156 ms-2


()
aBCM = w 2 R ≠ + Ra (¨) = 2 3 ≠ + 6 (¨)
(b) ()
\ aB = aBCM + aCM = 2 3 ≠ ()

(c) With respect to the COM the acceleration of P has two components –

ar

aP P

at
ar = w2r = r and at = ra = 3r
The resultant acceleration of P must be horizontal (¨) equal to 6 ms–2
1
fi ar cos q = at sin q fi = tan q
3
p
fiq =
6
and ar2 + at2 = 6 fi 2r = 6 fi r = 3m
Rotational Motion 6.61

60. The centre of the wheel is moving with constant speed on a circular path of radius 6 R. Hence it has a centripetal
v2
acceleration of ac = directed towards the centre of curvature of the convex surface.
6R
v
With respect to the centre of the wheel the contact point has acceleration equal to directed towards the centre
of the wheel. R

v 2 v 2 5v 2
Acceleration of the contact point in reference frame of ground is aP =
− =
R 6R 6R
61. The plank will lose contact with the horizontal surface after the centre of mass reaches the top point of the cylinder.
(Situation is shown in figure)
For this the displacement of plank relative to the centre of the sphere shall be equal to R. It means the centre will
move a distance R and the plank will move through a distance 2R.
Note that the speed of the plank is twice that of the centre of the sphere.
B C A

2R

62. Radius of curvature of the path is the radius of the circle that matches up with the path locally at a point.
Let’s first find the acceleration of point A with respect to the COM of the wheel. An observer at COM sees that
v
point A is spinning about it. Acceleration of A is
R
( )
↓ with respect to COM. Because the COM does not have
v
any acceleration hence acceleration of point A with respect to the ground is a A =
R
↓ . ( )
A

C V

For a person on ground, speed of point A is vA = v + wR = 2v.


If the radius of curvature of the path of A is r then
A
2V

aA


6.62 Problems in Physics for JEE Advanced

v 2 (2 v )
2
v2
\ a A = A i.e., =
r R r
fi r = 4R

63.

L
y

120°

x

Sine rule :
x y L
= =
sin (60 - q ) sin q sin 120∞
2L
\x = sin (60 - q )
3
dx 2 L Ê dq ˆ
\ = cos (60 - q ) Á - ˜
dt 3 Ë dt ¯
dq
Note - = w = angular speed.
dt
[q is decreasing, hence a negative sign]

When q = 20°,
dx
= 1.5m/s
dt
1.5 ¥ 1.732
\w = = 2.26 rad/s
2 ¥ 0.75 ¥ 0.766
64. (a) The top points of two cylinders have same speed as that of the plank. By looking at the motion of smaller
cylinder we can tell that this speed must be 2v. In pure rolling the speed of centre will be half the speed of the
top point. Hence speed of the centre of the bigger cylinder is v.
(b) The acceleration of two points is vertically downward with magnitudes:
v2 v2
abig = and asmall =
2R R
The required ratio is 1/2

65. Consider an element as shown


M

 d

Rotational Motion 6.63

Mass of the element dm = lRdq


Êqˆ
Distance of this element from point M is x = 2 R sin Á ˜
Ë 2¯
q
\ dI = x 2 dm = 4l R3 sin 2 dq
2
q0 q0
q
Ú Ú
3
\ I= dI = 4l R sin 2 dq
-q 0 -q 0
2

q0
= 2l R3 Ú (1 - cosq ) dq
-q 0

= 2l R 3 [q - sin q ]-0q
q
0

= 4l R (q 0 - sin q 0 )
3

66. (a) Let us first calculate the moment of inertia of the plate about the z – axis.
z

A R O
x

For part AOB : Mass = 3 m and Radius = R


and part AOB is one quarter of a circular disc of which z – axis is a diameter.
3mR 2
∴ I1 =
4
mR 2
[Q Moment of inertia of a disc of mass m and radius R about its diameter is . In the quarter of a disc mass
distribution remains same about the axis.] 4
A
O y
F
C


x
3mR 2
For part ACEO Moment of inertia about z axis is : I 2 =
Similarly for part EFOE
2
2
3 R 9
I3 = (6m)   = mR 2
2 2 4
3
[∵ MOI of a disc about a tangent perpendicular to its plane is mR 2 ]
2
\ Moment of Inertia of the plate about z – axis is
I = I1 + I2 + I3
Ê 3 3 9ˆ 9
= Á + + ˜ mR 2 = mR 2
Ë 4 2 4¯ 2

6.64 Problems in Physics for JEE Advanced

(b) Angular momentum is conserved about the z – axis.

R
O
y
R d

Moment of Inertia after the particle sticks to the plate


9 11
mR 2 + mR 2 = mR 2
I=
2 2
11
\ mR 2w = mvd
2
2 vd
fiw =
11R 2
67. Area of original plate A0 = a2
2
 a a2
A =
Area of bigger hole 1   =
 3 9
2
Ê aˆ a2
Area of each of 8 smaller holes A2 = Á ˜ =
Ë 9¯ 81

64 2
Area of plate with holes ; A = A0 − A1 − 8 A2 = a
81

 64 2 
Mass of area A =  a is M
 81 

81
Mass of area A0 (= a2) is = 0
64

 a2  9M
Mass of area A1 =

 9  is 64 = M 1

 a2  M
Mass of area A2  = is = M2
 81  64

Required MOI is
È1 Ê 2a ˆ ˘
2
1 1 Ê aˆ
2 È1 Ê aˆ
2
Ê aˆ ˘
2
Ê aˆ
2
2 Í
I = M 0 a - M1 Á ˜ - 4. Í M2 Á ˜ + M 2 Á ˜ ˙ - 4. M 2 Á ˜ + M 2 Á ˙
6 6 Ë 3¯ 6 Ë 9 ¯ Ë 3 ¯ Í 6 Ë 9 ¯ 3 ˜ ˙
ÍÎ ˙˚ Ë ¯
Î ˚
Rotational Motion 6.65

O 2a
OA 
3

A a
OB 
3

1 81 1 9 1 4 55 1 4 109 1
I= Ma 2 - Ma 2 - . . Ma 2 - . Ma 2
6 64 6 64 9 6 81 64 6 81 64
Ma 2 È 220 436 ˘ Ma 2 5824 91 2
= Í81 - 1 - - ˙ = 6 ¥ 64 ¥ 81 = 486 Ma
6 ¥ 6 4 Î 81 81 ˚
68. A square plate can be folded in the shape of described triangular plate without changing the mass distribution about
the said axis.
69. B

q/2 P
A D
dx
a


C
Êqˆ
h = AD = a cos Á ˜

Ë 2¯
Consider a strip as shown.
Êqˆ M 2M
dm = s 2 x tan Á ˜ dx where s =
= 2 \
Ë 2¯ 1 q q a sin q
.2a.sin .a.cos
2 2 2
q
tan
4M 2 . xdx
dm = 2
a sin q


MOI of strip about an axis through P perpendicular to the plane of the figure is
2
q
(dm ) ◊ ÊÁË 2 x tan 2 ˆ˜¯ 1 q
=
= (dm ) x 2 tan2
12 3 2

MOI about axis through A is

1 q Ê1 q ˆ
dI = dm ◊ x 2 tan 2 + dm ◊ x 2 = dm ◊ x 2 Á tan 2 + 1˜
3 2 Ë3 2 ¯
q
tan
4M
dI = 2 2 Ê 1 tan 2 q + 1ˆ x 3 dx
Á 2 ˜¯
a sin q Ë 3
6.66 Problems in Physics for JEE Advanced

q
tan h
4 M 2 Ê 1 tan 2 q + 1ˆ x 3 dx
2 ¯ Ú0

I= 2 Á ˜
a sin q Ë 3
4
q Ê qˆ
tan
q ÁË a cos 2 ˜¯ È q˘
= 4 M 2 Ê 1 2 ˆ Qh = cos
Á tan + 1˜¯ Í 2 ˙˚
a 2 sin q Ë 3 2 4 Î

q Ê 2q qˆ
sin sin + 3 cos2
2 q Á 2 2˜
= Ma 2 ◊ ◊ cos4 Á ˜
q q 2 q
2 ◊ sin ◊ cos2 Á 3 cos2 ˜
2 2 Ë 2 ¯
Ê q q qˆ
3 sin 2 + 3 cos2 - 2 sin 2
1
= Ma 2 Á
Á 2 2 2˜
2 3 ˜
Á ˜
Ë ¯
1 Ê 2 qˆ
= Ma 2 Á 1 - sin 2 ˜
2 Ë 3 2¯

70. (a) We will calculate the surface area of the end caps which got removed due to boring.

R/2

R
X
R
dq

60° q

Area of the ring element shown


dA = 2 (Rcosq) R.dq = 2 R2 cosq. dq
Area of one cap removed
q = 90∞
Ê 3ˆ
A = 2p R 2 Ú cos q dq = 2p R 2
[ sin q ]90∞
60∞
= Á 1 - ˜
2 ¯
2p R 2
q = 60∞ Ë
Area of both caps. 2A = (2 – 3) 2pR2
Mass of remaining sphere

=M-M
(2 - 3 ) 2p R 2

4p R 2
È 2- 3˘ 3
= M Í1 - ˙= M
Í
Î 2 ˙
˚ 2
Rotational Motion 6.67

(b) To calculate the MOI, we can take ring elements in the leftover part. We will add the MOI of each ring element.
M
Let mass per unit area be s =
4p R 2
MOI of a ring element
dI = dm (R cos q)2 = dA. (R cos q)2
M MR 2
= ◊ 2p R 2 cos q dq ◊ R 2 cos2 q = ◊ cos3 q dq
4p R 2 2
q = 60∞
MR 2
\I = 2 ¥
2 Ú cos3 q .dq
q =0
60∞
3 3
Ú cos
2 3
= MR q dq = MR 2
0
8

71. Note: This solution introduces you to a very different method of calculation of MOI based on use of dimensions and
parallel axes theorem.
The moment of inertia will be same as the moment of inertia of a triangular plate about an axis passing through its
centroid and perpendicular to its plane.

60°

a
I

Let the moment of inertia of such a plate of mass M and side length a about the central axis be I. If the triangle were
of side length 2a, its mass would have been 4M (Q Mass µ area)
Since MOI µ (mass) (side length)2
µ (4 M) (2a)2
Hence, MOI of a plate of side length 2a will be 16I about a similar axis, passing through G.

a
I G1

IV
G

II III

2a

16I = (MOI of part I about G) + ( MOI of part II about G) + (MOI of part III about G) + (MOI of part IV about G)
16I = 3I1 + I [since, first three terms are equal due to symmetry and the fourth term is I that we are trying to

calculate]
15I = 3I1 15I = 3[IG1 + M (GG1)2]
2
 a 
5 I = I + M   [you can show that GG = a ]
 3 1
3
6.68 Problems in Physics for JEE Advanced

1
I= Ma 2
12
Note : This method is very neat as compared to usual method by integration. As an additional exercise try to calculate
the MOI of a uniform rod of mass M and length L about an axis passing through its centre and perpendicular to its
length by using this method.
72. Let the mass of the block be m
Tco
sq

Ts
inq O

N
T f
Mg
sin
m q

Mg
mg co
sq q

For equilibrium of the block, T = mg...............(1)
For rotational equilibrium of the cylinder, net torque about its centre must be zero.
T◊R = F◊R. [R = radius of cylinder]
mg = f.........................(2)
For translational equilibrium of the cylinder
N = Mg cos q + T sin q
N = Mg cos q + mg sin q
and f + T cos q = Mg sin q
f (1 + cos q) = Mg sin q

Mg sin θ Mg sin θ
⇒ f = ⇒ mg =
(1 + cos θ) 1 + cos θ
M sin θ
m= ............(5)
1 + cos θ

Putting this in (3)


Mg sin 2 q Mg(cos q + 1)
N = Mg cos q + = = Mg
1 + cos q (1 + cos q )
\ f = mg £ m N
Mg sin q
\ £ m Mg
1 + cos q
sin q
£m
1 + cos q

Rotational Motion 6.69

73. f1

N1
N1

f2 N2
f1

mg mg

Cylinder Stick

For cylinder
f1 + f2 cos q = N2 sin q .....….. (1)
and N2 cos q + f2 sin q = mg + N1 ............….. (2)
Rotational equilibrium: f1R = f2R
f1 = f2 = f (say)
Equation (1) and (2) become
f + f cos q = N2 sin q .....….. (3)
and N2 cos q + f sin q = mg + N1 ............….. (4)
l
For stick mg = N1l
2
mg
fi N1 = .................(5)
2
Ê sin q ˆ
From (3) f = Á N ..........(6)
Ë 1 + cos q ˜¯ 2

Ê sin q ˆ
\Á N £ m N2
Ë 1 + cos q ˜¯ 2
sin q
\ £ m................(A)
1 + cos q

Substituting for N2 from (6) into (4)
Ê 1 + cos q ˆ
fÁ cos q + f sin q = mg + N1
Ë sin q ˜¯
È cos q + cos2 q + sin 2 q ˘ È mg ˘
fÍ ˙ = 3N1 ÍQ N1 = 2 ˙
ÍÎ sin q ˙˚ Î ˚

Ê 3 sin q ˆ
\f =Á N
Ë 1 + cos q ˜¯ 1
3 sin q
\ .N1 £ m N1
1 + cos q
3 sin q
\ £ m.....................(B)
1 + cos q
Note : f1 = f2 does not mean that N1 = N2
From A and B
3 sin q
m≥
1 + cos q
6.70 Problems in Physics for JEE Advanced

74. The object will perform oscillations if it is in stable equilibrium.

L
2

C
3R
8


It will be in stable equilibrium if its COM lies below centre of the sphere, C. Let the length of the rod be L. For COM
of the system to lie below C
3R  M  L
M > L
8  2R  2
3R2 3
fi > L2 fi L < ◊R
2 2
Note : For better understanding, consider three spheres : (A) a uniform sphere (B) a non uniform sphere whose COM
is above the geometrical centre (C) a non uniform sphere whose COM is below its geometrical centre.

CM

CM

(A) (B) (C)


In (A), the COM is always above contact point, hence force mg causes no torque about the point of contact. The
sphere is in neutral equilibrium. In (B), if the sphere rotates a little, the COM moves sideways and produces a torque
that tends to rotate the sphere farther away. The sphere is in unstable equilibrium. In (C), if the sphere moves a little,
the COM moves sideways and this produces a restoring torque. Hence this is position of stable equilibrium.
75. The diagram shows the relevant forces on the two cylinders.
f AB B

N
AB

A n q
gsi
MB fB

N
AB

f AB
q
g sin fA
MA


fAB = friction between the two cylinders;

fA = friction between A and the incline; fB = friction between B and the incline
For rotational equilibrium of the cylinders
Rotational Motion 6.71

fA = fAB [for A]
fB = fAB [for B]
fA = fB = fAB

For translational equilibrium of (A + B)
fA + fB = (MA+ MB) g sin
Ê M + MB ˆ
\ f A = f B = f AB = Á A ˜¯ g sinq
Ë 2
For A to be in equilibrium

Ê M + MB ˆ
N AB + M A g sin q = Á A ˜¯ g sin q
Ë 2
Ê M - MA ˆ
N AB = Á B ˜¯ g sin q
Ë 2
For the two cylinders to be in contact
NAB > O MB > MA
76.
B

60°

Mg
M N
T T
N1 N2

A C

L L
N1 L cos 60° = Tsin 60° + Mg cos 60°
2 4
3 Mg
N1 = T+ ...............(2)
2 4

L
N 2 L cos 60° = T sin 60°
2
3
N2 = T ..............(3)
2
6.72 Problems in Physics for JEE Advanced

3 Mg 3
T+ + T = Mg
2 4 2
3 3
3T = Mg ⇒ T = Mg
4 4

V0

dy

y
y

L
L
O
x
The x co-ordinate of COM of the sheet
M L M
− . − .L −3
x =
2 2 2 = L
cm
M 4
Torque of weight about y axis

Ê3 ˆ
t g = Mg ◊ Á L ˜ .............(1)
Ë4 ¯

Let’s calculate torque due to wind about y axis.

v
Speed of air at height y is v = y
L v0
Mass of air striking a strip of width dy in unit time is = r ◊ ( Ldy ) v = r Ldy ◊ y = r v0 y ◊ dy
L
Force on strip of width dy
v0 v2
dF = rate of transfer of momentum by the air = r v0 ydy ◊
y = r 0 y 2 dy
L L

0 3 rv2
Torque about y axis due to this force dt = ydF = y dy.
L
Total torque
L
r v02 3 r v02 3
t air = dt =

Ú L Ú0
y dy =
4
L .......(2)

The sheet will not topple if


r v02 L3 3
t air £ t g fi
£ Mg L
4 4
3Mg 3Mg
v02 £ 2
fi v0 £
r L r L2
Rotational Motion 6.73

78. For rotational equilibrium of B, the friction force on it due to A must be same as ground friction force on it.
N N

A
f f
Mg

N´ f

B
N Mg

f
For vertical equilibrium of A
2N cos 30° + 2f sin 30° = Mg

3N + f = Mg .........(1)
For Horizontal equilibrium of B

N cos 60° = f + f cos 30°
N 3f
= f+
2 2
N
f =
........(2)
2+ 3
Solving (1) and (2)
N
3N + = Mg
2+ 3
2+ 3 2+ 3 Mg
N= Mg = Mg =
2 3 + 3 +1 2 ( 3+2 ) 2
Solution to this question does not require any knowledge of torque. It has been deliberately included here to test
your conviction and faith on your self knowledge.
79. For equilibrium, the force applied by the incline (F ') must pass through point A, (otherwise rotational equilibrium
is not possible)

A
F

C
q
a F´
Mg
P

q=60∞
Let <CPA = a
then <CAP = q – a = 60° – a
6.74 Problems in Physics for JEE Advanced

Applying sine rule :


AC PC r R
= \ =
sin a sin (60∞ - a ) sin a sin (60∞ - a )
sin (60° – a) = 2 sin a
3 1 3
fi cos a - sin a = 2 sin a fi tan a =
2 2 5
 3
Required answer is 60° − tan −1    41°
80.  5 


C
G


A

C " Centre of circular base; G " Centre of mass CG = 3R 


A " Contact point  8 
CA is r to the inline.
The normal contract force (along AC), weight (along GA) and friction all pass through point A.
tan q = = 0.3 ; q = tan–1 (0.3)
In ACG
CG CA 3R R
= fi =
sin q sin (p - f ) 8 sin q sin f
8 ¥ 0.29
fi sin f = = 0.77 fi f = 50∞
3
81. A

2f


B F
f

C
Let friction on arm BC be f. Then friction on arm AB is 2f.
F = 3f  for uniform motion.
Net torque about B must be zero.
l
\ 2 fl sin a = f ◊ sin (90 - a )
2
1 Ê 1ˆ
fi tan a = fi a = tan -1 Á ˜
4 Ë 4¯
Ê 1ˆ
\ q = p - tan -1 Á ˜
Ë 4¯

Rotational Motion 6.75

82. (a) Stick is being rotated slowly. There is no acceleration.


Block remains in equilibrium.
Force by stick on the block is F1 = mg
F1
F1= mg

mg

 mg
0° 60°

(b) The angle of repose must be > 60°


m > tan 60° m> 3
(c) Consider Stick + Block as our system.
F

A f
(M + m)g

Torque about A = 0
L 1
\ FL = ( M + m ) g cos q fi F = ( M + m ) g cos q
2 2
For horizontal equilibrium
1
f = F sin q fi f = ( M + m ) g sin 2q
4
Plot of f vs q is as shown.
f

(M + m)g
4


O 45° 60°

83. (a) The three forces must be concurrent for equilibrium of the ladder. If the three lines (of action of the forces) are
not concurrent, then one force would produce a non zero torque about the intersection point of the other two.
(b) The intersection point of Fwall and W is O. The force Fg must also pass through point O.
6.76 Problems in Physics for JEE Advanced

Fwall O
Q

Fg
C



P

W = Mg

L sin q
tan a = = 2 tan q fi a = tan -1 (2 tan q )
L
cos q
2
(c) Friction is horizontal component of Fg. And vertical component of Fg must be equal to Mg.
f = Fg cos a
and N = Fg sin a = Mg [N = normal reaction of ground]
Mg Mg
\f = =
tan a 2 tan q
Mg
But f < mN fi £ m Mg
2 tanq

1
fi tan q ≥
2m
Ê 1 ˆ
fi q min = tan -1 Á ˜ .............(A)
Ë 2m ¯

(d) If COM is at a distance l from P then (as done in (b)) you can show that

L
tan a = tan q
l
And Fg sin a = Mg
F cos a = f
g

Mg l ◊ Mg
fi f = =
tan a L tan q

lMg
But £ m Mg
L tanq

l
fi tanq ≥
Lm

L
If l > ; then q > qmin [found in A]
Rotational Motion 6.77

84. Let the tension in the springs be T1, T2 and T3. The extension in spring 2 is larger than that in spring 1 (say by x).
Extension in spring 3 is larger than that in 2 by the same amount.
T2 – T1 = T3 – T2 .......(1)
Translational equilibrium
T1 + T2 + T3 = Mg.......(2)
Rotational equilibrium (tA = 0)
L 2L L
T2 + T3 = Mg
3 3 2
3
⇒ T2 + 2T3 = Mg........(3)
2
Solving (1), (2) and (3)
Mg Mg 7 Mg
T1 = ; T2 = ; T3 =
12 3 12

85. (a) When the end of the rod touching the wall has a tendency to slide up, forces are as shown.

N1

A
1 N1
B
N2

W
N2

N2 = N1 cosq1 + mN1 sin q1 .......…(1)


N1 sinq1 = mN2 + mN1 cos q1 +W.......…(2)
Taking torque about B

Ê a ˆ b
N1 Á ˜ = W sin q1 ...........(3)
Ë sin q1 ¯ 2
Eliminating N1 and N2 we get


2a È
b Î ( )
= 1 - m 2 sin q1 - 2 m cos q1 ˘ sin 2 q1........(A)
˚

(b) When the end B of the rod has a tendency to slide down, the direction of frictions mN2 and mN1 will be opposite.
Weight W will pass somewhere between B and A. The equation can be simply obtained by replacing m with –m
in equation (A).
2a È
b
( )
= 1 - m 2 sin q 2 + 2 m cos q 2 ˘ sin 2 q 2
Î ˚

3
86. The COM of the block + particle system is at a height of at a horizontal distance of from the geometric
centre (C). 4
6.78 Problems in Physics for JEE Advanced

L/4

CM
3h C
4
2Mg

It will be wise to consider torque about COM of the system (why?).


3h L 3h L
\f◊ = N ◊ fi m ◊ 2 Mg ◊ = 2 Mg
4 4 4 4
L L L
\h = = = .
3m 2 2
3◊
3
Note : you can make a gross mistake by balancing torque about C.
87. With the block at A, the cube is on verge of toppling.
N = mg
B A

C
L/ 4 L/ 4

O
P

Mg

Torque of Normal force (N = mg) by the block about O = torque due to weight of the cube
L L
⇒ mg = Mg ∴ m = M

Now as the block travels to the left, it exerts a friction on the cube to the left. Now there is no tendency of toppling
about O. The cube has tendency to topple about P if the friction is large. This tendency is maximum when the block
reaches point B. (Why?)
f

N
C
L
L/2

Mg

Cube will not topple if


L L
fL £ Mg fi m mgL £ Mg
2 2
1
fim£
2
[Q m = M ]
Rotational Motion 6.79

88. (a)

F
q

Toque about O
L
t =F cos q
2
ML2 L
\
◊ a = F cos q
3 2
3 F
fia = cos q
2 ML
dw 3 F
\w = cos q
dq 2 ML
w q
3 F
Ú w dw = 2 ML Ú0
cos q dq
0

w2 3 F F
= sin q fi w 2 = 3 sin q ......(i)
2 2 ML ML
3F
When q = 90°; w =
ML
Alternative :
Work done by F = k
L 1 Ê 1 ˆ 3F
F ◊ = Á ML2 ˜ w 2 fi w = .
2 2Ë3 ¯ ML
L 3 F
At angle q, the tangential acceleration of COM is acmt = a ◊ = cos q perpendicular to the rod.
2 4M

F
Ft
q acmr acmt

Fr

Let Fr and Ft be two components of hinge force along the rod and perpendicular to it.
F cos q – Ft = MaCM
3 1
Ft = F cos q - F cos q = F cos q
4 4
6.80 Problems in Physics for JEE Advanced

L 2 L
And Fr - F sinq = Mw 2 [ radial acceleration of COM is w ]
2 2

3 5
  Fr = F sin q + F sin q = F sin q [using (1)]
2 2
The hinge force makes 45° with the rod when
5 F
Fr = Ft fi F sin q = cos q
2 4
1
fi tan q =
10
89.
B

C w
C
L
2 q

Mg
A

Mechanical energy conservation


1 L
I Aw 2 = Mg (1 - cos q )
2 2
1 ML2 2 MgL Ê 4ˆ
◊w = ÁË 1 - ˜¯ .......(1)
2 3 2 5
3g 3 10
w= = ¥ = 2 rad /s
5L 5 1.5
L
vcm = ◊ w = 1.5 m /s
2
Momentum = Mvcm = 7.5 kg m/s
Torque about A:
IA ◊ a = tA

ML2 L
◊ a = Mg sin q ..........(2)
3 2
9 Ê gˆ È 3˘
fi a = Á ˜ = 6 rad / s2 ÍQ sin q = 5 ˙
10 Ë L ¯ Î ˚
COM is performing non-uniform circular motion with instantaneous angular acceleration a and angular speed w.
Acceleration of COM
C

q
ar q
at x
q
y
Rotational Motion 6.81

L L
◊ a = 4.5m / s2 and ar = w 2 = 3 m / s2
at =
2 2
4 3
\ a x = at cos q - ar sin q = 4.5 ¥ - 3 ¥ = 1.8 m / s2
5 5
a y = at sin q + ar cos q = 4.5 ¥ 3 + 3 ¥ 4 = 5.1 m / s2
5 5
Using Newton’s Second Law
N

aX
x
aY
f y

Mg

Fx = Max f = 5 × 1.8 = 9N
Fy = May Mg – N = May
5 × 10 – N = 5 × 5.1 N = 24.5 N
Friction force acting on the rod will be zero at the moment ax = 0
L L
fi at cos q = ar sin q fi a ◊ cos q = w 2 ◊ sin q
2 2
fi a cos q = w 2 sin q
3g 3g

2L
sin q ◊ cos q =
L
(1 - cosq ) [using (1) and (2)]
2
fi cos q =
3
L
T
T
L/2 a
O
A a
mg mg

Let angular acceleration of rod = a


Acceleration of block = acceleration of point A of the rod = a
a = La .........(1)
For block : mg – T = ma ........(2)

L 1 2
For rod : TL - mg = mL ◊ a
2 3

mg 1
T-
2
= ma.............(3)
3
[\ La = a ]
3
Solving (2) and (3) g=a
8
La a
a = La fi a1 =
(b) =
2 2
6.82 Problems in Physics for JEE Advanced

Net torque on the rod shall be zero


2 T = T0
a T T

O
A a
T0 mg
a1

mg
mg – T = ma ........… (1)
a ma
T0 - mg = m fi 2T - mg = ......(2)
2 2
(1) × 2 + (2) gives:
5ma 2g g
mg = ⇒a= and a1 =
2 5 5

Q B

T1 T2
a

N M
T1
R


A
Mg

Mg

1
T1 R − T2 R = MR 2 α
2
1 ∵ a
T1 - T2 = Ma
2
2g
a
11
4
11
7 Mg 6 Mg
T1 = and T2 =
11 11
Rotational Motion 6.83

0 3
92. Torque about O : τ = mg sin 60 = mg
2
l sin60°

60°
l
T3

2
l
T1
T2
l
T2
1 mg
a1

mg

t
3 3g
2 m 2 ◊ a = mg fi a =
2 4

3
a1 = α = g
4

3 3
T2 cos 300 = ma1 ⇒ T2 = mg
2 4
mg
⇒ T =

5
T1 = T2 sin 300 + mg = mg ..
4
Acceleration for particle 2
3
a2 = a = g
4
3
mg
T2 sin 60° 4

T3 T2

q T
2 co
s6
30° 0°
°
60
sin

mg
cos
mg

mg 60
°
a2
6.84 Problems in Physics for JEE Advanced

q
L
Mg cos q
2 3g
a0 = 2
= cos q
ML 2L
3
L cos q
2
A

Mg

x cos q

Mg

Mg

L
Mg cos θ + Mg x cos θ
2 3  L + 2x 
α = =  2 2 
g cos θ
1 2 2 2  L + 3 x 
ML + Mx
3
(a) For x = L
9 g
α = cos θ
8 L
q, a0 > a

3  L + 2x 
α = g cos θ
2  L2 + 3 x 2 

a =
dx
Rotational Motion 6.85

x2 ◊
2 6x2 L

6x 6Lx 2L2 fi
–3 + 9 + 12 21 – 3
x= fi x=
6 6
wr = u = a constant

1 dw 1 dR
 nw + nR = nu fi + =0
w dt R dt

dω ω dR
∴ =
dt R dt
w dR
a=
R dt

p RdR = (udt)d
dR u.d
∴ =
dt 2π R

w ud u2 d
a = ◊ =
R 2p R 2p R3

t a
u2d I u2d
⇒ F .R = I . 3
⇒ F =
2π R 2π R 4
95. F

N N
A a

L
3

O

L
For rod I 0 .a = FL - N
3
ML2 . NL
\ a = FL -
3 3
La N
M = F-
3 3
6.86 Problems in Physics for JEE Advanced

La
=
3

N .......................
∴ Ma = F −

.......................(

3F
a
4M
3F .
∴ N =
4
Acceleration of the COM of the rod is

L 3a 9F
acm = α = =
2 2 8M

3F
N= 4

Fx

3F 3F 9F
F- + Fx = Ma fi F - + Fx = M
4 4 8M
9F F 7F
Fx = - =
8 4 8
96.

a
c

N
a0
O

30°
A D
f
Rotational Motion 6.87

◊ 3R
..................(
fR = MR2◊ a0 f = MRa0 ..................(2
3
N = Ma + MR a 0
2
3
N = 2 Ma ÈÎQ a = Ra 0 ˘˚ ...................
2
1
( )
2
Mg 3R cos 600 - N 3R = M 2 3R ◊ a
3

c
N

B

A
Mg

Mg 4
-N = MRa
2 3
Mg 4 Ma 4 M
\ - = Ra
2 3 3
4M Mg
[ Ra + a ] = 2 ...................
3

3 ◊a

a sin 60° = 3 a fi a = 2Ra ...................


4 Èa ˘ g
Í 2 + a˙ = 2
3Î ˚
g
\a =
4 3
97.
Fy
A O B
Fx q
2R
p x
r
C
y
a
Mg

t
6.88 Problems in Physics for JEE Advanced

τA g
α = =
IA R

È 4 ˘
a = ra Í AC = r = R 1 + 2 ˙
ÍÎ p ˙˚

Direction of a is perpendicular to AC
2R . g g
a x = a sin q = ra sin q = =
p 2R p
g g
a y = a cos q = ra cos q = R. =
2R 2
Mg
\ Fx = Ma x =
p
Mg
Mg - Fy = Ma y \ Fy =
2

1 1
∴ F = Fx2 + Fy2 = Mg +
4 π2

3
8

R 3R
M −M
y0 = 2 8 = R
2M 16

R
2
C y0

O
3R
8
M


On disturbing the sphere, the torque of weight (about centre O) causes the motion. Obviously, the angular speed is
maximum when point C is at lowest position.

C
w
y0 O
O
y0
C

Rotational Motion 6.89

1 2
I ω0 = 2Mg ( 2 y0 )
2

2 2 16
I = MR 2 + MR 2 = MR 2
3 5 15

1 16 R 15 g
MR 2w o2 = 4 Mg. fi w0 =
2 15 16 32 R

R MgR
t = 2 Mg. =
16 8

O C

2Mg

MgR 16 MgR
Ia 0 = fi MR 2 .a 0 =
8 15 8
15 g
a0 =
128 R
The COM has horizontal acceleration as the sphere spins. This cannot happen in absence of a horizontal force.
The walls provide the necessary horizontal force. In absence of the walls, the centre of the ball will oscillate along
horizontal direction.
99.
T2
a
a0

2r
Mg

T1

4 Mg

a
6.90 Problems in Physics for JEE Advanced

0 a and a = Ra – a0 = 2ra – ra = ra
a .........................(
2 – T1 – Mg = Ma0 ..........................(

1 (2r) –T2 (r) = Ia


Mra È Mr 2 ˘
2T1 - T2 = ÍQ I = ˙
2 ÍÎ 2 ˙˚

Ê 1ˆ
3Mg = Á 4 + 1 + ˜ Ma0 ÈÎQ ra = a0 ˘˚
Ë 2¯
6
g = a0
11

12
V0 = a0 t = g
11

Velocity

a Mg

Mg – T = Ma .................(1)
Mr 2 . M.
Tr = a fi T= ra
2 2
2g
Solving (1) and (2); a =
3
If pulley stops ascending after time ‘t’ it means its upward velocity becomes zero.
V0 12 g 3 18
0 = V0 – at \ t = = ¥ = s
a 11 2 g 11
100.

a1

a2
r
a

mg
Rotational Motion 6.91

1 2 m
Tr =
2
mr . a 2 fi T=
2
( ra 2 )
1 M
TR =
2
MR 2 . a1 fi T=
2
( Ra1 )
Ra1 + ra2 = a
2T 2T T Mmg
\ + = g- \ T =
M m m 2m + 3M
Substituting the value of T in (2), (3) & (1)
2 Mg 2mg
a2 = a a1 =
( + 3M ) r
2 m ( + 3M ) R
2 m
2( M + m ) g
a =
( 3M + 2m )
(a) t0 = t
1 1
a t 2 fi 2p = 2 a 2 t0
2
\ q = w0t +
2
4p 2p ( 2 m + 3M ) r
\ t0 = =
a2 Mg
(b) Angular speed of lower pulley at time t0
A
ra 2

w 2r
x
cm y

a2

8p Mg
w 2 = w 02 + 2 a q fi w =
( 2m + 3M ) r
Acceleration of mark at A with respect to COM of the pulley is -

a ACoM = ra i + w 2 r ˆj
2

\

( )
aA = (ra 2 ) iˆ + w 2 r + a ˆj

Ê 2 Mg ˆ ˆ Ê 8p Mg 2 (M + m) g ˆ ˆ
=Á ˜ i +Á + j
Ë 2m + 3M ¯ Ë 2m + 3M 2m + 3M ˜¯

2g 2g
= È Miˆ + ( 4p M + M + m ) ˆj ˘ = È Miˆ + ( 4p M + M + m ) ˆj ˘
2 m + 3 M Î ˚
2 m + 3M Î ˚

2g
M 2 + ( 4p M + M + m )
2
\ aA =
2m + 3M
.
m ◊ R 1¥ R R 1
x= = = = m.
m + M 1+ 3 4 2
6.92 Problems in Physics for JEE Advanced

Icm = MR2 + Mx2 + m (R – x)2

x
O P
cm


2
R2 Ê Rˆ 3R 2 9 R 2 15R 2
= 3R 2 + 3 . + 1Á R - ˜ = 3R 2 + + =
16 Ë 4¯ 16 16 4
Let the acceleration of COM immediately after release be

acm = a x iˆ + a y ˆj
a

x
O cm
ax x

ay y
N (M+m)g

"), a.x (-­), ay (.)


Since, there is no acceleration in vertical direction for point O.

ax

ax

ay

R
\ ay = a ◊ x fi ay = a
4
ax
a \ ay =
4
Equations for translational motion are:
& f – 4ax

ax
( M + m) g - N = ( M + m ) ay fi 4g - N = 4 .
4
4g – N = ax
15 2
t a fi N . x - fR = Ra
4
Rotational Motion 6.93

N 15
- f = a x
4 4
Solving (1), (2) an d (3)
g g 31
ax = ;f = ; N = g
8 2 8
ax g 5
\a = = = rad / s2.
R 16 8
155
f = 5 N; N = newton
4

. =
a  =
a

a0

A
T
T

a mg

a
\ a


1 1
TR = MR 2 a fiT = MR a
2 2
a
T T 2T
g- = +
m M M
È3 1˘ Mmg
fi T Í + ˙ =g fiT=
ÎM m˚ 3m + M

= - = = (if m = M)
+ +
mg g
a0 = = (if m = M)
3m + M 4
2mg g
a = = (if m = M)
(3m + M ) R 2 R

P
a
x
O a0

A
6.94 Problems in Physics for JEE Advanced

xa = a0
xg g R
= ; fi x=
2R 4 2

T1
T2
T1 a

a1 T2

m 2m
3mg a0
a2

N
2mg
mg f

f= a
a
a
a

È a0 ˘
a ÍQ Ra = 2 ˙
Î ˚

a
a Q

mg
f=-
3

mg
Q f £ mN \ £ m 3mg
3
1
\ £ m
9
104. F

l
h

q
Rotational Motion 6.95

h
=
sinq
W = F ◊ 2 ¥ 2

\ F0 ◊ 4 = Mgh
4h Mg sin q
F0 ◊ = Mgh \ F0 =
sin q 4
Mg sinq
When F = 2 F0 =
2
WF = 2 Mgh
105. (a) If a is the angular acceleration (anticlockwise) of the cylinder with its centre at rest and a is the acceleration of
the plank to the right then there will be no slipping between the two if a = Ra
This ensures that the length of the thread that unwraps from the cylinder goes to increase the length of the lower
segment of the string and that the velocity of the point of contact of the cylinder and that of the plank are equal.
Therefore, the cylinder has no acceleration.
(b) For cylinder T = f and
f
1
TR + fr = mR 2 α T F  a
2
For the plank
F – T – f = ma
T
If a = Ra
2F
Solving we get a =
3m f
Displacement of the plank in time t is
1 1 2 F 2 Ft 2
S = at 2 = t =
2 2 3m 3m
F 2t 2
Work done by the force W = FS =
3m
As there is no loss of energy due to friction (there is no slipping), this work done is equal to gain in KE of the
system.
106. (a) Since the disc was rolling, the velocity of its top point at the instant of leaving the track was zero. It means
vH = wr
When the disc is in air vH and w both do not change. Hence the horizontal component of the velocity of the top
point P of the disc at every instant is zero and the vertical component of the velocity of the point P is equal to
the vertical component of velocity of the CM of disc.
2gh = 2.g.2( R - 0.1) = 6
4 × 10 (R – 0.1) = 36
R = 1 m.
6
(b) tan 30° =
vx
6
⇒ vx = =6 3
tan 30°
∴ v = vx2 + v y2 = 36 × 3 + 36
= 6 × 2 = 12 m/s.
6.96 Problems in Physics for JEE Advanced

(c) AC = vxt.
2 × 2 × 0.9
= 6 3 × = 12 3 × 0.3 = 3.6 3
10
107. Let acceleration on inclined part (while going up as well as while going down) be a. Length of inclined part be l
l1 and that of flat part be l2
Total time period is given as
 2 1 2 2 1 
T = + +  × 2.
 a 2 a 1 a 

If l1 and l2 is made 4 times, the time period doubles.
108.
a

a
R
f
a0

Let the friction force on the cylinder be f.


Translation : f = Ma.............................(1)

2 1
Rotation : fR = I cM a fi fR = MR ◊ a
2

1
∴ f = MRα........................(2)
2
For no slipping Ra + a = a0
Ra + a = 6 .....................(3)

1
Using (3) in (2) f = M (6 − a )
2

Put in (1) 1 M (6 - a ) = Ma
2
6 – a = 2a a = 2 m/s2
Let time required for the cylinder to roll off the sheet be t
Acceleration of sheet with respect to cylinder = a0 – a = 4 m/s2
1
∴10 = × 4 × t2;⇒ t = 5 s
2
109. Let the friction force between the cylinder and the plate be f1.
Note that maximum possible value of f1 is : f1max = 6◊g◊ m = 60 × 0.2 = 12 N
The friction force between the floor and the plate is
f2 = 7.5 × g × m = 7.5 × 10 × 0.2 = 15 N

Let acceleration of plate be a0 and acceleration of cylinder be a. If angular acceleration of cylinder is a, for no
slipping we have a + Ra = a0 ................. (i)
Writing the equation of translation and rotation for pipe :
Rotational Motion 6.97
a

a
f1 f1
F

f2 a0

f1 = Ma ......................(a)

f1R = MR ◊a 2
f1 = M (Ra).......................(b)

(a) and (b) imply that a = Ra
a0
From (1) a = Rα =
2
a0
Equation (a) becomes f1 = 6 = 3a0 ....................(2)
2
For plate : 25 – 15 – f1 = 1.5 a0...................(3)

10 10
(2) + (3) a0 = m /s2 fi a = ms -2
4.5 9
10
Putting in (2) f1 = 3 ¥= 6.67 N
4.5
Since, maximum permissible value of f1 is 12N hence it will adjust itself to 6.67N and ensure that the pipe does not
slide.
110. (a)
A B
F = 2kx0

F = 2kx0

fA (friction) fB (friction)

It can be shown that for no slipping condition:

F F
fA = and f B =
2 3
f A - fB F 2kx0 kx0
acm = = = =
2M 12 M 12 M 6 M

Initially, the acceleration of COM is towards left ( fA > fB) but once the spring gets compressed the direction of
forces will reverse and the acceleration of COM becomes towards right.
(b) Friction is static. It does not dissipate energy. The two cylinder will come to rest simultaneously (why?) when
the compression in the springs is x0.
F
aA F − f A 3
= = 2 =
aB F − f B 2F 4
3
x A 3 3 6x
∴ = ∴ x A = ( 2 x0 ) = 0
xB 4 7 7
6.98 Problems in Physics for JEE Advanced

111. Let angular displacement in extreme position be q.


A

L - L cos 

C D

Gain in potential energy UCD + UAB


L (1 - cos q ) 3
DU = mgL (1 - cos q ) + mg = mgL (1 - cos q )
2 2
Moment of inertia about rotation axis is
mL2 mL2 17
IA = + + mL2 = mL2
3 12 12
2
1 1 17 2  u  17 17
Kinetic energy in lowest position is k = I Aω 2 = mL   = mu 2 = mgL.
2 2 12  L 24 24
From energy conservation
3 17
mgL (1 - cos q ) = mgL
2 24
17 17 19
fi 1 - cos q = fi cos q = 1 - =
36 36 36
Torque in extreme position

mg

mg

L 3
t = mgL sin q + mg sin q = mgL sin q
2 2
3
mgL sin q
t 2 18 g 362 - 192 935 g
\a = = = ◊ =
I 17 17 L 36 34 L
mL2
12
112. (a)

M
6 f

3L w
4 a Mg
N
Mg 6


Rotational Motion 6.99

Let angular speed be w when rod gets horizontal.


Energy conservation
2
1Ê1 2ˆ 2 1 Ê M ˆ Ê 3L ˆ L M 3L
ÁË ML ˜¯ w + ÁË ˜¯ ÁË w ˜¯ = Mg + g
2 3 2 6 4 2 6 4
Ê1 3 ˆ 2 Ê 1ˆ
ÁË + ˜¯ w L = ÁË 1 + ˜¯ g
3 32 4
41 2 5 120 g
fi w L = g fi w2 = .............(1)
96 4 41 L
Torque in horizontal position

L Mg 3L 5 MgL
t = Mg + ◊ =
2 6 4 8
Angular acceleration in this position is given by Ia = t
È ML2 M Ê 3L ˆ 2 ˘ 5 MgL
Í + Á ˜ ˙a =
ÍÎ 3 6 Ë 4 ¯ ˙˚ 8
41 5 MgL 60 g
fi ML2a = \a=
96 8 41 L

3L 45
Tangential acceleration of the bead at = a. 4 = 41 g
Normal force by the rod on the bead is downward because downward acceleration of the bead is larger than g.
M M 45 2
N+ g= . g⇒N= Mg
6 6 41 123
If the bead just begin to slide outward then
M 2 3L
mN = w ◊
6 4
2 M 3 120
m◊ Mg = ¥ ¥ g [ using (1)]
123 6 3 41
3 ¥ 120 ¥ 123 45
m= = = 22.5
2 ¥ 6 ¥ 4 ¥ 41 2

113.

75 kg M

r2
CM

r1

m 50 kg

Distance of the astronauts from COM
6.100 Problems in Physics for JEE Advanced

Mr 75
r1 = = ¥ 10 = 6 m
M + m 75 + 50
mr 50
r2 = = ¥ 10 = 4 m
M + m 75 + 50

Motion of COM does not change (unless there is external force).


In reference frame attached to the COM, the angular momentum of the system remains conserved. Let the angular
speed be w when distance reduces to 5.0 m.
[74 × 22 + 50 × 32] w = [75 × 42 + 50 × 62] × 5
w = 20 rad/s
[Or, simply one could have argued that moment of inertia becomes one fourth and hence angular speed becomes 4
times]
Change in KE in COM frame = work done by astronauts.
1 2 1
\W = I w - I 0w 02
2 2
1ÊI ˆ 1 1
= Á 0 ˜ ( 4w 0 ) - I 0w 02 = I 0w 02 ( 4 - 1)
2
2Ë 4 ¯ 2 2
3 3
= I 0w 02 = ¥ ÈÎ75 ¥ 42 + 50 ¥ 62 ˘˚ 52 = 112.5KJ
2 2
114. (a) Let the final velocity of disc and plank are V and V1 respectively in the direction shown in the figure and angular
velocity of the disc be w.
For disc and plank system net force in horizontal direction is zero.
From conservation of momentum-
0 = mV – mV1 V1 = V
Since 50% of kinetic energy of system is lost V
1 2 1 1 1 È1 ˘
fi I w + mV 2 + mV12 = Í I w 02 ˙ .........(i) m, r
2 2 2 2 Î2 ˚
V1
Also for rolling without slipping m

2V
rw - V = V1 fi rw - V = V fi w = r

2
1 mr 2 Ê 2V ˆ 1 2 1 2 1 È1 2 ˘ rw 0
From (i) ◊ ÁË ˜¯ + mV + mV = Í 2 I w 0 ˙ fi V = 4 = V1
2 2 r 2 2 2Î ˚

(b) Change of angular momentum of disc about centre of mass of disc


Ê 2V ˆ
0 mr 2 Ê w ˆ mr 2w 0
DLC = I w - I w 0 = I ÁË - w0 ˜ = - w0 ˜ = -
r ¯ 2 ÁË 2 ¯ 4
f k µ mg
(c) Acceleration of plank till slipping ceases = = = µg
m m
2
Ê rw ˆ r 2w 02
V12 = 02 + 2as fi Á 0 ˜ = 2 m gs fi S =
Ë 4 ¯ 32 m g
115. Let w1, w2, & w0 be the angular velocity (all with respect to ground) of man 1 in anticlockwise direction, angular
velocity of man 2 in clockwise direction and angular velocity of the turntable in anticlockwise direction
respectively.
Rotational Motion 6.101

w1 – w0 = w2 + w0
From conservation of angular momentum –
1
m1r 2w1 + Mr 2w 0 = m2 r 2w 2
2
1
50w1 + 90w 0 = 60w 2 fi 10w1 + 9w 0 = 12w 2
2
10w1 + 9w 0 = 12 (w1 - 2w 0 ) fi 2w1 = 33w 0

If the angle through which the platform rotates is q


(90 + q ) = q 33
fi (90 + q ) = q
w1 w0 2
33 180
(90 + q ) = q fiq = = 5.8∞
2 31
116. There is no horizontal force on the rod and its COM must fall vertically. At the moment the rod BC is about to hit
the surface, velocity of AB will go to zero.
Let V and w be the speed of COM and angular speed of the rod when BC is about to hit the surface. For velocity of
part AB to be zero we must have -
C

B C

CM
l/4 cm
l
B 4
A V

l
V = w ....................(1)
4
3l
Velocity of point C at this moment is Vc = V + w ◊ 4 ....................(2)
Conservation of Energy

1 1 l
(2 m ) V 2 + I w 2 = (2 m ) g ◊
2 2 4
2 2
Ê lˆ ml 2 Ê lˆ 5ml 2
m
[I = MOI about the rotation axis = Á ˜ + + m ÁË ˜¯ =
Ë 4¯ 12 4 24
2
1 5ml 2 Ê 4V ˆ mgl
\ mV 2 + Á ˜ =
2 24 Ë l ¯ 2
8V 2 gl
=
3 2
3gl 4V 3g
fiV = and w = =
4 l l
3 3gl 3
\ From (2)Vc = V + w l = + 3gl = 3gl
4 4 4
6.102 Problems in Physics for JEE Advanced

117. (a)

on cylinder

R
f

f V

on belt
As long as the cylinder slips, friction force is f = mmg
As the belt keeps moving with constant speed, the extra power developed by the motor is
P = f◊V = mmg◊V...........................(1)
f ◊R 2f
(b) Angular acceleration is a = =
1 mR
mR 2
2
If cylinder stops slipping, it means
wR = V & a t◊ R = V [where t = time taken to stop slipping]
V mV
\t = =
Ra 2f

Work done by the motor in time t is (using (1))


1
W = Pt = fVt = 2 mV
2

Kinetic energy gained by the cylinder


1 2 1Ê1 ˆ 1
K = I w = Á mR 2 ˜ w 2 = mV 2
2 2Ë2 ¯ 4
50% Work done is spent as heat and remaining 50% is KE of the cylinder.
118. Friction force on the smaller disc is f = mmg◊ Torque due to friction (about axis AB)
tf = m mg◊r
t f m mgr 2 m g
Angular acceleration a = I = mr 2 = r
2
2 m gt
Angular speed of the smaller disc after time ‘t’ is w = a t =
r
Torque on larger disc about vertical axis is t = mmg L
MR 2 2 m mgL
fi a 0 = m mgL fi a 0 =
2 MR 2
Angular speed of larger disc at time t is
2 m mgLt
w 0’ = w 0 - a 0 t = w 0 -
MR 2
For no slipping wo' L = wr
2 m mgL2 t
fi w0 L - = 2 m gt
MR 2
MR 2w 0 L
fit =
2 m g ÈÎ MR 2 + mL2 ˘˚
Rotational Motion 6.103

119.

CM

X
O V
P R
2

R
2
2V

pR pR
As the disc moves through a distance , a length of thread gets unwind. Thus a point on the rope moves
through a distance pR. 2 2

Let’s apply law of conservation of mechanical energy to the system. Convince yourself that in original position the
COM of the rope was at a height of from the line OX.
p
È 2 R 3p R ˘ 1 1Ê1 ˆ 1
= MV 2 + Á MR 2 ˜ w 2 + m (2V )
2
\ mg Í + ˙
Îp 4 ˚ 2 2Ë2 ¯ 2

Ê 8 + 3p 2 ˆ 3 2 2
mgR Á ˜ = MV + 2mV
Ë 4p ¯ 4

8 + 3p 2 mgR
\V 2 =

p (3 M + 8 m )
Ê + p ˆÊ ˆ
= Á p ˜ ÁË ˜¯
Ë ¯ +

120. MOI of cube about rotation axis passing through P (lying in the face of the cube) is
2
Ma 2 Ê aˆ 5Ma 2
I= + MÁ ˜ =
6 Ë 2¯ 12
Loss in PE when the cube rotates by an angle q is

D = [ - q]

w
O O

a/2
a/2


P P
Mg

If angular speed is w when the cube rotates through q
1 2 a
I w = Mg (1 - cos q )
2 2
6.104 Problems in Physics for JEE Advanced

1 5Ma 2 2 a
w = Mg (1 - cos q )
2 12 2

12
\ Mw 2 a = Mg (1 - cos q ) ....................(1)
5
If the cube loses contact, the normal force (N) become zero.
È 2a ˘
\ Mg cosq = Mw
a ....................(2)
2 Íw 2 = centripetal acceleration of COM ˙
Î ˚
2
6 Mg 6
\ Mg cos q = (1 - cosq ) fi cosq 0 = 11
5

Put in (2) w 2 = 12 g
11 a
1 1 Ma 2 . 12 g Mga
\ Rotational KE is KE R = I cmw 2 = . =
2 2 6 11a 11
The rotational KE will not change after the cube leaves the table.
Speed of COM, at the instant of breaking off
a a 12 g 3 ga
Vcm = .w = =
2 2 11a 11
121. (a) Consider motion of the bead.
Energy conservation
1 mV 2
mV 2 = mgR (1 - cos q ) fi = 2mg (1 - cos q )
2 R
Equation for centripetal force
N + mg cos q = 2mg (1 – cosq)

N
q
V
mg



N = mg (2 – 3 cosq)

N is negative (i.e., outwards away from the centre) till 3 cos q > 2

Ê 2ˆ
fi cos q > 2 fi q < cos-1 Á ˜
3 Ë 3¯
Force by the bead on the ring will be radially inwards. This force will have a vertically downward component
and therefore the normal force by the ground will exceed Mg.
Ê 2ˆ
However, for q > cos-1 Á ˜ the normal force on the bead will be inward and the force applied by the bead on
Ë 3¯
the ring will be outward.
Rotational Motion 6.105

tN = N◊ R cosq
(b)
= mg (2 – 3 cosq)◊ R cosq = mgR (2 cosq – 3 cos2q)
N

q
A

dt N
tN is maximum when
(c) = 0
dq
1
fi - 2 sin q + 6 cos q sin q = 0 fi cos q =
3
Ê 1 1ˆ 1
\ tmax = mgR Á 2 ¥ - 3 ¥ ˜ = mgR
Ë 3 9¯ 3

The ring will rotate about A (i.e., it will rise) when this torque exceeds the torque due to Mg about A. If tmax remains
less than torque of Mg the ring will never rise.

\ Ring will not rise if


1 m
MgR ≥ mgR fi £ 3
3 M
122.

T = mg


m0g
For equilibrium : ,m0gR sinq = mgR
p
2 m sinq = m fi q=
4

B
p/ 4

p
4
A
P


6.106 Problems in Physics for JEE Advanced

Between P to A the pulley accelerates as torque due to tension will exceed the torque due to m0g. Between A to B
the pulley system retards and beyond B it will once again accelerate.
For m0 to climb to the top, we need to ensure that it just manages to cross point B. [Note that in no case m0 can reach
the top point with zero speed]
For kinetic energy of the system to be positive when m0 reaches B we must have

Ê pˆ Ê pˆ
mg R Á p - ˜ > m0 gR Á 1 + cos ˜
Ë 4¯ Ë 4¯

Ê 2 + 1ˆ
fi m.
3p
> m0 Á ˜ fi m0 <
3p
( 2m )

4 Ë 2 ¯ 4 2 +1( )
fi m0 < (0.975) 2 m

But it is given that m0 = 2 m


\ m0 will fail to cross point B.

123. Slope of the parabolic track at B can be obtained as

3 2 dy 3
y= x fi = x
2H dx H

q
X

H 3 2 H
At B ; = x ⇒x=
2 2H 3

dy 3 H
\ atB = ◊ = 3
dx H 3

fi tan q = 3 fi q = 60∞

KE of the sliding ring when it reaches B = loss in its potential energy

1 H
mV 2 = mg ⇒ V = gH
2 2

After B it will go like a projectile thrown at an angle of 60° to the horizontal.

\ Maximum height attained above B is

V 2 sin 2 60° gH 3 3H
= = =
2g 2g 4 8
Rotational Motion 6.107

V
3H
B 600 8
h1


H 3H 7 H
∴ h1 = + =
2 2 8
On track DEF the ring rolls. Hence, it acquires translational as well rotational kinetic energy. If speed of the centre
of the ring is V, when it reaches C, its kinetic energy is mV2.
H gH
∴ mV 2 = mg ⇒V =
2 2
The ring leaves the track at an angle of 60° to the horizontal.

V 2 sin 2 60∞ gH 3 3H
Further height attained = = =
2g 2.2 g. 4 16

\ Height of F above ground =

H 3H 11H h1 14
∴ h2 = + = ∴ =
2 16 16 h2 11

M 8M
124. Density r = =
4 3 4 Ê Rˆ
3 4
pR - p Á ˜ 7 ◊ p R3
3 3 Ë 2¯ 3

R
Mass of sphere of radius is
2
3
4 Ê Rˆ 8M M
mcavtiy = p Á ˜ ¥ =
3 Ë 2¯ 4 7
7 ◊ p R3
3
M 8M
Mass of sphere without cavity = M0 = M + 7 = 7

 8M   M
(a) Kinetic energy =  KE of a sphere of mass  –  KE of a sphere of mass 
7

1 2
[KE can be written as K = I w where I = MOI about an axis through contact point]
2

1Ê7 Ê Rˆ ˆ
2
1Ê7 ˆ
\ KE = Á M 0 R 2 ˜ w 2 - Á mcavity Á ˜ ˜ w 2
2Ë5 ¯ 2Ë5 Ë 2¯ ¯
7 8M 2 7 M 1 2
◊ = ◊V - ◊ ◊V
10 7 10 7 4
Ê4 1 ˆ 31
= Á - ˜ MV 2 = MV 2
Ë 5 40 ¯ 40

6.108 Problems in Physics for JEE Advanced

(b) Distance of COM from O = x


M R R
x◊M = ◊ \ x=
7 2 14

The sphere may be assumed to be in pure rotation about an axis through contact point P

CM

x
{
O

. P

VÊ R ˆ 15V
\ VCM = w ◊ ( R + x ) = ÁË R + ˜¯ =
R 14 14
The COM is moving in a circle of radius x about point O.
In the shown position, acceleration of point is zero (it will not be zero after some time !). We can write the equation
of motion for circular motion of COM as

CM

x
Mg w
O

Mw2x= Mg – N
2
V  MV 2
∴ N = Mg − M   x = Mg −
 R 14 R

MV 2
For N ≥ 0 ; Mg ≥
14 R
14gR > V

Note: (1) Had point O been accelerated, we would have to think about pseudo force
(2) Fext = M acm
125. Change in momentum of the bullet
p = m (u – v) = Impulse transferred to the ball.
The angular impulse transferred to the ball about X axis will be:
(Im)x = p◊y
Rotational Motion 6.109
y

x
y
O x


This impulse has vector direction along negative x direction.
For no rotation about x axis we must have -

2
Dpy = MR 2 ◊w ............................(1)
5

For the ball to gain same angular speed about y axis

2
(Im )y = Dpx = MR 2w ............................(2)
5
from (1) and (2) x = y

2 MR 2 ω
x=y= [∴ ∆p = mu − mv ]
5m ( u − v )

∴ r = x 2 + y2

2 2 MR 2 ω
r=
5 m (u − v )
126. We can apply conservation of angular momentum about a point on the surface just below A.

v Y
C C
a/2
vc
a/2 X
A
w

a a
LiA = M (4v0 ) (Clockwise) + M (2v0 ) ( Anticlockwise) = Mv0 a (Clockwise)
2 2
2
L fA = I Aw = Ma 2w (Clockwise)
3
2 3 v0
∴ Ma 2 ω = Mv0 a ∴ω =
3 2 a
Velocity of centre will be :
a
vc = w in the direction perpendicular to AC
2
6.110 Problems in Physics for JEE Advanced

3v0
\ vc =
2 2

3v  1 ˆ 1 ˆ  3v0 ˆ ˆ


∴ vc = 0 
2 2 2
i− j =
2  4
i−j ( )
127. (a) Let’s first locate the COM of the system. Let it be at a distance x from the centre of disc A
M (3 R )
x = =R
2M + M

A 2M w

B M

It means the COM is at point C where the two discs make contact.
Linear momentum of the system is zero. Hence the COM of the composite system will be at rest after collision.
The system will rotate about C with angular speed w after collision. We will apply conservation of angular
momentum about C
Lafter collision = Lbefore collision
IC◊ w = 2MVR + M◊2V◊2R

È3 3 2˘
Í 2 (2 M ) R + 2 ( M )(2 R ) ˙ w = 6 MVR
2
Î ˚
2V
fi 9 MR 2w = 6 MVR fi w =
3R
1 1 2 1
KEloss =  (2 M ) V 2 + ( M )(2V )  − ( IC ) ω 2
(b)
 2 2  2
2
= 3 MV 2 −
1
2
( )
 2V 
9 MR 2   = MV 2
 3R 

1 ML2
I CM = M ( 2 L )2 =
12 3

ML2
\ w = (J - J0 ) L
3
3 (J - J0 )
fiw =
ML
Rotational Motion 6.111

v0

O w

L L
JO J

Velocity of end O of the rod immediately after the impulse is applied is zero.

\ V0 = w L
J + J0
\ =3
(J - J0 )
M M
J
fi J + J 0 = 3J - 3J 0 fi J 0 =
2
J
fi J + J 0 = 3J - 3J 0 fi J 0 =
2
129. Angular impulse = change in angular momentum
3 È m 2 m 2 ˘ 36 P0 cos q Py vcm
fi P0 cos q = Í + ˙w fi w =
4 Î 12 16 ˚ 7 m
PO sinq
Px

Velocity of centre of mass after hit is
 9P0 cos q PO cosq
VCM = w =
4 7m
Let the rectangular components of impulse by the hinge be Px and Py
Px = P0 sin q and P0 cos q + Py = mVcm
2 P0 cos q
fi Py = mVcm - P0 cos q =
7
Impulse by the hinge has a magnitude
2
Ê2 ˆ
P = Px2 + Py2 = P0 sin 2 q + Á cos q ˜
Ë7 ¯
1
130. COM of rods AB and BC combine will lie on line BO at a distance m from B (at point C1)
2 2
1
COM of AD + DC will lie on DO at a distance from D (at point C2)
2 2
y u

A D
C
C2 C x
C x
12g x C
45 O
O O
C1 12g

B C

Fig (a) Fig (b) Fig (c)     Fig (d)


1 1 1
Distance OC1 = - = = OC2
2 2 2 2 2
6.112 Problems in Physics for JEE Advanced

COM of the square frame (indicated by C in diagram) will be at a distance x from O where

8 ¥ OC2 - 4 ¥ OC1 1
x= = m
8+4 6 2
For equilibrium, torque about O is zero
x 1
M monkey g y = 12 g \y = m
2 12

MOI of the square frame about an axis though O perpendicular to the plane of the figure is
Ê 2 ¥ 12 Ê 1ˆ ˆ
2
È 4 ¥ 12 Ê 1ˆ ˘
2

I =Á +2¥Á ˜ ˜ ¥2+ Í + 4 ¥ Á ˜ ˙ ¥ 2 = 4kg m 2


Ë 12 Ë 2¯ ¯ ÍÎ 12 Ë 2 ¯ ˙˚

Let the monkey jump with speed u and angular speed acquired by the frame be w.
From conservation of angular momentum
1
12.
I w = 12 ¥ u ¥ y fi w = 12 ◊ u = u
4 4
2
1 2 1 Ê uˆ u2
KE acquired by the frame K = Iw = ¥ 4 Á ˜ =
2 2 Ë 4¯ 8
This KE must be sufficient to raise the COM to the highest level shown in figure (d).

Ê x ˆ u2 1 Ê 2 - 1ˆ 2
\12 g Á x - ˜= \ 96 g Á ˜ =u
Ë 2¯ 8 6 2Ë 2 ¯
\ u2 = 8 ( )
2 -1 g

If height attained by monkey from its initial position is H


u2
H= = 4( 2 - 1)m
2g

131. Forces on the particle are – normal reaction of the frustum wall and weight of the particle.
These forces do not produce any torque about the central vertical axis. Hence, angular momentum of the particle
about the central vertical axis is conserved.

R
mvH r = mv0 R fi vH = v0
r

1 2 1 2
mv = mv0 + mgh \ v 2 = v02 + 2 gh
2 2
2
Ê Rˆ Ê R2 ˆ
vH2 + vv2 = v02 + 2 gh \ vv2 = v02 + 2 gh - Á v0 ˜ = 2 gh - v02 Á 2 - 1˜
Ë r¯ Ër ¯
Ê R2 ˆ
fi vV = 2 gh - v02 Á 2 - 1˜
Ër ¯

(b) The particle will not leave the frustum if vertical component of its velocity becomes zero before reaching the
Rotational Motion 6.113

bottom
 R2 
⇒ 2 gh = v02  2 − 1
r 

3v02
fi 2 gh = 3v02 fi h =
2g

v1
L
x
3
w C
v2
C

Before collision After collision

ML2 L MLw
w = mV1 fi V1 = ..........(ii)
12 3 4m

V1 - ( - V2 )
= 1 fi V1 + V2 = w L
L 3
w -0
3

Mw L w L w L
+ = ..............(iii)
4m 4 3
M 1
fi = fi m = 3M
m 3
L M L2 L2 w
(c) In above solution if we replace w + = wx
3 m 12 x 12 x
M L2 L2
⇒ =x−
m 12 x 12 x

L2 L
∴x > ⇒ x >
12 x 12

1 wL Ê Lˆ LÊ Lˆ 1
ML2w - M ÁË x - ˜¯ = Mw ¢ ÁË x - ˜¯ - ML2w ¢
12 2 2 2 2 12
6.114 Problems in Physics for JEE Advanced

O w w’ O

x x
CM vcm = w L/2
v’cm = w’ L/2

P P

Pole
Before collision After collision

ML2 Ê Lˆ L
fi (w + w ¢ ) = M Á x - ˜ (w ¢ + w )
12 Ë 2¯ 2
L L L L 2L
fi =x- fi x = + =
6 2 6 2 3
134. (a) Linear momentum of the system is zero. Hence, the centre of mass of the system (point P) is at rest after collision.
Let angular speed after collision be w0. Applying angular momentum conservation about point P gives:
v
M w P P
M w
w
v

Before collision after collision


LBC = LAC
2 2 2
ML w (CLK ) + MV L ( ACLK ) + ML w (CLK ) + MV L ( ACLK ) = 2 ML w (CLK )
0
12 2 12 2 3
Where CLK stands for clockwise and ACLK stands for anticlockwise.

ML2 2 w 3V
w - MVL = ML2 w 0
fi fi w0 = -
6 3 4 2L

V 1
For no motion w 0 = 0 fi =
wL 6
V 1 w
(b) If w L = 12 ; w 0 = 8

È ML2 2 Ê w L ˆ ˘ 13
2
È 1 ML2 2 1 ˘
KEinital = Í w + MV 2 ˙ ¥ 2 = Í w + MÁ 2 2
˜¯ ˙ = 144 ML w
ÍÎ 2 12 2 ˙˚ 12 Ë 12
ÎÍ ˙˚
2
1 2 ML2 2 ML2 Ê w ˆ ML2w 2
KEfinal = ◊ w0 = Á ˜ =
2 3 3 Ë 8¯ 192
È 13 1 ˘ 2 2 49 2 2
D KE = Í - ˙ ML w = 576 ML w
Î144 192 ˚
D KE 49
\ =
KEinital 52
Rotational Motion 6.115

1
And R F Dt =
2
MR 2w [Angular Impulse = change in angular momentum ]
1 2V
fi MVR = MR 2w fi w =
2 R
2x
fiq = fi Rq = 2 x
R
q
q = 2x =

\
K
◊ ⇒x=
F

2K
\ 2x =
3F

B
M/4
O
5M/4
O
A
M

1 M 45MR 2
I 2 = (20 M ) R 2 + MR 2 + R 2 =
2 4 4
1
I2 = (20M ) R 2 = 10MR 2
2
w w
2
45MR 9
10 MR 2 w 2 = w 0 fi w 2 = w 0 = 1.125w 0
4 8

w
w

w
vt


Situation at time t after start is as shown in figure.
6.116 Problems in Physics for JEE Advanced

M 2 È v ˆ ˘ MR 2
2
1 Ê
I = (20 M ) R 2 + M ( R - vt )2 + R = Í41 + 4 Á1 - t˜ ˙
2 4 ÍÎ Ë R ¯ ˙˚ 4

Angular momentum conservation gives :

È Ê Ê vt ˆ ˆ ˘ MR
2 2
45
Í41 + 4 Á 1 - Á ˜ ˜ ˙ w = MR 2 w 0
ÍÎ Ë Ë R ¯ ˙
¯˚ 4 4
45w 0
w = 2
Ê vt ˆ
41 + 4 Á 1 - ˜
Ë R¯

w w2

w1 w1

w0

t O t
O t0 t0 2t0

45
Angular speed when A crosses the centre w1 = w 0 = 1.098w 0
41
For motion of A from its initial position to B, the variation of w is as shown in first graph.
 5 
Similarly, one can write the expression of w when  + =  mass returns back from B to A. When at
4 4 
centre the angular speed is w2 (= 1.125w0) > w1. Hence the graph for complete journey is as shown in the second
graph.

137. The COM of the system is at A.


V
Momentum conservation gives : 2mV0 = mV ⇒ V0 =
2

w
V

CM
V0
A
w0


Before collision       After collision
Rotational Motion 6.117

1
I Aw 0 = mR 2 w + mVR
2
È3 2 1 2˘ mR 2 w + 2 mVR
Í2 mR + m ( 2 R ) w
˙ 0 =
Î 3 ˚ 2
17 mR 2 w + 2mVR 3( Rw + 2V )
mR 2 w 0 = fi w0 =
6 2 17.R
(a) If V = Rw
9
w0 = w
17
(b) Loss in energy:
1 1Ê1 ˆ È1 1 ˘
mV 2 + Á mR 2w 2 ˜ - Í (2 m)V02 + I Aw 02 ˙
DE =
2 2Ë2 ¯ Î2 2 ˚
V 17 3( Rw + 2V )
Put V0 = , I A = mR 2 ; w 0 =
2 6 17 R
5 7 3
D E = mV 2 + mR 2w 2 - mVRw
68 34 17

(c) Loss in kinetic energy is minimum when

d ( DE ) 5 3
=0 fi mV + 0 - mRw = 0
dV 34 17
6 6
fi V = Rw \ h =
5 5
w

m P P
V1 P’ u/2
u
x l / 6 x

w w
V2 u/2
C C

P’

In the diagrams the first, second and the third one represent the situations just before collision, just after
collision and just after second collision.
Momentum conservation
mu = mV1 + mV2 u = V1 + V2.............(i)
Apply conservation of angular momentum about a fixed point is space that coincides with the initial position of
the centre of the rod
m 2
mux = mV1 x + w
12
6.118 Problems in Physics for JEE Advanced

  
[Note: Angular momentum of the rod about said point is ICM w + R ¥ MV2 , where the second term is zero in
present context ]
   2w
u = V1 +
6 6 12
w
fi u = V1 + ........................(ii)
2 6
Collision is elastic hence we can use conservation of kinetic energy or the definition of coefficient of restitution
(e). Let’s use the later.

Relative velocity of point and the particle after collisiion


= -1
Relative velocity of point P and the particle before collision

V2 + w x - V1
= -1
o-u
w
fi V2 - V1 = u - .....................(iii)
6

u 6u
V1 = V2 = and w =
2 

'

q w

dL d
=
dt dt
( )
mR 2 w 0 sin 2 q = 2 mR 2 w 0 sin q cos q
dq
dt
t w w q q w0 w q
t q

dL
F^ R sin q =
dt
F^ R sin q = 2 mR 2 w 0 w sin q cos q fi F^ = 2 mRw 0 w cos q

V 30 m /s
w0 = = = 100 rad s -1
r 0.3 m
Rotational Motion 6.119

wC

w0 w
w

w
w w ◊w
ww (rad/s)

100

t
O t 2

dq
\ 80 = 0.3 ¥ 2 ÈÎ100 - (100 - 50t )˘˚
dt
q 2
\ 80 Ú0 dq = 0.6 Ú0 50 t dt
3 3
fiq = ¥ 2 = rad = 43∞
8 4

141. Let v = speed of COM before hitting at B


w = angular speed before hitting at B
v = wR [\ there is no slipping about A]

w’

O
O w
V
q

A A B
B
2R
(a) (b) (c)

Energy conservation
6.120 Problems in Physics for JEE Advanced

1 2 1
mv + ICM w 2 = mgR (1 - cos 45∞)
2 2
1 2 1Ê1 ˆ Ê 1 ˆ
mv + Á mR 2 ˜ w 2 = mgR Á 1 - ˜
2 2Ë2 ¯ Ë 2¯
1 2 1 2 Ê 2 - 1ˆ
v + v = gR Á ˜
2 4 Ë 2 ¯

2
4 ( 2 -1 ) gR = 2 2 ( 2 -1 ) gR
v = 3
3 2
Angular momentum conservation about B gives:

3 1
I Bw ¢ = ICM ◊ w fi mR 2w ¢ = mR 2 ◊ w
2 2
w v
w ¢ = 3 = 3R

[Note that before impact at B, the velocity vector of the COM passes through point B. Hence, the second term
in expression of angular momentum = 0]

142. Considering rotational equilibrium about the median BD gives -

T1

A
l2

T3 D
T2
G

B C

T1 = T2
Similarly, T1 = T3
Mg
Therefore T1 = T2 = T3 =
3
143. Originally, each spring is compressed by x0
4kx0 = Mg ........(i)

k (x0 + x) k (x0 – x)
kx0 kx0
3 2 3 2

kx0 C kx0 k (x0 – x) C k (x0 + x)

4 4
1 1

Mg Mg
6cm

    (a) (b)
Rotational Motion 6.121

After insertion of wooden block below spring at 1, let further compression in spring at 1 be x. Spring force at 1
increases by kx. The spring force at 3 must also increases by same amount so that net torque about diagonal 2 – 4
remains zero. Hence, spring at 3 compresses further by x. The resultant of all spring forces must remain Mg. Hence,
spring at 2 and 4 must extend by x [i.e. their compression will decreases by x].
Point 2 and 4 both rise by x (from their original position in figure (a)). C must lie on line 2-4. Hence C must also
rise by x.
C also lies on the line 1 – 3
1’
C’ 6-x
3 x
x C 1

3’

È M˘
df = ml dy g Íl = L ˙
Î ˚
F A

x dy

t ml
\
x L-x ml g È 2
x + (L - x ) ˘
2
t f = ml g Ú y dy + ml g Ú y dy =
0 0 2 Î ˚
◊ t

ml g È
Íx +
( L - x )2 ˙˘
fiF=
2 Í x ˙
Î ˚
6.122 Problems in Physics for JEE Advanced

dF L
Now F is minimum if =0⇒ x=
dx 2
L
x=

ml 2 –1 m 2–1
[Note : Net friction force on the rod is
È L/ 2
Ê 1 ˆ
L Á 1- ˜ ˘
f = ml g Í Ú dy - Ú Ë 2¯ dy ˙
ÍÎ 0 0 ˙˚

= ml g
2
L Ê
- ml gL Á 1 -
Ë
1 ˆ
˜=

( )
2 - 1 m Mg

This is equal to F0. Because of this, as F > F0 we see that COM of the rod beings to move.

w w

P T

A VA fB

w
fA V0 VB

v
fi v0 + w R = v fi 2v0 = v fi v0 =
2
w

∴ 0 =
2
q
L
t = Mg cos q
2
L
Mg cos q
t 2 3g
a= = = cos q
I 1 2 2L
ML
3
3
at = La = g cos q
2
Rotational Motion 6.123

3
at v = g cos2 q
2
A A

q0 a t
L/2 atv

O q q0

a
Mg

3 2 2
\ g cos2 q 0 ≥ g fi cos2 q 0 ≥ fi cos q 0 ≥
2 3 3

(b) Before the system is released, the block is at rest


\ Contact force = mg
(c) After release, Contact force = 0
Because acceleration of end A has a vertical component = g
147. When the string gets vertical, the centre of the rod has horizontal velocity and the angular speed of the rod will be
dq
zero. At this point angle q has become minimum (i.e. = 0 ). Velocity of C is horizontal. Velocity of B is also
horizontal. dt
\ Rod has a horizontal velocity and no angular velocity.

Energy conservation gives :



1 b  b − a  b
Mv 2 = Mg − Mg  ∴v = ga − 
2 4  2   2

148. (a) The median to the hypotenuse of a right angle triangle has half the length of the Hypotenuse.
L
∴ OC = (always)
6.124 Problems in Physics for JEE Advanced

q
L/2
L

C
L/2
L/2 cosq
q L/2
B

q
L L L
- cos q = (1 - cos q )
2 2 2

1 1 L
(2m)v 2 + I w 2 = 2 mg (1 - cos q )
2 2 2

1 È Ê Lˆ ˘Ê v ˆ2
2
mv 2 + Í Á ˜
m ¥ 2 ˙Á ˜ = mgL (1 - cos q )
2 ÍÎ Ë 2 ¯ ˙˚ Ë L / 2 ¯
v 2 + v 2 = gL (1 - cos q )
gL (1 - cos q )
v= = Speed of COM
2
q q
q
q w
Ê L ˆ
ÁË\ 2 w = v˜¯

gL
vx = v cos q = (1 - cos q ) ◊ cos q
2
dvx 2
= 0 fi cos q =
dq 3
q

gL (1 − 2 / 3) gL
v= =
2 6
Rotational Motion 6.125

=J ( a)
\ I A w 0 = 3 Ja
3 Ja
fi w0 =
I
J

3a

q
\ q
\ q)

C
C
P
a a
q B
30-q

N A
A
q
q

1 2
\ I w = Mga - Mga cos(30∞ - q )
2 0
2 Mga
\ w0 = È1 - cos (30∞ - q )˘˚
I Î
1
(d) The centre of mass of the pencil falls through a height a sin q
6
\ q

C
a
q

C’

a Sin q


6.126 Problems in Physics for JEE Advanced

m1 = l 
lp
l [L – l – p ]
\

T2 T1
T2
T1

m2 g
m1 g
Consider pulley + part of rope on pulley
È1 ˘
T1 R - T2 R = Í MR 2 + m0 R 2 ˙ a
Î 2 ˚
ÊM ˆ
T1 - T2 = Á + m0 ˜ a..................(iii)
Ë 2 ¯
[Q a = Ra ]
(i) + (ii) + (iii)
Ê M ˆ
(m1 - m2 )g = Á m1 + m2 + + m0 ˜ a
Ë 2 ¯

\a =
(m1 - m2 ) g =
l g [2 + p R - L ]
Ê Mˆ Ê Mˆ
ÁË m1 + m2 + m0 + 2 ˜¯ ÁË l L + 2 ˜¯

The x co-ordinate of COM of the pulley + rope system is


l R - l ( L -  - p R ) R
X=
M + lL
l R [2 + p R - L ]
\X =
M + lL
Y

X
O


Rotational Motion 6.127

\ X component of acceleration of COM is


d2 X 2l R d 2  2l R
ax = = = ◊a
dt 2 M + l L dt 2 M + lL
\ Fx = ( l L + M ) a x = 2 l Ra
2l 2 Rg (2 + p R - L )
\ Fx =
Ê Mˆ
ÁË l L + ˜¯
2
151. Since there is no slipping the toy travels a horizontal distance h in the time the block travels a vertical distance h.
v = velocity of block with respect to the toy (Ø)
v = velocity of toy with respect to the ground (Æ)
Velocity of block with respect to ground = 2 v
V
45°

V
2V

V
h

h
2V

Energy Conservation
1
( 2v) 1
2
m Mv 2 = mgh
+
2 2
2mgh
∴v =
2m + M
mgh
∴ Answer is 2v = 2
2m + M
152. w = angular speed
wR
= v0 for pure rolling condition
2
1 1
KE = Mv02 + I w 2
2 2

1 12 1 1 13
MR 2w 2 = MV02 + M (2 v0 ) = Mv02
2 2
= Mv0 +
2 25 2 5 10
153. Let a0 = acceleration of each ring and a = angular acceleration of each ring
a

N N
N
x 45
45° 45°
Mg

N0
a

f Mg
6.128 Problems in Physics for JEE Advanced

For one ring


N
− f = Ma0
2
◊a
N
= 2Ma0
2

N
mg - 2 ◊ = ma fi mg - 2 N = ma
2

dy dx
y 2 + x2 = R2 ⇒ 2y + 2x = 0
dt dt

R
y
q
x


2 2
 dy  d 2 y  dx  d2x
  + y +   + x =0
dt dt 2  dt  dt 2
dy dx
= =
dt dt
d y d x
∴y = −x
dt dt
\
q=
\
mg
a = a0 =
4M + m
154. At any given instant the ring and the rope around it is moving. Rest of the rope is not moving at any given instant.
The motion is identical to that of a wheel rolling without sliding
Taking torque about A :

A
a

2Mg
Rotational Motion 6.129

a t

g
◊a ◊ \a =
2R
g
\ a = Ra =
2
Ø
2u 2p gL L
T= = = 2p
g g g

2p g
w= =
T L

u2 p 2 gL p 2
H= = = L
2g 2g 2

w L/2
w
B A
B
C

H q C

V w

A B
C
Stick at top At time t after the centre begins to fall

g
w = wt = t
L
wL
2

wL g L Ê g ˆ
cos q = v fi ◊ cos Á t ˜ = gt
2 L 2 Ë L ¯
Ê g ˆ g
fi cos Á t˜ = 2 t
Ë L ¯ L

g L
∴ t = 0.45 ⇒ t = 0.45
L g
6.130 Problems in Physics for JEE Advanced

1 2 L
hB = H - gt + sin q
2 2
2
p2L 1 Ê Lˆ L
= - g Á 0.45 + sin(0.45)
2 2 Ë g ˜¯ 2
È 3.142 0.452 0.43 ˘
=Í - + ˙ L = 5.04 L
ÍÎ 2 2 2 ˙˚

1
mvr = mv1r + 2mr 2 w1 + 4m(2r )2 w1
12
10
fi v - v1 = rw1 ..............(ii)
3

6v
w1 =
13r

w w
10 2 10
And mvr + mr w1 = mv2 r + mr 2 w 2 .......... (iv)
3 3

6v 7
Solving (iii) and (iv) gives w 2 = + w1
13r 13
6v 7
It means w n +1 = + wn
13r 13
Say wn +1 = a + bwn
\ w1 = a + 0 = a
\ w2 = a + bw1 = a + ab
\ w3 = a + bw2 = a + ab + ab2
È1 - b n ˘
w n +1 = a + ab + ab2 + ........ + ab n = a Í ˙
ÍÎ 1 - b ˙˚
È Ê 7 ˆn ˘
Í1 - ˙
6 v Í ÁË 13 ˜¯ ˙ v È Ê 7 ˆ ˘
n
= = Í1 - Á ˜ ˙ n = 0,1, 2, 3......
13r Í 7 ˙ r Í Ë 13 ¯ ˙
Í 1- ˙ Î ˚
13 ˙
ÎÍ ˚
vÈ Ê 7ˆ
n -1 ˘
Or, w n = Í1 - Á ˜ ˙ n = 1, 2, 3,......
r ÍÎ Ë 13 ¯ ˙˚
n -1
Ê 7ˆ
When n Æ •, Á ˜ Æ0
Ë 13 ¯
v
\ wn Æ
r
Rotational Motion 6.131

157. (a) Let tension in string immediately after the projection of the particle be T
a = acceleration of COM of the rod
a = angular acceleration of the rod
Acceleration of end A is
a
A
a

1 1
TL = m ( 2 L )2 ◊ a fi T = m ◊ La ..........(ii)
12 3

T
maA

V0

mv02
T + ma A =
L
mv02
T + ma + mLa =
L
mv02 mv02
T + T + 3T = \T =
L 5L
2
T v0
\a = =
m 5L

v0
vcm =
2
A V0 /2

L/2
L/2
CM


V0 /2

2
6.132 Problems in Physics for JEE Advanced

2
w
0

CM

V0 /2
w
A

v0 L v L 1
m + m 0 = m ( 2 L )2 ◊ w
2 2 2 2 12
2 2 3 v0
\ mv0 L = mL w \w =
3 2 L
158. The vertical component of impulse on the rod will give it an upward velocity (say u) and there must be a horizontal
component of impulse which provides an angular speed.
In the first figure JH is perpendicular to the plane of the figure and in the second figure Jv is perpendicular to the
plane of the figure.
u
The time of flight t =
g
u JH


CM

JV JV JH

Angle rotated by the rod in this time = p


pg
\ wt = p fi w=
2u
Mu
Hence, J v = Mu ⇒ Jv =

ML2 p g È M ( 2 L )2 ML2 ˘
And 2 J H L = ◊ Í I CM = = ˙
3 2u ÍÎ 12 3 ˙˚
MLp g
\ JH =
12u
Impulse applied by one boy has magnitude J given by

J 2 = J H2 + J v2

M 2 L2 p 2 g 2 M 2 u2
J2 = + ..............(i)
144 u2 4
Rotational Motion 6.133

d (J 2 )
2
J is minimum when =0
du
M 2 L2 p 2 g 2 M2
fi -2 +2 u=0
144 u3 4
L2 p 2 g 2 Lp g
fi u4 = \ u2 =
36 6
Substituting in (i)
2 M 2 L2p 2 g 2 M 2 Lp g M 2p Lg
J min = + =
Lp g 4 ◊6 12
144 ◊
6
M p Lg
J min =
2 3
159. Conservation of angular momentum about the point of suspension gives
mv = I0 w……………(1)
m 2 m 2 Ê 2 ˆ 5
I0 = + + m Á +  2 ˜ = m 2
3 12 Ë 4 ¯ 3
5m  2 3v
\ mv = w fi w = .............(2)
3 5
w is the angular velocity of the combined system just after collision.
O

3,
h 4
a
a
q
CM
v

1 
I 0w 2 - mg - mg = -2mgh
2 2
3 È v ˘ 2
fih= Í - ˙ ..................(3)
2 ÍÎ 2 10 g ˙˚
3 / 4 3
cos a = =
92 2 10
+
16 16
h h  10
cos q = = fih= cos q ...............(4)
x Ê  10 ˆ 4
Á 4 ˜
Ë ¯
 10 3Ê v2 ˆ
\ cosq = Á  - ˜
4 4Ë 5g ¯

3 Ê v2 ˆ -
ÏÔ Ê ˆ ¸Ô - Ê ˆ
cosq = Á 1 - 5g ˜ q a Ì Á - ˝+
 ¯˜ Ô˛ ÁË ˜¯
10 Ë ¯ ÔÓ Ë
07 GRAVITATION

(a) 36% (b) 0.36% of its value on the surface of


Level 1 the earth. Radius of the earth R = 6400 km.
Q.1. Two lead balls of mass m and 2 m are placed at Q.5. An astronaut landed on a planet and found that
a separation d. A third ball of mass m is placed his weight at the pole of the planet was one third
at an unknown location on the line joining the of his weight at the pole of the earth. He also
first two balls such that the net gravitational force found himself to be weightless at the equator of
6 Gm 2 the planet. The planet is a homogeneous sphere of
experienced by the first ball is . What is radius half that of the earth. Find the duration of a
d2
day on the planet. Given density of the earth = d0.
the location of the third ball?
Q.6. A gravity meter can detect change in acceleration
Q.2. Four identical point masses m each are kept at the
due to gravity (g) of the order of 10–9 %. Calculate
vertices A, B, and C of a cube having side length
the smallest change in altitude near the surface of
‘a’ (see figure). Another identical mass is placed
the earth that results in a detectable change in g.
at the center point D of the cube.
Radius of the earth R = 6.4 × 106 m.
(a) Where will you place a fifth identical mass so
Q.7. The earth is a homogeneous sphere of mass M
that the net gravitational force acting on mass
and radius R. There is another spherical planet of
at D becomes zero?
mass M and radius R whose density changes with
(b) Calculate the net gravitational force acting on distance r from the centre as r = r0r.
the mass at D.
(a) Find the ratio of acceleration due to gravity
A
on the surface of the earth and that on the
B surface of the planet.
a
(b) Find r0.
D
Q.8. A planet having mass equal to that of the earth
(M = 6 × 1024 kg) has radius R such that a particle
C projected from its surface at the speed of light
Q.3. Two point masses m and M are held at rest at a large (c = 3 × 108 ms–1) just fails to escape.
distance from each other. When released, they Assuming Newton’s Law of gravitation to be
begin moving under their mutual gravitational valid calculate the radius and mass density of
pull. such a planet. Are the numbers realistic?
(a) Find their relative acceleration (a) when Note: The radius that you calculated is known
separation between them becomes x. as Schwarzschild radius. Actually we need to
(b) Integrate the expression of a obtained above use theory of general relativity for solving this
to calculate the relative velocity of the two problem.
masses when their separation is x. Q.9. Angular speed of rotation of the earth is w0. A
(c) Write the velocity of centre of mass of the train is running along the equator at a speed v from
system when separation between them is x. west to east. A very sensitive balance inside the
Q.4. Find the height above the surface of the earth train shows the weight of an object as W1. During
where the acceleration due to gravity reduces by the return journey when the train is running at
7.2 Problems in Physics for JEE Advanced

same speed from east to west the balance shows Q.15. It is known that if the length of the day were T0
the weight of the object to be W2. Weight of the hour, a man standing on the equator of the earth
object when the train is at rest was shown to be would have felt weightlessness. Assume that a
W0 by the balance. Calculate W2 – W1. person is located inside a deep hole at the equator
Q.10. If a planet rotates too fast, rocks from its surface at a distance of
R from the centre of the earth.
will start flying off its surface. If density of a 2
homogeneous planet is r and material is not flying What should be the time period of rotation of the
off its surface then show that its time period of earth for such a person to feel weightlessness?
[R = radius of the earth]
3p
rotation must be greater then .
Gr Q.16. A small satellite of mass m is going around a plan-
et in a circular orbit of radius r. Write the kinetic
Q.11. (a) The angular speed of rotation of the earth energy of the satellite if its angular momentum
is w = 7.27 × 10-5 rad s–1 and its radius is about the centre of the planet is J.
R = 6.37 × 106 m. Calculate the acceleration
Q.17. Suppose that the gravitational attraction between
of a man standing at a place at 40° latitude.
a star of mass M and a planet of mass m is given
[cos 40° = 0.77]
Mm
(b) If the earth suddenly stops rotating, the by the expression F K where K and n are
acceleration due to gravity on its surface will rn
become g0 = 9.82 ms–2. Find the effective constants. If the orbital speed of the planets were
value of acceleration due to gravity (g) at 40° found to be independent of their distance (r)
latitude taking into account the rotation of the from the star, calculate the time period (T0) of a
earth. planet going around the star in a circular orbit of
Ê 1ˆ radius r0.
Q.12. A planet has radius Á ˜ th of the radius of
Ë 36 ¯ Q.18. A near surface earth’s satellite is rotating in
the earth. The escape velocity on the surface of equatorial plane from west to east. The satellite is
1 exactly above a town at 6:00 A.M today. Exactly
the planet was found to be times the escape
6 how many times will it cross over the town by
velocity from the surface of the earth. The planet 6:00 A.M tomorrow. [Don’t count its appearance
is surrounded by a thin layer of atmosphere today at 6:00 A.M above the town].
having thickness h (<< radius of the planet). The Q.19. Imagine an astronaut inside a satellite going
average density of the atmosphere on the planet around the earth in a circular orbit at a speed
is d and acceleration due to gravity on the surface
gR
of the earth is ge. Find the value of atmospheric of where R is radius of the earth and g is
pressure on the surface of the planet. 2
acceleration due to gravity on the surface of the
Q.13. Using a telescope for several nights, you found a
earth.
celestial body at a distance of 2 × 1011 m from the
sun travelling at a speed of 60 km s–1. Knowing (a) What is weight experienced by the astronaut
that mass of the sun is 2 × 1030 kg, calculate after inside the satellite?
how many years you expect to see the body again (b) Assume that an alien demon stops the
at the same location. satellite and holds it at rest. What is weight
Q.14. A man can jump up to a height of h0 = 1 m on the experienced by the astronaut now?
surface of the earth. What should be the radius (c) The demon now releases the satellite (from
of a spherical planet so that the man makes a rest). What is weight experienced by the
jump on its surface and escapes out of its gravity? astronaut now?
Assume that the man jumps with same speed as
Q.20. The height of geostationary orbit above the
on earth and the density of planet is same as that
surface of the earth is h. Radius of the earth is R.
of earth. Take escape speed on the surface of the
The earth shrinks to half its present radius (mass
earth to be 11.2 km/s and radius of earth to be
remaining unchanged). Now what will be height
6400 km.
Gravitation 7.3

of a geostationary satellite above the surface of (b) Calculate the time period of the satellite if it
the earth? E
is given that 1 = 1 . Take mass of the earth
Q.21. (a) Estimate the average orbital speed of the earth E2
to be M.
going around the sun. The average Earth-sun
distance is 1.5 × 1011 m. Q.27. A satellite of mass m is going around the earth in
(b) An asteroid going around the sun has an a circular orbital at a height
R from the surface
average orbital speed of 15 km/s. Is the 2
asteroid farther from the sun or closer to the of the earth. The satellite has lived its life and
sun as compared to the Earth? Explain your a rocket, on board, is fired to make it leave the
answer. gravity of the earth. The rocket remains active
for a very small interval of time and imparts an
Q.22. Assume that the earth is not rotating about its
impulse in the direction of motion of the satellite.
axis and that Scientists have developed an engine
Neglect any change in mass due to firing of the
which can propel vehicles to very high speed on
rocket.
the surface of the earth. What is the maximum
possible speed for any such vehicle running on (a) Find the minimum impulse imparted by the
surface of the earth. Earth is a sphere of radius rocket to the satellite.
R = 6400 km and acceleration due to gravity on (b) Find the minimum work done by the rocket
the surface is g = 10 m/s2. engine. Mass of the earth = M, Radius of the
Q.23. A satellite of Earth is going around in an elliptical earth = R
orbit. The smallest distance of the satellite from Q.28. A small asteroid is at a large distance from a
the centre of the earth happens to be 2R (where planet and its velocity makes an angle f (! 0)
R = radius of the earth). Find the upper limit of with line joining the asteroid to the centre of the
the maximum speed of such a satellite. planet. Prove that such an asteroid can never fall
Q.24. Haley’s Comet is going around the Sun in a highly normally on the surface of the planet.
elliptical orbit with a period of 76 y. It was closest
to the sun in the year 1987 (I was 13 year old then

and heard a lot about it on radio). In which year of
Planet
21st century do you expect it to have least kinetic
energy?
Q.25. A planet goes around the sun in an elliptical orbit.
The minimum distance of the planet from the Sun Level 2
is 2 × 1012 m and the maximum speed of the planet
in its path is 40 km s–1. Find the rate at which its Q.29. Three identical particles, each of mass m, are
position vector relative to the sun sweeps area, located in space at the vertices of an equilateral
when the planet is at a distance 2.2 × 1012 m from triangle of side length a. They are revolving in
the sun. a circular orbital under mutual gravitational
attraction.
Q.26. To launch a satellite at a height h above the
surface of the earth (radius R) a two stage rocket (a) Find the speed of each particle.
is used . The first stage is used to lift the satellite (b) Find the acceleration of the centre of mass of
to the desired height and the second stage is used a system comprising of any two particles.
to impart it a tangential velocity so as to put it
(c) Assume that one of the particles suddenly
in a circular orbital. Assume (incorrectly) that the
loses its ability to exert gravitational force.
mass of rocket is negligible and that there is no
Find the velocity of the centre of mass of the
atmospheric resistance.
system of other two particles after this.
(a) If E1 and E2 are the energies delivered by the
Q.30. Imagine a hole drilled along the radius of the
first and the second stage of the rocket.
earth. A uniform rod of length equal to the radius
E1 (R) of the earth is inserted into this hole. Find the
Calculate the ratio .
E2 distance of centre of gravity of the rod from the
7.4 Problems in Physics for JEE Advanced

centre . (a) Calculate work done by the friction on the


block.
(b) Calculate m.
R Q.35. Diameter of a planet is 10d°, its mean density is
r and mass of its atmosphere is 10m where d ,
° °
4
r° and m° are diameter, mean density and mass

of atmosphere respectively for the earth. Assume
that mean density of atmosphere is same on the
Q.31. A large non rotating star of mass M and planet and the earth and height of atmosphere on
radius R begins to collapse under its own both the planets is very small compared to their
gravity and ultimately becomes very small radius.
(nearly a point mass). Assume that the density
(a) Find the ratio of atmospheric pressure on the
remains uniform inside the sphere in any
surface of the planet to that on the earth.
stage. Plot the variation of gravitational field
intensity (well, you can call it acceleration (b) If a mercury barometer reads 76 cm on the
R surface of the earth, find its reading on the
due to gravity) at a distance from the centre vs surface of the planet.
2
the radius (r) of the star. Q.36. A particle of mass m is projected upwards from
R the surface of the earth with a velocity equal to
Q.32. At a depth h1 = from the surface of the earth half the escape velocity. (R is radius of earth and
2
acceleration due to gravity is g1. It’s value M is mass of earth)
changes by Dg1 when one moves down further (a) Calculate the potential energy of the particle
by 1 km. At a height h2 above the surface of the at its maximum.
earth acceleration due to gravity is g2. It’s value (b) Write the kinetic energy of the particle when
changes by Dg2 when one moves up further by it was at half the maximum height.
1 km. If Dg1 = Dg2 find h2. Assume the earth to be Q.37. A uniform spherical planet is rotating about its
a uniform sphere of radius R. axis. The speed of a point on its equator is v
Q.33. Due to rotation of the earth the direction of and the effective acceleration due to gravity on
vertical at a place is not along the radius of the the equator is one third its value at the poles.
earth and actually makes a small angle f with the Calculate the escape velocity for a particle at the
true vertical (i.e. with radius). At what latitude (q) pole of the planet. Give your answer in multiple
is this angle f maximum ? of v.

Q.34. A tunnel is dug along a chord of non rotating earth at Q.38. A planet is a homogeneous ball of radius R having
R mass M. It is surrounded by a dense atmosphere
a distance d = [R = radius of the earth] from σ
2 having density ρ = where s° is a constant
its centre. A small block is released in the tunnel
from the surface of the earth. The block comes to and r is distance from the centre of the planet. It is
rest at the centre (C) of the tunnel. Assume that found that acceleration due to gravity is constant
the friction coefficient between the block and the throughout the atmosphere of the planet. Find s°
tunnel wall remains constant at m. in terms of M and R.
Q.39. A projectile is to be launched from the surface of
the earth so as to escape the solar system. Consider
C the gravitational force on the projectile due to the
earth and the sun only. The projectile is projected
d
O
perpendicular to the radius vector of the earth
relative to the centre of the sun in the direction
of motion of the earth. Find the minimum speed
Gravitation 7.5

of projection relative to the earth so that the the earth (R). Acceleration due to gravity on the
projectile escapes out of the solar system. Neglect surface of the earth is g.
rotation of the earth. Q.43. An astronaut on the surface of the moon throws a
Mass of the sun Ms = 2 × 1030 kg; Mass of the piece of lunar rock (mass m) directly towards the
earth Me = 6 × 1024 kg earth at a great speed such that the rock reaches
Radius of the earth Re = 6.4 × 106 m; Earth-Sun the earth.
M
distance r = 1.5 × 1011 m Mass of the earth = M, Mass of the moon =
81
Q.40. Assume that there is a tunnel in the shape of a Radius of the earth = R, Distance between the
circular arc through the earth. Wall of the tunnel centre of the earth and the moon = 60R
is smooth. A ball of mass m is projected into the
tunnel at A with speed v. The ball comes out of the (a) In the course of its journey calculate the
tunnel at B and escapes out of the gravity of the maximum gravitational potential energy of
earth. Mass and radius of the earth are M and R the rock
respectively and radius of the circle shaped tunnel (b) Find the minimum possible speed of the rock
is also R. when it enters the atmosphere of the earth.
(a) Find minimum possible value of v (call it v0) Q.44. The radius of the circular path of a geostationary
(b) If the ball is projected into the tunnel with satellite was inadvertently made Dr = 1 km larger
speed v0, calculate the normal force applied than the correct radius r = 42000 km
by the tunnel wall on the ball when it is (a) Calculate the difference in angular speed of
closest to the centre of the earth. It is given the satellite and the earth.
that the closest distance between the ball and (b) If the satellite was exactly above a house on
R the equator on a particular day, what will be
the centre of the earth is .
2 angular separation between the house and the
A satellite a year later?
V
Q.45. A spy satellite S1, travelling above the equator
is taking pictures at quick intervals. The satellite
O R/2 is travelling from west to east and is ready with
picture around the whole equator in 8 hours.
Another similar satellite S2, travelling in the same
B plane is travelling from east to west and is able to
take pictures around the whole equator in 6 hours.
Q.41. A celestial body, not bound to sun, will only
Find the ratio of radii of the circular paths of the
pass by the sun once. Calculate the minimum
satellite S1 and S2.
speed of such a body when it is at a distance of
1.5 × 1011 m from the sun (this is average distance Q.46. A comet is going around the sun in an elliptical
between the sun & the earth and is known as orbit with a period of 64 year. The closest
astronomical unit- A.U.) approach of the comet to the sun is 0.8 AU [AU =
astronomical unit]. Calculate the greatest distance
The mass of the sun is M ~ 2 × 1030 kg.
of the comet form the sun.
N - m2 Q.47. The astronomical phenomenon when the planet
G = 6.67 ¥ 10 -11
kg 2 Venus passes directly between the Sun and the
earth is known as Venus transit. For two separate
Show that this speed is 2 times greater than persons standing on the earth at points M and N,
speed of earth around the sun, assuming circular the Venus appears as black dots at points M’ and
trajectory N’ on the Sun. The orbital period of Venus is close
Q.42. A body is projected vertically upward from the to 220 days. Assuming that both earth and Venus
surface of the earth with escape velocity. Calculate revolve on circular paths and taking distance
the time in which it will be at a height (measured MN = 1000 km, calculate the distance M’N’ on
from the surface of the earth) 8 time the radius of the surface of the Sun. [Take (2.75)1/3 = 1.4]
7.6 Problems in Physics for JEE Advanced

A small asteroid is approaching a planet of mass


Sun
M and radius R from a large distance. Initially its
M velocity (u) is along a tangent to the surface of the
Earth V planet. It fall on the surface making an angle of
N N' 30° with the vertical. Calculate u.
Venus

M' Q.52. An asteroid was fast approaching the earth.


Scientists fired a rocket which hit the asteroid
Q.48. Satellite A is following a circular path of radius at a distance of 5 R from the centre of the
a around the earth another satellite B follows an earth (R = radius of the earth). Immediately
elliptical path around the earth. The two satellites after the hit the asteroid’s velocity (V0)
have same mechanical energy and their orbits was making an angle of q  =  30° with the line joining
intersect. Find the speed of satellite B at the point the centre of the earth to the asteroid. The asteroid
where its path intersects with the circular orbit of just grazed past the surface of the earth. Find V0
A. Take mass of earth to be M. [Mass of the earth = M]
A
V0
B Earth

5R

Q.53. A satellite is orbiting around the earth in a


circular orbit. Its orbital speed is V0. A rocket on
Q.49. A satellite of mass m is orbiting around the earth board is fired from the satellite which imparts a
(mass M, radius R) in a circular orbital of radius thrust to the satellite directed radially away from
4R. It starts losing energy slowly at a constant the centre of the earth. The duration of the engine
dE burn is negligible so that it can be considered
rate - = h due to friction. Find the time (t) in
dt instantaneous. Due to this thrust a velocity
which the satellite will spiral down to the surface variation DV is imparted to the satellite. Find the
of the earth. DV
minimum value of the ratio for which the
Q.50. Energy of a satellite going around the earth in an V0
GMm satellite will escape out of the gravitational field
elliptical orbit is given by - where M and of the earth.
2a
m are masses of the earth and the satellite
Q.54. In last question assume that circular orbit of the
respectively and 2a is the major axis of the
elliptical path. A satellite is launched tangentially satellite has radius r0. Find for which the
3GM
with a speed = from a height h = R above maximum distance of the satellite from the centre
5R
of the earth become 2r0 after the rocket is fired.
the surface of the earth. Calculate its maximum
distance from the centre of the earth Q.55. A satellite is at a distance r1 from the centre of the
earth at its apogee. The distance is r2 when it is at
Q.51.
perigee. Mass of the earth is M.
(a) Calculate the maximum speed of the satellite
in its orbit around the earth.
R
(b) Estimate the maximum speed of the moon
Planet
going around the earth. For moon
r1 ~ 400,000 km and r2 ~ 360,000 km

u
mass of the earth M = 6 × 1024 kg
Gravitation 7.7

Q.56. A satellite is going around the earth in an (b) Consider a large flat horizontal sheet of
elliptical orbit and has maximum and minimum material density r and thickness t, placed on
distance from the centre of earth equal to 10r and the surface of the earth. The density of the
r respectively. It was planned to fire on board earth is r0. If it is found that gravitational
rocket so as to increase the energy of the satellite field intensity just between the sheet is larger
by maximum amount. Assume that the rocket 3
is fired for a small time (almost instantaneous) than field just above it, prove that ro > r .
2
and gives an impulse J to the satellite in forward Assume t << R
direction. Take J to be small compared to overall Q.61. A spaceship is orbiting the earth in a circular orbit
momentum of the satellite. at a height equal to radius of the earth (Rc = 6400
(a) Show that firing the rocket when the satellite km) from the surface of the earth. An astronaut
is at perigee (nearest to earth) will result in is on a space walk outside the spaceship. He is
maximum gain in energy of the satellite. at a distance of l = 200 m from the ship and is
[The orbit of mass orbiter mission, informally connected to it with a simple cable which can
called Mangalyaan, was raised in five steps sustain a maximum tension of10 N. Assume that
using this principle; before it was given the the centre of the earth, the spaceship and the
escape speed] astronaut are in a line. Mass of astronaut along
with all his accessories is 100 kg.
(b) Find the impulse J that the rocket must impart
(a) Do you think that a weak cable that can only
to the satellite at perigee so that its maximum
take a load of 10 N, can prevent him from
distance from earth’s centre, during its course
drifting in space ? Make a guess.
of motion in elliptical path, becomes 12r. Take
mass of satellite as m and mass of earth as M. (b) Estimate the tension in the cable.
Assume that there is negligible change in mass [Acceleration due to gravity on the surface of
of the satellite due to firing of the rocket. Earth = 9.8 m/s2]
Q.58. Imagine a smooth tunnel along a chord of non- Q.62. Earth is rotating about its axis with angular
R speed w0 and average density of earth is r. It is
rotating earth at a distance from the centre. proposed to make a space elevator by placing a
2
R is the radius of the earth. A projectile is fired long rod with uniform mass density extending
along the tunnel from the centre of the tunnel at a from just above the surface for the earth out to
a radius nR (R is radius of the earth). Prove that
speed Vo gR [g is acceleration due to gravity
the rod can remain above the same point on the
at the surface of the earth]. 8p G r
equator all time if, n2 + n = , where r is
(a) Is the angular momentum [about the centre 3w 2
of the earth] of the projectile conserved as it density of the earth
moves along the tunnel?
(b) Calculate the maximum distance of the Q.63. A body is projected up from the surface of the
projectile from the centre of the earth during earth with a velocity half the escape velocity at an
its course of motion. angle of 30° with the horizontal. Neglecting air
Q.59. A geostationary satellite is nearly at a height of resistance and earth’s rotation, find
h = 6 R from the surface of the earth where R is (a) the maximum height above the earth’s surface
the radius of the earth. Calculate the area on the to which the body will rise.
surface of the earth in which the communication (b) will the body move around the earth as a
can be made using this satellite. satellite?
Q.64. A near surface earth satellite has cylindrical shape
Level 3 with cross sectional area of S = 0.5 m2 and mass
of M = 10 kg. It encounters dust which has density
Q.60. (a) There is an infinite thin flat sheet with mass of d = 1.6 × 10–11 kg/m3. Assume that the dust
density s per unit area. Find the gravitational particles are at rest and they stick to the satellite’s
force, due to sheet, on a point mass m located front face on collision. Take mean density of earth
at a distance x from the sheet. to be r = 5500 kg/m3
7.8 Problems in Physics for JEE Advanced

(a) Find the drag force experienced by the (b) If the dust extends throughout the orbit,
satellite find the change in velocity and radius of
the circular path of the satellite in one
revolution.

ANSWERS

1. Exactly midway between the two balls OR at a 14. 2.5 km


d 15. T0
distance of from the ball of mass m.
2 2
16. J2
2. (a) At the diagonally opposite corner of C on the
floor of the cube 2 mr 2

4 Gm 2 2p r0
(b) 17. T0 =
3a 2 KM
G( M + m) 18. 16
3. (a) 2 19. (a) Zero
x
mg
2G( M + m) (b)
(b) 4
x (c) zero
(c) zero R
20. h +
4. (a) 1600 km 2
(b) 11.52 km 21. (a) 30 km/s
(b) Farther
9p
5. T= 22. V0 = gR  7.9 km /s
2Gd0
6. 32 mm 23. gR  7.9 km /s
7. (a) 1 24. 2025
M 25. 4 × 1016 m2s–1
(b) 4
pR 2h
26. (a)
8. R = 8.9 mm; d = 2 × 1030 kg m –3 R

4W0w 0 v 27p 2 R3
9. (b)
g 2GM
11. (a) 0.026 ms–2
(b) 9.80 ms–2
27. (a) ( 2 -1 m ) 2GM
3R
12. 6 dgh GMm
(b)
13. The body will never return to the same location. 3R
Gravitation 7.9

Gm GMm
29. (a) 43. (a) -
a 243R
3 Gm 14045 GMm
(b) (b)
2 a2 14337 R
1 Gm 44. (a) 9.3 × 10-6 rad/hr
(c) (b) 4.6°
2 a
2 /3
r1 Ê 3 ˆ
2R 45. =
30. r2 ÁË 4 ˜¯
3
31. g 46. 31.2 AU
47. 2500 km
4GM
R2 GM
48.
a
GM
2R2
r 3GMm
O R R 49. t =
2 8h R

Ê 1 ˆ 50. 2R
32. h2 = R Á 2 3 - 1˜
Ë ¯ 2GM
51. u =
3R
33. 45°
3GMm 32 GM
34. (a) - 52. V0 =
8R 105 R
3 53. (1)
(b)
2 54. (1/2)
5 Ê ˆ
35. (a)
2 2GM Á 1 ˜
55. (a) 1+
(b) 7.6 cm r2 ÁÁ r2 ˜˜
Ë r1 ¯
-3GMm
36. (a) (b) 1.08 km/s
4R
3GMm GM
(b) 56. (b) J = 0.01 m
28 R r
37. 3v 58. (a) NO

M Ê 8 + 57 ˆ
38. s  = rmax = Á
(b) ˜R
2p R 2 Ë 14 ¯
39. 13.6 km/s 12 p R2
59.
2GM 7
40. (a) v0 =
R 60. (a) G2psm
27 GMm 61. (b) 0.01 N
(b)
4 R2 Ê 7 - 2ˆ
63. (a) Á ˜R
41. 4.2 × 104 m/s Ë 6 ¯
52 R (b) No
42. t =
3 2g 64. (a) 5 × 10 –4 N
(b) V = 0.25 m/s; R = –0.4 km
7.10 Problems in Physics for JEE Advanced

SOLUTIONS

1. There are two possibilities –


(1) the net force is towards right
(2) the net force is towards left (see figure)
When net force is towards right–
Gm (2 m) G mm 6 Gm 2
+ =
d2 x2 d2
1 4 d
2
= 2
x=
x d 2

m 2m

m
x Third ball

When the net force is to the left


m m 2m

1st ball
x d

G mm G m (2m) 6 G m 2
- =
x2 d2 d2
1 8
=
x2 d
d
x=
2 2
2. (a) The net force on mass at D will be zero if an identical mass is placed at E. In this case the resultant force due
to masses at A and B will be vertically upward and that due to masses at C and E will be vertically down.
A

D
E
C

(b) Required force = Force equal and opposite to that applied by the mass placed at E
Gm 2 4Gm 2
= =
( 3a / 2)2 3a 2

GmM
3. (a) Force between the two masses F =
x2
Gravitation 7.11
a1 a2
x
m F F M

F GM
Acceleration of m, a1 = = 2
m x
F Gm
Acceleration of M, a2 = = 2
M x
G ( M + m)
Relative acceleration a = a1 + a2 =
x2
dv dv dx dv
(b) a= = ◊ =v [v = relative velocity]
dt dx dt dx
dv G( M + m)
- v =
dx x2
[- sign has been placed because v is increasing with decreasing x]
dx
v dv = -G ( M + m)
x2
v x
dx
Ú v dv = -G( M + m)Ú x2
0 •

x
v2 È 1˘ G( M + m)
= -G( M + m) Í- ˙ =
2 Î x ˚• x
2G( M + m)
v=
x
(c) The centre of mass has no acceleration as there is no external force.
Hence, velocity of COM is zero throughout.
g
4. (a)
g´ = 2
Ê hˆ
ÁË 1 + ˜¯
R
g´ g – 0.36g = 0.64g
1
0.64 = 2
Ê hˆ
ÁË 1 + ˜¯
R
h 1
1+ =
R 0.8
h 5
= -1
R 4
R
h= = 1600 km
4
-
Ê ˆ
(b)
g´= = Á + ˜¯
Ê ˆ Ë
ÁË + ˜¯
7.12 Problems in Physics for JEE Advanced

È ˘
 Í - ˙ [expanding binomially and neglecting higher order terms]
Î ˚
g¢ 2h
=1-
g R
2h g - g ¢
=
R g
2h Dg
¥ 100 = ¥ 100
R g
2h
¥ 100 = 0.36
R
0.18
h= ¥ 6400 km = 11.52 km
100

5.
1
3
(
g pole ) earth
(
= g pole ) planet

1 G Me G M P
=
3 Re2 RP2

4 4
p R 3e ◊ d0 p R 3P ◊ dP
1 3 3
. =
3 Re2 RP2
d0 Re = 3 dP RP ….. (1)
For weightlessness at the equator
w2R = g

GM P
w 2 RP =
RP2

4 4 1 d0 Re
w2 = G p dP = Gp
3 3 3 RP
4 È Re ˘
= Gp d0 (2) Í = 2˙
9 R
Î P ˚

4p 2 8
2
= Gp d0
T 9
9p
T2 =
2Gd0

9p
T=
2Gd0

GM
6. g=
R2
dg 2GM
=- 3
dR R
Gravitation 7.13

dg 2g
=-
dR R
Dg 2 DR
-
g R

DR 1 Dg 1 10 -9
= = ¥
R 2 g 2 100
1
DR = R ◊ ¥ 10 -11
2
1
= 6.4 ¥ 106 ¥ ¥ 10 -11
2
= 3.2 ¥ 10 -5 = 32 mm

GM GM
7. (a) g1 = 2
; g2 =
R R2
g1
=1
g2
R

Ú r 4p r dr = M
2
(b)
0

R
fi 4pr0 Ú r 3 dr = M fi pr0 R 4 = M
0

M
r0 =
p R4
1 GMm
8. mc 2 - =0
2 R
2GM 2 ¥ 6.67 ¥ 10 -11 ¥ 6 ¥ 1024
R= 2 =
c (3 ¥ 108 )2
-3
= 8.89 × 10 m  =  8.9 mm (!!)
6 ¥ 1024
Mass density = = 2.0 × 1030 kg m–3
4 -3 3
¥ 3.14 ¥ (8.9 ¥ 10 )
3
The figures are unrealistic! A body having mass of the earth cannot act like a black hole.

When the train is at rest


mV02
W0 = mg - [V0 = w0R, w0 = angular speed of the earth]
R
When the train is moving from West to East V1 = V0 + v
m (V0 + v)2
W1 = mg -
R
For train running due west
V2 = V0 – v
7.14 Problems in Physics for JEE Advanced

m (V0 - v)2
W2 = mg -

R

W2 - W1 = (V0 + v)2 - (V0 - v)2 ˘˚

m 4m (w 0 R) v
= ÈÎ4V0 v ˘˚ = = 4mw0v
R R
W
= 4 0 w0 v
g
The rocks start flying away from the equator of the planet if
g
w2R > g w≥
R
2p g T R
≥ £
T R 2p g
R
T £ 2p
g
For rocks to not fly away
R R
T ≥ 2p
= 2p
g GM
R2

R3 3p
= 2p =
4 Gr
G ◊ p R3 ◊ r w
3
11. (a)  Radius of rotation at latitude q = 40° is r
a
r = R cos q
R
Centripetal acceleration q

a = w2r = w2R cos q


= (7.27 × 10–5)2 (6.37 × 106) (0.77)
= 0.026 ms–2
 
(b)  The resultant acceleration will be the vector sum of g0 and - a
w

–a

g0

2 2
g = g0 + a - 2ag cosq
Gravitation 7.15

= (9.82)2 + (0.026)2 - 2 ¥ 0.026 ¥ 9.82 ¥ 0.77


= 9.80 ms –2
12. Vescape = 2 gR

Vescape planet gP RP
=
Vescape earth ge Re

1 gP 1
=
6 ge 36
gp = 6ge
Atmospheric pressure = dgph
         = 6dgeh
Energy of the body
1 GMm
mV 2 -
2 r

(
È 60 ¥ 103
) ˘
2
Í 6.67 ¥ 10 -11 ¥ 2 ¥ 1030 ˙
= mÍ - ˙
2 2 ¥ 1011
ÍÎ ˙˚
= m[18 × 108 – 6.67 × 108]
= m(11.33 × 108)
Since total energy is positive, the body is in an unbound orbit. It will never return back.

For a jump of h0 = 1 m on the earth, speed required is given by


1
mV 2 = mgh0 V  20 m /s
2
Escape speed on the surface of a planet is

2GM 8p GR 2 r
Vesc =
=
R 3
planet R planet
Vesc
earth
=
Vesc Rearth
planet
We want Vesc = 20 m /s
earth
And it is given that Vesc = 11.2 km /s

20 ¥ (6400 km)
R planet = = 2.5 km
11200

15. For weightlessness at the equator


g
w 02 R = g w0 =
R
At a distance r from the centre (r < R) the acceleration due to gravity is
g
gr = r
R
7.16 Problems in Physics for JEE Advanced

For weightlessnes at a distance r from centre


g g
r = w 2r w=
R R
Required rotation speed of the earth is same for any depth.
J2 J2
16. K = =
2 I 2mr 2
[I = mr2 = moment of inertia of the satellite about the planet]

mV 2 Mm
17. =K n
r r
V is independent of r if n = 1
V2 = KM
V = KM
2p r0 2p r0
Time period T0 = =
V KM
R
18. Time period of a near surface satellite T = 2p  86 min = 1.43 hr
g
Number of revolutions in 24 hr
24
n= = 16.78
1.43
number of crossings = 16
19. (a) A person inside a satellite is in a state of weightlessness. The gravitational pull of earth provides the exact
centripetal force needed to keep moving with the satellite. There is no contact force between the walls of the
satellite and the person.
GM
(b) Orbital speed v =
r
gR GM
=
2 r

1 GM GM
2 R =
2 R r
Weight experienced at a distance r from the centre of the earth is
GMm GMm mg
2 = =
r (2 R ) 2 4
(c) Once again the man becomes weightless as he starts a free fall motion under gravity (along with the satellite).
20. The orbital radius of the satellite will not change because force acting on it does not change.
R
Since radius of earth has become R/2, distance of the satellite from the surface = h +
2
21. (a) Average speed of the earth is
2p r 2 ¥ 3.14 ¥ 1.5 ¥ 1011
v= = = 2.98 ¥ 10 4 m /s ~ 30 km/s
T 365 ¥ 24 ¥ 60 ¥ 60
(b) Speed of bodies orbiting the sun decreases with distance from the sun. Orbital speed for circular orbit is given
by
Gravitation 7.17

GM s
v =
r
Since speed of the asteroid is less than that of the earth, it is farther from the sun.
mV02 GMm
22. =
R R2
If the speed is increased beyond V0, the vehicle will leave the surface of the earth.

GM
V0 = = gR  7.9 km /s
R
23. Speed is maximum when the satellite is closest to the Earth.
For a bound orbit, total energy must be negative.
1 GMm
2
mVmax - <0
2 2R
GM
Vmax < = gR
R
24. Speed will be least when it is farthest from the sun. This will happen when it will be at aphelion.
76
t = = 38 y
2
1987 + 38 = 2025
25. The speed is maximum when the planet is at minimum distance from the Sun.
V = 40 km s–1 when r = 2 × 1012 m
V
w= [This equation is true at perigee and apogee]
r
40 × 103
ω= 12
= 2 × 10 −8 rad s−1
2 × 10
dA r 2w
Rate of sweeping of area =
dt 2
(2 × 1012 )2 × 2 × 10 −8
= = 4 × 1016 m 2 s−1
2
This rate remains constant for any position of the planet.
GMm Ê GMm ˆ
26. (a) E1 = - - -
( R + h ) ÁË R ˜¯

È1 1 ˘ GMmh
= GMm Í - ˙=
Î R R + h ˚ R( R + h )
GM GM
Orbital speed of satellite V = =
r R+h
1 GMm
\ E2 = mV 2 =
2 2( R + h )
E1 2h
\ =
E2 R
7.18 Problems in Physics for JEE Advanced

E1 2h
Now, = 1 means =1
E2 R

R
\h =
2
3R
Distance from the centre of earth r =
2
3
4p 2 3 4p 2 Ê 3R ˆ
T =
2
r = Á ˜
GM GM Ë 2 ¯

27π 2 R 3
T =
2GM
GM 2GM 2 gR
27. (a) Orbital speed V0 = = =
r 3R 3
1 GMm
For escape the satellite must have speed given by mV 2 - =0
2 r
2GM
V = = 2 V0
r
Impulse needed = m ( )
2 V0 - mV0

= ( )
2 - 1 mV0 = ( 2 -1 m) 2GM
3R
1
( ) 1 1 GMm
2
(b) W = m 2V0 - mV02 = mV02 =
2 2 2 3R
28. The angular momentum of the asteroid about the centre of the planet is not zero. Falling normally on the surface
will mean that its angular momentum about the centre is zero. But the angular momentum is conserved, hence it
cannot hit the surface of the planet normally.
a
29. (a) Consider one of the particle (say B). It is rotating in circle of radius r =
Resultant force on it towards the centre of the circle is 3

Gm 2 3 Gm 2
2 F cos 30∞ = 2 2
= 3 2
a 2 a
This must be equal to necessary centripetal force
A m

a a
F r
O
30° C
B a
m F m

mv 2 3Gm 2
\ =
r a2
Gm È a ˘
fi v 2 = ÍQ r = ˙
a Î 3˚
Gravitation 7.19

Gm
fi v =
a
(b) Consider our system to be made up of B and C. External force on this system is due to A. Net external force =
2F sin 60°
2
= 3F = 3Gm
a2 A

F
F
60° 60°
B C

Ê 3Gm 2 ˆ 1
\ acm = Á 2 ˜ ¥
Ë a ¯ 2m

= 3 Gm (towards the centre of the triangle)


2 a2
(c) Momentum of (B + C) system = mv cos 60° + mv cos 60° = mv [along CB]

v O

30°
B C
v

mv v 1 Gm
\ vcm = = =
2m 2 2 a
The velocity of CoM does not change after A stops attracting.

Ú xdw
30. xcg =
Ú dw

Where dw = Weight of an infinitesimal element whose x co-ordinate is x.


Let l = mass per unit length of the rod.
Consider an element at a distance x from the centre
Ê xˆ
dw = ( l dx ) Á g ˜
Ë R¯

dx

x
[Acceleration due to gravity at a distance x from the centre of the earth is = g ]
R
7.20 Problems in Physics for JEE Advanced

lg R 2 R3
Ú0 x dx
2R
\ xcg = R = 32 =
lg R R 3
R Ú0
xdx
2
R
31. As long as the point is inside the sphere (which means r > ) the field is given by
2
GM GM Ê R ˆ
g= x= Á ˜
r 3
r3 Ë 2 ¯
1
\gµ
r3
Gm
In the beginning r = R \g=
2 R2
R 4Gm
When r = ; g= 2
2 R
R
After the star shrinks to a radius r < , the field at the point (the point is now outside the sphere)
2
GM 4GM
becomes constant at g = 2
=
Ê Rˆ R2
ÁË 2 ˜¯ g

4GM
R2

GM
2R2
r
O R R
2

32. Acceleration inside the earth at a distance r from the centre


GM GM Ê R - h1 ˆ Ê h ˆ
g1 = .r = 2 Á ˜ = g Á1 - 1 ˜
R 3
R Ë R ¯ Ë R¯

Ê dg ˆ g
Dg1 = Á 1 ˜ Dh1 = - Dh1
Ë dh1 ¯ R
Dh1 = +1km
g
\ Dg1 = - (1 km) ........(i)
R
[This is independent of depth h1]
At a distance r from the centre outside the earth
GM GM R 2 R2
g2 = = =g
r2 R2 r 2 ( R + h2 )2
Ê dg ˆ -2 gR 2
\ D g2 = Á 2 ˜ ( Dh2 ) = Dh2
Ë dh2 ¯ ( R + h2 )3
Dh2 = 1km
Gravitation 7.21

2 gR 2
Dg2 = - (1 km) ..........(i)
( R + h2 )3
Since Dg1 = Dg2
g 2 gR 2
\- (1 km) = - (1 km)
R ( R + h2 )3
(R + h2)3 = 2R3
h2
fi1+ = (2)1/ 3
R
h2 = R [21/3 – 1]
33. The figure shows the true weight (mg) of a body, centrifugal force (Fc) acting on it and the resultant of these two
forces (mgeff).
Fc
 Fc
r


mg mg eff

Fc = m w2r = m w2R cos q



Since Fc is very small f will be maximum when component of Fc perpendicular to mg is largest, i.e., when
2
mw R cosq sinq is maximum. This will happen when q = 45°
GM
34. (a) Gravitational potential at A, VA = -
R
Potential at C

N
A C
f F
 R/2

GM È 3 2 1 Ê R ˆ ˘
2
11 GM
Vc = - Í R - ÁË ˜¯ ˙ = -
3 2 2 2 ˙˚ 8 R
R ÍÎ
Loss in gravitational PE of the block
GMm Ê 11GMm ˆ 3GMm
= - - Á- ˜ =
R Ë 8R ¯ 8R

3GMm
Work done by friction = -
8R
(b) At any intermediate position (q) shown in the figure
F cos q = N
GMm Ê R / 2 ˆ
Á ˜ .cos q = N
R3 Ë cos q ¯
7.22 Problems in Physics for JEE Advanced

GMm
\N = = a constant
2 R2
Work done by friction = – mN(AC)
3GMm GMm Ê 3R ˆ
- = -m Á ˜
8R 2 R2 Ë 2 ¯

3
fi m =
2

35. Ro = radius of earth


10Ro =radius of the planet [so = density of atmosphere; ho = height of atmosphere]
4
m = p ÈÍ( R + h ) - R3 ˘˙ s 
3

3 Î ˚

È h ˆ
3 ˘
4 3 Ê
= p R ÍÁ 1 + ˜ - R3 ˙ s 
3 ÍÎË R¯ ˙˚

4 3 È 3h ˘
 R Í1 + - 1˙ s  = 4p R02s 0 h0 ......(a)
3 Î R ˚
For planet m = 4pR2.sh = 4p(10Ro)2 soh ......(b)
Given m = 10 mo
fi 100 h = 10 ho
h
h=
10
(a) Ratio of atmospheric pressure

P s gh g 1 ...........(iii)
=  = .
P s  g h g 10
4
G p R 3 .r
GM 4
\g= = 3 2 = p GR r
R2 R 3
g Rr 1 5
\ = = 10 ¥ =
g R r 4 2
Using (iii)
r 5 1
= =
r 2 ¥ 10 4
rhggo × (76 cm) = Po
(b)
rhgg × h =P

h g P
fi ¥ =
76 g P
h 5 1
= ¥ =
76 2 4
38
fi h = = 7.6 cm
5
Gravitation 7.23

36. (a) At maximum height speed is zero. Conservation of mechanical energy gives -
2
-GMm 1 Ê ve ˆ
+ mÁ ˜ = U + 0
R 2 Ë 2¯

-GMm 1 Ê GM ˆ
+ mÁ =U
R 2 Ë 2 R ˜¯
-GMm GMm -3GMm
U= + =
R 4R 4R
(b) Looking at the expression of U obtained above it is easy to conclude that the particle is at a distance of 4R/3
from the center of the earth when it is at its maximum height. It means that the particle is at a distance R/3 from
the surface of the earth. At half the maximum height the particle is at a distance R/6 from the surface, i.e., at a
distance 7R/6 from the center of the earth.
Kinetic energy at this height = Total energy – Potential energy
-3GMm -6GMm 3GMm
= - =
4R 7R 28 R
37. Acceleration due to gravity at the pole is
GM
g= , since there is no effect of rotation at the pole.
R2
Acceleration due to gravity at the equator is
v2
ge = g - w 2 R = g -
R
g v2
=g-
3 R
2 g v2
= ------- (1)
3 R
1 GMm
A body at the pole has total energy - E = mV 2 –
It escapes when this energy is zero. 2 R
1 GMm
mVe2 – =0
2 R
2GM
Ve = = 2 gR = 3v 2 = 3v
R
38. Let’s first calculate the mass of the atmosphere between R < x < r
r
m = Ú r 4p x 2 dx
R

x2r È s ˘
= 4ps  Ú dx ÍQ r = x ˙
R x Î ˚
= 2pso [r2 – R2]
dx
x R
r
7.24 Problems in Physics for JEE Advanced

Acceleration due to gravity at a distance r from the centre is


G( M + m)
g=
r2
Since, the whole mass distribution (planet + atmosphere is spherically symmetric)
GM G 2ps 
\g= + Èr 2 - R 2 ˘
r 2
r 2 Î ˚

GM G 2ps ∞ R 2
= - + G.2ps ∞
r2 r2
This expression is independent of r if
M
GM = G.2ps ∞ R 2 fi s ∞ =
2p R 2

GM
\gconst = G.2ps ∞ =
R2
GM
Alter: Acceleration due to gravity on the surface g = 2
R
Dg DM 2 DR
= -
g M R
But Dg = 0
DM 2 DR
\ =
M R
s∞
4p R 2 . DR
R = 2 DR
M R
M
fi s∞ =
2p R 2
39. The projectile will escape out if
1 2 GM s m GM e m
mv = +
2 r Re

ÊM M ˆ
v = 2G Á e + s ˜
Ë Re r ¯

È 6 ¥ 1024 2 ¥ 1030 ˘
v = 2 ¥ 6.67 ¥ 10 -11 Í 6
+ ˙
ÍÎ 6.4 ¥ 10 1.5 ¥ 1011 ˙˚
= 43.6 × 103 ms–1 = 43.6 km/s

SUN
Earth
r
Gravitation 7.25

2p r 2 ¥ 3.14 ¥ 1.5 ¥ 1011 m


Speed of the earth orbiting around the sun is vE = = ~ 30 km/s
(365d ) 365 ¥ 24 ¥ 60 ¥ 60 s
\ Projection speed relative to the earth = 43.6 – 30 = 13.6 km/s
40. (a) From energy conservation, it is easy to see that
2GM
v0 = vescape =
R
R
(b) Gravitational potential at a distance r =
2

GM È 3 2 1 2 ˘ GM È 3 2 R 2 ˘ 11 GM
V = - R - r = - Í R - ˙=-
3 Í2 2 ˚˙ 3
R Î R ÍÎ 2 8 ˙˚ 8 R
R
Speed of the ball when it is at distance from the centre is given by
2

N
Fg

1 11 GMm
mu2 - =0
2 8 R
11 GM
fi u =
4 R
mu2
N - Fg =
R
mu2 11 GMm GMm
N = + Fg = +
R 4 R2 ( R / 2 )2
27 GMm
N =
4 R2
41. A celestial body not bound to the sun has sufficient KE to travel infinitely far away from it. To get its minimum
speed at any distance r, we can assume that it has v = 0 when it is infinitely far away from the sun.
1 2 GMm
mv - =0
2 r

2GM 2 ¥ 6.67 ¥ 10 -11 ¥ 2 ¥ 1030


\v = =
r 1.5 ¥ 1011
= 4.2 × 104 m/s
42. A body projected with escape speed will have total energy equal to zero. Let the speed of the body be v when it is
1 GMm
at a distance x from the centre of the earth. Energy conservation gives- mv 2 - =0
2 x
v 2 GM 2GM dx 2GM
fi - =0 fiv= fi =
2 x x dt x
7.26 Problems in Physics for JEE Advanced

9R t
fiÚ x dx = 2GM Ú0 dt
R

2 È 3/ 2 ˘9 R
fi x ˚ = 2GM .t
3Î R

2
fi (26) R3 / 2 = 2GM . t
3
52 R3 / 2 52 R
t= =
3 2GM 2GM
3
R2
52 R
=
3 2g

43. (a) As the rock moves up from the lunar surface, its KE decreases and PE increases. There is a point in its path
where the gravitational field of the earth balances the field due to the moon (say this point is at a distance x
from the centre of the earth). Beyond this point the KE of the rock once again begins to increases as gravity of
the earth becomes more powerful. Hence, PE is maximum at distance x from the centre of the earth.
GM G M / 81
2 =
x (60 R - x )2
2
Ê 60 R - x ˆ 1
fi Á
Ë ˜¯ = 81
x
60 R - x 1
fi =
x 9
fi 54R = x
M
G m
GMm 81
U max = - - Q
54 R 6R

GMm È 1 1 ˘ GMm È 1 ˘ 5GMm
=- Í + ˙ =- Í1+ ˙ = -
R Î 54 81 ¥ 6 ˚ 54 R Î 9 ˚ 243R

(b) If stone is projected such that its speed is just zero when it is at a distance of x from the earth, it will reach the
surface of the earth with least KE
Kmin + PEnear earth = PEat x

M M
G m G m
GMm 81 GMm 81
K min - - =- -
R 59 R 54 R 6R

GMm È 1 1 1 ˘ GMm È 81 ¥ 59 ¥ 6 + 6 - 9 ¥ 59 - 59 ˘
K min = Í1+ - - ˙ =
R Î 81 ¥ 59 54 81 ¥ 6 ˚ R ÍÎ 81 ¥ 59 ¥ 6 ˙
˚
14045 GMm
K min =
14337 R
44. (a) T µ r3/2 fi w µ r –3/2
Dw 3 Dr
\ =-
w 2 r
Gravitation 7.27

3 Ê Dr ˆ
Dw = - Á ˜ w [ -ve sign indicates that angular speed of satellite is less than that of the earth]
2Ë r ¯
3 Ê 1km ˆ 2p
= - Á ˜ . = –9.3 × 10 –6 rad/hr
2 Ë 42000km ¯ 24 hour
Dq = Dw.t
(b)
Ê rad ˆ
= Á 9.3 ¥ 10 -6 ¥ (365 ¥ 24 hr ) = 0.08 rad = 4.6°
Ë hr ˜¯
45. In 8 hour the earth rotates through 120° and point A on the equator moves to A’. The satellite will scan the entire
Ê 1ˆ
equator if it completer Á 1 + ˜ revolutions in 8 hrs
Ë 3¯
4
\ T1 = 8
3
T1 = 6 hour = time period of revolution of S1
S1'

A' 120°
S1
A

3
For S2, the satellite can scan the entire equator in 6 hours if it completes th revolution in that time
3 4
T1 = 6 fi T1 = 8 hour
4
2 /3 2 /3
r13 T12 r1 Ê 6 ˆ Ê 3ˆ
= Hence, = =Á ˜
r23 T2 2 r2 ÁË 8 ˜¯ Ë 4¯
S2'

A'

S2
A

46. Let the semi major axis of the elliptical path be a

0.8AU r
T2 µ a3
For earth (1y)2 µ (1 AU)3
[AU is mean distance between the earth and the Sun]
T2 a3
\ =
(1 y)2 (1 AU )3
7.28 Problems in Physics for JEE Advanced

\T2 = a3 [where T is an year and a is in AU]


2/3
fia=T = (64)2/3 = 16 AU
r + 0.8
If r = maximum distance of the comet from the sun, then a =
fi 32 – 0.8 = r 2
fi r = 31.2 AU
47. Let radius of circular orbit of the Earth and Venus be re and rv respectively
3 2
Ê re ˆ Ê 365 ˆ
ÁË r ˜¯ = ÁË 220 ˜¯ [Kepler’s third law]
v

re
fi = (2.75)1/ 3 = 1.4
rv
From the drawing given in the problem
M ¢N ¢ N ¢V
=
MN NV
Ê N ¢V ˆ r
= 1000 ¥
1
M ¢N ¢ = MN Á ˜ = 1000 ¥ v
Ë NV ¯ re - rv re
-1
rv
1
= 1000 ¥ = 2500 km
1.4 - 1
48. Hint: At the point of intersection both the satellites have same PE. Since they have same mechanical energy, their
KE will be same at the point of intersection.
GMm GMm
49. Ei = - =-
2r 8R
GMm
Ef = -
2r
GMm Ê GMm ˆ 3 GMm
Loss in energy DE = - - Á- ˜ =
8R Ë 2R ¯ 8R

3GMm
\ ht =
8R

3GMm
t=
8h R

GMm 1 GMm
50. Energy of the satellite - = mV 2 -
2a 2 2R
GMm 3GMm GMm
- = -
2a 10 R 2R

2R r

2a
Gravitation 7.29

1 3 1
- = -
2a 10 R 2 R
fi 2a = 5R
\ r = 5R – 2R = 3R \ hmax = 3R – R = 2R
51.

R
V
30°
O

u

The velocity (V) makes 30° angle with the radius (normal) near the surface.
Conservation of angular momentum (about centre O)
mVR sin 30° = muR
\ V = 2 u ………………….(i)
Energy Conservation
1 2 1 GMm
mu = mV 2 -
2 2 R
1 2 1 GM
u = (2u ) -
2

2 2 R
3 2 GM
fi u =
2 R
2GM
fi u=
3R

52.

V0 V
R
q
O
5R


Let V = speed where the asteroid grazes the earth’s surface.
Conservation of angular momentum about O gives
V = mVR = mV0 sinq.(5R)
5
\ V = V ………………(i)
2 0
Energy Conservation
1 GMm 1 GMm
mV 2 - = mV 2 -
2 0 5R 2 R
7.30 Problems in Physics for JEE Advanced

V2 V2 GM Ê 1 ˆ
\ - 0 = 1-
2 2 R ÁË 5 ˜¯

Ê 25 1 ˆ 2 4 GM
ÁË - ˜¯ V0 =
8 2 5 R

21 2 4 GM
V0 =
8 5 R
32 GM
V0 =
105 R

GM
53. V0 = ……………..(i)
r
Speed after firing of the rocket is given by- V 2 = V02 + DV 2
1 GMm
For escape mV 2 - =0
2 r

V02 + DV 2 GM
=
2 r

V02 + DV 2 = 2 V02
DV = V0

DV
=1
V0

m Vo

V
V
new path

54. Angular momentum does not change (with respect to the centre of the earth)
mV1 (2r0) = mV0r0
fi 2V1 = V0 ……………..(i)
Energy conservation
1 GMm 1 GMm
mV 2 - = mV12 -
2 r0 2 2r0

2GM V02 GM
V2 - = -
r0 4 r0

V02 GM
V02 + D V 2 - = ÈQ V 2 = V02 + D V 2 ˘
4 r0 Î ˚
Gravitation 7.31

3 2 È GM ˘
V0 + D V 2 = V02 ÍQ V0 = ˙
4 ÍÎ r0 ˙˚
V02
DV 2 =
4
DV 1
=
V0 2
V1
orbit
original
earth

2r0
r0
V0

V V new orbit

55. (a) Speed is maximum at perigee (= V2). Applying conservation of angular momentum between apogee and
perigee–
V1

m
r2 r1

V2

mV1r1 = m.V2r2

r
\ V1 = V2 2 …………. (1)
r1
Energy Conservation
GMm 1 GMm 1 2
- 2
+ mV2 = - + V1
r2 2 r1 2
1 V22 - GM = 1 V12 - GM
2 r2 2 r1
Using (1)
2
Ê ˆ 1 Ê r2 ˆ 2 GM
- = Á ˜¯ - = Á ˜ V2 -
Ë 2 Ë r1 ¯ r1

1 2 È Ê r2 ˆ ˘
2
È 1 1˘
\ V2 Í1 - Á ˜ ˙ = GM Í - ˙
2 Í Ë r1 ¯ ˙ ÍÎ r2 r1 ˚˙
Î ˚
È (r + r ) (r - r ) ˘ Èr - r ˘
V22 Í 1 2 2 1 2 ˙ = 2GM Í 1 2 ˙
ÍÎ r1 ˙˚ Î r1r2 ˚

Ê ˆ
2GM Á 1 ˜
V2 =
r2 Á 1 + r2 ˜
ÁË r1 ˜¯
7.32 Problems in Physics for JEE Advanced

(b) For moon

Ê ˆ
2 ¥ 6.67 ¥ 10 -11 ¥ 6 ¥ 1024 Á 1 ˜
V2 =
3.6 ¥ 108
Á 3.6 ˜
Á1+ ˜
Ë 4 ¯

= 1.08 × 103 m/s


= 1.08 km/s
56. (a) Speed of the satellite at perigee is maximum. If the satellite is moving at speed V and rocket is fired to give an
impulse J, then change in momentum of the satellite is J.
Dp = J
The resulting change in kinetic energy can be calculated as
p2 [p = mv]
K=
2m
2p
D K  Dp
2m
DK = VJ
Hence, change in kinetic energy is maximum when speed V of the satellite is maximum. This happens at
perigee.
(b)
V2

earth
m
r 10r

V1

Let the speed of the satellite at perigee and apogee be V1 and V2 respectively.
Conservation of angular momentum
mV1r = mV2(10r)
fi V1 = 10V2 ……………………(1)
Energy Conservation
1 GMm 1 GMm
mV12 - = mV22 -
2 r 2 10r
Ê 1 ˆ 9 GM
V12 Á 1 - ˜ = [Using (1)]
Ë 100 ¯ 10 r


10 GM GM
V1 =  0.95
11 r r
For perigee distance = r and apogee distance = 12 r – we can again solve as above to get

V ´ = 132 GM  0.96 GM
1
143 r r
Gravitation 7.33

Hence, required impulse at perigee is

J = m V1¢ - V1( ) =1.01 m


GM
r
58. (a) No. The tunnel wall applies a normal force which produces a torque about the centre.
(b) Let the speed of the projectile be V when it comes out of the tunnel.

Vo C
V A

R/2
Vcos R q
O

Energy conservation can be applied between point C and A.


GMm È 3 2 1 2 ˘
PE at a distance x (x < R) is given by - Í R -2x ˙
R3 Î 2 ˚
\ KA + UA = KC + UC
GMm È 3 2 R 2 ˘
1 GMm 1
( )
2
mV 2 - = m gR - Í R - ˙
2 R 2 R3 Î 2 8 ˚
GM
Simplifying after substituting g = , we get
R2

GM 1 GM
V 2 = fiV =
4R 2 R
When the projectile is at farthest distance its velocity u is perpendicular to its position vector r relative to the
centre of the earth. Applying conservation of angular momentum between A and B we get–

VR
mur = m (V cosq ) R fi ur =
2
1 ……………………(i)
fi ur = GMR
4
Energy conservation between A and B

1 GMm 1 GMm
mu2 - = mV 2 -
2 r 2 R
2
1 Ê GMm ˆ GMm 1 1 GM GMm
m Á - = m -
2 Ë 4r ˜¯ r 2 4 R R
R 1 7

2
- =-
32r r 8R

fi 28 r 2 - (32 R ) r + R 2 = 0
7.34 Problems in Physics for JEE Advanced

32 R ± (32 R )2 - 28 ¥ 4 R2
\ r=
56

Ê 8 ± 57 ˆ
= Á ˜R
Ë 14 ¯

Ê 8 + 57 ˆ
\ rmax = Á ˜R The other solution is less than R and not acceptable.
Ë 14 ¯

59.

R
q h=6R
earth
C R S


With satellites (S) at vertex imagine a cone inside which the spherical earth fits in. The surface area of the earth
inside this cone will receive the communication signals from the satellite.
R 1
From the figure cosq = =
7R 7
The solid angle subtended by the surface exposed to signal from the satellite at the centre of the sphere is
Ê 1 ˆ 12 p
W = 2p (1 - cos q ) = 2p ÁË 1 - ˜¯ = Steradian
7 7
\ Required area
S = W.R2
2
12 p R
S=
7
60. (a) Consider a ring element on the sheet as shown in figure. Mass of the ring element is –

m P

q
x
R

dr
r
Gravitation 7.35

dm = s 2p r dr
Force on mass m placed at P due to ring will be perpendicular to the sheet equal to
G m . dm . G.m. s .2p rdr. x
dF = 2
cos q =
R R3
rdr
= G.2p s m . x
(x )
3
2
+ r2 2

Force will be equal to sum of forces due to all concentric rings which make up the sheet.

r =•
rdr
\ F = G 2p s m x Ú
( )
3
r =0 x 2 + r 2 2

r =•
È 1 ˘
= - G 2p s m x Í ˙
ÍÎ x + r 2
2
˙˚r = 0

= G 2p s m [Independent of distance x]

Note: Students may also use gauss’ theorem for gravitation to arrive at the result.
(b) From the result obtained above, we can say that field due to a thin large sheet, of mass density s per unit area,
is G.2ps
If the sheet is of thickness t, then also the result will remain same because the sheet can be sliced into infinite
number of thin sheets each producing field G.2ps. [Remember this field is independent of distance from the
sheet]
And s = rt
\ Field due to sheet Esheet = G2prt
B

Earth

Resultant field at A is
GM
E A = 2 - 2p G r t
R

GM
Field at B is; E B = + 2p G rt
( R + t )2
4
Where mass of earth M = p R3 r0
3
As per question EA > EB

GM GM
2 - 2p G rt > + 2p G rt
R ( R + t )2
7.36 Problems in Physics for JEE Advanced

È ˘
Í ˙
fi 4 p R 3 r Í 1 - 1 ˙ > 4p rt
3
0
Í R 2
Ê tˆ ˙
2

Í R2 Á1 + ˜ ˙
Î Ë R¯ ˚

È Ê tˆ ˘
-2
fi r0 R Í1 - Á 1 + ˜ ˙ > 3 rt
ÍÎ Ë R ¯ ˙˚

-2
Ê tˆ 2t
We can write ÁË 1 + ˜¯  1-
R R

È 2t ˘ 3
\ r0 R Í1 - 1 + ˙ > 3rt fi r0 > r
Î R˚ 2
61.

R l
E ar t h

T T

M = mass of earth, m0 = mass of spaceship, T = tension in string


For spaceship
Mm0
G - T = m0w 2 R ............... (i )
R2
For Astronaut
GMm
+ T = mw 2 ( R + l ) ............... (2 )
( R + l) 2

From (1) and (2) and using R + l = R´


GM T GM T
- = +
R 3 m0 R ( R ¢ )3 m R ’

È 1 1 ˘ Ê 1 1 ˆ
fi TÍ + ˙ = GM ÁË 3 - 3 ˜¯
Î mR ¢ m0 R ˚ R R¢
Ê GM mmo ˆ Ê 1 1 ˆ
fi T =Á ˜ ÁË 3 - 3 ˜¯
Ë mo R + mR ¢ ¯ R R¢
This gives R¢3 – R3 = (R¢ – R)(R¢2 + RR¢ + R2)
~ l(3R2) ÈQ R ¢ 2  R 2  R ¢R ˘
Î ˚
GM mmo 3lR 2 GM . 3mmo l
\ T 
= 3
(mo + m ) R5 R mo + m
Gravitation 7.37

GM .
 3ml ÎÈQ mo + m  mo ˘˚
( 2 Re )3

GM . 3 ml
=
Re2 8 Re
3 ml
=g .
8 Re

3 100 ¥ 200
= 9.8 ¥ ¥
8 6400 ¥ 1000
= 0.01 N
62.
nR

w
O
R C
x
equator dx


The rod can stay above the same point only if it rotates with angular speed of earth from west to East (just like
geostationary satellites). The necessary centripetal force has to be provided by the gravitational pull of the earth.
Distance of centre of the rod from the centre of the earth is

r=
(n - 1) R + R = (n + 1) R
2 2
Mass of the rod m = l (n – 1) R
Where l = mass per unit length.
Gravitational pull on the rod can be calculated by writing force on an element of length dx.
GM ( l dx )
dF = [M = mass of earth]
x2
Total force on the rod
nR
dx GM l È n - 1 ˘
F = GM l Ú x 2
=
R ÍÎ n ˙˚
R

\ F = mw o2 r

GM l È n - 1 ˘ Ê n + 1ˆ
= l ( n - 1) R . w o2 Á R
Í
R Î n ˚ ˙ Ë 2 ˜¯

4 3
But M = pR r
3

\
4 Gp r
= w 02
(n + 1)
3 n 2
8 p Gr
\ n2 + n =
3 w o2
7.38 Problems in Physics for JEE Advanced

2GM
63. (a) Escape velocity Vo =
R
\ Velocity of projection
Vo

 = 30°

O
R h

V
r

1 2GM GM
Vo = = ………………..(i)
2 R 2R
When R = radius of earth, M = Mass of earth
At the point where the body is farthest & nearest to earth’s centre, its velocity is ^r to the position vector with
respect to earth’s centre.
\Angular momentum when it is farthest/nearest to earth’s centre is = mVr
Where r = distance from centre
V = velocity at farthest/nearest point
Angular momentum of body (about O) at the time of projection is
3
mV0 cos 30∞. R = mRV0
2
Since Angular momentum is conserved
3
mVr = mRV0
2
3
fi Vr = RV0
2
3 RV0
fi V = ……………………(ii)
2 r
From conservation of energy
1 GMm 1 GMm
mV02 - = mV 2 -
2 R 2 r
2GM 3 R 2 2 2GM
or, V02 - = V -
R 4 r2 0 r
GM 2GM 3 R 2 GM 2GM
Using (i) - = -
2R R 4 r2 2R r
1 2 3 R 2
fi - = -
2R R 8 r2 r
Gravitation 7.39

-3 3R - 16r
fi =
2R 8r 2

fi 12r2 – 16Rr + 3R2 = 0

16 R ± 256 R 2 - 144 R 2 Ê 4 ± 7 ˆ
or, r = =Á ˜ R ……………………(ii)
24 Ë 6 ¯

Q h cannot be less then R hence,



Ê 4 + 7ˆ Ê 7 - 2ˆ
\ h=Á ˜ R - R=Á ˜ R
Ë 6 ¯ Ë 6 ¯

Ê 4 - 7ˆ
(b) From (iii) rmin = Á ˜R < R
Ë 6 ¯
Hence, the body will move on elliptical path but it will hit the surface of earth at some point.
64. (a) Speed of dust particles before collision with satellite ~ 0
Speed of dust particles after collision with satellite = V
Where V is the orbital velocity of the satellite. Satellite encounters (VS d) kg of dust per sec
\ Force experienced = change in momentum per sec
F = (VS d) (V) = V2 Sd
= (8 × 103)2 × 0.5 × 1.6 × 10–11 ~ 5 × 10–4 N
[Q orbital speed of near surface satellite ~ 8 km/sec]
(b) Total energy of the satellite

1 GMMe È R = radius of earth ˘


E = - MV 2 = - Í = path radius ˙
2 2R Î ˚

GMMe
E = -
2R
GMMe
fi D E = + DR
2 R2

2 R2
fi D R = DE
GMMe
But change in energy in one revolution E = work done by F
= – 2 RF
2 R2
\ DR = (-2p R F )
GMMe
Ê4 ˆ
p R3
4 p R3 F -3 Á 3 ˜ -3F
= - = Á ˜ =
GMMe GM Me GM r
Á ˜
Ë ¯
Given r = mean density of earth = 5500 kg/m3
7.40 Problems in Physics for JEE Advanced

- 3 ¥ 5 ¥ 10 -4
\ DR = = - 0.4 km
6.67 ¥ 10 -11 ¥ 10 ¥ 5500
– ve sign indicates reduction in radius.
1
Similarly, E = - MV 2
2
E = – MV V
D E 2p RF
fi DV = - =  0.25 m /s [Velocity increases]
MV MV
08 FLUIDS

Level 1 rod attached to it. Water rises in the pipe along


with the piston. Why? To what maximum height
water can be raised in the pipe using this method?
Q. 1. We know that the atmospheric pressure on the
What will be the answer to your question if water
surface of the earth is because of weight of
is replaced with mercury? Atmospheric pressure
the air. The radius of the earth is 6400 km and
is Patm = 1.01 × 105 Pa.
atmospheric pressure on the surface of earth is
1 × 105 N/m2.Estimate the mass of the earth’s Q. 5. A hypothetical planet has an ocean of water which
atmosphere assuming that acceleration due to is 50 km deep. The top 5 km is frozen as ice (i.e,
gravity remains constant at 10 m/s2 over the entire 45 km is water). Radius and average density of the
height of the atmosphere. planet are both half the respective values for the
earth. There is no atmosphere. Obtain an estimate
Q. 2. Why mercury is used in a barometer, though it of the pressure at the bottom of the ocean.
is costly? Why cannot we use water in place of
mercury. Q. 6. Two identical beakers are filled with water. One
of them has an ice block floating in it. The level of
Q. 3 Look at the barometer shown in the figure. If a water in both the beakers is same. Which beaker
small hole is developed in the wall of the tube at will weigh more? Will your answer change if
point A, will the mercury leak out of it? water is replaced with a liquid of higher density
in the beakers?
Q. 7. (i) A toy boat made of steel is floating in a
beaker having water. The beaker is placed on
A a spring balance. The boat tilts and sinks into
water.
(a) Will the level of water in the beaker go
up or fall down?
Hg
(b) Will the reading of spring balance
Q. 4. decrease or increase?

Rod

Piston

(ii) You are in a boat on a calm lake. There is a


floating log near you. You pick the log and
put it into the boat. What happens to the level
A tightly fitted piston can slide along the inner of water in the lake? Does it rise or fall?
wall of a long cylindrical pipe. With the piston Q. 8. A closed cubical box of negligible mass has large
at the lower end of the pipe, the lower end of the number of spherical balls arranged neatly inside
pipe is dipped into a large tank, filled with water. it as shown in the figure. When placed in water,
Now the piston is pulled up with the help of the the box floats with 80% of its volume remaining
8.2 Problems in Physics for JEE Advanced

submerged. What is specific gravity of material


of the balls? Neglect thickness of the wall of the density d with of its length outside the liquid.
box. (a) Find d
4 1
(b) Show that
3

Q. 9. Two identical containers have the same volume l


d1
of water in it. Each of them is placed on a balance
d
and readings of the two balances are same. There
is a hollow ball and a solid ball that have same l
d2
volume. The hollow ball floats in water and the
solid ball sinks. A string from the ceiling suspends
the solid ball so that it remains completely
Q. 12. A sealed balloon, filled with air, floats in water
submerged in the water in the first container. The
1
hollow ball is held submerged in the water in the with of its volume submerged. It was found
second container and is held by a string fastened 3
that if it is pushed inside water at a depth h, it
to the bottom of the container. Which balance
remains in equilibrium, neither sinking nor rising.
will show higher reading? How will your answer
Find h. Given that height of water barometer is
change if the string in the second container is cut?
10 m and temperature is constant at all depth.

Q. 10. Three tanks T1, T2 and T3 are sitting on three


Q. 13. Long back our Earth was made of molten material.
weighing scales S1, S2 and S3 respectively. Tank
Assume it to be a uniform sphere of radius R
T1 has a spout, as shown and water has been filled
having density d. Take acceleration due to gravity
in it to a level just below the spout. The other two
at the surface to be g and calculate the gauge
tanks are empty. Reading of the three scales are
pressure (P0) at the centre of this fluid Earth.
20 kg, 4 kg and 3 kg respectively. A 4 kg body is
Calculate P0 for following data: R = 6000  km;
put into the tank T1 and it floats in the water. Now
d = 5500 kg m–3 and g = 10 ms–2.
the reading of scale S3 was found to be 4.5 kg.
What is the reading of other two scales? Q. 14. A device used to measure the specific gravity
of a liquid is called a hydrometer. In a simple
hydrometer there is a cylindrical glass tube with
some lead – weight at its bottom. The device
floats in liquid while remaining vertical. The
top part of the tube extends above the liquid and
the divisions marked on the tube allows one to
directly read the specific gravity of the liquid.

Assume that the water in the system remains 1.0

inside three tanks.


Q. 11. A cylindrical block of length 2l is made of two Lead
different materials. The upper half has density
d1 and lower half, which is heavier, has density The scale on the tube is calibrated such that in
d2. The block is floating in a liquid of unknown pure water it reads 1.0 at the water surface and a
Fluids 8.3

length z0 of the tube is submerged. Calculate the Q. 18. A light cylindrical tube of length L = 1.5 m
specific gravity of the liquid if the liquid level is
z above the 1.0 mark. Disregard the curvature of and radius r = m is open at one end. The tube
π
the tube bottom.
containing air is inverted and pushed inside water
Q. 15. A sphere of radius R and having negligible mass is as shown in figure. A block made of material
floating in a large lake. An external agent slowly of relative density 2 has been placed on the flat
pushes the sphere so as to submerge it completely. upper surface of the tube and the whole system
How much work was done by the agent? Density is in equilibrium. Neglect the weight of air inside
of water is r. the tube and find the volume of block placed on
Q. 16. Two identical communicating containers have the tube.
water filled into them. A spherical ball of ice
(relative density = 0.9 ) having volume 100  cm3 Block

is put into the left vessel. Calculate the volume Air L/2
of water flowing into the right container,
immediately after placing the ball (i.e., don’t L/2
consider any melting of the ice ball). Give your
answer for following two cases
Q. 19. A solid wooden cone has been supported by a
string inside water as shown in the figure. The
radius of the circular base of the cone is R and the
volume of the cone is v. In equilibrium the base of
the cone is at a depth H below the water surface.
Density of wood is d (< r, density of water).

(i) The ice ball floats in the water in the left


container.
H
(ii) The ice ball gets exactly half immersed in the
water.
(iii) What will happen to the water level after
the ice melts? Answer for both (i) and (ii)
above.
(a) Find tension in the string.
Q. 17. An open cylindrical container has a cross sectional
(b) Find the force applied by the water on the
area A0 = 150 cm2 and water has been filled in it up
slant surface of the cone. Take atmospheric
to a height h. A cylinder made of wood (relative
pressure to be P0
density = 0.6) having cross sectional area A = 125
cm2 and length 10 cm is now placed inside the Q. 20. A large container has a sliding vertical wall of
container with its axis vertical. Find the distance height H so as to divide it into two parts. The
(x) of the base of the wooden cylinder from the partition wall is connected to the left container
base of the container in equilibrium for following wall by an ideal spring of force constant k. When
three cases : the spring is relaxed the dimensions of the floor
of the right part is L × b. Now water (density r) is
(a) h = 8 cm slowly poured into the right chamber. What is the
(b) h = 12 cm maximum volume of water that can be stored in
(c) h = 0.8 cm the right chamber without spilling it into the other
part. The partition wall slides without friction.

h k H
x

L
8.4 Problems in Physics for JEE Advanced

Q. 21. A cylindrical container has cross sectional area of


0.20 m2 and is open at the top. At the bottom, it
has a small hole (A) kept closed by a cork. There a
is an air balloon tied to the bottom surface of the q
container. Volume of balloon is 2.2 litre. Now
water is filled in the container and the balloon
gets fully submerged. Volume of the balloon r
reduces to 2.0 litre. The cork is taken out to
open the hole and at the same moment the whole
container is dropped from a large height so as
to fall under gravity. Assume that the container
remains vertical. Find the change in level of water
inside the falling container 2 second after it starts
falling.
Q. 22. A wooden stick of length L, radius R and density
r has a small metal piece of mass m (of negligible
volume) attached to its one end. Find the
minimum value for the mass m that would make
the stick float vertically in equilibrium in a liquid
of density s.
A B
Q. 23. A rod of length 6 m has a mass 12 kg. It is hinged
at one end at a distance of 3 m below water
surface.
(a) What weight must be attached to the other
end of the rod so that a length of rod equal to Q. 28. (i) A ball is projected in still air. With respect
5 m is submerged in water in equilibrium? to the ball the streamlines appear as shown
in the figure. At which point is the pressure
(b) Find the magnitude and direction of the force larger – 1 or 2?
exerted by the hinge on the rod. (Specific 2
gravity of rod is 0.5).
1

(ii) In the above figure if the ball is also spinning


in clockwise sense, in which direction it will
get deflected – up or down?
r Q. 29. (i) In the arrangement shown in the figure, the
tank has a large cross section and the pipes
have much smaller cross sections. The
g
a opening at A is unplugged and the water
jet hits the ground surface at a horizontal
distance x.

H
q
q
Fluids 8.5

(a) Find the level of water (h) in the tube B


as water flows out of A.
(b) Find x.
(ii) A flat horizontal surface has a small hole A B
at its centre. A circular glass plate of radius
R is placed symmetrically above the hole
with a small gap h remaining between the h2
h1
plate and the surface. A liquid enters the
gap symmetrically from all sides and after
travelling radially through the gap finally v
exits from the hole. The volume flow rate (a) The diagram depicts that height of liquid in
of the liquid coming out from the hole is Q tube B (= h2) is more than the height of liquid
(in m3s–1). in tube A (= h1). Is it correct?
(a) If the flow speed just inside the (b) Calculate the difference in height of the
circumference of the circular plate is liquid in two tubes.
V0 find the speed (Vx) of flow inside the
gap at a distance x (see figure) from the
centre of the hole.
(b) Write Vx in terms of Q, h and x.

x R glass plate

h
h

h0

Q. 30. A horizontal tube having cross sectional area v


A1 = 10 cm2 has a venturi connected to it having
cross sectional area A2 = 4 cm2. A manometer,
having mercury as its liquid is connected to the
tube as shown in the figure. The manometer tube
has uniform cross section and it has a horizontal
part of length L = 10 cm. When there is no flow
in the tube the height of mercury column in both
vertical arms is H = 12 cm. Calculate the minimum r
flow rate (in m3/s) of air through the tube if it is
required that the entire amount of mercury move Vacuum
to one vertical arm of the manometer. Given:
h
density of Hg = 13.6 × 103 kg m–3; density of air =
1.2 kg m–3. v
A1 A3
A2

A1 A2 Piston
(i) Find the force needed to push the piston
assuming that friction force between the
piston and the tube wall is = 40 N
(ii) Find the height (h) of mercury column in the
H attached vertical tube. What happens to this
height if the piston is pushed with smaller
L
speed?
8.6 Problems in Physics for JEE Advanced

(a) Plot the variation of pressure inside the lake


as a function of height y from the base. Let
the height of piston from the base, after the
pipe is made vertical, be y. Plot the variation
of gas pressure as a function of y in the first
graph itself.
(a) A homogeneous solid cylinder of length L (L
(b) In equilibrium the gas pressure and the
< H/2), cross-sectional area (A/5) is immersed
pressure due to water on the piston must be
such that it floats with its axis vertical at the
equal. Using this solve for equilibrium height
liquid-liquid interface with length (L/4) in
y0 of the piston. You get two answers. Which
the denser liquid. Determine (i) The density
one is correct and why?
of solid and (ii) The gauge pressure at the
bottom of the container. Q. 36. A centrifuge has a horizontal cylinder rotating
(b) The cylinder is removed and original about a vertical axis as shown in the figure. Water
arrangement is restored. A tiny hole of area inside it has density r.
s(s << A) is punched on the vertical side of the
container at a height h(h < H/2). Determine
(i) the initial speed of efflux of the liquid at
the hole (ii) the horizontal distance x travelled dP
by the liquid initially and (iii) the height hm dr
at which the hole should be punched so that w
the liquid travels the maximum distance xm
initially. Also calculate xm.

r r
H/2 d

H/2 2d h w
w

X
A
Level 2
r
r
w

10
r

Q. 37. (i) A cubical metal block of side 10 cm is floating


in a vessel containing mercury. The vessel
Y has a square cross section of side length
15 cm. Water is poured into the vessel so
that the metal block just gets submerged.
Calculate the mass of water that was poured
into the vessel. It is given that relative density
of the metal and mercury are 7.3 and 13.6
respectively.
gas

H y
10
gas
(ii) In the last question, in place of water if we
O
Fluids 8.7

poured another liquid of relative density ‘r’


it was found that when the metal block was
just completely submerged it was no longer
touching mercury. What is value of r?

r
A r
h
A

h
r

2r

(a) How does the centre of mass of the system


(container + liquid) move as the height (x)
of liquid column changes from zero to H?
Explain your answer qualitatively. Draw a
q graph showing the variation of height of
centre of mass of the system (xcm) with x.
Bubble (b) Find the height of liquid column x for which
the centre of mass is at its lowest position.

q H

Q. 44. A cubical ice block of side length ‘a’ is floating in


water in a beaker. Find the change in height of the
centre of mass of (water + ice) system when the
ice block melts completely. It is given that ratio
8.8 Problems in Physics for JEE Advanced

of mass of water to mass of ice originally in the


container is 4 : 1.

Q. 45. A cylindrical ice block is floating in water. 10%


of its total volume is outside water. Kerosene oil
(relative density = 0.8) is poured slowly on top of
water in the container. Assume that the oil does not
mix with water. Height of the ice cylinder is H.
r

water
h h
2

(a) As kerosene is poured, how does the volume


of ice block above the water level change?
(b) What is the thickness of kerosene layer above h
2
the water when 20% of the volume of the ice
block is above the water surface?
(c) Find the ratio of volume of ice block in
kerosene to its volume in water after the
kerosene layer rises above the top surface of
ice and the block gets completely submerged.
Neglect any melting of ice
Q. 46. A cylindrical container contains water. A cubical
block is floating in water with its lower surface
connected to a spring
(a) Suppose that the spring is in relaxed state. Now,
if the whole container is accelerated vertically
upwards, will the spring get compressed?
(b) Suppose that the spring is initially compressed. h3 = 4 m

Now, what will happen to the state of the


spring when the container is accelerated
upwards? 2r

h2 = 10 m
(c) Assume that mass of the block is 1 kg and
initially the spring (force constant k = 100
h=3m
N/m) is compressed by 5 cm. When the
container is accelerated up by an acceleration h1 = 1 m
of 5 m/s2, the spring has a total compression of
B1 B2
6 cm. Calculate the change in volume of block
submerged inside water when the container
gets accelerated. Density of water is 103 kg/
m3.
r r
Fluids 8.9

r
a
2

B
a
A

(ii) A rectangular concrete block (specific gravity


= 2.5) is used as a retaining wall in a reservoir
of water. The height and width of the block
are x and y respectively. The height of water
3
in the reservoir is z = x . The concrete block
4
cannot slide on the horizontal base but can
rotate about an axis perpendicular to the plane
of the figure and passing through point A
y

Concrete x
z block

A
y
(a) Calculate the minimum value of the ratio
x
for which the block will not begin to overturn
about A.
(b) Redo the above problem for the case when
there is a seepage and a thin film of water is
present under the block. Assume that a seal
at A prevents the water from flowing out Gauge pressure at a point is difference in
underneath the block. absolute pressure at the point and atmospheric
Q. 50. A container having an ideal liquid of density pressure.
r is moving with a constant acceleration of Rear Front

a = a x iˆ + az kˆ where x direction is horizontal and
z is vertically upward. The container is open at the
top. In a reference frame attached to the container 3m
with origin at bottom corner (see figure), write the 2m a
pressure at a point inside the liquid at co-ordinates
B A
(x, y, z). The pressure is P0 at origin.
Z 5m

Q. 53. A water clock consist of a vessel which has a small


orifice O. The upper container is filled with water
which trickles down into the lower container. The
r shape of the (upper or lower) container is such that
height of water in the upper container changes at
O X
a uniform rate. What should be the shape of the
8.10 Problems in Physics for JEE Advanced

container? Assume that atmospheric air can enter the specific gravity of the material of the
inside the lower container through a hole in it and cylinder.
that the upper container is open at the top. Vessel (b) the time period of oscillations if the cylinder
is axially symmetric.  
is depressed by some small distance  < 
 
Neglect viscosity and change in level of
O
liquids when the cylinder moves.
Q. 57. (i) In the figure shown, the heavy cylinder (radius
R) resting on a smooth surface separates two
liquids of densities 2r and 3r. Find the height
‘h’ for the equilibrium of cylinder.
q
R 3r
2r
h R

q)

R
O


R
w

Water L
4

3 1
4 4 Dh
D P0
Fluids 8.11


rw V = (u0 + bx ) i - by j; u0

r
P1 P2 .
A1

Zero

Water h

Mercury

P1 9A A
P2
Scale

 r
(a) Find the speed of the piston
(b) Find the total work done by the external agent
in emptying the syringe.

.
3 –1
Load Calculate the discharge rate in m s

a
b
A1 F

A2
8.12 Problems in Physics for JEE Advanced

d0
d

(a) (b)
(i) Prove that the tank will move with a constant
acceleration till it is emptied. Find this
acceleration.
(ii) Find the find speed acquired by the tank when
it is completely empty.

H H

H
I 1 A II 1

h h

2 A
2

to A. Point 1 marked in both figures, is a point just


below the opening in the tank and point 2 marked
in both figures, is a point h below point 1 [In fig
II, point 2 is just outside the opening in the pipe.].
(a) Find the speed of flow at point 2 in both
figures.
(b) Find the ratio of speed of flow at point I is
first figure to that in second figure.
(c) Find the difference in pressure at point 1 in
both figures.
Q. 66. To illustrate the principal of a rocket, a student
designed a water rocket as shown in the figure. It
is basically a container having pressurized gas in
its upper part and water in its lower part. Pressure
of the gas is 4.0 Mpa. Mass of empty container is r
1.0 kg and mass of its content is also1.0 kg. The
(a) Calculate the gage pressure in the tube.
nozzle at the bottom is opened to impart a vertical
acceleration to the container. If it is desired that (b) Calculate the volume flow rate (Q) of the air.
the initial upward acceleration of the container be R
0.5g, what should be the cross sectional area (A)
of the exit of the nozzle?

A B
x
gas

water

b
A
Fluids 8.13

r)

A1 A2

A1 A2 A1 atmosphere
h1
H h2

r
(ii) A non viscous liquid of constant density r
flows in a streamline motion along a tube
of variable cross section. The tube is kept
inclined in the vertical plane as shown in the
figure. The area of cross section of the tube at
two points P and Q at heights of h1 and h2 are
respectively A1 and A2. The velocity of the (a) Will the water drain out in this siphon? If yes,
liquid at point P is v. Find the work done on a at what speed (V)?
small volume V of fluid by the neighbouring
(b) Find pressure at the top of the siphon tube
fluid as the small volume moves from P to Q.
(call it P´
Q

P h2
h1

´

 R3 
V = V0 1 + 3  V0
 x 

H
r
h

x=–R
Level 3
A X
V0 B O
O origin s
8.14 Problems in Physics for JEE Advanced

r r

(a) If density of water is r0 r

s r
O
2r
water
r
Q. 77. In a machine, a fluid from a compressor, which is at
high pressure, is allowed to pass through a nozzle.
Cross section of the nozzle is shown in the figure.
hole
The nozzle consists of two sections of radii r1 and
r2. The nozzle is fixed to a stand with the help of
m a clamp. The clamp is a circular ring of radius r1
R and width b .The fluid from the compressor is at
a pressure of n times the atmospheric pressure P0.
Assume that the entire system is horizontal, the
fluid is ideal and the flow is steady.
b
A
r1 B
r2
High pressure
fluid
clamp
r
(a) What should be the volume flow rate so that
pressure of the fluid at end B reduces to half
π  1 of its value at end A?
tan −1    57° cos 57∞ 
2 2

O Cable
A

m
B

C
Fluids 8.15

ANSWERS

4 × 1018 kg 21. Water level will rise by 1 mm.

Ê s ˆ
22. p R L r Á - 1˜
2

Ë r ¯
23. (a) 2.33 kg
(b) 56.7 N
24. 18 Nm–2
10
25. q = tan–1 (3); T mg
3
7. (i) (a) Fall down
26. P0 + 2 rgR
(b) Not change
27. A
(ii) Does not change.
28. (i) P1 > P2
8. 1.54
(ii) up
9. First balance will show higher reading.
29. (i) (a) h = 0
Answer will not change if string in 2nd container is
cut. (b)
x = 2 h0H

10. Reading of S1 = 20 kg; Reading of S2 = 6.5 kg R


(ii) (a) Vx V
2 x
11. (a) = ( 1 + 2 )
3
12. 20 m Q
(b) Vx =
π hx
1
13. P0 = g.d .R = 1.65 × 1011 Pa 30. 12.1 m3s–1
2
31. (a) Yes
14.
+∆ (b)

16. (i) 45 cc v2
(ii) 25 cc 32. h0
2g
(iii) When ice melts there will be no change in water
level in case (i) and the water level will rise in 33. (i) 371.2 N
case (ii). (ii) 26 cm. Height h will increase
17. (a) 7 cm
5
(b) 0 34. (a) (i)
4
(c) 0 1
(ii) (6 H L)dg
18. 0.75 m3 4
19. (a) vg (d – r)
(b) (i) ( g / 2)(3H 4h)
(b) R2 [r0 + rgH] – vrg
(ii) h(3H 4h)
20. Hb  L + ρ gbH 
 k  3
(iii) (3/8)H,
4
8.16 Problems in Physics for JEE Advanced

35. (a) r r
P
r

Ê aˆ
h = L Á1 - ˜
Pwater Ë g¯
y
O 2g
4 m /s 2
5
H 15
y=
(b) − H
2 10
dP
36. (a) = ρω r
dr
 ρ 2
(b) 1 −  ω m
 ρ '
w 10
37. (i) 625 g
(ii) r = 7.3 7
1 4
38. gAh 2
2 7L  1 
π 1 + 
π
39. ρ gLr  +  4g  2
  3
40. a = g tan q towards right
2
41. 0.4 mm
1
42. W = Aρ0 gh 2
16

43. (a) The COM first falls, attains a minimum height
1
s=
and then it rises to original height  A2 
1 +  ρ Hg .g
(b)
x = ( 6 – 2)H  A1 
44. 0.01a q
45. (a) First increases then become constant
c
(b) y=
u + bx

(c)
46. (a) No 4 3
Q= πη r 2 g H 2
(b) spring will get compressed more 3
(c) 100 cm3 a
47. V rg/8 u = u0 = 5 mm /s
A
48. 179 KN
9
49. (i)
8
3
(ii) (a) v = g ( H + h)
4 10
3
(b) H
4 7
H h
Fluids 8.17

P0
H0 =  10.3 m
ρg
2 A0 g
V gH s
A
s

s0
−1
2 ρ0 gH  1 1 
Q=  2 − 2 1.0
ρ A
 2 A 1 

1  A2  V
ρ∆V  12 − 1 v12 + ρ∆Vg ( h2 − h1 ) O


2  A2  πR ρ
m=
  R3 
P = P0 + ρV02 1 − 1 + 3  
  x  4 +π 2 .
ρ gR 2 L
8
P
(ii) rV02
P0 + 2 ρ0 123R
2 ρ= T = 2π
3 40 g
nP0
77. (a) Q = p r1 r2
2 2
P0
X {
r r14 - r2 4 }
-R F
(b)
2 A1 A2 h1 - h2 2πµ r1b
g A0 ( A1 + A2 ) 2(n − 1) P0
(c ) Q = π r12 r22
ρ[r14 − r24 ]
gh

SOLUTIONS

pA 1 × 105 × 4π (6.4 × 106 ) 2


1. m= =  5.14 × 1018 kg
g 10
3. The pressure in the tube at the level A will be below the atmospheric pressure. Therefore, the atmospheric pressure
will not allow the water to flow out when a hole develops. Air will enter the tube through the hole until the
atmospheric pressure is reached inside the tube and the mercury level will sink to the level outside the tube.
4. The water will rise along with the piston till the pressure produced by the water column at point 1 becomes equal to
atmospheric pressure.

2
1
8.18 Problems in Physics for JEE Advanced

= P2

1.01 × 105
rw ⇒ h= = 10.30 m
103 × 9.81
rHggh = Patm h = 0.76 m

4 3
G πR ρ
GM 3 4π
5. g= 2 = = Gρ R
R R2 3
ρ Re
ρ=
Since and R

g e 9.8
gP = = = 2.45 ms −2
4 4
P = rice gP dice + rwgP dw

 kg   m  kg   m
=  900 3  ×  2.45 2  × ( 5000 m ) + 1000 3  ×  2.45 2  × (1000 m )
 m   s   m   s 

= 1.35 × 107 Pa

6. Weight of ice block = Weight of liquid displaced.


Hence weight of both the beakers will be same.
8. Think of each ball tightly packed inside a cube.
4 3
πr
π
Fraction of volume occupied by balls f = 3 3 = = 0.52
( 2r ) 6
If volume of the box is v, then 0.52v is occupied by the balls.
Mass of box = 0.52v.d [d = density of balls]
Buoyancy force when 80% of the box is submerged is
FB = 0.8vrw.g
0.52vdg = 0.8vrwg
0.8
& ρ = 0.52 = 1.54
ω

9. Buoyancy on both balls, when completely submerged will be same (say FB).
This FB is large than weight of the hollow ball.
In first container, the ball exerts a force FB on the water + container system.
This increases the reading of the balance by FB
In second container, the reading increases by an amount equal to weight of the hollow ball (that is less then FB).

11. W = FB
3
Ald1 g + Ald 2 g = A ldg
2
2
= ( 1 + 2)
3
Fluids 8.19

3 3
2 = − 1> 1 1
2 4
12. Let mass of the balloon be M and its volume on the surface be v0
v
Mg = 0 ρ g   ......…(i) [r = density of water]
3
At depth ‘h’, the volume of the balloon will decrease due to hydrostatic pressure. Let new volume be v.
p2v2 = p1v1 gives

10 v0
(h + 10)v = 10v0 v =
h + 10
For equilibrium vrg = Mg

10 v0 v
ρ g = 0 ρ g [using (i)]
h + 10 3
30 = h + 10 h = 20 m

dP
13. If P is pressure at depth h below the surface = g h ⋅ d [gh = acceleration due to gravity at depth ]
dh
dP  h
=d⋅g − 
dh  R

 h
dP = g ⋅ d  −  dh

 R
P0
R 1
R

∫0 dP = gd  ∫0 dh − R ∫0 hdh 

 R 1
P0 = g ⋅ d  R −  = g ⋅ d ⋅ R
 2 2
For given data
1 11
P0 = × 10 × 5500 × 6000 × 103 Pa = 1.65 × 10 Pa
2
14. When the hydrometer is in water, the buoyant force is equal to its weight. A length z0 of the tube is submerged in
water.
W = rwgAz0
…(i) [A = cross section of tube]
In a liquid of density less than water (r < rw), the hydrometer will sink deeper and the liquid surface will be at a
distance z above z0.
W = rgA (z0 + z) …(ii)

Z
1.0
r
Z0
8.20 Problems in Physics for JEE Advanced

From (i) and (ii)


rgA (z0 + z) = rwgAz0
ρ
=
ρω +∆

specific gravity of liquid =


+∆

15. Consider the sphere when it is submerged up to depth h.

R
FB

h
Fext

Volume of the submerged part


1
v = π h 2 (3R − h) [Prove this result yourself]
3
ρ gπ h 2
Buoyancy FB = ρ vg = (3R − h)
3
To push the ball slowly, the external agent must apply a downward force equal to FB
Fext = FB
Work done in pushing the sphere further down by dh is
ρ gπ
dW = Fext ⋅ dh =
3Rh 2 − h3  dh
3 
Work required to completely submerge the ball is
2R
W= ∫ dW
h=0

πρ g  2 R 2 2R

W=  ∫
3 R h dh − ∫ h3 dh 
3  0 0 

πρ g  ( 2 R )3 ( 2 R ) 4 
= 3R ⋅ − 
3  3 4 

πρ g
= 8 R 4 − 4 R 4  = 4 π R 4 ρ g
3  3
Alter:

4 3
pR
3

4 3
Since lake is large we can assume that the entire water ( = π volume) has been brought to the surface. Earlier
3
Fluids 8.21

this volume of water was having its centre of mass at a depth R below the surface.

4  4
=  π R3 ρ ⋅ g  R = π R 4 ρ g
3  3
16. Let volume of water on each side (before ball is put in) be v0 and v be the volume of submerged part of the ice ball.
(i) The level of liquid on both side must remain same.

Total volume of liquid on right side after dipping the ball is

2 +
= 0
= 0 + …..(i)
2 2

Volume of water moving to right part


v = volume of submerged part of the ice ball = 90 cc
Answer is 45 cc
(ii) The water in the container is too little and the ice block cannot float. It must rest on the floor of the container.

Once again [as given by equation (i)] amount of water moving to right is .

But this time v = 50 cc.


Answer is 25 cc
(iii) When ice melts
In case (i) no change in water level will be seen as volume of water formed due to melting of ice ball = v
In case (ii) water level will rise as volume of water formed will be larger then v.
17. Because relative density of wood is 0.6, it can float in water with 60% of its volume remaining submerged.
The given wooden cylinder can float with its 6 cm length inside water.
When h = 8 cm, the wooden cylinder will certainly float.
Let the level of water in the container move up by y when the cylinder is placed inside it

F K
y Original level
J
E A 6 cm B

h = 8 cm
D C
8.22 Problems in Physics for JEE Advanced

(b) When h = 1 cm, the cylinder will just float with x = 0

6 cm

(c) With h < 1 cm, the cylinder cannot float


x = 0
18. Consider the tube + liquid inside it + Block as our system.
For equilibrium
Weight = Buoyancy
L  L
2 ρω vg + π r 2 . ρω g = v + π r 2  ρω g
2  2

19. (a) Buoyancy force is FB = vrg


Weight of the cone W = vdg
Tension T = W – FB = vg (d – r)
(b)
FB = vrg
Buoyancy is resultant of vertically upward force (F1) applied by water pressure on the base of the cone and the
vertically downward force (F2) applied by water on the slant surface. [The horizontal component of this force
sums up to zero due to symmetry]

H
F2

F1
F1 – F2 = FB
R2 . [r0 + rgH] – F2 = vrg
F2 = pR2 [r0 + rgH] – vrg
20. Let the sliding wall shift to left by a distance x when water is filled upto height H into the right chamber.

Fw

Fs
L+x
Fluids 8.23

Force by water on the movable wall (FW) must equal the spring force (Fs) in this position.
gH
To calculate the force applied by water on the wall we can assume that the average pressure is .
[This is because the pressure changes from zero to rgH linearly as we move from water surface to the bottom. You
are suggested to prove this by performing integration to calculate the force on the wall. Also, atmospheric pressure
has not been considered as its effect is on both the faces of the wall]

ρ gH ρ gbH
FW = ⋅ bH =

ρ gbH ρ gbH
kx = ⇒ x=
k

 ρ gbH 
Volume of water = Hb (L + x) = Hb  L + 
 k 
21. Pressure inside the freely falling container will become equal to atmospheric pressure. Water will not leak out of the
hole. The balloon will grow in size by
2.2 – 2.0 = 0.2 litre
Water level will rise.
−3 3
0.2 × 10 m
∆h = = 1 mm
0.2 m 2

22. For the stick to be in stable equilibrium, center of gravity should be below the center of buoyancy. For minimum m,
the two will just coincide.
Let h be the length of immersed portion. For translational equilibrium,
Weight of rod + mass attached = force of buoyancy
(M + m)g = R2hsg   ........…(i)
where M = R2Lr.

(M ) L / 2 + m × 0 ML
The height of center of mass from bottom = =
m+M 2(m + M )
For rotational equilibrium and for minimum m, this should be equal to h/2.

h ML
= ........…(ii)
2 2(m + M )    

ML
h=
(m + M )

L
2 h
C
m
8.24 Problems in Physics for JEE Advanced

Substituting for h in Equation (i), we get


( M + m) g = πR 2 σg. ML
(m + M )
(M + m)2 = R2sML
2 2 2
( M + m) = M πR σL = πR Lρ.πR σL
m = R2L sr – pR2Lr
 σ 
= πR 2 Lρ  − 1
 ρ 
23. As shown in figure, the forces acting on the rod are:
(1) The weight of rod 12 g acting downwards through the CG of the rod, i.e., at a distance of 3 m from the hinge.

w
B
R
3m q
W

(2) Force of buoyancy through the CG of displaced liquid vertically upwards.

Weight of displaced water


RD
Force of buoyancy

 5  12
Force of buoyancy =   × = 20
 6  0.5
and acts at a distance 2.5 m from the hinge.
(3) Extra weight w at the other end of the rod at a distance 6 m from O acting vertically downwards.
(4) Reaction R at the hinge at O will be vertical (as all other forces are vertical)
So for translational equilibrium of rod,
R+B–W–w=0
i.e., w – R = 20g – 12g = 8 g …(i)
And for rotational equilibrium of rod (taking moments about O)

6 5
−12 g × sin θ + 20 g × sin θ − w.6 sin θ = 0
2 2
Or, w = (14/6)g = 2.23gN = 2.33 kg …(ii)
Substituting the value of w from Eqn. (ii) in (i) and solving for R, we get
R = (2.33 – 8)g = – 5.67 kg
Negative sign implies that R is directed vertically downwards.
24. Consider a cylindrical tube of air as shown.
L

PF
PR
a
Fluids 8.25

Area of cross section = A



PR and PF are pressure at rear and front.
PR A – PF A = rAL.a

PR – PF = rLa
P = 1.2 × 3.0 × 5 = 18 Nm–2

∆ 18
= 5
= 1.8 × 10−4
0 1 × 10
25. Consider a volume of water at the location of the wooden block. The volume of water is of same shape and size as
the wooden block.
Mass of volume of water = 2m [Q density of water is double the density of wood]
Consider equilibrium of this water volume in the non-inertial frame of the container (fig.(a))

FP
2 ma
q

FR 2 mg

The resultant of the force of gravity and the pseudo force is FR. The force due to pressure of surrounding water,
FP balances FR.
g
tan θ = = 3 [Q a = g/3]
a
2 10
and FP = 2m a 2 + g 2 = mg
3
Force due to pressure of surrounding water on the wooden block is also FP since the wooden block and considered
volume of water are of same shape and size.
Considering equilibrium of the block in the frame attached to the container
g
tan θ = =3
a

and T + m a + g = FP

T = m a +g −m a +g

10
T = m a2 + g 2 =
mg
3
26. The liquid surface will get inclined to the horizontal by an angle q given by

P0

2R a=g
h0

q g
A
8.26 Problems in Physics for JEE Advanced

a
tan θ =
=1 q
g
Maximum vertical depth of liquid from free surface is

h0 = 2R sin 45º = 2 R
Pressure is maximum at A.
Pmax = P0 + rgh0 = P0 + 2 rgR

27. High density of streamlines at B indicates that speed is higher at B


Bernoulli’s equation gives
1 1
PA + ρVA 2 = PB + ρVB 2

2 2
1
PA − PB = ρ (VB2 − VA2 ) > 0
2
28. (i) The stream lines crowd near the sides of the ball. For the air to pass through the reduced width of the flow
channel it has to move faster. Air speeds up as it approaches the ball and then slows again as it departs at the
rear. At point 2 pressure is low due to high speed.
(ii) The spinning ball, to some extent, carries the air in the direction of the spin. This means that the flow velocity
on the side of the ball moving with the airflow is increased and hence pressure gets reduced on that side.

R
Vx V
x

Q
V0 =
2π Rh
R Q
Vx = ⋅ V0 =
x 2π hx

30.
A2
A1
v1 v2
1 2

2H + L

The required situation has been shown in the figure. Height of Hg column in right part of the tube

H = 2H + L = 34 cm
P1 – P2 = rHg . gH = 13.6 × 103 × 10 × 0.34 = 4.62 × 104 Nm–2
Fluids 8.27

Using Bernoulli’s equation

1 1
P1 + ρV12 + 0 = P2 + ρV22 + 0
2 2

1 1  Q2 Q2 
P1 − P2 = ρ (V22 − V12 ) = ρ  2 − 2
2 2  A2 A1 

[ Flow rate Q = A1V1 = A2V2]

2( P1 − P2 ) 2 × 4.62 × 104 2 × 4.62 × 1600


Q2 = = = = 146.7
 1 1   104 104  1.2 × (100 − 16)
ρ  2 − 2  1.2  2 − 2 
 A2 A1   4 10 

Q = 12.1 m3s–1
31. There is no speed of the liquid inside tubes. Speed at a point like 2 is zero.
1 2
Using Bernoulli’s equation between point 1 and 2 give P1 + ρ v1 = P2 + 0
2
A B

h2
h1

1 2
v

1 2
∴ P2 − P1 = ρ v [v1 = v and v2 = 0]
2
1 2 v2
∴ρ gh2 − ρ gh1 = ρv ⇒ h2 − h1 =
2 2g
32. Let the depth of the tube entrance be y. Consider point 1 (at the entry) and point 2 (at the maximum height of the
fountain) on a streamline. Apply Bernoulli’s theorem
1 2 1
P1 + ρ gh1 +
ρ v1 = P2 + ρ gh2 + ρ v22
2 2
2

h0

y
v
1

Consider h1 = 0 and P0 = atmospheric pressure


1 2
P0 + ρ gy + 0 +
ρ v = P0 + ρ g ( y + h0 + h) + 0 [Since speed at top point 2 is zero]
2
1 v2
∴ ρ v 2 = ρ g (h0 + h) ∴h =
− h0
2 2g
8.28 Problems in Physics for JEE Advanced

3.V

9
V1 = × 6 = 3 m /s
18
Applying Bernoulli’s equation for point 1 and 3

1 1
P1 + ρ Hg v12 = Patm + ρ Hg v 2
2 2

1
∴ ( P1 − Patm ) = ρ Hg ( 62 − 32 )
2

1
= × 1.36 × 104 × 27 = 1.84 ×105 Pa
2

2 V2 3
1

V1 V = 6 m/s

&

1 1
P2 + ρ HgV22 = Patm + ρ Hg .V 2
2 2
1
2 = 1.01 × 105 + × 1.36 × 104 (62 − 6.752 )
2

r
0.36 × 105
h= = 0.26 m = 26 cm
1.36 × 104 × 10

34. (a) (i) As for floating, W = FB


Vrg = V1d1g + V2d2g

Ê Aˆ Ê 3 ˆ Ê Aˆ Ê 1 ˆ Ê Aˆ 3 2 5
Or, L Á ˜ r = Á L ˜ Á ˜ d + Á L ˜ Á ˜ 2d i.e., r= d+ d= d
Ë 5¯ Ë 4 ¯ Ë 5¯ Ë 4 ¯ Ë 5¯ 4 4 4
(ii) Total pressure = P0 + (weight of liquid + weight of solid) / A

H H 5 A  1
i.e., P − P0 = dg + 2dg + d ×  × L  × g ×
2 2 4 5  A
3 1 1
i.e., P − P0 = Hdg + Ldg = (6 H + L)dg
2 4 4
(b) (i) By Bernoulli’s theorem for a point just inside and outside the hole
Fluids 8.29

1 1
1 + ρυ12 = 2 + ρυ22
2 2

H H  1
i.e., P0 + dg +  − h  2dg = P0 + (2d )υ2
2  2  2
2
or, g(3H – 4h) = 2 or υ = ( g / 2)(3H − 4h)

(ii) At the hole vertical velocity of liquid is zero. So time taken by it to reach the ground,
t ( 2h / g )

g 2h
so that x = υt = (3H − 4h) × = h(3H − 4h)
2 g
(iii) For x to be maximum x2 must be maximum, i.e.,
d 2 d
( x ) 0 or (3Hh − 4h 2 ) = 0
dh dh
Or, 3H – 8h = 0, i.e., h = (3/8)H
3H 3 3
and xmax = (3H − H ) = H .
8 2 4
35. Water pressure at height y is Pwater = rg(H – y)
For gas in the pipe

 H
Pgas ( Ay ) = ( ρ gH )  A    [Q P1V1 = P2V2]
 10 
ρ gH 2
∴ Pgas =
10 y
P

Pwater

Pgas
y
O y1 y2 H

(b) In equilibrium
Pgas = Pwater

ρ gH 2
= ρ g ( H − y)
10 y

10y2 – 10Hy + H2 = 0

H 15
y = ± H
2 10
8.30 Problems in Physics for JEE Advanced

H 15
Only y1 = − H is acceptable solution
2 10

H 15
At y2 = + H the equilibrium will be unstable
2 10
You can argue this looking at the graph plotted in part (a) of the problem
37. (i) Let height of water column be x.
Height of cubical block inside mercury = 10 – x
rwgx + rHg g(10 – x) = rm g.10

ρ Hg ρm
⇒ x + (10 − x) = 10
ρw ρw
x + (10 – x) × 13.6 = 10 × 7.3 x = 5 cm
Volume of water in the container
v = (15cm × 15 cm – 10 cm × 10 cm) × 5 cm = 625 cc
Mass water m = 625 × 1 = 625 g
38. Since tank is wide, the displaced water will spread in a thin layer on the surface. It means that water level in the tank
will not change. In equilibrium the block is completely submerged. It means that density of wood = density of water
h CM
CM

The COM of the wood falls by h and the COM of the displaced water rises by .
Heat produced = loss in PE
h 1
= ( ρ hA ) gh − ( ρ hA ) g = ρ gAh 2
2
39. Assume that half the cylinder (cut along its length) is held in the position shown
Buoyancy force on half cylinder
π r2
FB = L ρ g (↑)
2

P1A

Force on the flat surface at depth r is


F1 = P1A ( ) = (rgr) (2rL) ( ) = 2 rgLr2 ( )
Upward force on curved part due to water pressure is given by
π r 2 Lρ g 2 π 
F = FB + F1 = + 2 ρ gLr 2 = ρ gLr  + 2  (↑)
2 2 
Fluids 8.31

40. g eff = a + g and is directed as shown in the figure.


[Direction of acceleration of the tube is towards right]
a
q

geff g

a
tan θ = a = g tan q
g
The air bubble experiences a net buoyancy force opposite to geff.
41. Let the outer radius of the spherical pot be r
Buoyancy = Mg

2 3
π r ρ w g = (500) g ............(i)
3
3 452.1 × 3
∴ = = 216 r = 6 cm
2 × 3.14 × 1
If the sphere sinks further by h, the additional buoyancy is approximately
r2 h r.g = 5.g ..........(ii)
(ii) (i) gives -
∆h 5 2
= ×
r 500 3
2 4
∆ = ×6 = cm = 0.04 cm = 0.4 mm
3 × 100 100

42. Originally, half length   of the cylinder is inside water.


 
When the wooden cylinder is pushed down by x, water level in the container will also rise by x.
[Q cross section of container = 2A = twice that of cylinder]
Buoyancy force rises by A(2x) rog
To gradually push the cylinder, force F needed is F = 2Arogx
F
x
h/2
x{

Work done in pushing through dx


dW = Fdx = 2 A r0g x dx
h/4 1
Total work done W = 2 Aρ0 g ∫ x dx = Aρ0 gh 2
0 16
8.32 Problems in Physics for JEE Advanced

H
43. (a) COM of empty container is at x cm . When liquid is poured, the mass of the system increases on the lower
2
side and hence, the COM moves down. But the final position of COM when container is completely filled is
H
once again xcm . It means the COM begins to rise after
2
falling through a certain distance.
xcm

H
2

x
O

M
(b) Mass for unit length of liquid column is
H
Height of COM when liquid column has length x is
H M x
+ M x. 2 2

xcm = 2 2H 2 = 2H + x ........(i)
Mx 2[2 H + x]
M+
2H
d xcm
xcm is minimum when 0
dx
i.e., (2H + x) (2x) – (2H2 + x2) = 0
x2 + 4Hx – 2H2 = 0

−4 H ± 16 H 2 + 8H 2
∴x=
2
x cannot be negative
x0 = ( 6 – 2)H
When x = x0, the COM of the system is at lowest position
To find the minimum value of xcm you can put x = ( 6 – 2)H in equation (i)
44. After the ice melts, the level of water in the container remains unchanged. The COM is at geometrical centre ‘C’
after the ice melts.
Let upward direction be positive. In figure (a), height of COM above C is given by
water ice
− M ABDE y + M BDFG ( y + 0.05a)
ycm = water
M + M ice

G F
A E
O
y H
2
B D
C C
H/2

(a) (b)

Here y = height of COM of ABDE above C


And y + 0.05a = height of COM of complete ice block above C
Fluids 8.33

G
 0.9a 0.1a 
ice F
M  AEFG +  0.1a
ice  2 2  A E
ycm = y +
M ice
BDFG

= y + 0.1 × 0.5a = y + 0.05 a 0.9a

y
B D

ice
M BDFG × 0.05a
ycm = water ice  M ABDE
water ice
= M BDFG 
C
M + M BDFG
0.05
= = 0.01
4 +1

45. Because 90% of the volume of the ice is inside water, the relative density of ice must be 0.9.
(a) The ice block is pushed out of the water as the kerosene is poured. This is due to increased pressure at the bottom
of the ice block. After the ice block gets completely submerged, there is no change in pressure difference at
the bottom surface and the top surface. The ice block does not move after this. Solving part (b) and (c) of this
problem will further help you to understand this.
(b) Let A = area of cross section of the ice cylinder

H = height of the cylinder
F1 = force on top surface due to atmospheric pressure (Po)


F2 = force on the bottom surface due to water pressure
A
W = weight of the cylinder F1
For equilibrium -
K.oil x

0.8H water
w
r r r
ρk ρ F2
x + 0.8 H = ice H
ρw ρw

H
x
[Alternative way of writing equation (i) is thinking in terms of Archimedes principle]
Buoyancy = W
Wt. of water displaced + wt. of kerosene displaced = W
0.8H Arwg + xArkg = HAriceg
(c) Once again one can frame equation using any of the two methods given above.
Using Archimedes principle
Ah1rkg + Ah2rwg = A(h1 + h2)riceg
Solving, h1 = h2

h1
K.oil

h2 water
8.34 Problems in Physics for JEE Advanced

(c) Let v´ =

5
v′ρ w g = 1 × 10 − 100 ×
100
v´ × 104 = 5
5
v′ = 4 m3 = 500 cm3
10
When lift is accelerated geff = 15 m/s2
FB = W’ – kx2

6
v′′ ρ w g eff = 1 × 15 − 100 ×
100
v'' × 103 × 15 = 15 – 6

6 3
v′′ = m = 600 cm3
104
Change in submerged volume = 100 cm3
1
= π R2h
3
2
1 R h V
Volume of protruding part Vout = π   =
3 2 2 8
7
Volume of cone inside water in = − =
8 8
Imagine the cone to have that part of it which protrudes through the hole removed and the space under the container
filled with water. The buoyancy force would then be
7V ρ g
F0 = ρ g Vin =
8

R
As there is no water beneath the hole, a contribution π   ρ gh is missing
 

R2 7 3V V ρg
Actual buoyancy force is F = F0 − π ρ gh = V ρ g − ρg =
4 8 4
48. Let the density of the liquid be rL
v
ρ L g = v(0.6 ρ w ) g
2
rL = 1.2 rw = 1200 kg m –3
Liquid pressure at the upper surface of the tank is P = rLg [h1 + h2 + h3 – h] = 1200 × 10 × 12 = 144000 Nm–2
Fluids 8.35

Upward force applied by the liquid on the upper surface is



F = PA = 144000 × 3.14 × 22 = 1.81 × 106 N
Force by bolts is FB = F – Mg = 1810000 – 20000 = 179000 N
49. (i) The cube will topple when torque of buoyancy exceeds the torque due to gravity about the side passing through
point A.
FB
B
CM

A
x
Mg
a
FB ⋅ ≥ Mg ⋅ x

a3 a a
ρ ⋅ g ⋅ ≥ a 3 (2 ρ ) g ⋅ x ⇒ ≥ x
2 2
a
Since the cube just beings to topple when water reaches height , hence x .

(ii) (a) Let the dimension of the block perpendicular to the given figure be b. Consider a strip of width dh on the wall
of the block in contact with water (see figure)
Hydrostatic force on the strip is dF = rgh(b dh)
Torque of this force about rotation axis through A is dt = (z – h)dF

3
 z z
  z3 z3  z3 ρ gb  3  9
τ = ρ gb z ∫ h dh − ∫ h dh
2
= ρ gb  −  = ρ gb =  x = ρ gbx3
 0 0   2 3  6 6 4  128
Torque of weight of concrete block is
y y 5
τ g = Mg. = ( xyb)(2.5ρ ) g = ρ gbxy 2
2 2 4
In critical case tg = t
2
5 9  y 9 y 3
∴ ρ gbxy 2 = ρ gbx3 ⇒   = ⇒ =
4 128  x  32 × 5 x 4 10

h dF

dh

(b) when water is present below the block, there will be an upward force due to water pressure equal to
F = ( ρ gz )(by ) = 3 ρ gbxy
0
4
8.36 Problems in Physics for JEE Advanced

The force may be assumed to be effectively acting along the central line
y
In critical case τ = τ g − F°
9 3 5 2 3 2
ρ gbx = ρ gbxy − ρ gbxy
128 4 8
9 2 7 2
⇒ x = y
128 8
y 3
∴ =
x 4 7

∂P ∂P ∂P
50. = − ρ ax , = and = − ρ ( g + az )
∂x ∂y ∂z
Pressure is independent of y
Total differential of pressure P will be

 ∂P   ∂P 
dP =   dx +   dz = – raxdx – r(g + az)dz
 ∂x   ∂z 
P x z
∴ ∫ dP = − ρ ax ∫ dx − ρ ( g + az ) ∫ dz
P° 0 0

P – Po = – raxx – r(g + az)z


P = Po – rax·x – r(g + az)z
51. (a) Pressure at point 2 can be found by considering the motion of a cylindrical element of water as shown in the
figure.
P2S P1S

S – P1 S = SL ra P2 = raL [Q P1 = 0]

a
θ = tan −1  
g

4 line of constant pressure

L- h
3 a
L

L−h h a
= tan θ ⇒ − =
L L g

 a
h = L  − 
 g
Fluids 8.37

a0
q tan θ =
g

3m a0

5m

q
1
= 5 × 4 × + × (3 − )×5× 4 = 5× 4× 2
2
5x + 15 = 20 or x = 1 m
2 a 2
∴ tan θ = or 0 2g
5 g 5 ⇒ a0 = 5
(b) The acceleration of tank is a = 9 m/s2

A B C

3m a

y x

a 9.0 3 9.0
∴ tan θ = = or, or, x = 3.27 m.
g 9.8 9.8
1
= 3× y × 4 + × 3× x × 4 = 2 × 5× 4
or, 3y + 1.5x = 10 2
or, 3y = 10 – 1.5x = 10 – 1.5 × 3.27
or, y = 1.698 m
x + y = 4.698 m

P
S x
r
8.38 Problems in Physics for JEE Advanced

z
x

O x
v

Let us choose a co-ordinate system as shown. When height of water surface above O is z, the speed of efflux is
v gz

dQ
= vA0 = 2 gz ⋅ A0
If area of orifice is A0, the flow rate is
dt
Let x be the radius of cross section at height z. If water level falls by dz in interval dt then

dQ dz
=πx
dt dt

dz
−π x 2 = 2 gz ⋅ A0
dt

dz
It is desired that − = a constant = v0 (say)
dt

2 gz A0 = p x 2 ◊ v0

p 2 v02
z = kx4 where k =
2 gA02
54. Acceleration of the container down the incline is
q m q

ma


A
Horz

Liquid surface

mg Line parallel to
incline surface
Fluids 8.39

q
q a q a q m q a
tan a = m. a = tan–1 m

h1
P1
h2
L
P2
a

q m q

q
r r
P1 = rgh1 cos q + P0; P2 = rgh2 cos q + P0 [P0

r q r q r q r q m q
h2 − h1
m q q ⇒ =µ
L

mw 2 x
x y0

H 
O
mg

The resultant of centrifugal force and weight must be normal to the liquid surface. [Since liquid is non viscous and
equilibrium of the particle will not be possible if it experiences any tangential force].

mω 2 x dy ω x
tan θ = ⇒ =
mg dx g
8.40 Problems in Physics for JEE Advanced

y
ω x
∫ dy = ∫ xdx
o g o

ω
⇒ y= x
g

ω2 ω2
(1) =
2
y0 =
2g 2g

The volume of empty part of container = volume of paraboloid of revolution obtained by rotating the parabola
(1) about its axis. This paraboloid can be divided into horizontal discs and volume can be obtained by adding
the volumes of all such discs.
y0 2π g y0 p g 2 pw 2
∴ V = ∫ π x 2 dy =
ω 2 ∫0
ydy = y0 =
0
w2 4g

p p
pw 2
p (1) (2 ) = p (1) (3) -
2 2

4g

ω2
∴ =1 w 40 10
4

56. (a) For equilibrium

 3L  L
A   ( 2 ρω ) g + A ρω g = ALdg
 4  4

rw

7
fi r =d
4 w

7
=
ρω 4
(b) When depressed by x restoring force is
F = Ax (2 rw) g – Axrwg = Arwgx

d x
m = − Aρω gx
dt
d x d2 x Ê 4g ˆ
ALd = − Aρω g x fi = -Á x
dt dt 2
Ë 7 L ˜¯

4g 7L
ω12 = ⇒ T1 = 2π
7L 4g
Fluids 8.41

T1 7L
t1 = = π
2 4g

rw
d x 2 d2x  8g 
\ ALd = - 2 A r g x ⇒ 2
= − x
dt 2 w dt  7L 
8g 7L
ω2 = ⇒ T2 = 2π
7L 8g

T1 T  7L 7L  7L  1 
T = + 2 =π  +  =π 1 + 
2 2  4g 8g  4g  2

57. (i) Horizontal force on the cylinder due to two liquids must cancel out.

1 1
⇒ 2 ρ gh.hL 3 gR.RL
2 2

3
⇒h= R
2

O A C


r
R

È p R2 1 R 3R ˘ 
2 π 3
= b Í - ˙ r g = b ρ gR  − 
ÍÎ 6 2 2 2 ˙˚ 6 8 
B

= 10 × 103 × 10 × 202  3.14 − 1.732  = 1.2 ×107N


 6 8 

=
1 
= =  ρ gh  ( bh )
2 

1 3  3   3 
=  ρ g R b R Q h = AB = R
2 2   2   2 

3
× 103 × 10 ×10 × ( 20 ) = 1.5 × 10 N
2 7
=
8

F= FH2 + FV2
8.42 Problems in Physics for JEE Advanced

w
P1 + rHg gh0 = P2 + rwg hw
P1 – P2 = rwghw – rHg gh0
P0 = rwghw – rHg gh0

A1 x .
A2
A1  A 
∆ h0 = x + x = 1 + 1  x
A2  A2 
rw w r
Ê A ˆ
= 0 - r Hg g D h0 = r Hg g Á 1 + 1 ˜ x
Ë A2 ¯

A2
∴ x= ∆h
A1

 A A ∆ P0  A 
∴ ∆ P0 = ρ Hg .g 1 + 1  2 ∆h ∴ = ρ Hg .g 1 + 2 
 A2  A1 ∆h  A1 

∆h 1
∴ =
∆ P0  A2 
1 +  ( ρ Hg .g )
 A1 

9A P1
A P2

x
h2

h1

q q
r r
x  1 
= 0.74 ρω g  + x sin θ  = 0.74 ρω g x  + sin θ 

9  9 
rw

1 
∴ 9 × 10−5 × ρω = 0.74 ρω × 5 ×10−4  + sin θ 
 9 
Fluids 8.43

0.9 1
= + sin θ
0.74 × 5 9
sin q = 0.24 – 0.11 = 0.13
q = sin–1 (0.13)

F2 .
A1
A2
 
Q
1
= 5
 2 

F2 a 4000 ¥ 4
\ F= = = 400 N
a+b 4 + 36
61. A tangent on a streamline gives the direction of flow velocity.
Vy
S
Vx
dy by
= −
dx u + bx

dy dx

⇒ ∫ y
=−b∫
u + bx ⇒ n y = −n ( u + bx ) + c

c nc a
c
ny = − n ( u + bx ) + n co ⇒ n y = n
u + bx
c
⇒ y =
u + bx


y

2gy

8.44 Problems in Physics for JEE Advanced

y
dy
H

h p
Rate (volume per unit time) at which the liquid flows out through the strip is
= vA = 2phr 2g y dy

H 2  32  H 4 3

πη r g ∫ y dy = 2 πη r 2g y = πη r 2g H 2
= 3   O
O 3

4 3
Q= πη r 2 g H 2
3
V0 40
h= = = 5 cm
A 8

a 2 mm 2
fiu= u0 = ¥ 2 m /s = 0.005 m /s u0 = 2 m/s
A 8 cm 2
1

1 1
P0 + ρ u02 + ρ gh = P + ρ u 2 h
2 2
P u
2
1 1
⇒ P = P0 + ρ u02 − ρ u 2 + ρ gh P0
2 2 F
Mg
For piston to go up at constant speed
1
P0A + F = PA + Mg ⇒ F = ρ A ( u02 − u 2 ) + Aρ g h + Mg
2


0
1 1
W = − ∫ Fdh = ρ A ( u02 − u 2 ) h + Aρ gh 2 + Mgh
h 2 2

1 1
= ρ A u02 h + A ρ gh 2 + Mgh Q 2
0 >> 2

2 2
= 0.115 J
Fluids 8.45

Alternate
W = Increase in KE of water + Increase in potential energy of water + Increase in potential energy of piston.
1 h
= ρ Ah ( u02 − u 2 ) + ρ Ahg + Mgh = 0.115 J
2 2

v = 2 gh = 2 × 10 × 2 = 6.32 ms −1

3.14 × ( 0.01)
2
π d 02 .
Q = v = × 6.32 = 4.96 × 10−4 m3 s −1
4 4

2gh

P1 v2 P v2
+ 1 + h1 = 2 + 2 + h2
ρ g 2g ρ g 2g

v22
(10 m ) + 0 + ( 2m ) = ( 2.5m ) + +0
2g
∴ v22 = 9.5 × 2 g
∴ v2 = 9.5 × 2 × 10 = 13.78 ms −1

( 0.01)
2
d2
Q = π 0 v2 = 3.14 × × 13.78 = 1.08 × 10−3 m3 s −1
4 4
πd2 .
Q= v
4
3.14
∴ 1.08 × 10−3 = × 6.32 × 2

4
d2 = 2.18 × 10–4 fi d = 1.48 × 10–2 m = 1.48 cm
65.
3 3

H
H

1 1
h h
2 2

(a) Applying Bernoulli’s equation in both cases between point 3 and 2 gives speed at point 2 as v = g ( H + h)

v1 2 gH

gH H
= =
g ( H + h) H +h
8.46 Problems in Physics for JEE Advanced

(c) In first figure P1 = P0 =

1
P1 + ρ v12 = P0 + ρ gH
2

r r Q v1 = 2 g ( H + h ) 
 
= P0 – rgh
DP = rgh

1
ρ v 2 + Patm = Pgas
2
1
× 103 × 2
= ( 4 − 1) ×106 fi v2 = 6 × 103
2

dm
= ρ Q = ρ Av
dt
 dm 
Fth = v   = ρ Av = 10 × 6000 × A = 6 × 10 A
3 6

 dt 

Fth = m (a + g) = 2 × (0.5g + g) = 3g
6 × 106 A = 3 × 10
1
A= ×10−5 m 2 = 5 × 10−6 m 2 = 5 mm 2
2

2 gh

dm
= ( uA ) ρ
dt
dv Ê dm ˆ
m = uÁ
dt Ë dt ˜¯
m = Ah r =

dv
\ Ah r = u2 A0 r
dt
dv
Ah = A0 ( 2 gh )
dt
dv 2 A0 g
∴ = = a
dt A
Fluids 8.47

r r
dh A0 u A0
⇒ dt = − A = − A 2 gh

dv 2 A0 g
dt A

dv dh 2 A0 g dv  A0  2A g
⇒ = ⇒  − 2 gh  = 0
dh dt A dh  A  A
dv 1
∴ = − 2g
dh h
v 0
dh
∴ ∫ dv = − 2 g ∫
0 H h

2 gh

Kx
. Pgage =
πR

1 2
P= ρ v + P0
2
1 2
Pgage = P − P0 =ρv
2
2    
Kx 1 Q  Q
∴ = ρ  

π R2 2  bx  QV = bx 
 

3 2K
∴ Q = bx 2

ρπ R 2

Applying Bernoulli’s equation between two points of a streamline at the constriction and at the open end of the
tube -

1 1
ρV22 + P2 = ρV12 + P0
2 2
1
∴ P0 − P2 = ρ (V22 − V12 )
2

1  Q2 Q2 
ρ0 gH = ρ − 
2  A22 A12 

−1 −1

2 ρ0 gH  1 1  2 ρ0 gH  1 1 
∴ Q2 =  2 − 2 ⇒ Q =  2 − 2
ρ A
 2 A 1  ρ A
 2 A 1 
8.48 Problems in Physics for JEE Advanced

(ii) Work done on small element of liquid by neighbouring liquid = Change in KE of element + Change in
gravitational PE of the element.
1
W = ∆K + ∆U = ρ∆V ( v22 − v12 ) + ρ∆Vg ( h2 − h1 )
2

1  A2 
= ρ∆V  12 − 1 v12 + ρ∆Vg ( h2 − h1 )
2  A2 
70.
A P
X
VA =V0 O

Let us consider a point P whose x co-ordinate is –x. Apply Bernoulli’s equation between P and A.
Ê 2 ˆ Ê rV 2 ˆ
rV
Pp + Á ˜ + 0 = P A + Á 2 ˜ +0
Ë 2 ¯P Ë ¯A

ρV 2 ρV02
P+
= P0 +
2 2
ρ 2   R3  
2
ρ 2
∴ P = P0 + (V0 − V ) = P0 + V0 1 − 1 + 3  
2

2 2   x  

ρV02
Æ ; P → Po +
2
dP

dx
P 2

P0 + V0
2

-R P0 X

71.
A2
A1
1

h1 2 h2

P1 V2 P V2
+ 1 + h1 = 2 + 2 + h2
ρg 2g ρ g 2g
P0 P + ρ gh2 V22
+ 0 + h1 = 0 + + 0 [P0 =
ρg ρg 2g
V22
∴ h1 = h2 + ⇒ V2 = 2 g ( h1 − h2 )
2g
Fluids 8.49

\ 2gh 
2gh

\
dh1 dh
Q = − A1 = A2 2
dt dt
fi Q dt = – A1 dh1 = A2dh2
A1 A2 dh
−dh1 − dh1 = −dh ∴ dh1 = A + A
A2 1 2

A1 A2 dh ⇒ A A1 A2
∴ Qdt = − 0 2 gh dt = − dh
A1 + A2 A1 + A2
t
A1 A2 0 dh
∫ dt = − ∫
0 A0 ( A1 + A2 ) 2 g h
h

2 A1 A2 2 A1 A2
t = h = h1 − h2
2 g A0 ( A1 + A2 ) g A0 ( A1 + A2 )

3
72. (a)

1 H

2
V

1 1
P1 + ρV12 + ρ gh1 = P2 + ρV22 + ρ gh2
2 2
1
P0 + 0 + ρ gh = P0 + ρV 2 + 0 [V1 = 0]
2
\ V = 2gh 

(b) Applying Bernoulli’s theorem between point 1 and 3–
1
P0 + 0 + ρ gh = P3 + ρV 2 + ρ gH
2
1
P′ = P0 + ρ gh − ρV 2 − ρ gH = P0 – rgH
2

The pressure at point 3 can also be obtained by using Bernoulli’s theorem between point 3
and 2.
8.50 Problems in Physics for JEE Advanced

P0
H0 =  10.3 m
ρg

'
s s fi s s s

s – s = ds

\ s s

σ
dσ V
⇒ V0
∫ σ −1
σ0
=− ∫ 0
dVw

σ  σ −1 
⇒ ln (σ − 1) σ = − ⇒ ln   = −
 σ0 −1
0
0 0

fi s = 1 + (s0 – 1)e–V/V0
s

(c)
s

s0

1.0

O V

r
Ø ­­­­­­­­­­­­­­­­
F = P. pR2 = r.g.pR3


2
ρ .g.π R 3 = ρ . π R 3 g + mg

3

πR ρ
∴ m =

75. Vertical force on the plate due to water


π R2
=ρ. Lg
4
Fluids 8.51

r gR 1
= ¥ RL = r gR 2 L
2 2
q Fv π
tan θ = = .
FH 2
Cable
R
t q O
 2T

R
t d
\ q
C F
F 2 2 1
T = cos θ = Fv + F H .
2 2

1 π2 1
= ρ gR 2 L 1 + = ρ gR 2 L 4 + π 2
4 4 8

4
V = π R3 =
3

V V
V r0 g = r g + (2 r ) g
2 2
3 2ρ
⇒ ρ0 = ρ⇒ρ= 0
2 3

 3R   3R 
2M  − M  
 8   8 = R
x0 =
2M + M 8
3
Q
8

FB
M 

O
O X0 X0
CM
2M CM

W
8.52 Problems in Physics for JEE Advanced

2 2 6
I 0 = MR 2 + ( 2 M ) R 2 = MR 2
5 5 5

6 R 2 369
∴I= MR 2 − 3M = MR 2
5 64 320

\ a q
369 R
∴ MR 2 . α = − FB θ [for small q, sin q = q]
320 8

Q FB = 3Mg ∴ α = −  40 g  θ
 123 R 
40 g 123R
∴ ω= ⇒ T = 2π
123 R 40 g

77. (a) If rate of fluid flow is Q then
2 2
1  Q  nP0 1 Ê Q ˆ
ρ 2  = + rÁ 2 ˜
2  π r1  2 2 Ë p r2 ¯
nP0
\ Q = p r12 r2 2
{
r r14 - r2 4 }
m
F F F
N= = PA ∴ P = =
µ A µ 2π r1 b µ

1 1
PA + ρ VA 2 = PB + ρVB2
2 2

1 È 1 1 ˘
nP0 - P0 = r Q2 Í - 2 2 ˙
2 (
Î 2p r 2 2
) (p r1 ) ˚

2(n - 1)P0
Q = p r12 r22
r[r14 - r24 ]
09 SURFACE TENSION

What is the contact angle between the liquid


Level 1 and the container wall?
Q.1. A circular ring has inner and outer radii equal to
10 mm and 30 mm respectively Mass of the ring
is m = 0.7 g. It gently pulled out vertically from
a water surface by a sensitive spring. When the
spring is stretched 3.4 cm from its equilibrium
position the ring is on verge of being pulled Q.4. A conical pipe shown in figure has a small water
out from the water surface. If spring constant is drop. In which direction does the drop will tend to
k  =  0.7 Nm–1 find the surface tension of water. move?

Q.5. A narrow tube of length l and radius r is sealed


at one end. Its open end is brought in contact
with the surface of water while the tube is held
vertical. The water rises to a height h in the tube.
water The contact angle of water with the tube wall
Q.2. A long thin walled capillary tube of mass M and is q, density of water is r and the atmospheric
radius r is partially immersed in a liquid of surface pressure is Po. Find the surface tension of the
tension T. The angle of contact for the liquid and liquid. Assume that the temperature of air inside
the tube wall is 30°. How much force is needed to the tube remains constant and the volume of the
hold the tube vertically? Neglect buoyancy force meniscus is negligible.
on the tube. Q.6. The internal radius of one arm of a glass capillary
U tube is r1 and for the second arm it is r2(> r1).
The tube is filled with some mercury having
surface tension T and contact angle with glass
equal to 90° + q.
(a) It is proposed to connect one arm of the U
Q.3. (i) Water drops on two surfaces A and B has been tube to a vacuum pump- so that the mercury
shown in figure. Which surface is hydrophobic level in both arms can be equalized. To which
and which surface is hydrophilic? arm the pump shall be connected?
(b) When mercury level in both arms is same,
how much below the atmospheric pressure is
the pressure of air in the arm connected to the
A B pump?
(ii) A liquid is filled in a spherical container of Q.7. In a horizontal capillary tube, the rate of capillary
radius R till a height h. In this position the flow depends on the surface tension force as
liquid surface at the edges is also horizontal. well as the viscous force. Lueas and washburn
9.2 Problems in Physics for JEE Advanced

showed that the length (x) of liquid penetration density of water is r = 103 kg/m3 and atmospheric
in a horizontal capillary depends on a factor pressure is Po = 105 N / m2
(k) apart from time (t). The factor is given by r
1
È rT cosq ˘ 2
k=Í ˙ ; where r, T, q and h are radius
Î 2h ˚
of the capillary tube, surface tension, contact
angle and coefficient of viscosity respectively. If
the length of liquid in the capillary grows from h
zero to x0 in time t0, how much time will be
needed for the length to increases from x0 to 4x0.
Q.8. A glass tube of radius R is covered with a liquid
Find the excess pressure to be applied on the
film at its one end. Air is blown slowly into the
water in the capillary tube so that -
tube to gradually increase the pressure inside.
What is the maximum pressure that the air inside (a) The water level in the tube becomes same as
the tube can have? Assume that the liquid film that in the vessel.
does not leave the surface (whatever its size) and (b) Is it possible to blow out an air bubble out of
it does not get punctured. Surface tension of the the tube by increasing the pressure?
liquid is T and atmospheric pressure is Po. (ii) A container contains two immiscible liquids
of density r1 and r2 (r2 > r1). A capillary of
radius r is inserted in the liquid so that its bottom
Liquid film
reaches up to denser liquid and lighter liquid does
not enter into the capillary. Denser liquid rises
in capillary and attain height equal to h which
air is also equal to column length of lighter liquid.
Assuming zero contact angle find surface tension
of the heavier liquid.
Q.9. Why bubbles can be formed using soap water
but we do not have bubbles formed out of pure h
water?
Q.10. A tapering glass capillary tube A of length 0.1 m
has diameters 10–3 m and 5 × 104 m at the ends.
Q.12. The radii of two columns in a U tube are r1 and
When it is just immersed in a liquid at 0ºC with
r2 (r1 > r2). A liquid of density r is filled in it.
larger radius in contact with liquid surface, the
The contact angle of the liquid with the tube
liquid rises 8 × 10–2 m in the tube. In another
wall is q. If the surface tension of the liquid is
experiment, in a cylindrical glass capillary tube
T then plot the graph of the level difference
B, when immersed in the same liquid at 0ºC, the
(h) of the liquid in the two arms versus contact
liquid rises to 6 × 10–2 m height. The rise of liquid
angle q. Plot the graph for angle q changing from
in tube B is only 5.5 × 10–2 m when the liquid is
0° to 90°. Assume the curved surface of meniscus
at 50ºC. Find the rate at which the surface tension
to be part of a sphere.
changes with temperature considering the change
to be linear. The density of liquid is (1/14) × 104
kg/m3 and the angle of contact is zero. Effect of
temperature on the density of liquid and glass is
h
negligible.
Q.11. (i) One end of a uniform glass capillary tube
of radius r = 0.025 cm is immersed vertically
in water to a depth h = 1cm. Contact angle is
0°, surface tension of water is 7.5 × 10–2 N/m,
Surface Tension 9.3

that the excess pressure inside a bubble is small


Level 2 compared to P0.
Q.13. A glass prism has its principal section in form of
an equilateral triangle of side length l. The length
of the prism is L (see fig.). The prism, with its r
base horizontal, is supported by a vertical spring
of force constant k. Half the slant surface of the
R
prism is submerged in water. Surface tension of
water is T and contact angle between water and Q.16. In the siphon shown in the figure the ends A and
glass is 0°. Density of glass is d and that of water B of the tube are at same horizontal level. Water
is r ( < d). Calculate the extension in the spring in fills the entire tube but it does not flow out of the
this position of equilibrium. end B. With the help of a diagram show how the
water surface at end B changes if the end B were
slightly lower than the position shown.

L
l/2

60°
l A B

Q.14. Two capillaries of small cross section are


connected as shown in the figure. The right tube Q.17. A glass capillary tube sealed at the upper end has
has cross sectional radius R and left one has a internal radius r. The tube is held vertical with its
radius of r (< R). The tube of radius R is very long lower end touching the surface of water. Calculate
where as the tube of radius r is of short length. the length (L) of such a tube for water in it to rise
Water is slowly poured in the right tube. Contact to a height h (< L). Atmospheric pressure is Po
angle for the tube wall and water is q = 0°. Let h and surface tension of water is T. Assume that
be the height difference between water surface in water perfectly wets glass (Density of water = r)
the right and left tube. Surface tension of water is Q.18. In the last question let the length of the tube be
T and its density is r. L and its outer radius be R. Water rises in it to
a height h. Calculate the vertical force needed to
hold the tube in this position. Mass of empty tube
h is M.
Q.19. A glass capillary tube is held vertical and put into
contact with the surface of water in a tank. It was
observed that the liquid rises to the top of the tube
before settling to an equilibrium height h0 in the
tube. Assume that water perfectly wets glass and
viscosity is small. Is the length of the capillary
tube larger then 2h0?
(a) Find the value of h if the water surface in the
Q.20. Two soap bubbles of radii r1 and r2 are attached
left tube is found to be flat.
as shown. Find the radius of curvature of the
(b) Find the maximum value of h for which water common film ACB.
will not flow out of the left tube . A
Q.15. A soap bubble of radius r is formed inside another
soap bubble of radius R (> r). The atmospheric r1 C r2
pressure is P0 and surface tension of the soap
solution is T. Calculate change in radius of the B
smaller bubble if the outer bubble bursts. Assume
9.4 Problems in Physics for JEE Advanced

Q.21. (a) In the last question find the angle between the
tangents drawn to the bubble surfaces at point Level 3
A. Q.24. Consider a rain drop falling at terminal speed. For
(b) In the above question assume that r1 = r2 = r. what radius (R) of the drop can we disregard the
What is the shape of the common interface influence of gravity on its shape? Surface tension
ACB? Find length AB in this case. and density of water are T and r respectively.
(c) With r1 = r2 = r the common wall bursts and Q.25. A soap bubble has radius R and thickness of its
the two bubbles form a single bubble find wall is a. Calculate the apparent weight (= true
the radius of this new bubble. It is given that weight - Buoyancy) of the bubble if surface
volume of a truncated sphere of radius R and tension of soap solution and its density are T and
p
height y is y 2 (3R - y ) [see figure] d respectively. The atmospheric pressure is P0
3 and density of atmospheric air is r0. By assuming
a  =  10–6 m, R = 10 cm, P0 = 105 Nm–2, r0 = 1.2
kg  m–3, d = 103 kg m–3, T = 0.04 Nm–1; show that
y the weight of the bubble is mainly because of
R
water in the skin. What is weight of the bubble?
Q.26. A soap bubble is blown at the end of a capillary
tube of radius a and length L. When the other end
Q.22. Two soap bubbles of radius R1 and R2 (< R1) are
is left open, the bubble begins to deflate. Write
joined by a straw. Air flows from one bubble to
the radius of the bubble as a function of time if
another and a single bubble of radius R3 remains.
the initial radius of the bubble was R0. Surface
tension of soap solution is T. It is known that
volume flow rate through a tube of radius a and
length L is given by Poiseuille’s equation-
R2 p a 4 DP
Q=
8h L

R1 L R0
2a

(a) From which bubble does the air flow out ?


(b) Assuming no temperature change and
atmospheric pressure to be Po, find the Where DP is pressure difference at the two ends of
surface tension of the soap solution. the tube and h is coefficient of viscosity. Assume
Q.23. In the last problem, one of the bubbles supplies that the bubble remains spherical.
its entire air to the other bubble and a film of soap Q.27. Two blocks are floating in water. When they are
solution is formed at the end of the straw which brought sufficiently close they are attracted to
keeps it closed. What is the radius of curvature each other due to surface tension effects. When
of this film if the bigger bubble has grown in size the experiment is repeated after replacing water
and its radius has become R3. with mercury, once again the two blocks are
attracted. Explain the phenomena. It is given that
water wets the material of the block where as
Film of soap mercury does not.
solution
Q.28. A long thin string has a coat of water on it. The
radius of the water cylinder is r. After some time
it was found that the string had a series of equally
R3 spaced identical water drops on it. Find the
minimum distance between two successive drops.
Surface Tension 9.5

Q.29. A liquid having surface tension T and density r B


is in contact with a vertical solid wall. The liquid
surface gets curved as shown in the figure. At the
V
bottom the liquid surface is flat.
The atmospheric pressure is Po.
h
A
C
h M

L
A

h
(i) Find the pressure in the liquid at the top of the
meniscus (i.e. at A)
(ii) Calculate the difference in height (h) between h0
the bottom and top of the meniscus.
Q.30. Is it possible that water evaporates from a
spherical drop of water just by means of surface
energy supplying the necessary latent heat of
vaporisation? The drop does not use its internal
thermal energy and does not receive any heat
t
from outside. It is known that water drops of
size less than 10–6 m do not exist. Latent heat of Q.32. A curved liquid surface has radius of curvature R1
vaporisation of water is L = 2.3 × 106 Jkg–1 and and R2 in two perpendicular directions as shown
surface tension is T = 0.07 Nm–1. in figure. Surface tension of the liquid is T. Find
the difference in pressure on the concave side and
Q.31. In the arrangement shown in the figure, A is a jar
the convex side of the liquid surface.
half filled with water and half filled with air. It
is fitted with a leak proof cork. A tube connects
Liquid surface
it to a water vessel B. Another narrow tube
fitted to A connects it to a narrow tube C via
a water monometer M. The tip of the tube C is
R2 R1
just touching the surface of a liquid L. Valve V
is opened at time t = 0 and water from vessel B
pours down slowly and uniformly into the jar A.
An air bubble develops at the tip of tube C. The
cross sectional radius of tube C is r and density of Q.33. A capillary tube of radius r and height h1 is
water is r. The difference in height of water (h) in connected to a broad tube of large height as shown
the two arms of the manometer varies with time in the figure. Water is poured into the broad tube –
't' as shown in the graph. Find the surface tension drop by drop. Drops fall at regular intervals. Plot
of the liquid L. the variation of height of water in both tubes with
time. Initially the tube and capillary are empty.
Neglect the volume of the connecting pipe.

h1
9.6 Problems in Physics for JEE Advanced

ANSWERS

1. 0.076 Nm–1 È L r2 ˘
2. 2 3prT + Mg 18. Mg + p P∞ Í R 2 - ˙ + 2p ( R + r )T
ÍÎ L - h ˙˚
3. (i) A Æ hydrophilic , B Æ hydrophobic 19. No, l < 2h0
Ê R - hˆ
(ii) cos-1 Á r1r2
Ë R ˜¯ 20.
r1 - r2
4. Towards the tapered end. 3r
21. (a) 120°  (b) 3r   (c)
r È P∞ h ˘ 2(2)1/ 3
5. T= Í + r gh ˙
2 cosq Î  - h
6.
˚
22. (a) From smaller bubble (b) T =
(
P∞ R33 - R13 - R23 )
4 (R )
(a) To capillary of smaller radius 2
1 + R22 - R32
2T sin q (r2 - r1 )
(b) 23. R3
r1r2
T
7. 15t0 24. R <<
4T rg
8. P∞ +
R 16p R 2 r∞
N 25. g + 4p R 2 a.d.g
10. -1.4 ¥ 10 -4 3 P∞
m∞C 1
r È a 4Tt ˘ 4
11. (i) (a) 600 Pa (b) Yes. (ii) T = ( r2 - r1 ) gh 26. R = R0 Í1 - ˙
2 ÍÎ 2hL R04 ˙˚
12. h
9
28. r
2
2T
29. (i) Po – rgh (ii)
rg
q
O° 90° 30. No
È 3 2
1 3 3 2 ˘ rgh0 r
13. x = Í l L d.g - l L r g + 3TL + Tl ˙ 31.
ÍÎ 4K 16 ˙˚ 2
2T 2T Ê r + R ˆ Ê 1 1ˆ
14. (a) h = (b) Á ˜ 32. DP = T Á + ˜
Rr g r g Ë rR ¯ Ë R1 R2 ¯
4Tr h
15. Dr = 33. Tube
3P0 R
16. The radius of curvature decreases h1
Capilary
P∞ hr h0
17. L = +h
2T - r grh
t
t0 t1 t2
Surface Tension 9.7

SOLUTIONS

1. When the ring is about to leave the water surface, surface tension force on it is
FST = 2pRT + 2prT = 2p(R + r)T
Spring force FS = kx
\ kx = 2p(R + r)T + mg
kx - mg 0.7 ¥ 3.4 ¥ 10 -2 - 7 ¥ 10 -4 ¥ 9.8
\T = = = 0.076 Nm–1
2p ( R + r ) 2 ¥ 3.14 ¥ (30 + 10) ¥ 10 -3
2. Force = Mg + force due to surface tension on the inner wall + force due to surface tension on the outer wall
= Mg + 2prT cos q + 2prT cos q = Mg + 2 3prT

q
T

 

T T
Mg
3. (i) The correct contact angle (q) has been shown in figure
Tl
Tl

q q
TS A TS B
Hydrophobic
Hydrophilic

If q is acute the surface is hydrophilic (i.e. water wets the surface) and if q is obtuse the surface is hydrophobic.
(ii) Draw a tangent on the container wall at point of contact. Angle between this tangent and the liquid surface is
the contact angle.
4. Pressure difference on two sides of a curved surface is inversely proportional to the radius of curvature.
5. PA = P° – rgh
2T
Pressure at B is higher than at A by cosq B
r
l
2T cosq 2T cos q A
\ PB = PA + = P∞ - r gh + P0 h
r r
P1V1 = P2V2 for air inside the tube

È 2T cos q ˘
P∞ A = Í P∞ - r gh + ˙ A( - h )
Î r ˚
2T cosq P Ph
fi = ∞ - P∞ + r gh = ∞ + r gh
r -h -h
9.8 Problems in Physics for JEE Advanced

Ê 1 1 ˆ r È P∞ h ˘
DP = T Á + ˜ fi T= Í + r gh ˙
Ë R1 R2 ¯ 2 cosq Î  - h ˚
r
6. Relation between radius of curvature (R) of the Hg surface and tube radius (r) is = sinq
R
Pump r
P

P 
2 
R

P1 = P2
2T sin q 2T sin q Ê 1 1ˆ Êr -r ˆ
P+ = P∞ + ; \ P∞ - P = 2T sinq Á - ˜ = 2T sinq Á 2 1 ˜
r1 r2 Ë r1 r2 ¯ Ë r1r2 ¯
È - ˘
1
One can easily show that dimension of the factor k are: [ k ] = Í M ∞L1T 2 ˙
7.
ÍÎ ˙˚
1
If x = kta, then a =
2
1
\ x µ t2
Time needed for x to go from zero to x0 is t0
Time for x to grow from zero to 4x0 will be 16t0
\ required answer is 15t0
9. When water sprays from a tap in a small container, one can see bubbles, but they burst very soon. This is because
surface tension of water is high and it tends to draw water molecules into the bulk, to the point where thickness of
bubble wall is too thin to remain intact. On the other hand, the surface tension of soap water is much lower. The
molecules of the bubble are less stressed and can last longer.
10. The situation is shown in figure.
Let r1 and r2 be radii of upper and lower ends of the conical capillary tube. The radius r at the meniscus is given by
Ê l - hˆ
r = r1 + (r2 - r1 ) Á
Ë l ˜¯
Ê 0.1 - 0.08 ˆ
= (2.5 ¥ 10 -4 ) + (2.5 ¥ 10 -4 ) Á ˜
Ë 0.1 ¯
r1

l h
r2

0°C
Surface Tension 9.9

rh r g (3.0 ¥ 10 -4 )(8 ¥ 10 -2 )(1 / 4 ¥ 10 4 )(9.8)


T0 = =
2 2
T0 h 6 ¥ 10 -2 12
For tube B; = 0 = - 2
=
T50 h50 5.5 ¥ 10 11

11 11
T50 =
Or, ¥ T0 = ¥ 0.084 = 0.077 N /m
12 12
Considering that change in surface tension as linear, the change in surface tension with temperature is given by
T - T0 0.077 - 0.084 N
a = 50 = = -1.4 ¥ 10 -4
50 50 m∞C
11. (i) (a) P1 = P2
T
P° + ∆P − = P°
R

P0+DP
P0
1 2

T
[R = radius of curvature of the curved surface. Pressure on convex side is lesser by than pressure on
R
concave side]
For q = 0°; R = r
2T 2 × 7.5 × 10−2
∆P = = = 600 Pa
r 2.5 × 10−4
2T
(ii) P0 + ρ1 gh − ρ 2 gh + = P0
r
r
T = ( ρ 2 − ρ1 ) gh
2
12. P1 = P3
2T 2T 2T
P° − = P2 + ρ gh ⇒ P° − = P° − + ρ gh
R1 R1 R2
r1 r2

3
1

r
R1 & R2 are radii of curvature of the meniscus and it is known that R =
cos θ
1 1 2T cos θ  1 1 
ρ gh = 2T  −  ( h=  − 
R
 2 R1 ρ g  r2 r1 
9.10 Problems in Physics for JEE Advanced

2T  r1 − r2 
h=   cos θ \ h µ cos q
ρ g  r1 r2 
h


O 90
FS
3 2 FB
13. Volume of prism , V l L
4
2
3 2 3l  3 3 2
Volume of submerged part, V ' = l L−   L l L
4 4 2 16
l/2
FST FST
FST

W
For equilibrium -
Spring force (FS) + Buoyancy (FB) = weight (W) + surface tension force (FST)

3 3 2 3 2 l
kx + l L ρg = l L dg + 2.TL cos 30° + 2T
16 4 2

1  3 2 3 3 2 
x =  l L d .g − l L ρ g + 3TL + Tl 
K 4 16 
14. When water on two sides has same level (with left tube completely filled) the radius of curvature of the water
surface on two sides will be same - equal to R. As more water is added, the surface in the left tube gets flatter, then
becomes completely flat and then becomes convex up.
Figure shows change in curvature of the left surface as height of water in the right tube increases.

In extreme case left surface is hemispherical (radius = r ) with convex side up. After this, water starts flowing out of
the tube.
(a) The meniscus in the right tube is hemispherical (because q = 0°) . Radius of curvature of the surface is = R

1
2
Surface Tension 9.11

When surface on left is flat, pressure at point 1 (just below the surface) is atmospheric pressure (Po)
P1 = P2
T T
P° = P° − + ρ gh ⇒h=
R Rρ g
(b) In extreme case, when water does not come out of left tube, the surface gets convex with radius of curvature r
P1 = P2
T T T r + R
P° + = P° − + ρ gh ⇒ =h
r R ρ g  rR 

3
h
1
2

15. P0 = atmospheric pressure


4T
P0 + = pressure inside the larger bubble.
R
4T 4T
P0 + + = pressure inside the smaller bubble.
R r
4T
After the larger bubble bursts, the new pressure inside the smaller bubble is = P0 +
r
Using P2V2 = P1V1

 4T   4T 4T  4 3
 P0 +  V2 =  P0 + +  πr
 r   R r 3
 4T 
4 3  R 4 3
π r1 = 1 +  πr
3  4T  3
P0 +
 r 
Where r1 is new radius of the bubble.
1/ 3
 4T   4T 
r1  r 1 + ∵ P0 + r  P0 

 RP0   
 4T   4T 
 r 1 + 
∵ << 1
 3RP0   RP0 

4Tr 4Tr
\ r1 − r = ⇒ ∆r =
3P0 r 3P0 R

16. Pressure at end B inside the tube is


PB = Po + rgh [Po = atmospheric pressure]
9.12 Problems in Physics for JEE Advanced

h
A B
B

Water Surface

\ PB – Po = rgh
T
∴ = ρ gh [r = radius of curvature of the surface]
r
With increase in h, the curvature of water surface increases (i.e. r decreases) so as to prevent water from flowing out.

The figure shows the change in surface as B is lowered.


17. When the tube is brought into contact with water, it is filled with air at atmospheric pressure. When water rises to a
height h, the air pressure (P) is given by

PA(L – h) = PoAL
P° L
∴P=
L−h

A
h

Radius of curvature of meniscus R = r, since contact angle is zero.


Pressure at A is PA = Po – rgh
T
∴ P = PA +
r
P° L T  L  2T
∴ = P° − ρ gh + ⇒ P°  − 1 = − ρ gh
L−h r L−h  r
P° h T P° h
⇒ = − ρ gh ⇒L−h=
L−h r T
− ρ gh
r
P° r h
⇒L=h+
T − ρ ghr
Surface Tension 9.13

18. Surface tension force on outer wall of the tube is 2pRT (Ø)
Surface tension force (at meniscus) on the inner tube wall is 2prT (Ø)

Force due to air pressure inside the tube pr2P(-)


Force due to atmospheric pressure = pR2Po(Ø)
For equilibrium of the tube, let the upward force needed be F
F + Ppr2 = Mg + pR2Po + 2pRT + 2prT
P° Lπ r 2
F = Mg + π R 2 P° − + 2π ( R + r )T
L−h
 Lr 2 
F = Mg + π P°  R 2 −  + 2π ( R + r )T
 L − h
2T
19. It is known that h0 = [Here cosq = 1]
ρ gr
If the tube has large length and viscosity is not there, the upward pulling surface tension force can cause the water
to rise to a maximum height h. Where
h
F h = π r 2 h ρ .g
2
Where left side is work done by surface tension force and right side is the rise in potential energy of the water in
the glass tube
1
(2π rT cos θ )h = π r 2 ρ gh 2
2
T
cos q = 1 ∴ =h
ρ gr
\ h = 2h0
In presence of viscosity h < 2h0. If water touches the brim of the tube, it means length of tube must be less than 2h0
20. Let the radius of curvature of surface ACB be R.
4T
P2 − P1 =
R
A

r1 r2
C
P2
B
P1

T
[ because there are two surfaces in the wall ACB]
R
4T 4T 4T rr
∴ − = fiR= 12
r2 r1 R r1 - r2
9.14 Problems in Physics for JEE Advanced

21. (a) Consider a particle on the common meeting line of the three surface. It experience three forces of same
magnitude (surface tension is same for all three films) and is in equilibrium. This is possible only if the three
forces are at 120° to each other
F

F
120°
120°

(b) When r1 = r2 = r the pressure on two sides of the common wall is same. The common wall remains flat in shape
of a disc. AB is diameter of the disc.
120º

90º A

r 30º

C
C1 C2

3r
AC = r cos 30° = & AB = 2AC = 3r
2
r
(c) In above figure CC1 = r sin 30° =
2
r 3r
∴ y=r+ =
2 2 2 3
π 3   3  9π
volume of each bubble =    3 −  =
3 2   2 8
3
4 9π
If radius of new bubble = R, then π 3 = 2 ×
3 8
3r
R 1/ 3
2(2)
22. (a) As excess pressure for a soap bubble is (4T / r) and external pressure p0,

pi = p0 + (4T / r)
Pressure inside the smaller bubble is higher, Hence, air flows from smaller bubble to the larger one.
(b) The larger bubble grows in size till the entire air of smaller bubble is transferred into it.
 4T   4T   4T 
p1 =  p0 + , p2 =  p0 +  and p3 =  p0 +  …(i)
 R1   R2   R3 
4 4 4
and V1 = πR13 , V2 = πR23 and V3 = πR33 …(ii)
3 3 3
Now, as mass is conserved, n1 + n2 = n3
P1V1 P2V2 P3V3 È PV ˘
i.e., + = Ías PV = nRT , i.e., n = RT ˙
RT1 RT2 RT3 Î ˚
Surface Tension 9.15

At temperature is constant, i.e., T1 = T2 = T3 , the above expression reduces to


P1V1 + P2V2 = P3V3
Which, in the light of Eqn. (i) and (ii) becomes
 4T   4 3  4T   4 3  4T   4 3
 P0 +   πR1  +  P0 +   πR2  =  P0 +   πR3 
 R1  3   R2  3   R3  3 
2 2 2 3 3 3
i.e., 4T ( R1 + R2 − R3 ) = P0 ( R3 − R1 − R2 )

P0 ( R33 − R13 − R23 )


i.e., T = .
4( R12 + R2 2 − R32 )
23. Hint: the pressure must be same everywhere inside the straw and the bubble.
24. Since the drag force balances the gravity, the necessary condition must be that the variation in hydrostatic pressure
inside the drop should be negligible compared to the excess pressure due to surface tension
T T
ρ g R << ⇒ R <<
R ρg
T
25. Excess pressure inside the soap bubble ∆P =
R
From ideal gas equation, pressure of a gas (at a given temperature) is directly proportional to its density
Pµr
∆ ∆ρ
∴ = where Po and ro are pressure and density of atmospheric air.
° ρ°
ρ
⇒ ∆ρ = ° ∆
°
Apparent weight True weight – Buoyancy
= weight of air inside Bubble – Buoyancy + weight of water skin
4 4
= π R 3 ( ρ° + ∆ρ ) g − π R 3 ρ° g + 4π R 2 a d .g
3 3
4
= π R 3 ∆ρ g + 4π R 2 a d .g
3
4 ρ 4T
= π R3 ° g + 4π R 2 . a.d .g
3 P° R
4 1.2
= ¥ 3.14 ¥ (0.1)2 ¥ 5 ¥ 4 ¥ .04 ¥ 10 + 4 ¥ 3.14 ¥ (0.1)2 ¥ 10 -6 ¥ 103 ¥ 10
3 10
= 0.8 × 10–6 + 1.3 × 10–3 ~ 1.3 × 10–3 N
T π a 4 ∆P
26. ∆P = ⇒ Q=
R 8η L
4
dV π a T 4
⇒ = ; But V = π R 3
dt 2η LR 3

dV dV dR dR
= = 4π R 2
dt dR dt dt
dR π a 4 T
∴ − 4π R 2 =
dt 2η LR
9.16 Problems in Physics for JEE Advanced

R a 4T t a 4T
−4∫ R 3 dR =
2η L ∫0
dt ⇒ R 4 − R04 = t
R0 2η L
1 1
 a 4T  4  a 4 Tt  4
∴ R =  R04 − t = R0 1 − 
 2η L   2η LR04 
27. Water rises between the blocks due to capillary effect. Pressure below the curved water surface (like at a point A
shown in figure) is below the atmospheric pressure.

P0
P0
P0 P0
A

B P0 C
A

But outer walls of the blocks experience atmospheric pressure. This causes the blocks to be pushed towards each
other. In case of Hg, the situation is as shown in the figure. At points like B and C the pressure is higher than
atmospheric pressure which pushed the blocks closer.
28. Consider a length L of the string. Total surface energy of water on it is E = 2prLT
If spherical drops are of radius R and separation between two successive drops is l then conservation of volume
gives
4 L 4 3
π R3 = π r 2 L ⇒ R = r 2 l .................(i)
3 l 3
The total surface energy of liquid drops over a length L is
2/3
2 L  3r 2 l  L
E ' = 4π R . T . = 4π   .T [using (i)]
l  4  l
2/3
 3r 2  TL
∴ E ' = 4π   1/ 3
 4  l
The final surface energy must be less than the original surface energy
\ E' < E
2/3
 3r 2  TL
4π   < 2π rLT
 4  l1/ 3
2/3 2/3
 3r 2  1/ 3 3
2  < rl ⇒ 2  r1/ 3 < l1/ 3
 4  4
9 9
8× × r < l ⇒ r < l
16 2
29. (i) Pressure at the lowest level of meniscus is = Po [∵ liquid surface is flat]
\ Pressure in liquid at A is PA = Po – rgh
(ii) We will consider the horizontal equilibrium of the liquid in the meniscus. We will consider a depth L
perpendicular to the plane of the figure.
Force due to wall Fwall = Pav hL
 P + P° − ρ gh   ρ gh 
= °  hL =  P° −  hL
   
Surface Tension 9.17

Fatm
Fwall
FST

Force due to atmosphere Fatm = PohL


Force due to surface tension FST = T.L
\ Fwall + FST = Fatm
 ρ gh 
 P° −  hL + T .L = P° hL
 2 
ρ gh T
⇒ P° h − + T = P° h ∴ h=
ρg
30. Consider a water drop of radius R
Surface area S = 4pR2
If radius decreases by DR, the surface area changes by DS = –8pRDR
\ Loss in surface energy DE = 8pRDR.T
4
The volume of the drop V = π R 3
3
DV = 4pR2DR
\ Energy required for evaporation of a water layer DR thick is
DE' = rDVL = 4pR2DR. r.L
Loss in surface energy can cause water to evaporate if DE > DE'
T 2 × 0.07
8pR DRT > 4pR2 DRrL ⇒ R < =
ρ L 103 × 2.3 × 106
6 × 10–11 m
Drops of this size do not exist. In fact the above number is close to size of one molecule of water.
31 When water level rises in , the air pressure rises. This blows a bubble at the tip of C. The monometer reading
gives the pressure difference of the air inside the container and the atmospheric pressure (Po). The various stages
of bubble has been shown in the figure

The inside pressure is maximum when radius of the bubble is equal to the radius of the capillary (i.e. when the
bubble is hemispherical )
Pmax – Po = rgho
2T ρ gh0 .r
⇒ = ρ gh0 ⇒T =
r 2
32. Consider a small patch on the liquid surface. Angle subtended by arc abc at the centre of curvature is dq and the
angle subtended by arc a f e at its centre of curvature is df.
9.18 Problems in Physics for JEE Advanced

B
d

e f b
c
A
a
R2
d
R1

d

Area of the patch

dS = (R1dq) (R2df) ...........(i)


Pressure on concave side (at point A) is PA and pressure on convex side (at point B) is PB. Net outward force on the
patch due to pressure difference is F = (PA – PB) dS ............. (ii)
This force is balanced by surface tension force. Look at the two figures shown. FT and FT are surface tension force
on arcs of length R2df and R1dq respectively.
\ Net inward force on the patch of liquid surface is
b
a c
a e

FT FT´
FT FT´
dq dq R1 R1
df df
2 2 2 2

dθ dφ
+ 2 F 'T sin
= 2 FT sin =  FT dq + F'T df [Q sin dq ~ dq]
2 2
= T.R2df.dq + TR1dq.df
Ê R + R2 ˆ
= T ( R1 + R2 )df.dq = T Á 1 dS [using (i)]
Ë R1 R2 ˜¯
This force balances the force given by equation (ii)
 R1 + R2   1 1 
( PA − PB ) dS = T   dS ⇒ PA − PB = T  + 
 R1 R2  R
 1 R2 

Note: If the surface is spherical R1 = R2 = R (say)


T
∴ PA − PB =
R
If the liquid surface is cylindrical
T
R1 = R ; R2 = • ∴ PA − PB =
R R2  
R1

33. First the water level rises in the capillary till the time the pressure at the bottom of the capillary becomes equal to
atmospheric pressure.
Surface Tension 9.19
P0

P0
h0

2T cos θ 2T cosq
PO − + ρ gh0 = PO fi h0 =
r r rg
After this the height difference between levels of water in the capillary and the tube remains constant at h0. It means
height in both of them increases by same amount when some water is poured. h
Tube
At time t1 water touches the brim of the capillary. When more water is poured
the radius of curvature of the water surface in the meniscus of the capillary
h
changes (surface gets flatter) and the water level in the wide tube continues to
1
Capilary
rise. Note that the level of water in the broad tube will now rise at a slightly h 0

faster pace as quantity of water is not increasing in the capillary. The time t2
when water level becomes same in both tubes the meniscus surface become t
t t t
flat (as in the broad tube). Thereafter, as the height of water increases in broad 0 1 2

tube, the meniscus gets convex. After time t2 water begins to flow out of the capillary and water level in both tubes
becomes constant.
10 VISCOSITY

Level 1 viscous liquid is made to flow through the pipe at


the same volume flow rate (measured in m3s–1).
Q.1. During a painting process, a thin, flat tape of Find the maximum speed of a fluid particle in the
width b (dimension perpendicular to the plane of pipe.
the figure) is pulled through a paint filled channel Q.5. A near surface earth satellite is in the shape of a
of length L. The density and viscosity of the paint sphere of radius r. It encounters cosmic dust in
liquid is r and h respectively. The tape is pulled at its path. The viscous force experienced by the
a constant speed v and width of the channel is h. satellite follows stoke’s law. The coefficient of
Find the minimum force needed to pull the tape. viscosity is h. Mass and radius of the earth are M
L and R respectively.

v
(a) Calculate the power of the rocket engine that
h
must be put on to keep the satellite moving as
usual.
Q.2. A liquid is flowing through a horizontal channel. (b) Calculate the equilibrium temperature of the
The speed of flow (v) depends on height (y) from surface of the satellite assuming that it radiates
  y   y 2  like a black body and no outer radiation falls
the floor as v = v0  2   −    . Where h is
  h   h   on it. Assume that the heat generated due to
viscous force is absorbed completely by the
the height of liquid in the channel and v0 is the
satellite body.
speed of the top layer. Coefficient of viscosity is
h. Calculate the shear stress that the liquid exerts
Q.6. Two balls of radii r and are released inside a
on the floor.
deep water tank. Their initial accelerations are
y
v0 Free surface found to be and respectively. Find the

velocity of smaller ball relative to the larger


h ball, a long time after the two balls are released.
Coefficient of viscosity is given to be h.

r
Q.3. A car having cross sectional area of its front equal r/2
to A is travelling on a highway at a speed v. The
viscous drag force acting on the car is known to
be given as Fv = CArv .Where r is density of air
and C is a constant which depends on the shape
of the car. The petrol used by the car produces Q.7. The coefficient of viscosity h of a gas depends on
E joules of energy per kg of it burnt. Calculate mass of the gas molecule, its effective diameter
the mileage (in km/kg) of the car if the combined and its average speed. It is known that diameter
efficiency of its engine and transmission is f. of helium atom is 2.1 × 10–10 m and its coefficient
Q.4. An ideal fluid flows through a pipe of circular of viscosity, at room temperature is 2.0 × 10–5
cross section of radius r at a speed v0. Now a kg m–1s–1. Estimate the effective diameter of CO2
10.2 Problems in Physics for JEE Advanced

molecule if it is known that h at room temperature


for CO2 is 1.5 × 10–5 kg m–1s–1.
Q.8. When hard brakes are applied (so as to lock the
wheels) in a car travelling on a wet road it can b
“hydro-plane”. A film of water is created between
the tires and the road and, theoretically, the car R
can slide a very long distance. [In practice film
l
is destroyed much before such distances can be
achieved]. Consider a car of mass M moving on
a wet road with speed v0. Hard brakes are applied. Level 3
Let the area of film under all four tires be A and
thickness of the film be h. Coefficient of viscosity Q.12. A vertical steel rod has radius a. The rod has a
is h. coat of a liquid film on it. The liquid slides under
(a) Calculate the distance (x) to which the car gravity. It was found that the speed of liquid layer
will slide before coming to rest. at radius r is given by
(b) Calculate the value of x for M = 103 kg,
A = 0.2 m2, h = 0.1 mm, v0 = 20 ms–1, and
–3
h kg m–1
Where b is the outer radius of liquid film, h is
Level 2 coefficient of viscosity and r is density of the
liquid.
Q.9. A spherical ball of radius r and density d is (i) Calculate the force on unit length of the rod
dropped from rest in a viscous fluid having due to the viscous liquid?
density r and coefficient of viscosity h. (ii) Set up the integral to calculate the volume
(a) Calculate the power (P1) of gravitational flow rate of the liquid down the rod. [you may
force acting on the ball at a time t after it is not evaluate the integral]
dropped. rod
(b) Calculate the rate of heat generation (P2) due
to rubbing of fluid molecules with the ball, at
time t after it is dropped.
Liquid
(c) How do P1 and P2 change if the radius of the r
ball were doubled? g
(d) Find P1 and P2 when both become equal.
a
Q.10. Two balls of same material of density r but
radius r1 and r2 are joined by a light inextensible b
vertical thread and released from a large height
in a medium of coefficient of viscosity = h. Find Q.13. A viscometer (an instrument used to study
the terminal velocity acquired by the balls. Also characteristics of a non-ideal fluid) consists of
find the tension in the string connecting both a flat plate and a rotating cone. The cone has a
the balls when both of them are moving with large apex angle and the angle q shown in figure
terminal velocity. Neglect buoyancy and change is very small (typically less than 0.5° ). The apex
in acceleration due to gravity. of the cone just touches the plate and a liquid fills
the narrow gap between the plate and the cone.
Q.11. A car windshield wiper blade sweeps the wet
The cone has a base radius R and is rotated with
windshield rotating at a constant angular speed of
constant angular speed w. Consider the liquid to
w. R is the radius of innermost arc swept by the
be ideal and take its coefficient of viscosity to be
blade. Length and width of the blade are l and b
h. Calculate the torque needed to drive the cone.
respectively. Coefficient of viscosity of water is w
h. Calculate the torque delivered by the motor to R
rotate the blade assuming that there is a uniform
layer of water of thickness t on the glass surface. q q
Viscosity 10.3

ANSWERS

1. 9. (a)

2.
(b)

3. (c) P1 and P2 become 32 times.

4. (d) P1 = P2 =

r È ˘ ;
È h ˘ 10. - + pr -
5. (a) (b) Í ˙ h Î ˚
Î s ˚

6. upwards 11.

7. 4.4 × 10–10 m
12. (i) (ii)

8. (a) (b) 10 km
13. 

SOLUTIONS

1. The paint layer in contact with channel wall is at rest and that in contact with the tape is v. The viscous force acts
on two surfaces of the tape. If gap between the tape and upper surface of the channel is x then velocity gradient at
the two surfaces of the tape is
and

Total viscous force on the tape is Fvis = Fupper + F lower


È ˘
=h Í ˙
Î - ˚

This force is minimum when x(h – x) is maximum, i.e., when

2. Shear stress is tangential force applied by the liquid on unit area of the floor.

Velocity gradient
10.4 Problems in Physics for JEE Advanced

At y = 0,

Viscous force per unit area

3. Viscous drag force acting on the car is F = CArv2


Work required to travel through a distance x is
W = Fx = CArv2◊x
Let mass of petrol burnt = m ; Energy produced = mE
Since efficiency is f fmE = W

fmE = CArv2x

Mileage = Distance travelled per unit mass of fuel burnt

4. Volume flow rate Q = pr2 .v0


When a viscous liquid flows, speed is maximum along the axis of the pipe (let us call this speed as u0) and decreases
to zero at the circumference. The variation in speed will be almost linear if radius of the pipe is small.

x dx

Speed at a distance x from the axis is

volume flow rate through a region of width dx is

Total flow rate is

5. (a) Orbital speed of the satellite


Viscous force fv = 6phrv
Power required P = v. fv = 6phrv2

(b)

6. Initial acceleration for larger ball is given by -


ma = mg – FB
Viscosity 10.5

[r density of water, d1= density of ball]

& d1 = 2r
4r
Similarly, for second ball d2 =
3
After long time both of them will acquire terminal speed ( u01 for larger ball and u02 for smaller ball)

6phr u01 = mg – FB

fi Similarly,

Velocity of smaller ball with respect to the larger ball is

Ê ˆ r r r

Ë
- ˜
¯ h
= -
h
=
h
(≠)

7. From method of dimensions


h = kmvd–2 [ m = mass, v = average speed, d = diameter of molecule]

But

At a given temperature

= 4.4 × 10–10m

8. (a) Viscous force [ v = instantaneous speed of the car]

Or,

Or, &

&

(b) =10 km (!)


10.6 Problems in Physics for JEE Advanced

9.

= a – bv (say)

&
&

& &

2 g (d − ρ ) r 2  −

t 
& & v=
1 − e 2⋅d ⋅r 
2

9η  

P1 = m◊g◊v
(a)

P2 = Fv ◊ v = 6phrv ◊ v = 6phrv


(b)

P1 µ r5 and P2 µ r5
(c)

On doubling the radius, both become 32 times


(d)
P1 = P2, when
[i.e., at t = • ]


6ph r2VT
10. Co nsidering both balls together
T W2


T 6ph r1VT

=
W1
Viscosity 10.7

Consider only ball 1 and write W1 = T + 6phr1Vr

Put the value of VT and calculate T.


11.
w

dx

Consider an element of length dx on the blade. Speed of this element is v = wx


Viscous force on this element is

[ = velocity gradient]

Torque due to this force

Net viscous torque on the blade is

The same torque must be applied by the motor to keep the blade moving.

12. (a) Velocity of fluid layer at radius r is

Velocity gradient along radial direction is

At r = a (i.e., on the surface of the rod)

Area of unit length of the rod’s surface A = (2pa)(1) = 2pa


10.8 Problems in Physics for JEE Advanced

(b) Volume flow rate is


You can evaluate the above integral if you want some practice in mathematics. To help you it is being given that


13. w

x dr
r y
q

x
dr

Consider a ring shaped element in the cone as shown in the figure.


&

Area of the ring element

Velocity gradient at the location of this ring is

Viscous force on ring element

Torque on the element

Net torque on the cone

 [∵ q is small]
11 ELASTICITY

modulus of steel is Y, calculate the longitudinal


Level 1 stress developed in it. Also calculate the tension
Q.1. Human bones remain elastic if strain is less force developed in the ring.
than 0.5%. However, the young’s modulus for [Take b << r]
compression (Yc) and stretch (Ys) are different.
The typical values are Yc = 9.4 × 109 Pa and
R
Ys  =  16 × 109 Pa. The shear modulus of elasticity
for the bone is h = 1010 Pa
Answer following questions with regard to a leg
bone of length 20 cm and cross sectional area d

3  cm2
Disc
(a) Calculate the maximum stretching force that
the bone can sustain and still remain elastic. b
(b) A man of mass 60 kg jumps from a height of
10 m on a concrete floor. Half his momentum r
is absorbed by the impact of the floor on the
d
particular bone we are talking about. The
impact lasts for 0.02 s. Will the compressive
stress exceed the elastic limit?
(c) How much shearing force will be needed to Ring
break the bone if breaking strain is 5°. Q.4. A water tank is supported by four pillars. The
Q.2. The elastic limit and ultimate strength for steel is pillars are strong enough to sustain ten times
2.48 × 108 Pa and 4.89 × 108 Pa respectively. A the stress developed in them when the tank is
steel wire of 10 m length and 2 mm cross sectional completely full. An engineer decides to increase
diameter is subjected to longitudinal tensile stress. the every dimension of the tank and the pillars
Young’s modulus of steel is Y = 2 × 1011 Pa by hundred times so as to store more water. Do
(a) Calculate the maximum elongation that can you think he has taken a right decision? Assume
be produced in the wire without permanently that material used in construction of the tank and
deforming it. How much force is needed to pillars remain same.
produce this extension?
(b) Calculate the maximum stretching force that
can be applied without breaking the wire.
Q.3. A steel ring is to be fitted on a wooden disc of
radius R and thickness d. The inner radius of the
ring is r which is slightly smaller than R. The
outer radius of the ring is r + b and its thickness is
d (same as the disc). There is no change in value
of b and d after the ring is fitted over the disc;
Q.5. Two bars A and B are stuck using an adhesive.
only the inner radius becomes R. If the Young’s
The contact surface of the bars make an angle
11.2 Problems in Physics for JEE Advanced

q with the length. Area of cross section of each with angular velocity w. Determine the amount
bar is S0. It is known that the adhesive yields if (assumed small) by which its circumference
normal stress at the contact surface exceeds s0. increases.
Find the maximum pulling force F that can be Q.10. A steel wire of radius r is stretched without
applied without detaching the bars. tension along a straight line with its ends fixed at
S0
A B A and B (figure). The wire is pulled into the shape
ACB. Assume that d is very small compared to
F F length of the wire. Young’s modulus of steel is Y.
q
(a) What is the tension (T) in the wire?
Q.6. A very stiff bar (AB) of negligible mass is (b) Determine the pulling force F. Is F larger
suspended horizontally by two vertical rods as than T?
shown in figure. Length of the bar is 2.5 L. The
A 2l B
steel rod has length L and cross sectional radius
of r and the brass rod has length 2L and cross d
sectional radius of 2r. A vertically downward C
force F is applied to the bar at a distance x from
the steel rod and the bar remains horizontal. Find F
the value of x if it is given that ratio of Young’s Q.11. A uniform material rod of length L is rotated in a
Y
modulus of steel and brass is s = 2 . horizontal plane about a vertical axis through one
YB of its ends. The angular speed of rotation is w.
Find increase in length of the rod. It is given that
Brass
density and Young’s modulus of the rod are r and
2L Steel Y respectively.
L Q.12. A rectangular bar is fixed to a hard floor. Height
F x
of the bar is h and its area in contact with the floor
is A. A shearing force distorts the bar as shown.
A 2.5L B
Prove that the work done by the shearing force is
Ês2 ˆ
Q.7. A closed steel cylinder is completely filled with W = Á ˜ ¥ volume of the bar. Here s is shear
water at 0°C. The water is made to freeze at 0°C. Ë 2h ¯
Calculate the rise in pressure on the cylinder modulus of elasticity. Assume the deformation to
wall. It is known that density of water at 0°C is be small.
1000 kg/m3 and the density of ice at 0°C is 910 F
A
kg/m3. Bulk modulus of ice at 0°C is nearly 9 ×
109 Pa. [Compare this pressure to the atmospheric
h
pressure. Now you can easily understand why
water pipelines burst in cold regions as the winter
sets in.]
Q.13. A thin uniform rod of mass M and length L is
Q.8. (i) Two identical rods, one of steel, the other of free to rotate in vertical plane about a horizontal
copper, are stretched by an identical amount. axis passing through one of its ends. The rod
On which operation more work is expended? is released from horizontal position shown in
(ii) Two identical rods, one of steel, the other of the figure. Calculate the shear stress developed
copper, are stretched with equal force. On at the centre of the rod immediately after it is
which operation is more work needed? released. Cross sectional area of the rod is A. [For
calculation of moment of inertia you can treat it to
Level 2 very thin]
A
Q.9. A thin ring of radius R is made of a wire of M L B
density r and Young’s modulus Y. It is spun in
its own plane, about an axis through its centre,
Elasticity 11.3

Q.14. A rigid cylindrical container has inner radius r. thread of length l. It is raised to a height and
A cork having radius r + Dr and length L is to released. Find the least height to which the load
be fitted so as to close the container. Uniform must be raised so that it will break the thread
pressure (DP) is needed on the curved cylindrical when allowed to fall.
surface of the cork. Poisson’s ratio of a cork is Q.16. Atmospheric pressure is P0 and density of water
almost zero, and its bulk modulus is B. at the sea level is r0. If the bulk modulus of water
P
is B, calculate the pressure deep inside the sea at
a depth h below the surface.

2 (r + r)
2r
Level 3
P Q.17. A metal cylinder of length L and radius R is
L
fixed rigidly to ground with its axis vertical.
A twisting torque t0 is applied along the
(a) Calculate DP circumference at the top of the cylinder. This
(b) After the cork is fitted how much force will causes an angular twist of q0 (rad) in the top
be needed to pull it out of the container? surface. Calculate the shear modulus of elasticity
Coefficient of friction between the container (h) of the material of the cylinder.
and the cork is m. t

Q.15. Assume that the least load which would break a


thread when simply suspended from it is M and
that this load produces a strain of 1 percent at the L
moment of breaking. Also assume that Hooke's
law applies to the thread right up to breaking-
point. A load of mass m is suspended from a

ANSWERS

1. (a) 2.4 × 104 N  (b) Yes  (c) 2.6 × 105 N rw 2 L


2. (a) 1.24 cm, 779 N (b) 1535 N 3Y
Ê R - rˆ Ê R - rˆ Mg
3. YÁ ; Ybd Á
Ë r ˜¯ Ë r ˜¯ 16 A
4. No
2 BDr
s 0 S0 DP = pm D
5. r
sin 2 q
0.01M 
6. x = 1.25 L 2m
7. 8.1 × 108 Pa Ê r gh ˆ
P = P0 - B ln Á 1 - 0 ˜
Ë B ¯
2pr R3w 2 2lt 0
Y p R 4q 0
p
11.4 Problems in Physics for JEE Advanced

SOLUTIONS

F DL
1. (a) s = Ys
A L
\ Fs = AYs
DL
L
( ) ( ) ( )
4
= 3 ¥ 10 -4 m 2 ¥ 16 ¥ 109 N / m 2 ¥ 5 ¥ 10 -3 = 2.4 × 10 N
(b) Momentum absorbed by the bone
1 1
D P = m 2 gh = ¥ 60 ¥ 2 ¥ 10 ¥ 10 = 423 N – s
2 2
D P 423
\ Force = = = 2.12 ¥ 10 4 N
D t 0.02
Maximum compressive force that the bone can sustain to remain elastic is

Fc = AYc
DL
L
( ) (
= 3 ¥ 10 -4 m 2 ¥ 9.4 ¥ 109 N /m 2 ) (5 ¥ 10 -3
) = 1.4 × 104 N
The impact force is larger than elastic limit.
Ê p ˆ 5
F = hq A = 1010 ¥ Á 5 ¥
(c) rad ˜ ¥ 3 ¥ 10 -4 = 2.6 × 10 N
Ë 180 ¯
2. (a) Area of cross section A = pr2 = 3.14 × (1 × 10–3)2
The stress should not exceed the elastic limit otherwise the wire will suffer permanent deformation
F
\ = 2.48 ¥ 108
A
F = 2.48 × 108 × 3.14 × 10–6 = 779N
stress 2.48 ¥ 108
Strain = = = 1.24 ¥ 10 -3
Y 2 ¥ 1011
DL
= 1.24 ¥ 10 -3 fi DL = 1.24 × 10–3 × 10 m = 1.24 cm
L
(b) The stress should not exceed the ultimate strength.
F 8
A = 4.89 ¥ 10
F = 4.89 × 108 × 3.14 × 10–6 = 1535N
3.
Increase in circumference of the ring = 2p(R – r)
2p ( R - r ) R - r
\ Strain = =
2p r r
Ê R - rˆ
Stress = Y (strain) = Y Á
Ë r ˜¯
Ê R - rˆ
\ Tension = (stress)A = Ybd Á
Ë r ˜¯
W
4. If weight of the tank (completely filled with water) is W, then load on each pillar = .
4
W
Stress on each pillar =
4A
A = Area of cross section of the pillar.
W
Each pillar can support a stress of 10 ¥ .
4A
Elasticity 11.5

When each dimension is made 100 times, the total load will become 106 W.
106 W W
\ Stress on each pillar = 4
= 25 ◊
4 ¥ 10 A A
W
But each pillar can support a stress of 10 only.
4A
Hence, the pillars will break.
5. S
Fn

F
S0
q

Normal force Fn = F sinq


S
Area S = 0
sinq
Fn F sin q F
\ s n = = = sin 2 q
S S0 S0
sin q
Fmax sin 2 q s S
fi = s0 \ Fmax = 0 2 0
S0 sin q

6. If Tb & Ts are tension developed in the steel and the brass rod then

Tb(2.5L – x) = Ts . x ….. (1)
and Tb + Ts = F ….. (2)
xF
Solving (1) and (2) we get Tb =
2.5L
Ê x ˆ
and Ts = Á 1 - F
Ë 2.5L ˜¯
For the bar to remain horizontal, extension in the two rods must be same.
Tb (2 L ) Ts L
\ = [where A = pr2]
4 AYB AYs
Ts Y
= s
Tb 2YB
2.5L - x
fi =1 fi x = 1.25 L
x
7. If v = volume of water at 0°C, then volume of ice formed will be
1000 100
v¢ = v ¥ = v
910 91
9v
Change in volume Dv = v ¢ - v =
91
But the ice is not allowed to expand due to pressure exerted by the cylinder wall.
9v
Dv 9
Volume strain = = 91 =
v¢ 100 v 100
91
DP
\  Using definition of bulk modulus B =
Dv

11.6 Problems in Physics for JEE Advanced

9
D P = 9 ¥ 109 ¥ = 81 ¥ 107 = 8.1 ¥ 108 Pa
100
8. (i) The modulus of normal elasticity (Young’s modulus) is greater for steel than for copper. Therefore if the rods
are of equal dimensions and are to be stretched by the same amount, a greater force is necessary for the steel
rod than for the copper one. So the steel rod requires that more work should be done.
(ii) If the process of stretching is carried out with equal forces for both rods, the steel rod will be stretched less than
the copper one. Therefore more work will be done this time on stretching the copper rod.
9. Let the tension in the ring be T.
Its resolved component acting towards the centre of rotation is
Ê Dq ˆ
2T sin Á  T Dq
Ë 2 ˜¯
This must balance the centripetal force = RDqArRw2 (A is area of cross section of the wire of the ring)
T T
Dq
R q /2 T T

2T sin Dq /2
w

TDq = RDqArRw2
Longitudinal stress in the ring = T/A = rR2w2.
r R 2w 2
Strain = 
Y
r R 2w 2 2pr R3w 2
Increase in circumference = 2p R =
Y Y
10. Increase in length of the wire = (AC + CB) – 2l = [2 (l 2 + d 2 ) - 2l ]
Longitudinal stress = T / pr2, (where r is radius of wire).
2 2
[2 (l + d ) - 2l ]
Longitudinal strain =
2l
A 2l B
q q
d
T T
C

Longitudinal stress
Young’s modulus =
Longitudinal strain
T 2l
Y= 2¥
or,
pr [2[ (l + d 2 ) - l ]]
2

Y ¥ p r 2 ¥ [ (l 2 + d 2 ) - l ]
\ T =
l
If d is very small compared to l, then
1/ 2
2 2
Ê d2 ˆ Ê d2 ˆ
(l + d ) = l Á 1 + 2 ˜ = l Á 1 + 2 ˜
Ë l ¯ Ë 2l ¯
2 2 2
\ T = Y × pr (d / 2l ).
Elasticity 11.7

F = 2Tsinq = 2T(d / AC) ~ 2T(d / l)


This will be a number much smaller than T when d is very small compared to length of the string.

11. Consider an element of length dx at a distance x from the rotation axis. Centripetal force required for this element
is (rAdx)w2x
w
P
B
A

x
dx

Tension at a point P, at a distance r from the axis is equal to sum of centripetal forces on all elements lying between
P and B.
\ Tension at P
L
r Aw 2 2
Tr = r Aw 2 Ú x dx =
(L - r 2 )
r
2
Now assume that dl is extension in an element of length dr located at a distance r from the axis.
dl
Strain =
dr
Tr 1
Stress = = rw 2 [ L2 - r 2 ]
A 2
dl 1
\ Y = rw 2 [ L2 - r 2 ]
dr 2
1 rw 2 2
fi dl = [ L - r 2 ] dr
2 Y
\ Change in length of the entire rod
L
rw 2
Dl = Ú dl =
2Y Ú0
( L2 - r 2 )dr

rw 2 Ê 3 L3 ˆ 1 rw 2 L3
= Á L - ˜ =
2Y Ë 3¯ 3 Y
12. x F
F

h
q
x

Stress = h . strain
F x hA
\ =h fi F = ◊ x ….. (i)
A h h
Since force changes linearly with x, we can write its work done as
1 1 Ê shˆ sh
W= Fx = (s A) Á ˜ [using (1) x = ]
2 2 Ë h¯ h

s2 s2
= Ah = ¥ volume
2h 2h
11.8 Problems in Physics for JEE Advanced

13. A
C B

a
Mg
Immediately after release
ML2 L 3g
Ia = t fi ◊ a = Mg fi a =
3 2 2L
Consider the half rod BC. Its COM has a downward acceleration
3L 9
aBC = ◊a = g
4 8
FShear = Shear force applied by part AC on the part BC.
Mg M C
+ Fshear = aBC B
2 2
9 Mg Mg Mg
\ Fshear = - = M
g
16 2 16 2
Fshear
Mg
\ Shear stress =
16 A
14. (a) V = p(r + Dr)2 L ~ pr2L
DV = 2prLDr
DV 2p rL Dr 2 Dr DP
\ = = fi B=
V pr2L r DV
V
Ê DV ˆ 2 BDr
\ D P = BÁ =
Ë V ˜¯ r
(b) Pulling force = friction force
2 BDr
F = m(DP2prL) = m ◊ ◊ 2p rL = 4pmBLDr
r
15. A load m, falling from a height h, acquires a kinetic energy equal mgh.
When the stretch in the string is maximum this kinetic energy must be turned into energy of elastic deformation of
the thread. If k is the force constant of the string-
kx 2
= mgh ----- (i)
2
In the problem it is given that maximum stretch is x = 0.01 l.
kx = Mg.
Substituting these relationships in the equation (i)
0.01M 
We get h = .
2m
16. If volume of mass m of water is V and its density is r then
rV = m
fi rdV + Vdr = 0

dr dV
fi =- … (i)
p V
If volume (V) of an element of water changes by dV due to an isotropic pressure increase dP,
dP
B = -
dV / V
Elasticity 11.9

r
fiB= dP [using (i)]
dr

dr
fi dP = B … (ii)
r
If depth changes by dh
dr
dP = rgdh \ r g dh = B ◊
r
h r
g dr g dr
fi fi Ú dh = Ú 2
dh = 2
B r B 0 r r 0

gh 1 1 r r gh
fi = - fi 0 = 1 - 0 … (iii)
B r0 r r B
dr
From (ii) dP = B
r
r
P
dr Ê rˆ
\ dP = B

Ú Ú r
\ P - P0 = B ln Á ˜
Ë r0 ¯
P0 r0

Êr ˆ Ê r gh ˆ
P = P0 - B ln Á 0 ˜ = P0 - B ln Á 1 - 0 ˜

Ë r¯ Ë B ¯
rq 0
17. The shear strain at a radius r within the cylinder at its surface is =
l
hrq 0
\ Stress = h (strain) =
l
hr 2q 0
Torque per unit area at radius r is r ¥ stress =
l
The total torque can be obtained by integrating this quantity over the entire area.
R
Ê hr 2q 0 ˆ
\ t 0 = Ú (2p rdr ) Á ˜
0 Ë l ¯

p h R 4q 0
t0 =
2 l
2lt 0
\ h=
p R 4q 0
12 SIMPLE HARMONIC MOTION

Level 1 of oscillation is a. A large number of photographs


of the particle are shot at regular intervals of
time with a high speed camera. It was found that
Q. 1. (i) The acceleration (a) of a particle moving
photographs having the particle at x1 + x were
along a straight line is related to time (t) as
maximum in number and photographs having the
per the differential equation . b is particle at x2 + x were least in number. What are
a positive constant. Is the particle performing values of x1 and x2?
SHM? If yes, what is the time period? Q. 5. Position vector of a particle as a function of time

(ii) A particle is executing SHM on a straight is given by

line. A and B are two points at which its    R = (a sin wt) î + (a cos wt) ĵ + (b sin w0t) k̂
velocity is zero. It passes through a certain The particle appears to be performing simple
point P (AP < PB) with a speed of 3 m/s at harmonic motion along z direction, to an observer
times recorded as t = 0, 0.5 s, 2.0 s, 2.5 s, moving in xy plane.
4.0 s, 4.5 s......Determine the maximum speed
(a) Describe the path of the observer.
of the particle and also the ratio AP/PB.
(b) Write the distance travelled by the observer
Q. 2. The position – time graph for two particles- 1 and
himself in the time interval he sees the
2- performing SHM along X axis has been shown
particle completing one oscillation.
in the fig.
Q. 6. A wheel is revolving at an angular speed of w.
(a) Write the velocity of the two particles as a
A pin welded at the circumference of the wheel
function of time.
forces a T shaped body to move up and down. The
(b) If the energy of SHM for the two particles is pin slides freely inside the slot of the yoke as the
same write the ratio of their masses. wheel rotates. The T shaped body is constrained
x
2
to move vertically by a set of walls.

A 1
yoke
t
t0 2t0

-2A
Pin
Q. 3. A particle moves along X axis such that its w
acceleration is given by a = – b(x – 2),where b is a
positive constant and x is the position co-ordinate.
(a) Is the motion simple harmonic? A
Wall
(b) Calculate the time period of oscillations.
(c) How far is the origin of co-ordinate system
(a) Find the time period of oscillatory motion of
from the equilibrium position?
point A at the base of the T shaped body
Q. 4. A particle is performing simple harmonic motion
(b) Is the motion of A simple harmonic?
along the x axis about the origin. The amplitude
12.2 Problems in Physics for JEE Advanced

Q. 7. (i) A particle is performing simple harmonic other end of the massless spring has a particle
motion with time period T. At an instant its of mass m connected to it. With what maximum
speed is 60% of its maximum value and is amplitude can the particle oscillate up and down
increasing. After an interval t its speed such that the block does not lose contact with the
becomes 80% of its maximum value and is table?
decreasing. Find the smallest value of t in M
terms of T.
(ii) A particle is doing SHM of amplitude
0.5 m and period p seconds. When in a
position of instantaneous rest, it is given an k
impulse which imparts a velocity of 1 m/s
towards the equilibrium position. Find the m
new amplitude of oscillation and find how
much less time will it take to arrive at the next Q. 11. A block of mass m is moving along positive x
position of instantaneous rest as compared to direction on a smooth horizontal surface with
the case if the impulse had not been applied. velocity u. It enters a rough horizontal region at
x = 0. The coefficient of friction in this rough
Q. 8. A block of mass M is tied to a spring of force region varies according to m = ax, where ‘a’ is
constant k and placed on a smooth horizontal a positive constant and x is displacement of the
surface. The natural length of the spring is L. P block in the rough region. Find the time for which
is a point on the spring at a distance from its the block will slide in this rough region.
fixed end. The block is set in oscillations with Q. 12. (i) In the shown arrangement, both springs are
amplitude A. Find the maximum speed of point P relaxed. The coefficient of friction between
on the spring. m2 and m1 is m. There is no friction between
m1 and the horizontal surface. The blocks are
P k displaced slightly and released. They move
A M
together without slipping on each other.

Q. 9. A particle of mass m is suspended with the help k2


of a spring and an inextensible string as shown k1 m2
in the figure. Force constant of the spring is k. m1
The particle is pulled down from its equilibrium
position by a distance x and released. (a) If the small displacement of blocks is x then
find the magnitude of acceleration of m2.
What is time period of oscillations?
String
1
(b) Find the ratio so that the frictional force
2

on m2 acts in the direction of its displacement


Spring
from the mean position.
m (ii) Two small blocks of same mass m are
connected to two identical springs as shown
(a) Find maximum value of x for which the in fig. Both springs have stiffness K and they
motion of the particle will remain simple are in their natural length when the blocks
harmonic. are at point O. Both the blocks are pushed
(b) Find maximum tension in the string if so that one of the springs get compressed
mg by a distance a and the other by a/2. Both
x .
k the blocks are released from this position
Q. 10. A block of mass M is placed on top of a hole in simultaneously. Find the time period of
a horizontal table. A spring of force constant k oscillations of the blocks if - (neglect the
is connected to the block through the hole. The dimensions of the blocks)
Simple Harmonic Motion 12.3

a
a/2 calculate the period. If there is a device which can
change the tension in the wire at will , how will
k m k
O
the time period change if tension in the wire is
increased?
(a) Collisions between them are elastic. A
(b) Collisions between them are perfectly l T
inelastic. W

Q. 13. Two blocks of mass 10 kg and 2 kg are connected x


l
by an ideal spring of spring constant K = 800 N/m T
and the system is placed on a horizontal surface B
as shown.
10 kg 2 kg Q. 16. A simple pendulum oscillating with a small
µ = 0.5 µ=0
amplitude has a time period of T = 1.0s. A
horizontal thin rod is now placed beneath the
The coefficient of friction between 10 kg block point of suspension at a distance equal to half the
and surface is 0.5 but friction is absent between length of the pendulum. The string collides with
2 kg and the surface. Initially blocks are at rest and the rod once in each oscillation and there is no
spring is relaxed. The 2 kg block is displaced to loss of energy in such collisions. Find the new
elongate the spring by 1 cm and is then released. time period T´ of the pendulum.
(a) Will 10 kg block move subsequently? Q. 17. (i) A small steel ball (B) is at rest on the edge of a
(b) Draw a graph representing variation of table of height h. Another identical steel ball
magnitude of frictional force on 10 kg block (A) is tied to a light string of length L =1.0 m
with time. Time t is measured from that and is released from the position shown so
instant when 2 kg block is released to move. that it swings like a pendulum. At the lowest
position of its path it hits the ball B which
Q.14. A particle of mass m is tied at the end of a light
string of length L, whose other end is fixed at is at rest. Ball B flies off the table and hits
the ground in time t. After collision the ball
point C (fig), and is revolving in a horizontal
A keeps moving for a time t´ before coming
circle of radius r to form a conical pendulum. A
to rest for the first time. Find the value of h if
parallel horizontal beam of light forms shadow of
t = t´. Collision between the balls is head on
the particle on a vertical wall.
and coefficient of restitution is e = 0.995.
C
L=1m O
A
L L
Light
r
O Wall B
m
If the tension in the string is F find - h

(a) The maximum acceleration of the shadow


moving on the wall.
(b) The time period of the shadow moving on the (ii) A pendulum has a particle of mass m attached
wall. to a massless rod of length L. The rod is
Q.15. A small ball of mass m is attached to the middle released from a position where it makes an
π
of a tightly stretched perfectly flexible wire AB angle θ 0  <  with the vertical. The time
of length 2 l ( ). The ball is given a small  2
lateral displacement in horizontal direction and period of oscillation is observed to be T0.
released. The initial tension (T) in the wire is high Another similar pendulum has a rod of length
and change in it due to small lateral displacement 2L. Time period of this pendulum when
of the ball can be neglected. Prove that the ball released from position q0 is T. Which is larger
will perform simple harmonic motion, and T or T0?
12.4 Problems in Physics for JEE Advanced

Q. 18. A disc of mass M = 2m and radius R is pivoted at separation between A and B is always less than or
its centre. The disc is free to rotate in the vertical equal to a.
plane about its horizontal axis through its centre (a) Find the phase difference between the
O. A particle of mass m is stuck on the periphery of particles.
the disc. Find the frequency of small oscillations
of the system about its equilibrium position. (b) If distance between the two particles is
plotted with time, with what frequency will
the graph oscillate?
Q. 22. (i) A particle of mass m executes SHM in xy-
O plane along a straight line AB. The points A
(a, a) and B (– a, – a) are the two extreme
R positions of the particle. The particle takes
time T to move from one extreme A to the
other extreme B. Find the x component of
m
the force acting on the particle as a function
Q. 19. A rigid body is to be suspended like a physical of time if at t = 0 the particle is at A.
pendulum so as to have a time period of T = 0.2p

(ii) Two particle A and B are performing SHM
second for small amplitude oscillations. The
along X-axis and Y-axis respectively with
minimum distance of the point of suspension from
equal amplitude and frequency of 2 cm and
the centre of mass of the body is l1 = 0.4 m to get
1 Hz respectively. Equilibrium positions for
this time period. Find the maximum distance (l2)
the particles A and B are at the coordinate
of a point of suspension from the centre of mass
(3, 0) and (0, 4) respectively. At t = 0, B is at its
of the body so as to get the same time period.
equilibrium position and moving toward the
[g = 10 m/s2]
origin, while A is nearest to the origin. Find the
Q. 20. A square plate of mass M and side length L is maximum and minimum distances between A
hinged at one of its vertex (A) and is free to rotate and B during their course of motion.
about it. Find the time period of small oscillations
Q. 23. A particle is performing SHM along x – axis and
if
equation for its motion is x = a cos (pt)
t
A Let the time t be expressed as = n + m

Where n = 0,1,2,3,4.... and m is a positive fraction.


Calculate the distance travelled by the particle
during the interval from t = 0 to t = t if
(a) m < 0.5 (b) m > 0.5
(a) the plate performs oscillations in the vertical
Q. 24. Two blocks A and B having mass m = 1 kg and
plane of the figure. (Axis is perpendicular to
M = 4 kg respectively are attached to a spring
figure.)
and placed vertically on a weighing machine as
(b) the plate performs oscillations about a shown in the figure. Block A is held so that the
horizontal axis passing through A lying in the spring is relaxed. A is released from this position
plane of the figure. and it performs simple harmonic motion with
angular frequency w = 25 rad s–1. The spring
Level 2 remains vertical.
A

Q. 21. Two particles A and B are describing SHM of same


amplitude (a) and same frequency (f) along a
common straight line. The mean positions of the
two SHMs are also same but the particles have B
a constant phase difference between them. It is
observed that during the course of motion the
Simple Harmonic Motion 12.5

(a) Find the reading of the weighing machine of the block (represented by line AB) always
as a function of time. Take t = 0 when A is remains horizontal.
released.
(b) What is the maximum reading of the weighing
machine?
Q. 25. A block of mass M rests on a smooth horizontal
table. There is a small gap in the table under
the block through which a pendulum has been
k1 k2
attached to the block. The bob of the simple
pendulum has mass m and length of the pendulum A B
is L. The pendulum is set into small oscillations
M
in the vertical plane of the figure. Calculate its
time period. The table does not interfere with the
motion of the string. Q. 28. (i) In the system shown in figure, find the
time period of vertical oscillations of the
M block A. Both the blocks A and B have
equal mass of m and the force constant of
the ideal spring is k. Pulley and threads are
massless.
L

m k
Q. 26. A circular wire frame of radius R is rotating about A
its fixed vertical diameter. A bead on the wire
remains at rest relative to the wire at a position
in which the radius makes an angle q with the
vertical (see figure). There is no friction between
the bead and the wire frame. Prove that the bead
will perform SHM (in the reference frame of the B
wire) if it is displaced a little from its equilibrium
position. Calculate the time period of oscillation. (ii) In the arrangement shown in the figure
the spring, string and the pulley are mass
less. The force constant of the spring is k.
A rope of mass per unit length equal to l
(kg m–1) hangs from the string as shown. In
equilibrium a length L of the rope is in air
O
and its bottom part lies in a heap on the floor.
The rope is very thin and size of the heap is
R
q negligible though the heap contains a fairly
long length of the rope. The rope is raised by
a very small distance and released. Show that
motion will be simple harmonic and calculate
the time period. Assume that the hanging part
Q. 27. In the system shown in the figure the string, of the rope does not experience any force
springs and pulley are light. The force constant from the heap or the floor (For example there
of the two springs are k1 = k and k2 = 2k. Block is no impact force while the rope hits the
of mass M is pulled vertically down from its floor while moving downward and there is no
equilibrium position and released. Calculate the impulsive pull when the vertical part jerks a
angular frequency of oscillation. The top surface small element of heap into motion).
12.6 Problems in Physics for JEE Advanced

(a) Find maximum value of mass (m) of the ball


for which the block will not lose contact with
the ground?
(b) If the stand is not tall enough and the ball
String makes elastic impact with the block, will your
answer to part (a) change?
k
Q. 31. Two ideal springs of same make (the springs differ
Rope L
heap
in their lengths only) have been suspended from
points A and B such that their free ends C and D
are at same horizontal level. A massless rod PQ
Q. 29. A box B of mass M hangs from an ideal spring of is attached to the ends of the springs. A block of
force constant k. A small particle, also of mass M, mass m is attached to the rod at point R. The rod
is stuck to the ceiling of the box and the system remains horizontal in equilibrium. Now the block
is in equilibrium. The particle gets detached from is pulled down and released. It performs vertical
the ceiling and falls to strike the floor of the box. m
oscillations with time period T = 2π where
It takes time ‘t’ for the particle to hit the floor after 3k
it gets detached from the ceiling. The particle, k is the force constant of the longer spring.
on hitting the floor, sticks to it and the system A
thereafter oscillates with a time period T. Find the
T
height H of the box if it is given that t . B
6 2 k
Assume that the floor and ceiling of the box always
remain horizontal.
P R Q
C D

k m

(a) Find the ratio of length RC and RD.

A
(b) Find the difference in heights of point A and
B if it is given that natural length of spring BD
H is L.
B
Q. 32. A block of mass M connected to an ideal spring of
Q. 30. A block has a L shaped stand fixed to it. Mass force constant k lies in equilibrium on the smooth
of the block with the stand is M. At the free end floor of a room. The other end of the spring is
of the stand there is a spring which carries a ball fixed to the left wall of the room. The room begins
of mass m. With the spring in its natural length, to move to the right with a constant acceleration
the ball is released. It begins to oscillate and the a0. In the reference frame of the room the block
stand is tall enough so that the ball does not hit the begins to perform simple harmonic motion.
block.

a0

k k M
x
x=0
m
At a certain instant (say t = t0) when the block was
M at its left extreme, the acceleration of the room
vanishes. Plot the x – t graph for the block taking
time t = 0 when the room started accelerating.
Simple Harmonic Motion 12.7

Show the graph till time t0 and beyond that. Take


origin to be at the left wall and positive x direction
towards right (as shown in figure). Assume no d d
collision of the block with walls.
Q. 33. A block of mass M connected to an ideal spring
of force constant k, is placed on a smooth surface. v
The block is pushed to the left so as to compress
the spring by a length A and then it is released. Q. 36. Two blocks rest on a smooth horizontal surface.
3 They are connected by a spring of force constant
The block hits an elastic wall at a distance
2 k. If the system is set into oscillation find its time
from its point of release. Assume the collision to period.
be instantaneous. k
m1 m2
(a) Calculate the time required by the block to
complete one oscillation
Q. 37. Two blocks A (2 kg) and B (3 kg) rest on a
(b) Draw the velocity – time graph for one
smooth horizontal surface, connected by a spring
oscillation of the block.
of stiffness k = 120 N/m. Initially, the spring is
A A/2
relaxed. At t = 0, A is imparted a velocity u = 2 m/s
towards right. Find displacement of block A as a
k
M
function of time.
x 3 kg 2 kg
O
B A
(c) Find the value of k for which average force
experienced by the wall due to repeated hitting Q. 38. A spring has force constant k = 200 N/m and its
of the block is F0. one end is fixed. There is a block of mass 2 kg
Q. 34. A particle of mass m is constrained to move along attached to its other end and the system lies on a
a straight line. A and B are two fixed points on the smooth horizontal table. The block is pulled so
line at a separation of L. When the particle is at that the extension in the spring becomes 0.05 m.
some point P, between A and B, it is acted upon by At this position the block is projected with a speed
two forces of 1 m/s in the direction of increasing extension of

  6mg   
  3mg   the spring. Consider time t = 0 at the moment the
F1 =   PA and F2 =   PB block is projected and find
 L   L 
(a) the extension (or compression) in the spring
At time t = 0, the particle is projected from A as a function of time.
towards B with a speed of gL .
(b) the maximum extension in the spring and the
At what time ‘t’ will the particle reach at B for the time at which it occurs for the first time.
first time? (c) the time after which the speed of the block
A P B becomes maximum for the first time.
Given: sin–1 (0.446) = 0.46 radian
Q. 35. An equilateral prism of mass m is kept on a smooth
table between two identical springs each having Q. 39. Two identical simple pendulums A and B are
a force constant of k. The two springs have their fixed at same point. They are displaced by very
lengths perpendicular to the inclined faces of the small angles a and b ( = 2 a) respectively and are
prism and are constrained to remain straight. The simultaneously released from rest at time t = 0.
ends of the springs have light pads aligned parallel Collisions between the pendulum bobs are elastic
to the faces of the prism, and distance between and length of each pendulum is l .
pads and the incline faces is d. The prism is (a) What is the minimum number of collisions
imparted a velocity v to the right. Find time period between the bobs after which the pendulum
of its oscillation. B will again reach its original position from
where it was released?
12.8 Problems in Physics for JEE Advanced

(b) Find the time (t) at which B reaches its initial


M
position for the first time after the release.
0 cm A
(c) Write the kinetic energy of pendulum B just 30
after nth collision? Take mass of each bob to
be m.

B
k
a b

30°

and then gets pushed back. How much time after


B its release, the block will be back to point A?
A

Q. 43. Two tunnels - T1 and T2 are dug across the earth as


Q. 40. Two spheres A and B of the same mass m and shown in figure. One end of the two tunnels have a
the same radius are placed on a rough horizontal common meeting point on the surface of the earth.
surface. A is a uniform hollow sphere and B is Two particles P1 and P2 are oscillating from one
uniform solid sphere. They are tied centrally to end to the other end of the tunnels. At some instant
a light spring of spring constant k as shown in particles are at mid point of their tunnels as shown
figure. A and B are released when the extension in figure. Then –
in the spring is x0. Friction is sufficient and
the spheres do not slip on the surface. Find the T1
frequency and amplitude of SHM of the sphere A.
A B
T2
K
P1
v2
v1 P2
60°
Q. 41. Two small blocks of mass m and 4m are connected
to two springs as shown in fig. Both springs have
stiffness K and they are in their natural length J
when the blocks are at point O. Both the blocks are
pushed so that both the springs get compressed by (a) Write phase difference between the particle
a distance a. First the block of mass m is released P1 and P2. Can the two particles ever meet?

a a (b) Write the ratio of maximum velocity of


m 4m
particle P1 and P2.
k k
O
Q. 44. The given figure shows the variation of the kinetic
energy of a simple pendulum with its angular
 
and after it travels through a distance 1 − 3  , displacement (q) from the vertical. Mass of the
 2  pendulum bob is m = 0.2 kg. Find the time period

the second block is also released. of the pendulum. Take g = 10 ms–2.
(a) At what distance from point O will the two
kE (10–3J)
blocks collide? 20
(b) How much time the two blocks need to
collide after the block of mass 4m is released?
10
Q. 42. A block of mass M = 40 kg is released on a smooth
incline from point A. After travelling through a
length of 30 cm it strikes an ideal spring of force q (10–3 rad)
-100 O 100
constant K = 1000 N/m. It compresses the spring
Simple Harmonic Motion 12.9

Q. 45. Two identical small elastic balls have been line AP.
suspended using two strings of different length (b) Assuming that the triangle APB makes a
(see fig (a)). Pendulum A is pulled to left by a small angle q with the vertical plane, write the
small angle q0 and released. It hits ball B head restoring torque acting on the rod.
on which swings to angle 2q0 from the vertical.
Calculate the time period of oscillation of A if its (c) Calculate the time period of small oscillations.
length is known to be L. A 1.25 L P

0.75 L
L

q0
L
2q0 B

Q. 48. A railway tank wagon with its 2m diameter and


6m long horizontal cylindrical body, half full of
petrol is driven around a curve of radius 100m, at
a speed of 8.33 m/s. The curve runs smoothly into
A B a straight track and the train maintains a constant
(a) (b) speed. Find the angular amplitude and frequency
of subsequent oscillation of the petrol due to this
Q. 46. A simple pendulum of length L has a bob of mass change of motion. Neglect viscosity and consider
m. The bob is connected to light horizontal spring petrol as a solid semi cylinder sliding inside the
of force constant k. The spring is relaxed when the tank. Given: tan–1 (0.07) ª 4º
pendulum is vertical (see fig (i)).
Q. 49. A pendulum consists of an inextensible thread
(a) The bob is pulled slightly and released. Find connected to a solid spherical ball of radius r.
the time period of small oscillations. Assume The distance between the point of suspension
that the spring remains horizontal. and the centre of the ball is L (>> r). Calculate
(b) The spring is replaced with an elastic cord the percentage difference in the time period of this
of force constant k. The cord is relaxed when pendulum to the time period of a simple pendulum
the pendulum is vertical (see fig (ii)). The bob of length L. How large is this difference for
is pulled slightly and released. Find the time r = 5 cm and L = 100 cm.
period of oscillations. Q. 50. A disc of radius r is connected to a string of length
L. The string is tied to a point on the circumference
of the disc. This system is made to oscillate in
vertical plane of the disc with a small angular
l l
amplitude q0. Find the speed of the lowest point of
the disc at the moment the string become vertical.
O
k k

(i) (ii)

Q. 47. A uniform rod AB of mass m and length L is tied, L


at its end B, to a thread which is attached to point
P on the ceiling. Length of the thread PB is 0.75
L. The other end A of the rod is hinged at a point r
on the ceiling. Distance AP = 1.25 L. End B of the
rod is pushed gently perpendicular to the plane of
the figure and it starts oscillating Q. 51. (i) A cylindrical container has area of cross
section equal to 4A and it contains a non
(a) Find the moment of inertia of the rod about
viscous liquid of density 2r. A wooden
12.10 Problems in Physics for JEE Advanced

cylinder of cross sectional area A and length Find the frequency of small oscillations of the
L has density r. It is held vertically with particle about its equilibrium position
its lower surface touching the liquid. It is
released from this position. Assume that the Level 3
depth of the container is sufficient and the
cylinder does not touch the bottom.
Q. 54. Two particles of mass m1 and m2 are connected
(a) Find amplitude of oscillation of the wooden by a spring of natural length L and force constant
cylinder. k. The masses are brought close enough so as
(b) Find time period of its oscillation. to compress the spring completely and a string
is used to tie the system. Assume that length of
spring in this position is close to zero. The system
L
is projected with a velocity V0 along the positive x
direction. At the instant it reaches origin at time t
= 0, the string snaps and the spring starts opening.
Vo
m2 m1
X
o
(a) Show that the mass m1 (or m2) will perform
(ii) Two cubical blocks of side length a and 2a SHM in the reference frame attached to the
are stuck symmetrically as shown in the centre of mass of the system. Find the time
figure. The combined block is floating in period of oscillation.
water with the bigger block just submerged
(b) Write the amplitude of m1 and m2 as a
completely. The block is pushed down a
function of time.
little and released. Find the time period of its
oscillations. Neglect viscosity. (c) Write the X co ordinates of m1 and m2 as a
a function of time
Q. 55. Two simple pendulums A and B have length 4 l
and l respectively. They are released from rest
from the position shown. Both the angles a and b
2a
are small. Calculate the time after which the two
string become parallel for the first time if–
Q. 52. A hollow cylindrical shell of radius R has mass M. (a) a = b (b) b = 1.5 a
It is completely filled with ice having mass m. It is
placed on a horizontal floor connected to a spring
(force constant k) as shown. When it is disturbed a l
b
it performs oscillations without slipping on the
B
floor. 4l

A
(a) Find time period of oscillation assuming that Q. 56. A simple pendulum has a bob of mass m and it
the ice is tightly pressed against the inner is oscillating with a small angular amplitude of
surface of the cylinder. q0. Calculate the average tension in the string
(b) If the ice melts into non viscous water, find averaged over one time period. [For small q take
the time period of oscillations. (Neglect any θ2
cos θ  1 − ]
volume change due to melting of ice) 2
Q. 53. A particle of mass m is free to move along x axis Q. 57. Assume a smooth hole drilled along the diameter
under the influence of a conservative force. The of the earth. If a stone is dropped at one end it
potential energy of the particle is given by reaches the other end of the hole after a Time T0.
U = – axne–bx [a and b are positive constants]
Now instead of dropping the stone, you throw it
Simple Harmonic Motion 12.11

into the hole with an initial velocity u. How big two SHMs. Write frequency of the component
should u be, so that the stone appears at the other SHMs.
T0
end of the hole after a time . Express your
2
answer in terms of acceleration due to gravity on k m k m k
1 2
the surface of the earth (g) and the radius of the +ve
earth (R).
Q. 58. A large horizontal turntable is rotating with Q. 61. Four identical mass less rods are connected by
constant angular speed w in counterclockwise hinged joints to form a rhombus of side length L.
sense. A person standing at the centre, begins to Rods can rotate freely about the joints. The joints
walk eastward with a constant speed V relative B and D are connected by a mass less spring of
to the table. Taking origin at the centre and X relaxed length 1.5 L. The system is suspended
direction to be eastward calculate the maximum X vertically with a load of mass M attached at C
co-ordinate of the person. (see fig). In equilibrium the rods form an angle
of 30° with the vertical. Find time period of small
Q. 59. A spherical cavity of radius is removed from oscillations of the load.
a solid sphere of radius R as shown in fig. The A
sphere is placed on a rough horizontal surface as
shown. The sphere is given a gentle push. Friction
L L
is large enough to prevent slippage. Prove that the
sphere perform SHM and find the time period.
B D

R/2
R
L L

o C

Q. 62. Two identical blocks 1 and 2, each of mass m, are


Q. 60. Two blocks 1 and 2, each having mass m, are placed kept on a smooth horizontal surface, connected to
on a smooth table connected to three identical three springs as shown in the figure. Each spring
springs as shown in the figure. Each spring has a has a force constant k. Under suitable initial
force constant K. Initially, all springs are relaxed. conditions, the two blocks oscillate in phase and
The system is disturbed and starts moving. Let their respective displacement from the mean
x1 and x2 represent the displacements of the two position is given by
blocks from their respective mean positions.
m m
(a) Prove that the quantity A = x1 + x2 varies k k k
sinusoidally and calculate its angular 1 2
frequency wa.     
(b) Prove that the quantity B = x1 – x2 varies x1 = A sin wt and x2 = A sin wt
sinusoidally and calculate its angular (i) Suggest one such initial condition that will
frequency wb. result in such oscillation.
(c) Prove that motion of block 1 is superposition of (ii) Find w
12.12 Problems in Physics for JEE Advanced

ANSWERS

π  
(i) T =
b 16.  2 + 1 
 2 2 
 
(ii) Vmax = 3 2 m/s, AP/BP = ( 2 –1) / ( 2 + 1)
17. (i) h = 1.25 m (ii) T > T0

π π  π π 
(a)
v1 = A cos  t  and v2 = 2 A sin  t 
t0 1 g
 t0  t0  t0  18. f =
2π 2R
(b) 2 : 1
19. 0.6 m
. (a) Yes (b) T = 2 (c) 2 unit
20. (a) 2p 2 2 a
4. x1 = a; x2 = 0 3 g
 
5. (a) circle of radius a.   (b) 2π  ω  (b) 7 a
2p
 ω0  6 2g
π
6. (a) = (b) yes 2π
ω . (a) φ1 − φ2 = (b) 2f
3
1 π
7. (i) ∆ = (ii)
A m; t = s 4π 2  2π 
2
. (i) − ma cos  t
T2  T 
A k
8. (ii) 7 cm and 3 cm.
M

mg 3 23. (a) s = 4an + a(1 – cos pt)
9. (a) (b) mg
k 2
Mg mg s = 4an + a(3 + cos pt)
(b)
10.
k k 24. (a) 50 – 10 cos (625t) (b) 60 N

11. p 1
25. T = π
LM
2 ag g ( M + m)

 k +k 
12. (a) a =  1 2  x ; T = 2π m1 + m2 R cos θ
. T = 2π
 m1 + m2  k1 + k2 g
m1 k1 m
(b) ; T = π for both the blocks in both 8k
m2 k2 k 27. ω =
cases. 3M

f m λL
28. (i) T = π (ii) T = π
k k + λg
13. (a) No (b)
t
29. H = Mg 1 + π 
2

1
1
2k  9 
 m2 r 2  4
(a) F2 2
(mg ) (b) 2π  2 2 
m  F − (mg )  30. For both (a) and (b) the block will not lose contact
with the ground for any value of m.
ml
15. 2p , Time period increases.
T
Simple Harmonic Motion 12.13

RC 2 1 46 k 21
(a) (b)
L. f= ; A1 = x0
RD 1 2p 35 m 46
32.
5π 5p
(a) cos   (b) m
x

 18  9 k
L + 2b
42. 1.54 s
L (a) 180°, No (b) 2 : 1
L – 2b T = 2.80 s

1  −
1
− 
1
g k  2  g k  2  g  2
(a) 2 π  +  (b) π  +  +   
t  l m  l m  l 
O t0  
3 3
(a) mL2 (b) mgL
33. (a) 4p M 25 10
3 k
(c) 2L
(b) v 2p
5g
v0
Amplitude = tan–1 (0.07) = 4º; frequency = 0.46 Hz.
v0
2
20r 2
% ; 0.05 %
O
t0 2t0
t L2

v0 g (L + r)
2 θo ( L + r )
– v0
r + (L + r)
3
(i) (a)     (b) 2p 3L
4p F0 8 8g
k=
(c)
3 3A
(ii) 3p 2a
2π L g
34. t =
3 3g
4M + 3m
(a) 2π ; ;
8d m 2K
35. + 4p
3v 3k
M +m
(b) π .
µ K
36. T = π ; where
k
1 ae− n n n −1
. xA = 0.12 sin (10t) + 0.8t f=
2π mb n − 2
38. (a) x = 0.112 sin (10t + 0.46) m
(b) 0.112 m, 0.111 s
. (a) T = 2π m1m2
(c)
t = 0.268 s ( m1 + m2 ) k
39. (a) 2 (b) t = 2p
l m2 L
(b) = A1
g m1 + m2
1 2 1 2 X1 = V0t + A1 (1 – cos wt);
(c)
E = ml
(c) g if n is odd and E = ml g if n is
2 2 X2 = V0t – A2 (1 – cos wt)
even Where
12.14 Problems in Physics for JEE Advanced

m2 L m1 L k ( m1 + m2 ) k
A1 = ; A2 = ; ω= (a) ωa =
m1 + m2 m1 + m2 m1m2 m

2p l l  19 − 1  k
2 . cos −1  ωb =
(b)
(a) (b)  m
3 g  6 
1 wa and wb
(c)
Tav = mg + mg q 02
4
u= gR L
. T = 2π
2 3g

k
. (ii) ω =
177 R m
T = 2π
10 g

SOLUTIONS

(i) The given equation has a standard solution given by


a = a0 sin (wt + ). Where w = b.
This is an equation of SHM.
π π
T= =
ω b
(ii) Careful observation of the data tells us that the time period of SHM = 2 s
0.75 s
0.25 s
O 3 m/s
B A
P
0.75 s 0.25 s

If we consider t = 0 when the particle is at origin and travelling in positive direction, we can write the equation
of motion as-
x = A sin wt v = Aw cos wt
Particle will reach at P when time is t = 1/4 s
2π  2π 1  3 2
∴3 m / s = A cos  .  ⇒ =
2  2 4 π
vmax = Aw = 3 2 m/s

OP = A sin ωt =
3 2 π  3
sin   = m
π 4 π
Now, it is easy to work out the ratio AP/PB.
2.
(a) A careful observation of the given graphs reveals that the time period is same for both the particles. Amplitude
p
of 1 and 2 are A and 2A respectively and particle 2 lags in phase by .
2
Position – time equation for the two particles is –
 π
x1 = A sin wt and x2 = 2 A sin  ωt −  = – 2A cos wt
 2
Simple Harmonic Motion 12.15

2π π
Where ω = =
20 0

We get velocity by differentiating the above two equations.

π π  π π 
v1 = Aw cos wt = A cos  t  and v2 = 2 Aw sin wt = 2 A sin  t 
t0  t0  t0  t0 
1
m1ω 2 A
E1 2
(b) = =1
E2 1 m ω 2 2 A
2
2

Hence, 1 2
2 1
3. Motion is simple harmonic. The origin of the co-ordinate system is not the equilibrium position for the particle. In
equilibrium force on the particle shall be zero. Thus x = 2 is the equilibrium position.

d2x
= − β ( x − 2)
dt 2
Let (x – 2) = X

d x d X
Then
dt dt

d X
Hence, = −β X
dt
1
\ w2 = b fi T = 2p
b
5. (a) The observer is moving in xy plane with his position vector changing with time as

R = (a sin wt) î + (a cos wt) ĵ
This is a circle with radius a. Angular speed of the observer is w and his linear speed is v = aw.
He finds that the particle oscillates along z direction with angular frequency = w0.
(b) Time period of rotation of the particle is

0 =
ω0
ω 
s = v. T0 = 2pa  
ω 
7. (i) x = A sin wt
v = v0 cos wt
w

v0

q2 q1
B O A
12.16 Problems in Physics for JEE Advanced

If we consider a vector of length v0 rotating with angular speed w then x co-ordinate of its tip gives the
instantaneous velocity of the particle performing SHM.
3
From figure OA v0
5
3
sin θ1 = ; ⇒ q1 = 37º
5
4
OB v0
5
4
sin θ 2 =
⇒ θ 2 = 53º
5
At A speed is increasing and at B it is decreasing. In interval t the phasor rotates through 90°.

T
∆t =

(ii) 2p
w= = 2 rad /s
T
Angular frequency is property of the system and it does not change with change in energy. Let A be the new
amplitude. Speed at a distance x from equilibrium is given by

v = ω A −x
⇒ 1 = 2 2
− 0.52 ⇒ A = 1 m
2
In the described position the particle is at x = 0.5 m travelling towards mean position. Since the time period
has not changed the required time is time of travel from the positive extreme to x = 0.5 m. This time can be
calculated as-

1 1 π
cos(2 ) ⇒ t = s
2 2

k
8. ω=
M
k
For the block Vmax = Aw = A
M
Speed of any point on the spring is proportional to its distance from the fixed end A.

Vmax A k
(VP )max =
=
4 4 M
. Hint: The spring shall remain stretched – always.
Mg
. The block will lose contact if the spring compresses more than
. In equilibrium position the spring is stretched
k
mg
by . If the spring oscillates with amplitude A , it will move up above its equilibrium position by A. It means
k
mg
compression in this extreme position will be A . For the block to remain on the table –
k
Mg mg
A − =
k k
Simple Harmonic Motion 12.17

Mg mg
A = +
k k

d x
m = − µ mg
dt
d2 x
= - agx
dt 2
This is equation of SHM whose time period is
1
T = 2π
ag


The block will slide for a time
T π 1
t = =
4 2 ag

12. (a) As springs are in parallel


Fnet (k + k ) x
a = = 1 2
mass (m1 + m2 )

ω 2 = k1 + k2 T = 2π
m1 + m2
m1 + m2 k1 + k2

(b) Frictional force on m2 will act in direction of displacement if k2x > m2 a


(k1 + k2 ) x m1 k1
k2 x > m2
(m1 + m2 ) m2 k2
(ii) Hint: The time period does not depend on amplitude or energy.
(a) The maximum friction force that can act on 10 kg block is mmg = 0.5 × 10 × 10 = 50 N
Maximum spring force = kx = 800 × 0.01 = 8N
The friction acting on 10 kg block is large enough to prevent its slipping. So it will not move.
(b) The 2 kg block will perform SHM. Equation of it motion will be –
x = A cos (wt)
K
Where ω = and A = 0.01 m
m
The instantaneous friction acting on the block = fs = KA cos (wt)
f s = KA cos ωt f

t
(a) The vertical and horizontal component of tension force is equal to mg and mw2r respectively (w is angular
speed of rotation of the particle). Hence,
F2 = (mg)2 + (mw2r)2
1
w r = F 2 − (mg )2
m
1
 F 2 − ( mg )2  4

ω =  
 m2 r 2 
 
12.18 Problems in Physics for JEE Advanced

1
 m2 r 2 4
∴ T = 2π  2 
 F − ( mg ) 
2
 
This will be the maximum acceleration of the shadow as it will perform SHM with amplitude r and angular
frequency w.
(b) from the above equation
1 1
 F 2 − (mg )2  4  m2 r 2  4
ω =   ∴ T = 2π
2 2  2 
 mr
2
  F − (mg ) 
When the ball has a small displacement as shown in the figure, the wires will be inclined slightly to the vertical.
The horizontal component of tension will be the net unbalanced force on the ball

ma = – 2T
l
T
Or, a = – x
m l
This is the equation of simple harmonic motion where
T
ω =
ml
Time period is

π ml
= = 2p
ω T
We see that the time period is inversely proportional to the square root of the tension. Thus by increasing the tension
the time period of oscillation will decrease.
16.

l
2

l
T= π
g

T l / 2 T T 1 T  2 +1
T '= +π
= + =  
2 g 2 2 2 2  2 

17. (i) Because e  1, the ball A will have a very small velocity after collision and ball B will have speed slightly less
than that of A before collision.
Hence, A will move like a simple pendulum after collision with time period

L
T = 2π  2.0 s .
g
A will come to rest 0.5 s after collision.
Simple Harmonic Motion 12.19

It means time of fall for B is 0.5 s. Time of fall of B is independent of the horizontal speed acquired by it.
h 2h
t 0.5
g g
h = 1.25 m

(ii) Dimensional analysis tells us that

l
T k where k is a constant dependent on q0.
g

2
0

18. In equilibrium the particle is at the lowest position. Consider the system at an angular position q.

R
q

mg
t = mg R sin q  mg R q
(for small q)
I a = t

1 2
 2 ( 2m ) R + mR  α = −mgRθ
2

 
 g  1 g
∴α = − 
θ ∴ f =
 2 R  2π 2R
I0 = MOI of the body about an axis through COM.
l = distance of point of suspension from the COM.
Time period of oscillation of a physical pendulum is
I
T= π
[I = MOI about rotation axis = I0 + ml2]
mgl

I 0 + ml 2
0.2π = 2π

mgl

I 0 + ml 2
0.01 =
mgl
I0 + ml2 = 0.01 mgl

I0 – 0.01 mgl + ml2 = 0



The two solutions to this equation have sum of

l1 + l2 = 0.01g = 0.1
12.20 Problems in Physics for JEE Advanced

If l = 0.4 m then l2 = 0.6 m


All points lying on two concentric circles around COM having radii 0.4 m and 0.6 m, are point of suspensions
which will give T = 0.2p s.
20. (a) Moment of inertia about the axis of oscillation is
2
I A Ma 2
3
Distance of COM from point of suspension is
l a

IA
T = π
mgl

2
Ma 2
3 2 2 a
T = 2π = 2π
a 3 g
Mg
2
(b) In this case
7
I A Ma 2
12
7
Ma 2
7 a
T = 2π 12 = 2π
a 6 2g
Mg
2
21. (a) x A = a sin (wt +f1 ) and x B = a sin (wt + f2 )
φ +φ φ −φ
x A – x B = a sin (wt + f1 ) – a sin (wt + f2 ) = 2a sin  ω + 1 2  cos  1 2 
 2   2 
The distance between the two particles will be expressed by the modulus of the above expression.
Obviously, the sine term in the last expression can have a maximum value of 1, therefore maximum separation
of a implies that

φ1 − φ2  1
cos  =
 2  2

φ −φ π 2π
 1 2  = φ1 − φ2 =
 2  3 3
 φ +φ  φ −φ 
(b) Distance between the two particles is = 2a sin  ωt + 1 2  cos  1 2 
 2   2 
This will complete two oscillation in the period when the sine term makes one complete oscillation. Hence,
frequency of oscillation will be double the frequency of oscillation of the particles.

π
(i) ω = and amplitude A = 2a

 2π 
Equation of SHM along AB will be r = 2a cos  t
 T 
Simple Harmonic Motion 12.21

X co-ordinate as a function of time is


 2π 
x = r cos 45° = a cos  t
 T 

d2x 4π 2  2π 
Fx = m 2
= − 2 ma cos  t
dt T  T 
(ii) The co-ordinates of the two particles as a function of time can be written as
x = 3 – 2 cos wt, and y = 4 – 2 sin wt,

Distance between them is l = x + y = 29 − 12 cos ω − 16 sin ω


Maximum and minimum values of 12 cos wt – 16 sin wt are 20 and –20 respectively.
Hence, maximum and minimum values of distance are 7 and 3 respectively.
π π
23. Time period of oscillation = = =
ω π
t t
=   expresses the number of completed oscillations = n (an integer) + m (a fraction)
T  
When m = 0, particle has made n number of complete oscillations and distance travelled is
s = (4a) (n)
(a) If m is a fraction less than 0.5, it means the particle is moving towards the negative extreme and its distance
from the positive extreme is
a – a cos (pt)
for m < 0.5
s = 4 an + a (1– cos pt)
(b) If m > 0.5, the particle has completed n and half oscillations and is travelling back towards the positive extreme.
s = 4 an + 2a + (a + y)
= 4 an + 3 a + a cos pt
= 4 an + a (3 + cos pt)

k
24. ω =
m

25 k = 625 N/m
1
Compression in the spring in equilibrium position is
mg 1 × 10 2
x0 =
= = m = 1.6cm
k 625 125
One extreme position of A is the natural length position of the spring. Therefore, amplitude of oscillation is
A = 1.6 cm.
For SHM of block A
x = x0 cos (wt) [with equilibrium as origin and upward direction as positive]
   
d2x
acceleration a = 2 = − x0ω 2 cos(ωt )
dt
For (A + B) as system:     [N = normal force by the weighing machine]
N – (M + m) g = ma
12.22 Problems in Physics for JEE Advanced

N = 5g – mx0w2 cos (wt)


2
N = 50 − 1 × × 625 cos(625t )
125
= 50 – 10 cos (625t)
25. Consider the string at a small angle q to the vertical.
a0

If T is tension in the string,


T sin q = Ma0 (i)
Now, we will consider the motion of the pendulum in the reference frame attached to the block.

x
ma0

mg
­

For small oscillations, the path of pendulum bob can be approximated to be a straight line and
T = mg cos q  mg
The restoring force = mg sin q + ma0

T sin θ
= mg sin θ + m
M
m 2 g sin θ
= mg sin θ +

M
 m
 mg  +  θ
 M

d2x  m
m 2 = −mg 1 +  θ

dt  M

d x M + m x
= − g     [SHM]
dt  M L

gM +m
ω =  
L M 
Simple Harmonic Motion 12.23

LM
∴ T = 2π
g ( M + m)

26. In reference frame of the wire, the equilibrium of bead gives
mw2 r cos q = mg sin q [Equilibrium along tangent]
w2r = g tan q    … (i)
If q is increased by a small amount, say q, the tangential force [along upward tangent] also changes.

O mwr cosq

q R
r mw2r

mg
mg sin q

Ft = mw2r cos q – mg sin q


d
∆Ft = [m w2r cos q – mg sin q] q = [– m w2r sin q – mg cos q ] q

d 2 ( R∆θ )
m = [−mω 2 r sin θ − mg cos θ ]∆θ
dt 2
d 2 ( ∆θ ) 1 g  sin 2 θ 
= [− g tan θ .sin θ − g cos θ ]∆θ = −  + cos θ  ∆θ = −  g  ∆θ
dt 2 R R  cos θ   R cos θ 
Hence motion is SHM.

g R cos θ
ω2 = ⇒ T = 2π
R cos θ g
27. Let x0 be stretch in spring of force constant k2 in equilibrium position. Then stretch in the other spring must be 2x0
so that tension is same in both of them.
2kx0 + k (2x0) = Mg
4 kx0 = Mg    … (i)
Let the block be displaced further by x causing the two springs to stretch further by x1 and x2 respectively.
k1x1 = k2x2 x1 = 2x2
And x1 + x2 = 2x
2 4
2
and 1
3 3
d x
M Mg – k1 (x1 + 2x0) – k2 (x2 + x0)
dt
d2x 4 4kx
M 2
= − kx − [using (i)]
dt 3 3   
12.24 Problems in Physics for JEE Advanced

d2x 8k
2
=− .x   [SHM]
dt 3M

8k
ω=
3M

28. (i) Let the extension in spring be x0 in equilibrium.

kx0 k (x + x0)

A A

mg
mg
T = T'
2
T2 T2

T'
T1
B
B
mg
mg

For B T' = mg
T' 3
For A kx0 = mg + ⇒ kx0 = mg    … (i)
2 2
Consider the block A in position that is displaced x from equilibrium. The corresponding displacement of B
from its equilibrium position will be .

d2x
For A m = T2 + mg − k ( x + x0 )     … (ii)
dt 2
d 2 ( x / 2)
For B m = mg − 2T2 [Q T1 = 2T2]
dt 2    
d2x
m = 2mg − 4T2 … (iii)
dt 2     
Multiplying equation (ii) with 4 and adding to equation (iii)
d2x
5m = 6mg − 4kx − 4kx0
dt 2
But from (i) 6mg = 4 kx0
d2x 4k
2
=− x
dt 5m
d x
Comparing with = −ω x
dt
4k
We get ω = 5m

5m 5m
T = 2π =π
4k k
Simple Harmonic Motion 12.25

(ii) In equilibrium the string tension (which is equal to the spring force) applies an upward force equal to weight of
the hanging part of the rope. If the rope moves a distance x downward, the restoring force is equal to increase
in spring force plus the decrease in weight of the hanging part.
Restoring force = kx + lxg
Mass of rope in motion = lL.
This mass changes a little as the rope oscillates but that can be neglected compared to lL.
d2x
λ L = −( k + λ g ) x
dt 2

d x  k g
= − + x
dt  λL L 
k g
ω = +
λL L

1 λL
T = 2π = 2π
k g k + λg
+
λL L
29. Originally, the system keeps the spring stretched by a length x0 where -
kx0 = 2Mg   … (i)
As soon as the particle gets detached, it begins to fall with acceleration g and the box begins to perform SHM with
amplitude

Mg Mg Mg
A=
− =
k k k
1 2
Distance travelled by the particle in time t is s1 gt
2
Distance travelled (upward) by the floor in time t will be
s2 = A – A cos (wt)

k
Where ω =
M

Mg Mg  k 
s2 = − cos  t 
k k  M 
H = s1 + s2

1 2 Mg Mg  k 
H= gt + − cos  t  ……(ii)
2 k k  M      
T M
It is given that t where T = π
6 2 k
π M
t=
k

1 π 2 M Mg Mg  π  Mg  π 
2
H= g + − cos   = 1 + 
2 9 k k k  3  2k  9 
12.26 Problems in Physics for JEE Advanced

30. (a) The ball is released from a position where the spring is relaxed.
mg
The mean position is below the initial position.
k
mg
Ball will oscillate with amplitude A
k

Natural
length
Fs A

Equilibrium

mg

The spring will never get compressed as the upper extreme position of the block will be the natural length
position of the spring.
Hence spring will never exert any upward force on the stand.
Block will never lose contact with ground.
(b) In elastic collision the ball will not lose any KE. Hence, its upper extreme position does not change.
Hence, our answer remains same.
m
31. The time period for vertical oscillations of the block is T = 2π
3k
This means that the effective force constant is 3k.
Spring BD must have a force constant of 2k.
[Springs AC and BD are in parallel]
(a) Rod PQ is massless. Net force and torque on it must be zero.
2kx

kx

R
C D

3kx

(RC) (kx) = (RD) (2kx)

RC 2

RD 1

1
(b) Force constant of spring
length of spring
length of AC must be double that of spring BD.
Height difference between A and B will be L.
Simple Harmonic Motion 12.27

When the room moves with acceleration a0, the equilibrium position of the block (In reference frame of the room)
will be where the compressed spring balance the pseudo force ma0.
b
Mao
kb
x
x=o x=L
EQ

Let compression in equilibrium be b


Ma0
kb = Ma0 b .
k
As the block was initially at X=L, it will start oscillating about X = L–b with an amplitude equal to b.
At t = t0 the block is at x = L-2b when the constant force (Ma0) acting on it vanishes. This does not change the
frequency of oscillation but the equilibrium position now shifts to X = L. The block oscillates with an amplitude of
2b about X = L.
Hence, the X – t graph will be as shown in fig below.
x

L + 2b

L – 2b

t
O t0

33. (a) Time required for a particle performing SHM to travel from x = 0 to x = A/2 is given by
A 2π 
= A sin  t
2  T 
T
t    [T = time period of SHM]
12
Time period of oscillation of the block is
T T 2T 4π M
T0 = 2 × + 2× = = … (i)
4 12 3 3 k    
(b) Equation of motion of the block from the time of start to the time it hits the wall is

 k 
x = – A cos wt ω = 
 M

dx
Velocity v = = Aω sin ωt
dt
2π M
[for t = 0 to t0 = ]
3 k
After this the block hits the wall and direction of velocity changes suddenly.
Graph will be as shown in the figure.
12.28 Problems in Physics for JEE Advanced
v

v0

v0
2

O t
t0 2t0
v0

2

– v0

2
 A 3
(c) Speed of block just before hitting the wall is v = ω A2 − x 2 = ω A2 −   = Aω
2 2

F0 = (change in momentum of block during collision) × (frequency of collisions)

 
= 2.M  3 Aω  × 1 = 3MAω. 3ω [from (1)]
 2  T 4π
  0
    
3 3 k
= MA
4π M

3 3
F0 = Ak     [This is independent of M!!]

4π F0
k =
3 3A
34. Let the x axis be along AB
x (L – x)

x
A B
P

Let AP = x and PB = L – x
When the particle is at P
d2 x mgx ˆ 3mg
m 2 = -6
i+ ( L - x ) iˆ
dt L L

3mg
=
L
[ L - x - 2 x ]î
3mg
=-
L
[3 x - L ]î
d2 x 3g

2
=-
L
[3 x - L ]iˆ
dt
L g
This is equation of SHM with equilibrium position at x , and ω = .
L
L
T = 2π
3g
Simple Harmonic Motion 12.29

At A; x = and v = gL

 L2 
v2 = ω 2 a2 − 
 9

3g  2 L2 
gL = a − 
L  9

L2 L2
+ = a2
3 9
2
a L
3
It means B is extreme position of SHM; and distance of A from mean position is .
a
Time needed to reach mean position from x = ± is given by

a
= a sin ωt
2
π T
t= =
6ω 12
T T T 2π L
time from A to B is t = + = = .
12 4 3 3 3g
35. The prism will hit the spring to the right if it travels through a distance x0 such that
2d
x0 cos 30º = d x0
3
2d
Time needed to cover this distance x0 is t1
3v
3
If the prism moves further by x it compresses the spring by x cos 30º =
2
3
Spring force on it will be k x perpendicular to incline
2
 3  3
Horizontal component of this force =  kx  cos 30º = kx
 2  4

The prism will perform half oscillation with the spring consuming a time of

m m
t2 = π = 2π
3k 3k
4
The prism is back to its original position moving in opposite direction at a time
t = t1 + t2 + t1
The same story is repeated in opposite direction with the other spring.
T = 2t = 4t1 + 2t2
8d m
= + 4π
3v 3 k
12.30 Problems in Physics for JEE Advanced

m1 m2 Equilibrium

x1 x2

F F

F = kx

Let x1 and x2 be displacement of the two blocks at any point of time.


Extension in the spring is x = x2 – x1
d 2 x1 d 2 x1 k
For motion of m1; m1 kx x …(i)
dt 2 dt 2 m1
  

For motion of m2; m d x = − kx d x


=−
k
x …(ii)
dt dt m    
Equation (ii) – (i)

d 2 ( x2 − x1 )  1 1 
= −k  + x
2
dt  m1 m2 
d x k m1m2
= − x [where m = ]
dt µ      m1 + m2

k µ
ω= ⇒T = π
µ k
37. From the last problem we know that

d x k 3× 2
= −   x where µ = = 1.2kg
dt µ 3+ 2

k 120
ω= = = 10 rads/ s
µ 1.2
x = x0 sin (10t)
Where x = xA – xB
xA – xB = x0 sin (10t)   …. (i)

B A Equilibrium

xB xA

B A

The velocity of COM of the system is


Simple Harmonic Motion 12.31

2× 2 + 0
vcm = = 0.8m / s = a constant.
2+3
Displacement of COM in time t is xcm = vcm t = 0.8t
2 x A + 3 xB
But xcm =
2+3
2 x A + 3 xB
= 0.8t
5
2xA + 3xB = 4t   …. (ii)
Solving (i) and (ii)
xA = 0.6 x0 sin 10t + 0.8    …. (iii)
Now, we need to find x0 (maximum extension in the spring)
At the point of maximum extension, the velocity of two blocks will be equal to vcm = 0.8 m/s
From energy conservation
1 1 1 1
× 2 × 22 = × 120 × 2
0 + × 2 × 0.82 + × 3 × 0.82
2 2 2 2
x0 = 0.2
From (iii) we get xA = 0.12 sin (10t) + 0.8t
38. Let extension be x at any time t.
x

kx

Natural
length
d x
Then m = −kx
dt
d2x d2x
or, 2 = −200 x or, = −100 x
dt 2 dt 2
Motion is S.H.M. with w = 100 = 10 /s
x = A sin (wt + d) = A sin (10t + d)
dx
And v = = Aω cos (ωt + δ ) = 10 A cos (10t + δ )
dt
At t = 0, x = 0.05m 0.05 = A sin d     … (a)
At t = 0, v = 1 m/s 1 = 10 A cos d     … (b)
From (a) and (b) (0.05)2 + (0.1)2 = A2
or A2 = 0.0025 + 0.01 = 0.0125 [you can also use v2 = w2 [A2 – x2] to get A]
A = 0.112 m
0.05
From (a) sin δ = = 0.446
0.112
d = 26.5º = 0.46 radian
x = 0.112 sin (10t + 0.46)
12.32 Problems in Physics for JEE Advanced

(b) Maximum extension = A = 0.112 m


Time at maximum extension is given by
0.112 = 0.112 sin (10t + 0.46)
10t + 0.46 = p/2 = 1.57
10t = 1.11 t = 0.111 sec
(c) Speed is maximum when v = |10A cos (10t + d)| is maximum
i.e., 10t + d = p
or, 10t = 3.14 – 0.46 = 2.68
t = 0.268 sec
39. (a) After first collision, B acquires amplitude of A and after second collision it acquires its original amplitude.
Therefore, after 2 collisions B will once again go back to its original extreme position.
l
(b) Time period of both A and B is T = 2 p
g
Time consumed before B reaches its original extreme for the first time is
T T T T l
t = + + + = T = 2π
4 4 4 4 g
(c) If n is odd , the energy of B will be equal to original energy of A.
1 2 2 1 2 g 1
E = ma Aω = m ( lα ) = mlα 2 g
2 2 l 2
If n is even, the energy of B will be equal to original energy of B.
1 2 2 1 2 g 1
E = maBω = m ( l β ) = ml β 2 g
2 2 l 2
40. Let x & x be the displacement of the two spheres from the equilibrium position. Let their respective speed in this
1 2
position be v & v
1 2
Compression in the spring = (x1 + x2)
For hollow sphere, applying tA = I A a about the point of contact
5
k (x1 + x2) r = m r2 a1
3
5
k (x1 + x2) = ma1      ………(1)
3
Let’s apply angular momentum conservation about point of contact of ball A for the entire system,
5 7
mv1r mv2 r
3 5
25 v1 = 21 v2        ………(2)
25 x1 = 21x2        …...….(3)

Using (1) and (3) we get, k = m

46 k
a1 x1 (SHM)
35 m   
1 46k
f =
2π 35m
Simple Harmonic Motion 12.33

Now, sum of amplitudes of the two spheres is A1 + A2 = x0   …… (4)

25
From equation (3) we get 2 1
        …….. (5).
21
21
By (4) and (5) we get, A1 x0
46
x1 x2

A a2
B
f1 a1 f2

Till the point of collision, motion of both the blocks remain simple harmonic. For spring block system of mass m,
m
the time period of SHM will be 2π = t ( say )
k
π k
Angular frequency ω = =
T m
For spring block system of mass 4m, the corresponding time period is 2T [and angular frequency = ]
If a particle P performs uniform circular motion in x-y plane with constant angular speed w, foot of perpendicular
drawn from it on the X axis [or Y axis] performs SHM with angular frequency w.
y
w P''1

P'1
2f
Q1 p/6 o Q1
t=0 X
P1 3 f P2
2a Q2

w/2

P'2

Let P1" and P2 be two points rotating on a circle of radius a in clockwise sense with angular speed w and
respectively. Motion of foot of perpendicular from P1 on X axis represents the motion of mass m and the motion of
foot of perpendicular from P2 on the X axis represents the motion of mass 4m.
3
According to the question, when P1 reaches P1' (i.e., block of mass m moves to Q1 getting displaced by )
2
then P2 starts rotating. Q2 is foot of perpendicular of P2 representing the motion of 4m. The blocks collide when P1
reaches P1'' and P2 reaches P'2 (see fig). Angle rotated by P1 is twice that covered by P2 due to its double angular
speed.
If < P2OP'2 = f then <P1' OP1" = 2f
Then < P1" OP2 = < P2OP'2 = f
But < P1" OP2 = < P2OP'2 = f
π
∴ + 2φ + φ = π
6

⇒φ =
18
12.34 Problems in Physics for JEE Advanced

 5π 
OQ1 = distance from O where collision takes place = a cos f = a cos   = cos 50°
 18 
5p
Time required for collision = time required by P2 to rotate through
18
2T 5π 5T 5π m
= . = =
2π 18 18 9 K
Time to travel from A to B (t1) is given by –
1 2
x = ut + at 2     [Q a = g sin q = 5 m/s ]
2
1
0.3 = × 5 × 12 t1 = 0.35s
2
Speed of the block when it hits the spring is given by–

V2 = 02 + 2ax = 2 × 5 × 0.3
V= 3 m/s

The motion of the block in contact with the spring can be regarded as a part of SHM. In equilibrium, compression
in the spring is given by
kx0 = Mg sin q
1
40 × 10 ×
x0 = 2 = 0.2m
1000
Let the block compress the spring by a length x1.
Energy conservation gives–

1 2 1
kx1 = MV 2 + Mgx1 sin θ
2 2
1 1 1
( 3) 
2
2
× 1000. 1 = × 40 × +  40 × 10 ×  1
2 2  2
5x21 – 2x1 – 0.6 = 0
Solving x1 = 0.6 m
Hence, amplitude of SHM is A = x1 – x0 = 0.4m
A

Xo
X1
B
A
O

c O Equilibrium

X0 = A/2

The motion from B to C and back to B can be regarded as motion of a particle


A A
performing SHM [from x = + to negative extreme and back to x = + ].
Simple Harmonic Motion 12.35

Time for this motion can be obtained from fig given below.

P1 B P2

A/2
A p/3 p/3
p/6
O

Particle on circle moves from P1 to C to P2. Time required for completing this two third
2
circle
3
2T 2 m
Hence, desired time t2 = = 2π
3 3 k
4π 40 8π
= = = 0.84
3 1000 30
Time required for the block to come back to A is t = 2t1 + t2
= 2 × 0.35 + 0.84 = 1.54 s
43. At the instant shown, both particle are at their mean position and moving in opposite direction.
Phase difference = 180°
 GM 
As w is same for both particle ω =  the phase difference will be maintained throughout and they will never
 R 
meet.
vmax = Aw
v1 R 2

v2 R/2 1
If V0 is the speed at the mean position
1
mV 2 = 20 ¥ 10 –3
2 0
1
× 0.2 × 02 = 20 × 10−3 V0 = 0.2 m/s
2
If linear amplitude is A then V0 = Aw = 0.2
g
Lθ 0 = 0.2     [Q A = Lq0 where q0 = angular amplitude]
L
q0 gL = 0.2
0.2
L= = 2.0 m
10 × (100 × 10−3 )
2

L 2
∴ T = 2π = 2 × 3.14 = 2.80 s
g 10

12.36 Problems in Physics for JEE Advanced

There is no loss of energy during collision because the collisions are elastic. Ball A stops and B acquires speed of
A. Let the length of pendulum B be l .
mg L (1 – cos q0) = mg l (1 – cos 2q0)

θ 
. 2 sin 2  0  = l . 2 sin 2 θ 0
 2
2
 θ0 
 2   l (θ 0 ) [sin ce sin θ 0  θ 0 ]
2

 
1
A completes of its oscillation and comes to rest after hitting B. Then B completes half of its oscillation and hits
4
A. B comes to rest and A goes back to its starting point. This completes one oscillation of A

1 L 1 l
∴ TA = 2π + 2π
2 g 2 g

π  L  = 3p L
=  L + 
g   2 g

(a) When the bob displaces by an angle q (small), restoring torque is


t = (mg sin q) l + kx. l
= mg l q + k l 2q Q x = l q

d 2θ
∴ ml 2 . = − ( mgl + kl 2 )θ
dt 2

d 2θ g k
2 = −  +  θ [SHM ]
dt  l m
1

g k g k  2
ω = + ⇒ T = 2π  + 
l m  l m

(b) In this case the cord is similar to the spring when it is stretched but it will exert no force during half the motion
when it is loose.

T l  −
1
− 
1

∴T' = + π  g k  2  g  2

2 g = π  +  +   
 l m  l 
 
. (a) ABP is right angled at B.

A q0
x sin q0
x

dx

B
Simple Harmonic Motion 12.37

sin θ 0 = 0.75 = 3
1.25 5
L m  mL2 . 2 mL2 9 3
( )
2
I = ∫0  dx
L 
 x sin θ 0 =
3
sin θ 0 =
3
×
25 25
mL2

L  L 
τ = mg
(b) sin θ 0 . sin θ   mg sin θ 0 θ [for small q]
2  2     

= 3 mgL θ
10

(c) 3
Iα = − mgL θ
10
3 3 5 g
mL2 . α − mgLθ ⇒ α = −  θ
25 10 2 L

2L
∴ T = 2π
5g
V = wR

8.33 1
\ w= = rad /s
100 12
Let m = mass of petrol
1
Moment of inertial of petrol about an axis through O is I mr 2
2
where 2r = 2m r = 1m
When the wagon is driven on curved track q is such that

È 4r ˘
mg x sin q = mw2 x cos q Íwhere x = 3p ˙
Î ˚
2
ω2R  1  100
tan θ = =   × = 0.07
g  12  9.8
q = tan–1 (0.07) = 4º
When the wagon enters the straight segment of track, liquid oscillates about an axis through O with an amplitude of
4º.

o
4r x G
3p q mw2R
G

mg
I a = – mg x sin q
1 2 d 2θ
⇒ mr = − mg x sin θ
2 dt 2
12.38 Problems in Physics for JEE Advanced

Since, q is small

d 2θ 2 g  4r   4r 
2
= − 2  θ Q x = 3π 
dt r  3π   

8g
= − θ
3π r

8g 1 8g 1 8 × 9.8
∴ω = ; ⇒ f = =
3π r 2π 3π r 2 × 3.14 3 × 3.14 × 1

2.88
= = 0.46 Hz
2 × 3.14

For simple pendulum T0 = 2π


L
g

I
For compound pendulum T = π
mgL

2 2
mr + mL2
∴ T = 2π 5
mgL
1/ 2
L  2 r2 
T = 2π 1 + 2 
g  5 L 

2r 2 n
Since 1 therefore, we can use (1 + x) = 1 + nx
5L2

 1 2 r2   r2 
∴ T = T0 1 + 2  ⇒ T = T01 + 2
 2 5 L   5L 

T - T0 r2
\ =
T0 5L2
∆T 20r 2 20 × 52 20 × 25
× 100 = 2 = 2
= = 0.05%
T0 L 100 10000
50. This is a compound pendulum with angular

mgl mg ( L + r ) 2g ( L + r )
frequency ω = = =
I 1 2 r + 2(L + r )
2 2
mr + m ( L + r )
2

2
Angular displacement can be expressed as a function of time q = q0 sin (wt)

dq
\ = q 0 cos (w t )
dt
When the string is vertical q = 0
sin (wt) = 0 and cos (wt) = 1
Simple Harmonic Motion 12.39

dq
\ = q 0w
dt
speed of lowest point

V = (L + r) = (L + 2r) q0w
dt
g (L + r)
= θo ( L + r )
r + (L + r)
51. (i) (a)

When the cylinder moves down by x, the liquid level rises by y.


Since volume of liquid is constant
x
3 Ay = Ax ⇒ y =
 4x 
Buoyancy force on cylinder =  A  ( 2ρ ) g
 3 
8 x0
In equilibrium Ar g = AL r g
3
3L
∴ x0 =
8
Since, equilibrium position is x0 below the original position (where, cylinder was at rest), hence amplitude is
3L
A x0
8
(b) When the cylinder is displaced x from its equilibrium position, restoring force on it will be the extra buoyancy
force which is

4x
=  A  ( 2 ρ ) g
 3 
d2x 8 
∴m 2
= −  Aρ g  x
dt 3 
d2 x 8
AL r g = - Ar g. x
dl 2 3

d2x 8 g 
∴ 2
= − x
dt 3 L

8 g 3L
∴ω = ⇒ T = 2π
3 L 8g
12.40 Problems in Physics for JEE Advanced

(ii) In equilibrium FB = W
rwg (2a)3 = rg [(2a)3 + a3]
ρ 8
=
ρω 9
When the block is depressed by x from its equilibrium position, the excess buoyancy is the restoring force.
Restoring force = (a2x) rwg
2
( 9a ) ρ ⋅ ddt x = − ( a ρ g ) x
3
2
2
ω

d2x  g   ρ 8
2
= −  x Q = 
dt  8a   ρω 9 
g
ω1 =
8a

Time for half oscillation t1 = p 8a


g
When the block is raised above its mean position, the restoring force is = (2a2 rwg)x
d2x
9a 3 ρ = −4a 2 ρω g x
dt 2
d x  g 
= −  x
dt  a

g
ω =
a

a
Time for half oscillation t = π
g

2a 2a
T = t1 + t2 = π [ 2 + 1] = 3π
g g

52. (a) Ice and cylinder together move as a single rigid body. Moment of inertia about axis of the cylinder is -
2 1 2  m 2
I = MR + mR =  M +  R
2  2

In equilibrium, the spring is relaxed. Consider the body at a displaced position from equilibrium.
x

a
kx
a

– kx = (M + m)a     ……………….(1)

– R=Ia
Simple Harmonic Motion 12.41

 m
− f R =  M +  R α
 

 m
− f =  M +  a     ……….(2) (Q Ra = a)
 

Add (1) and (2)

3m 
 2M +  a = − kx
 2 

2k
a = −  
x [SHM ]
 4M + 3m 
2k 4M + 3m
ω = hence , T = 2π
4M + 3m 2k

(b) Non viscous water will not rotate with the cylinder. It will only perform translational motion.
Equation (1) remains same as above.
Equation (2) changes as -
– R = MR2. a
– = Ma     ………….(3)
Adding (1) and (3)

k M +m
a = −  
 x ∴T = π
 M +m k

dU
F =− = ae− bx x n − ( n − bx )
dx
n
At equilibrium position F = 0 bx = n ⇒ x =
b
If the particle is displaced a little (say x) from its equilibrium position, the force that it will experience is calculated
as follows

dF
= − ab e− bx x n −1 ( n − bx ) + e− bx  −bx n −1 + ( n − bx ) ( n − 1) x n − 2 
dx
= – axn–2 e–bx [(n – bx) (bx – n + 1) + bx]
n
Put x
b
n−2 n−2
 dF  n n n n −1
  = −a   e −n
n = − an e− n   = − a e− n
 dx  at x0 b b bn−2

 dF   − n n n −1 
∴∆F =   ∆x = − ae  ∆x
 dx   bn−2 
Negative sign indicates that the force is restoring. It is proportional to displacement ( x). Hence motion is SHM.

a e− n n n −1 and 1 a e− n n n −1
ω2 = f =
mb n − 2 2π mb n − 2
12.42 Problems in Physics for JEE Advanced

Let us work in the reference frame attached to COM of the system. It is an inertial frame moving with constant
velocity V0.
m2 X2 X1
m1
X
O

If X1 (towards tight) and X2 (to left) are displacement of the two masses at time ‘t’ then.
m1X1 = m2X2
Equilibrium position means X1 + X2 = L
 m2 
In equilibrium, x co-ordinate of m1 is X 10 =  L
 m1 + m2 
equilibrium

X
c B
X10 X10

In the reference frame of COM, mass m1 will oscillate between points O and B with amplitude equal to
m2 L
A1 = X 10 =
m1 + m2
Equation of motion for m1
m1a1 = k [L – (X1 + X2)]

  m 
m1a1 = k  L −  X 1 + 1 X 1   [Q m2 X 2 = m1 X 1 ]
  m2  

KL K  m1m2 
∴ a1 = − X1 µ = 
m1 µ  m1 + m2 
µL
This is equation of SHM with equilibrium at X 10 = and angular frequency
m1

k  m + m2  m1m2
ω = = k 1  ⇒ T = 2π
µ  m1m2  ( m1 + m2 ) k
At t = 0, mass m1 is at negative extreme of the SHM and the origin of co-ordinate system is at the negative extreme
itself. Hence, displacement of m1 is
X1 = A1 – A1 cos wt [In COM frame]
In ground frame X1 = V0t + A1 (1 – cos wt)
[Q COM will travel V0t distance in time t]
m1 X 1 m
In COM frame X 2 = = 1 A1 (1 − cos ωt )
m2 m2
X2 = A2 (1 – cos wt)
m1 A1 m1 L
Where A2 = =
m2 m1 + m2

In ground frame X2 = V0t – A2 (1 – cos wt)


Simple Harmonic Motion 12.43

l
(a) The time period of B T = π
g

4l
Time period of A = 2π = 2T
g
The phasor representing B will rotate with an angular speed twice that of phasor representing A.
Let wA = w; then wB w

2w B B

120°

60°
w
B A
A

(A) (B)

Fig (A) represent the initial positions of the phasors representing A and B. Fig (B) represent their position when
the two pendulums are parallel and crossing each other.
Obviously, time required for this is the interval in which the phasor representing A (time period = 2T) rotates
p
by
3

∴ t = 2T = T = 2π l
6 3 3 g
(b) Amplitude of B is twice that of A. The initial phasors and the phasors when strings are parallel has been shown
below in figure (a) and (b) respectively.
2w B 2w

2q
q
w A
B
A

(a) (b)

Radius of the outer circle is 1.5 times that of the inner circle
R cos q = 1.5 R cos (180º – 2q)
2 cos q = – 3 cos 2q 2 cos q = – 3 (2 cos2 q – 1)
6 cos2 q + 2 cos q – 3 = 0

76 − 2 19 −1
∴ cos θ = =
12 6
It can be seen from simple observation that q is acute and cos q will be positive
 19 − 1 
∴ θ = cos −1  
 6 
12.44 Problems in Physics for JEE Advanced

q q q l l
\t= = = 4p = θ
w 2p 2p g
TA
56. let the pendulum be at its positive extreme at t = 0

È g˘
q = q0 cos (wt) Íwhere w = ˙
ÍÎ l ˙˚

dq
Angular velocity at time ‘t’ is = - q 0w sin (w t )
dt
velocity of the bob V = q0 wl sin (wt)
Tension (T) is given by–
mV 2
T = mg cos θ +
l
 θ2 
T = mg cos q + mq02 w2l sin2 wt = mg 1 −  + mq02 w2l sin2 wt
 2
1
= mg − mgq 20 cos2 wt + mq 20 g sin2 wt
2
1
cos 2 ω = sin 2 ω =
2
1
\ Tav = mg + mg q 02
4
When the stone is at a distance x from the centre of the earth, gravitational force on it is
GMm .
Fg = − x     [– sign indicates force is towards the Centre]
R3
d2x GM
∴ 2
= − 3 .x ……….(1)
dt R    
Stone performs SHM with
GM GM R3
ω2 = 3 ⇒ ω = ⇒ T = 2 π
R R3 GM
If the stone is dropped into the hole, it performs SHM of amplitude R and the journey from one end to the other
will take half the time T.

T R3
∴ T0 = = π
2 GM
The general solution to equation (1) is X = A cos (wt + d)
at t = 0; X = R A cos d = R         ----------------(2)
Also, V = – Aw sin (wt + d)
at t = 0; V = – u
u
– u = – Aw sin d ⇒ A sin δ = ------------------(3)
ω
u u
From (2) and (3) A = R + and tan δ =
ω ωR
Simple Harmonic Motion 12.45

T0
If the stone has to reach X = – R at t then,
2
X = A cos (wt + d)

Ê u2 ˆ Êp ˆ
- R = Á R 2 + 2 ˜ cos Á + d ˜
ÁË w ¯˜ Ë 2 ¯

È GM . T0 GM . p R3 p˘
ÍQ w t = = = ˙
ÍÎ R3 2 R3 2 GM 2˙
˚
R
∴ sin δ =         -------------------(4)
2 u2
R + 2
ω
R
From (2) cos δ =      -------------------(5)
u2
2
R + 2
ω
Squaring and adding (4) and (5) gives
2R2 u2 g
2
=1 ⇒ R2 = ⇒ u = Rω = R = gR
u
R + 2
2 ω2 R
ω w
58. Let the position of the man at time ‘t’ be at P (see fig).
Vo
P
q = wt ; V0 = wr
v
Vx = V – V0 sin (wt) = V – wr sin (wt) r
q
Vx = V – wy              ------------------(1)
O X
And Vy = V0 cos (wt) = wr cos (wt) = wx   ------------------(2)
Differentiating (1) wrt time
dVx dy
= −ω or, d x = − ωV
dt dt dt
y

d x
Using (2) = −ω x
dt
Solution of this equation is x = A sin (wt + d)
As per the question, at t = 0, x = 0
dx
d = 0 x = A sin (wt) and = a ω cos (ω t )
dt
at t = 0, Vx = V
V
∴ A=
ω
Maximum X co-ordinate is

59. Location of centre of mass of the cavitied sphere is given by -

4  3 R3  4 R3 R
π R − ρ x = π ρ
3  8  3 8 2
7 R
⇒ x = [ ρ = density]
8 16
R
⇒ x =
14
12.46 Problems in Physics for JEE Advanced

X
cm

P
Moment of inertia of the cavitied sphere about an axis ( r to plane of the fig) through point of contact (P) is
calculated as follows -
Let M = mass of cavitied sphere
4  1 3M 8
π R3 1 −  ρ = M ; ρ = 3
×
3  8 4π R 7
4 R3 M
Mass of sphere of radius is m = ρ π =
3 8 7
8M
Mass of sphere without cavity M 0 = m + M =
7
Required moment of inertia
I = (moment of inertia of complete sphere without cavity about an axis through P)–
(moment of inertia of the cavity about the same axis)
7 2
 2  R 2  3R  
2
= M0 R −  m  m +  
5  5  2   2  

7 8M 2  47 M 2  177
= R − R = MR 2
5 7  20 7  140
A purely rolling sphere can be considered to be is pure rotation about the point of contact. Consider the sphere at a
slightly displaced position q, as shown.
Restoring torque in this position is

R
t = Mg x sin q  Mg
14
O
R x
∴ I α = − Mgθ CM
14 q

177 R
⇒ MR 2 α = − Mg θ mg
140 14
140 g P
⇒ α =− θ
177 × 14 R

10 g
∴ ω2 =
177 R

177 R
⇒ T = 2π
10 g
Simple Harmonic Motion 12.47

Middle spring is stretched by x2 – x1

d 2 x1
For block 1 m = − k x1 + k ( x2 − x1 ) ------------(1)
dt 2     
d 2 x2
For block 2 m = − k x2 − k ( x2 − x1 ) ------------(2)
dt 2    
(a) Adding (1) and (2) gives

d 2 ( x1 + x2 )
m = − k ( x1 + x2 )
dt 2

⇒ d A = − kA
dt m
k
A = a1 sin (wat + d1) where ωa = ------------(3)
m     
(b) Subtracting (2) from (1) gives

d 2 ( x1 − x2 )
m = − 3 k ( x1 − x2 )
dt 2
d 2B 3k B
⇒ 2
= −
dt m

k
B = a2 sin (wbt + d2) where ωb = ----------(4)
m    
Adding (3) and (4) gives [A = x1 + x2, B = x1 – x2]
2x1 = a1 sin (wat + d1) + a2 sin (wb + d2)
a1 a
∴ x1 = sin (ωa t + δ1 ) + 2 sin (ωb t + δ 2 )
2 2
Thus x1 is combination of two SHMs having angular frequencies wa and wb.

61.
A


L
T T
y
B D
Fs
T T

q
T
T

C
Mg

In equilibrium 2T cos q = Mg [convince yourself that all rods have same tension]
And 2T sin q = Fs
12.48 Problems in Physics for JEE Advanced

Fs
⇒ tan θ =
Mg

Mg tan 30º = k [1.5L – 2L sin 30º]

1 kL kL 2
⇒ Mg = ⇒ = -----------(1)
3 2 Mg 3       
If y changes by y then we proceed as follows to calculate the restoring force.
Let length of the spring be l.
l = 2 L sin q
l = 2L cos q q       ------------(a)
And y = 2L cos q
y = – 2L sin q q       ------------(b)
Spring force changes by k l = 2kL cos q q
Change in rod tension will be given as
2 T sin q = 2kL cos q q
T = kL cot q. q
Restoring force on mass M is

cos 2 θ
2 ∆ T cos θ = 2kL ∆θ
sin θ

d2y cos 2 θ
∴M 2
= 2kL ∆θ
dt sin θ

d2y kL cos 2 θ ∆y
2
= −2 . [using (b)]
dt M sin θ 2 L sin θ     

k cos 2 θ 2g g
( 3)
2
= −
2
∆y = − . ∆=y − 2 3 ∆y
M sin θ 3L L
g
\ w2 = 2 3
L
L
T = 2π
2 3g

62. (i) If both blocks are simultaneously given equal velocity when they are at their mean positions, they will oscillate
as suggested.
(ii) The distance between 1 and 2 does not change. Hence, middle spring does not exert any force on the blocks.
Each block experiences force due to one spring only.

k
ω =
m
13 WAVE MOTION

the direction of wave propagation. Out of the


Level 1 6 particles marked (1, 2, 3, 4, 5, 6 ) how many
Q. 1. A boy is jerking one end of a taut string. The wave have their instantaneous velocity and acceleration
train propagating to the right has been shown in both directed towards their mean position.
the figure. Q. 4. Consider a function
- - -
=
a b
(a) Does this represent a travelling wave?
(b) What is direction of propagation of the wave?
(a) Why the crests are farther apart as we move (c) Find wave speed.
away from the boy. (d) Sketch the wave at t = 0
(b) Which particle on the string a or b is having Q. 5. A hypothetical pulse is travelling along positive x
higher speed? direction on a taut string. The speed of the pulse is
Q. 2. A transverse wave is travelling along a horizontal 10 cm s –1. The shape of the pulse at t = 0 is given
string. The first figure is the shape of the string as
at an instant of time. The second picture is a x
graph of the vertical displacement of a point on y = +1 for –6<x<0
6
the string as a function of time. How far does this
wave travel along the string in one second? = – x + 1 for 0<x<1
cm 4 = 0 for all other values of x
2 x and y are in cm.
0 cm (a) Find the vertical displacement of the particle
–2 2 4 6 8 10 12 14 16 18 at x = 1 cm at t = 0.2 s
–4 (b) Find the transverse velocity of the particle at
4 x = 1 cm at t = 0.2 s.
cm 2
Q. 6. Which of the following functions does not satisfy
1 2 3 4 5 6 7 8
0 the differential wave equation -
–2
seconds (i) y = 4ek(x – vt)
–4 (ii) y = 2 sin (5t) cos (6px)
Q. 3. Q. 7. A transverse harmonic wave of amplitude 4 mm
1 5 and wavelength 1.5 m is travelling in positive
x direction on a stretched string. At an instant,
the particle at x = 1.0 m is at y = + 2 mm and
2 6 is travelling in positive y direction. Find the co-
4
ordinate of the nearest particle (x > 1.0 m) which
is at its positive extreme at this instant.
3
Q. 8. A transverse harmonic wave travels along a taut
The figure shows the shape of three strings string having a tension of 57.6 N and linear mass
on which sinusoidal transverse waves are density of 100 g/m. Two points A and B on the
propagating. The arrows in the diagram indicate string are 5 cm apart and oscillate with a phase
13.2 Problems in Physics for JEE Advanced

p joining the speakers and record the intensity


difference of . How much does the phase of of sound at various points. On which path you
6
observe the loudness to alternate between faint
oscillation of point A change in a time interval of
and loud? Explain.
5.0 ms? l

Q. 9. A distant source of sound has frequency 800 Hz. S1 S2


An observer is facing 90° away from the direction
of the source. Estimate the phase difference
between the oscillations of her left and right
eardrums. Speed of sound in air = 340 ms –1.
Q. 10. A sinusoidal wave travels along a taut string m
of linear mass density 0.1 g/cm. The particles
oscillate along y-direction and the disturbance Q. 14. (i) A wire is stretched between two rigid supports.
moves in the positive x-direction. The amplitude It is observed that the wire resonates at a
and frequency of oscillation are 2 mm and 50 Hz frequency of 420 Hz. If a wooden bridge is
respectively. The minimum distance between two placed at the midpoint of the wire (so that the
particles oscillating in the same phase is 4 m. midpoint becomes a node), it was observed
(a) Find the tension in the string. that the smallest frequency at which the
wire resonates is 420 Hz. Find the smallest
(b) Find the amount of energy transferred frequency at which the wire will resonate
through any point of the string in one second. when there is no wooden bridge.
(c) If it is observed that the particle at x = 2 m is (ii) A string of length L is fixed at one end and
at y = 1 mm at t = 2 s, and its velocity is in is under tension due to a weight hanging
positive y-direction, then write the equation from the other end, as shown in the figure.
of this travelling wave. The point of the string on the pulley behaves
Q. 11. A and B are two point sources of sound (of same as a fixed point. Coordinate axes are chosen
frequency) and are kept at a separation. At a point so that the horizontal segment of the string
P, the intensity of sound is observed to be I0 runs from x = – L / 2 to x = L / 2. The string
when only source A is put on. With only B on the is vibrating at one of its resonant frequencies
intensity is observed to be 2I0. The distance AP is with transverse displacement (y) given by
higher than distance BP by half the wavelength of

y(x,t) = 0.05 cos(12.0x) sin(360t)
the sound. Find the intensity recorded at P with
both sources on. Give your answer for following with x, y in meter and t in second. Write two
cases: smallest possible values of L consistent with
(a) The sources are coherent and in phase. the given equation?
(b) The sources are coherent and 180° out of y
phase.
x = +L/2
(c) The sources are incoherent. x=0
x = –L/2
Q. 12. Two sound sources oscillate in phase with a
frequency of 100 Hz. At a point 1.74 m from one M
source and 1.16 m from the other, the amplitudes
of sound from the two sources are A and 2 A Q. 15. Two strings of same material are joined to form
respectively. Calculate the amplitude of the a large string and is stretched between rigid
resultant disturbance at the point. [Speed of sound supports. The diameter of the second string
in air is v = 348 ms –1] is twice that of the first. It was observed in an
Q. 13. Two speakers S1 and S2 are a driven by same experiment that the whole string was oscillating in
source. You walk along a line l that is perpendicular 4 loops with a node at the joint. Find the possible
bisector of the line joining the two speakers and lengths of the second string if the length of first
record the intensity at different points. Then you string is 90 cm.
walk along a line m that is parallel to the line Q. 16. (i) The equation of wave in a string fixed at both
Wave Motion 13.3

end is y = 2 sin p t cos p x. Find the phase as shown in figure. The end A of the string has a
difference between oscillations of two points small light ring which can slide on a smooth rod.
located at x = 0.4 m and x = 0.6 m. The wave reaches A at time t = 0.
(ii) A string having length L is under tension with
both the ends free to move. Standing wave is a
set in the string and the shape of the string at A
time t = 0 is as shown in the figure. Both ends
are at extreme. The string is back in the same
shape after regular intervals of time equal to
T and the maximum displacement of the free (i) Write the slope of the string at point A as
ends at any instant is A. Write the equation of function of time.
the standing wave.
(ii) If the incoming wave has amplitude a, with
what amplitude will the end A oscillate?
x=L
Q. 21. Fundamental frequency of a stretched sonometer
x=0
wire is f0. When its tension is increased by 96%
Q. 17. One type of steel has density 7800 kg/m3 and will and length decreased by 35%, its fundamental
break if the tensile stress exceeds 7.0 × 108 N/ frequency becomes h1 f0. When its tension is
m2. You want to make a guitar string using 4.0 decreased by 36% and its length is increased by
g of this type of steel. While in use, the guitar 30%, its fundamental frequency becomes h2 f0.
string must be able to withstand a tension of h
900 N without breaking. Find 1 .
h2
(a) Determine the maximum length and Q. 22. The linear mass density of the string shown in
minimum radius the string can have. the figure is m = 1 g/m. One end (A) of the string
(b) Determine the highest possible fundamental is tied to a prong of a tuning fork and the other
end carries a block of mass M. The length of the
frequency of standing waves on this string,
string between the tuning fork and the pulley is
if the entire length of the string is free to
L = 2.0 m. When the tuning fork vibrates, the
vibrate.
string resonates with it when mass M is either
Q. 18. A string, of length L, clamped at both ends is 16 kg or 25 kg. However, standing waves are not
vibrating in its first overtone mode. Answer the observed for any other value of M lying between
following questions for the moment the string 16 kg and 25 kg. Assume that end A of the string
looks flat is practically at rest and calculate the frequency of
(a) Find the distance between two nearest the fork.
particles each of which have half the speed of L
the particle having maximum speed.
(b) How many particles in the string have one A
eighth the speed of the particle travelling at
highest speed?
Q. 19. Two transverse waves travel in a medium in same
M
direction.
Ê 2p ˆ Ê 2p ˆ 18
y1 = a cos Á w t - x˜ ; y2 = a cos Á 2w t - x˜ Q. 23. Wavelength of two musical notes in air are m
Ë l1 ¯ Ë l2 ¯ 35
Êl ˆ and ÊÁ 90 ˆ˜ m . Each note produces four beats per
(a) Write the ratio of wavelengths Á 1 ˜ for the Ë 173 ¯
two waves. Ë l2 ¯
second with a third note of frequency f0. Calculate
(b) Plot the displacement of the particle at x = 0 the frequency f0.
with time (t). Q. 24. In a science – fiction movie the crew of a ship
Q. 20. A sine wave is travelling on a stretched string observes a satellite. Suddenly the satellite blows
13.4 Problems in Physics for JEE Advanced

up. The crew first sees the explosion and after instantaneous transverse velocity of points
a small time gap hears the sound. Do you think along the string, excluding its end-points,
there was a technical lapse? must be same everywhere except at nodes.”
Q. 25. A man is swimming at a depth d in a sea at a Is this statement correct?
distance L (>> d) from a ship (S). An explosion Q. 30 Sound of wavelength 100 cm travels in air.
occurs in the ship and after hearing the sound the At a given point the difference in maximum
man immediately moves to the surface. It takes and minimum pressure is 0.2 Nm –2. If the
0.8 s for the man to rise to the surface after he bulk modulus of air is 1.5 × 105 Nm –2, find the
hears the sound of explosion. 0.2 s after reaching amplitude of vibration of the particles of the
the surface he once again hears a sound of medium.
explosion. Calculate L.
Q. 31. (i) An organ pipe has one end closed and at the
L
other end there is a vibrating diaphragm.
S The diaphragm is a pressure node. The
pipe resonates when the frequency of the
d diaphragm is 2 KHz. Distance between
M
adjacent nodes is 8.0 cm. When the frequency
is slowly reduced, the pipe again resonates at
1.2 KHz.
Given: Speed of sound in air = 340 ms –1; Bulk (a) Find the length of the tube.
modulus of water = 2 × 109 Pa
(b) Find the next frequency above 2 KHz at
Q. 26. Speed of sound in air is 331 ms –1 at 0°C. Prove which the pipe resonates.
that it increases at a rate of 0.6 ms–1°C –1 for small
temperature increase. (ii) The figure shows an arrangement for
measuring the speed of sound in air. A glass
Q. 27. (a) Calculate the speed of sound in hydrogen gas tube is fitted with a movable piston that
at 300 K allows the indicated length L to be adjusted.
(b) At what temperature the speed in oxygen There is enough gap between the piston and
will be same as above. [Assume oxygen the tube wall to allow the air to pass through
molecules to remain diatomic] it. A speaker is placed near the open end of
Q. 28. A harmonic source (S) is driving a taut string. The the tube. A microphone is placed close to the
other end of the string is tied to a wall that is not speaker and it is connected to a waveform
so rigid. It is observed that standing waves are display. The display is a pure sinusoidal
formed in the string with ratio of amplitudes at waveform making 750 oscillations in 5 s.
the antinodes to that at the nodes equal to 8. What Initially, the piston is held at end A and is then
percentage of wave energy is transmitted to the slowly pulled back. Loud sound is produced
wall? by the tube when L = 50 cm and L = 157 cm.
Calculate the speed of sound in air.
S L
A

Wall Movable piston


Speaker Mic Tube
Q. 29. (a) Two identical sinusoidal pulses move in
Display
opposite directions on a stretched string.
Kinetic energy of each pulse is k. At the Q. 32. A rigid cylindrical container having a cross
instant they overlap completely, what is sectional area of 0.2 m2 is filled with water up to
kinetic energy of the resulting pulse? a height of 5.0 m. There is a piston of negligible
mass over the water. Piston can slide inside the
container without friction. When a weight of
2000 kg is placed over the piston, it moves down
(b) “A string clamped at both ends is vibrating. by 0.25 mm compressing the water.
At the moment the string looks flat, the r = Density of water = 103 kg/m3; Patm

Wave Motion 13.5

= Atmospheric pressure = 105 N/m2 and emitted by the source initially.


g = 10 m/s2. With this information calculate the (b) The frequency detected by both O1 and
speed of sound in water. O2 corresponding to the sound emitted by
Patm the source at height h/2 from the ground.
Q. 36. (i) A source of sound emits waves of frequency
0.25 mm Weight
f0 = 1200 Hz. The source is travelling at a
speed of v1 = 30 m/s towards east. There is a
5m large reflecting surface in front of the source
which is travelling at a velocity of v2 = 60
m/s towards west. Speed of sound in air is
A = 0.2m2 v = 330 m/s.

Q. 33. A point source of sound is located inside sea (a) Find the number of waves arriving per
water. Bulk modulus of sea water is Bw = 2.0 × second at the reflecting surface.
109 N / m2. A diver located at a distance of 10 m (b) Find the ratio of wavelength (l1) of sound
from the source registers a pressure amplitude of in front of the source travelling towards
D P0 = 3000 p N/m2 and gives the equation of the reflecting surface to the wavelength
sound wave as (l2) of sound in front of the source
y = A sin (15 p x – 21000 pt), y and x are in meter
approaching it after getting reflected.
and t is in second. (ii) A sound source (S) and an observer (A) are
(a) Find the displacement amplitude of the sound moving towards a point O along two straight
wave at the location of the diver. lines making an angle of 60° with each other.
The velocities of S and A are 18 ms –1 and
(b) Find the power of the sound source.
12 ms –1 respectively and remain constant
Q. 34. A point source of sound is moving uniformly with time. Frequency of the source is 1000
along positive x direction with velocity V0. At Hz and speed of sound is v = 330 ms –1.
time t = 0 the source was at origin and emitted
a compression pulse C1. After time T it emitted (a) Find the frequency received by the
another compression pulse C2. Write the equation observer when both the source and
of the wave front representing the compression observer are at a distance of 180 m from
pulse C2 at time t (>T). Speed of sound is V. point O (see figure).
Q. 35. (i) In a car race sound signals emitted by the (b)
Find the frequency received by the
two cars are detected by the detector on the observer when she reaches point O.
straight track at the end point of the race. Q. 37. A source of sound, producing a sinusoidal wave,
Frequency observed is 330 Hz and 360 Hz. is moving uniformly towards an observer at a
The original frequency of horn is 300 Hz for velocity of 20 m/s. The observer is moving away
both cars. Race ends with the separation of from the source at a constant velocity of 10 m / s.
1000 m between the cars. Assume both cars Frequency of the source is 200 Hz and speed of
move with constant velocity and velocity of sound in air is of 340 m/s.
sound is 330 m/s. Find the time (in seconds) (a) How many times, in an interval of 10 second,
taken by the winning car to finish the race. the eardrums of the observer will sense
(ii) A source of sound of frequency f is dropped maximum change in pressure?
from rest from a height h above the ground. (b) What will be apparent wavelength of sound
An observer O1 is located on the ground for the observer?
and another observer O2 is inside water at a
Q. 38. Two trains A and B are moving on parallel tracks
depth d from the ground. Both O1 and O2 are
in opposite direction at same speed of 30 ms –1.
vertically below the source. The velocity of
Just when the engines of the two trains are about
sound in water is 4V and that in air is V. Find to cross, the engine of train A begins to sound
(a) The frequency of the sound detected by a horn. The sound of the horn is composed of
O1 and O2 corresponding to the sound components varying in frequency from 900 Hz to
13.6 Problems in Physics for JEE Advanced

1200 Hz. The speed of sound in air is 330 ms –1.


Level 2
(a) Find the frequency spread (range of
frequencies) for the sound heard by a Q. 41. A long taut string is plucked at its centre. The
passenger in train A. pulse travelling on it can be described as y (x, t) =
e-( x + 2 t ) + e-( x - 2 t ) . Draw the shape of the string
2 2
(b) Find the frequency spread heard by a
passenger in train B. at time t = 0, a short time after t = 0 and a long
time after t = 0.
Q. 39. Two tuning forks produce 4 beats per second
when they are sounded together. Now both the Q. 42. A sinusoidal harmonic wave is propagating along
forks are moved towards the observer at same a string stretched along x – axis. A particle on the
speed (u). The beat frequency now becomes 5 Hz. string at x = 1 m is found to be at its mean position
If the observer also begins to run with speed u travelling in positive y direction at t = 1 s. The
towards both the forks, what beat frequency will amplitude, wavelength and frequency of the wave
he hear now? p
are 0.01 m, m and 20 Hz respectively. Write
2
Q. 40. (i) A sound source emitting sound at a single
the equation of the wave if-
frequency moves with constant speed along
x‑axis as shown in figure (a). A and B are two (a) it is travelling along negative X direction
stationary observers. The three plots shown (b) it is travelling along positive X direction.
in figure (b) indicate the pressure function
Q. 43. A circular loop of radius R is made of a perfectly
P(x) of the sound wave as recorded by the
elastic wire and is rotating with a constant angular
observer A, by B, and by another observer velocity w lying on a smooth horizontal table.
C who is at rest in the frame of the source. The rotation axis is vertical passing through the
Which plot (marked as 1, 2 and 3) correspond centre. A small radial push given to the loop at a
to which observer? point P on the table causes a transverse pulse to
x propagate on it. Find the smallest time in which
Detector B Source Detector A
(a)
the pulse will be back to its originating point P on
the table.

(ii) Each of the two figures is rough illustration


of the resulting waveform (y versus t ) due Q. 44. Two waves y1 = a sin ÊÁ p x - w t ˆ˜ and y2 = a sin
Ë2 ¯
to overlapping of two waves. The four p p
Ê ˆ
component waves have frequencies of ÁË + w + ˜ get superimposed in the region
¯
300 Hz, 200 Hz, 204 Hz and an unknown
frequency f = 300 + Df. Is Df higher than or
x > 0. Find the number of nodes in the region 0 <
less than 4 Hz? x < 6 m.
Q. 45. Two sine waves of same frequency and amplitude,
travel on a stretched string in opposite directions.
Speed of each wave is 10 cm/s. These two waves
superimpose to form a standing wave pattern
on the string. The maximum amplitude in the
standing wave pattern is 0.5 mm.
The figure shows the snapshot of the string at
t = 0.Write the equation of the two travelling
waves.
Wave Motion 13.7

Y (in mm) amplitudes observed.


(ii) A speaker (producing a sound of a single
0.5 wavelength l) and a microphone are placed as
shown in the figure. The microphone detects
-3 1 X (in cm) the sound and converts it into electrical signal.
O 5
This way we can obtain the waveform of the
sound. Assume that there is no attenuation
of the sound. The waveform detected by the
Q. 46. (i) A sinusoidal wave is travelling along positive microphone is sinusoidal with amplitude a. In
x direction and the displacements at two one experiment 6 microphones are placed in
positions x = 0 and x = 1 m are given by y (0, t) = front of the speaker with distance between two
Ê pˆ l
= neighbouring microphones being =
0.2 cos (3 p t) and ÁË p + ˜¯
Find all possible wavelength of the wave if
(a) The output from all the 6 microphones is
it is known that wavelength is greater than superimposed. What is amplitude of the
0.4 m. resultant?
(ii) A transverse sine wave of amplitude a = 0.1 (b) If large number of microphone are kept with
cm is travelling along a string laid along the separation L between two consecutive ones,
x-axis. The displacement (y) – time (t) graph how will the combined output change with L?
of the string particle at x = 0.1 m is shown Given that L π nl (n = 1, 2, 3...........)
in first figure. The shape of the string at time Microphone 1 Microphone 2 Microphone 6

t = 0.1 s is shown in second figure. At this time


the particle at x = 0.11 m is having velocity
Speaker
in positive y direction write the equation of
wave. Q. 48. The figure shows y (transverse displacement) vs x
Y (position) graph for a sinusoidal wave travelling
along a stretched string. P is power transmitted
through a cross section of the string at the instant
0.1 cm
x = 0.1m shown. Plot the graph of P versus x.
y
0.1
t(s)
O 0.2
0.5

Y –0.5

t = 0.1s
Q. 49. A string in a guitar is made of steel (density 7962
O
kg/m3). It is 63.5 cm long, and has diameter of
0.1 x(m)
0.05 0.4 mm. The fundamental frequency is f = 247 Hz.
0.1 cm (a) Find the string tension (F).
(b) If the tension F is changed by a small amount
DF, the frequency f changes by a small
Q. 47. (i) Two sinusoidal wave are given as y1 = a1 sin
D D
(wt + kx + d) and y2 = a2 sin (wt – kx). They amount Df. Show that =
superimpose.
(c) The string is tuned with tension equal to that
(a) Calculate the resultant amplitude of calculated in part (a) when its temperature
oscillation at a position x. Is amplitude time is 18°C. Continuous playing causes the
dependent?
temperature of the string to rise, changing its
(b) Calculate the ratio of maximum and minimum vibration frequency. Find Df if the temperature
13.8 Problems in Physics for JEE Advanced

of the string rises to 29°C. The steel string (a) Write the equation of the wave
has a Young’s modulus of 2.00 × 1011 Pa and (b) Draw y versus x graph for the wave at t = 0
a coefficient of linear expansion of 1.20 ×
Y (in mm)
10–5 (°C) –1. Assume that the temperature of
the body of the guitar remains constant. Will
3
the vibration frequency rise or fall?
Q. 50. A long taut string is connected to a harmonic 2 4 6 8 t (in s)
oscillator of frequency f at one end. The oscillator O
oscillates with an amplitude a0 and delivers power
P0 to the string. Due to dissipation of energy
the amplitude of wave goes on decreasing with
distance x from the oscillator given as a = a0e –kx. Q. 55. A string SQ is connected to a long heavier string
at Q. Linear mass density of the heavier string is
Ê ˆ 4 times that of the string SQ. Length of SQ is
In what length of the string ÁË ˜¯ of the energy
9.5 cm. Both the strings are subjected to same
supplied by the oscillator gets dissipated? tension. A 50 Hz source connected at S produces
Q. 51. A transverse harmonic wave is propagating along transverse disturbance in the string. Wavelength
a taut string. Tension in the string is 50 N and of the wave in string SQ is observed to be 1 cm.
its linear mass density is 0.02 kg m –1 The string If the source is put on at time t = 0, calculate the
is driven by a 80 Hz oscillator tied to one end smallest time (t) at which we can find a particle in
oscillating with an amplitude of 1mm. The other the heavier string that oscillates in phase with the
end of the string is terminated so that all the wave source at S.
Y
energy is absorbed and there is no reflection
(a) Calculate the power of the oscillator.
(b) The tension in the string is quadrupled. What
is new amplitude of the wave if the power of S
the oscillator remains same? Q
(c) Calculate the average energy of the wave on a 9.5 cm
1.0 m long segment of the string. Q. 56. The figure shows the snapshot at time t = 0 of a
Q. 52. A small steel ball of mass m = 5g is dropped transverse pulse travelling on a string in positive
from a height of 2.0 m on a hard floor. 0.001% x direction.
of its kinetic energy before striking the floor gets (a) Sketch the pulse at a slightly later time.
converted into a sound pulse having a duration of
0.4 s. Estimate how far away the sound can be (b) With the help of the given sketch draw a
heard if minimum audible intensity is 2.0 × 10 –8 graph of velocity of each string segment
Wm –2 [Actually it is much less but to account versus position. Take upward direction as
for background sound we are assuming it to be positive.
-1 0 1
high]. Assume no attenuation due to atmospheric X
absorption.
Q. 53. Three travelling waves are superimposed. The
equations of the wave are
y1 = A0 sin (kx – wt), y2 = 3 2 A0 sin (kx – wt + f)
Q. 57. A uniform string of length 6.5 m is subjected to
and y3 = 4 A0 cos(kx – wt)
a tension of 40 N. Mass of the string is 162.5 g.
Find the value of f (given 0 < f < p / 2) if the One end of the string is fixed and the other end is
phase difference between the resultant wave and tied to a source (s). which produces a transverse
first wave is p / 4. oscillation. The displacement of the end of the
Q. 54. A sinusoidal wave having wavelength of 6 m string tied to the source can be expressed as y =
propagates along positive x direction on a string. (3 mm) sin (40 p t), where ‘t’ is time. Find the
The displacement (y) of a particle at x = 2 m displacement of point P of the string at a distance
varies with time (t) as shown in the graph of 3.75 m from the fixed end, at time t = 0.3 s.
Wave Motion 13.9

2.75 m 3.75 m Q. 62. The figure shows the y – x graph at an instant for
a small amplitude transverse wave travelling on a
S
stretched string. Three elements (1, 2 and 3) on
the string have equal original lengths (= Dx). At
P
the given instant-
Q. 58. A longitudinal harmonic wave is travelling along (i) which element (among 1, 2 and 3) has largest
positive x direction. The amplitude, wavelength kinetic energy?
and frequency of the wave are 8.0 × 10 –3 m, 12 cm (ii) which element has largest energy (i.e., sum of
and 6800 Hz respectively. The displacement (s) its kinetic and elastic potential energy)
versus position graph for particles on the x axis at D
an instant of time has been shown in figure. Find (iii) Prove that energy per unit length of
D
the separation at the instant shown, between the
the string is constant everywhere equal to
particles which were originally at x1 = 1 cm and
x2 = 3 cm Ê ∂ ˆ where T is tension the string.
ÁË ˜¯

S
y

O X x x a
x X
O
a
2 3
Ê p ˆ

Q. 59. A sinusoidal wave = ÁË -w ˜ is Q. 63. A string has linear mass density m = 0.1 kg / m.
l ¯
A L = 60 cm segment of the string is clamped at
travelling on a stretched string. An observer A and B and is kept under a tension of T = 160 N
is travelling along positive x direction with a [The tension providing arrangement has not been
velocity equal to that of the wave. Find the angle shown in the figure]. A small paper rider is placed
that the velocity of a particle on the string at on the string at point R such that BR = 20 cm. The
l string is set into vibrations using a tuning fork of
= makes with – x direction as seen by the
frequency f.
observer at time t = 0. (a) Calculate all values of f below 1000 Hz for
Q. 60. A standing wave y = A sin kx .cos wt is established which the rider will not vibrate at all.
in a string fixed at its ends. (b) Calculate all values of f below 1000 Hz
(a) What is value of instantaneous power transfer for which the rider will have maximum
at a cross section of the string when the string oscillation amplitude among all points on the
is passing through its mean position? string.
L = 60 cm
(b) What is value of instantaneous power transfer A R B
at a cross section of the string when the string
l = 20 cm
is at its extreme position?
Q. 64. A sinusoidal longitudinal wave is travelling in
(c) At what frequency is the power transmitted positive x direction. Wave length of the wave is
through a cross section changing with time? 0.5 m At time t = 0, the change in pressure at
Q. 61. A sinusoidal transverse wave of small amplitude various points on the x axis can be represented
is travelling on a stretched string. The wave as shown in figure. Consider five particles of the
equation is y = a sin(k x – wt) and mass per unit medium A, B, C, D and E whose x co-ordinates
length of the string is m. Consider a small element are 0.125 m, 0.1875 m, 0.250 m, 0.375 m and
of length Dx on the string at x = 0. Calculate the 0.50 m respectively.
elastic potential energy stored in the element at (a) Which of the above mentioned five particles
time t = 0. Also write the kinetic energy of the of the medium are moving in positive x
element at t = 0. direction at t = 0.
13.10 Problems in Physics for JEE Advanced

(b) Find the ratio of speed of particles B and D at b = 0.006 °C/m, M = molar mass of air ~
t = 0. 29 g mol –1, g = 9.8 ms –2
P R = gas constant = 8.31 J mol –1 K –1
Consider air to be a mixture of diatomic gases
and calculate the atmospheric temperature and
pressure at height h0. Also find h0. Take (0.82)5.7
O X = 0.32.
Q. 67. In resonance column experiment a tuning fork
of frequency f = 400 Hz is held above the pipe
as shown in figure. The reservoir is raised and
Q. 65. (i) Two cylindrical pipes are each of length L = lowered to change the level of water and thus the
30 cm. One of them contains hydrogen and length of the column of air in the tube. The area
the other has oxygen at the same temperature. of cross section of the reservoir is 6 times that of
The ends A, B, C and D of the pipes are fitted the pipe. Initially, the reservoir is kept so that the
with flexible diaphragms. The diaphragms A pipe is full up to the brim. Tuning fork is sounded
and C are set into oscillations simultaneously and the reservoir is lowered. When the reservoir
using the same source having frequency is lowered by 21 cm, first resonance is recorded.
f = 600 Hz. Calculate the difference in phase When the reservoir is lowered further by 49 cm
of oscillations of the diaphragms D and B if it the second resonance is heard. Find the speed of
is known that the speed of sound in hydrogen sound in air.
at the temperature concerned is 1200 m/s.
A C

H2 O2 L

B D

(ii) The air column in a pipe closed at one end is


made to vibrate in its second overtone by a Q. 68. (i) In a travelling sinusoidal longitudinal wave,
tuning fork of frequency 440 Hz. The speed the displacement of particle of medium is
of sound in air is 330 m/s. P0 is mean pressure represented by s = S (x, t). The midpoint of a
in the pipe and DP0 is maximum amplitude of compression zone and an adjacent rarefaction
pressure variation. Neglect end correction. zone are represented by letter ‘C’ and ‘R’
(a) Find the length L of air column. respectively. The difference in pressure at ‘C’
(b) What is amplitude of pressure variation at and ‘R’ is DP and the bulk modulus of the
the middle of the column? medium is B.
(c) What is maximum and minimum pressure ∂ ∂
at the closed end? (a) How is ∂ related to

Q. 66. Speed of sound in atmosphere at a height h0 is
1080 km hr –1. The variation of temperature and ∂
pressure of the atmosphere with height h from the (b) Write the value of in terms of DP

surface is given by and B.

Ê (c) What is speed of a medium particle


b ˆ b
T = T0 – bh and
= ÁË - ˜¯ located mid-way between ‘C’ and ‘R’.

(ii) A standing wave in a pipe with a
Where T0 = temperature at the surface of the earth
length of L = 3 m is described by
= 273 K,
P0 = atmospheric pressure at the surface of the Ê p ˆ Ê p ˆ where v is wave
= ÁË ˜¯ ÁË ˜¯
earth,
Wave Motion 13.11

speed. The atmospheric pressure and density Q. 72. The string of a musical instrument was being
are P0 and r respectively. tuned using a tuning fork of known frequency,
f0 = 1024 Hz. The tuning fork and the string were
(a) At =
the acoustic pressure at = is set to vibrate together. Both vibrated together for
n
10 s and no beat was heard. What prediction can
0.2 percent of the atmospheric pressure . Find
be made regarding the frequency of the string?
the displacement amplitude A.
Q. 73. A wooden platform can be rotated about its vertical
(b) In which overtone is the pipe oscillating?
axis with constant angular speed w with the help
Q. 69. Two sources A and B give out sound waves in of a motor. A buzzer is fixed at the circumference
coherence and in phase. The sources are located of the platform and it rotates in a circle of radius
at co-ordinates (0, 0) and (0, 9 m) in xy plane. R. The buzzer produces sound of frequency f0. A
There is a detector located at (40 m, 0). It was mic is placed just beneath the platform near its
found that the detector records continuous circumference. An electronic frequency analyzer
increase in intensity of sound when it is moved in connected to the mic records the frequency (f)
positive y-direction for 4.5 m but the intensity was received by the mic. Take time (t) to be zero when
found to fall for some distance when it is moved the buzzer is just above the mic and express f as a
in negative y direction. What frequency of sound function of time. Plot f versus t. Speed of sound =
is consistent with these observations? Speed of V0.
sound = 340 ms –1.
w
Q. 70. In the figure shown, S1 and S2 are two identical B
point sources of sound which are coherent 180°
out of phase. Taking S1 as centre, two circular arcs
l and m of radii 1 m and 2 m are drawn. Taking
S2 as centres, two circular arcs p and q are drawn
having radii 2 m and 4 m respectively. Out of the
Q. 74. (i) A harmonic wave in a stationary medium is
four intersection points A, B, C and D which point
will record maximum intensity and which will represented by y = a sin (kx – wt). Write the
record the least intensity of sound? equation of this wave for an observer who is
moving in negative x direction with constant
It is given that wavelength of wave produced by speed v0.
each source is 4.0 m.
(ii) The Doppler flow meter is a device that
q
measures the speed of blood flow, using
p transmitting and receiving elements that are
A
placed directly on the skin. The transmitter
B
emits a continuous sound wave whose
C
frequency is 5 M Hz. When the sound is
D reflected from the red blood cells, its frequency
is changed in a kind of Doppler effect. The
cells are moving with the same velocity as
l S1 S2 the blood. The receiving element detects the
m
reflected sound, and an electronic counter
Q. 71. Stationary wave of frequency 5 K Hz is produced measures its frequency, which is Doppler-
in a tube open at both ends and filled with air at shifted relative to the transmitter frequency.
300 K. The tube is oscillating in its first overtone From the change in frequency the speed of
mode. the blood flow can be determined. Typically,
(a) Find the length of the tube assuming that air the change in frequency is around 600 Hz for
contains only nitrogen and oxygen in molar flow speeds of about 0.1 m/s. Assume that the
ratio of 3 : 1. red blood cell is directly moving away from
the source and the receiver.
(b) What shall be the frequency of sound wave
used so that the same tube oscillates in its (a) Estimate the speed of the sound wave in
second overtone mode? the blood?
13.12 Problems in Physics for JEE Advanced

(b) A segment of artery is narrowed down by the registered frequency changes from 420
plaque to half the normal cross-sectional
Hz to Ê ¥
ˆ and Dt2 be the time
area. What will be the Doppler change in ÁË ˜¯
frequency due to reflection from the red
interval during which the observed frequency
blood cell in that region?
Q. 75. A sound source emits waves of frequency f0 and changes from 399 Hz to ÊÁ ¥
ˆ
˜¯ .
wavelength l0 in still air. When there is a wind Ë
blowing with speed u from left to right what will Which is larger Dt1 or Dt2?
be wavelength of sound to the right of the source Q. 79. (i) A straight railway track is at a distance ‘d’
and to the left of the source. from you. A distant train approaches you
Q. 76. There are two horns H1 and H2 in a car. When travelling at a speed u (< speed of sound)
sounded together, the driver records 35 beats and crosses you. How does the apparent
in 10 second. With horn H2 blowing and car frequency (f) of the whistle change with time
moving towards a wall at a speed of 5 ms –1, the (f0 is the original frequency of the whistle).
driver noticed a beat frequency of 5 Hz with the Draw a rough f vs t graph.
echo. When frequency of H1 is decreased the
(ii) A bat is tracking a bug. It emits a sound,
beat frequency with two horns sounded together
which reflects off the bug. The bat hears
increases. Calculate the frequency of two horns.
Speed of sound = 332 ms –1
the echo of the sound 0.1 seconds after it
originally emitted it. The bat can tell if the
Q. 77. A toy train in a children amusement park runs insect is to the right or left by comparing
on an elliptical orbit having major and minor when the sound reaches its right ear to
axis in the ratio of 4 : 3. The length of the train is when the sound reaches its left ear. Bat’s
exactly equal to half the perimeter of the elliptical ears are only 2 cm apart. Bats also use the
track. The train is travelling at a constant speed of
frequency change of the sound echo to
20 ms –1. The engine sounds a whistle when its
determine the flight direction of the bug.
acceleration is minimum. The whistle has a
While hovering in the air (not moving),
frequency of f0 = 3460 Hz and speed of sound in
the bat emits a sound of 40.0 kHz. The
air is V = 330 ms –1
frequency of the echo is 40.4 kHz. Assume

(a) What frequency of whistle is received by a that the speed of sound is 340 m/s.
passenger in the last compartment of the (a) How far away is the bug?
train? (b) How much time delay is there between
(b) What frequency of whistle is received by a the echo reaching the two ears if the bug
passenger sitting in the central compartment is directly to the right of the bat?
of the train? (c) What is the speed of the bug?
Q. 78. A small source of sound has mass M and is Q. 80. A source of sound is located in a medium in which
attached to a spring of force constant K. It is speed of sound is V and an observer is located in
a medium in which speed of sound is 2V. Both
oscillating with amplitude = where the source and observer are moving directly
V is speed of sound in air. The source of sound towards each other at velocity . The source has
produces a sound of frequency f0 = 399 Hz. a frequency of f0.
(a) Find the frequency of sound registered by (a) Find the wavelength of wave in the medium
a stationary observer standing at a distant in which the observer is located.
point O. (b) Find the frequency received by the observer.
M Observer
k
Level 3
O
Smooth Ê ˆ

Q. 81. A transverse wave = wÁ - ˜ is
(b) Let Dt1 be the time interval during which Ë ¯
Wave Motion 13.13

travelling in a medium with speed V1. Plane Q. 83. Two sound waves, travelling in same direction
x = 0 is the boundary of the medium. For can be represented as
x > 0 there is a different medium in which the È - Ê ˆ˘
wave travels at a different speed V2. Part of =( ) Í p ( ÁË -
- ˜˙
¯˚
)
Î
the wave is reflected and part is transmitted.
For x < 0 the wave function is described as And
Ê x ˆ Ê x ˆ È ˆ˘
y- = A1 sin w Á - t ˜ + A2 sin w Á + t ˜ , while
Ë V1 ¯ Ë V2 ¯
=( ) (
Í p -
) ÊÁË -
- ˜˙
¯˚
Î
Ê x ˆ The waves superimpose.
for x > 0 ; y+ = A3 sinw Á - t ˜

Ë V2 ¯ (i) Find distance between two nearest points
where an intensity maximum is recorded
(a) Using the fact that the wave function must be simultaneously.
continuous at x = 0, show that A1 – A2 = A3 (ii) Find the time gap between two successive
intensity maxima at a given point.
∂y Q. 84. There are three sinusoidal waves A, B and C
(b) Using the fact that must be continuous at
∂x represented by equations-
V2 A
x = 0, prove that A1 = A3 - A2 A Æ y = A sin k x ; B Æ y = sin 2k x;
V1 2

Ê V1 - V2 ˆ A
(c) Show that = and A2 = Á A1 CÆy=
sin 3 k x
+ Ë V1 + V2 ˜¯ 2
Q. 82. A longitudinal wave is travelling at speed u in (a) To get a waveform of nearly the shape given
positive x direction in a medium having average in fig (a) which of the two waves B or C shall
density r0. The displacement (s) for particles of be superimposed with wave A?
the medium versus their position (x) has been (b) To get a waveform close to that in fig (b)
shown in the figure. which of the two waves B or C shall be
S superimposed with A?
y

O 6 8
X (in cm)
2 4 10

x
fig (a)
Answer following questions for 0 < x < 10 cm
(a) Write x co-ordinates of all positions where the
y
particles of the medium have maximum negative
acceleration. What is density at these locations –
higher than r0, less than r0 or equal to r0?
x
(b) Write x co-ordinates of all locations where the
fig (b)
particles of the medium have negative maximum
velocity. What do you think about density at these
positions?
(c) Knowing that the change in density (Dr) is Q. 85. A taut string is made of two segments. To the left
proportional to negative of the slope of s versus x of A it has a linear mass density of m kg/m and to
r V
graph, prove that µ - , where a is acceleration 2cm 1cm
4
of the particles at position x. At which point  2cm A
dr
(0 < x < 10) is positive maximum.
dx the right of A its linear mass density is 4 m kg/m.
13.14 Problems in Physics for JEE Advanced

A sinusoidal pulse of amplitude a is travelling Q. 87. A shock wave is a region of high acoustic pressure
towards right on the lighter string with a speed V propagating at speed of sound (v). Assume that
= 2 cm/s. the pressure in one such shock wave is 2P0
Draw the shape of the string after where P0 is the atmospheric pressure. This shock
(a) 1 s wave is travelling horizontally along x direction
(b) 2.5 s and hits a small wedge whose dimensions are
Q. 86. A wire having mass per unit length m and length as shown in the figure. The wedge has a mass
L is fixed between two fixed vertical walls at a m and is lying on a smooth horizontal surface.
separation L. Due to its own weight the wire sags. Determine the velocity u acquired by the wedge
The sag in the middle is d (<< L). Assume that immediately after the shock wave passes through
tension is practically constant along the wire, it. The velocity acquired by the wedge should be
owing to its small mass. Calculate the speed of assumed to be much lower than the velocity of the
the transverse wave on the wire. wave (u << v).
p
L
c
2p0
v
d p0
a
m
x b

ANSWERS
1. (a) The boy is jerking the string with gradually p2
increasing frequency (b) J
25
(b) Particle ‘a’ Êpx ˆ
y = 2 sin Á - 100 p t - 30∞˜
(c)
Ë 2 ¯
2. 3.09 cm
3. Three 2
4. (a) Yes
(b) Negative x direction
7A
(c) 0.6 unit
13. Along path m the loudness alternates between faint
(d) y
and loud due to phenomena of interference.

p p
,
12 4
x
15. 15 cm, 4 cm, 135 cm
5 16. (i) p
5. (a) cm
6 px 2p
5 (ii) y = A cos L cos T t
(b) - cm s -1
3 17. (a) 0.40 m , 6.4 × 10 –4 m
6. Both satisfy the wave equation
(b) 376 Hz
7. 2.25 m L
2p 18. (a)
8. 6
5 (b) 4
16
9. p l1 2
17 19. (a) =
l2 1
10. (a) 400 N
Wave Motion 13.15

y 40. (i) A - 1 B - 2 C - 3
(b)
(ii) Less than 4 Hz

t 41.

20. (i) zero


(ii) 2 a 42. (a) y = 0.01 sin [40 p (t – 1) + 4(x – 1)]
21. 3.5 y = 0.01 sin [40 p (t – 1) – 4(x – 1)]
(b)
23. 696 Hz p
43.
24. Yes, sound cannot travel in free space. w
44. 3
25. 447.6 m
Êp 5p 3p ˆ
27. (a) 1320 ms –1 45. y1 = 0.25 sin ÁË 4 x + 2 t + 4 ˜¯ ;
(b) 4800 K
Êp 5p 3p ˆ
28. 39.5% y2 = 0.25 sin Á x - t+ ˜
Ë4 2 4¯
29. (a) 4 k
(b) No 46. (i) and

30. 0.1 mm
y = (0.1 cm) cos [(20 pm –1x] + (10 p s –1)t]
(ii)
31. (i) (a) 20 cm

(b) 2800 Hz 47. (i) (a) a12 + a22 + 2a1a2 cos(2kx + d )


(ii) 321 m/s
Amax a1 + a2
32. 1414 m/s (b) =
Amin | a1 - a2 |
33. (a) 0.1 mm
(ii) (a) zero
(b) 3.9 × 104 watt
(b) zero
34. (x – V0T)2 + y2 + z2 = V2 (t – T)2
48.
35. (i) 40 s P
2
2vf
(ii) (a) f2 = f1 =
2vf - g
0
2
2vf
(b) f = f =
1 2 2(v - gh ) f - g
x

36. (i) (a) 1560 49. (a) 98.4 N


13 (c) 4.2 Hz decrease.
(b)
9
ln 2
(ii) (a) 1047 Hz 50.
k
(b) 991 Hz 51. (a) 0.126 W
37. (a) 4125 1
(b) mm
(b) 1.6 m 2
38. (a) 900 Hz to 1200 Hz (c) 2.5 mJ
(b) 1080 Hz to 1440 Hz 52. 3.15 m
39. 6 Hz p
53. f = 12
13.16 Problems in Physics for JEE Advanced

Êp p pˆ ∂s DP
54. (a) y = (3mm)sin Á x - t + ˜ (b) =
Ë3 2 3¯ ∂x C 2 B
55. 0.2 s (c) Zero
56. 0.0004 L P0
0 1 (ii) (a) ,
-1 X 3p v 2 r
(b) Second

O 1
X
-1

2.4 cm t
Ê ap ˆ
q = tan -1 Á ˜
Ë l ¯
w

w
p
l + l -
1 2 2 1
DU = m a w Dx ; Dk = m a 2w 2 Dx
2 2

D D

p
f0

D
t
D D

∂s ∂s
68. (i) (a) =
∂x C ∂x R
Wave Motion 13.17
y

82. (a) x = 4; equal to r0


(b) x = 2, x = 10 ; Dr is maximum negative

(c) x = 4.

83. (i) 150 m


(ii) 0.5 s fig (b)

85. (a)
84. (a) B
a
(b)
C A

y 2a
A 3

(b) 2a
3

a
3 A
a
3
B


2

fig (a)
86. =

87.

SOLUTION

2. The wavelength is = 9 cm
8
The particle makes 2.75 oscillations in 8 s, hence time period is T = = 2.91 s
2.75
l -
Wave speed, = = =

3. All particles perform SHM. Their acceleration is always towards their mean position. Velocity of 3, 4 and 5 are
towards mean position. This can be observed by looking at the direction of wave and predicting the position of a
particle a moment later.
5. Shape of the string at t = 0 and t = 0.2 s has been shown below
y

t=0 t = 0.2s
1 1
x x
0 1
–6 –4 0 2 3

(a) From the diagram


5
y = cm at x = 1 cm
6
(b) Velocity of the particle
13.18 Problems in Physics for JEE Advanced


v = – (wave velocity)

=- ( -
) =- -

6. y = 4 ek(x – vt)
∂y k ( x - vt ) ∂2 y
= 4 k e ; = 4k 2 ek ( x - vt ) ............(i)
∂x ∂x 2
∂y ∂2 y
= - 4vk ek ( x - vt ) ; 2 = 4v 2 k 2 ek ( x - vt ) ............(ii)
∂t ∂t
∂2 y 1 ∂2 y
Using (i) and (ii) one can show that =
∂x 2 v 2 ∂t 2
Similarly one can prove that the second function also obeys wave equation.
7.


We have to find x co-ordinate of particle B. Phase difference between A and B is
p p p
f= fi (D )=
l
5 ¥ 1.5
Dx = = 1.25 m ; \ x B = 2.25 m
6

T 57.6
8. Speed of wave v = = = 24 ms -1
m 0.1
2p p
Given ( Dx ) = for Dx = 0.05 m
l 6
v
\ l = 0.6 m \ f = = 40 Hz
l
1 1
\ T= = = 25 ms
f 40
T
Given time interval = 5 ms =
5
T 2p
\ Phase difference corresponding to a time interval of is Df =
5 5
9. Distance between two ears ~ 20 cm
The waves reaching the two eardrums have a path difference of 20 cm.

2p 2x
Df = ( Dx ) = f Dx
l v
2p ¥ 800 16p
= ¥ 0.2 =
340 17
Wave Motion 13.19

10. (a) l = 4 m and f = 50 Hz.


\ V = fl = 200 m/s
T
QV=
m

\ T = µ v 2 = (0.1) × (200) 2 = 400 N


(b) Number of waves that will cross a point is 1 second is 50, therefore power transmitted can be written as average
power (If a fractional number of waves were crossing a point in a given interval then we cannot multiply
average power with time to get the total energy transmitted)

= <P> = 2p 2 f 2 A 2 µv 2
p
= 2 × p 2 × (50) 2 × (2 × 10 –3 ) 2 × (0.01) × 200 = J.
(c) The equation of wave is
y = A sin(kx – wt + f )
0
p p
\ where K = = , w = 2pf = 100 p and A = 2
l
at x = 2 and t = 2; y = 1 mm
\ 1 = 2 sin(p – 200p + f )
0
solving f = –30°
0
Êp ˆ
\ y = 2 sin Á - p - ∞˜
Ë ¯
11. Phase difference due to path difference

p pl
Df = D = =p
l l

(a) When sources are coherent and in phase, the phase difference at P will be p.
\ I = I0 + 2I0 + 2 I0.2I0 cos p = 3I0 – 2 2 I0 = 0.2I0
(b) When sources are 180° out of phase, the resulting phase difference at P will be zero or 2 p
\ I = I0 + 2I0 + 2. I0(2I0) cos 0 = 3(3 + 2 2)I0
(c) When sources are incoherent the intensities add
\ I = 3I0
12. Path difference for two waves arriving at the point is
Dx = 1.74 – 1.16 = 0.58 m
\ Phase difference between the two waves arriving at the point is
p p
Df = D = D
l
p¥ p
= ¥ =

\ Resultant amplitude
Êpˆ
A0 = A2 + (2 A) + 2. A.2 A.cos Á ˜ = 7 A
2

Ë 3¯
13.20 Problems in Physics for JEE Advanced

14. (i) Let the fundamental frequency of the wire be f0.


After wooden bridge is inserted, length of each segment becomes half and fundamental frequency becomes
2 f0.
It is given that 2 f0 = 420
\ f0 = 210 Hz
(ii)
A = 0.05 cos(12.0x)
For x = –L / 2 and x = L / 2, A must be zero.
This is possible if
p p
- =- -

p p p p
fi = =

15. Fundamental frequencies of the two strings are

=
r

= =
r r

There are 3 possibilities:


(i) When first string has 1 loop and the other than 3 loops.
f10 = 3 f20

1 T 3 T
=
2l1 r A 4l2 rA
3 3
\ l2 = l1 = ¥ 90 = 135 cm
2 2

(ii) When both strings have two loops



l l
2 f10 = 2 f20 fi =
2l1 4l2
l1
fi l2 = 2 = 45 cm
(iii) When the first string has 3 loops and the second has one loop only.


16. Equation of wave is y = 2 sin pt cos px



x = 0 m is an antinode and the next antinode is at x = 1.0 m. In between node is at x = 0.5 m. Two given points are
on two sides of a node. Hence they differ in phase by p
Wave Motion 13.21

(ii) Hint : The wavelength is 2L.


900
17. (a) The breaking stress is = 7.0 × 108 N/m2,
pr 2
Solving for ‘r’ gives the minimum radius

900 N
r= = 6.4 × 10 –4 m.
p(7.0 ¥ 108 N /m 2 )
Ê ˆ
The mass and density are fixed ÁË r = p ˜¯ so the minimum radius gives the maximum length

M 4.0 ¥ 10 -3 kg
L= = = 0.40 m
p r 2 r p (6.4 ¥ 10 -4 m)2 (7800 kg /m 3 )

(b) The fundamental frequency is f1 = = =


m
The highest fundamental frequency occurs when F = 900 N,

= -
=
¥

18. (a) There are two nodes at the ends and one at the centre of the string.
Speed of a particle at a distance x from a node is given by-

Ê p ˆ where v is the maximum speed possessed by a particle at the antinode.


= ÁË ˜ 0
l ¯

l l l l
v = v0 /2 for =
l
Two nearest particles are at separation = =

(b) There will be two such particles in each loop. So the answer is 4.

19. (a) Speed of wave is property of the medium. Hence, both waves have same speed.
Since frequency of second wave is double that of the first, therefore, its wavelength will be half that of the first.
l1 2
=
l2 1
(b) At x = 0
y1 = a cos (wt)
y2 = a cos (2wt)
Disturbance due to individual waves and their superposition has been shown below.
y1 y2

t t


13.22 Problems in Physics for JEE Advanced
y3

1 T
21. f0 =
2L m
1 T + DT
f = = h f0
2 ( L + DL ) m
DT
1+
L T + DT T
\ h= =
L + DL T DT
1+
T
1 - 0.96 1.4 1 - 0.36 0.8
\ h1 = = and h2 = =
1 - 0.35 0.65 1 + 0.3 1.3
h1 1.4 1.3
\ = ¥ = 3.5
h2 0.65 0.8

T Mg
22. Wave speed v = =
m m
If the string oscillates with n loops when
M = 16 kg then there will be (n – 1) loops for M = 25 kg

Q n T      But frequency is same in both cases,


f =
2L m

n 16 g (n - 1) 25g
\ =
2L m 2L m
\ 4 n = 5 (n - 1)
fi n=5
5 16 ¥ 10 5
\ f = -3
= ¥ 4 ¥ 102 = 500 Hz
2 ¥ 2 1 ¥ 10 4

18 90
23. l1 = = m
35 175
90
and l2 = m
173
\ l2 > l1 f1 > f2
\ f1 – f0 = 4 ..............… (1)
and f0 – f2 = 4 ..............… (2)
(1) + (2)
f1 – f2 = 8, i.e., - =
l l
Wave Motion 13.23

È175 173 ˘ 8 ¥ 90
vÍ - ˙ =8 fiv= = 360 ms -1
Î 90 90 ˚ 2
v 360 ¥ 35
\ f1 = = = 700 Hz
l1 18
\ f0 = 696 Hz
25. Speed of sound in water
B 2 ¥ 109
Vw = = = 1414 ms -1
r 103
First sound is heard due to sound wave travelling in water. It takes t1 time for sound to travel from S to M in water.
Second sound is heard because of sound travelling through air reaching the man.
This sound takes t2 time to travel through L distance.
L L
t2 – t1 = 0.8 + 0.2 i.e., - = 1.0
Vair Vw

[Distance SM ~ L since d << L]


Vair Vw 340 ¥ 1414
\ L= = = 447.6 m
Vw - Vair 1414 - 340

g RT
26. v = ..........….(1)
M
D g
= ..........….(2)
D

(2) (1) gives


Dv 1 DT
=
v 2 T
Dv 1 v 1 331
= = ¥ = 0.61 ms -1K -1 = 0.61 ms -1 ∞C -1
DT 2 T 2 273
Change in temperature has same magnitude in K scale and °C scale.
g RT 1.4 ¥ 8.314 ¥ 300
27. (a) VH = = -3
= 1320 ms -1
MH 2 ¥ 10

g RT
V0 =
(b)
M0
As per question
g RT g R ¥ 300
=
M0 MH
M 32
\ T = 0 ¥ 300 = ¥ 300 = 4800 K
MH 2

Amax Ai + Ar
28. = =8
Amin Ai - Ar
Where Ai = amplitude of the incident wave
Ar = amplitude of the reflected wave
13.24 Problems in Physics for JEE Advanced

9
\ Ai = Ar
7
Ai2 - Ar2
Percentage of energy transmitted = ¥ 100
Ai2
49 2
Ai2 - A
= 81 i ¥ 100
Ai2
= 39.5%
29. Speed of a particle on the string is proportional to the slope of the string at that location.
If y = A sin kx, then -
∂y
= Ak sin kx
∂x
∂y
After overlapping : y = 2A sin kx fi = 2 Ak sin kx
∂x
Since the slope doubles, speed of each particle doubles. It means K.E. will be 4 k.
30. Pmax – Pmin = 2DP0
Where DP0 is pressure amplitude.
\ 2 DP0 = 0.2
fi DP0 = 0.1 Nm –2 &
BAk = 0.1
0.1 ¥ l 0.1 ¥ 1.0
fi A= = = 1.0 ¥ 10 -7 m
2p B 2 ¥ 3.14 ¥ 1.5 ¥ 105

31. (i) (a) f = 2000 Hz and l = 2 × 8 = 16 cm
\ Speed of sound in air in the pipe is
v = fl = 2000 × 0.16 = 320 ms –1
320 4 80
When f´ = 1600 Hz ; l ¢ = = m= cm
1200 15 3
Length of the tube is
l l’
= [2(n - 1) - 1]
L = (2n - 1)
4 4
80
fi (2n - 1) ◊ 16 = (2n - 3) ◊ fin=3
3
l 16
\ L = (2 n - 1) = (6 - 1) ¥ = 20 cm
4 4
v 320
Fundamental frequency f0 = 4 L = 4 ¥ 0.2 = 400 Hz
\ Resonant frequencies are 400 Hz, 1200 Hz, 2800 Hz …..
\ Answer is 2800 Hz
750
(ii) Frequency of sound f = = 150 Hz
5
Loud sound is produced when resonance occurs. Let the end correction be e.
l
e + 50 = ..........… (1)
4
Wave Motion 13.25

3l
And e + 157 = .........… (2)
4
l
(2) – (1) gives : = 107
2
= 214 cm = 2.14 m
–1
\ v = lf = 2.14 × 150 = 321 ms
32. Volume of water in the cylinder is V = 5 × 0.2 = 1 m3
Decrease in volume due to increased pressure
DV = 0.2 × 0.25 × 10 –3 m3= 50 × 10 –6 m3 = 50 cc
DV 50 ¥ 10 -6
Volume strain = = 50 ¥ 10 -6
V 1
Volume stress or Bulk stress = change in pressure
Mg 2000 ¥ 10
DP = = = 105 N /m 2
A 0.2
D
\ Bulk modulus of water = = = ¥
D ¥ -

B 2 ¥ 109
\ Speed of sound in water v = = = 1414 m /s
r 103
33. k = 15 p ; w = 21000 p
w
Speed of sound in sea water V = = 1400 m /s
k
Bw 2 ¥ 109
But V = \ rw = = 1.02 ¥ 103 kg /m 3
rw (1400) 2

DP0 = BAk
(a)
DP0 3000p
fi A= = = 10 -7 m = 0.1 m m
Bk 2 ¥ 109 ¥ 15p
(b) Intensity received
DP02 (3000p ) 2

I = = = 31.07 W /m 2
2rw V 2 ¥ 1.02 ¥ 10 ¥ 1400
3

\ Power of source
P = I × 4pr2 = 31.07 × 4 × 3.14 ×102 = 3.90 ×104 Watt
34. Wave front will be a sphere centered at (V0T, 0,0) having radius V (t – T)
(x – V0T)2 + y2 + z2 = V2 (t – T)2
35. (i) Let speed of winning car be v1 and for the other car be v2
-
= ¥ =
-
330
And 330 = ¥ 300; v2 = 30 ms -1
330 - v2
13.26 Problems in Physics for JEE Advanced

As per the question v1t – v2t = 1000 m


1000 m
t = = 40 s
25ms -1
(ii) (a) Let distance between the observer and source be x when the source is released at t = 0. If the source
releases a compression pulse immediately when it is released; the pulse will reach the observer at a time
x
t1 = .
v
The second pulse is released after a time = . At this time the distance between the source and the observer
is - .

The second pulse will reach the observer when the time is
1 2
x- gT
2
t2 = +T
v
The interval between the two successive pulses reaching the observer is the apparent time period.

1 2
x- gT
2 x
T ’ = t2 - t1 = +T -
v v
1 2
gT
T =T - 2

v
2
1 Ê 1ˆ
g
1 1 2 ÁË f ˜¯ 2 vf 2
= - fi f¢=
f ¢ f v 2 vf - g
This frequency will be same for both the observer.
Part (b) can be solved with similar approach
36. (a) Frequency received by the reflector is

È V + V2 ˘
f = f0 Í ˙
Î V - V1 ˚
È 330 + 60 ˘ 390
= 1200 Í ˙ = 1200 ¥ 300 = 1560 Hz
Î 330 - 60 ˚

V1 = 30 m/s V1 = 60 m/s

Reflector
Wave Motion 13.27

(b)
V - V1 330 - 30 1
l1 = = = m
f0 1200 4
V - V2 330 - 60 27 9
l2 = = = = m
f 1560 156 52
l1 1 52 13
\ = ¥ =
l2 4 9 9

È 340 - 10 ˘
37. (a) f = 200 Í ˙ = 206.25 Hz
Î 340 - 20 ˚
No. of times maximum change in pressure is sensed by the observer in 10 s is
= 206.25 × 2 × 10 [In each cycle, the change in pressure is maximum twice]
= 4125
(b) Wavelength changes due to motion of source
340 1
l = l0 - VsT = - 20 ¥ = 1.7 - 0.1 = 1.6 m
200 200
38.
(a) Source and observer are travelling in same direction with same speed. Hence there will be no change in
frequency. Frequency received will be from 900 Hz to 1200 Hz.
È 330 + 30 ˘ È 360 ˘
(b) Lower frequency f1 = f0 Í ˙ = 900 ¥ Í ˙ = 1080 Hz
Î 330 - 30 ˚ Î 300 ˚
È 360 ˘
Higher frequency f2 = 1200 ¥ Í ˙ = 1440 Hz.
Î 300 ˚
39. The frequencies of two forks are assumed to be f and f + 4.
When forks are moving towards the observer, the frequencies received by the observer are

Ê v ˆ and f = ( f + 4 ) Ê v ˆ
f1 = f Á ÁË v - u ˜¯
Ë v - u ˜¯
2

Ê v ˆ
\ D f = f2 - f1 = 4 Á
Ë v - u ˜¯
Ê v ˆ
\ 5 = 4Á
Ë v - u ˜¯
v
fi 5v - 5u = 4 v fi u =
5
When observer also begins to run, the frequencies received by him will be

Ê v + uˆ Ê v + uˆ
f1¢= f Á ˜ and f2¢ = ( f + 4) Á
Ë v - u¯ Ë v - u ˜¯
Ê v + uˆ
\ D f ¢ = f2¢ - f1¢= 4 Á
Ë v - u ˜¯
Ê vˆ
v+
Á 5 ˜ 6
= 4Á = 4 ◊ = 6 Hz
v˜ 4
Áv- ˜
Ë 5 ¯
13.28 Problems in Physics for JEE Advanced

40. (i) The wavelength will decrease in front of the source and it will increase behind the source
(ii) The first figure has higher average wave frequency but lower beat frequency. Thus the first graph represents
superposition of 300 Hz and 300 + Df. The other graph has a beat frequency of 4 Hz = (204 – 200). Therefore,
Df < 4 Hz.
42. (a) If the origin is assumed be at x = 1 and time is reset to be zero at t = 1s, the equation of wave travelling in
negative x will be

y´ = sin (wt + kx)


È ˘
Í 2p ˙
= 0.01sin Í2p .20t + . x = 0.01sin [ 40p t + 4 x ] ………….(i)
Í p ˙˙
Î 2 ˚
In the given co-ordinate system, the displacement of a particle at x = 1 m at t = 1 s is represented by the value
of y obtained in (i) after putting t = 0 and x = 0
\ In given system

y = 0.01 sin [40 p (t – 1) + 4 (x – 1)]


(b) once again if origin is taken at x = 1 m and time is reset to be zero at t = 1 s, then the equation of the wave

travelling in positive x can be written as follows:
y’ = 0.01 sin [40 p t – 4x]
dy ’
[Note that at t = 1, x = 1 gives a positive value meaning that the velocity of the particle is positive]
dt
Once again adjusting for shift in origin of space and time -
y = 0.01 sin [40 p (t – 1) – 4 (x – 1)]

43. First let us calculate the tension in the wire loop. Consider a small element of angular width dq on the loop.
q
= m qw

q
\ =m qw fi =m w
T
Speed of transverse wave relative to the string is V = = wR
m
Relative to the ground, the pulse will travel with speed 2V (or angular speed 2w)
\ Time required to reach back at P is
p
=
w
[Note: The pulse that starts travelling in direction opposite to the direction of rotation of the ring will remain static
at P]
È p˘
44. y1 = a sin(kx - w t ) Ík = 2 ˙
Î ˚
Ê pˆ
y2 = a sin Á kx + w t + ˜
Ë 3¯

È Ê p ˆ˘
= + = Í -w + ÁË +w + ˜˙
Î ¯˚

Wave Motion 13.29

Ê pˆ Ê pˆ
= ÁË + ˜ ÁË w + ˜¯
¯

Ê pˆ
Node are positions where A = 2a sin Á kx + ˜ = 0
Ë 6¯

p p
fi x + = p , 2p , 3p ......... [for x > 0]
2 6
x Ê 1ˆ Ê 1ˆ Ê 1ˆ
fi = Á 1 - ˜ , Á 2 - ˜ , Á 3 - ˜ ..........
2 Ë 6¯ Ë 6¯ Ë 6¯
5 11 17
= , , .........
6 6 6
5 11 17 23
\ x = , , , .....,
3 3 3 3
We have not considered negative value of x as those are not required.
17
Obviously, = and lie between x = 1 and x = 6 m
3
So there are 3 nodes between x = 1 and x = 6 m.

45. At x = –3, x = 1, x = 5 we have nodes. Two consecutive nodes are at separation.


l
\ = fil=

p p
= =
l
w
Also, wave speed V =
k
p p
\w= = ¥ = rad / s

As shown in the figure, all particle in the string are at their extreme at t = 0. Hence the equation of standing wave
is like
y = A cos wt   [At t = 0, y = A for all particles]
Where A is the amplitude which changes harmonically with x
Let A = A0 sin (kx + d)
From figure A0 = 0.5 mm
At x = 1; A = 0
\ k (1) + d = p [kx + d¹ π 0 at x = 1. Why ?]
p 3p
fi +d = p fid =
4 4
Ê p 3p ˆ
\ A = 0.5 sin Á x + ˜
Ë4 4¯
Êp pˆ
And = ÁË + ˜¯ w

Êp 3p ˆ 5p
y = 0.5 sin Á x + ˜ cos t
Ë4 4¯ 2
13.30 Problems in Physics for JEE Advanced

Êp p pˆ Êp p pˆ
= ÁË + + ˜¯ + ÁË - + ˜¯

Ê p ˆ
46. (ii) Let the equation of the wave be = ÁË p - ˜
l ¯

p
Obviously phase angle at x = 1 is shifted by - p =
As compared to the phase at x = 0.
2p p
\ - = - 2np (n = 1, 2.....)
l 8
16
fi l=
16n - 1

For n = 1; l =

For n = 2; l =

For n = 3; l =

Only l1 and l2 are within given limit of l > 0.4 m


47. (i) The two waves can be represented by phasors as shown in figure.
The resultant amplitude is
= + + +d

kx +
kx


(a) No, amplitude does not depend on time.
Amax = a1 + a2 [When cos (2 kx + d) = 1]

= - [when cos (2kx + ) = – 1

+
=
-
48. The instantaneous power transmitted is proportional to the magnitude of the slope of the y versus x graph.
49. (a) m = p (0.2 × 10–3 m)2 (7962 kg/m3) = 1.0 × 10–3 kg/m

Fundamental frequency of a string, f =


m
Wave Motion 13.31

Rearranging this and solving for F gives


F = 4 m L2f2
F = 4 (1.0 × 10–3 kg/m) (0.635 m)2 (247.0 Hz)2 = 98.4 N.

(b)
f= …………….. (1)
m

Ê ˆ Ê ˆ
D = D Á ˜ = DÁ ˜
Ë m¯ Ë m ¯

D =
m
D ( )
D
D =
m
Now divide this equation by (1)
1 1 DF
Df 2L m 2 F
=
f 1 F
2L m
Df 1 Df
=
f 2 F
(c) Stress = Y (strain)
D
= - aD

DF = – (2.00 × 1011Pa) (1.20 × 10–5/°C) × p (0.2 × 10–3m)2 × (11°C) = – 3.3 N.


DF/F = – 3.3/98.4 = – 0.033,

D D
=

Df/f = – 0.017,
Df = –0.017 × 247 = – 4.2 Hz.
Negative sign indicates that the tension falls and the frequency also falls.
1
50. P0 = mw 2 a02 v = 2p 2 m f 2 a02 v
2
At distance x, a = a0e–kx

( )
2
\ P = 2p 2 m f 2 a0 e- kx v

Given P = P0
4
1 ln 2
fi e-2 kx = fi -2kx = ln(1) - ln (4) fi x =
4 k

T 50
51. (a) Wave speed v = = = 2500 = 50 ms -1 
m 0.02
1
Power of oscillator = average power transmitted through a cross section = mw 2 a 2 v
2
13.32 Problems in Physics for JEE Advanced

\ P = 2p2m f 2 a2v ..........(i)


P = 2 × (3.14)2 × (0.02) × (80)2 × (1 × 10 –3)2 × 50 = 0.126 W
(b) If tension is made 4 times the wave speed becomes double.
1
To keep the power same amplitude will become times , i.e.,
2
1
(c) Pav = mw 2 A2 v
2
Average energy on a segment of length L is
= (Average power) × (time required for the wave to travel a distance L)
L 1.0
Eav = Pav = 0.126 ¥ = 0.0025 J = 2.5 mJ
v 50
52. Energy of ball E = mgh = 5 × 10 –3 × 10 × 2 = 0.1 J
0.001
Energy of sound pulse = ¥ 0.1 = 10 -6 J
100
10 -6 J
Power of the sound source P = = 2.5 × 10 –6 W
0.4 s
P
Intensity at a distance r from a point source is I =
4p r 2
Given I = 2 × 10 –8 Wm –2
P 2.5 ¥ 10 -6
\ r2 = = = 9.95
4p I 4 ¥ 3.14 ¥ 2 ¥ 10 -8
\ r = 9.95 = 3.15 m
53. From phase diagram

/4 

Êpˆ (+ f)
ÁË ˜¯ =
(+ f)

π
Solving, φ =
12
54. (a) l = 6 m
Y ( mm)

2 5 8 X
O

-3

Wave Motion 13.33

2p p
\k = =
l 3

From the graph T = 4s


2p p
\w = =
T 2
Let the equation of the wave be
Êp p ˆ
= ÁË - +d˜
¯
At x = 2; t = 0, y is zero
Ê p ˆ
\ sin Á 2 + d ˜ = 0
Ë 3 ¯
2p p
fi d = - or
3 3
∂y
Also, > 0 at t = 0 at x = 2
∂t
∂y p Êp p ˆ
= -3 cos ÁË x - t + d ˜¯
∂t 2 3 3
At t = 0, x = 2
∂y 3p Ê 2p ˆ
= - cos Á +d˜
∂t 2 Ë 3 ¯
for this to be positive
2p
cos ÊÁ ˆ
+d˜ < 0
Ë 3 ¯

p 2p
\d = [and d π - ]
3 3
Êp p pˆ
\ y = (3mm)sin Á x - t + ˜
Ë3 2 3¯

Êp pˆ
(b) at t = 0, y = 3 sin Á x + ˜
Ë3 3¯

T
55. Speed of wave v =
m
Hence speed of wave in heavier string is half that in the lighter string.
Speed of wave in SQ is v = lf
\ v = (1 cm) (50 Hz) = 50 cm/s
v
Speed in heavier string = = 25 cm /s
2
1 cm
Wavelength in heavier string = = 0.5 cm
2
Phase of point Q is ahead of S by
13.34 Problems in Physics for JEE Advanced

2p 2p
Df = Dx = (9.5 cm) = (9.5)(2p )
l (1cm)
\ A point that will oscillate in phase with S must be ahead in phase by (0.5) (2 p) with respect to Q.
If x is distance of this point from Q then -

2p
x = (0.5)(2p )
0.5 cm

\ x = 0.25 cm
9.5 0.25
Time required for the disturbance to reach this point is + = 0.19 + 0.01 = 0.20 s
50 25

T 40 È 0.1625 ˘
57. Wave speed along the string v = = ÍQ m = 6.5 = 0.025 kg /m ˙
m 0.025 Î ˚
= 40 m /s

Frequency f = 20 Hz
v
Wavelength; l = = 2m and k = 2p = p m -1
f l
Wave propagating along the string can be expressed as

y = a sin (40pt – px) [x = 0 at source]


Displacement at P due to reflected wave at t = 0.3 s is [x = 6.5 + 3.75 = 10.25 m]
a
y1 = a sin (40p × 0.3 – 10.25 p + p) = a sin(2.75p ) =
2
[‘p’ due to phase change during reflection]
Displacement at P due to direct wave at t = 0.3 s is
a
y2 = a sin (40p × 0.3 – p × 2.75) = a sin(9.25p ) = -
2
\ Displacement at P at t = 0.3 s is y = y1 + y2 = 0
Ê 2p ˆ Êp ˆ
58. Displacement of the medium particles at the instant shown can be represented by - s = a sin Á x = a sin Á x˜
Ë 12 ˜¯ Ë6 ¯
Êp ˆ a
The displacement of the particle at x1 = 1 cm is s1 = a sin ÁË .1˜¯ = = 0.4 cm
6 2

Êp ˆ
and displacement of the particle at x2 = 3 cm is s2 = a sin Á .3˜ = a = 0.8 cm
Ë6 ¯
Required separation is = x2 + s2 – (x1 + s1)
= 3 + 0.8 – ( 1 + 0.4 ) = 2.4 cm

59. To the observer, the wave (disturbance) does not appear to propagate. The string moves backwards. Every point on
the string appears to be moving tangentially. At time t = 0
Ê 2p ˆ dy 2p Ê 2p ˆ
y = a sin Á x˜ ; =a cos Á x˜
Ë l ¯ dx l Ë l ¯
2p Ê 2p l ˆ ap
\ tan q = a. cos Á ˜ ; tanq =
l Ë l 6¯ l
Wave Motion 13.35

x

VA

60. (a) Velocity at a point is perpendicular to the tension. Hence power is zero.
(b) Velocity at all points is zero. Hence power is zero.
Ê ∂y ˆ Ê ∂y ˆ
y = a sin kx cos w t; P = - Á T ˜ Á ˜
(c) Ë ∂x ¯ Ë ∂t ¯
Where the first component is transverse component of tension and the term in second bracket is the velocity at
a point.
P is positive when energy is transmitted in the direction of positive x. When energy is transferred in other
direction P is negative.
The (–) sign in the above expression is to ensure this sign convention.
T
\ P = -T (kA sin w t cos kx )(w A cos w t sin kx ) = - A2 w k (sin 2kx )(cos 2w t )
4
Hence frequency of oscillation of P at a given point is twice that of the wave.
w
Wave frequency, f =
2p
w
Power frequency =
p
61. Consider an element of small length Dx. At some instant it is stretched to length Dl.
T

l
y

x
1
Extension = Dl - Dx = ( Dx 2 + Dy 2 ) 2 - Dx

È ÊD ˆ ˘ È ÊD ˆ ˘
= D Í +Á ˜ ˙ -D D Í + Á ˜ ˙-D
ÍÎ Ë D ¯ ˙˚ ÍÎ ËD ¯ ˙
˚

D
[Because of small amplitude the slope is small ]
D

\ Extension ~ ÊD ˆ Ê∂ ˆ
ÁË D ˜¯ D ÁË ∂ ˜¯ D

Elastic PE = work done by tension in stretching = Tension × extension

2 2
1 Ê ∂y ˆ 1 Ê ∂y ˆ È T˘
DU = T Á ˜ Dx = m v 2 Á ˜ Dx ÍQ v = ˙
2 Ë ∂x ¯ 2 Ë ∂x ¯ ÍÎ m ˙˚

13.36 Problems in Physics for JEE Advanced

w
= m -w D

1
x = 0, DU =
At mw 2 a 2 Dx cos2 (w t )
2
1
At t = 0, DU = mw 2 a 2 Dx
2
2
1 Ê ∂y ˆ 1
Kinetic energy DK = ( mDx ) Á ˜ = m D x a 2w 2 cos2 (kx - w t )
2 Ë ∂t ¯ 2

1 2 2
At x = 0; t = 0 DK = m a w Dx
2
62. From the last problem
2 2
DU 1 2 Ê ∂y ˆ 1 Ê ∂y ˆ
= mv Á ˜ = T Á ˜
Dx 2 Ë ∂x ¯ 2 Ë ∂x ¯ ∂y
2 2 [Q particle velocity Vp = - v ]
DK 1 Ê ∂y ˆ 1 Ê ∂y ˆ ∂x
= mÁ ˜ = mÁ-v ˜
Dx 2 Ë ∂t ¯ 2 Ë ∂x ¯
2 2
1 2 Ê ∂y ˆ 1 Ê ∂y ˆ
= mv Á ˜ = T Á ˜
2 Ë ∂x ¯ 2 Ë ∂x ¯
2
DE DU + DK Ê ∂y ˆ
\ = =TÁ ˜
Dx Dx Ë ∂x ¯
Obviously kinetic energy, potential energy and total energy is largest for the element where the slope is steepest,
i.e., for element 2.

63. (a) wave speed = = =


m

If R is node and segment BR has n loops then the segment AR must have 2n loops (as AR = 2BR)

l
\ = =

f = n × 100 Hz for n = 1, 2, 3, 4,.............9


We get f = 100, 200, 300,..........., 900 Hz
(b) If R is antinode then
l l
= + [n = 0,1,2,.....]

l l
= = +

l l l l l
\ AB = 3n + + = (3n + 1) +
2 2 4 2 4
l
But length of string can only be integral multiple of
2
Hence, it is not possible to have antinode at R.
Wave Motion 13.37

64. (a) Displacement position graph will be as shown (at t = 0)


S

B C
X
O A D E


Ê∂ ˆ
Velocity of a particle is =- ÁË ∂ ˜¯ [V0 = wave velocity]


At A and B slope is negative, hence VA and VB are positive.

(b) Displacement wave equation can be written as

Ê p ˆ
= ÁË -w ˜
l ¯
=
Ê p ˆ aw
= w = = w ÁË ¥ ˜¯ =
= 2

VB 1
\ =
VD 2

65. (i) g g
= =

\ = = =

\ = =

l = = = l = = =

A and C are in same phase.


p p
\ Phase difference between D and B will be Df = -
l l Pressure
node
È ˘ È ˘
= p Í - ˙= p¥ Í - ˙= p
ÎÍ l l ˚˙
Î ˚

¥
(ii) (a) = l= = =
¥
l 
(b) Distance of mid point from closest Node = 8
8
Ê p lˆ D
D =D =D ÁË ˜=
l ¯
At the closed end Pmax = P0 + DP0 ; Pmin = P0 – DP0

13.38 Problems in Physics for JEE Advanced

66. 1080 km/hr = 300 m/s



g
=
- - -
¥
\ = = - -
=
g ¥

T = 273 – (0.006)h0 \ 224 = 273 – (0.006)h0


h0 = 8167 m
29 ¥10 -3 ¥ 9.8

Ê 0.006 ¥ 8167 ˆ 8.31¥ 0.006


= P0 (1 – 0.18)5.7 = (0.82)5.7 P0 = 0.32 P0
P = P0 Á 1 - ˜¯
Ë 273

67. When reservoir is lowered by x, let the level of water fall by y


y 7y 6x
x- = y \x= fiy=
6 6 7
For x = 21 cm, y1 = 18 cm
For x = 21 + 49 = 70 cm ; y2 = 60 cm
l
\ + = ........…….(i)

l
+ = ........…….(ii)

(ii) – (i) gives

l
= 42 fi l = 84 cm = 0.84 m
2
\ V = lf = 0.84 × 400 = 336 ms –1

68. (ii)

L
At = and x =
2
p Ê pˆ Êpˆ p
D = - ÁË ˜¯ ÁË ˜¯ =
p
As per the question = 0.0002 P0

\ = =
p p r
69. At C we have maxima (zero path difference). Conditions in the question can be met if the detector (D) lies between
C and the first minima. The figure shows the variation of intensity around C.
l
The path difference Dx = BD – AD must be less than

= + = \D = - =

l
\ < fi < fi <
Wave Motion 13.39
y

B
C
9m

O
A 40 m D

< <

70. Let amplitude of wave on arc m and p be a due to individual sources


Amplitude on arc l due to S1 = 2a

Amplitude on arc q due to =

p p
At point A : Df = D +p = +p = p
l

Ê ˆ
\ = +Á ˜ + p =
Ë ¯
At B ; Df = p [Q no path difference]
\ aB = 0
p p
At C ; Df = ( )+p =
Ê pˆ
\ = + + ÁË ˜¯ =

p p p
At D : Df = +p = +p =

Ê ˆ Ê ˆ p
= Á ˜ + + Á ˜ =
Ë ¯ Ë ¯

71. (a) Molar mass of air


Mair = 0.75 × MN + 0.25 × MO
= 0.75 × 28 + 0.25 × 32
= 29 g/mol = 29 × 10 –3 kg/mol
Because, N2 and O2 both are diatomic hence g = 1.4
Speed of sound in air at 300 K
g ¥ ¥
= = -
=
¥

\ Wavelength of 5kHz sound will be l = = =

In first overtone mode, the length of the tube (L) = l = 6.9 cm


(b) In second overtone mode
13.40 Problems in Physics for JEE Advanced

l ¥
= fi l= =

\ == =
l ¥ -
72. Beat frequency Df must be less than 1 in 10 s.
&Df < 0.1 Hz
\ Difference in frequencies of the two sources is less than 0.1 Hz.
\ 1023.9 Hz < f string < 1024.1 Hz
73. Let the position of Buzzer at time t be B and that of the mic be M.
V

R b


p -q
=w q =w b=

p q w
a= -b = =

È ˘
È V0 ˘ Í V0 ˙
\ f = f0 Í ˙ = f0 Í ˙
Ê wt ˆ ˙
Î V0 + Vcosa ˚ Í
V + w Rcos
Í 0 ÁË 2 ˜¯ ˙
Î ˚
graph is as shown
f

f0 t


74. (i) Frequency observed by the observer is
Ê + ˆ Ê ˆ w
= Á ˜¯ \ w = w ÁË + ˜¯ = w + =w +
Ë
Amplitude and wavelength do not change for moving observer.
\ y0 = a sin [(w + kv0) t – kx]
Wave Motion 13.41

V - VR
f =
(ii) f0 where V and VR are speed of sound and speed of red blood cells respectively.
V + VR
-
D = - = - = ……. (i)
+ +

¥
fi = ¥ ¥ =

(b) The continuity equation in fluid mechanics tells us that VR will get double if cross sectional area becomes half.
From (i) Df will become 2 times.
76. f1 = f2 + 3.5 ................(1)

H2


Consider wall as observer. Frequency received by the wall is

Ê ˆ
= ÁË - ˜¯

Now wall acts as a source of frequency f´. Frequency of echo received by the observer is
Ê + ˆ Ê + ˆ Ê ˆ
= ÁË ˜¯ = ÁË ˜= ÁË ˜¯
- ¯

È ˘
Given f´´ – f2 = 5 \ Í - ˙=
Î ˚

= =

From (1)
f1 = 163.5 + 3.5 = 167 or 160 Hz
When frequency of H1 decreases, it gives more beats with H2.
\ f1 = 160 Hz
77. Acceleration of the engine is minimum at position where radius of curvature is maximum i.e., at position E shown
in the figure.
E
u 
u

3


4 C

u
L

In the figure - E, C and L are engine, centrally located passenger and the last passenger respectively.
Speed of the train u = 20 ms –1
13.42 Problems in Physics for JEE Advanced

(a) Since observer and source both are moving perpendicular to the line joining them there is no change in
frequency.
(b) Frequency received by C is
È ˘
Í + ¥ ˙
Ê + bˆ
= ÁË ˜= Í ˙
+ a¯ Í + ¥ ˙
ÎÍ ˚˙
= ¥ =

78. The observer registers sound of different frequencies ranging from a minimum to a maximum.
Observed frequency is maximum for the sound emitted by the source when it is crossing its mean position travelling
towards the observer [remember that Doppler’s effect does not depend on distance between the source the observer]
È
È V ˘ K˘
\ fmax = f0 Í ˙ Íw = ˙
Î V - Aw ˚ ÍÎ M ˙˚
È ˘
Í V ˙ 20
= 399 ¥ Í ˙ = 399 ¥ = 420 Hz
V
ÍV - ˙ 19
ÍÎ 20 ˙˚
The minimum frequency is registered for the sound emitted by the source at its mean position travelling away from
the observer
È ˘
Í ˙
= Í ˙= ¥ =
Í + ˙
ÎÍ ˚˙

\ Observer receives sound of frequency ranging from 380 Hz to 420 Hz


380 Hz f 420 Hz.

(b) For = ¥

È ˘
= Í - ˙
Î ˚
È ˘
¥ = Í - ˙ fi =
Î ˚ -

w
fi = = (approaching)

for = ¥

È ˘ w
=Í ˙ fi + = ( )
Î + ˚

\ Dt1 is time interval during which the source moves from its mean position to =
È w ˘
Í =w - fi =w - fi = ˙
ÎÍ ˚˙
Wave Motion 13.43

And Dt2 is time interval during which it moves from x = A (extreme) to =


\ Dt1 > Dt2

È ˘
79. (i) = Í -
Î q ˙˚
p
As train moves from a large distance towards point A, angle q changes from near zero to and then increases
beyond p / 2 to approach p. The graph is as shown in the diagram.
f

u A
 f0
d

o t

¥
(ii) (a) Distance = =

-
(b) D = = ¥

(c) The bug is moving towards the bat since the received frequency is higher.
È + ˘
Frequency received by the bug Í ˙ =
Î ˚
Now, the bug can be treated as a moving source of frequency f1. Frequency received by the bat is
È 340 ˘ È 340 + v ˘ È 340 ˘ È 340 + v ˘
f2 = f1 Í ˙ fi f2 = 40 Í ˙ Í 340 - v ˙ = 40 Í 340 - v ˙
Î 340 - v ˚ Î 340 ˚Î ˚ Î ˚

È + ˘
fi = Í
Î - ˙˚
fi =

\ v = 1.7 m/s
80. (a) Wavelength in front of the source (in medium of the source itself) is
l
l = l - ◊ But =

l l
\ l=l - fi l=
When a wave charges medium, frequency does not change, it is wavelength that changes.
\ Wavelength in the medium of the observer is
l
l ¢ = l = =

(b) Apparent time period = time interval between two successive compression pulses striking the observer =
l¢ l¢ l¢
¢ = = =
+
+

= ◊
¢

fi ¢=
13.44 Problems in Physics for JEE Advanced

Ê ˆ Ê ˆ Ê ˆ
81. (a) wÁ - ˜+ wÁ + ˜= wÁ - ˜
Ë ¯ Ë ¯ Ë ¯
&A1 sin (–wt) +A2 sin (wt) = A3 sin (–wt)
& – A1 + A2 = – A3
& A1 – A2 = A3..............(1)

∂y- A1w
(b) Ê x ˆ Aw Ê x ˆ
= cos w Á - t ˜ + 2 cos w Á + t ˜
∂x V1 Ë V1 ¯ V2 Ë V2 ¯

Ê ∂y- ˆ Aw Aw
Á ˜ = 1 cos (-w t ) + 2 cos (w t )
Ë ∂x ¯ x = o v1 v2

A1w Aw
= cos w t + 2 cos (w t )
V1 V2
∂y+ A3w Ê x ˆ
= cos w Á - t ˜
∂x V2 Ë V2 ¯

Ê ∂y ˆ Aw
Á + ˜ = 3 cos w t
Ë ∂x ¯ x = 0 V2

But Ê∂ -ˆ Ê∂ ˆ
ÁË ∂ ˜¯ =Á +˜
=
Ë ∂ ¯ =

w w w
\ + =

= - .............(2)

Solving (1) and (2)

= and =
( - )
+ +
82. (a) At x = 4, particle is at positive extreme. Hence acceleration is maximum and negative

Dr µ -

At = =

\ Dr = 0 & density is equal to r0

(b) At x = 2 and x = 10; is maximum and positive

= -

V is maximum negative at these points.

Dr µ - & Dr is maximum and negative


Wave Motion 13.45

ds
Dr = -
(c)
dx

But =- = particle velocity (V)

∂r ∂
\ Dr µ fi µ

In a travelling wave V = g (x – ut)
∂ ∂
\ =- =-

∂r
\ µ-

∂r
is positive maximum at positions where a is negative maximum; i.e., at x = 4 cm

83. The given equations are
y1 = a sin (K1x – w1t) and y2 = a sin (K2x – w2t)
Where a = 0.02 mm ; w1 = 400 p rad s –1 ; w2 = 404 p rad –1
400 p 404p
K1 = and K 2 =
300 330
\ y = y1 + y2
Ê DKx Dw ˆ
= 2a cos Á - t sin ( K av x - w av t ) ..............(1)
Ë 2 2 ˜¯
+ w +w
Where = w =

DK = K2 – K1 ; Dw = w2 – w1
(a) At t = 0, the equation becomes
È ÊD ˆ˘
= Í ËÁ ¯˜ ˙˚
( )
Î
This represents a wave with wave number = Kav and amplitude varying with position as
ÊD ˆ
= ÁË ˜¯

\ A is maximum when
D
= p p

p p
fi =
D D
p p
\ Required answer is = =
D p

(b) At x = 0; equation (i) becomes


Ê Dw ˆ
=- ÁË ˜¯ (w )
13.46 Problems in Physics for JEE Advanced

The amplitude changes with time (at a fixed location) as given by


Ê Dw ˆ
= ÁË ˜¯
Ê Dw ˆ
\ Intensity varies as = ÁË ˜¯ = ÈÎ (Dw ) + ˘˚

Angular Frequency of this (Known as beat frequency) is


Dw = w2 – w1
\ D f = f2 – f1 = 202 – 200 = 2 Hz

\ = = =
D
at a point maximum are recorded at a gap of 0.5 s.

85. Speed of wave on second string is . At A, a part of wave energy gets reflected and a part is transmitted

Ê ˆ
-
Á ˜
Amplitude of reflected pulse =Á ˜ =-
Á + ˜
Ë ¯
Negative sign indicates a phase charge of p.

Ê V ˆ
2
Amplitude of transmitted pulse at = Á 2 ˜ a = 2 a
ÁV ˜ 3
Á +V˜
Ë2 ¯
At t = 1 s incident, reflected and transmitted pulses are as shown below –
Incident pulse

a
A

a

Reflected pulse Transmitted pulse Wavelength reduces by a factor of 2.

a/3 A
2a
A
3


Shape of the string is obtained by superposition of above 3 pulses.

a
A

2a
3

In 2.5 s the complete pulse strikes A and shape of the string is as shown.
Wave Motion 13.47

2a
3

a
3 A
a
3



2

86. Let tension at mid point be T. Consider rotational equilibrium of half of the wire about its fixed end.
Torque due to tension = torque due to its weight

◊ =m ◊

Since line of action of weight is nearly at a distance from the fixed end.
m
\ =

\ Wave speed ==
m
87. Time required for the wave to travel through length b is D = .

The horizontal force on the wedge during the interval 0 to Dt increases linearly from 0 to P0ac.

1
Favg =P0 ac
2
P ac b P acb
\ mu = 0 fiu= 0
2 v 2mv

You might also like